You are on page 1of 282

A ("^ 13 American College of Physicians

internal medicine | Doctors for Adidts

I
MKSAP16
Medical Knowledge Self-Assessment Program

General Internal
'
Medicine
All New Content, Including 168
Multiple-Choice Questions

7 CUMULATIVE

INDEX
A /^ T) American College of Physicians
1 _lV__>JL internal medicine I Doctors for Adults

MKSAP16
Medical Knowledge Self-Assessment Program

General Internal Medicine


Welcome to the General Internal
Medicine section of MKSAP 16!
Here, you will find updated information on routine care of the healthy patient; patient safety; professionalism and ethics; pal
liative care; chronic pain; acute and chronic cough; chronic fatigue; musculoskeletal pain; dyslipidemia; obesity; men's and
women's health; eye, ear, nose, mouth, and throat disorders; mental and behavioral health; geriatric medicine; perioperative
medicine; and many other clinical challenges. All of these topics are uniquely focused on the needs of both generalists
and those who practice subspecialty internal medicine.

The publication of the 16th edition of Medical Knowledge Self-Assessment Program heralds a significant event, culminating
2 years of effort by dozens of leading subspecialists across the United States. Our authoring committees have strived to help
internists succeed in Maintenance of Certification, right up to preparing for the MOC examination, and to get residents ready
for the certifying examination. MKSAP 16 also helps you update your medical knowledge and elevates standards of self-learning
by allowing you to assess your knowledge with 1,200 all-new multiple-choice questions, including 168 in General Internal
Medicine.

MKSAP began more than 40 years ago. The American Board of Internal Medicine's examination blueprint and gaps between
actual and preferred practices inform creation of the content. The questions, refined through rigorous face-to-face meetings,
are among the best in medicine. A psychometric analysis of the items sharpens our educational focus on weaknesses in prac
tice. To meet diverse learning styles, we offer MKSAP 16 online and in downloadable apps for PCs, tablets, laptops, and
smartphones. We are also introducing the following:

High-Value Care Recommendations: The General Internal Medicine section starts with several recommendations based on
the important concept of health care value (balancing clinical benefit with costs and harms) to address the needs of trainees,
practicing physicians, and patients. These recommendations are part of a major initiative that has been undertaken by the
American College of Physicians, in collaboration with other organizations.

Content for Hospitalists: This material, highlighted in blue and labeled with the familiar hospital icon (), directly addresses
the learning needs of the increasing number of physicians who work in the hospital setting. MKSAP 16 Digital will allow you
to customize quizzes based on hospitalist-only questions to help you prepare for the Hospital Medicine Maintenance of
Certification Examination.

We hope you enjoy and benefit from MKSAP 16. Please feel free to send us any comments to mksap_editors@acponline.org
or visit us at the MKSAP Resource Site (mksap.acponline.org) to find out how we can help you study, earn CME, accumulate
MOC points, and stay up to date. I know I speak on behalf of ACP staff members and our authoring committees when I say
we are honored to have attracted your interest and participation.

Sincerely,

Patrick Alguire, MD, FACP


Editor-in-Chief
Senior Vice President
Medical Education Division
American College of Physicians
General Internal Medicine

Committee Scott Herrle, MD, MS1


Assistant Professor of Medicine
Gary H. Tabas, MD, FACP, Editor1
Professor of Medicine University of Pittsburgh School of Medicine
VA Pittsburgh Healthcare System
Division of General Internal Medicine
Pittsburgh, Pennsylvania
Department of Medicine
University of Pittsburgh School of Medicine Christopher L. Knight, MD, FACP2
University of Pittsburgh Medical Center Presbyterian- Associate Professor
Shadyside Department of Medicine
Pittsburgh, Pennsylvania University of Washington
Seattle, Washington
Jack Ende, MD, MACP, Associate Editor1
Professor of Medicine Megan McNamara, MD, MSc1
University of Pennsylvania Assistant Professor of Medicine-
Chief, Department of Medicine Director of Student Assessment, Program Evaluation, and
Penn Presbyterian Medical Center the Center for the Advancement of Medical Learning
Philadelphia, Pennsylvania Case Western Reserve University School of Medicine
Paul B. Aronowitz, MD, FACP1 Cleveland, Ohio

Adjunct Associate Professor of Medicine Mohan Nadkarni, MD, FACP1


Dartmouth Medical School Professor of Medicine
Associate Professor of Clinical Medicine Chief, Section of General Internal Medicine
University of California, San Francisco University of Virginia Health System
Program Director Charlottesville, Virginia
Internal Medicine Residency Program
California Pacific Medical Center
San Francisco, California Consulting Contributor
P. Preston Reynolds, MD, PhD, FACP1
Rosemarie L. Conigliaro, MD, FACP1
Professor of Medicine
Professor of Medicine
Division of General Medicine, Geriatrics and Palliative
Senior Assistant Dean for Curriculum
Care
University of Kentucky College of Medicine Center for Biomedical Ethics and Humanities
Lexington, Kentucky University of Virginia Health System
Rosanne Granieri, MD, FACP1 Charlottesville, Virginia
Professor of Medicine
Division of General Internal Medicine
Editor-in-Chief
Department of Medicine
Patrick C. Alguire, MD, FACP1
University' of Pittsburgh School of Medicine
Senior Vice President, Medical Education
Pittsburgh, Pennsylvania
American College of Physicians
Eric H. Green, MD, MSc, FACP1 Philadelphia, Pennsylvania
Associate Professor of Clinical Medicine
Drexel University College of Medicine
Philadelphia, Pennsylvania Deputy Editor-in-Chief
Associate Program Director Philip A. Masters, MD, FACP1
Internal Medicine Residency Program Senior Medical Associate for Content Development
Mercy Catholic Medical Center American College of Physicians
Darby, Pennsylvania Philadelphia, Pennsylvania
Senior Medical Associate for Content Becky Krumm1
Development Senior Staff Editor

Cynthia D. Smith, MD, FACP2 Ellen McDonald, PhD1


American College of Physicians Senior Staff Editor
Philadelphia, Pennsylvania
Katie Idell1
Senior Staff Editor
General Internal Medicine
Randy Hendrickson1
Clinical Editor Production Administrator/Editor
Michele Heisler, MD1
Megan Zborowski1
Staff Editor
General Internal Medicine Reviewers
Linnea Donnarumma1
Stewart Babbott, MD, FACP1 Assistant Editor
Elizabeth A. Cerceo, MD, FACP1
John K. Chamberlain, MD, MACP1 John Haefele1
Timi Edeki, MD2 Assistant Editor
Douglas Einstadter, MD, MPH, FACP1 Developed by the American College of Physicians
Stephanie L. Elkins, MD2
Richard M. Hoffman, MD, MPH, FACP2
1. Has no relationships with any entity producing, marketing, re-selling, or distributing
Medha Munshi, MD2 health care goods or services consumed by. or used on, patients.
Asher Tulsky, MD, FACP1 2. Has disclosed relationships with entities producing, marketing, re-selling, or distributing
health care goods or services consumed by, or used on, patients. See below.
Peter H. Wiernik, MD, FACP2

General Internal Medicine Reviewers Conflicts of Interest


Representing the American Society for The following committee members, reviewers, and ACP
Clinical Pharmacology & Therapeutics staff members have disclosed relationships with commercial

John Thomas Callaghan, MD, PhD2 companies:


Anne N. Nafziger, MD, FACP2 John Thomas Callaghan, MD
Employment
Eli Lilly & Co. (Retiree)
General Internal Medicine
Stock Options/Holdings
ACP Editorial Staff Eli Lilly, Abbott, Isis
Becky Krumm1, Senior Staff Editor Consultantship
Sean McKinney1, Director, Self-Assessment Programs Marcadia, Biogen Idee
Margaret Wells1, Managing Editor
Timi Edeki, MD
John Haefele1, Assistant Editor
Employment
AstraZeneca
ACP Principal Staff Stock Options/Holdings
Patrick C. Alguire, MD, FACP1 Abbott Laboratories
Senior Vice President, Medical Education
Stephanie L. Elkins, MD
D. Theresa Kanya, MBA1 Speakers Bureau
Vice President, Medical Education Celgene, Cephalon Oncology, GlaxoSmithKline

Sean McKinney1 Richard M. Hoffman, MD, MPH, FACP


Director, Self-Assessment Programs Employment
Foundation for Informed Medical Decision Making
Margaret Wells1 Resca rch Gra n ts/Con tra cts
Managing Editor NIH
Valerie Dangovetsky1 Royalties
Program Administrator UpToDate

IV
Christopher L. Knight, MD, FACP online answer sheets for print subscribers; access to MKSAP
Royalties 16 Digital, Board Basics 3, and MKSAP 16 Updates; the
Oakstone Medical Publishing latest details on Continuing Medical Education (CME) and
Maintenance of Certification (MOC) in the United States,
Medha Munshi, MD
Canada, and Australia; errata; and other new information.
Consultantship
Novartis (spouse), Celgene (spouse), Millenium (spouse)
ABIM Maintenance of Certification
Anne N. Nafziger, MD, FACP
Check the MKSAP Resource Site (mksap.acponline.org) for
Consultantship
Bertino Consulting the latest information on how MKSAP tests can be used to
apply to the American Board of Internal Medicine for
Cynthia D. Smith, MD, FACP Maintenance of Certification (MOC) points.
Stock Options/Holdings
Merck and Company
RCPSC Maintenance of Certification
Peter H. Wiernik, MD, FACP
In Canada, MKSAP 16 is an Accredited Self-Assessment
Honoraria
Program (Section 3) as defined by the Maintenance of
Celgene Certification Program of The Royal College of Physicians
and Surgeons of Canada (RCPSC) and approved by the
Canadian Society of Internal Medicine on December 9,
Acknowledgments
2011. Approval of Part A sections of MKSAP 16 extends
The American College of Physicians (ACP) gratefully
from July 31, 2012, until July 31, 2015. Approval of Part B
acknowledges the special contributions to the development sections of MKSAP 16 extends from December 31, 2012,
and production of the 16th edition of the Medical
to December 31, 2015. Fellows of the Royal College may
Knowledge Self-Assessment Program (MKSAP 16) made earn three credits per hour for participating in MKSAP 16
by the following people: under Section 3. MKSAP 16 will enable Fellows to earn up
Graphic Services: Michael Ripca (Technical to 75% of their required 400 credits during the 5-year
Administrator/Graphic Designer) and Willie-Fetchko MOC cycle. A Fellow can achieve this 75% level by earning
Graphic Design (Graphic Designer). 100 of the maximum of 174 AMA PRA Category 1
Credits available in MKSAP 16. MKSAP 16 also meets
Production/Systems: Dan Hoffmann (Director, Web Services
multiple CanMEDS Roles for RCPSC MOC, including that
& Systems Development), Neil Kohl (Senior Architect), and
of Medical Expert, Communicator, Collaborator, Manager,
Scott Hurd (Senior Systems Analyst/Developer).
Health Advocate, Scholar, and Professional. For informa
MKSAP 16 Digital: Under the direction of Steven Spadt, tion on how to apply MKSAP 16 CME credits to RCPSC
Vice President, ACP Digital Products & Services, the digital xVIOC, visit the MKSAP Resource Site at
version of MKSAP 16 was developed within the ACP's mksap.acponline.org.
Digital Product Development Department, led by Brian
Sweigard (Director). Other members of the team included The Royal Australasian College of
Sean O'Donnell (Senior Architect), Dan Barron (Senior
Physicians CPD Program
Systems Analyst/Developer), Chris Forrest (Senior Software
In Australia, MKSAP 16 is a Category 3 program that
Developer/Design Lead), Jon Laing (Senior Web
Application Developer), Brad Lord (Senior Web may be used by Fellows of The Royal Australasian College
of Physicians (RACP) to meet mandatory CPD points.
Developer), John McKnight (Senior Web Developer),
and Nate Pershall (Senior Web Developer). Two CPD credits are awarded for each of the 174 AMA
PRA Category 1 Credits available in MKSAP 16. More
The College also wishes to acknowledge that many other information about using MKSAP 16 for this purpose is
persons, too numerous to mention, have contributed to the available at the MKSAP Resource Site at mksap.acpon-
production of this program. Without their dedicated efforts, line.org and at www.racp.edu.au. CPD credits earned
this program would not have been possible. through MKSAP 16 should be reported at the MyCPD
site at www.racp.edu.au/mycpd.

Introducing the MKSAP Resource Site


(mksap.acponline.org) Continuing Medical Education
The MKSAP Resource Site (mksap.acponline.org) is a con The American College of Physicians is accredited by the
tinually updated site that provides links to MKSAP 16 Accreditation Council for Continuing Medical Education
(ACCME) to provide continuing medical education for your test for same-day CME credits. There is no addi
physicians. tional fee for this service.
The American College of Physicians designates this endur 3. Pay a S10 processing fee per answer sheet and submit the
ing material, MKSAP 16, for a maximum of 174 AMA printed answer sheet at the back of this book by mail or
PRA Category 1 Credits. Physicians should claim only the lax, as instructed on the answer sheet. Make sure you cal
credit commensurate with the extent of their participation culate your score and fax the answer sheet to 215-351-
in the activity. 2799 or mail the answer sheet to Member and Customer
Service, American College of Physicians, 190 N.
Up to 24 AMA PRA Category 1 Credits are available
from December 31, 2012, to December 31, 2015, for the Independence Mall West, Philadelphia, PA 19106-1572,
MKSAP 16 General Internal xVledicine section. using die courtesy envelope provided in your MKSAP 16
slipcase. You will need your 10-digit order number and
8-digit ACP ID number, which are printed on your pack
Learning Objectives ing slip. Please allow 4 to 6 weeks for your score report
The learning objectives of MKSAP 16 are to: to be emailed back to you. Be sure to include your email
Close gaps between actual care in your practice and pre address for a response.
ferred standards of care, based on best evidence
If you do not have a 10-digit order number and 8-digit ACP
Diagnose disease states that are less common and some
ID number or if you need help creating a username and pass
times overlooked and confusing
Improve management of comorbid conditions that can word to access die MKSAP 16 online answer sheets, go to
mksap.acponline.org or email custserv@acponline.org.
complicate patient care
Determine when to refer patients for surgery or care by
subspecialists Permission/Consent for Use of Figures
Pass the ABIM Certification Examination Shown in MKSAP 16 General Internal
Pass the ABIM Maintenance of Certification Examination
Medicine Multiple-Choice Questions
The figures shown in Self-Assessment Test Item 29, Item
Target Audience 148, and Item 61 appear courtesy of Edward A. Jaeger,
General internists and primary care physicians MD, Jefferson Medical College, Wills Eye Institute,
Subspecialists who need to remain up-to-date in internal Philadelphia, PA. The figure shown in Self-Assessment Test
medicine Item 163 is reprinted with permission from Physicians1
Residents preparing for the certifying examination in Information and Education Resource (ACP PIER).
internal medicine Philadelphia, PA: American College of Physicians.
Physicians preparing for maintenance of certification in
internal medicine (recertification)
Disclosure Policy
It is the policy of the American College of Physicians (ACP)
Earn "Same-Day" CME Credits Online to ensure balance, independence, objectivity, and scientific
For the first time, print subscribers can enter their answers rigor in all of its educational activities. To this end, and con
online to earn CME credits in 24 hours or less. You can sistent with the policies of the ACP and the Accreditation
submit your answers using online answer sheets that are Council for Continuing Medical Education (ACCME), con
provided at mksap.acponline.org, where a record of your tributors to all ACP continuing medical education activities
MKSAP 16 credits will be available. To earn CME credits, are required to disclose all relevant financial relationships
you need to answer all of the questions in a test and earn a with an}- entity producing, marketing, re-selling, or distrib
score of at least 50% correct (number of correct answers uting health care goods or services consumed by, or used
divided by the total number of questions). Take any of the on, patients. Contributors are required to use generic names
following approaches: in the discussion of therapeutic options and are required to
identify an)' unapproved, off-label, or investigative use of
1. Use the printed answer sheet at the back of this book to
commercial products or devices. Where a trade name is
record your answers. Go to mksap.acponline.org, access
the appropriate online answer sheet, transcribe your used, all available trade names for the same product type are
also included. If trade-name products manufactured by
answers, and submit your test for same-day CME credits.
There is no additional fee for this service. companies with whom contributors have relationships are
discussed, contributors are asked to provide evidence-based
2. Go to mksap.acponline.org, access the appropriate online citations in support of the discussion. The information is
answer sheet, directly enter your answers, and submit reviewed by the committee responsible for producing this

VI
text. If necessary, adjustments to topics or contributors' that may result from the use of information, publications,
roles in content development are made to balance the dis technologies, products, and/or services discussed in this
cussion. Further, all readers of this text are asked to evaluate program.
the content for evidence of commercial bias and send any
relevant comments to mksap_editors@acponline.org so that
future decisions about content and contributors can be
Publisher's Information
made in light of this information. Copyright 2012 American College of Physicians. All
rights reserved.

Resolution of Conflicts This publication is protected by copyright. No part of this

To resolve all conflicts of interest and influences of vested publication may be reproduced, stored in a retrieval system,
or transmitted in any form or by any means, electronic or
interests, the ACP precluded members of the content-cre
ation committee from deciding on any content issues that mechanical, including photocopy, without the express con
sent of the ACP. MKSAP 16 is for individual use only. Only
involved generic or trade-name products associated with
one account per subscription will be permitted for the pur
proprietary entities with which these committee members
had relationships. In addition, content was based on best pose of earning CME credits and MOC points/credits and
for other authorized uses of MKSAP 16.
evidence and updated clinical care guidelines, when such
evidence and guidelines were available. Contributors' dis
closure information can be found with the list of contribu Unauthorized Use of This Book Is
tors' names and those of ACP principal staff listed in the
Against the Law
beginning of this book.
Unauthorized reproduction of this publication is unlaw
ful. The ACP prohibits reproduction of this publication
Hospital-Based Medicine or any of its parts in any form either for individual use or
For the convenience of subscribers who provide care in hos for distribution.

pital settings, content that is specific to the hospital setting The ACP will consider granting an individual permission
has been highlighted in blue. Hospital icons (Q) highlight to reproduce only limited portions of this publication for
where the hospital-only content begins, continues over his or her own exclusive use. Send requests in writing to
more than one page, and ends. MKSAP Permissions, American College of Physicians, 190
N. Independence Mall West, Philadelphia, PA 19106-1572,
Educational Disclaimer or email your request to mksap_editors@acponline.org.

The editors and publisher of MKSAP 16 recognize that the MKSAP 16 ISBN: 978-1-938245-00-8
development of new material offers many opportunities for (General Internal Medicine) ISBN: 978-1-938245-08-4
error. Despite our best efforts, some errors may persist in
Printed in the United States of America.
print. Drug dosage schedules are, we believe, accurate and
in accordance with current standards. Readers are advised, For order information in die U.S. or Canada call 800-
however, to ensure that the recommended dosages in 523-1546, extension 2600. All other countries call 215-
MKSAP 16 concur with the information provided in the 351-2600. Fax inquiries to 215-351-2799 or email to
product information material. This is especially important in custserv@acponline.org.
cases of new, infrequently used, or highly toxic drugs.
Application of the information in MKSAP 16 remains the
Errata and Norm Tables
professional responsibility of the practitioner.
Errata for MKSAP 16 will be available through the MKSAP
The primary purpose of MKSAP 16 is educational.
Resource Site at mksap.acponline.org as new information
Information presented, as well as publications, technolo
becomes known to die editors.
gies, products, and/or services discussed, is intended to
inform subscribers about the knowledge, techniques, and MKSAP 16 Performance Interpretation Guidelines with
experiences of the contributors. A diversity of professional Norm Tables, available July 31, 2013, will reflect the
opinion exists, and the views of the contributors are their knowledge of physicians who have completed the self-
own and not those of the ACP. Inclusion of any material assessment tests before the program was published. These
in the program does not constitute endorsement or rec physicians took the tests without being able to refer to the
ommendation by the ACP. The ACP does not warrant the syllabus, answers, and critiques. For your convenience, the
safety, reliability, accuracy, completeness, or usefulness of tables are available in a printable PDF file through the
and disclaims any and all liability for damages and claims MKSAP Resource Site at mksap.acponline.org.

VII
Table of Contents

Interpretation of the Medical Literature Professionalism and Ethics


Study Design 1 Professionalism 22
Threats to Va l i d i t y 1 Decision-Making and Informed Consent 24
Informed Consent 24
Experimental Studies 1
Observational Studies 1 Assessing Decision-Making Capacity 24
Advance Directives and Surrogate Decision-Making . 24
Other Study Designs 3
Statistical Analysis 3 Withholding or Withdrawing Treatment 25

Sources of Evidence 4 Physician-Assisted Suicide and Euthanasia 25


C o n fi d e n t i a l i t y 25
C o n fl i c t s of Interest 26
Routine Care of the Healthy Patient Medical Error Disclosure 26
Important Health Care Initiatives and Trends 4 Sexual Contact between Physician and Patient 27
Screening 6 The Impaired Physician and Colleague Responsibility ... 27
Screening During the History and Physical
Examination 8
Palliative Care
Periodic Health Examination 8
Introduction 27
S p e c i fi c Screening Te s t s 8
Deciding When Hospice Palliative Care Is Indicated .... 28
F a m i l y H i s t o r y a n d G e n e t i c Te s t i n g 11 Assessment and Communication 29
Ta k i n g a Family History 11 Symptom Management 29
Caveats to Genetic Te s t i n g 11 Pain 29
R e f e r r a l f o r G e n e t i c C o u n s e l i n g 11 Constipation 30
Immunization 12 Fatigue 30
Vaccinations Recommended for All Adults 13 Dyspnea 32
Vaccinations Recommended for Some Adults 13 Nausea 32
Lifestyle Risk Factors 16 Anorexia and Nutrition 32
Behavioral Counseling 16 Depression 32
Diet and Physical Activity 16 Delirium 32
Substance Use Disorders 17 Bereavement and Grief. 32
Sexual Behavior 18
Domestic Violence 18 Common Symptoms
Overview 32
Patient Safety Chronic Noncancer Pain 33
Assessment 33
Introduction 19
Management 34
Principles of Patient Safety 19
Cough 36
Quality Improvement Models 19 Acute Cough 37
Measurement of Quality Improvement 20
Subacute and Chronic Cough 37
Sources of Error 20
Cough in die Immunocompromised Patient 39
Diagnostic Errors 20 Hemoptysis 39
Medication Errors 20 Chronic Fatigue and Chronic Fatigue Syndrome 39
Transitions of Care 20
Diagnosis and Evaluation of Chronic Fatigue 39
Health Information Technology and Patient Safety 22 Management of Chronic Fatigue and Chronic
National Patient Safety Goals 22 Fatigue Syndrome 40

IX
Dizziness 40 Iliotibial Band Syndrome 60
Vertigo 40 Baker Cyst 61
Presyncope 43 Aikle and Foot Pain 61
Dysequilibrium 43 Aiklc Sprains 61
N o n s p e c i fi c Dizziness 43 Hind Foot Pain 61
Insomnia 43 Midfoot Pain 62
Evaluation of Insomnia 44 Forefoot Pain 62
Management of Insomnia 44
Syncope 45 Dyslipidemia
Neurocardiogenic Syncope 45 Screening 62
Orthostatic Hypotension 46 Evaluation of Lipid Levels 62
Cardiac Causes of Syncope 46 LDL Cholesterol 62
Diagnostic Evaluation of Syncope 46 Triglycerides 63
Risk Stratification and Management of Syncope .... 47 HDL Cholesterol 64
Chest Pain 47 Nonstandard Lipid Risk Factors 64
Differential Diagnosis 47 Management of Dyslipidemias 64
Chest Pain and Decision to Hospitalize 49
Therapeutic Lifestyle Changes 64
Edema 50
Drug Therapy 64
Combination Drug Therapy 65
Musculoskeletal Pain Management of Hypertriglyceridemia 66
Acute Low Back Pain 50 Management of Low HDL Cholesterol 67
Diagnosis and Evaluation 50 Metabolic Syndrome 67
Treatment 51 Epidemiology and Pathophysiology 67
Neck Pain 52 Management of Metabolic Syndrome 67
Diagnosis and Evaluation 52 Dyslipidemia Management in Older Patients 68
Treatment 53 Dyslipidemia Management and Stroke Prevention 69
Shoulder Pain 53 Aspirin as an Adjunct to Dyslipidemia Management 69
Diagnosis and Evaluation 53
Rotator Cuff Disorders 54 Obesity
Adhesive Capsulitis 55 D e fi n i t i o n and Epidemiology 69
Acromioclavicular Joint Degeneration 56 Screening and Evaluation 70
Elbow Pain 56 Treatment 71
Diagnosis and Evaluation 56 Lifestyle M o d i fi c a t i o n 72
Epicondylitis 56 Pharmacologic Therapy 72
Olecranon Bursitis 56 Surgery 73
Ulnar Nerve Entrapment 56
Wrist and Hand Pain 57 Men's Health
Indications for Imaging 57 Male Sexual Dysfunction 74
Carpal Tunnel Syndrome 57 Erectile Dysfunction 74
O t h e r C a u s e s o f Wr i s t a n d H a n d P a i n 5 7 Premature Ejaculation 76
Hip Pain 58 Decreased Libido 77
Diagnosis and Evaluation 58 Androgen D e fi c i e n c y 77
S p e c i fi c C a u s e s o f H i p P a i n 58 Benign Prostatic Hyperplasia 78
Knee Pain 58 A c u t e Te s t i c u l a r a n d S c r o t a l P a i n 7 8
Diagnosis and Evaluation 58 Te s t i c u l a r To r s i o n 79
Degenerative Joint Disease 59 Epididymitis 79
Trauma 59 Hydrocele and Va r i c o c e l e 79
Patellofemoral Pain Syndrome 59 Epididymal Cyst 79
Bursitis 60 Va r i c o c e l e 79
Acute and Chronic Prostatitis and Pelvic Pain 80 Glaucoma 96
Hernia 81 Primary Open Angle Glaucoma 96
Acute Angle Closure Glaucoma 97
Women's Health Cataract 97
Female Sexual Dysfunction 81 Dry Eye 97
Approach to die Patient 81 Excessive Te a r i n g 98
Classification of Female Sexual Disorders 81 Retinal Detachment 98
Evaluation of a Breast Mass 82 Retinal Va s c u l a r Occlusion 99
Clinical Presentation 83 Central Retinal Vein Occlusion 99
Evaluation 83 Central Retinal Artery Occlusion 99
Breast Pain 84 Eye Emergencies 99
Clinical Presentation 84
Evaluation 84 Ear, Nose, Mouth, and Throat Disorders
Treatment 84 Evaluation of Hearing Loss 100
Contraception 84 Tinnitus 101
Oral Contraceptive Pills 84 Otitis Media 102
Long-Acting Contraceptives 86 Otitis Externa 102
Intrauterine Devices 86 Cerumen Impaction 103
Barrier Methods 86
Upper Respirator)' Tract Infections 103
Emergency Contraception 86 Sinusitis 103
Sterilization 86
Allergic Rhinitis 103
Preconception Counseling 86
Nonallcrgic Rhinitis 104
Menopause 87 Pharyngitis 104
Overview 87
Epistaxis 105
M a n a g e m e n t o f Va s o m o t o r S y m p t o m s 88 Oral Health 105
Management of Urogenital Symptoms 89 Oral Infections and Ulcers 106
Abnormal Uterine Bleeding 90
Dental Infection 106
Clinical Presentation and Evaluation 90
Halitosis 106
Management 90
To n g u e Syndromes 106
Dysmenorrhea 90
Burning Mouth Syndrome 106
Chronic Pelvic Pain 91
Te m p o r o m a n d i b u l a r Disorders 106
Va g i n i t i s 92
Clinical Presentation and Evaluation 92
Anorectal Disorders
Bacterial Va g i n o s i s 92
Trichomoniasis 92 Approach to the Patient with Anorectal Disorders 107
Hemorrhoids and Rectal Bleeding 107
Vulvovaginal Candidiasis 93
Anal Fissure 108
Anorectal Abscess 108
Eye Disorders
Chronic Anorectal Pain 108
Red Eye 93
Clinical Evaluation 93 Pruritus Ani 108

Conjunctivitis 93
Mental and Behavioral Health
Subconjunctival Hematoma 94
Corneal Conditions 94 Depression 108
Episcleritis and Scleritis 95 Diagnosis of Depressive Disorders 108
Uveitis 95 Management of Depression 109
Blepharitis 95 Aixiety Disorders 11 0
Macular Degeneration 96 Posttraumatic Stress Disorder Ill
Pathophysiology and Clinical Presentation 96 Social Anxiety Disorder Ill
Treatment 96 Obsessive-Compulsive Disorder Ill
Intermittent Explosive Disorder 11 2 Perioperative Medicine
Bipolar Disorder 11 2 General Recommendations 124
Somatoform Disorders 11 2
Perioperative Te s t i n g 124
C l i n i c a l P r e s e n t a t i o n a n d E v a l u a t i o n 11 2
Perioperative Medication Management 125
Management 11 3 Cardiovascular Perioperative Management 125
Eating Disorders 11 3 Pulmonary Perioperative Management 128
Types of Eating Disorders 11 3 Hematologic Perioperative Management 129
Medical Complications of Eating Disorders 114 Ve n o u s T h r o m b o e m b o l i s m P r o p h y l a x i s 1 2 9
Tr e a t m e n t o f E a t i n g D i s o r d e r s 11 4
Perioperative Management of Warfarin Therapy ... 129
Schizophrenia 11 4 Perioperative Management of Antiplatelet
A t t e n t i o n - D e fi c i t / H y p e r a c t i v i t y D i s o r d e r 11 5 Medications, Coagulopathies, and
Autism Spectrum Disorders 11 5 Thrombocytopenia 130
D i f fi c u l t Patient Encounters 11 6 Perioperative Management of Anemia 131
Perioperative Management of Endocrine Diseases 132
Geriatric Medicine Diabetes Mellitus 132
Functional Assessment 11 6 Thyroid Disease 132
F a l l P r e v e n t i o n a n d H o m e S a f e t y 11 7 Adrenal I n s u f fi c i e n c y 132
Mild Cognitive Impairment and Dementia 118 Perioperative Management of Kidney Disease 133
Depression 11 9 Perioperative Management of Liver Disease 133
Hearing 11 9 Perioperative Management of Neurologic Disease 134
Vision 11 9
The Older Driver 120 Bibliography 134
Levels of Care 120
Polypharmacy 120 Self-Assessment Te s t 141
Urinary Incontinence 121
Epidemiology 121 Index 259
Evaluation 121
Treatment 121
Pressure Ulcers 123
Clinical Presentation 123
Prevention and Management 123

XII
General Internal Medicine
High-Value Care Recommendations
The American College of Physicians, in collaboration with The American Cancer Society does not recommend using
multiple other organizations, is embarking on a national ini MRI for breast cancer screening in average-risk women
tiative to promote awareness about the importance of stew and finds the evidence regarding breast self-examination
ardship of health care resources. The goals are to improve to be insufficient.
health care outcomes by providing care of proven benefit Owing to poor specificity, cervical cancer screening with
and reducing costs by avoiding unnecessary and even harm human papillomavirus (HPV) DNA testing alone is not
ful interventions. The initiative comprises several programs recommended, although clinicians can consider using
that integrate die important concept of health care value HPV DNA testing along with cervical cytology in women
(balancing clinical benefit with costs and harms) for a given age 30 years and older to help guide further investigation
intervention into various educational materials to address die and decrease the frequency of testing (see Item 34).
needs of trainees, practicing physicians, and patients. Owing to limitations of currently available screening tests
and unclear benefits of screening, prostate cancer screen
To integrate discussion of high-value, cost-conscious care into
MKSAP 16, we have created recommendations based on the ing remains controversial (see Item 49).
The American College of Physicians and the American
medical knowledge content that we feel meet the below defi
nition of high-value care and bring us closer to our goal of Academy of Family Physicians both recommend diat clini
cians have individualized discussions widi dieir patients
improving patient outcomes while conserving finite resources.
regarding obtaining prostate-specific antigen (PSA) meas
High-Value Care Recommendation: A recommendation to urements and support obtaining PSA levels after such dis
choose diagnostic and management strategies lor patients in cussions in patients 50 years and older who have life
specific clinical situations diat balances clinical benefit widi cost expectancies of at least 10 years (see Item 49).
and harms with die goal of improving patient outcomes. Palliative care consultation programs are associated widi

Below are the High-Value Care Recommendations for the significant hospital cost savings, widi an adjusted net sav
General Internal Medicine section of MKSAP 16. ings of SI 696 in direct costs for patients discharged alive
from the hospital and $4908 net savings for patients dying
The value of the periodic health examination for healthy, in die hospital as compared with patients who receive
asymptomatic adults is debatable and there is no consen usual care (see Item 76).
sus interval. Evidence suggests that more aggressive care at the end of
The U.S. Preventive Services Task Force recommends lifewhether prolonged hospitalization, intensive care
against screening for the following conditions: carotid unit admission, or performance of proceduresdoes not
artery stenosis, COPD, hereditary hemochromatosis, and improve either quality or duration of life (see Item 89).
peripheral arterial disease. Feeding tubes are not recommended for terminal cancer.
According to the U.S. Preventive Services Task Force, There is no specific role for diagnostic testing in the assess
screening for coronary artery disease is not recommended ment and management of chronic noncancer pain because
in low-risk persons, and the evidence for screening high- abnormalities that are identified may not be die source of
risk persons is inconclusive, as is the evidence for screening the patient's pain (see Item 117).
using nontraditional risk (actors, such as high-sensitivity Routine antibiotic treatment of uncomplicated upper res
C-rcactive protein, homocysteine level, Lp(a) lipoprotein piratory tract infections and acute bronchitis in nonelderly
level, anklc-brachial index, carotid intima-media thickness, immunocompetent patients is not recommended (see
and coronary artery calcium score. Item 13).
The U.S. Preventive Services Task Force recommends Patients with chronic fatigue for longer than 1 mondi
against routine screening for hepatitis C virus infection in rarely have abnormalities on either physical or laboratory
the general population. evaluation; testing should thus be judicious and per
The U.S. Preventive Services Task Force recommends formed only when clearly indicated (see Item 7).
against screening for asymptomatic bacteriuria in men and Patients with chronic fatigue syndrome should have regu
nonpregnant women. lar follow-up to monitor their symptoms, for support and
Evidence for the benefits of screening mammography is validation, and to avoid unnecessary diagnostic and treat
lacking for women age 75 years and older. ment interventions.

XIII
Tests with the lowest likelihood of affecting diagnosis or Initial evaluation of women with dysmenorrhea includes a
management of syncope include head CT scan, carotid thorough history, with particular attention to sexual activ
Doppler ultrasonography, electroencephalography, and ity and risks for abuse or infection; unless pelvic pathology
cardiac enzyme levels; these studies may be indicated if is suspected (previous radiation, trauma, infection, foreign
symptoms point to specific etiologies but otherwise body), treatment may be initiated without further evalua
should be omitted from the work-up (see Item 86 and tion (see Item 131).
Item 156). There is no role for antibiotic eye drops in the treatment
Neurocardiogenic and ordiostatic syncope both are gener of viral conjunctivitis (see Item 11).
ally benign in nature and do not require hospitalization Treatment of allergic conjunctivitis includes oral antihista
(see Item 86). mines, topical antihistamines, and artificial tears; antibiotic
Patients with nonspecific low back pain and no symptoms treatment is not indicated.
or signs to suggest systemic illness should not routinely Imaging of the central nervous system is not considered
receive additional diagnostic testing (see Item 91). part of a routine evaluation for bilateral hearing loss.
Mechanical neck pain outside of die setting of acute For die diagnosis of sinusitis, imaging is rarely necessary in
trauma rarely requires imaging, although plain films can an average-risk patient; however, it should be considered
be helpful in patients older than 50 years to exclude in immunocompromised patients at risk for unusual
malignancy and to assess for osteoarthritic changes (see organisms, such as fungal or pseudomonal sinusitis (see
Item 37). Item 107).
A repeat lipid screening interval of 5 years is considered Empiric antibiotic treatment for acute pharyngitis not
appropriate in low-risk patients, with a shorter interval in based on a clinical decision tool (such as the Centor crite
those with borderline results and a longer interval in diose ria) should be discouraged (see Item 161).
with consistently normal results. In the management of epistaxis, unless the patient has
Several nontraditional risk factors may be related to car severe bleeding or has an associated systemic disease, labo
diovascular outcomes, including levels of Lp(a) lipopro ratory studies and imaging are usually not necessary (see
tein, small LDL particles, HDL subspecies, apolipopro- Item 4).
teins B and A-l, and the total cholesterol/HDL In patients with dental infections without cellulitis or sys
cholesterol ratio; however, the U.S. Preventive Services temic symptoms, antibiotic therapy is not necessary if den
Task Force and updated National Cholesterol Education tal intervention can be performed within several days.
Program Adult Treatment Panel III (ATP III) guidelines Visualizing a hemorrhoid or odier source of rectal bleed
do not recommend measuring or treating any of these risk ing in a low-risk patient younger than 40 years widiout
factors when managing lipid levels (see Item 66). other symptoms to suggest inflammatory bowel disease or
The benefits of statin therapy are generally class specific, colon cancer may spare the patient further endoscopic
and there is no compelling evidence that newer agents are evaluation (see Item 48).
more effective than established statin medications, which When a somatoform disorder is suspected, laboratory and
may be more cost effective. other testing should be ordered logically to evaluate plau
Because the risk of significant liver or muscle damage is sible medical diagnoses; extensive and elaborate testing to
very low in patients on statin therapy, routine follow-up explore unsupported or very unlikely diagnoses should be
testing is not indicated and should be performed based on avoided (see Item 85).
die development of symptoms or other clinical findings Frequent, routine review to verify need for medication and
while on therapy (sec Item 158). appropriate dosing is an important aspect of optimal geri
Cardiovascular primary prevention with statin therapy in atric care (see Item 46).
older patients (ages 65-80 years) is controversial. Interventions to prevent pressure ulcers are much more
Because many episodes of erectile dysfunction are tran cost effective than the prolonged and intensive efforts
sient, an extensive laboratory evaluation is not mandatory required for treatment of existing ulcers (see Item 24).
at presentation without symptoms or findings suggestive Comprehensive batteries of laboratory testing, chest radi
of an underlying systemic disorder or before implement ographs, and electrocardiograms should not be routinely
ing lifestyle modification or counseling therapy. performed in the preoperative setting without specific
In adult men, asymptomatic hydroceles and varicoceles indication as they may result in further testing, delay sur
can usually be diagnosed clinically and generally do not gery, and add expense, and such testing rarely influences
require advanced imaging or treatment. perioperative care (see Item 8).
Intrauterine devices (IUDs) combine the highest contra Preoperative tests should be based on known or suspected
ceptive efficacy (typical failure rate <1%) with the lowest comorbidities and should only be ordered when a result
cost. will alter management (see Item 8).

XIV
Comprehensive preoperative testing has not been shown required as a routine component of preoperative evalua
to be helpful in cataract surgery and is not endorsed by tion (see Item 8).
any major specialty' society or payor. Blood transfusion is reserved for patients with sympto
Preoperative pulmonary function testing should be reserved matic anemia, a preoperative hemoglobin concentration
for patients with unexplained dyspnea (see Item 47). below 6 g/dL, postoperative hemoglobin concentration
Laboratory testing for underlying bleeding disorders and below 7 g/dL, or patients with symptomatic cardiovascu
anemia should be reserved for patients in whom there is a lar disease and hemoglobin concentrations between 6 and
reasonable probability of an abnormal test and is not 10 g/dL.

xv
General Internal Medicine

Experimental Studies
Interpretation of the In an experimental study, patient selection, treatment, and
Medical Literature analysis are determined from the outset to minimize error and
bias. In addition, many experimental studies blind patients,
Study Design treating physicians, and investigators to which treatment a
patient is receiving in order to reduce bias, because knowledge
Threats to Validity of which treatment a patient receives can affect patient report
Investigators attempt to infer validity, or "truth," by com ing and investigator assessment of outcomes. To minimize
paring two groups of people in a well-designed study. Many bias associated with measurement, assessors of clinical out
factors can threaten a study's validity, including errors in comes are typically blinded even if patients and treating physi
measurement, data collection, selection of subjects, or analy cians cannot be blinded.
sis. Internal validity refers to the degree to which the inves Various types of study designs are compared in Table 1.
tigators' conclusions (usually implying cause and effect) are In a randomized controlled trial (RCT), randomization is per
supported by the study External validity refers to the gen- formed in an effort to distribute all potential prognostic fac
eralizability of the study. There is always some error in any tors equally across both the experimental and die control
research study. Error is typically shown in scientific publica groups, minimizing confounding and bias. Although histori
tions by way of a confidence interval (CI), typically the 95% cally, most RCTs compared a new intervention with a placebo,
confidence interval, which signifies that the investigators can RCTs can also compare a new therapy with an existing one.
be 95% certain that the value derived from a study truly lies The objective of these trials, which are often used when the
within that interval. If die CI is wide, the point estimate is new therapy is less costly or easier to use, is typically to prove
said to have less certainty, or precision; this is often due to that the new dierapy is "noninferior" to accepted therapies.
small sample size. A small sample size can also decrease the These studies require careful attention to the power of the

power of a study. Power is the probability of detecting a dif study; studies with small numbers of patients can mask a true
ference between two groups when a true difference exists. difference.
Random error due to chance alone can sometimes result in Well-designed RCTs typically have a high degree of pre
cision and internal validity'. However, RCTs are typically
uneven distribution of patient characteristics, affecting study
conducted on patients with a narrow spectrum of disease
results. When error is not random, but is applied differen
and use treatment protocols that may be difficult to imple
tially to one group, it is called bias. Bias can occur in selec
ment outside of a research setting. Therefore, many RCTs
tion of patients, measurement, and analysis. Selection bias
lack generalizability.
occurs when patients chosen for a study group have charac
Two odier experimental study designs are used when ran
teristics that can affect the results of the study. Bias can be
domization of individual patients is unfeasible or unethical (for
minimized using carefully constructed research protocols
example, evaluating a new patient safety initiative). In a quasi-
that ensure that the comparison groups arc selected, meas
experimental study design, data can be compared in the same
ured, and analyzed in the same way. group of patients bodi before and after an intervention. In
Another major challenge in interpreting studies is con cluster-randomized studies, groups of patients are random
founding. A confounder is a diird factor that influences bodi ized, rather than individual patients.
exposure (treatment) and outcome. For example, die false
conclusion that smokeless tobacco in the form of snuff poses Observational Studies
a greater risk for developing coronary artery disease (CAD) In an observational study, the investigator has no role in
dian cigarette smoking could occur because being male (die
assigning individuals to interventions, but radier compares the
confounder, which is associated with CAD) is more likely effects of exposures or treatments among two or more
among snuff users than among cigarette smokers. A failure to observed groups. By their very nature, observational studies
recognize the presence of a confounder distorts the cause- are more susceptible to bias and confounding than experi
effect relationship. Confounding can be minimized by using mental studies. Strengths of observational studies, however,
a randomized study design or widi statistical techniques if ran include dieir ability to include a broader spectrum of disease
domization is not possible. (and diseases or exposures diat are rare) and diat treatments
Interpretation of the Medical Literature

table 1. Types of Study Designs

Study Design Description Strengths Weaknesses Key Threats to Validity


Experimental Studies

Randomized controlled Patients receive one Strongest design for Expensive, If randomization is ineffective
trial (RCT) of two interventions, determining causation time-consuming, If data are not analyzed
often one being a not practical for many
clinical situations according to initially assigned
placebo
group
Limited follow-up If key individuals are aware of
duration
group assignment (not
Limited number of blinded)
outcomes that can If follow-up is incomplete
be assessed
Limited generalizability
Cluster-randomized trial Patients grouped by Same as for RCTs Same as for RCTs Same as for RCTs
clusters (e.g., nursing
Can be used if Challenging to analyze If analysis does not account
unit) rather than
randomization of for clustering
assigned randomly
patients is not ethical
or feasible

Quasi-experimental desigr Review of data Can be used if Patients not If no adjustment for possible
collected before and randomization of randomized confounding
after an intervention patients is not ethical
or feasible

Observational Studies

Cohort study Studies outcomes of Able to detect Requires complicated Selection bias in cohort
groups using observed associations, but these statistical techniques to
are not always cause- minimize confounding Bias in measurement of
assignment
effect relationships exposures and outcomes
Prospective designs can
Able to study multiple be expensive and take If important confounders not
outcomes over a long many years before accounted for
period of time results are available
Large sample size
Case-control study Compares past Useful for rare High risk for bias Selection bias, especially in
exposures in patients diseases or exposures controls
with and without High risk for
disease Inexpensive confounding Measurement bias, especially
recall bias
Cannot assess
incidence/prevalence

are administered in a "real-world" environment. Two types of bias, especially recall bias, as patients with disease may be
observational studies are cohort studies and case-control more likely to remember previous exposures. Careful atten
studies. A cohort study compares the outcomes of groups tion is needed in both measurement of exposures and selec
with and without exposures or treatments not initiated by the tion of controls.
investigator; for example, rates of lung cancer between smok Other observational study designs are limited in their
ers and nonsmokers. Cohorts are compared by following ability to establish causality but may be useful as relatively
them forward in time (prospectively) or by looking backward inexpensive means of generating hypotheses for future
in time (retrospectively); prospective design minimizes recall research or for determining estimates of prevalence of a dis
bias (inaccurate recall of past events). ease. A cross-sectional study assesses for both exposure and
A case-control study retrospectively compares die expe disease at the same time point (rather than prospectively or
rience of patients who have a disease with those who do not retrospectively). A case series is a report of clinical outcomes
have the disease. For example, patients with and without in a group of patients; the absence of a control group prevents
lung cancer can be compared with respect to their exposure any conclusions about the effectiveness of the treatment.
to asbestos. Case-control studies are particularly useful to Epidemiologic studies compare outcomes, in aggregate, of
study rare diseases or diseases that occur many years after two different populations (countries, socioeconomic groups).
specific exposures. These studies are highly susceptible to These studies are potentially subject to the ecologic fallacy,
Interpretation of the Medical Literature

erroneously assuming that population-level associations imply KEY POINTS


individual-level associations. Internal validity refers to the degree to which die inves
tigators' conclusions are supported by the study; exter
Other Study Designs nal validity refers to die generalizability of die study.
Systematic reviews summarize existing experimental or obser Bias occurs when systematic differences between
vational studies in a rigorous way. Systematic reviews are char
acterized by a focused clinical question, exhaustive review of groups affect the outcome of a study.
the published literature, a systematic protocol for selecting Meta-analyses are considered the highest quality
articles and abstracting data (often utilizing independent sources of evidence, followed (in descending order)
reviewers), qualitative or quantitative combination of the by qualitative systematic reviews, RCTs, cohort stud
results, and narrative summary of strengths and limitations of ies, and case-control studies.
the analysis. Systematic reviews that quantitatively combine
data are called meta-analyses. The strength of systematic
reviews lies in combining the data from many small studies to Statistical Analysis
minimize the impact of random error. Potential weaknesses, Published studies of therapies typically express results in eidier
in addition to the weaknesses of the composite studies, relative or absolute terms (Table 2). Relative comparisons,
include study identification and selection variability in the such as relative risk, odds ratio, and hazard ratio, compare the
design of die included studies. ratio between rates of events (such as death or hospitalizations)
In general, meta-analyses are considered die highest in two groups. Absolute comparisons, such as absolute risk dif
quality sources of evidence, followed (in descending order) by ferences, express absolute differences in rates of disease or
qualitative systematic reviews, RCTs, cohort studies, and case- events in two groups. Relative risk differences may exaggerate
control studies. Within each category, however, quality may die impact of an intervention, especially for relatively uncom
vary, so a well-designed prospective cohort study may be mon outcomes, For example, an intervention that reduces die
superior to a small RCT with a high rate of drop-out. rate of a disease from 20% to 10% and an intervention diat
Comparative effectiveness research (CER) is intended to reduces it from 2% to 1% each have a relative risk reduction
produce evidence that can help patients, physicians, and pol (RRR) of 50%. However, the absolute risk reduction (ARR)
icy makers better understand the effectiveness, benefits, and for the first case is 10%, whereas the ARR. for the second is 1%.
harms of treatments or procedures. CER employs systematic The effect size, a measure of the general impact of an inter
reviews as well as new clinical trials to determine effectiveness vention, is best characterized by the number needed to treat
in routine clinical practice. (NNT), which is number of patients needed to receive a

TA B L E 2 . C o m m o n Te r m s U s e d i n t h e I n t e r p r e t a t i o n o f t h e M e d i c a l L i t e r a t u r e f o r T h e r a p e u t i c s 1
Term D e fi n i t i o n Calculation Notes

Absolute risk The probability of an event occurring in a AR = patients with event in Also known as event rate; can be
(AR) group during a specified time period group/total patients in group for benefits or harms. Often, an
experimental event rate (EER) is
compared with a control event rate
(CER)
Relative risk (RR) The ratio of the probability of developing RR-EER/CER Used in cohort studies and
a disease with a risk factor present to the randomized controlled trials
probability of developing the disease
without the risk factor present Any two ARs can be substituted for
EER and CER
Absolute risk The difference in rates of events between ARR = | EER-CER | Any two ARs can be substituted for
reduction (ARR) experimental group (EER) and control EER and CER
group (CER)
Relative risk The ratio of absolute risk reduction to the RRR = | EER - CER \l CER Any two ARs can be substituted for
reduction (RRR) event rate among controls EER and CER
Number needed Number of patients needed to receive a N N T- 1 / A R R A good estimate of the effect size
to treat (NNT) treatment for one additional patient to
benefit
Number needed Number of patients needed to receive a NNH-1/ARI ARI = absolute risk increase and
to harm (NNH) treatment for one additional patient to equals | EER - CER | when the event
be harmed is an unfavorable outcome (e.g.,
drug side effect)
Routine Care of the Healthy Patient

treatment for one additional patient to be expected to benefit KEY POINTS


from die intervention. NNT is the reciprocal of the ARR. In
Defining risk in absolute terms and calculating num
the example above, the NNT in the first case is 10 (1/0.10)
ber needed to treat/harm is the best way to under
and in the second case, 100 (1/0.01). Number needed to
stand the magnitude of difference in the sample
harm (NNH) is die reciprocal of the absolute risk increase
(effect size).
(ARI), and reflects die number of patients diat would need to
The positive predictive value of a test increases widi
bc treated to expect one of diem to be harmed by the inter
vention. Defining risk in absolute terms and calculating die increasing prevalence of the disease being tested for,
whereas the negative predictive value increases with
NNT/NNH is the best way to understand the magnitude of
difference in die sample (effect size). decreasing prevalence of the disease.
A P value allows the reader to assess how likely any dif
ference seen is due to chance alone. For example, a P value of
less than 0.05 (corresponding to a less than 1 in 20 chance of
Sources of Evidence
The Cochrane collection offers a single source for systematic
getting the results found in the trial assuming there is no dif
ference between die treatments) is often accepted as a cut-off reviews (www.mrw.interscience.wiley.com/cochrane/);
for statistically significant results. Pvalues are related to both PubMcd's clinical queries page offers assistance for common
the degree of difference found between groups and the num clinical searches (wwAv.ncbi.nlm.nih.gov/pubmed/clinical),
ber of patients in a study, so studies with many patients often including searches for systematic re\ie\vs and odier clinical study
produce highly statistically significant results. However, it is categories; and PubMed (www.ncbi.nlm.nih.gov/pubmed/)
can be used for more comprehensive searches. Textbooks and
important to recognize that differences found to be statisti
review articles typically offer "predigested" evidence on a clin
cally significant in large trials may not be clinically important.
For example, a large study could find a statistically significant ical topic, although they lack the formal rigor of a systematic
decrease in an event rate from 0.2% in the control group to review. Guidelines, in contrast, allow groups of experts to syn
0.1% in the treatment group. However, the ARR would be thesize and interpret available evidence. The audiors grade
their recommendations based on the quality of die evidence
0.1%, yielding a NNT of 1000.
reviewed. As guidelines are evidence-supported expert opin
Many measures are used to define the properties of diag
nostic tests (Table 3), including sensitivity, specificity, and ion, there are often differences between guidelines put out by
different organizations. These differences may reflect the
predictive value. Understanding these characteristics for a
unconscious biases of the authors and sponsoring organiza
given test is essential in knowing how effective and helpful a
tion. The American College of Physicians (ACP) publishes
study will be in the diagnostic process. For example, die sen
clinical practice guidelines, guidance statements, and best
sitivity is an indicator of the ability of a test to detect a disease
if it is present, and the specificity reflects how effectively a test practice advice based on reviews of available evidence
can exclude illness in a patient without the disease. Bodi sen (www.acponline.org/clinical_information/guidelines). A
useful compilation of guidelines is also available at
sitivity and specificity are properties of the test itself and do
not vary with the prevalence of disease. The predictive value www.guidelines.gov. Critical evaluation tools and e\idence-
based medicine (EBM) calculators are available at
of a test, however, does vary according to the prevalence of
www.cebm.net. Other EBM resources include ACP's PIER
die disease. For example, consider a test with 90% sensitivity
and 90% specificity. In a population in which 80% of people (http://picr.acponline.org), ACP Journal Club (www
have the disease (prevalence of 80%), positive test results will .acpjc.org), ACP JournalWise (www.journalwise.org), and
Clinical Evidence (http://clinicalevidence.bmj.com).
be true positives 97% of the time (positive predictive value
[PPV]); however, negative test results will be true negatives
only 69% of the time (negative predictive value [NPV]).
Conversely, if only 8% of the population has die disease Routine Care of the
(prevalence of 8%), the PPV of the test is 44% and the NPV
is 99%. Healthy Patient
The Bayes theorem uses sensitivity, specificity, and the
pretest probability of a disease to calculate the posttest prob Important Health Care Initiatives
ability of die disease. The likelihood ratio (LR) is die ratio of and Trends
the probability of a particular test result (positive or negative) Many recent national initiatives attempt to improve qualitv
among patients widi a disease to the probability of that same and safety of patient care, standardize practice patterns, pro
result among patients without the disease. The pretest odds vide more transparency about hospital outcomes, and nudge
of a disease multiplied by the LR equals the posttest odds of heterogeneous health care systems into the electronic age
the disease. The posttest odds can be converted to a percent (Table 4). In a health care system with many stakeholders
age to yield die more commonly used posttest probability. widi varying and sometimes conflicting goals, there is an
Routine Care of the Healthy Patient

table 3. Common Terms Used in the Interpretation of the Medical Literature for Diagnostic Tests
Term D e fi n i t i o n Calculation Notes

Prevalence Proportion of patients with Prev = (TP + FN) / (TP + FP + FN + TN)


(Prev) disease in the population

Sensitivity (Sn) Proportion of patients with Sn = TP/(TP + FN)


disease who have a positive test

Specificity (Sp) Proportion of patients without Sp = TN/(FP + TN)


disease who have a negative test
Positive Proportion of patients with a PPV = TP/(TP + FP) Increases with increasing
predictive positive test who have disease prevalence
value (PPV)

Negative Proportion of patients with a NPV = TN / (TN + FN) Increases with decreasing
predictive negative test who do not have prevalence
value (NPV) disease
Positive The likelihood that a positive test LR+ = Sn / (1 - Sp)
likelihood result would be expected in a
ratio (LR+) patient with the disease
that a positive test result would
be expected in a patient without
a disease

Negative The likelihood that a negative LR- = (1-Sn)/Sp


likelihood test result would be expected in
ratio (LR-) a patient with the disease
compared with the likelihood
that a negative test result would
be expected in a patient without
a disease
Pretest odds The odds that a patient has the Pretest odds
pretest odds-
- 1 pretest
_ p_etestprobability_
probabj|ity
disease before the test is
performed
Posttest odds The odds that a patient has the Posttest odds = pretest odds x LR LR+ is used if result of test is
disease after a test is performed positive; LR- is used if result of test
is negative.
A nomogram is available to
calculate posttest probability using
pretest probability and LR without
having to convert pretest
probability to odds (see
www.cebm.net/index.aspx7o-1043)
Pretest Proportion of patients with the Pretest probability can be estimated
probability disease before a test is performed from population prevalence, clinical
risk calculators, or clinical experience
if no evidence-based tools exist
Posttest Proportion of patients with the Posttestr probability
' 1 + p o= s posttestodds
ttest odds
probability disease after a test is performed

FN = false negative; FP = false positive; TN = true negative; TP = true positive.

increasing drive to stress valuehealth outcomes achieved and obesity, as well as die costs associated widi potential adverse
per dollar spentfor the patient above all else. effects of treatments.
Value in health care is defined by the outcomes achieved Frail elderly persons with limited life expectancies
radier than by the volumes of procedures performed or services should have different outcome measures than younger
rendered. To determine value, die measurement of processes adults. For example, seeking to achieve a hemoglobin Alc
and dieir improvement is trumped by the measurement of clin level below 7% in an 80-year-old woman with multiple
ical outcomes and die economic costs of attaining diose out comorbidities who is recently diagnosed with diabetes mel
comes. This measurement can be complex. Eor example, meas litus would have less value (in terms of improved clinical out
urement of die outcome of care of a patient with cardiovascular comes, potential adverse effects, and economic costs) than
disease should include the costs of treating comorbidities or achieving that clinical target in a young adult recently diag
contributory risk factors, such as hypertension, hyperlipidemia, nosed with diabetes.
Routine Care of the Healthy Patient

table 4. Important Health Care Initiatives, Organizations, and Terms

Term D e fi n i t i o n Additional Resources

Diabetes A voluntary NCQA program designed to provide www.ncqa.org/tabid/139/Default.aspx


Recognition clinicians with tools to support the delivery and
Program (DRP) recognition of consistent high quality care in diabetes
Electronic Health Medicare program to provide incentive payments to physicians www.cms.gov/ehrincentiveprograms
Record (EHR) and hospitals that demonstrate meaningful use of certified
incentive program EHR technology
Healthcare Measures that are widely used across health systems www.ncqa.org/tabid/187/default.aspx
Effectiveness Data (e.g., p-blocker use after myocardial infarction or breast
and Information Set cancer screening)
(HEDIS)
Hospital Compare U.S. Health and Human Services (HHS) Web site that compares hospitalcompare.hhs.gov
Medicare data between hospitals
Physician Quality The 2006 Tax Relief and Health Care Act required the www.cms.gov/pqri
Reporting System establishment of a physician quality reporting system,
(PQRS [formerly including an incentive payment for eligible professionals
PQRI1) who satisfactorily report data on quality measures for covered
professional services furnished to Medicare beneficiaries
National Committee Not-for-profit organization dedicated to improving health www.ncqa.org/tabid/675/Default.aspx
for Quality care quality
Assurance (NCQA)
Accountable care A formally organized entity comprising physicians (primary www.acponline.org/ppvl/policies/aco.pdf
organization (ACO) care, specialists, subspecialists) and other health service
professionals that is responsible through contracts with
payers for providing a broad set of health care services to a
specific population of people. A key goal of the ACO structure
is to control growth of health care costs while maintaining or
improving quality of care.
Concierge medicine Relationship between a patient and a physician in which the www.physiciansnews.com/business/
patient pays an annual fee for increased access to 204.kalogredis.html
that physician
Electronic prescribing Computer-based electronic generation, transmission, www.ama-assn.org/ama/pub/physician
and filling of a prescription, taking the place of paper -resources/health-informationtechnology/
and faxed prescriptions health-it-basics/eprescribing.page
Patient-centered Team-based model of care led by primary care physician www.acponline.org/running_practice/pcmh/
medical home (PCMH) providing continuous, coordinated care for the patient
Pay for performance Payment to providers of health care based upon quality of www.ahrq.gov/qual/pay4per.htm
(P4P) care rather than just services provided

In practice, many quality performance measures assess of metrics by which they are measured, both responding
compliance with care processes rather than the outcomes to and influencing the development of specific process and
of those processes. For example, all of the more than 70 outcome care measures.
Healthcare Effectiveness Data and Information Set
(HEDIS) measures updated by the National Committee
for Quality Assurance (NCQA) each year are process Screening
measures. None are outcome measurements. This method Screening, which typically refers to the identification of a
of quality improvement attempts to standardize the pro condition in the asymptomatic state, should be reserved for
vision of care by different providers (for example, meas common conditions, with well-understood natural histories,
urement of hemoglobin Alc in patients with diabetes) but that have significant negative consequences on society and
rarely measures the actual outcomes of that care (for for which early detection provides clinical benefits. Early
example, risk of microvascular or macrovascular disease in detection and treatment of the condition should lead to
diabetes). Performance measures stressing value will increased survival or improved quality of life compared with
emphasize outcomes and the costs to achieve those out identification at a later, symptomatic stage. The screening
comes more heavily than the processes utilized in care. test needs to be acceptable to persons available for screening
Practicing physicians will need to be vigilant for the tvpes and should possess adequate sensitivity and specificity such
Routine Care of the Healthy Patient

that the frequency of false-positive and false-negative results patient anxiety; and possible harm to patients requires an
is minimized. understanding of specific screening tests and their appropri
Evaluating the effectiveness of screening tests in reduc ate use. The American College of Physicians (ACP) has
ing morbidity and mortality is best accomplished with ran developed a number of clinical practice guidelines, guidance
domized controlled trials. Three types of bias are commonly statements, and best practice statements to help understand
observed in such studies. Lead-time bias occurs when a the optimal use of specific screening tests. The U.S.
screening test leads to earlier identification of a condition, and Preventive Services Task Force (USPSTF) has systemati
an apparent improvement in 5-year survival, but does not
cally reviewed the available evidence and published evi
actually result in improved mortality. Length bias occurs dence-based recommendations on screening for a wide
when the variable rate of progression of a disease is not
range of conditions (www.uspreventiveservicestaskforce
accounted for. For example, a patient with a prolonged
.org/recommendations.htm). Table 5 summarizes these
asymptomatic phase (for example, a slowly progressing can recommendations (see Geriatric Medicine for preventive
cer) has a greater likelihood of being identified in a screening care measures specific to the geriatric patient). A Web-based
study than a patient with a more rapidly progressing cancer. and mobile application for use at the point of care to indi
This results in an apparentbut not actualsurvival benefit.
vidualize screening recommendations from the USPSTF is
Overdiagnosis m. screening refers to the identification of can
cers that are not destined to progress, thereby inflating sur available (http://epss.ahrq.gov/PDA/index.jsp). Other
vival statistics. organizations also publish focused guidelines for proper use
of screening tests, and multiple clinical calculators and risk
Balancing the benefits of screening with the potential to
identify inaccurate or insignificant findings that lead to addi assessment tools are available to assess the need for specific
tional low yield, high cost, and low value testing; increased screening tests.

table 5. Summary of USPSTF Screening Recommendations


Condition Recommendation

Depression All adults, when appropriate support system available


Alcohol misuse All adults

Obesity All adults


Hypertension All adults
Lipid disorders All men >35 years of age; consider in men 20-35 years of age with increased cardiovascular risk. Women >45 years
of age with increased cardiovascular risk; consider in women 20-45 years of age with increased cardiovascular risk.
Diabetes mellitus All adults with sustained blood pressure >135/80 mm Hg
Osteoporosis Women >65 years of age; any other woman whose fracture risk is > that of a 65-year old white woman without
additional risk factors.
Abdominal One-time screening in all men 65-75 years of age who have ever smoked
aortic aneurysm
HIV infection All persons at increased risk of HIV infection

Hepatitis B All pregnant women at the first prenatal visit


virus infection

Chlamydial All women <24 years of age who are sexually active; all women >24 years of age who are at increased risk of
infection infection.
Gonorrhea Sexually active women who are at increased risk of infection
Asymptomatic Pregnant women at 12-16 weeks' gestation or at the first prenatal visit, whichever comes first.
bacteriuria
Syphilis High-risk persons and pregnant women
Breast cancer Biennial screening mammography for average-risk women 50-74 years of age; initiation of screening between 40
and 49 years of age should be individualized.
Cervical cancer Screen with Pap smear: initiate no sooner than 21 years of age; test every 3 years thereafter or, for women aged 30-
65 years who want to lengthen the duration of screening, every 5 years if combined with HPV testing. Screening is
not indicated in women following hysterectomy and without previous high-risk Pap smears. Screening may be
discontinued at age 65 years in non-high-risk women with no recent abnormal Pap smears.
Colon cancer All adults 50-75 years of age (see MKSAP 16 Gastroenterology and Hepatology)

HPV = human papillomavirus; USPSTF = U.S. Preventive Services Task Force.


Routine Care of the Healthy Patient

Screening During the History and Physical Periodic Health Examination


Examination Although most Americans view the periodic health examina
The USPSTF recommends screening adults for depression tion as essential to high quality care, die value of the periodic
when appropriate supports are available for accurate diag health examination for healthy, asymptomatic adults is debat
nosis, treatment, and follow up. There is little evidence to able and there is no consensus interval. At die very least, it
support one screening method over another, although evi appears that the periodic health examination improves the
dence suggests that asking two questions ("Over the past 2 delivery of some preventive services and reduces patient
weeks, have you felt down, depressed, or hopeless?" and worry. Executive periodic physical examinations, frequently
"Over the past 2 weeks, have you felt little interest or pleas performed on behalf of many corporations, remain contro
ure in doing things?") will detect almost all cases of signifi versial without clear evidence of benefit.
cant depression and may be as effective as longer instru
ments. There is also evidence that asking only one question, Specific Screening Tests
"Have you felt sad or depressed much of the time during the The USPSTF strongly recommends screening for dyslipi
past year?" is also effective. A positive response to any of demia in certain populations based on age, sex, and cardio
these questions requires additional assessment to determine vascular risk factors (see Table 5). The National Cholesterol
diagnosis and treatment. Education Program's Adult Treatment Program III recom
Several organizations, including the USPSTF, the mends obtaining a fasting lipid profile once every 5 years in
American Medical Association, the American Society of all adults over die age of 20 years with a normal initial lipid
Addiction Medicine, and the Canadian Task Force on
profile (see Dyslipidemia for specific lipid values).
Preventive Health Care recommend screening and perform The USPSTF recommends screening for type 2 dia
ing behavioral counseling interventions to reduce alcohol betes in asymptomatic adults with a sustained blood pressure
misuse by adults. The USPSTF recommends that all adults be of greater than 135/80 mm Hg. In contrast, the American
asked about tobacco use. Aldiough the USPSTF concludes Diabetes Association recommends screening all adults age
that there is insufficient evidence to recommend screening for 45 years and older without risk factors and all adults with a
illicit substance use, they also conclude that there is little evi BMI of equal to or greater than 25 who have one or more
dence of harms associated with either screening or subsequent of the following risk factors: gestational diabetes, hyperten
behavioral interventions. Screening for alcohol misuse, smok sion, hyperlipidemia, and family history of type 2 diabetes in
ing, and drug use is discussed further in Lifestyle Risk Factors. a first-degree relative. Appropriate screening tests include a
Owing to the increasing rate of obesity in our society, fasting plasma glucose level, hemoglobin Alc level, or a 2-h,
the USPSTF recommends screening all adults for obesity 75-g oral glucose tolerance test. The USPSTF concludes
and, for those determined to be obese, to offer intensive that there is insufficient evidence regarding screening for
counseling and behavioral interventions to promote sus gestational diabetes.
tained weight loss. The USPSTF recommends screening for osteoporosis in
The USPSTF recommends screening all adults age 18 all women age 65 years or older and also in younger women
years and older for hypertension but concludes that evidence with an elevated fracture risk. Although die USPSTF con
is lacking regarding the optimal interval of screening. The cludes that evidence is insufficient to recommend screening in
Seventh Report of the Joint National Committee on men, ACP practice guidelines from 2008 recommend screen
Prevention, Detection, Evaluation, and Treatment of High ing those men who are at increased risk. The USPSTF cur
Blood Pressure (JNC 7) recommends screening every 2 years rently has no guidelines regarding screening for vitamin D
for those with blood pressures of less than 120/80 mm Hg deficiency.
and every year for those with systolic blood pressures of 120 According to the USPSTF, screening for abdominal aor
to 139 mm Hg and diastolic blood pressures of 80 to 89 mm tic aneurysm (AAA) should be performed with abdominal
Hg. According to JNC 7 guidelines, the mean of two or more ultrasonograph}' on a one-time basis in all men between the
seated clinic measurements should be used to make the diag ages of 65 and 75 years who have ever smoked (defined as
nosis of hypertension. A recent study confirms the impor 100 lifetime cigarettes). They make no recommendation for
tance of using multiple measurements to make the diagnosis or against screening men who have never smoked and rec
of hypertension and the potential error in using only one ommend against routine screening for AAA in women
measurement. regardless of smoking history.
Because of insufficient evidence, die USPSTF does not The USPSTF recommends against screening for the
recommend for or against screening for glaucoma in adults or following conditions: carotid artery stenosis, COPD, hered
for either visual acuity or dementia in older adults. There was itary hemochromatosis, and peripheral arterial disease.
also insufficient evidence for the USPSTF to recommend for Screening for coronary artery disease is not recommended in
or against screening for family and intimate partner violence low-risk persons, and the evidence for screening high-risk per
among children, women, and older adults. sons is inconclusive, as is the evidence for screening using
Routine Care of the Healthy Patient

nontraditional risk factors, such as high-sensitivity C-reactive breast cancer can be determined using the breast cancer risk
protein level, homocysteine level, Lp(a) lipoprotein level, assessment tool (www.cancer.gov/bcrisktool), which is based
ankle-brachial index, carotid intima-mcdia thickness, and on die Gail model.
coronary artery calcium score. Owing to insufficient evi Mammography, the most widely used method of screen
dence, the USPSTF does not recommend for or against ing for breast cancer, is the only available breast cancer screen
screening for thyroid disease. ing modality that has been shown to reduce mortality, with
die strongest evidence for its use being in average-risk women
Screening for Infectious Diseases between the ages of 50 and 69 years. Screening mammogra
The Centers for Disease Control and Prevention (CDC) rec phy may be less effective for average-risk women between die
ommend that all persons between the ages of 13 and 64 ages of 40 and 49 years owing to the lower incidence of breast
years be screened for HIV infection, whereas the USPSTF cancer and lower accuracy of mammography in this age
recommends screening only persons at increased risk of group. Whereas two randomized controlled trials have
infection. In addition, the USPSTF recommends that all demonstrated the benefits of screening mammography in
pregnant women be screened for HIV infection. average-risk women between the ages of 70 and 74 years, evi
The USPSTF strongly recommends screening for hepati dence for the benefits of screening mammography is lacking
tis B virus infection in all pregnant women at their first prena for women age 75 years and older.
tal visit but otherwise recommends against routine screening. The optimal frequency of mammography screening is
The USPSTF recommends against routine screening for unclear for all age groups. For average-risk women, the USPSTF
hepatitis C virus infection in the general population and found recommends performing biennial screening mammography
insufficient evidence to recommend for or against screening in for women ages 50 to 74 years. This approach yields a
persons at increased risk. The CDC recommends screening median breast cancer reduction of 16.5% compared widi no
persons at increased risk of infection (history of illicit injection screening (number needed to invite [NNI] to screening to
drug use, history of receiving clotting factors before 1987 or prevent one breast cancer death = 1339, ages 50-59 years;
blood products or organs before 1992, or on chronic NNI = 377, ages 60-69 years). In contrast, the USPSTF
hemodialysis at any time). Additionally, the CDC recommends states that initiation of screening between die ages of 40 and
one-time testing for baby boomers (born 1945-1965) regard 49 years (NNI = 1904) should be individualized, taking into
less of risk factors, followed by a brief screening for alcohol use account both the patient's risk and die testing characteristics
in diose identified as having hepatitis C virus infection. of mammography. The ACP provides similar recommenda
The USPSTF recommends screening for chlamydial tions for women in their 40s. A recent study supports these
infection in all women 24 years of age or younger who are recommendations, finding that mammography performed
sexually active and all women older than 24 years who are at every 2 years was effective in women with relatively high
increased risk of infection (history of sexually transmitted breast density or who possess additional risk factors for devel
infection [STI], new or multiple sexual partners, inconsistent oping breast cancer. The American Cancer Society' (ACS),
condom use, or exchanging sex for drugs or money). There American Medical Association, American College of
is insufficient evidence to recommend for or against chlamy Obstetricians and Gynecologists (ACOG), and the National
dial infection screening in men. Comprehensive Cancer Network (NCCN) all recommend
According to the USPSTF, screening for gonorrhea beginning annual mammography at the age of 40 years. The
infection should be limited to sexually active women who are ACS does not recommend using MRI for breast cancer
at increased risk of infection (same risk factors as for chlamy screening in average-risk women and finds the evidence
dial infection). The task force recommends against screening regarding breast self-examination to be insufficient. No men
low-risk men and women and states that there is insufficient tion is made of what age to stop annual screening mammog
evidence for screening in all other groups. raphy in the ACOG, ACS, or NCCN guidelines.
Although the USPSTF recommends against screening Women who are either BRCA1 or BRCA2 mutation
for asymptomatic bacteriuria in men and nonpregnant carriers have a high risk of developing breast cancer by the
women, it supports screening with urine culture for pregnant age of 70 years (65% and 45%, respectively). Despite most
women at 12 to 16 weeks' gestation or at the first prenatal patients having a family history of breast cancer, less than
visit, whichever comes first. Syphilis screening is recom 10% of all women with breast cancer have an inherited
mended in all high-risk persons and in all pregnant women genetic mutation. Features of the family history that should
but not in other persons. raise concern for an inherited syndrome include having mul
tiple affected family members, young age at time of diag
Cancer Screening Tests nosis, and the presence of multiple primary tumors. The
Breast Cancer Screening USPSTF and NCCN both recommend referral to a geneti
Age is the most important risk factor for women developing cist if concern exists for an inherited syndrome. The ACS
breast cancer. A woman's individual risk of developing recommends screening with annual MRI alternating every
Routine Care of the Healthy Patient

6 months with annual mammography for women with a Prostate Cancer Screening
greater than 25% lifetime risk of developing breast cancer Owing to limitations of currently available screening tests and
beginning at the age of 30 years and continuing as long as unclear benefits of screening, prostate cancer screening
the woman is in good health. remains controversial. The two most commonly used meth
Based on the results of the Study of Tamoxifen and ods of screening for prostate cancer include prostate-specific
Raloxifene (STAR) trial, which demonstrated a 50% reduc
antigen (PSA) measurement and digital rectal examination
tion in the incidence of hormone receptor-invasive breast
(DRE), with the former being more sensitive.
cancer with the use of these agents, postmenopausal women The USPSTF has concluded that the harms of screen
with a >1.66 risk of developing breast cancer over the next 5
ing for prostate cancer outweigh the benefits in men of any
years based on the Gail model should be offered a 5-year age regardless of risk factors. In contrast, the ACS and
course of either tamoxifen or raloxifene. American Urological Association (AUA) recommend offer
ing both PSA measurement and DRE to men on an annual
Cervical and Anal Cancer Screening
basis beginning at the age of 50 years. The ACP and
Screening with conventional cervical cytology (Pap smear) American Academy of Family Physicians (AAFP) both rec
results in a 95% decrease in mortality from cervical cancer.
ommend that clinicians have individualized discussions with
Table 6 summarizes the recommendations from major pro
their patients regarding obtaining PSA measurements and
fessional organizations. Owing to poor specificity', screening
with human papillomavirus (HPV) DNA testing alone is not support obtaining PSA levels after such discussions in
patients 50 years and older who have life expectancies of at
recommended, although clinicians can consider using HPV
DNA testing along with cervical cytology in women age 30 least 10 years. Men should be informed about the gaps in
the evidence and should be assisted in considering their per
years and older to help guide fiirdier investigation and
decrease the frequency of testing. sonal preferences before deciding whether or not to be
If cervical cytology is interpreted as unsatisfactory, the tested. The presence of benign prostatic hyperplasia symp
test should be immediately repeated. When interpreted as toms should not increase the propensity to screen for
atypical squamous cells of undetermined significance, prostate cancer with PSA testing.
acceptable options include referring for colposcopy, obtain
ing HPV DNA testing and then referring for colposcopy if Additional Cancer Screening Tests
positive, or repeating die Pap smear in 6 to 12 months. With The USPSTF recommends against routine screening for
any other abnormal result, the patient should be referred for bladder, ovarian, and pancreatic cancers. The USPSTF con
colposcopy. cludes that the evidence is insufficient to recommend for or
Although some experts recommend anal cytologic against screening for skin and oral cancer. The USPSTF rec
screening for anal cancer in high-risk persons (persons ommends that fair-skinned persons aged 10 to 24 years be
who practice receptive anal intercourse, have known anal counseled about reducing dieir exposure to ultraviolet radia
HPV infection, or who have HIV infection), the USPSTF, tion to reduce die risk of skin cancer. Colon cancer screening
ACS, and the CDC do not currently recommend such is discussed in depdi in MKSAP 16 Gastroenterology and
screening. Hepatology.

table 6. Recommendations for Cervical Cancer Screening from Major Professional Organizations
Professional Age to Initiate Screening Interval Age to Discontinue Screening
Organization Screening

American Cancer Society 21 y Ages 21-29 y: cytology alone every 3 y 65 y if no recent abnormal Pap
smears and no risk factors
Ages 30-65 y: cytology alone every 3 y, or
cytology with HPV co-testing every 5 y
U.S. Preventive Services 21 y Every 3 y, or every 5 y with HPV co-testing for 65 y if no recent abnormal Pap
Task Force women ages 30-65 y smears and no risk factors
American College of 21 y Ages 21-29 y: cytology alone every 2 y 65-70 y if three or more consecutive
Obstetrics and Gynecology negative Pap smears, including no
Ages 30-65 y: cytology alone every 3 y if three abnormal Pap smears within the
consecutive Pap smears have been negative and
no risk factors past 10 years, and no risk factors

NOTE: Risk factors for cervical cancer: history of in utero diethylstilbestrol exposure, immunocompromise, HIV positivity.

HPV = human papillomavirus.

10
Routine Care of the Healthy Patient

KEY POINTS family medical issues, desire to hide or protect, anxiety about
The U.S. Preventive Services Task Force recommends subsequent testing or procedures, and fear of psychological
harm. Provider barriers include limited clinic time, lack of
periodic screening for alcohol misuse, smoking, obe
tools to analyze and interpret data, and lack of understanding
sity, and hypertension in all adults and for hyperlipi
of die implications of the data, especially regarding necessity
demia, osteoporosis (women), and abdominal aortic
of further genetic testing.
aneurysm (men, one-time screening) based on age
and risk factors.
Caveats to Genetic Testing
The U.S. Preventive Services Task Force recommends
The advent of greater availability to the public of genetic tests
pregnant women be screened for HIV infection, hep
atitis B virus infection, syphilis, and asymptomatic through direct-to-consumer marketing of personal DNA
bacteriuria. analysis is likely to have an effect on the process of genetic
testing and counseling. Current recommendations support
The U.S. Preventive Services Task Force recommends
the concept that knowledge of one's DNA is only one com
screening persons at increased risk for sexually trans ponent of a complex process. As demand for genetic knowl
mitted infections for HrV infection, hepatitis C virus
edge increases, primary care providers may be asked to weigh
infection, chlamydial infection (women), and gonor in on issues for which they were not specifically trained. The
rhea infection (women). American College of Medical Genetics recommends that a
The U.S. Preventive Services Task Force recommends
knowledgeable health care provider be involved in the process
periodic screening for breast cancer (women >50 of ordering and interpreting genetic tests, that die patient be
years of age), cervical cancer, and colon cancer (50- informed about what the test can actually reveal, and that
75 years of age). attention be paid to privacy issues as well as die potential psy
chological impact of knowing that one is a carrier of a genetic
disease.
Family History and Genetic Specific components of genetic testing should include
Testing the following: ediical considerations, including patient auton
omy regarding the decision to test and the amount of infor
Taking a Family History mation the patient wishes to know (the right "not to know");
Family history has an important role in the practice of medi
legal considerations, including informed consent and confi
cine; it can motivate behavior change, help prevent or predict
dentiality; and issues of potential genetic discrimination by die
disease, and improve health outcomes. As many common dis
employment or insurance sectors. In addition, religious and
eases have genetic contributions, obtaining a family history
cultural considerations may come into play. Patients should be
may identify patients who would benefit from genetic testing. made aware that not all genetic testing and counseling is cov
For die clinician, the details of die family history help estab
ered by insurance plans. Finally, providers may be faced with
lish the risk (or pretest probability) of genetically associated
disease in an individual patient, and forms the basis upon family members who do not wish to disclose information to
others who may be affected, thus creating an ediical conflict
which clinical decisions regarding the need for additional test
for the provider.
ing may be made.
The definition of family may differ among patients, so
Referral for Genetic Counseling
clarification may be necessary. The accuracy of the informa
tion obtained may vary based on die method by which it is Controversy exists as to whether primary care providers have
obtained (for example, through standardized form, com the training to provide adequate genetic counseling.

puter, or in person) as well as on patient characteristics. However, die supply of trained genetic specialists may cur-
Key elements of a family history include number of rela rendy be inadequate for providing all recommended genetic
tives affected by a condition, relationship degree, gender, age counseling services. Different types of genetic tests require
at onset, ethnicity, and lineage (maternal or paternal). different levels of genetic counseling. In more routine testing,
Additional demographic and environmental data may be in which adequate physician training and patient education
needed, as these may modify risk. Available evidence suggests materials are available (for example, routine prenatal testing
diat patients are better at reporting information on first-degree for a-fetoprotein and markers for inborn errors of metabo
relatives and are more accurate in reporting the absence than lism), referral may not be necessary. Recommendations for
the presence of diseases. Older persons report more accurate referral for genetic counseling include situations in which
family histories dian younger persons; women and persons patients, families, or providers require assistance in interpret
widi higher educational levels report more information. ing family and medical histories to assess chance of disease and
Barriers to obtaining an accurate family history include whether genetic testing is an option, and to promote
patient understanding, lack of available knowledge about informed choices about furdier actions based on risk.

11
Routine Care of the Healthy Patient

Specific indications for genetic testing include the fol Immunization


lowing: to diagnose a condition in a person with signs and
Immunization is a cornerstone of bodi pediatric and adult
symptoms of a disease; to assess risk status for a family mem
ber who may be an asymptomatic carrier or at risk to develop preventive care and a major component of public health pol
a disease (for example, muscular dystrophy, cystic fibrosis, icy. Vaccination, or the administration of live, attenuated virus
or inactivated virus proteins, can protect a person from
Huntington disease); concerns about cancer risk based on
family history; an abnormal prenatal or newborn screening acquiring, having complications from, or dying of an infec
test that may indicate increased risk of a genetic disorder; con tion. Vaccination also reduces transmission and spread of a
cern about risk to a future pregnancy because of family his disease in the population at large, thereby helping even unvac-
tory, maternal age, or a previous abnormal pregnancy out cinated persons. More than 24 different vaccines are currently
come; maternal exposure to teratogens; or an ethnic available for clinical use. Recommendations for their use are
background suggesting an increased risk for a genetic disease updated frequently by die CDC's Advisory Committee on
(for example, sickle cell disease, thalassemia, Tay-Sachs dis Immunization Practices (ACIP). Current recommendations
ease, or Fanconi anemia). are available at www.cdc.gov/vaccines/pubs/ACIP-list.htm.
The genetic counseling process includes a complete Aldiough a few vaccinations are recommended for all adults,
genetic pedigree, usually of three generations; risk assessment; most are limited to persons of a specific age group or with cer
review of medical records; physical examination of die patient tain comorbid illnesses (Table 7).
and/or relatives; laboratory testing as appropriate; a discussion A comprehensive approach to vaccination requires infor
of options and potential courses of action; and a diorough mation about previous vaccination as well as current health and
explanation of the potential consequences of the genetic test comorbid conditions. Many recommendations have changed
ing process. Genetic counselors may also recommend to test or over time, and adults may need "catch-up" vaccinations.
not to test and must always provide posttest counseling. Clinicians should pay particular attention to previous vaccina
KEY POINT tion for pertussis, pneumococcal disease, human papillo
mavirus, and herpes zoster, as recommendations regarding
Recommendations regarding genetic testing include
these vaccinations are newer or have recently changed.
that a knowledgeable health care provider be involved
in die process of ordering and interpreting genetic tests, Unfortunately, patients may not remember previous vaccina
that die patient be informed about what the test can tions, and documentation of childhood vaccination may be dif
ficult to obtain. Patient self-report should only be considered
actually reveal, and that attention be paid to privacy valid for influenza and pneumococcal vaccination. For all other
issues as well as the potential psychological impact of
vaccines, clinicians should either revaccinate for age-appropri
knowing that one is a carrier of a genetic disease. ate diseases or obtain serology to document immunity viruses.

table 7. Summary of Recommendations for Vaccines for Adults3

Vaccine Type Indications

Influenza Live, attenuated or All adults


inactivated
Tetanus, diphtheria, Inactivated All adults. Booster every 10 y. One-time Tdap for all (see text)
pertussis (Td, Tdap)
Varicella Live, attenuated Persons born after 1980, HCWs, persons with T risk of disseminated varicella
without documented vaccination or immunity.
Herpes zoster Live, attenuated Adults >60y
Pneumococcal disease Inactivated Adults >65 y; adults 19-64 y with risk factors (see Table 8)
Human papillomavirus Inactivated Females 11-26 y; males 11-21 y (permitted 21-26 y)
Measles, mumps, rubella Live, attenuated Adults born after 1957 without documented vaccination or immunity. One dose
usually sufficient; second dose indicated in HCWs, international travelers,
college students, and post-exposure.
Meningococcal disease Inactivated Adolescents; persons living in dormitories; persons with HIV or asplenia
Hepatitis A Inactivated Travelers to endemic areas, men who have sex with men, users of illicit drugs,
persons with chronic liver disease.
Hepatitis B Inactivated Adults with increased risk of transmission, morbidity, or exposure (see Table 9)
HCW = health care worker.

'Full recommendations are available a www.cdc.gov/vaccines/pubs/ACIP-list.htm.

12
Routine Care of the Healthy Patient

Vaccines with live, attenuated virus are generally con influenza should be prioritized. This includes patients older
traindicated in patients with immunodeficiency, although than 50 years and those younger than 50 years who have car
most patients with HIV infection with a CD4 cell count diovascular disease, pulmonary disease, or immunodeficiency.
above 200/microlitcr can receive diese vaccines. Inactivated In addition, health care workers should be prioritized.
vaccines typically are contraindicated only when the patient Vaccination can start whenever the vaccine is available; mass
has had an allergic reaction to die vaccine. In order to achieve vaccination events should be planned starting in the fall.
a sustained immunologic response, most immunizations Clinicians should continue offering vaccination until influenza
require a scries of vaccinations. Manufacturer's guidelines for activity fades in their community, typically in March or April.
vaccines indicate the minimum interval between vaccinations, Each season, vaccines contain influenza A (including
and good immunologic response is typically seen if longer H1N1) and influenza B.
intervals are used. Thus, in patients whose vaccination series
was interrupted (for example, a patient who received only two Tetanus, Diphtheria, and Pertussis
of three injections in a series), die clinician can resume a vac Primary vaccination for tetanus and diphdieria (Td) with a
cination series where it was interrupted. Restarting a vaccina three-shot series in childhood followed by booster vaccination
tion sequence is rarely, if ever, required. Although most vac every 10 years is highly effective in preventing these diseases.
cines should not be given to a patient with a serious acute The incidence of adult pertussis has been rising, likely due in
illness, a patient with mild or moderate disease (such as upper part to a declining immunity among adults. Therefore, all
respiratory tract infection), even if associated with fever, can adults 19 years and older who have not received a dose of Tdap
be vaccinated. Multiple vaccines can be given at the same (tetanus and diphtheria combined with acellular pertussis)
time, although each should be administered at a different site. should be administered a dose regardless of die interval since
their last tetanus and diphdieria toxoid-containing vaccine
Vaccinations Recommended for All Adults (although it may also be given in place of a scheduled decen
Influenza nial Td booster). Because recent data suggest that the actual
burden of pertussis in persons older than 65 years may be at
Influenza is a respiratory virus that spreads seasonally, with
least 100 times greater than previously reported, pertussis vac
peak activity typically in the fall and winter. Because of anti
cination with at least one dose of Tdap is particularly important
genic drift, a different strain typically circulates each year, and
in this patient population. Postpartum women, health care
immunity to the previous year's influenza (vaccine-induced or
natural) is not protective of future infection. Vaccination has workers, and adults who have close contact with infants
been shown to reduce incidence, morbidity, and mortality younger than 12 months (such as child-care workers and
from influenza, and is thought to provide a public health ben grandparents) should also receive a one-time booster of Tdap
efit by reducing community spread of influenza. Two types of regardless of the timing of their last Td booster. As diis is a rel
vaccination are used: a live, attenuated vaccine and an inacti atively recent update, many adults currendy coming due for
their Td booster should more appropriately receive Tdap.
vated vaccine. Each is developed during the spring based on
projections of the most likely strains to be responsible for the
Vaccinations Recommended for Some Adults
following winter's infections. The live, attenuated vaccine is
administered as a single intranasal infusion and is approved for Varicella and Herpes Zoster
nonpregnant persons from 2 years to 49 years of age without The primary focus of varicella immunization is in pediatric
medical conditions that predispose to influenza or its compli care, and childhood vaccination has been recommended
cations (see below). The inactivated vaccine is given as a sin since 1996. Persons born before 1980 in the United States
gle intramuscular injection and is approved for adults of any are considered likely to have immunity resulting from child
age. Both types are contraindicated in patients with a history hood exposure. Persons born after 1980, health care workers,
of Guillain-Barre syndrome after influenza or severe egg and those born before 1980 who have a high risk for dissem
allergy (for example, anaphylaxis), as the vaccine is produced inated varicella should receive a two-dose varicella vaccination
in chicken eggs. Patients without a history of severe egg series unless they have serologic evidence of varicella immu
allergy can be vaccinated, although some patients should be nity or physician-documented evidence of either varicella or
observed for 30 minutes after inoculation. The inactivated varicella vaccination. Patient or parent self-report is not con
vaccine often produces local reaction at the vaccine site and sidered reliable. Childhood recommendations changed in
may induce a brief period of myalgia and low-grade fever. The 2007 from a single to a two-step varicella vaccination, and
live, attenuated vaccine frequently has side effects of rhinor some persons may need a single catch-up immunization.
rhea, headache, and cough. Between 10% and 78% of recipi Because the varicella vaccine is a live, attenuated vaccine,
ents of either vaccine will have a mild adverse effect. vaccination should be done with caution in immunocompro
Influenza vaccination is currendy recommended for all mised patients. Pregnant women should not be vaccinated.
adults, although in a time of scarcity, patients more likely to Women who become pregnant within 1 month of vaccination
become infected by or develop serious complications from should be counseled that there is a low risk of birth defects.

13
Routine Care of the Healthy Patient

A more concentrated version of the live attenuated virus functional or anatomic asplenia, HIV infection, cancer,
is used for immunization again herpes zoster. In clinical trials, advanced kidney disease, or other immunocompromising con
vaccinated patients had a much lower rate of both herpes ditions; however, the recommended dosing regimen for these
zoster and postherpetic neuralgia. All adults aged 60 years and patients is not yet available. Aldiough it is also FDA-approved
older should be vaccinated unless vaccination is contraindi for use in persons 50 years of age and older, the ACIP has not
cated because of immunodeficiency, regardless of whether yet made a recommendation for its routine use in this patient
they have experienced previous episodes of zoster. population as an alternative to die polysaccharide vaccine.

Pneumococcal Disease Human Papillomavirus


Immunization with the 23-valent pneumococcal polysaccha Human papillomavirus (HPV) is the most common STI in
ride vaccine decreases the risk of invasive pneumococcal dis die United States, and exposure to certain serotypes can lead
ease. A single dose is recommended for all persons aged 65 to genital warts and cervical cancer. Vaccination against the
years or older and those 19 to 64 years with specific risk factors most pathogenic serotcypes has been shown to reduce infec
(Table 8); these risk factors were broadened in 2010 to include tion with HPV as well as the development of precancerous
smokers and persons with asthma. Revaccination after 5 years cervical lesions and genital warts. The vaccine is an inactivated
is recommended in patients who have asplenia or immuno vaccine and is licensed for males and females between the ages
compromise, including kidney failure. Patients vaccinated of 9 and 26 years. Vaccination is recommended for all girls
before age 65 years should receive a booster at 65 years, or 5 and boys starting at age 11 or 12 years. The CDC recom
years after their initial vaccination if die)' were initially vacci mends catch-up vaccination of women to age 26 years and
nated between die ages of 60 and 64 years. There is no current men to age 21 years but permits vaccination of men up to age
recommendation for additional boosters and no recommen 26 years. Both a bivalent and quadrivalent vaccine exist; both
dation for a booster for those vaccinated after age 65 years. A are licensed and recommended for females, but only the
13-valent pneumococcal conjugate vaccine is also available. quadrivalent vaccine is recommended for males. Ideally, vac
The ACIP has approved its use in patients 19 years of age and cination should begin before a person has sexual intercourse,
older with immunocompromising conditions, defined as but vaccination is still indicated in individuals who have had

table 8. Indications for Pneumococcal Polysaccharide Vaccination in Adults 19 to 64 Years of Aqea


Patient Category Specific Indications Dosing Interval

Immunocompetent Chronic cardiovascular disease (including hypertension) Single vaccination; single booster at age
patients6 65 y or 5 y after initial vaccination,
Chronic pulmonary disease (including asthma and COPD)
whichever is later
Diabetes mellitus
Chronic liver disease (including cirrhosis)
Alcoholism

Cigarette smoking
Cerebrospinal fluid leak
Cochlear implant

Immunocompromised HIV infection Two doses separated by 5 y


Chronic kidney disease or nephrotic syndrome

Malignancy (leukemia, lymphoma, generalized malignancy)


Use of immunosuppressive treatment (corticosteroids,
antirejection medication, radiation therapy)
Multiple myeloma
Congenital or acquired immunodeficiency
Patients with Functional asplenia (sickle cell disease, other Two doses separated by 5 y
asplenia0 hemoglobinopathies)
Anatomic asplenia (congenital, surgical, others)

^Vaccination recommended for all adults aged 65 years or older.

DA 13-valent pneumococcal conjugate vaccine has been FDA approved for use in patients 50 years of age and older, although the ACIP has not made a recommendation for routine use in
this patient population.

CA 13-valent pneumococcal conjugate vaccine has been approved by the ACIP for use in patients 19 years of age and older with immunocompromising conditions, defined as functional or anatomic
asplenia, HIV infection, cancer, advanced kidney disease, or other immunocompromising conditions; however, the recommended dosing regimen for these patients is not yet available.

14
Routine Care of the Healthy Patient

intercourse or even have evidence of HPV infection, as extent of hepatitis A outbreaks among adults. Currently, hepati
immunization can prevent infection with other serotypes. The tis A vaccination is recommended for adults with increased like
vaccine is administered as a series of three injections. lihood of exposure, including travelers to endemic areas, men
who have sex widi men, and users of illicit drugs (both injected
Measles, Mumps, and Rubella and noninjected drugs). Hepatitis A vaccination is also recom
Measles, mumps, and rubella have been nearly eliminated in mended for adults widi chronic liver disease. The vaccination is
the United States after die introduction of universal child not recommended for patients widi chronic hepatitis B or C
hood vaccination. Adults born before 1957 are considered vvidiout evidence of liver dysfunction. Immunization typically
immune to these diseases. Younger adults vvidiout either a consists of two injections separated by at least 6 months.
documented history of immunization or documented immu
nity should receive a single dose of measles, mumps, and Hepatitis B
rubella (MMR) vaccine; this is especially important in women Hepatitis B is a vaccine-preventable disease with potential seri
of reproductive age. Health care workers, persons exposed to ous complications, and universal vaccination of children is cur
measles or mumps, international travelers, and students in rently recommended. In adults, vaccination is recommended
post-secondary education should receive a second vaccina for unvaccinated persons with increased risk of exposure from a
tion. MMR is a live, attenuated vaccine that is contraindicated sexual, percutaneous, or mucosal source; those planning travel
in severe immunodeficiency and pregnancy. to an endemic area; those widi increased risk for morbidity; and
any other person who requests vaccination (Table 9). Several
Meningococcal Disease vaccination strategies exist, all requiring at least three injections.
Vaccination against meningococcus is primarily recom
mended for adolescents. Unvaccinated adults living in college Special Considerations for Health Care Workers
dormitories or who are in the military should receive a single Heal til care workers have additional vaccination requirements,
dose, whereas those with asplenia, complement deficiencies, both because of their potential exposures to disease and their
or HIV infection should receive two doses. Patients with ability to act as a vector to spread diseases to or between
asplenia and those with complement deficiencies should patients. All health care workers should be vaccinated against
receive boosters every 5 years. or have serologic evidence of immunity to hepatitis B, varicella,
measles, mumps, and rubella; most employers routinely
Hepatitis A request or provide pre-employment screening and/or vacci
Universal vaccination of children against hepatitis A is currently nation. In addition, health care workers should receive a one
recommended by the ACIP. As hepatitis A epidemics are typi time Tdap vaccination. Annual influenza vaccination is also rec
cally spread by children, diis strategy provides a combination of ommended for all health care workers, and there have been
bodi individual and "herd" immunity and has decreased die efforts by some states to require these vaccinations.

table 9. Indications for Adult Hepatitis B Vaccination


Patient Category Specific Indications

Increased risk of sexual transmission More than one sex partners/6 months
Men who have sex with men
Evaluation or treatment for sexually transmitted infection
Sex partner of a patient with hepatitis B
Increased risk of percutaneous or mucosal transmission Current or recent injection drug user
Close contact of a person with hepatitis B
Resident or staff of long-term facility for developmentally disabled
Health care worker or public-safety worker who may be exposed to
blood or blood-contaminated fluids

End-stage kidney disease


Increased risk of morbidity HIV infection
Chronic liver disease
Diabetes mellitus (type 1 or 2) in ages 19-59 y (and at clinician's
discretion for age >60 y)
Others Travelers to countries with endemic hepatitis B

Anyone who requests hepatitis B vaccination

15
Routine Care of the Healthy Patient

KEY POINTS following a positive screen for unhealthy behaviors. Sessions as


Annual influenza vaccination is recommended for all short as 5 minutes, referred to as brief advice, may also be effec
tive. Brief interventions can easily be administered in a busy
adults without contraindications.
office setting, an emergency department, or odier urgent-care
Herpes zoster vaccination is recommended for all
or community-based settings. One framework for administer
adults older than 60 years without contraindications.
ing a brief intervention is the FRAMES (Feedback,
Pneumococcal polysaccharide vaccination is recom
Responsibility, Advice, Menu, Empathy, Self-efficacy) model,
mended for all adults aged 65 years or older; younger whereby clinicians provide feedback on risks, with an emphasis
persons who should receive the vaccine include those on the patient's responsibility to change behavior; clear advice,
with specific risk factors, including smokers and those with a menu of change options; support and cmpatln, and facil
with asthma. itate die patient's self-efficacy. Brief interventions may be deliv
All health care workers should be vaccinated against ered by physicians, nurses, health educators, counselors, or via
or have serologic evidence of immunity to hepatitis B, printed or computer-based information. Follow-up is critical to
assess patient progress.
varicella, measles, mumps, and rubella, receive a one
time Tdap vaccination, and receive an annual
influenza vaccination. Diet and Physical Activity
Despite a large body of literature attesting to the benefits of
counseling interventions for increasing physical activity, few
studies have been performed in the primary care setting. Most
Lifestyle Risk Factors studies that have demonstrated a benefit relied on specially
Behavioral Counseling trained health educators or nurses, counselors or psycholo
Motivational interviewing describes a patient-centered coun gists, dietitians or nutritionists, or exercise instructors or phys
seling process whereby the provider engages the patient in iologists; very few involved primary care providers.
behavior change and allows die patient to direct many aspects A recent systematic review showed diat medium- to high-
of die change process, exploring how behavior changes relate intensity dietary behavioral counseling, widi or widiout physical
to their key goals and values. Many studies attest to the suc activity counseling, resulted in small but statistically significant
cess of motivational interviewing for addressing dietary and improvements in adiposity, blood pressure, and cholesterol level,
lifestyle changes, substance abuse, and other behaviors known as well as moderate to large changes in self-reported dietary and
to be detrimental to patients' health. Many providers give physical activity behaviors. The evidence for changes in physio
patients information and advice in an effort to convince them logic outcomes was strongest for high-intensity counseling inter
to change behavior. In contrast, motivational interviewing ventions. Benefits beyond 1 year are limited and require contin
uses patient-identified issues to initiate and continue the ued high-intensity interventions, defined as frequent (monthly)
change process. Classic motivational interviewing teaches that contacts in person, by phone, or by e-mail. Several advisory
patients may be ambivalent about making behavioral changes, groups recommend 150 minutes of leisure-time physical activity
and that the purpose of motivational interviewing is to per week, usually broken into five 30-minute sessions weekly.
encourage patients themselves to identify and voice reasons Several national organizations recommend seeing a physician
for making behavioral changes. before embarking on an exercise program only if diere is a his
Components of motivational interviewing include using tory of heart disease or hvpertension, musculoskeletal disorder,
or if symptoms such as chest pain or dizziness have been experi
open-ended questions; inviting patients to consider how and
enced when trying to exercise in the past.
why they might change behavior; eliciting patients' under
standing of die problem or process and what they feel might KEY POINTS
be barriers to and facilitators of change; providing informa
Key features of motivational interviewing are that the
tion and support; and accepting their current level of com
patient chooses the agenda, the provider is not in
mitment to change. Key features of motivational interviewing "control" and does not tell the patient what he or she
are that the patient chooses the agenda, die provider is not in should do, and the provider assesses the patient's
"control" and does not tell the patient what he or she should sense of the importance of issues for him or her and
do, and the provider assesses the patient's sense of the impor level of confidence in making changes, which are usu
tance of issues for him or her and level of confidence in mak
ally small and incremental.
ing changes, which are usually small and incremental.
Motivational interviewing is one form of a brief interven Physician evaluation before embarking on an exercise
program is recommended only if there is a history of
tion, which consists of delivering clear, concise advice designed heart disease or hypertension, musculoskeletal disor
to increase patients' awareness of an unhealthy behavior and its
der, or if symptoms such as chest pain or dizziness have
negative consequences. Brief interventions usually involve one been experienced when trying to exercise in die past.
or two counseling sessions of 10 to 30 minutes each, usually

16
Routine Care of the Healthy Patient

Substance Use Disorders agreed upon and arranged. The various options offer alterna
tives for patients with contraindications, side effects, or who
Tobacco
have relapsed with one type of treatment.
Smoking is a leading cause of illness and death in the United
States. Smoking is no longer viewed as a habit but as a chronic Hospitalization provides a unique counseling opportu- ^H
disease, and scientifically validated treatments are available for nity for patients with smoking-related diseases, especially car- *
tobacco use and dependence. Thus, tobacco use assessment diovascular disease. Smoking cessation counseling that begins
should be performed for all patients. Brief advice from a during hospitalization and continues after discharge increases
the odds of long-term abstinence. The Joint Commission
physician has been shown to increase tobacco cessation rates.
Despite the availability of several FDA-approved medications (formerly JCAHO) has instituted an accreditation require
for smoking cessation, the proportion of smokers who make ment to provide tobacco cessation interventions for patients
quit attempts has increased very little, and many smokers who hospitalized for acute myocardial infarction, heart failure, and
do attempt to quit do not take advantage of these new treat pneumonia. Q
ment options. In addition, innovative and more effective
counseling strategies are needed, especially in adolescents and Alcohol
young adults. Alcohol misuse includes "risky" or "hazardous" and "harm
Current recommendations are that all clinicians assess ful" drinking patterns. Hazardous drinking is a pattern of
tobacco use at every visit, given the likely need for repeated alcohol consumption that increases the risk of harmful con
quit attempts and the importance of reinforcing positive sequences. Harmful drinking refers to alcohol consumption
behaviors. Clinicians should encourage every patient to make resulting in negative psychological, physical, and social effects
a quit attempt and counsel the patient appropriately. One sim but without meeting criteria for dependence. Alcohol
ple approach for use in the clinical setting is the 5 A's (Ask, dependence is a cluster of behavioral, cognitive, and physio
Advise, Assess, Assist, and Arrange), wherein the clinician asks logic phenomena that may develop after repeated alcohol use,
patients about their smoking status at every visit, advises them which includes a strong desire to consume alcohol, impaired
to quit, assesses their willingness/readiness to quit at this time, control over its use, persistent drinking despite harmful con
assists them with a quit plan, and arranges for follow-up. For sequences, increased tolerance, and physical withdrawal when
smokers not ready to quit, motivational interviewing, with use is discontinued. A standard alcoholic beverage is consid
emphasis on nonconfrontational strategies and discussion of ered to be one 16-ounce beer, one 5-ounce glass of wine, or
patient choices, has shown higher cessation rates than use of one shot (1.5 ounces) of spirits.
brief advice or usual care. High-intensity counseling (greater The USPSTF recommends screening and counseling for
time and number of sessions) is more effective than low- all adults for alcohol use and abuse, identifying quantity and fre
intensity strategies; for clinicians with limited time, recom
quency of drinking, adverse consequences, and patterns of use.
mendations arc to use adjunctive telephone counseling, as The Alcohol Use Disorders Identification Test (AUDIT) is the
every state in the United States has telephone counseling quit most studied screening tool for detecting alcohol-related prob
line services. In addition, counseling that addresses practical
lems in primary care settings. It consists often questions and is
problem-solving skills and social support has been shown to
easy to administer; a three-question version (the AUDIT-C) is
be more effective.
more sensitive but less specific than the ten-question AUDIT
Combining counseling with medication use is more
effective than either intervention alone. In addition, combi (www.hepatitis.ya.goy/provider/tools/audit-c.asp). The
four-item CAGE questionnaire may also be used. (Have you
nations of nicotine replacement therapy (available as nicotine
ever felt you should Cut down on your drinking? Have peo
gum, inhaler, lozenge, nasal spray, and patch) with bupropion
have been shown to be more effective than either alone. ple Annoyed you by criticizing your drinking? Have you ever
felt bad or Guilty about your drinking? Have you ever had a
Varenicline has been demonstrated to be more effective than
drink first thing in the morning to steady your nerves or get
bupropion, and combinations of varenicline with various
nicotine replacement therapies have showed cessation rates rid of a hangover [Eye-opener]?) With a cutoff of two posi
tive answers, the CAGE questionnaire is 77% to 95% sensitive
higher than nicotine replacement or bupropion alone.
Precautions for bupropion include contraindication in and 79% to 97% specific for detecting alcohol abuse or
patients on monoamine oxidase inhibitors or with seizure or dependence in primary care settings and indicates that further
eating disorders; varenicline must be used with caution in assessment is warranted. The AUDIT is more sensitive than
die CAGE questionnaire in identifying hazardous drinking
patients with kidney impairment or on dialysis, and in patients
with cardiovascular disorders; both drugs must be used with and alcohol dependence, whereas the CAGE is easier to
caution in patients with serious psychiatric illness, as they may deliver in a primary care setting. The TWEAK test, designed
cause neuropsychiatric symptoms such as personality changes, specifically for pregnant women, identifies a lower level of
vivid dreams, or suicidal ideation. The specific method or alcohol use, as any amount of alcohol may be considered haz
medication chosen is less critical than that a cessation plan is ardous to the fetus. Clinicians should choose a screening test

17
Routine Care of the Healthy Patient

appropriate to their practice, and provide counseling and Challenges unique to this population include the high likeli
intervention when patients screen positive. hood of drug dependence and polysubstance use; the legal
Effective interventions for alcohol-use disorders include ramifications of illicit drug use, and, in die case of prescription
counseling regarding appropriate amounts and negative con drug use, the distinction between appropriate and inappro
sequences, and agreeing on goals for reducing alcohol intake. priate use.
Sessions may be performed by the provider or other members
KEY POINTS
of die healdi care team and are most effective when combined
All patients who smoke should be offered cessation
with other assistance such as Alcoholics Anonymous or spe
cialty referral. Frequent follow-up and reassessment are assistance, including counseling and medical therapy.
Smoking cessation counseling that begins during hos
important.
No guidelines clearly delineate how often alcohol screen pitalization and continues after discharge increases
ing should be performed; however, all adults should be the odds of long-term abstinence.
screened and all women who are pregnant or contemplating
pregnane}' should be advised of the known ill effects of alco Sexual Behavior
hol on die fetus. Patients with a history of alcohol or other All sexually active adolescents are considered at increased risk
substance abuse are at higher risk for alcohol abuse or relapse for STIs, including chlamydial infection, hepatitis B and C,
and should be screened frequently. The effects of screening HPV, HPV, and herpes simplex virus. Adults considered at
adolescents are not known. increased risk include those with multiple sexual partners, a
current or previous (in the past year) STI, and all non-
Drugs monogamous sexually active partners in communities with
Unlike alcohol use, all illicit drug use may be considered high rates of STIs. The USPSTF recommends that providers
harmful as any use carries a risk for health and legal ramifica take a sexual history and perform risk assessment during peri
tions, even amounts and patterns of use that do not meet cri odic and other health-related visits, particularly in high-risk
teria for drug dependence. Patients with drug abuse are more
patients.
likely to experience anxiety, depression, psychosis, chronic Once risk and behavioral factors have been identified,
back and other pain disorders, peptic ulcer disease, headache, targeted counseling should be provided. Counseling issues
chronic lung disease (usually from associated tobacco use), may include numbers of partners, partner selection, what
and alcohol-related illnesses, such as pancreatitis, hepatitis, constitutes consensual sexual behavior, proper use of con
and gastritis. Additional risks associated with illicit drug use doms and other contraceptives, and appropriate vaccines
include unsafe sexual practices, STIs, and intravenous injec (HPV, hepatitis). Current USPSTF recommendations are to
tion-associated complications. provide high-intensity counseling targeted to sexually active
Prevalence of drug use (illicit and prescription) is much adolescents and adults at risk for STIs. "High-intensity"
lower than for alcohol and depends on the clinical setting. counseling refers to a single 4-hour session or a minimum of
Several screening instruments have been validated for drug three 1 -hour sessions, with longer duration or counseling up
use in the primary care setting. The Alcohol Smoking and to 16 hours or 10 sessions; this intensity is unlikely to be
Substance Involvement Screening Test (ASSIST) may be used achieved in the primary care setting. Evidence exists, however,
for tobacco, alcohol, marijuana, cocaine, stimulants, sedatives, diat as little as one or two 20-minute sessions in a primary
opioids, and several other agents. It has excellent sensitivity care office are effective for reducing STIs. Recommendations
and specificity but is considered too long for use in die pri regarding counseling for low-risk adults and non-sexually
mary care setting. The 10-item Drug Abuse Screening Test active adolescents are lacking.
(DAST-10) is similar to the AUDIT and used for assessing
drug use. A single-item screening question, "How many Domestic Violence
times in the past year have you used an illegal drug or used Domestic violence, which refers to intimate partner violence
prescription medications for nonmedical reasons?" was 100% as well as child and elder abuse, is defined as intentional con
sensitive and 74% specific in a single urban primary care prac trolling or violent behavior by a person in an intimate rela
tice for die detection of a current drug use disorder, similar tionship with the victim. Risk factors for intimate partner
to die longer DAST-10, and may be more appropriate in a violence include low socioeconomic status, young age, psy
busy primary care setting with time constraints. Currently no chiatric illness, alcohol or substance abuse, separated or
practice guidelines exist for screening patients for unhealthy divorced status, and history of childhood sexual or physical
drug use. abuse. Risk factors specific for elder abuse include increasing
Once patients have been identified as using drugs, the age, non-white race, functional impairment, cognitive dis
same interventional techniques (brief interventions, motiva ability, low self-esteem, and lack of social support. Victims of
tional interviewing) may be used for counseling, although domestic violence have a high rate of disability and somatic
data are limited regarding outcomes of these interventions. symptoms, such as chronic pain, headache, and abdominal

18
Patient Safety

pain, as well as vaginal and urinary tract infections, STIs, and from accidental injur)'." Since publication of this report in
depression. Providers are often unaware that their patients are 2000, the patient safety movement has evolved to focus on
victims of domestic violence, despite its pervasiveness in clin systems of care and the potential to do harm to patients inher
ical practice. Domestic violence is associated with high-risk ent in the structure of those systems. Emphasis is placed on
behaviors such as substance abuse and risky sexual behavior, creating systems of care that have built-in safety nets for
eating disorders, and limited access to health care. avoiding patient harm. The construction of these systems has
Recommendations for screening for domestic violence relied on the concept of the "Swiss cheese model" of medical
errors (Figure 1). In this model, bad outcomes result when
vary, as there is no gold standard screening test. Flowever, a
2012 USPSTF systematic review concluded that screening errors occur at several layers in the system. Efforts to improve
instruments can accurately identify women experiencing inti patient safety also encourage the growth and discussion of
mate partner violence, and that screening may improve healdi patient safety, wherein participants feel they can openly dis
outcomes in this population. Studies suggest that women cuss and address medical errors or concerns that an error may
occur if the system is not changed.
(who constitute the majority of victims) are comfortable with
questions from their health care providers regarding abuse.
Patients with repeated traumas, even minor, should be asked
about abuse. However, many providers may not feel com Quality Improvement Models
fortable asking, nor be versed in further counseling strategies Most health care quality' improvement programs utilize the
and legal issues if their patient is a domestic violence victim. Model for Improvement, which emphasizes methods with
Health care providers' primary responsibilities are to assist specific and measurable results. The key to quality improve
ment using this model is to establish what is to be accom
with health, assess for safety, and maintain a supportive rela
plished with a change, clarify how the results of a change will
tionship. Improved health outcomes have been shown even
be measured, and determine what changes will be made that
if victims remain in the abusive relationship when their
will lead to improvement. These changes are tested and
providers have given validation, support, empathy, and non-
implemented using the Plan-Do-Study-Act (PDSA) cycle
judgmental, patient-centered discussions about available
(Figure 2).
options and services. Many domestic violence victims are Several important quality improvement models have
helped by the acknowledgment that violence is unacceptable,
that they do have choices, and that they may proceed at their gained popularity in the United States over die past decade.
One is the "lean thinking" model developed by Toyota,
own pace.
which aims to optimize efficiency, value, and safety on a con
KEY POINT tinuous basis. Other systems sometimes utilized are Six Sigma
Health care providers' primary responsibilities in car and the Malcolm Baldrige Model for Performance
Excellence. Six Sigma is a disciplined, data-driven approach
ing for a patient who may be a victim of domestic
violence are to assist with health, assess for safety', and for identifying and removing the causes of defects (errors) and
maintain a supportive relationship. minimizing variability in patient care processes (aiming for six
standard deviations between the mean and the "customer's"
specification limit). A Six Sigma process is one in which there
are only 3.4 errors per million events. For example, in
Patient Safety
Introduction
Overall, patients in the United States receive only approxi
mately 50% of recommended preventive care measures, while
unnecessary procedures and tests continue to be performed,
partially owing to payment systems that incentivize quantity
over quality as well as to a culture that tends to demand more Error!
testing rather than less. The harms to patients from such
excessive testing are being increasingly recognized and docu
mented, such as recent reports of harm from radiation expo
sure from excessive imaging.

Principles of Patient Safety FIGURE 1. "Swiss cheese" model of error.


In the report To Err is Human: Building a Safer Health
Adapted with permission from Reason J. Human error: models and management. BMJ.
2000:320(7237):768-770. |PMID: 10720363] Copyright 2000, BMJ Publishing Group Ltd.
System, the Institute of Medicine defined safety as "freedom

19
Patient Safety

n^\
What is the goal of
the change?
(Aim)

Plan \ \
/ / Act

\ \ Study Do / /

>'
How will the change What changes can
be measured to see be made to
if quality improves? improve quality? Test Ideas and Changes
(Ideas) PDSA CYCLE
(Measures)

FIGURE 2. PDSA Model for Improvement.

assessing prescribing errors, 99.99966% of prescriptions and that tort claims for diagnostic errors are twice as common
would have no errors. The Baldrige model focuses on seven as claims for medication errors, the science of diagnostic
components: leadership, strategic planning, student and errors is in its infancy. Approximately 40% of malpractice pay
stakeholder focus, information and analysis, faculty and staff ments in 2003 were related to diagnostic errors, with average
focus, educational and support process management, and payouts of S300,000/claim. The lack of attention to this
organizational performance results. important area is largely because of the complexity' of most
diagnostic errors. Diagnostic errors tend to result from a
Measurement of Quality Improvement faulty cognitive approach (for example, settling on a diagno
Quality improvement measurement began in 1996, when the sis too soon or not re-examining new information ade
Joint Commission (formerly JCAHO) implemented the quately) or from systems-based issues (for example, commu
ORYX initiative, a national program for the measurement of nication breakdown or failure to obtain important data in a
hospital quality. This movement gathered momentum in timely way). Suggestions to help avoid diagnostic errors are
2004, when the Joint Commission began requiring the col provided in Table 10.
lection and reporting of core measure sets (myocardial infarc
tion, congestive heart failure, pneumonia, and pregnancy). A Medication Errors
set of quality indicators have been developed to measure Between 500,000 and 1.5 million preventable adverse drug
health care quality that make use of readily available hospital events occur each year in the United States, with an estimated
inpatient and outpatient administrative data. For example, 1 medication error daily for each hospitalized patient.
prevention quality indicators identify numbers of hospital Common medication errors by individual physicians include
admissions that evidence suggests could have been avoided failure to recognize drug interactions among prescribed med
through access to high-quality outpatient care. Inpatient ications or to adjust dosing to account for impaired kidnev or
quality indicators assess quality of care inside hospitals as well liver function or other conditions requiring dosing adjust
as across geographic areas, including inpatient mortality for ments. The focus of efforts to reduce medication errors has
medical conditions and surgical procedures. Patient safety been on computerized order entry, banning of abbreviations
indicators measure potentially avoidable complications and causing medication errors, bar code implementation in phar
iatrogenic events. macies, and medication reconciliation at the time of hospital
admission and discharge.

Sources of Error Transitions of Care


Diagnostic Errors Transitions of care between the inpatient and outpatient set
A diagnostic error is defined as a diagnosis that is missed, tings present unique challenges in patient safety. Forty per
delayed, or wrong; this error may or may not result in harm. cent of discharged patients will have pending studies of which
Misdiagnosis-related harm is preventable harm resulting from patients and outpatient physicians are not aware despite the
a diagnostic error. Despite the fact diat 40,000 to 80,000 need for action on them. One in five patients discharged from
deadis result from misdiagnosis annually in the United States die hospital will suffer an adverse event related to medical

20
Patient Safety

table 10. Twelve Tips for Avoiding Diagnostic Errors

Technique Comments

(1) Understand heuristics3 Availability heuristic: Diagnosing based upon what is most easily available in the
physician's mind (e.g., because of a patient recently seen) rather than what is
most probable
Anchoring heuristic: Settling on a diagnosis early in the diagnostic process
despite data that refute the diagnosis or support another (premature closure)
Representativeness heuristic: Application of pattern recognition (a patient's
presentation fits a "typical" case, therefore it must be that case)
(2) Utilize "diagnostic timeouts" Taking time to periodically review a case based on data but without assuming
that the diagnosis is that which was previously reached

(3) Practice "worst-case scenario medicine" Consider the most life-threatening diagnoses first:
Lessens chance of missing these diagnoses
Does not mandate testing for them, however

(4) Use systematic approach to common problems For example, anatomic approach to abdominal pain beginning from exterior to
interior

(5) Ask why For example, when a patient presents with diabetic ketoacidosis or a COPD
exacerbation, ask what prompted this acute exacerbation of a chronic condition
(6) Utilize the clinical examination Decreases reliance on a single test and decreases chance of premature closure

(7) Use Bayesian theory Utilize pre- and posttest probabilities


Helps avert premature closure based on a single test result

(8) Acknowledge the effect of the patient How does the patient make the physician feel?
Physicians may avoid making bad diagnoses in patients they identify with
Physicians may discount important data in patients with whom they have
difficult encounters

(9) Look for clinical findings that do not fit the Encourages a comprehensive approach and incorporates healthy skepticism
diagnosis
(10) Consider "zebras" Resist temptation to lock onto common diagnoses at risk of missing the
uncommon

(11) Slow down and reflect Difficult to do in most health care systems, which stress the economy of "getting
it right the first time"

(12) Admit mistakes Awareness of one's own fallibility may lead to fewer diagnostic errors later
Heuristics are shortcuts in reasoning used in discovery, learning, or problem solving.

Based on Trowbridge RL. Twelve tips for teaching avoidance of diagnostic errors. Med Teach. 2008;30(5):496-500. [PMID: 18576188)

management within 3 weeks of hospital discharge, 66% of pending studies. The quality of discharge summaries is
which are related to medications. Fourteen percent of elderly improved with the use of standardized content (Table 11).
CONT.
patients widi medication discrepancies between prescribed Medication reconciliation can be arduous and time-con
outpatient and discharge medications are rehospitalized suming. Information gathered at the time of hospitalization
within 30 days, compared with 6% of those without medica should include that provided by the patient, caregivers, pri
tion discrepancies. Timely follow-up with a primary care mary care providers, and, when necessary, prescription infor
physician after hospital discharge, particularly within 1 mation from the patient's outpatient pharmacy. This infor
month, leads to lower rates of ^hospitalization. mation should be entered on a standardized form or
The key components of good transitions of care include electronic record in the patient's medical chart. This form can
effective hospitalist-to-priniary care physician communica then be compared with medications prescribed throughout
tion both during hospitalization and at the time of discharge the hospitalization and at the time of discharge. Medication
(hand-off), pre-discharge patient education, medication rec reconciliation is a dynamic process that should occur
onciliation, and timely post-hospitalization follow-up. diroughout the hospital stay as new information is obtained
Despite the importance of the discharge summary, it often or medications are altered or discontinued. Patients should
fails to arrive to the primary care physician in a timely fashion receive a list of medications at the time of discharge and be
and frequently does not convey important information about informed of previous medications which have been discon
diagnosis, medications, test results, follow-up plans, or tinued or changed (and the reason for the change). When

21
Professionalism and Ethics

TABLE 11. Suggested Content of a Standardized handwriting, ensuring that the medication, dose, and direc
Discharge Summary tions are exactly what is requested. CPOE improves efficiency
Dates of admission and discharge by eliminating delays between order entry and order receipt.
A patient's EHR comprises a collection of clinical data
Reason for hospitalization
(notes, reports) in a computerized format. The EHR should
Discharge diagnosis allow multiple users, including specialists, primary care physi
Significant findings from admission work-up: cians, and the patient, to maintain an accurate, dynamic
History and physical examination health record.
Laboratory studies Clinical decision support refers to the use of these tech
Imaging studies nologies to supplement a practitioner's clinical reasoning. For
Other tests example, a CPOE system can warn the ordering clinician
about drug interactions, need for renal dosing, or potential
Procedures performed contraindications to studies. An EHR interface can prompt
Results of procedures and significant testing physicians to order needed medications, screening tests, or
Condition at discharge chronic disease management.
Health information technology is not a panacea. The
Discharge medications and reasons for any changes from
admission medications potential for new errors is present in all systems; for example,
Follow-up issues CPOE can allow multiple physicians to enter potentially con
Pending studies and laboratory tests flicting orders simultaneously. Poorly designed systems can
introduce even more errors.
Counseling provided to patient and family
Follow-up appointments/plans
National Patient Safety Goals
Each year, the Joint Commission publishes national patient
safety guidelines (www.jointcommission.org/standards
1 medication reconciliation efforts are led by pharmacists, die
Jnformation/npsgs.aspx). These guidelines span the contin
C0NT rate of adverse drug events at 30 days is l% versus 11% in con uum of care, including acute care hospitals, ambulatory prac
trol patients. tices, nursing homes, and free-standing surgical centers,
KEY POINTS although not every standard is applicable at every site. The
Common medication errors include failure to recog goals are focused on those that have the highest impact for
both quality and patient safety. Each goal provides specific
nize drug interactions among prescribed medications
or to adjust dosing to account for impaired kidney or objectives and metrics. For example, under the goal Improve
liver function. accuracy of patient identification are objectives such as "Use
at least two patient identifiers when providing care, treatment,
Medication reconciliation should occur throughout
and services." The Joint Commission also includes measura
the hospital stay; patients should receive a list of med ble elements of performance such as "Label containers used
ications at the time of discharge. for blood and other specimens in the presence of the patient."
Timely follow-up with a primary care physician after These safety goals provide a frame for interdisciplinary col
hospital discharge (<1 month) leads to lower rates of laboration around quality and safety and can provide concrete
rehospitalization. objectives and criteria for quality improvement projects.

Health Information Technology


and Patient Safety Professionalism and
Health information technology comes in three general forms: Ethics
computerized physician order entry (CPOE), electronic
health record (EHR), and clinical decision support. Each of Professionalism
these technologies can be used in an inpatient or outpatient In exchange for the authority to train, license, certify, and cre
environment and can improve patient safety. Health informa dential physicians, the medical profession has an obligation to
tion technology systems should allow information sharing society to ensure that physicians uphold ethical and profes
and integration across the continuum of care. sional behaviors, including maintenance of clinical compe
CPOE refers to the direct entry of orders (for example, tence and the fulfillment of clinical and hospital responsibili
prescriptions, radiology) into a computer interface by a physi ties in a timely manner. This obligation requires that
cian. CPOE eliminates medication error related to physician physicians honestly assess their knowledge and skills and

22
Professionalism and Ethics

pursue learning where gaps exist. It also creates the expecta professionalism with the public's health: the commitment to
tion that physicians will disclose errors to patients and others improving quality of care, the commitment to improving
and perform duties and meet standards established by local access to care, and the commitment to a just distribution of
medical and hospital staffs. finite resources.
In the routine delivery of patient care, physicians must Together, these commitments create an expectation that
model exemplary interactions with trainees and colleagues, every physician advocate for and work to ensure that all per
both physician and nonphysician. Teamwork and collabora sons have access to quality health care, and that medical care-
tion, with mutual respect and recognition of one another's is delivered equitably across racial, ethnic, religious, and
contributions to the healdi care team, are fundamental in socioeconomic groups. A commitment to social justice
today's healdi care environment and fundamental to demon requires physicians to guard the health of the public by elim
strating professionalism with trainees. inating unsafe and low-value services that generate expenses
As part of their contract with society, physicians must for care with potential harm and little to no benefit to the
also advance the public good, which requires they serve as patient. The American College of Physicians' High-Value,
judicious stewards of public resources. The Charter on Cost-Conscious Care initiative strives to promote this com
Medical Professionalism comprises three principles and mitment by helping physicians assess die value and the costs
ten commitments (Table 12). Three commitments link of specific interventions.

table 12. Principles and Commitments of Professionalism

Principle or Commitment Comment


Fundamental Principle

Primacy of patient welfare Altruism is a central trust factor in the physician-patient relationship. Market forces, societal
pressures, and administrative exigencies must not compromise this principle.
Patient autonomy Patients' decisions about their care must be paramount, as long as those decisions are in keeping with
ethical practice and do not lead to demands for inappropriate care.
Social justice Physicians should work actively to eliminate discrimination in health care, whether based on race,
gender, socioeconomic status, ethnicity, religion, or any other social category.
Professional Commitment

Competence Physicians must be committed to lifelong learning and to maintaining the medical knowledge and
clinical and team skills necessary for the provision of quality care.

Honesty with patients Obtain informed consent for treatment or research. Report and analyze medical errors in order to
maintain trust, improve care, and provide appropriate compensation to injured parties.
Patient confidentiality Privacy of information is essential to patient trust and even more pressing with electronic medical
records.

Appropriate patient relations Given the inherent vulnerability and dependency of patients, physicians should never exploit patients
for any sexual advantage, personal financial gain, or other private purpose.

Improve quality of care Work collaboratively with other professionals to reduce medical errors, increase patient safety,
minimize overuse of health care resources, and optimize the outcomes of care.

Improve access to care Work to eliminate barriers to access based on education, laws, finances, geography, and social
discrimination. Equity requires the promotion of public health and preventive medicine, as well as
public advocacy, without concern for the self-interest of the physician or the profession.
Just distribution of resources Work with other physicians, hospitals, and payers to develop guidelines for cost-effective care.
Providing unnecessary services not only exposes one's patients to avoidable harm and expense but
also diminishes the resources available for others.
Scientific knowledge Uphold scientific standards, promote research, create new knowledge, and ensure its appropriate use.
Manage conflicts of interest Medical professionals and their organizations have many opportunities to compromise their
professional responsibilities by pursuing private gain or personal advantage. Such compromises are
especially threatening with for-profit industries, including medical equipment manufacturers,
insurance companies, and pharmaceutical firms. Physicians have an obligation to recognize, disclose
to the general public, and deal with conflicts of interest that arise.
Professional responsibilities Undergo self-assessment and external scrutiny of all aspects of one's performance. Participate in the
processes of self-regulation, including remediation and discipline of members who have failed to
meet professional standards.

Adapted with permission from ABIM Foundation. American Board of Internal Medicine; ACP-ASIM Foundation. American College of Physicians-American Society of Internal Medicine;
European Federation of Internal Medicine. Medical professionalism in the new millennium: a physician charter. Ann Intern Med. 2002;136(3):243-246. [PMID: 11827500] Copyright 2002.
American College of Physicians.

23
Professionalism and Ethics

KEY POINT particularly in patients with underlying mental health disor


Physicians have a responsibility to guard the health of ders or cognitive deficits. A diagnosis of dementia or a men
the public by eliminating unsafe and low-value serv tal illness does not necessarily mean that a patient is incapable
ices that generate expenses for care with potential of making health care decisions. The clinician must assess
harm and little to no benefit to the patient. whether or not the patient's decision appears consistent with
his or her values and goals of" care. If it does, it can probably
be accepted as valid. A decision that seems inconsistent is a
prompt to further explore the patient's beliefs, values, and
Decision-Making and Informed comprehension of die situation and the decision. A given
Consent decision does not have to be rational, however, nor need it
Decision-making is at die core of medicine. A common reflect what most people would do in that situation. Patients
source of ethical challenges in practice is that while the who are able to understand the consequences may refuse life-
authority' for decision-making rests with the patient, the saving dierapy, even for reasons that may be difficult to under
knowledge needed for informed decision-making is often in stand, such as religious preferences, community beliefs, or
the hands of the clinician. other values. The decisional capacity of these patients should
only be questioned if*they are acting in a way that is inconsis
Informed Consent tent with dieir personal beliefs or those of" their community.
Informed consent requires that the patient understand the
nature of the decision being made or intervention being pro Advance Directives and Surrogate Decision-
posed, alternative options to the proposed intervention or Making
decision, and the risks and benefits of each of the various alter When patients lack the capacity to make a decision on their
natives. To obtain informed consent, the clinician explains the
own, diey may have provided guidance in the form of" an
options and their risks and benefits to the patient and makes advance directive. Advance directives fall into two broad cat
a recommendation if there is a preferred course of action. In
egories: instructive directives and proxy directives. Advance
addition, the clinician must assess the patient's understanding directive documents may include both components, for
of the options and verify the patient's final decision. Although
example, a living will (instructive) and durable power of attor
clinicians should make a recommendation if they feel that a
ney for health care (proxy).
particular choice is the most medically appropriate, they must Instructive directives provide guidance about what the
not coerce or entice the patient into making that choice.
patient would want to have done in certain situations.
The greater the complexity of the decision, the greater These may be straightforward, such as a request not to
the importance of detailed informed consent. Complexity' can receive cardiopulmonary resuscitation. They may also be
be characterized by three domains: the potential effect on the
complicated, such as instructions for what specific life-sus
patient; the consensus within the medical community as to taining treatment the patient may or may not want if they
the appropriate action; and the number and uncertainty of are unable to communicate their wishes (for example, in a
possible outcomes. By convention, most invasive procedures persistent vegetative state). Instructive directives are subject
such as surgery require written informed consent, which to interpretation, as they cannot capture all the nuances of a
should reflect die outcome of die conversation described
given clinical situation. Many U.S. states have legal guide
above. However, other complex decisions may also benefit lines and forms that address the scope of advance directives,
from the principles of informed decision-making, even if an but unfortunately the legal forms may be difficult for
explicit signed consent document is not required. For exam patients to read and understand.
ple, deciding whether to obtain a screening mammography in Proxy directives designate a surrogate decision maker
a woman younger than 50 years is a complex decision by die (also known as a durable power of attorney for health care).
criteria above, and the principles of informed consent apply
Ideally, before incapacity occurs, the patient will have desig
even if one does not obtain a specific signed consent for the nated a surrogate to make health care decisions. If not, the cli
test in question. In a study of outpatient practices, it was nician must try to identify the legal next of kin who is empow
shown that fewer than 10% of all clinical decisions made met ered to act as a surrogate decision maker. This is typically
standards for informed decision-making, and fewer dian 1% determined by state law in the United States. Most jurisdic
of complex decisions were considered fully informed. tions also have a mechanism to appoint a legal guardian in sit
uations in which the patient is incapable of making health care
Assessing Decision-Making Capacity decisions and there is no designated surrogate or next of kin.
The core components of decisional capacity are understand The role of the surrogate is to strive to make decisions
ing the situation at hand, understanding the risks and bene based on what the patient would have wanted in that situa
fits of the decision being made, and the ability to communi tion (the ethics concept of substituted judgment). A living
cate a decision. Assessing understanding can be challenging. will or instructive directive can help inform the surrogate's

24
Professionalism and Ethics

decisions. If the surrogate has no knowledge of what the patient wants them, they may need to request ethical and legal
consultations in such cases.
patient would have wanted, either because this was never dis
cussed or because the surrogate did not know the patient Populations that have historically faced discrimination by
well, a secondary standard would be to make decisions the the health care system and may have greater mistrust of health
surrogate considers to be in die patient's best interest. If the care providers' intentions, such as blacks in the United States,
surrogate's decisions seem inconsistent with the patient's val are more likely to request aggressive end-of-life treatment.
ues or previous directives, die clinician should proceed with Physicians need to be sensitive to such cultural and historical
extreme caution. Ethics consultations are helpful in reconcil factors that may shape patients' and their families' attitudes
ing conflicts. about end-of-life care.

KEY POINT KEY POINT


The core components of" decisional capacity are Competent patients may refuse care even if the clini

understanding the situation at hand, understanding cian believes that the patient's death is not imminent
the risks and benefits of the decision being made, and or inevitable; unless the patient is a minor or lacks
the ability to communicate a decision. decisional capacity, the clinician should abide by the
patient's wishes.

Withholding or Withdrawing
Treatment Physician-Assisted Suicide and
Euthanasia
Although a primary goal of care is usually to sustain life, there
are circumstances under which life-sustaining treatment The issue of when it is appropriate to provide a treatment that
should be withheld or withdrawn. The emotional implica may hasten the death of" a dying patient is a heavily debated
tions may differ whether one withholds care (never starts a area of medical ethics. Most agree by virtue of the principle
treatment) or withdraws care (stops a treatment previously of double effect that it is ethically permissible to give a termi
started), but the two are equivalent from the standpoint of" nally ill patient a treatment that may hasten death when the
medical ethics. primary intent is therapeutic. A common example is high
The two most common reasons to withhold or withdraw doses of"opiate analgesics used to relieve pain or dyspnea in a
treatment are that the patient or the surrogate has decided patient who is dying.
that the patient no longer wishes to receive such treatment, The practice of prescribing medications or interventions
or that life-sustaining treatment no longer offers benefit to with the primary intent of"hastening a patient's death (physi
die patient. If the patient has decided that he or she no longer cian aid in dying, physician-assisted suicide) remains intensely
wishes treatment, the decision to withhold or withdraw is controversial. Although the American Medical Association
and the American College of"Physicians have both taken posi
usually straightforward. In most cases, it is reasonable to con
tinue efforts at relief of symptoms and discomfort, while stop tions against the practice, it is legal in some states under spe
ping invasive or uncomfortable treatments that serve only to cific circumstances.
sustain life. In some circumstances, clinicians may disagree Active administration of" a drug to cause death (active
with the patient's decision to withhold or withdraw treat euthanasia) is illegal in all states, regardless of consent. All
ment, particularly if they feel diat the patient's death is not requests for a hastened death should be carefully evaluated
imminent or inevitable. However, there is ample legal and and responded to with empathy and compassion, regardless
ethical precedent for competent patients to refuse care. of" die physician's ethical position on the specifics of" die
Unless the patient is a minor or lacks decisional capacity, the request.
clinician should abide by the patient's wishes. KEY POINTS
A scenario in which life-sustaining treatment no longer
A terminally ill patient may be given a treatment that
offers benefit to the patient who wants it can be more chal
may hasten death when the primary intent is to pro
lenging. Although one is on solid ethical ground denying vide therapy for another condition.
futile treatment, the precise definition of" futility can be quite
difficult. A treatment is futile either when it has a very low Active administration of a drug with die intent to
cause death is illegal in all states, regardless of"consent.
probability of producing any benefit whatsoever (quantitative
futility), or when the amount or quality of benefit produced
is so small as to be trivial (qualitative futility). If"the physician
feels that a specific intervention is unlikely to produce an out C o n fi d e n t i a l i t y
come that the patient would find to be meaningfully benefi Implicit in the physician-patient relationship is a commitment
cial, the intervention is likely futile. Although physicians have to confidentiality. Information disclosed by a patient to a
no ethical obligation to provide futile treatments, even if the physician should not be disclosed to anyone not directly

25
Professionalism and Ethics

involved in the patient's care (and therefore bound by a sim can generate conflicts of" interest. In f'ee-for-service systems
ilar requirement for confidentiality). Patients may specify such as those prevalent in the United States, physician com
other persons, such as friends or family members, with whom
pensation often increases with the number and complexity of
they wish the physician to share information, but in die services provided. Moreover, if"the physician has a financial
absence of" specific permission, the physician should not dis
interest in facilities used for diagnosis or treatment, he or she
close anything.
has an incentive to provide additional diagnostic or thera
Confidentiality is not absolute, however. Situations in
which there is an established risk of patients harming them peutic services, or to provide those services in a manner that
selves or others may require the clinician to disclose confi increases compensation (for example, intravenous instead of"
dential information. If the clinician believes that the patient subcutaneous or oral medication administration). Aldiough
these relationships are regulated by federal law in the United
poses a serious risk of"harm to a specific person, he or she has
a duty to warn that person. This is usually best handled with States, regulation has not eliminated them or the associated
die assistance of law enforcement authorities, and it is also financial incentives.
often helpful to get legal advice, if available. Another situation The relationships between physicians and companies in
in which confidentiality may be sacrificed for the public wel die health care industry that profit from physician decisions
fare is for reportable communicable diseases, which are usu have also come under increasing scrutiny. The manufacturers
ally determined by local jurisdictions and public health of prescription drugs and medical devices devote considerable
departments. funds to marketing their products, using both direct and indi
Constraints on decisional capacity also have implications rect approaches to influence physician behavior. A consider
for confidentiality. If" the patient is unable to make an impor able amount of"medical research is funded by companies that
tant medical decision, then the person who is making that
stand to gain from the results, which has led to concerns
decision for him or her is entitled to be fully informed as to
about bias in the content or publication of research papers. In
die aspects of"the patient's condition that pertain to the deci
sion. In the case of"minor children, parents usually have a legal addition, concern has been raised regarding whether experts
whose research is heavily funded by industry can make impar
right to bc informed about the child's condition, and die
tial decisions when serving on panels to draft clinical practice
physician has an obligation to disclose any suspicion of abuse.
Both ethics and law regarding confidentiality can be compli guidelines. Many physicians also benefit indirecdy from
cated for adolescents. Some U.S. states have laws specifically industry funding for continuing medical education programs.
protecting adolescent confidentiality for certain medical issues Both industry and accrediting organizations have developed
such as reproductive healdi. When interviewing adolescents, standards to help manage these relationships. The Institute of
the physician should perform at least some of the interview Medicine has published recommendations for controlling
without die parents present and explicitly discuss with the conflicts ofinterest (Table 13).
patient what will and what will not be confidential, informed
by an awareness of applicable local law.
Medical Error Disclosure
KEY POINTS
Research shows that patients expect disclosure of" harmful
A physician may be required to disclose confidential
medical errors. The National Quality Forum endorsed guide
information if the patient poses a serious risk of harm
to self or others or has a reportable communicable lines in 2006 that included three key components oferror dis
disease. closure:

If a patient is unable to make an important medical Provide facts about the event (including an analysis of

decision, the person who is making that decision for system failure, if"available)
him or her is entided to be fully informed as to the Express regret for the unanticipated outcome
aspects of" the patient's condition that pertain to the Give a formal apology if the unanticipated outcome was
decision.
caused by error or system failure

Apologizing for errors can be particularly difficult owing


Conflicts of Interest to concern that an apology or admission of responsibility may
The fiduciary nature of the physician-patient relationship cause problems in subsequent malpractice litigation. A major
dictates that the physician place die interests of the patient ity of U.S. states now have laws diat offer at least limited pro
above his or her own. There are, however, numerous oppor tection for apologies or expressions of regret. Caution is indi
tunities for conflicts of interest. A common area of conflict cated, however; whereas apologizing for errors is both
with the physician's own interests is that of financial rela ethically and interpersonal!}' desirable, it is reasonable to
tionships. At the most basic level, physician payment structures obtain additional counsel on the legal implications.

26
Palliative Care

TABLE 13 . A Selection of Institute of Medicine Recommendations for Individual Physicians to Control Conflicts of Interest

Forgo all gifts or items of material value from pharmaceutical, medical device, and biotechnology companies, accepting only payment
at fair market value for a legitimate service in specified situations.
Do not make educational presentations or publish scientific articles that are controlled by industry or contain substantial portions
written by someone who is not identified as an author or who is not properly acknowledged.
Do not meet with pharmaceutical and medical device sales representatives except by documented appointment and at the physician's
express invitation.
Do not accept drug samples except in certain situations for patients who lack financial access to medications.
Until institutions change their policies, physicians and trainees should voluntarily adopt these recommendations as standards for their
own conduct.

Adapted with permission from Steinbrook R. Controlling conflict of interestproposals from the Institute of Medicine. New Engl J Med. 2009;360(21):2160-2163. (PMID: 194038981
Copyright 2009, Massachusetts Medical Society.

Sexual Contact between TABLE 14. Signs of Physician Impairment in the Work
Setting
Physician and Patient
Late to appointments; increased absences; unknown
The inherent asymmetry of power and trust in the physician- whereabouts
patient relationship makes it ethically unacceptable to main Unusual rounding times, either very early or very late
tain a sexual relationship with a patient, even if the patient ini
Increase in patient complaints
tiates sexual contact.
Sexual relationships with former patients are also con Increased secrecy
Decrease in quality of care; careless medical decisions
cerning. The greater the depth and duration of the previous
professional physician-patient relationship, the more caution Incorrect charting or writing of prescriptions
is needed. Relationships with patients whom the physician Decrease in productivity or efficiency
was treating for a mental health disorder are particularly prob Increased conflicts with colleagues
lematic and may result in civil liability or professional discipli
Increased irritability and aggression
nary action in some jurisdictions, regardless of patient consent
Smell of alcohol; overt intoxication; needle marks
or the time elapsed since terminating the physician-patient
Erratic job history
relationship. In general, a physician considering a sexual rela
tionship with a former patient is advised to solicit the opin Adapted with permission from Ross S. Identifying an Impaired Physician. Virtual Mentor.
December 2003. Volume 5. Number 12. Accessed at http://virtualmentor.ama-assn.org/
ions oflegal counsel or an ediicist before proceeding with the 2003/12/cprll-0312.html. 22 December 2011. The viewpoints expressed in Virtual
Mentor are those of the authors and do not necessarily reflect the views and policies of
relationship. the AMA. Copyright 2003, American Medical Association.

The Impaired Physician and


Colleague Responsibility physicians, and many states stipulate those licensed to practice
by the state must report impaired colleagues. Once the PHP
Members of the medical profession have an obligation to pro
is notified of a possibly impaired physician, assessment, treat
tect the welfare of"patients, which includes taking action when
ment, and monitoring can be arranged. If a physician volun
a colleague puts patients at risk. Many states have mandatory
tarily participates in the PHP's treatment and monitoring
reporting statutes diat require a physician to report to appro program, the PHP will often advocate for him or her with the
priate authorities when a colleague continues to practice state medical board, resulting in mitigation or even avoidance
despite his or her inability to do so safely because of" impair of formal disciplinary action as long as die physician remains
ment from substance use or illness. in compliance.
In a 2009 survey, the majority of respondents endorsed
a duty to report, but of the 17% who had direct knowledge
of" an impaired colleague, only two-thirds actually reported
him or her to the relevant authority. Beliefs that someone else Palliative Care
was taking care of"the problem or that nothing would happen
were the most common reasons for failure to report. Introduction
Common signs ofphysician impairment at work are shown in The primary focus of palliative care is to relieve patient suffer
Table 14. ing and to improve the quality' of" patients' lives and those of*
All U.S. states now have physician health programs their caregivers. Palliative care involves a multifaccted
(PHPs) that allow for anonymous reporting of impaired approach that includes clarifying goals of treatment, managing

27
Palliative Care

symptoms, mobilizing resources to optimize care and social Prognostication can be even more difficult when a patient
support, and integrating care across settingswhether home, does not have cancer (Figure 3). In patients with cancer, the
nursing home, or hospital. Despite its widespread availability, clinical course frequently ends with an obvious decline in the
palliative care continues to be underutilized. Much of" die final weeks or months of"disease. Patients with organ failure
resistance to initiating palliative care stems from the traditional often have gradual decline with serious exacerbations fol
care dichotomy between "doing everything" versus providing lowed by improvement; it can be very challenging to predict
comfort carea dichotomy that often fails to focus on reliev timing of'death in these patients. A third group of patients are
ing suffering and improving quality of"care. diose with frailty, sometimes with concomitant dementia.
Palliative care is often thought of as end-of-life care only, These patients have long, slow declines but vvidiout discrete
but palliative care addresses pain, suffering, and quality' of life exacerbations of disease.
across all stages of" treatment and does not exclude life-pro
longing treatment and rehabilitation. Nonhospice palliative High
care may be offered along with curative or life-prolonging Mostly cancer

therapies for patients with complex, life-threatening disorders.


Hospice palliative care is offered when patients reach their
final weeks or months of life, when the likely harm of life-pro
longing or curative therapies exceeds benefit, and these ther
apies are discontinued. Hospice services can be provided in a
patient's home, in specialized hospice units in hospitals, or in
community-based facilities, depending on patients' and their
Death
family's individual needs and preferences.
Time
Emerging data for concurrent palliative care intervention Low
in die treatment ofcancer strongly suggest that patients using Short Period of Evident Decline

palliative care services have higher scores for quality of life and
mood than diose undergoing cancer treatment alone. In a
study of" 151 patients with newly diagnosed metastatic High
non-small cell lung cancer randomly assigned to standard
oncologic therapy alone versus early palliative care with stan
dard oncologic therapy, patients in the palliative care group
had a mean survival of 11.6 months as compared with 8.9
Mostly heart and lung failure
months in the group not provided early palliative care. The
intervention group also experienced less depression and bet
ter quality of"life.
Palliative care consultation programs are also associated
Death
with significant hospital cost savings, with an adjusted net sav
ings of"S1696 in direct costs for patients discharged alive from Low

the hospital and S4908 net savings for patients dying in the Long-Term Limitations with Intermittent Serious Episodes

hospital as compared with patients who receive usual care.


KEY POINT
High
Palliative care addresses pain, suffering, and quality of
life across all stages of treatment and does not
exclude life-prolonging treatment and rehabilitation.

Mostly frailty and dementia

Deciding When Hospice


Palliative Care Is Indicated
Approximately 30% of Medicare's expenditures occur in
patients' last year of life. Evidence suggests diat more aggres
Low
sive care at the end of lifewhether prolonged hospitaliza
Prolonged Dwindling
tion, intensive care unit admission, or performance of proce
duresdoes not improve either quality or duration of" life. FIGURE 3. Trajectories of late-life illness.
The problem with estimating when the last year of" life has Reprinted with permission from Lorenz KA, Lynn J, Dy SM. et al. Evidence for improving pal
liative care at the end of life: a systematic review. Ann Intern Med. 2008c 148(2): 147-159.
arrived is that patients do not progress in identical patterns. [PMID: 18195339] Copyright 2008. American College of Physicians.

28
Palliative Care

Recent evidence suggests that minority patients receive add one additional symptom and rate it. These symptoms
more intensive care at the end of life than they would if they may occur because of the underlying disease or secondary to
were more adequately communicated with regarding advance treatments for symptoms of the disease.
care planning, prognosis, and treatment alternatives, such as
nonhospice or hospice palliative care. The National Pain
Comprehensive Cancer Network has recommended that Undertreated patient pain is a major problem in both the
patients be screened for palliative care needs at initial presen inpatient and outpatient settings. Assessment of pain can
tation with cancer as well as at subsequent visits, and that pal be difficult, particularly in patients with dementia, delir
liative care services be available to all patients with cancer. ium, or somnolence caused by medications being used to
treat pain.
KEY POINT
Strong evidence from randomized trials supports
More aggressive care at the end of life does not
treating cancer-related pain with NSAIDs, opioids, and
improve either quality or duration of life. radiation therapy; evidence regarding the use of" bisphos
phonates for cancer pain is less convincing. There is a lack
of studies providing strong evidence on effective ways to
Assessment and Communication treat pain in advanced heart failure or dementia. Opioid
The cornerstone of both hospice and nonhospice palliative use in the treatment of noncancer pain continues to be
care is communication with patients, families, and caregivers. controversial.
The first step in this communication is to establish goals of The World Health Organization analgesic ladder pro
care in a patient-centered, open-ended format. Adequate time vides a stepwise approach to the management of pain
needs to be allotted to ensure that these discussions provide (Figure 4). Pain management in palliative care, as with
die opportunity' for actively listening to the patient and key other conditions, should begin with nonopioid analgesia,
care providers (rather than being physician-centeredori including acetaminophen and NSAIDs (see Common
ented toward efficiency and completing the task). The initial Symptoms, Chronic Noncancer Pain). Adjuvant pain med
ications are used to treat pain symptoms that respond
meeting or meetings should emphasize discovery of what the
patient knows and understands regarding the diagnosis and poorly to analgesic agents, such as neuropathic or bony
prognosis. This discussion should allow opportunity to clar
ify how much the patient actually wants to know and should
respect the diverse ways in which patients and their families
process information about life-altering or life-threatening
medical conditions. Medical jargon should be avoided and
medical information conveyed succinctly and empathetically.
Several meetings may be necessary to allow further process
ing of information and preparation of more questions by the
patient or family members. As issues are addressed and
worked through, a plan should be presented in a clear and
succinct manner. Next steps should be outlined and, when
necessary, carefully reiterated.
KEY POINT
The first step in communication with patients, fami
lies, and caregivers regarding palliative care is to
establish goals of care in a patient-centered, open-
ended format.

Symptom Management
Symptoms common in patients with cancer and other life-
threatening illnesses are myriad. Numerous symptom assess
ment instruments have been validated in the medical litera
ture. The Edmonton Symptom Assessment Scale is a brief
survey that asks patients to rate several symptoms on a scale
(pain, dyspnea, fatigue, nausea, depression, anxiety-, drowsi
ness, appetite, sense of well being) and also allows them to FIGURE 4. World Health Organization analgesic ladder.

29
Palliative Care

D metastatic pain. Examples of these medications include


E anticonvulsants, corticosteroids, and antidepressants.
CONT.
relieve it. Other important side effects of morphine include
xerostomia, constipation, sedation, myoclonus, urinary hesi-
Corticosteroids are useful for treating spinal cord com tanq', nausea, and itching.
pression or brain metastases, but data are lacking in sup
port of their use in other cancer-related pain. Benzo Constipation
diazepines should generally be avoided as they may actually Constipation is common in cancer patients, particularly in the
worsen pain and also cause confusion and agitation in elderly. It is defined as fewer than three bowel movements per
patients who are elderly or have dementia. week, die subjective sensation of incomplete bowel move
Various opioid analgesics arc listed in Table 15. A
ments, or difficult passage of stool. It is important to obtain
weaker opioid analgesic (such as oxycodone) can be added at
a history about a patient's premorbid bowel habits in order to
any point along the World Health Organization ladder. If establish a baseline. Forty percent to 95% of" cancer patients
pain persists or worsens, stronger opioid analgesia should be suffer from constipation, and 95% of" patients on opioids
started. The usual starting dose of"short-acting morphine is 5
to 15 mg orally in opiate-naive patients and the elderly or 1 report constipation. Opioids cause constipation by binding to
the p receptor in the bowel, diminishing intestinal activity and
to 5 mg intravenously. Shorter-acting morphine can be used
for breakthrough pain but should bc changed to long-acting reducing intestinal secretions. Constipation is also exacer
bated by decreased activity and, toward the end of life, by
morphine if a patient develops persistent pain throughout the
reduced mobility and access to die toilet. Other contributing
day or beyond 24 hours of"treatment with shorter-acting opi
oids. A starting dosage of long-acting morphine is usually factors in cancer patients include hypercalcemia and bowel
30% to 50% of a patient's total average 24-hour usage. This obstruction (from primary or metastatic disease or treatment-
related adhesions).
dosage is increased every 3 to 4 days. Analgesia dosage for
breakthrough pain should be 10% to 20% of" the total daily A bowel regimen should bc prescribed for any patient
opioid dose or 25% to 30% of the single standing dose. starting opioid analgesic agents and should include both a
Analgesia should be gradually increased, with the firm goal of stool softener and a laxative. "Rescue laxatives," such as lac
controlling pain. Beliefs about "maximum" dosages do not tulose, magnesium citrate, or enemas, may be needed if die
usually apply. patient fails to have at least three bowel movements weekly
Although many health care providers worry about res or reports symptoms of constipation on a standard bowel
piratory depression as a side effect of morphine, respiratory regimen.
depression occurs rarely in patients who are actively in pain. Some data indicate that opioid rotation can improve
In the elderly, however, it is generally best to start at the constipation. Three studies have showed reduced reliance on
lower range. Hydromorphone is a semisynthetic opioid laxatives when oral morphine was switched to a fentanyl trans
agonist that is similar to morphine but has a more favorable dermal patch.
pharmacokinetic profile. Fcntanyl is more expensive but can
be administered as a lollipop, transdermally, or intra
Fatigue
venously. Methadone, a long-acting opioid, has an unpre
Fatigue, the subjective feeling of physical, emotional, or
dictable half-life and has been reported to cause QT inter
mental exhaustion, is one of the most common and dis
val prolongation and arrhythmias. It should be used with
tressing symptoms related to cancer and cancer treatment.
caution, with monitoring of the QT interval. Meperidine
In contrast to fatigue in a healthy person, fatigue in a cancer
should be avoided because it can cause seizures, especially
in the setting of kidney failure, as well as confusion and patient is not relieved by rest. It affects 60% to 90% of
mood alterations. A fentanyl transdermal patch is an excel patients with cancer and up to 96% of patients undergoing
radiation or chemotherapy. Notably, as cancer progresses,
lent alternative to decrease frequency of dosing but is sub
stantially more expensive than long-acting morphine and fatigue worsens. Fatigue can also be an early harbinger of
takes up to 24 hours to take effect, making dose adjustment recurrent cancer.
Treatment of fatigue in cancer and other end-stage ill
slightly more complicated.
When using opioids, it is important to be sensitive to nesses, such as heart failure and COPD, begins with educa
patient concerns about addiction, as diese concerns may pre tion of die patient and family members about modes of
sent an obstacle to attaining adequate pain control. It is energy conservation and distraction. Various modalities have
extremely rare for patients to become addicted to opioids in been evaluated in reducing cancer-related fatigue, including
die acute hospital setting and rare to become addicted when biofeedback and exercise programs, but the small patient pop
being treated in the setting of cancer-related pain. ulations in these studies make it difficult to establish firm
Occasionally, opioid use may result in opioid-induced hyper evidence-based recommendations about interventions.
algesia. This is a poorly understood phenomenon whereby However, given die low morbidity of such treatments, it is
increased dosages of opioids exacerbate pain rather than reasonable to try an exercise or rehabilitation program.

30
Palliative Care

table 15. Common Noninjected Narcotics Used in Palliative Care for Chronic Pain [

Agent Drug:Morphine Form Starting Dose Onset Duration Comments


Potency Ratio3

Morphine Immediate 10mgq3-4h 30 min 4h Tablet, solution, and


release rectal suppository
Controlled/ 15-30 mgq 12 h 2-4 h 12h Tablets, ranging from
sustained 15-200 mg
release6

Oxycodone 2:1 Immediate 2.5-5 mg q 6 h 10-15 min 3-6 h Tablet and solution form
release
Extended 10 mgq 12 h 1h 12 h Tablets 60 mg or higher for use
releaseb only in opioid-tolerant patients.
Fentanyl 4:1c Immediate 200-pg lozenge:
release may repeat once 5-15 min 4-8 h Not recommended for
in 15 min, then opioid-naive patients.
q6h Transmucosal lozenge or buccal
100-pg buccal tablet should be used only in
tablet: may patients who are already
repeat once in 30 receiving narcotics and are
min, then q 4 h opioid tolerant.
Limit to 4 or fewer daily
additional doses mark need for
adjustment of basal pain
medication.
Extended 25-pg patch q 72 h 12-24 h 72 h Not recommended for
release opioid-naive patients.
Patients should be on at least
60 mg oral morphine
equivalents/d before starting.
Dose should not be adjusted
upward based on supplemental
opiate need for 3 days after
initial placement or 6 days after
subsequent dose changes.
17 hours are required for 50%
decrease in fentanyl levels
after removal.
Codeine 1:3-8 (variable) Immediate 30-60 mg q 4-6 h 30-60 min 4-6 h Tablet and liquid, usually taken
release with adjunct analgesics due to
weak strength.
Variable efficacy due to
differences in metabolism to
morphine with CYP2D6 enzyme.
Hydromorphone 4:1 Immediate 2-4 mg q 3-6 h 15-30 min 4-5 h Also available as liquid, rectal
release suppository.
Dose adjustment required with
kidney failure.
Extended 8-64 mg q 24 h 1-2 h 24 h For use in opioid-tolerant
release patients only.
Dose adjustment required
with kidney failure.

Hydrocodone 1:1 Immediate 5-15 mg 30-60 min 4-8 h Available as combination


release q3-8h product with adjunct analgesics.

q = every; pg = microgram.
aA ratio of 2:1 indicates that the medication is twice as powerful as an equivalent mg strength of morphine. No fixed conversion ratio is likely to be satisfactory in all patients, especially
patients receiving large opioid doses.

"Divide cumulative daily dose of short-acting narcotic into two divided doses of the longer-acting narcotic.

The fentanyl comparison is a pg-to-mg conversion.

31
Common Symptoms

Dyspnea Depression

Dyspnea is a common and troubling symptom for patients Depression is a common symptom in patients approaching
with cancer as well as advanced COPD and heart failure. Up the end of" life. Tricyclic antidepressants and selective sero
to 70% of" cancer patients suffer from dyspnea in the last 6 tonin reuptake inhibitors have been found to be equally effec
weeks of" life, and for 30%, it is rated as moderate to severe tive, provided treatment duration is 6 weeks or longer.
in intensity. Strong evidence supports the use of (3-agonists, Randomized controlled trials have also found consistent
morphine, pulmonary rehabilitation, and oxygen for symp efficacy in nonpharmacologic interventions, such as psycho
tom relief in COPD. Although opioids are fiequendy used therapv', education, and individual and group support.
in cancer-related dyspnea, evidence is weak for their use in
this situation. However, options are limited in treating can Delirium
cer-related dyspnea, and opioid use is standard of" care. Delirium is common at the end of" life and can be caused
Morphine use in cancer-related dyspnea is prescribed die by advanced medical conditions, the medications being
same way it is used in cancer pain. Benzodiazepines are also used to palliate symptoms, or a combination of both.
sometimes used to treat cancer-related dyspnea but have not Family members frequently need to be reassured that
been well studied. Oxygen may also be used in treating can delirium is common. Opioid analgesia can be reduced pro
cer-related dyspnea, but evidence for functional improve vided pain remains controlled. If opioid dosage cannot be
ment is lacking. reduced owing to pain or dyspnea, haloperidol can be
given at a dose of"0.5 to 1 mg intramuscularly. Additional
Nausea sedative agents (such as benzodiazepines) may bc required
Chronic nausea, defined as nausea lasting longer than 1 week, but can further exacerbate delirium in patients who are eld
affects up to 60% of cancer patients in the last 6 months of erly or have dementia. Q
life regardless of whether they receive chemotherapy.
KEY POINTS
Metoclopramide is a dopamine agonist with prokinetic activ
Respiratory depression as a side effect of morphine
ity through die cholinergic system. It has a short half-life but
can be given intravenously, subcutaneously, or orally and occurs rarely in patients who are actively in pain.
results in symptomatic improvement in 50% of" patients. A bowel regimen should be prescribed for any patient
Serotonin antagonists (ondansetron) are effective for starting opioid analgesic agents.
chemotherapy-related nausea but are expensive and unproven
in die chronic palliative care setting. Dronabinol is a cannabi-
noid that is effective in chemotherapy-related nausea and Bereavement and Grief
AIDS-related wasting, but it has not been well studied for
Although providing psychosocial, spiritual, and bereave
chronic palliative care patients. It may cause somnolence, con ment support for patients and their families is considered
fusion or euphoria. Dexamethasone has been increasingly an essential part of" hospice palliative care, effective ways
used for palliative care as an antiemetic. A trial of 2 to 4 mg to do this are not well demonstrated in the medical liter
intravenously can be tried for patients not responding to ature. Approximately 15% of" bereaved survivors in the
other antiemetic agents. U.S. experience complicated grief", defined as grief per
sisting more than 6 months after a death. Its chief symp
Anorexia and Nutrition tom is a yearning for a loved one so intense that it inca
Loss of" appetite can bc one of the most disturbing symptoms
pacitates all other desires. Many interventions have been
for patients and their providers. Reassurance often needs to be studied, but thus far, none have shown results adequate
provided, and advance directives regarding feeding tubes to provide evidence-based guidance in this area. It is
should ideally be clarified in advance of the anorexia diat
important, however, for physicians to be sensitive to the
occurs at die end of life. Feeding tubes are controversial and needs of bereaved survivors given the depth of" despair for
probably do not confer a survival advantage to cancer patients which they are at risk.
unless used as a temporizing measure in the treatment of head
and neck or gastrointestinal cancers. Feeding tubes are not
recommended for terminal cancer. Two classes of drugs have
been shown to improve appetite in patients with cancer but Common Symptoms
do not provide a survival advantage. Progestational agents,
such as megestrol, improve appetite and weight gain. Side Overview
effects include thromboembolism, impotence, edema, and Common symptoms generally fall into a few major cate
vaginal bleeding. Corticosteroids have also been found to gories, including pain, upper respiratory, neurologic, der
improve appetite in patients with cancer, but the proper dos matologic, musculoskeletal, and psychological. Common
ing regimen has not been established. symptoms account for more than half of visits to the general

32
Common Symptoms

internist. Most symptoms improve within 2 weeks, but up to exacerbating factors, and previous treatments. Patients should
25% of these common symptoms become recurrent or be questioned regarding the impact of" pain on their activity
chronic. Patients may present with symptoms as the primary level, ability to work, mood, sleep, and relationships with oth
reason for a visit or as a secondary concern during an ers. The patient's symptoms and goals for treatment need to
appointment for follow-up of chronic medical conditions. be explored in a thorough, patient-centered manner. The
Symptoms for which no demonstrable pathology is found Brief Pain Inventory (BPI), a patient-completed question
are designated as somatic. Up to one third of symptoms pre
naire, is a standardized tool for obtaining information about
sented in primary care visits have no demonstrable cause. a patient's pain symptoms efficiendy. Screening for depression
Some patients may feel uncomfortable discussing sensitive or
and anxiety, as well as substance abuse, is essential, since these
difficult issues, and talking about a common symptom may
feel more acceptable to them. Examples of these hidden diagnoses may significantly affect treatment options. A history
of" physical, verbal, or sexual abuse has been associated with
agendas include concerns about sexual dysfunction, sexually
chronic pain syndromes, and providers should carefully
transmitted infection, incontinence, and depression or other
assess for current or previous threats to a patient's well-being
psychological issues. Asking patients if they have other con
cerns during the visit can often uncover these issues. and safety. Table 16 lists the key elements of the CNCP

Depression, anxiety, or somatization should be considered in assessment, as well as suggested strategies for obtaining this
patients presenting with multiple symptoms in different information.
parts of the body. All patients with CNCP should receive a complete phys
Clinical decision rules (CDRs) aid in the diagnosis and ical examination, with special attention to the musculoskele
management of many common symptoms. Examples of" tal and nervous systems. Muscle strength and tone should be
CDRs include the Centor score to diagnose group A strep assessed, joints inspected for signs of" effusion or erythema,
tococcal pharyngitis and the Ottawa ankle rule to determine and trigger points identified. Testing of sensation is essential
which patients with ankle injuries require imaging. There is a for diagnosing absent or abnormal sensory states, such as
growing literature regarding the evidence-based approach to allodynia (pain with stimuli that are usually not painful) or
evaluating and managing many common symptoms; for
hyperalgesia (increased sensitivity to pain).
example, the Rational Clinical Examination series published There is no specific role for diagnostic testing in the
by the Journal of the American Medical Association. assessment and management of" CNCP, as abnormalities that
are identified may not be the source of" the patient's pain.
Chronic Noncancer Pain Tests for certain conditions may be considered, such as an
Chronic pain is defined as persistent pain of sufficient dura MRI for evaluating back pain if conditions such as spinal
tion and intensity to have a significant impact on a patient's stenosis or herniated disk disease are suspected. Similarly,
quality of life, functional status, and well-being. Pain may nerve conduction studies may be helpful in select patients if"
be considered chronic if it persists for longer than the the diagnosis or etiology of neuropathy is uncertain.
anticipated healing time, although various time frames Based on findings from the history and physical exami
have been proposed (persistence of pain for more than 6 nation, a patient's pain should be classified into one of" five
weeks, 3 months, or 6 months). Chronic noncancer pain types: neuropathic pain, muscle pain, inflammatory pain,
(CNCP) affects as many as 50 million Americans and is mechanical/compressive pain, and mixed (Table 17). This
associated with many conditions encountered in primary classification is helpful for choosing mechanism-specific treat
care, including fibromyalgia, postherpetic neuralgia, dia ment options that are most likely to alleviate the pain. Before
betic neuropathy, and osteoarthritis. CNCP exacts a sig
initiating treatment, providers should assess for common
nificant personal and economic burden, with costs esti
behavioral, social, or systems barriers diat may prevent a patient
mated at $85 billion annually as a result of job loss, missed
from experiencing functional improvement, irrespective of the
work days, and medical visits. (Cancer pain is discussed in
treatment plan. Behavioral barriers include low motivation,
Palliative Care).
unrealistic expectations, poor adherence, chemical dependency,
Assessment and passivity. Social barriers include time constraints, lack of
CNCP may affect many aspects of a patient's life, and guide social support, cultural and language barriers, and financial
lines from the Institute for Clinical Systems Improvement issues. Systems barriers include formulary and coverage
restrictions and difficulty accessing behavioral health care.
emphasize that an initial assessment should evaluate each
patient's pain within the context of his or her psychological Identifying these barriers can help providers develop a realis
state and beliefs, family supports, and social and work envi tic expectation of what can be accomplished in management
ronments. Physicians should first obtain information about of a patient's pain. Patients involved in litigation may display
pain location, character, intensity, duration, relieving and limited response to treatment.

33
Common Symptoms

ta b l e 16. Key Elements in the Assessment of the Patient with Chronic Noncancer Pain

Key Element Notes3


General Assessment

Pain location, intensity, quality, onset, relieving and The Brief Pain Inventory, Chronic Pain Grade, and Neuropathic Pain
exacerbating factors Scale are all helpful for assessing multiple aspects of a patient's pain.
Functional status The Physical Functional Ability Questionnaire (FAQ 5) is brief and easy
to complete.
Mental health disorders (depression, anxiety, The Patient Health Questionnaire (PHQ-9) screens for major depressive
substance abuse) disorder. The CAGE Questionnaire is a brief screen for alcohol use
disorders (see Routine Care of the Healthy Patient).
Verbal, physical, or sexual abuse Sensitivity and empathy are essential when eliciting this information.
Assessment for Opioid Therapy

Risk stratification for initiating therapy The Screener and Opioid Assessment for Patients with Pain (SOAPP)
and Opioid Risk Tool (ORT) are patient-completed. The Diagnosis,
Intractability, Risk, and Efficacy (DIRE) tool is provider-completed.
Follow-up The 6 A's are a useful framework for follow-up visits: Analgesia,
Activities of daily living, Adverse events, Aberrant behavior,
Assessment, Action plan.
'Several tools are available for assessments; listed scales are examples.

table 17. Classification of Chronic Pain Mechanisms and Recommended Pharmacologic Therapies
Mechanism of Pain Description Examples Medications

Neuropathic Burning, shooting, stabbing Diabetic peripheral neuropathy, Systemic: gabapentin,


postherpetic neuralgia, pregabalin, TCAs, duloxetine,
fibromyalgia3, multiple sclerosis, venlafaxine, tramadol,
trigeminal neuralgia opioids, carbamazepine
Local: topical lidocaine
5% patch, capsaicin
Muscle Tender trigger points Myofascial pain syndrome, TCAs, milnacipran (for
Pain often involves the neck, fibromyalgia3 fibromyalgia)
shoulders, arms, low back,
hips, lower extremities
Inflammatory Involved joints are warm, Rheumatoid arthritis and other DMARDs, NSAIDs, TCAs
erythematous, and swollen inflammatory arthropathies
Mechanical/compressive Aggravated by activity, Back pain, neck pain, NSAIDs, acetaminophen,
relieved by rest musculoskeletal pain TCAs, duloxetine
DMARD = disease-modifying antirheumatic drug; TCA = tricyclic antidepressant.

"Fibromyalgia pain may be both neuropathic and muscular in origin.

Management and flexion/extension exercises are equally beneficial. "Passive"


modalities (transcutaneous electrical nerve stimulation
General Principles
[TENS], massage, and ultrasound) should only be used in the
The goal of CNCP treatment is to improve function and
context of an active exercise program; massage therapy may
quality of life in the context of" pain that may be ongoing. A be particularly beneficial for patients with low back pain,
comprehensive treatment plan may include physical rehabili fibromyalgia, and knee osteoarthritis. All patients should be
tation, cognitive-behavioral therapy (CBT), management of" taught self-management strategies that can improve pain,
comorbid psychiatric conditions, complementary and alter such as ice and heat therapy.
native therapy, and pharmacologic therapies. A graded exer CBT techniques include biofeedback, niindf'ulness-based
cise program will help to improve functional status, and stress reduction, imagery, and hypnosis, and have been shown
machine muscle strengthening, aerobic low-impact exercises, to have small but positive effects on pain, disability, and

34
Common Symptoms

mood. Depression and CNCP are frequently coexistent, and are treated with an NSAID should also be prescribed a pro
simultaneous treatment should be initiated if the depression ton pump inhibitor to reduce gastric toxicity; this should
is mild to moderate in severity. In contrast, psychiatric ther bc considered in any patient on long-term NSAIDs.
apy is the initial focus of treatment in the setting of severe Contraindications to NSAID therapy include current peptic
major depressive disorder and CNCP, as untreated patients ulcer disease, chronic kidney disease, and heart failure. For
will not be able to actively work toward achieving rehabilita patients with high cardiovascular risk, naproxen may be a safer
tion and treatment goals. choice than diclofenac or ibuprofen. Diclofenac has been
Acupuncture has been studied for various CNCP syn associated with increased cardiovascular risk as compared with
dromes. The limited evidence for effectiveness for neck pain other NSAIDs, and ibuprofen interferes with the antiplatelet
does not appear to be clinically significant. The improvement effects of"aspirin.
with acupuncture in chronic back pain, osteoarthritis- knee Adjuvant therapies such as gabapentin and prega
pain, and fibromyalgia has not been demonstrated to be supe balin are efficacious for the treatment of"neuropathic pain
rior to sham acupuncture. Recent studies have shown that and have few drug interactions. According to a Cochrane
manipulative therapy, when included as part of an interdisci systematic review, pregabalin dosed at 600 mg/d pro
plinary treatment program, can reduce chronic pain. Herbal vided substantial pain relief for postherpetic neuralgia,
medications should be used cautiously because of"the risk for diabetic neuropathy, and fibromyalgia (number needed
drug interactions and adverse effects. Feverfew and willow to treat = 3.9, 5.0, and 11.0, respectively), but treatment-
bark appear to be effective for treating headaches and back associated dizziness and somnolence were common.
pain, respectively, but litde evidence supports the use of glu Tricyclic antidepressants (TCAs) can be effective for
cosamine, chondroitin, dimediylsulfoxide, or devil's claw. fibromyalgia and other central sensitization pain syn
dromes, but may produce significant adverse reactions,
Nonopioid Medical Therapies including constipation, dry mouth, conduction abnor
Although medication is not die sole focus of the treatment malities, and urinary retention; their use should be
plan, it is a useful adjunct to care for many patients with CNCP. avoided in the elderly. Duloxetine, a norepinephrine
The World Healdi Organization analgesic ladder (see Palliative transporter (NET) inhibitor that has been approved for
Care), although developed for the palliative care setting, is the treatment of fibromyalgia and diabetic neuropathy,
widely used in the management of CNCP. Pharmacologic pain and tramadol both also inhibit serotonin uptake, making
management should begin widi nonopioid agents, such as them effective for neuropathic pain. In patients with pos
acetaminophen, NSAIDs, and die selective cyclooxygenase-2 therpetic neuralgia and diabetic neuropathy, combination
(COX-2) inhibitor celecoxib. Adjuvant therapies include anti therapy with gabapentin and nortriptyline, as compared
depressants, anticonvulsants, muscle relaxants, and topical with monotherapy with each agent, produces better pain
medications. Tramadol is a unique analgesic that activates u- relief" with fewer adverse reactions.
opioid receptors and also inhibits serotonin reuptake. It is
effective for the treatment of"moderate to severe chronic pain, Opioid Therapy
but may increase the risk for suicide in certain patients; more Opioid therapy should be reserved for patients with moder
over, tramadol may be abused or subject to criminal diversion. ate to severe neuropathic pain that has been unresponsive to
Caution is necessary when initiating tramadol in patients who other pharmacologic therapy. It is typically not beneficial in
are taking serotonin reuptake inhibitors, as cotreatment can patients with inflammatory or mechanical/compressive pain.
increase the risk for serotonin syndrome. The significant side effects associated with opioid therapy
Factors to consider when selecting a medication include (constipation, fatigue, nausea), as well as the attendant risks
the type of pain being treated, side effects, drug interactions, for abuse and addiction, make appropriate patient selection
and patient comorbidities. Medications that are appropriate essential. The most important risk factors for aberrant drug-
for each type of pain are listed in Table 17. taking behaviors are a personal or family history of" drug or
Acetaminophen is generally safe but should not exceed 4 alcohol abuse, age younger than 45 years, and a history of"
g/d and should be avoided or used cautiously in patients with psychiatric disease. Other risk factors include female sex, cig
liver disease. NSAIDs are reasonable alternatives or supple arette smoking, preadolescent sexual abuse (in women), pre
ments to acetaminophen, especially if pain is associated with vious legal problems, history of" motor vehicle accidents,
inflammation. NSAIDs and COX-2 inhibitors should be used and poor family support. The DIRE score (Diagnosis,
cautiously, as they can increase die risk of gastritis, kidney dys Intractability, Risk, and Efficacy) is a physician-completed risk-
function, and adverse cardiovascular outcomes. In older strati fication tool that can be helpful for determining which
patients, the risk for adverse reactions is increased, and the patients are most suitable for opioid therapy; higher scores
American Geriatrics Society' recommends acetaminophen, (that is, a more severe diagnosis, clearly intractable pain, lower
rather than NSAIDs, as first-line therapy. Older patients who psychosocial risk, no chemical dependence history, and higher

35
Common Symptoms

efficacy of opioids already used) predict greater success with and false-negative tests, and correlate results clinically
treatment. The DIRE score can be accessed at www (that is, a specific type of urine testing is required for
.icsi.org/pain chronic assessment_and_management_of hydrocodone).
_14399/pain chronic assessment_and_management_of" Patients should be seen monthly for the first 3 months
guideline_.html (Appendix E). Table 16 lists additional after opioid therapy is initiated; once a stable regimen is
tools that may be helpful for assessing risk. achieved, visits may occur every 3 to 6 months. More fre
Patients who are selected for opioid therapy should have quent visits, with possible therapy restructuring or discon
a thorough understanding of" its risks and benefits. The tinuation of opioid therapy, should occur if" there is evidence
patient and the physician should work together to develop of" drug abuse or misuse, significant side effects, or lack of
an opioid management plan, or pain contract, that outlines functional improvement. Patients on high doses of opioid
agreed-upon goals and rules of" treatment. (An example is therapy should also be seen more frequently, as a recent ret
available in the chronic pain guideline linked to above,
rospective cohort study suggests that such patients are at
Appendix F.) Typically, pain contracts include stipulations higher risk for medically serious or fatal overdose. That study
diat pain medications will not be sought elsewhere, that the found that patients prescribed 100 mg/d of"opioid medica
patient will abstain from illicit drugs, will keep clinic tion were nine times more likely to have an overdose than
appointments as scheduled, and will obtain randomly sched patients prescribed 20 mg/d or less. Patients prescribed
uled urine toxicology screens. A copy of this signed docu 50 mg/d to 99 mg/d were at four times the risk of" those
ment can be given to the patient and reviewed periodically on lower doses.
at follow-up visits to ensure that treatment expectations are
KEY POINTS
being met.
Medication selection should be influenced by the sever Before initiating treatment for chronic noncancer
ity and frequency of"pain; long-acting opioids, which main pain, providers should assess for common behavioral,
tain more consistent drug levels, are preferred for the treat social, or systems barriers that may prevent a patient
ment of CNCP. Physicians should be cautious when from experiencing functional improvement.
initiating methadone, which can cause QT-interval prolon All patients with chronic noncancer pain should be
gation, hypotension, and cardiac arrhythmias. An electro taught self-management strategies that can improve
cardiogram should be obtained at baseline, after 30 days of pain.
treatment, and annually thereafter. Methadone should be Pharmacologic management of chronic pain should
started at low doses and gradually increased to effective
start with acetaminophen, NSAIDs, and adjuvant
doses. Although methadone requires regular monitoring of
pain therapies.
QT intervals, it can be effective when other opioids are not. Opioid therapy should be reserved for patients with
In addition, methadone lacks the euphoric effects of" mor
chronic moderate to severe neuropathic pain that has
phine and other opioids that can contribute to dose escala
tion and potential abuse. been unresponsive to other treatments.
Careful monitoring of" patients treated with opioid
therapy is essential, and clinicians can use the "6 A's" as a
framework for assessment during follow-up visits. Patients Cough
should be queried about the effectiveness of Analgesia, the Cough is one of the most common symptoms for which
benefit of therapy on their Activities of daily living, Adverse patients seek medical attention from either a primary care
events associated with treatment, and Aberrant behaviors physician or a pulmonologist. In addition, it has significant
suggesting drug abuse; providers should also Assess mood impact on health care expenditures. According to the
and review the Action (treatment) plan. Clues to aberrant American College of" Chest Physicians, up to 30,000,000
drug-taking behaviors include multiple episodes of" pre physician visits annually are for cough-related symptoms,
scription loss, repeated requests for dose increases or early and billions of dollars are spent on medication for sympto
refills, drug requests by name, missed appointments, matic relief.
repeatedly seeking prescriptions from other clinicians, not A cough is triggered by chemical or mechanical stim
following through with other components of the treat ulation of cough receptors. These receptors are located in
ment plan, and aggressive complaining about needing the upper and lower respiratory tracts as well as in the
more of" the drug. Random urine drug testing, which is stomach, gastroesophageal junction, diaphragm, esopha
recommended for all patients on opioid therapy, may be gus, pericardium, and ears. The knowledge of the location
conducted more frequently if patients exhibit any of these of" cough receptors and the chemical and mechanical trig
behaviors. When interpreting the results of urine drug test gers of the cough reflex contributes to understanding the
ing, providers should consider the possibility of false-positive differential diagnosis. Recent guidelines suggest an empiric

36
Common Symptoms

and integrative approach to the management of cough the common cold, first-generation antihistamines, deconges
based on duration of"cough. tants, inhaled ipratropium bromide, cromolyn sodium, and
naproxen are helpful in decreasing sneezing and rhinorrhea.
Acute Cough Newer-generation nonsedating antihistamines are ineffective.
Acute cough is a cough that is present for less than 3 weeks. A review of" 17 trials in adults concluded that centrally acting
Upper respiratory tract infections (rhinosinusitis, pharyngitis) (codeine, dextromethorphan) or peripherally acting (mogu-
and acute bronchitis are the most common causes. Other isteine, benzonatate) antitussive dierapy results in little, if any,
considerations include exacerbations of COPD, pneumonia, improvement in cough associated with upper respiratory tract
infection. The American College of'Chest Physicians does not
allergic rhinitis, left ventricular failure, asthma, medications,
and aspiration. recommend their use. P,-agonists should not be used unless
Rhinosinusitis (the common cold) and acute bronchitis cough is accompanied by wheezing.
are most commonly caused by viruses (influenza A and B,
Subacute and Chronic Cough
parainfluenza, coronavirus, rhinovirus, and respiratory syncy
tial virus). Nonviral causes include Mycoplasma pneumoniae, Subacute cough, a cough of" 3 to 8 weeks' duration, is most
Chlamydophila pneumoniae, and Bordetella pertussis commonly postinfectious. If an infectious origin is unlikely,
(whooping cough). Fever may or, more likely, may not be upper airway cough syndrome (UACS, previously called
present. Cough with purulent sputum is not a reliable indi postnasal drip syndrome), asthma, pertussis, acid reflux, or
cator of a bacterial infection. In acute bronchitis, the cough acute exacerbation of primary lung disease should be consid
generally lasts more than 5 days and, although most resolve ered. Chronic cough is defined as die persistence of cough for
in 3 weeks, bronchial hyperreactivity can lead to persistence longer than 8 weeks. The most common causes are UACS,
of cough for up to 8 weeks. asthma, nonasthmatic eosinophilic bronchitis (NAEB), and
The incidence of pertussis has increased over the past two gastroesophageal reflux disease (GERD). In several case
decades. If suspected, culture via nasopharyngeal aspirate or series, UACS, asthma, and GERD accounted for 90% of
swab and macrolide antibiotic treatment are indicated. patients with chronic cough (excluding those with cough
Routine antibiotic treatment of uncomplicated upper related to smoking or ACE inhibitors). Other, less common,
respiratory tract infections and acute bronchitis in causes are chronic bronchitis, bronchiectasis, lung cancer,
nonelderly immunocompetent patients is not recom aspiration, irritation of the external auditory canal, and psy
mended. Despite recommendations to avoid antibiotics in chogenic. There are often multiple causes for a case of
most patients, antibiotic overuse is common. Up to 60% of chronic cough.
patients who present with upper respiratory tract infection The medical history and physical examination may sug
symptoms or acute bronchitis arc given antibiotics. Patient gest a potential cause or causes, but neither is reliable for
satisfaction with care for acute bronchitis depends primarily definitively ruling in or ruling out specific disease. Patients
on physician-patient communication rather than on antibi with chronic cough, especially smokers, should undergo
otic prescription. chest radiography. If" the chest radiograph is normal, the
Lower respiratory tract infection or pneumonia can pre physician should consider UACS, asthma, NAEB, and
sent with cough, but these infections generally are accompa GERD and begin a stepwise, sequential approach for evalu
nied by fever, constitutional symptoms, pleuritic chest pain, ation and treatment (Figure 5). The definitive diagnosis
and abnormalities on pulmonary examination. In patients may be known only after successful individual or joint
with such findings and moderate to severe symptoms, a chest empiric treatment.
radiograph should be obtained. Influenza should be consid The use of a systematic algorithmic approach in the
ered in any patient during the appropriate season who pres immunocompetent patient with chronic cough can lead to
ents with cough, fever, myalgia, and headache. successful outcomes in more than 90% of patients. In general,
Approximately 15% of" patients on an ACE inhibitor unless symptoms point to a specific diagnosis or there is a
develop cough. The cough usually begins within 1 week of" definitive finding on chest radiograph, empiric therapy for
starting therapy, although in some patients it may be delayed. UACS for 2 to 3 weeks is started first. If there is no response,
The medication should be discontinued. Cough generally evaluation and treatment for asdima, NAEB, and GERD
abates within 1 to 4 weeks. Since this is a class-specific effect, should ensue. All patients with chronic cough who smoke
rechallenge with a different ACE inhibitor is not recom should receive smoking cessation counseling.
mended. An angiotensin receptor blocker can be substituted Specific therapy for each diagnosis should be optimized.
for an ACE inhibitor as these medications generally do not For patients with UACS, first-generation antihistamines and
cause cough. decongestants remain first-line therapy. Patients with cough-
Treatment of" the patient with acute cough is based on variant asthma may demonstrate reversible airflow obstruction
primary diagnosis and is mainly supportive. For patients with or airway hyperreactivity with bronchoprovocation testing.

37
Common Symptoms

Chronic Cough

A cause of cough History, examination,


is suggested chest radiograph Smoking, ACEI

Investigate
and treat Discontinue

Inadequate response
to optimal therapy No response

Upper airway cough syndrome (UACS)


Empiric treatment
Asthma
Ideally evaluate (spirometry, bronchodilator
reversibility, bronchial provocation challenge)
or empiric treatment
Non-asthmatic eosinophilic bronchitis (NAEB)
Ideally evaluate for sputum eosinophilia
or empiric treatment

Gastroesophageal reflux disease (GERD)


Empiric treatment

For initial treatments see box below

Inadequate response
to optimal therapy

Further investigations to consider:


24-h esophageal pH monitoring
Endoscopic or videofluoroscopic
swallow evaluation
Barium esophagram
Sinus imaging
High-resolution CT
Bronchoscopy
Echocardiogram
Environmental assessment
Consider other rare causes

General considerations: Initial treatments:


Optimize therapy for each diagnosis UACS: antihistamine/decongestant
Check compliance Asthma: inhaled corticosteroid, bronchodilator, LTRA
Due to possibility of multiple causes, NAEB: inhaled corticosteroid
maintain all partially effective treatments GERD: proton-pump inhibitor, diet/lifestyle

FIGURE 5. Evaluation of chronic cough. ACEI = angiotensin-converting enzyme inhibitor; LTRA = leukotriene receptor antagonist.
Reproduced with permission from the American College of Chest Physicians. Irwin RS, Baumann MH, Bolser DC. et al; American College of Chest Physicians (ACCP). Diagnosis and management
of cough executive summary: ACCP evidence-based clinical practice guidelines. Chest. 2006; 129(1 suppl): 1S-23S. |PMID: 16428686] Copyright 2006. American College of Chest Physicians.

38
Common Symptoms

However, as bronchoprovocation testing may yield false-pos embolism, mitral stenosis, left-sided heart failure). All patients
itive results, asthma should be diagnosed as a cause of chronic with hemoptysis should undergo chest radiography and, if
cough only if symptoms abate after 2 to 4 weeks of standard indicated, chest CT or bronchoscopy.
antiasthmatic therapy with an inhaled bronchodilator and KEY POINTS
inhaled corticosteroids. NAEB is diagnosed in patients who
Routine antibiotic treatment of uncomplicated
have sputum eosinophilia but are without airway hyperreac
tivity. These patients are treated with inhaled corticosteroids. upper respiratory tract infections and acute bronchi
tis in nonelderly immunocompetent patients is not
GERD may be diagnosed in patients with typical symptoms
or in those who fit a typical clinical profile and in whom near- recommended.
Neither centrally acting nor peripherally acting anti
complete or complete resolution of symptoms occurs widi
antireflux treatment. Typical heartburn symptoms may be tussive agents have demonstrated improvement in
present in only 60% patients widi chronic cough caused by acute cough associated with upper respiratory tract
GERD. Although 24-hour pH monitoring may be helpful in infection.
the evaluation of patients widi suspected GERD, empiric
treatment can be initiated before testing. Effective treatment
modalities include dietary and lifestyle modification and acid Chronic Fatigue and Chronic
suppressive therapy with proton pump inhibitors for 1 to 3 Fatigue Syndrome
months (see MKSAP 16 Gastroenterology and Hepatology).
When disease-based specific therapy fails, cough sup Fatigue is a common symptom in primary care, occurring in
a fifth to a quarter of patients. Fatigue is difficult to define and
pressants may be helpful. Unlike in treatment of acute
quantify and is often viewed as a minor problem. Chronic
cough, some clinical trials suggest that centrally acting nar
cotic (morphine or codeine) or nonnarcotic (dextromethor fatigue is variably defined as lasting longer than 30 days or
longer than 3 months, with a resulting inability to perform
phan) medications may be effective in the treatment of
desired activities. Chronic fatigue may be secondary to vari
chronic cough. Peripherally acting antitussives may also be
ous diseases, including malignancy; autoimmune and
beneficial. As with any long-term medication, risks and ben
endocrine disorders; neurologic diseases (multiple sclerosis,
efits need to be considered.
Parkinson disease); chronic kidney, lung, heart, or liver dis
ease; HIV infection; substance abuse; medication side effects;
Cough in the Immunocompromised Patient
and heavy metal poisoning.
In addition to being at risk for the common community-
Chronic fatigue syndrome (CFS) is a distinct entity of
acquired infections seen in the immunocompetent host, the
fatigue that persists for 6 months or more. Diagnostic crite
immunocompromised patient is at risk for various oppor
ria developed for research purposes have been applied widely
tunistic infections that may present with cough, such as
in die clinical setting. The International CFS Study Group
tuberculosis, Pneumocystis jirovccii pneumonia, and
definition includes medically unexplained fatigue of longer
aspergillosis. The degree and duration of immunosuppres
dian 6 months' duration after clinical evaluation, with four or
sion, along with whether the primary impairment is in
humoral or cell-mediated immunity, can assist in determin more of the following symptoms: subjective memory impair

ing the more likely causes. Definitive work-up is indicated ment, sore throat, tender lymph nodes, muscle or joint pain,
in immunocompromised patients with cough. Empiric headache, unrefreshing sleep, and postexertional malaise last
antibiotic therapy should be initiated while diagnostic test ing longer than 24 hours; exclusion criteria include the pres
ence of substance abuse, an eating disorder, an underlying
ing is used. Treatment can be modified based on subse
quent microbiologic test results. psychiatric disorder, dementia, or severe obesity (BMI > 45).
Chronic fatigue of longer than 6 months' duration that does
Hemoptysis not meet criteria for CFS is designated idiopathic chronic

Hemoptysis is defined as coughing up any amount of blood fatigue.


from the lower respiratory tract. Hemoptysis must be distin
guished from hematemesis or nasopharyngeal bleeding. Diagnosis and Evaluation of Chronic Fatigue
Massive hemoptysis (>200 mL/d) can be life-threatening. There are no specific recommendations regarding diagnostic
The most common causes of hemoptysis are infection (airway evaluation for chronic fatigue. Patients with fatigue greater
inflammation) and malignancy. Other sources include the tra thanl month rarely have abnormalities on either physical or
cheobronchial tree (bronchitis, bronchiectasis, tumor), pul laboratory evaluation; testing should thus be judicious and
monary parenchyma (abscess; pneumonia, including tubercu performed only when clearly indicated. The degree of func
losis; Goodpasture syndrome, granulomatosis with polyangiitis tional limitation of patients widi fatigue is often underesti
[also known as Wegener granulomatosis]), and pulmonary mated by health care providers, resulting in inadequate or
vasculature (arteriovenous malformation, pulmonary incomplete attention and treatment.

39
Common Symptoms

Historical elements associated with CFS include unre- KEY POINTS


freshing sleep, subjective memory impairment, and substan Fever, lymphadenopathy, or muscle wasting in a
tial curtailment in previous level of" functioning. CFS has
patient with chronic fatigue warrants further evalua
been associated with various conditions, including post-viral
tion for organic causes and should not be attributed
infection (parvovirus B19), childhood trauma, and preexist
to chronic fatigue syndrome.
ing psychiatric disorders; many associations have not been Treatment of chronic fatigue syndrome is largely
reproduced in other studies, or are only seen in small num
bers of patients, and thus cannot be clearly attributed as nonpharmacologic and includes lifestyle modifica
tion, sleep hygiene, graded activity, and cognitive-
causes of" CFS.
behavioral therapy.
Physical examination in patients with chronic fatigue is Patients with chronic fatigue syndrome should have
usually normal and performed with the intent to exclude
other possible organic causes. Findings of" fever, lym regular follow-up to monitor their symptoms, for
support and validation, and to avoid unnecessary
phadenopathy, and muscle wasting warrant further evalua
tion for organic causes and should not be attributed to CFS. diagnostic and treatment interventions.
Selected laboratory or other diagnostic studies may help
identify the cause of" chronic fatigue or rule out treatable
causes.
Dizziness
Dizziness is a frustrating and acutely debilitating symptom for
many patients. It is more common in women, and prevalence
Management of Chronic Fatigue and Chronic in die elderly may be as high as 37%. Patients with dizziness
Fatigue Syndrome are at increased risk for falls and nursing home placement.
Management of CFS is challenging and requires a compre
Although most causes of dizziness have a benign course, they
hensive strategy tailored to the patient's individual goals and
may be associated with life-threatening consequences, includ
needs. Providers must consider the risks of over-investiga
ing stroke and death.
tion, including reinforcement of the patient's belief" that a The evaluation of dizziness is challenging and without
treatable organic cause may be found, the potential hazards
universally accepted guidelines. The history and physical
of testing itself, and false-positive findings, as well as time examination are the most effective diagnostic tools and are
and cost issues. Treatment is directed at the underlying ill used to classify dizziness into four categories: (1) vertigo,
ness, if identified; nonspecific therapies include counseling, (2) presyncope, (3) dysequilibrium, and (4) other causes. The
exercise, and possibly medications. Nonpharmacologic ther history and physical examination can also help to distinguish
apy includes lifestyle modification, sleep hygiene, and graded peripheral or otologic disease from central disease. Although
activity; selected patients may benefit from referral for CBT, categorization is attractive, it is not always possible, especially
physical rehabilitation, or psychiatric management. One in die elderly. Up to one half of" geriatric patients with dizzi
goal should be to prevent further deterioration in functional ness have multiple causes in more than one category.
ability, which may be accomplished with supportive man Nonetheless, this schema forms the current best framework
for diagnosis, evaluation, and management.
agement and by limiting largely ineffective treatments, such
as corticosteroids and immunotherapy. Patients widi comor In general, key elements of the history include timing of
bid depression should be offered antidepressant therapy, but symptoms, duration, provocative or palliative measures, and
risk factors for atherosclerotic disease. All patients should have
no specific class of medications is recommended specifically
orthostatic vital signs taken and undergo thorough cardiac
for CFS. The limited number of" small randomized con
and neurologic examinations. Routine laboratory testing is
trolled trials (RCTs) do not provide conclusive evidence for
not helpful.
the effectiveness of dietary supplements and herbal remedies
for CFS.
Patients with CFS should have regular follow-up to
Vertigo
Vertigo is the illusion of" movement, either personal or envi
monitor their symptoms, for support and validation, and to
ronmental, caused by unilateral or asymmetric disruption of
avoid unnecessary diagnostic and treatment interventions.
peripheral or central vestibular structures. It is typically, but
Prognosis is variable and related primarily to severity and not always, rotational. In studies that examined dizziness in
degree of impairment. In studies, patients with less impair primary care, specialty, or emergency settings, vertigo was the
ment or fatigue of" shorter duration were more likely to most common type, present in approximately half of patients.
recover, although functional outcome is often not reported or It may be accompanied by severe nausea, vomiting, nystag
standardized, thus limiting the definition of recovery. Other mus, and postural instability. Central causes of vertigo include
predictors of"poor outcome include self-reported poor health vascular disease and stroke, mass lesions of the brainstem and
and coexisting somatic or mental health disorders. cerebellum, multiple sclerosis, migraine, and seizures.

40
Common Symptoms

Peripheral causes of"vertigo include benign paroxysmal posi in bed. Auditory or associated neurologic symptoms are
tional vertigo (BPPV), vestibular neuronitis, and Meniere dis absent. BPPV is caused by movement of otoliths or odier
ease. Less common peripheral causes include aminoglycoside debris in the semicircular canals (most commonly die poste
toxicity, herpes zoster, otitis media, and perilymph fistulas. rior semicircular canal) induced by head movement. This
Diagnosing the cause of" vertigo is critical because tar leads to perturbation of sensory receptors in die vestibular
geted, disease-specific treatment can improve symptoms and labyrinth. Recurrences are common, reported to be 18% at 1
prognosis. The duration of symptoms can guide the differen year and 30% at 3 years.
tial diagnosis (Table 18), and results from the Dix-Hallpike Vestibular neuronitis is acute in onset (hours) and is fre
maneuver can distinguish central from peripheral disease quently associated with a viral infection that affects the
vestibular portion of the eighth cranial nerve. Nausea and
(Table 19). In the Dix-Hallpike maneuver, the patient is
instructed to sit upright, turn die head 45 degrees, and keep vomiting are common; no brainstem symptoms are present
both eyes open during the entire maneuver. The examiner sup (distinguishing it from central vertigo). Hearing usually is not
affected, but if it is, the term labyrinthitis is used. Symptoms
ports the head and, while instructing the patient to lie down,
can be very severe, and although symptoms usually peak dur
rapidly places the head below the level of the examining table.
The examiner notes nystagmus and subjective symptoms. The ing the first 24 hours and resolve within 7 days, full recovery
test is repeated widi die head turned to the opposite side. may take longer. Residual dizziness can last for months.
Meniere disease, also known as idiopathic endolymphatic
Peripheral Vertigo hydrops, is characterized by the classic triad of" vertigo, uni
lateral low frequency hearing loss, and tinnitus, occasionally
Clinical Presentation associated with aural fullness. Endolymphatic hydrops refers
The most common cause of vertigo is BPPV. Patients with to a condition of increased hydraulic pressure in die inner ear
BPPV classically report recurrent, intense, and brief episodes
endolymphatic system leading to these symptoms. In Meniere
of"vertigo (1 minute or less) with a rapid change in head posi disease, the cause of the increased pressure is not known.
tion, such as turning the head while driving or turning over Vertigo may be the first presenting sign. The diagnosis may
be secured only after repeated attacks of vertigo with associ
ated hearing loss.
TABLE 18. Duration of Vertigo and Suggested Causes j

Duration of Vertigo Treatment


Underlying Cause
BPPV can be treated by the Epley maneuver (Figure 6),
Seconds Benign paroxysmal positional which can be curative. This maneuver attempts to move the
vertigo
debris floating in the semicircular canal to a position where it
Minutes to hours Transient ischemic attack can exit into the utricular cavity. An evidence-based review of
Meniere disease all major studies analyzing repositioning procedures con
Perilymph fistula cluded that this therapy is beneficial. In one study, 61% of the
Migraine treated group had complete resolution of" symptoms at 4
Acute vestibular weeks compared with 20% in the control (sham-treated)
Days
neuronitis/labyrinthitis group. The number needed to treat was 2.4. Medications
Ischemia/stroke generally are ineffective in BPPV.
For other causes of vertigo, particularly vestibular neuroni
Migraine
tis, treatment options are generally limited to symptom relief".
Multiple sclerosis
Vestibular suppressants and antiemetic drugs (antihistamines,
Weeks Psychogenic
benzodiazepines, and phenothiazines) are die three major drug

table 19. Interpretation of Dix-Hallpike Maneuver Findings in E v a l u a t i o n of Vertigo [


Characteristic Peripheral Disease Central Disease

Latency of nystagmus 2-40 s No latency


Duration of nystagmus <1 min >1 min

Severity of symptoms Severe Less severe


Habituation Yes No

Fatigability Yes No
Direction of nystagmus Horizontal, with rotational component; Can be vertical, horizontal, or torsional; may change
never vertical with position

41
FIGURE 6. Epley maneuver for relieving benign paroxysmal positional vertigo. The patient sits on the examination table, with eyes open and head turned 45
degrees to the right (A).The physician supports the patient's head as the patient lies back quickly from a sitting to supine position, ending with the head hanging 20
degrees off the end of the examination table and still turned to the right (fi). The physician turns the patient's head 90 degrees to the left side. The patient remains
in this position for 30 seconds (Q.The physician turns the patient's head an additional 90 degrees to the left while the patient rotates his or her body 90 degrees in
the same direction. The patient remains in this position for 30 seconds (D).The patient sits up on the left side of the examination table (Q.The procedure may be
repeated on either side until the patient experiences relief of symptoms.
Maria Hartsock. CMI. Reprinted from Swartz R, Longwell P. Treatment of Vertigo. Am Fam Physician. 2005;71(6):1115-1122, 1129-1130. [PMID: 15791890]

42
Common Symptoms

classes that may modify die intensity of symptoms. The most Dysequilibrium
common drugs are centrally active antihistamine anticholinergic Dysequilibrium is defined as imbalance or unsteadiness
medications. These medications have been shown to reduce while standing or walking, without the sensation originat
symptom severity. Benzodiazepines act centrally to suppress ing in the head. It is relieved with sitting or recumbency.
vestibular responses. Phenothiazinc antiemetics are useful when Dysequilibrium is caused by defective sensory input (vision,
nausea and vomiting accompany vertigo. Side effects of" all of" vestibular), impaired proprioception or motor function, gen
these medications include sedation. Some experts advocate cor eralized weakness, Parkinson disease, joint pain, or anxiety or
ticosteroid dierapy, but robust outcome data are lacking. Data other psychiatric disorders. Medications can be contributing
for calcium channel blockers, betahistine, corticosteroids and factors. A multidisciplinary approach to treatment is fre
ginger root are either weak or conflicting. Medications are rec quently indicated. The plan may include physical therapy,
ommended only for short periods (24-48 hours). More pro assistance devices for ambulation, audiometry testing, vision
longed use may suppress vestibular feedback and central com testing, and medication reconciliation.
pensation mechanisms, leading to prolongation of symptoms.
Referral to a trained therapist for vestibular rehabilitation Nonspecific Dizziness
(VR) is helpful for peripheral vertigo, especially if initiated early. Vague signs of dizziness, lightheadedness, or "vvoozincss"
In a review of 27 moderate- to high-quality trials enrolling
comprise the last and typically most frustrating category of"
community-dwelling adults, VR proved to be effective. dizziness. Although frequently attributed to psychiatric dis
Compared with control groups, VR led to improvement in orders, especially anxiety and depression, there may be over
symptoms, walking, balance, vision, and activities of daily liv lap with other categories. Other specific causes include hypo
ing. Exercises include learning to bring on symptoms to glycemia, hyperglycemia, electrolyte abnormalities, thyroid
desensitize the vestibular system, learning to coordinate head
disorders, anemia, and multiple classes of" medications.
and eye movements, and improving balance and walking skills.
KEY POINTS
Although caffeine restriction, salt restriction, and diuretic
therapy have been advocated for Meniere disease, a recent All patients being evaluated for dizziness should have
review concluded that there is insufficient high-quality evidence orthostatic vital signs taken and undergo thorough
to recommend diese interventions. Similarly, there are no cardiac and neurologic examinations; routine labora
strong data for corticosteroids or immunosuppressive dierapy. tory testing is not helpful.
Benign paroxysmal positional vertigo (BPPV) can be
Central Vertigo treated by the Epley maneuver, which attempts to
Ischemia, infarction, and hemorrhage of the brainstem or move the debris floating in the semicircular canal to a
cerebellum are life-threatening causes of vertigo. Patients at
position where it can exit into the utricular cavity;
risk arc diose with hvpertension, tobacco use, hyperlipidemia, medications generally are ineffective for BPPV.
diabetes mellitus, atrial fibrillation, and preexisting athero
Patients with vertigo accompanied by diplopia,
sclerotic vascular disease. Up to one quarter of patients with
risk factors for stroke who present with severe vertigo or nys dysarthria, dysphagia, focal numbness, gait abnormal
ities, or headache may have brainstem or cerebellar
tagmus or who are unable to stand without support have
disease requiring immediate medical or neurosurgical
infarction of"the inferior cerebellum.
The distinction between vertigo due to vascular disease intervention.
and vertigo due to vestibular neuronitis is critical. Neurologic
symptoms arc absent in patients with vestibular neuronitis.
However, patients widi vertigo due to brainstem ischemia,
Insomnia
infarction, or hemorrhage may demonstrate diplopia, Insomnia is defined as any difficulty with sleep initiation, dura
dysarthria, dysphagia, and focal numbness or weakness. Those tion, consolidation, or quality that occurs despite adequate
with cerebellar disease may present with gait abnormalities and opportunity for sleep (in contradistinction to sleep deprivation)
headache. MRI of"the brain with angiography is the preferred and results in daytime sleepiness or odier adverse effects on
diagnostic test. Patients with brainstem or cerebellar disease daytime activities. It can be primary or secondary, die latter
require immediate medical or neurosurgical intervention. being more common. Causes include underlying medical
problems such as chronic pain, depression, GERD, and
Presyncope obstructive sleep apnea. Insomnia also may be secondary to a
Presyncope is the sensation of near loss of consciousness. The poor sleep environment, medications, or odier substances. It
main cause is a global and temporary decrease in cerebral per can be acute (transient) or chronic (occurring at least three
fusion owing to cardiovascular disease (ischemia, arrhythmia, times weekly for at least 1 mondi). Insomnia is sometimes
valvular heart disease), hypotension, carbon monoxide poi defined as initial (difficulty falling asleep), middle (awakenings
soning, anemia, or vasovagal reaction (see Syncope, below). during die night), and terminal (early morning awakening).

43
Common Symptoms

Insomnia is more common in women, the elderly, sessions weekly are required over a several-week period.
patients with comorbid medical problems, shift workers, and Adjunct medications may be used with CBT initially, but
persons of lower socioeconomic status. Patients with chronic long-term studies indicate that CBT alone is best for mainte
insomnia are more likely to use health resources, be absent or nance therapy. Other, less common, treatments include sleep
late to work, and be involved in motor vehicle collisions, and restriction, biofeedback, and relaxation techniques.
they have an increased risk of suicide, depression, anxiety, and The decision to initiate pharmacologic treatment should
substance abuse. As many as 50% of patients identify some take into account the patient's response to other therapies,
aspect of" sleep disorder; clinicians should consider screening patient preferences, treatment goals, comorbid conditions,
for insomnia by asking every patient about their sleep. medication interactions, adverse effects, and cost. Initial rec
ommendations are for short- and intermediate-acting benzo
Evaluation of Insomnia diazepine y-aniinobutyric acid (GABA)-receptor agonists,
Several sleep questionnaires exist for use in patients in whom a nonbenzodiazepine GABA-receptor agonists, and type A
sleep problem has been identified. Alternatively, physicians can melatonin-receptor agonists (Table 20). Nonbenzodiazepine
ask about key components of" the problem, including charac GABA-receptor agonists are preferred over other sedating
terization of die sleep disturbance (duration, frequency, sever agents, including sedating antidepressants, owing to their lack
ity, progression), precipitating factors, past problems, and pre of effect on sleep architecture and their superior safety profile.
vious treatments and response. A thorough sleep history With benzodiazepines, caution is warranted regarding
includes patterns of"sleep and wakefulness (time to bed, time to dependence and tolerance, as well as side effects of daytime
sleep, number of" awakenings), nocturnal symptoms, pre-sleep sedation and psychomotor impairment. They should not be
and sleep environment, and daytime activities. Interviewing a used in patients with a history of drug or alcohol abuse.
bed partner may provide additional information. A formal sleep Nonbenzodiazepine GABA-receptor agonists, such as Zolpi
diary may also be used. Important questions include informa dem and zaleplon, have fewer adverse effects, mostly owing
tion about medication and other substance use, especially over- to their shorter half-life. Adverse effects of" these agents
the-counter (OTC) medications, caffeine, and alcohol. include nausea, vertigo, nightmares, disorientation, and agi
Physical examination should assess for obesity, body tation. Zolpidem is associated widi cases of somnambulism,
habitus consistent widi obstructive sleep apnea (enlarged such as nocturnal eating, driving, and walking. Data are lim
tongue or tonsils, increased neck circumference), thyroid dys ited regarding hypnotic agents for long-term therapy. The
function, heart failure, and neurologic disease. Mental status FDA recommends that treatment be limited to 1 month, and
examination should focus on mood and level of" alertness. sleep specialists do not recommend long-term therapy. Small
Laboratory and other diagnostic evaluations should be per studies have show effectiveness for Zolpidem for up to 8
formed judiciously. An overnight polysomnography may be weeks and for cszopiclone for up to 6 months.
indicated if obstructive sleep apnea or other primary sleep dis Other sedative-hypnotic agents (chloral hydrate, barbitu
turbance (for example, restless legs syndrome or periodic limb rates), gabapentin, and antipsychotic medications are not rec
movements of" sleep) is strongly suspected, or in patients ommended for use for primary insomnia. Antidepressant med
refractory to initial dierapy. ications are recommended for use only when insomnia is one
of the manifestations of an underlying depressive disorder. Use
Management of Insomnia of tricyclic antidepressants is discouraged owing to their sig
Therapy for insomnia is directed at treating any underlying cause nificant side effect profile; antidepressants that are most effica
or associated comorbid conditions. Pain, esophageal reflux, heart cious for use in insomnia are low-dose trazodone or mirtazap-
failure, and obstructive sleep apnea should be treated as appro ine. Antihistamines are often used to treat insomnia, especially
priate. Nonpharmacologic interventions are preferred, especially diphenhydramine, owing to its OTC availability' and inclusion
in the elderly and in those widi chronic insomnia. in OTC products marketed specifically for sleep. Because these
Sleep hygiene refers to behavioral and environmental fac drugs antagonize central H, receptors, their side-effect profile
tors that affect sleep. Improving sleep hygiene is an important is significant and is predominantly anticholinergic with carry
component ofinsomnia management; alone, however, this is over sedation and should be avoided for chronic insomnia.
often insufficient to treat chronic insomnia. Components of" Diphenhydramine has a long half-life and is an inhibitor of
sleep hygiene include maintenance of a stable bedtime and CYP2D6 drug metabolism. Thus, widi its anticholinergic pro
awakening time, appropriate exposure to light during daytime file, it is a particularly bad choice for elderly patients, especially
and darkness during nighttime, avoidance of stimulants and those with mild cognitive impairment or Alzheimer disease or
exercise after 6 I'M, use of the bed for sleeping and sex only, a who are exposed to polypharmacy. OTC melatonin is available,
maximum of 8 hours in bed, adjusting the bed and room and in this form is a nonspecific agonist of melatonin recep
comfort level, and relaxation strategies before bedtime. tors. It may be helpful for short-term use for jet lag and
CBT has been to shown to be more effective for both other circadian rhythm disorders; however, its effectiveness
primary and secondary insomnia than drug therapy. Several compared with specific melatonin receptor agonists available

44
Common Symptoms

TABLE 20. Drug Treatment of Primary Insomnia

Agent, Dosage Half-Life (hour) Side Effects

Benzodiazepines Daytime sedation, dizziness, anterograde amnesia, falls,


rebound insomnia

Estazolam, 1-2 mg 10-24

Flurazepam, 15-30 mg 2-3

Quazepam, 7.5-30 mg 40

Temazepam, 7.5-30 mg 8-15


Triazolam, 0.125-0.5 mg 2-5

Nonbenzodiazepine sedative - Daytime sedation, dizziness, anterograde amnesia, falls,


hypnotic agents rebound insomnia

Zolpidem, 5-10 mg 3

Zaleplon, 5-10 mg 1

Eszopiclone, 2-3 mg 5-7 Unpleasant taste, dry mouth, drowsiness, dizziness

Melatonin-receptor agonist

Ramelteon, 8 mg 2-5 Drowsiness, dizziness, increased serum prolactin levels

Adapted with permission from Wilson JF. In the clinic Insomnia. Ann Intern Med. 2008; 148< 1 >:ITC 13-1-1TC13-16. (PMID: 18166757] Copyright 2008. American College of Physicians.

by prescription for acute and chronic insomnia is not known. Antihistamines such as diphenhydramine have a long
Many patients drink alcohol to help with sleep; although alco half-life, have significant anticholinergic side effects,
hol may help in falling asleep, it interferes with sleep architec
and often have carry-over sedation; they are a poor
ture and often causes sleep disruption in the latter half of the
choice for treating insomnia in the elderly.
night. Dopaminergic agents may be helpful for patients with
insomnia associated with restless legs syndrome (see MKSAP
16 Neurology).
Referral to a sleep specialist is indicated if the etiology of"
Syncope
insomnia remains unclear, ifdaytime functioning is impaired, Syncope is defined as the transient loss of"consciousness with
loss of postural tone and spontaneous recovery, resulting from
if" the insomnia is refractory to therapy, or if the patient
global cerebral hypoperfusion. Loss of consciousness distin
requests it. Consultation may also be helpful for patients with
restless legs syndrome or other primary sleep disorders, such guishes syncope from pseudosyncopal events, such as drop
attacks and simple falls, and global cerebral hypoperfusion dis
as narcolepsy. Pulmonary, otolaryngology, or dental referral
tinguishes syncope from other causes ofloss of'consciousness,
may be helpful for patients with obstructive sleep apnea or if such as seizure and stroke. The loss of"consciousness of"syn
specific upper airway anatomic abnormalities are suspected.
cope is usually less than 1 minute, with complete restoration
Psychiatric referral is helpful for patients with concurrent psy of" orientation and function at the time of recovery.
chiatric disorders, patients requiring high doses of medica
tions for treatment, patients with insomnia refractory to
Neurocardiogenic Syncope
treatment, or patients requiring tapering or titration of med
ications or combinations of medications. Neurocardiogenic syncope, die most common type, is pre
dominantly a clinical diagnosis. Vasovagal neurocardiogenic
KEY POINTS syncope (the common "faint") results from a reflex withdrawal
The first-line treatment of insomnia is improving of"sympathetic tone accompanied by an increase in vagal tone,
sleep hygiene; additional options include cognitive- precipitating a drop in blood pressure and heart rate. Without
behavioral therapy and medications. die surge in vagal tone, bradycardia is absent; diis variant is
Short- and intermediate-acting benzodiazepine y- called vasodepressor syncope. Patients with neurocardiogenic
syncope often experience a prodromal phase, usually longer
aminobutyiic acid (GABA)-receptor agonists, nonben
than 10 seconds, characterized by palpitations, nausea, blurred
zodiazepine GABA-receptor agonists, and melatonin-
vision, warmth, diaphoresis, or lightheadedness, aldiough these
receptor agonists are all effective in the treatment of
symptoms are less common in die elderlv'. Provoking factors
insomnia, but have different risks of long-term
include prolonged standing, postural change, hot environ
dependence and tolerance as well as side effects.
ments, emotional distress, and preload-reducing situations

45
Common Symptoms

(dehydration, use of" diuretics or vasodilators). The first and cardiomyopathy. Clues to structural heart disease include
episode usually occurs at a young age, and recurrences are relationship to exercise or exertion, sensitivity to volume sta
common. Variants of" neurocardiogenic syncope that are tus, and association with medications. Cardiac ischemia, pul
specifically situational include cough, sneeze, defecation, monary embolism, and aortic dissection are unusual causes of
swallow, micturition, laughter, post-exercise, and post-pran syncope and rarely occur without other symptoms.
dial syncope. Carotid sinus syncope occurs after mechanical
manipulation of the carotid sinuses, altering sympathetic and Diagnostic Evaluation of Syncope fn
parasympathetic tone; it may be reproduced by carotid sinus The diagnostic evaluation of"syncope begins with a thorough *
massage and is more common in the elderly, in men, and in history and physical examination. The history and physical
those with underlying structural heart disease. examination reveal an etiology in one-third to two-thirds of
patients. In addition, die history often helps with risk stratifi
Orthostatic Hypotension cation as well as with diagnosis. Any witnesses should be inter
Orthostatic hypotension is characterized by an abnormal drop viewed, as patients may not remember important aspects of"
in blood pressure with standing (greater than 20 mm Hg sys the event. Important aspects of"the history include age, posi
tolic or 10 mm Hg diastolic). Since orthostatic changes may tion, prodrome, triggers, associated symptoms, duration of"
be "initial" (immediate), "classic" (within 3 minutes), or symptoms, previous episodes and duration between episodes,
delayed, syncope may occur immediately (0-3 minutes) or be medications, family history (especially in young patients), and
delayed (up to 30 minutes). Orthostatic syncope is more underlying medical conditions. Vital signs, including ortho
common in the elderly; in diose taking vasoactive drugs, static vital signs, often contribute to the diagnosis. Elderh'
diuretics, or alcohol; and in the setting of volume depletion patients may have asymptomatic orthostasis. Thus, it is
or autonomic failure, such as primary or idiopathic autonomic important to document symptom reproduction with dicta
neuropathy. It may occur in association with Parkinson dis tion of orthostatic change; otherwise, an alternative cause for
ease, diabetes, amyloidosis, Shy-Drager syndrome, and lower syncope should be sought. Carotid artery palpation and aus
motor neuron injuries. Patients commonly have symptoms of cultation and detailed cardiopulmonary, abdominal, and neu
dizziness, weakness, and fatigue, both before and after the rologic examinations should be performed. Carotid sinus
event. massage should be performed in patients suspected of"having
A unique variant of orthostatic intolerance is postural carotid sinus hypersensitivity but is contraindicated in patients
orthostatic tachycardia syndrome, usually seen in young with recent myocardial infarction or cerebral ischemia.
women and related to inadequate venous return with signifi Despite its low diagnostic yield, a 12-lead electrocardio
cant tachycardia; patients may experience symptoms of light gram (ECG) remains the first and most widely recommended
headedness and palpitations, but not syncope. test to perform in patients being evaluated for syncope, partly
owing to its noninvasive nature, availability, and low cost.
Cardiac Causes of Syncope Aldiough this test is relatively insensitive for finding a specific
Cardiac diseases account for die remaining causes of true syn cause of syncope, its specificity is high, and an abnormal ECG
cope and predominate in die elderly. Arrhythmias are the is used to identify and stratify patients for additional testing.
most common and the most worrisome causes of syncope and The remainder of the evaluation should be directed by
include bradycardias (sinus and atrioventricular node dys- die unique circumstances of" die patient's event. Guidelines
ftinction) as well as tachyarrhydimias (supraventricular and for syncope generally do not dictate the degree of detail con
ventricular; atrial tachyarrhydimias rarely cause syncope). sidered necessary for fiirdier testing. Echocardiography is rec
Patients with an arrhydimogenic cause of syncope usually ommended in patients suspected of" having structural heart
have had only one or two episodes, with less than 5 seconds disease. If an arrhythmia is suspected, documentation of die
of warning symptoms before each episode. Patients often arrhythmia is indicated either by inpatient telemetry or ambu
have underlying structural heart disease as a contributing latory monitoring (sec MKSAP 16 Cardiovascular Medicine).
cause. A prolonged QT interval may result in arrhythmia No specific routine laboratory studies are recommended,
causing syncope and can be drug-related (see MKSAP 16 although women of reproductive age warrant a pregnancy
Cardiovascular Medicine). Clues to arrhythmia include brief" test. Tests with the lowest likelihood of"affecting diagnosis or
or absent prodrome, palpitations immediately preceding the management include head CT scan, carotid Doppler ultra
episode, and syncope occurring in the supine position. (An sonography, electroencephalography, and measurement of
exception is ventricular tachycardia, which usually has a warn cardiac enzyme levels; these studies may be indicated if"symp
ing prodrome of more than 5 seconds and associated toms point to specific etiologies but otherwise should be
diaphoresis). omitted from the work-up. Recommendations to not rou
Other cardiac causes resulting in inadequate cardiac tinely perform cardiac enzyme testing are supported by evi
output and subsequent cerebral hypoperfusion include dence that patients with cardiac ischemia almost always pre
valvular heart disease, cardiac tumors, pericardial disease, sent with chest pain or ECG changes. Neurologic studies

46
Common Symptoms

have only been shown to be useful in patients with new neu signs, hemorrhage, suspected or known heart disease, and fre
CONT.
rologic findings on initial evaluation. Tilt-table testing should quent recurrent episodes.
be reserved for patients with suspected neurocardiogenic syn KEY POINTS
cope not confirmed by history and physical examination, for
those widi recurrent syncopal episodes, and for patients sus Tests in the evaluation of" syncope with the lowest
likelihood of affecting diagnosis or management
pected of"having arrhydimogenic syncope or who have a high
risk profile for cardiovascular events in whom previous testing include head CT scan, carotid Doppler ultrasonogra
has not been revealing. phy, electroencephalography, and measurement of
cardiac enzyme levels; these studies should not bc
Risk Stratification and Management of performed in the absence of" symptoms pointing to
Syncope specific etiologies.
Several risk scores have been developed and validated in order Indications for hospitalization in patients with syn
to identify patients with syncope at high risk of adverse events. cope include chest pain, heart failure, syncope with
Causes of syncope vary according to the clinical setting in out warning signs, hemorrhage, suspected or known
which the evaluation was conducted (office versus emergency heart disease, and frequent recurrent episodes.

department), and a significant number of patients (40%-50%) High-risk patients with syncope requiring immedi
never present for evaluation; this number is higher in younger ate in-hospital telemetry are those with exertional
persons. Risk stratification dius is subject to patient popula or supine syncope, palpitations before the event, a
tion and setting. None of" the current risk assessment tools is family history of" sudden death, nonsustained ven
widely accepted in emergency practice. tricular tachycardia, or abnormal electrocardio
Neurocardiogenic and orthostatic syncope both are gen graphic findings.
erally benign in nature and do not require hospitalization. An
exception is an elderly patient in whom a secondary cause of
orthostasis is suspected or in whom recurrent episodes carry Chest Pain
risk for harm from trauma. For neurocardiogenic causes, Chest pain accounts for 2% of all outpatient visits to primary
management may consist of patient education with specific care physicians and 5.5 million visits to emergency depart
instructions on abortive and preventive strategies. These iso ments in the United States annually. It is the most common
metric counter-pressure maneuvers include leg crossing, symptom in persons 50 to 64 years of" age presenting for
hand-grip, squatting, and muscle tensing. (i-Blockers are no emergency care. Approximately 3 million patients are hos
longer indicated in vasovagal syncope. pitalized each year for further evaluation and treatment.
Ordiostatic hypotension is associated with a two-fold Although a few patients presenting with chest pain have
increase in mortality from underlying causes; these should be true medical emergencies or require inpatient care, prompt
treated appropriately. For benign orthostatic causes, treat and accurate diagnosis is nonetheless essential. The physi
ment consists of the maneuvers listed above, as well as posi cian must not only distinguish ischemic from nonischemic
tional changes, maintenance of adequate fluid intake, com chest pain but also distinguish emergent causes of" chest
pression stockings, and possibly niidodrine or fludrocortisone pain from nonemergent ones. Misdiagnosis can be cata
if indicated (frequent episodes with subsequent risk of strophic for the patient and a source of a malpractice claim
trauma; syncope during high-risk or competitive activities, against the physician.
such as in pilots or adiletes). These patients have an excellent The most likely cause of chest pain varies according to ^H
prognosis and no increase in mortality. Patients with carotid patient characteristics and the presentation setting. In the out- UJ
sinus hypersensitivity benefit from insertion of a dual-cham patient setting, musculoskeletal causes are die most common,
ber permanent pacemaker. accounting for approximately 40% of cases, followed by gas
Cardiac causes of syncope carry a high mortality in all age trointestinal disease (19%). In die emergency department,
groups; 6-month mortality is greater than 10% and is related acute coronary syndromes are more common, accounting for
to the underlying cause, not die syncope itself. Although up to 13% of" patient visits.
younger patients are more likely to have benign causes of syn
cope, patients with suspected cardiac causes warrant further Differential Diagnosis
inpatient evaluation regardless of" age. High-risk patients The differential diagnosis of chest pain largely consists of
requiring immediate in-hospital telemetry are those with cardiac, pulmonary, gastrointestinal, and musculoskeletal
exertional or supine syncope, palpitations before die event, a disease. Dermatologic disease (herpes zoster affecting tho
family history of" sudden death, nonsustained ventricular racic dermatomes) and psychiatric conditions (anxiety,
tachycardia, and abnormal ECG findings (conduction abnor panic attack) also can result in chest pain. Whereas most
malities, bradycardia). Other indications for hospitalization causes of chest pain can be evaluated and treated in a nonur
include chest pain, heart failure, syncope without warning gent fashion, others, such as acute coronary syndromes,

47
Common Symptoms

aortic dissection, tension pneumothorax, and pulmonary pleuritic pain (LR+ 0.2); pain described as either positional,
CONT.
embolism, require rapid triage for definitive diagnosis and sharp, or reproducible with palpation (LR+ 0.3); and pain
intervention. diat is eidier intramammary in location or not associated with
exertion (LR+ 0.8). In one study of 48 patients, no patient
Cardiac Causes of Chest Pain widi sharp or stabbing pain that was positional, pleuritic, or
Cardiac causes of chest pain include ischemia (acute coronary reproducible with palpation and who had no history of"angina
syndromes, stable angina), aortic dissection, myocarditis, or myocardial infarction was diagnosed with acute myocardial
pericarditis, and aortic stenosis. A standard diagnostic infarction at discharge. There is no predictable association
approach is to distinguish ischemic chest pain from nonis between acute myocardial infarction and relief" of chest pain
chemic causes as the first step. Characterization of the chest with nitroglycerin. A high index of"suspicion is necessary in
pain is helpful, as pain descriptors provided by the patient can women, the elderly, and patients widi diabetes, as they may
increase or decrease the likelihood of ischemia. (Assessment not present with classic symptoms.
of pretest probability of ischemic heart disease based on age, Patients presenting with acute chest pain can bc strati
sex, and description of the pain is discussed in MKSAP 16 fied into high, intermediate, or low risk of" having an acute
Cardiovascular Medicine.) In a meta-analysis describing the coronarv' syndrome based on information easily available in
value and limitations of" die chest pain history in the evalua the office setting, including pain descriptors, history of heart
tion of patients widi suspected acute myocardial infarction, disease, cardiac risk factors, and 12-lead ECG (Table 21).
radiation to die right arm or shoulder and radiation to both The ECG can be diagnostic (new ST-segment elevation or
arms or shoulders had die highest positive likelihood ratios depression, T-wave inversion in multiple leads). In some
(LR+) of 4.7 and 4.1, respectively. Odier pain descriptors diat patients, however, the ECG is normal or near-normal even
increase die likelihood of acute myocardial infarction are pain in the presence of"acute ischemia, and serial evaluations may
associated widi exertion (LR+ 2.4), radiation to the left arm be necessary.
(LR+ 2.3), diaphoresis (LR+ 2.0), nausea or vomiting (LR+ Several more detailed prediction tools and algorithms
1.9), and pain diat is pressure-like (LR+ 1.3). Descriptors diat have been published in an attempt to guide ftirdier evaluation
decrease the likelihood of acute myocardial infarction include and therapy and minimize unnecessary testing. These tools,

TABLE 21. Likelihood that Signs and Symptoms Represent an Acute Coronary Syndrome Secondary to CAD

High Likelihood Intermediate Likelihood Low Likelihood


Feature Any of the following: Absence of high-likelihood Absence of high- or intermediate-
features and presence of likelihood features but may have:
any of the following:

History Chest or left arm pain or Chest or left arm pain or Probable ischemic symptoms in absence of
discomfort as chief symptom discomfort as chief symptom any of the intermediate-likelihood characteristics
reproducing prior
documented angina Age >70 y Recent cocaine use
Male sex
Known history of CAD,
including Ml Diabetes mellitus
Physical Transient MR murmur, Extracardiac vascular disease Chest discomfort reproduced by palpation
examination hypotension, diaphoresis,
pulmonary edema, or crackles
ECG New, or presumably new, Fixed Q waves T-wave flattening or inversion less than 1 mm in
t r a n s i e n t S T- s e g m e n t d e v i a t i o n _ _ . , _ _ . leads with dominant R waves
n
(1 mmmm or nr greater)
nrMt*r\ or
orT-wave
Tumv. ST-segment depression 0.5 to
1 mm or T-wave inversion Normal ECG
inversion in multiple precordial
leads greater than 1 mm

Cardiac markers Elevated serum cardiac Normal Normal


troponin I, troponin T, or
creatine kinase MB

CAD = coronary artery disease; ECG = electrocardiogram; Ml = myocardial infarction; MR = mitral regurgitation.

Reprinted with permission from Anderson JL, Adams CD, Amman EM, et al; American College of Cardiology; American Heart Association Task Force on Practice Guidelines (Writing
Committee to Revise the 2002 Guidelines for the Management of Patients With Unstable Angina/Non ST-Elevation Myocardial Infarction); American College of Emergency Physicians; Society
for Cardiovascular Angiography and Interventions; Society of Thoracic Surgeons; American Association of Cardiovascular and Pulmonary Rehabilitation; Society for Academic Emergency
Medicine. ACC/AHA 2007 guidelines for the management of patients with unstable angina/non ST-elevation myocardial infarction: a report of the American College of Cardiology/American
Heart Association Task Force on Practice Guidelines (Writing Committee to Revise the 2002 Guidelines for the Management of Patients With Unstable Angina/Non ST-Elevation Myocardial
Infarction): developed in collaboration with the American College of Emergency Physicians, the Society for Cardiovascular Angiography and Interventions, and the Society of Thoracic
Surgeons: endorsed by the American Association of Cardiovascular and Pulmonary Rehabilitation and the Society for Academic Emergency Medicine. Circulation. 2007;! 16(7):el64. [Erratum
in: Circulation. 2008;117(9):e180|. [PMID: 17679616] Copyright 2007, Lippincott Williams & Wilkins.

48
Common Symptoms

utilizing history, physical examination, ECG, and cardiac bio arteries. On physical examination, jugular venous pressure is
markers (creatine kinase MB, troponin), are predominantly often elevated, there may be a parasternal heave, the S, heart
CONT.
used in emergency departments. sound is widely split, and the P, (pulmonic valve) compo
Aortic dissection is characterized by sudden and severe nent of S, is loud.
pain at onset, often with a tearing, splitting, or ripping qual
ity that radiates to the back. Asymmetric intensity of periph Gastrointestinal Causes of Chest Pain
eral pulses (pulse deficit) is a strong predictor (LR+ 5.7), and The most common gastrointestinal cause of chest pain is
the chest radiograph may demonstrate a widened medi GERD. Although often described as burning, it can mimic
astinum. These classic features are often absent, however, and angina and be relieved by nitroglycerin. It generally is wors
an acute aortic condition must be considered in any patient ened with bending over or recumbency and relieved with
with severe thoracic pain. Pericarditis and myocarditis usually antacids, H, blockers, or proton pump inhibitors. Other, less
present with pleuritic chest pain. The pain in pericarditis is common, gastrointestinal causes of"chest pain include chole
unaffected by exercise, worsens with recumbency, and is cystitis, cholangitis, esophageal spasm, peptic ulcer disease,
relieved with sitting forward. A pericardial rub may be pre and pancreatitis.
sent. Classic ECG findings are diffuse ST-segment elevation
and PR-segment depression. Aortic stenosis may cause exer Musculoskeletal Causes of Chest Pain
tional chest pain, which may be accompanied by syncope, dys Musculoskeletal chest pain is common, especially in women.
pnea, and other signs of heart failure. Etiologies include costochondritis, osteoarthritis, and muscle
strain. The pain is sharp, occasionally pleuritic, worsened or
Pulmonary Causes of Chest Pain reproduced with movement or palpation, and relieved with
Pulmonary causes of chest pain include pulmonary embolism, NSAIDs. It may last for weeks. Pain that worsens with move
pneumothorax, pleuritis, pneumonia, pulmonary hyperten ment can raise concern for ischemia, and a careful history is
sion, and other parenchymal lesions. In general, the pain with essential. Imaging is usually not indicated or helpful.
pulmonary embolism, pneumothorax, pleuritis, and pneu
monia is pleuritic in nature and accompanied by dyspnea. Chest Pain and Decision to Hospitalize 1X1
With pulmonary embolism, pain may be accompanied The decision to hospitalize a patient with chest pain is dial- "
by cough, wheezing, hemoptysis, tachypnea, and tachycar lenging. The goal is to identify patients with life-threatening
dia. Risk factors include immobilization, recent surgery, disease who require immediate attention while minimizing
paresis, history of'deep venous thrombosis, malignancy, obe unnecessary work-up and treatment in others. A rapid clini
sity, tobacco use, and estrogen therapy. The diagnosis is cal determination of the likelihood of an acute coronary syn
made by ventilation-perfusion scan, chest CT angiography, drome (see Table 21) is the essential first task for the physi
or pulmonary artery angiography (see MKSAP 16 cian and should guide the decision. In addition, the physician
Pulmonary and Critical Care Medicine). In low-probability should consider the likelihood of short-term adverse out
scenarios, a negative D-dimer test can be helpful in ruling comes (deadi, nonfatal myocardial infarction) in patients with
out thromboembolism. an acute coronary syndrome. Patients suspected of"having an
Pneumothorax can be primary (spontaneous) or sec acute coronary syndrome should be admitted. Low-risk
ondary. Rupture of a subpleural bleb underlies a spontaneous patients can be further stratified with stress testing.
pneumothorax, and lifetime recurrence rates are up to 50%. The use of formal risk scores (TIMI, GRACE, PUR
Secondary pneumothorax occurs in patients with underlying SUIT) can be useful in further risk stratification and manage
pulmonary disease such as COPD, q'stic fibrosis, or tubercu ment. While some centers utilize chest pain evaluation units
losis. Physical examination demonstrates decreased chest (CPEUs) to streamline evaluation and limit unnecessary
expansion, decreased breath sounds, and hyperresonance to coronary care unit admissions, die true cost-effectiveness of
percussion, all on die affected side. Diagnosis is made by chest these units is unknown.
radiograph. Tension pneumothorax, heralded by hypotension Despite a comprehensive history, physical examination,
and tracheal deviation to the unaffected side, is a medical and ECG, approximately 2% of patients with an acute coro
emergency. nary syndrome are missed in the emergency department.
Pulmonary hypertension includes pulmonary arterial Women younger than 55 years old and those who are non-
hypertension and pulmonary hypertension secondary to white, those who report dyspnea as a chief symptom, or
other conditions, including heart, lung, and thromboem those who have normal ECGs are more likely to be dis
bolic disease. The chest pain associated with pulmonary charged. In one study, the risk-adjusted mortality ratio for
hypertension is not pleuritic but usually is accompanied by diose not hospitalized compared with diose who were was
exertional dyspnea and fatigue. In most patients, die pain is 1.9 (95% CI, 0.7-5.2).
due to ischemia from either subendocardial hypoperfusion Patients with pneumothorax, pulmonary embolism,
or compression of coronary arteries by enlarged pulmonary and pericarditis are generally hospitalized for further care.

49
Musculoskeletal Pain

Aortic dissection and tension pneumothorax always require KEY POINT


admission. Q
CONT. In patients with chronic venous insufficiency or lym
KEY POINTS phedema, diuretic therapy should be avoided because
Characteristics of chest pain that increase the likeli fluid mobilization from die interstitial to the vascular
hood of ischemia include radiation of pain to either space does not predictably occur.
or bodi arms or shoulders; pain associated with exer
tion, diaphoresis, or nausea or vomiting; and pain
that is pressure-like.
Characteristics of chest pain with a lower likelihood Musculoskeletal Pain
of ischemia include pleuritic pain; pain that is posi Acute Low Back Pain
tional, sharp, or reproducible with palpation; pain
that is not associated with exertion; and pain diat is Diagnosis and Evaluation
inframammary in location. The prognosis for acute musculoskeletal low back pain is
There is no predictable association between acute excellent. More dian 70% of patients presenting to primary
myocardial infarction and relief of chest pain widi care clinics recover completely, and fewer than 10% seek
nitroglycerin. ongoing care after 3 months. Because of this, initial diagnos
An acute aortic condition must be considered in any tic evaluation of acute low back pain should focus on identi
patient with severe thoracic pain. fying patients at risk for nonmusculoskeletal causes, such as
malignancy, infection, or underlying systemic illness.
Particular attention should be paid to diagnosis of conditions
Edema that require urgent evaluations (tumor, infection, rapidly pro
Excess fluid accumulation in die interstitial space leads to gressive neurologic symptoms, cauda equina syndrome) and
those for which specific treatments may be more effective
edema. Such fluid accumulation results from increased capil
than symptomatic therapy (rheumatologic diseases, compres
lary hydrostatic pressure, increased capillary permeability, or
sion fractures).
decreased plasma oncotic pressure, as well as from retention
of sodium and water by die kidneys. Lymphedema, a type of Psychosocial factors are more useful dian the physical
examination in predicting the course of recovery in acute back
nonpitting edema due to extravasation of high-protein con
tent lymphatic fluid, is caused by congenital lymphatic disease pain. Presence of depression, passive coping strategies, job
or damage to die lymphatic vessels from surgery, radiation, dissatisfaction, higher disability levels, disputed compensation
obstruction, or recurrent cellulitis. claims, and somatization all may predict poorer outcomes. In
The differential diagnosis of the patient with edema is patients with risk factors for ongoing pain, quantitative assess
broad but can be narrowed by history and physical exami ment of"the patient's pain and function can be valuable.
nation. Causes of bilateral edema include heart failure,
nephrotic syndrome, cirrhosis, hypoproteincmia, constric History
tive pericarditis, chronic venous insufficiency, lymphedema, Age is the most useful screening factor in low back pain.
and medications (including minoxidil, nifedipine, amlodip- Cancer, compression fractures, and spinal stenosis are all sig
ine, thiazolidinediones, NSAIDs, and fludrocortisone). If nificantly more likely in older patients. Age older dian 50
central venous pressure is elevated, cardiac disease or pul years has a 75% to 90% sensitivity and 60% to 70% specificity
monary hypertension is likely; if normal, other causes should for these three diagnoses. Spondyloarthropathies almost
be considered. The most common causes of unilateral leg always present before the age of 40 years, but diis finding is
edema are deep venous thrombosis, cellulitis, and malignant very nonspecific (sensitivity 100%, specificity 7%). Other
lymphedema. important factors in the history are shown in Table 22.
The management of" edema involves identifying and Neurologic history and physical examination should
reversing die underlying disorder. Diuretics should be used focus on differentiating disk herniation and accompanying
carefully, especially in patients whose sodium retention is sec sciatica (unilateral sharp or burning pain radiating down the
ondary to a reduction in cardiac output. In diese patients, back or side of the leg, usually to the foot or ankle) from
diuretics may ftirdier reduce intravascular volume. In patients lesions diat compress the central spinal cord (cauda equina
with chronic venous insufficient (stasis edema) or lym syndrome). Key findings associated widi central spinal cord
phedema, diuretic dierapy should be avoided because fluid compression include bowel and bladder dysfunction (partic
mobilization from the interstitial to the vascular space does ularly urinary retention or incontinence), diminished sensa
not predictably occur. Sodium restriction, leg elevation, and tion over the perineum, bilateral or multilevel neurologic
compressive stockings can be helpful. deficits, and rapidly progressive neurologic deficits.

50
Musculoskeletal Pain

table 22. History Features and Suggested Diagnoses in in the straight-leg raise tests; what matters is that the pain
Low Back Pain radiates below the knee.
Lumbar spinal stenosis is more common in patients older
Suggested Diagnosis History Feature
than 65 years and is anecdotally associated with increased pain
Cancer Unexplained weight loss when walking and relief when sitting, often called neurogenic
Failure to improve after 1 claudication or pseudoclaudication. Evidence regarding the
month
diagnostic value of neurogenic claudication is limited, how
No relief with bed rest ever, and the available data suggest a poor positive likelihood
Infection Fever ratio. An abnormal Romberg test or a wide-based gait in the
Injection drug abuse presence of pain suggesting spinal stenosis are specific but not
sensitive findings.
Urinary tract infection
Skin infection
Further Diagnostic Testing
Inflammatory/rheumatologic Presence of morning
stiffness Patients widi nonspecific low back pain and no symptoms or Hrt
condition
Pain not relieved when supine signs to suggest systemic illness should not routinely receive UJ
additional diagnostic testing. The American College of
Pain persisting for >3 months
Physicians recommends that diagnostic imaging for low back
Gradual onset
pain be obtained in patients acutely only if they have evidence
Involvement of other joints of a severe progressive neurologic deficit or signs or symp
Nerve root irritation Sciatica toms suggestive of"a serious or specific underlying condition.
(radiculopathy) Increased pain with cough, There is no evidence to suggest an optimal imaging strategy
sneeze, or Valsalva maneuver for such patients if pain persists beyond 1 to 2 months,
Spinal stenosis Severe leg pain although plain radiography may be reasonable in this cir
No pain when seated cumstance. For patients with a history diat suggests malig

Pseudoclaudication nancy or fracture, plain radiography is recommended for ini


tial imaging. Complete blood count and erythrocyte
Compression fracture Trauma
sedimentation rate can also be helpful in evaluating patients
Corticosteroid use for infection or evidence of systemic illness. In patients with
Osteoporosis rapidly progressive neurologic symptoms (but not stable mild
Cauda equina syndrome Bowel or bladder dysfunction neurologic symptoms), cauda equina syndrome, or suspicion
Saddle sensory loss for epidural abscess or osteomyelitis, MRI is die preferred

Rapidly progressive modality because of better visualization of soft tissues and die
neurologic deficits spinal canal. If MRI is not feasible, CT myelography is a viable
alternative. Noncontrast CT is useful for detecting sacroiliitis
and is a reasonable diagnostic study in patients in whom an
inflammatory spondyloardiropadiy is suspected.
Patients whose symptoms specifically suggest radicu
Physical Examination lopathy or spinal stenosis should not receive additional imag
Clinicians should inspect for scoliosis and kyphosis, noting ing unless they have an inadequate response to noninvasive
any erythema or edema that suggests underlying infection or management and are considering surgery or epidural corti
inflammation. Skin findings (for example, psoriasis) that may costeroid injection. For patients who are considering an inva
suggest an underlying inflammatory arthritis should also be sive intervention, MRI is usually die initial study of choice.
noted. Palpation and percussion of the back can help differ Asymptomatic herniated disks are common findings on MRI,
entiate vertebral tenderness from pain originating in the so disk herniations that do not cause nerve root impingement
paraspinous soft tissues. The presence of vertebral tender or correlate anatomically with the patient's symptoms should
ness should increase suspicion of" compression fracture or be approached with suspicion.
infection. The straight-leg raise test (reproduction of pain
extending below the knee with 10 degrees to 60 degrees of" Tr e a t m e n t
leg elevation), weakness and diminished reflexes at the Because the overall prognosis for acute musculoskeletal low
ankles, and sensory loss in the feet are all associated with disk back pain is excellent, therapeutic interventions should focus
herniation. The crossed straight-leg raise test (lifting the on mitigating symptoms and maintaining function while the
unaffected side causing pain in the opposite leg) is less sen patient recovers. Most patients vvidiout sciatica show sub
sitive but more specific than performing the straight-leg raise stantial improvement within 2 weeks. Patients with sciatica
test on the affected side. The quality of" pain is not important may be slower to improve, but diree quarters of" patients are

51
Musculoskeletal Pain

substantially better after 3 months. Pain from spinal stenosis benefit in pain relief in acute low back pain. All can cause
is less likely to improve, although even with this condition, dizziness and sedation, however, and should be used widi
some fluctuation is common. caution in older patients.

Nonpharmacologic Treatment Interventional and Surgical Treatment


Patients should be encouraged to maintain their daily activi Many injection therapies have been tried in the management
ties as best they can. Bed rest has been shown in several trials of low back pain. Epidural corticosteroid injections may pro
to decrease functional recovery and increase pain in patients vide short-term relief in patients with disk herniations caus
bodi with and without sciatica. Spinal manipulation therapy ing radiculopathy, but evidence is mixed and long-term
has been shown to be associated with modest benefits in treat outcomes are unchanged. Facet joint injections, trigger
ment of acute low back pain, comparable to conventional point injections, and prolotherapy lack compelling support
therapy. Most randomized trials of" manipulation involved 2 ive evidence; American Pain Society guidelines recommend
to 3 weeks of dierapy; there is no evidence to suggest diat against their use.
long-term manipulation is any more helpful. Massage and Surgery has clearly shown benefit only in patients with
yoga have been shown to be helpful in small studies. The disk herniation causing persistent radiculopathy, patients
improvement seen with acupuncture in chronic back pain is with painful spinal stenosis, and patients with cauda equina
not superior to sham acupuncture. Supervised exercise dier syndrome. For patients with radiculopathy, diskectomy and
apy and physical therapy have not been shown to be effective microdiskectoniy show improved outcomes at 6 to 12 weeks
early in the course of low back pain, but are helpful in patients compared with nonsurgical therapy, although both surgical
whose pain persists for more than 4 weeks and to prevent and nonsurgical groups tend to continue to improve with
recurrences. time, and differences in outcomes diminish over 1 to 2 years.
In spinal stenosis, decompressive laminectomy shows mod
Pharmacologic Treatment erate benefits compared with nonsurgical therapy for the
First-line pharmacotherapy for most patients widi acute low first 1 to 2 years, again with diminishing effects in long-term
back pain includes acetaminophen and NSAIDs. NSAIDs follow-up.
should be used with caution in patients at increased risk for Cauda equina syndrome is considered an emergency F^t
nephrotoxicity (older patients and patients with diabetes mel because of die risk of rapid, irreversible loss of neurologic *'
litus, heart failure, or preexisting kidney disease) or gastroin function and because of its association with urgent underly
testinal ulcer. Concurrent treatment with a proton pump ing conditions, such as neoplasia or infection. Management
inhibitor or misoprostol or use of'a cyclooxegenase-2 (COX- typically involves prompt surgical decompression of" the
affected area of the spinal cord.
2) selective NSAID reduces risk of ulcers in patients at high
risk and is specifically recommended for patients older dian For patients with neither radiculopathy nor spinal
75 years. Some patients may also be at increased risk of hyper stenosis, the value of surgery is questionable. Available evi
tension and cardiovascular events when using NSAIDs, dence is of moderate quality and suggests that outcomes
from surgery are similar to those of" nonsurgical therapy.
aldiough different drugs have shown these effects to varying
When benefit has been shown in clinical trials, it is limited
degrees in observational studies. When NSAIDs are used, the
lowest effective dose should be used for the shortest possible to a narrow group of patients with more than 1 year ofmod-

period, especially in older patients and those with comorbidi erately severe pain and disability and without serious med
ties that put them at risk for complications. Acetaminophen ical or psychiatric comorbidities.
has fewer adverse effects, but doses should be limited to a KEY POINTS
maximum of 4 g/d to reduce the risk of hepatotoxicity, Patients with nonspecific low back pain and no symp
aldiough even this dose may be linked to hepatotoxicity in toms or signs to suggest systemic illness should not
patients with moderate to high alcohol use. routinely receive additional diagnostic testing.
Opioid analgesics or tramadol may be helpful in patients Most patients with acute low back pain improve
with acute low back pain for whom NSAIDs or acetamino
within 3 months with conservative therapy.
phen do not provide adequate relief". Although reasonably
safe for short-term use in modest doses, opioids should be
used widi caution in elderly patients and patients with chronic
mechanical low back pain and used only with extreme caution
Neck Pain
or avoided altogether in patients with a history of addiction Diagnosis and Evaluation
or substance abuse. Neck pain typically arises from three broad categories of"
There are several drugs with diverse mechanisms of conditions: mechanical neck pain arising from muscles,
action that are collectively referred to as skeletal muscle relax joints, and associated tissues; neurogenic neck pain arising
ants. Evidence suggests these drugs may have some modest from a cervical nerve root to the spinal cord; and neck pain

52
Musculoskeletal Pain

associated with systemic disease. Initial approach to the his back pain, acetaminophen and NSAIDs are first-line pharma
tory and physical examination should aim to differentiate cotherapy for most patients widi acute neck pain. Opioids and
these categories. skeletal muscle relaxants should be reserved for patients with
Mechanical neck pain is typically an aching pain localized a poor response to acetaminophen and NSAIDs or who are
to the neck, although it may be referred up toward the head having difficulty sleeping (in which case sedation is a desirable
or down toward die shoulder girdle. The history may include effect). Patients with neurogenic neck pain may find addi
injury (for example, whiplash or acute strain) or unaccus tional benefit from medications targeting neuropathic pain
tomed activity, suggesting an overuse syndrome. Physical (see Common Symptoms, Chronic Noncancer Pain,
examination often reveals decreased range of motion, tender Management).
ness over soft tissues, and pain caused or exacerbated by flex Interventional therapy is not beneficial for patients with
ion or extension of" the neck. mechanical neck pain. Epidural corticosteroid injection may
Neurogenic pain is typically burning and often radiates to be beneficial in patients with stable radiculopathy diat fails to
the shoulder or down the arm. Patients may have der respond to conservative therapy. Surgery is clearly indicated
matomal numbness or muscle weakness in the distribution of in patients widi progressive neurologic symptoms that stem
a cervical nerve root. Involvement of multiple spinal levels, from a defined anatomic abnormality. The role of surgery in
spasticity, hvperreflexia, gait abnormality, or leg weakness all patients with chronic neck pain without progressive neuro
suggest central spinal cord compression. logic deficits is controversial; limited data suggest faster short-
Patients with underlying systemic illnesses frequently term pain relief but no long-term difference compared with
have symptoms suggesting the underlying problem. These conservative management.
may include fever, weight loss, joint pain or ardiritis else KEY POINTS
where, symptoms of" polymyalgia rheumatica (headache, Mechanical neck pain outside of" the setting of acute
visual changes, shoulder and hip girdle pain), or history of
trauma rarely requires imaging, although plain films
immunosuppression, cancer, or injection drug use. Anterior can be helpful in patients older than 50 years to
neck pain is an unusual presentation of cervical spine disease
exclude malignancy and to assess for osteoarthritic
and should lead the clinician to suspect other anatomic struc
changes.
tures as possible sources of pain. Physical examination should
Most patients with neck pain recover with conserva
be directed toward identifying underlying systemic illness
based on symptoms and risk factors. tive therapy.

Imaging studies should be directed based on suspicions


raised during history and physical examination. Mechanical
neck pain outside of" the setting of acute trauma rarely Shoulder Pain
requires imaging, although plain films can be helpful in Diagnosis and Evaluation
patients older dian 50 years to exclude malignancy and to Important historical features to elicit in die evaluation of"
assess for osteoarthritic changes. Patients widi weakness, shoulder pain include location, severity, chronicity, circum
hyporetlexia, or symptoms or signs of spinal cord involvement stances of onset, history of trauma, and associated symptoms.
should be evaluated with MRI or CT myelography. Patients Shoulder pain may arise from structures of" the shoulder
with symptoms or signs of systemic illness and a suspected
(intrinsic disorders) or may be referred from other sites
anatomic abnormality in the neck (tumor, abscess, pathologic (extrinsic disorders). Pain widi movement of die shoulder
fracture, or disease of anterior neck structures) should have along with stiffness, locking, catching, and instability all sug
imaging directed at the suspected underlying cause. gest an intrinsic disorder, whereas a normal shoulder exami
Blood tests are not routinely needed for evaluation of" nation or the presence of constitutional symptoms and respi
patients with neck pain, but should be ordered as appropriate ratory or gastrointestinal symptoms suggests an extrinsic or
to exclude or confirm underlying systemic illness. Erythrocyte systemic etiology. Neck pain, decreased range of" motion of
sedimentation rate, C-reactive protein, and complete blood die neck, paresdiesias, and pain that radiates down the arm
count can be useful in evaluating for infection or inflamma past the elbow suggest cervical spine disease. Pain that wors
tory conditions. ens with intake of food raises die possibility of" pain referred
from a gallbladder disorder. Discomfort widi physical (non-
Treatment shoulder) exertion should raise concern for cardiac ischemia.
Most patients widi neck pain recover with conservative ther Other extrinsic causes of" shoulder pain include pneumonia,
apy. For patients with mechanical neck pain, mobilization and apical lung masses, and diaphragmatic irritation due to a vari
a gentle home exercise program have been shown to be ben ety of causes.
eficial. For patients with cervical radiculopathy, the role of Examination of" the shoulder should include inspection,
either hard or soft cervical collars is unclear, and acupuncture palpation, range-of-motion testing (both passive and active),
does not appear to offer clinically significant benefit. As in low- and provocative maneuvers. It is important to begin by fully

53
Musculoskeletal Pain

exposing both shoulders and examining for any asymmetry or


muscle wasting. Palpation should include the acromioclavic
ular (AC) joint, bicipital groove, all bony structures, and the
cervical spine. Pain that is only present with active but not
passive range-of"-motion testing suggests an extraarticular
condition; pain with both active and passive range-of-motion
testing suggests an intraarticular condition. Pain that occurs
between 60 and 120 degrees of abduction suggests a rotator
cuff impingement syndrome, whereas pain with more dian
120 degrees of"abduction favors AC joint pathology.
Many provocative tests are commonly performed in the
assessment of shoulder pain (Table 23). The Apley scratch
test can be used to assess range of motion. The Neer (Figure 7)
and Hawkins (Figure 8) tests are both useful for diagnosing
rotator cuff" impingement. The Yergason test (Figure 9) is
used to assess inflammation of" the long head of the biceps
and lesions of die glenoid labrum. The apprehension test
(Figure 10) assesses anterior glenohumeral instability.
FIGURE 7. Neer test. The examiner applies forced flexion to the affected arm
Rotator Cuff Disorders while the arm is fully pronated.This test is considered positive if pain is elicited
Rotator cuff tendinitis (inflammation of* the rotator cuff ten and suggests either rotator cuff tendinitis or subacromial impingement.
dons) can result from repetitive overhead motions, shoulder Maria Hartsock, CMI. Reprinted from Woodward TW. Best TW. The painful shoulder: part I.
Clinical evaluation. Am Fam Physician. 2000;61(10):3079-3088. [PMID: 10839557]
instability, or trauma. The supraspinatus tendon is most com
monly affected and is sometimes accompanied by tendinitis of"
die long head of"the biceps muscle.
Impingement syndrome results from compression of the symptoms are worse with abduction of" the arm, as the space
rotator cuff* between the inferior surface of the acromion and between die acromion and humeral head is smallest widi this
the superior surface of" the humeral head. Impingement movement. Impingement can be caused by spurs on die

T A B L E 2 3 . T e s t s U s e d i n S h o u l d e r E v a l u a t i o n a n d S i g n i fi c a n c e o f P o s i t i v e F i n d i n g j

Test Maneuver Diagnosis Suggested by Positive Result

Apley scratch test Patient touches superior and inferior aspects Loss of range of motion: rotator cuff problem
of opposite scapula
Neer test Forced flexion of pronated arm Subacromial impingement (89% sensitivity; 31 %
specificity)
Hawkins test Forward flexion of the shoulder to Supraspinatus tendon impingement (92%
90 degrees and internal rotation sensitivity; 25% specificity)
Drop-arm test Attempted slow lowering of arm to waist Rotator cuff tear
Cross-arm test Forward elevation to 90 degrees and active Acromioclavicular joint arthritis
adduction
Spurling test Spine extended with head rotated to Cervical nerve root disorder
affected shoulder while axially loaded

Apprehension test Anterior pressure on the humerus Anterior glenohumeral instability


with external rotation
Relocation test Posterior force on humerus while externally Anterior glenohumeral instability
rotating the arm
Sulcus sign Pulling downward on elbow or wrist Inferior glenohumeral instability
Yergason test Elbow flexed to 90 degrees with forearm pron ated Biceps tendon instability or tendinitis
Speed maneuver Elbow flexed 20 to 30 degrees and forearm su pinated Biceps tendon instability or tendinitis
"Clunk" sign Rotation of loaded shoulder from extension to forward Labral disorder
flexion

Reprinted with permission from The Painful Shoulder: Part 1. Clinical Evaluation., May 15, 2000, Vol 6 , No 10, Amer can Family Physician Copyright 2000 American Academy of Family
Physicians. All Rights Reserved.

54
Musculoskeletal Pain

FIGURE 10. Apprehension test. The patient is placed supine on a table.


FIGURE 8. Hawkins test for shoulder impingement. The patient holds the With the arm abducted at 90 degrees and the forearm flexed, the examiner
arm extended anteriorly at 90 degrees with the forearm bent to 90 degrees (at stands at the bedside facing the patient and places one hand under the
12 o'clock), as if holding a shield. The scapula should be stabilized by the affected shoulder. With the other hand, gentle pressure is placed on the fore
examiner. The arm is then internally rotated to cross in front of the body. A arm. Pain or apprehension constitutes a positive test.

positive test elicits pain in the shoulder.

abduction between 60 degrees and 120 degrees. Pain may


acromion, calcification of the coracoacromial ligament, or be elicited with deep palpation of" the lateral deltoid inferior
to the acromion process. Both the Neer and Hawkins tests
superior migration of die humeral head during abduction. If"
left untreated, impingement may lead to a full-thickness rota are sensitive but less specific for diagnosing impingement
tor cuff tear. syndrome.
Patients with either rotator cuff" tendinitis or impinge Weakness and loss of function should raise concern for a
ment have pain located over the lateral deltoid that is present rotator cuff tear. Examination findings include supraspinatus
with abduction and internal rotation of the arm. Nighttime weakness, weakness with external rotation, evidence of
symptoms are common. Examination reveals pain with impingement, and a positive drop-arm test. When the diag
nosis is not clear or if concern exists for a rotator cuff" tear,
imaging of die shoulder should be obtained. MRI is die pre
ferred imaging modality (>90% sensitivity), aldiough ultra
sonography, in experienced hands, is also an option.
Management of" rotator cuff" disease is multif'acetcd.
Initially, management includes rest, modification of" activi
ties, and NSAIDs to help reduce inflammation and provide
pain relief. Subacromial corticosteroid injection has been
shown to improve pain for up to 9 mondis. Physical therapy
focused on strengthening the rotator cuff muscles using
low-resistance exercises may bc beneficial in stabilizing the
head of the humerus and reducing impingement. Partial-
thickness tears can bc managed similar to tendinitis. Full-
thickness tears may require surgical intervention depending
on other patient characteristics (such as age, type of daily
activities, or profession).

Adhesive Capsulitis
Adhesive capsulitis (frozen shoulder) is caused by thickening
of the capsule surrounding the glenohumeral joint. It is asso
FIGURE 9. The Yergason test consists of resisted supination with the arm ciated with diabetes, Parkinson disease, stroke, previous
at the patient's side and the elbow at 90 degrees of flexion. The test is positive
if shoulder pain is elicited. trauma, and hypodiyroidism but may be idiopathic, particu
Maria Hartsock, CMI. Reprinted from Woodward TW, Best TW. The painful shoulder: part
larly in the elderly. It can also occur in die setting of a shoul
I. Clinical evaluation. Am Fam Physician. 2000;61(10):3079-3088. [PMID: 10839557] der or arm injury with concomitant restriction in movement

55
Musculoskeletal Pain

of" the shoulder by the patient. Pain is characteristically slow where the extensor muscles attach to the lateral epicondyle.
in onset and is located near the insertion of the deltoid mus Pain increases with forced extension of"the wrist.
cle. Patients often avoid lying on the affected side. On exam Medial epicondylitis (golfer's elbow) is characterized by
ination, there is loss of both active and passive range of" pain in the medial elbow and proximal forearm that occurs
motion. Radiographic findings are usually absent. with activities that require wrist flexion. On examination,
Initial treatment consists of" physical therapy and med there is tenderness to palpation from the medial epicondyle to
ication widi NSAIDs. Subacromial corticosteroid injection the pronator teres and flexor carpi radialis muscles. Pain
(and intra-articular corticosteroids for refractory cases) is increases with wrist flexion and resisted forearm supination.
most useful in patients with concurrent rotator cuff pathol Rest, avoidance of activities that worsen pain, stretching,
ogy. Recovery is typically prolonged (up to 2 years). and resistance exercises constitute initial therapy.
Consideration for surgical intervention is reserved for those Corticosteroid injection and oral NSAIDs may provide short-
in whom conservative therapv' has failed. term (but not long-term) relief of symptoms when other
measures fail. Surgery may be required for refractory cases.
Acromioclavicular Joint Degeneration
AC joint degeneration typically results from trauma (in Olecranon Bursitis
Olecranon bursitis, characterized by painful swelling of the
younger patients) or osteoarthritis (in older patients).
Bilateral involvement should raise concern for rheumatoid posterior elbow, can result from repetitive trauma, inflamma
arthritis. On examination, there is pain on palpation of the tion (gout, rheumatoid arthritis), or infection. On examina
AC joint. Palpable osteophytes may be present. Pain charac tion, there is localized swelling posterior to die olecranon
teristically occurs with shoulder adduction and abduction process without limitation of range of" motion of the elbow.
above 120 degrees. Plain radiography, if obtained, reveals When pain, inflammation, or fever is present, fluid should be

degenerative changes. Therapy consists of NSAIDs and pos aspirated for Gram stain, culture, and crystal analysis to rule
out septic bursitis and to evaluate for gout. Treatment options
sibly intra-articular corticosteroid injections. Exercises
focused on improving scapular retraction may be beneficial. include NSAIDs and corticosteroid injection (once infection
If these therapies fail, surgical referral is warranted. has been ruled out). Chronic effusions may require surgical
intervention.
KEY POINTS
Management of rotator cuff" disease includes rest, Ulnar Nerve Entrapment
activity modification, and anti-inflammatory ther Entrapment of the ulnar nerve at die elbow (cubital tunnel syn
apy; after resolution of" symptoms, an exercise pro drome) commonly manifests with pain in the elbow widi flex
gram to strengthen the rotator cuff" muscles should ion that radiates to the hand accompanied by paresthesias and
be initiated. sensory loss involving the fourth and tilth fingers (Figure 11).
Management of adhesive capsulitis consists of physical Weakness is a late finding. Treatment consists of splinting,
NSAIDs, and surgical decompression when severe.
therapy and oral anti-inflammatory agents; recovery is
typically prolonged.

Elbow Pain
Diagnosis and Evaluation
One should begin by obtaining a complete description of the
patient's pain. Examination of"the elbow includes inspection,
palpation, raiige-of-motion testing, and special tests (see
individual conditions below). The neck, shoulder, and wrist
should also be examined, as pain can be referred to the elbow
from each of"these sites.

Epicondylitis
Lateral epicondylitis (tennis elbow) is caused by overuse of
the wrist extensor muscles and is most commonly observed in
nonathletes, such as computer users who use a mouse. The
Ulnar Radial Ulnar
pain is located laterally, characteristically radiates down the
nerve nerve nerve
forearm to the dorsal hand, and worsens with activity and at
night. On examination, there is tenderness to palpation FIGURE 11. Innervation of the hand.

56
Musculoskeletal Pain

De Quervain tenosynovitis is caused by inflammation of"


Wrist and Hand Pain die abductor pollicis longus and extensor pollicis brevis ten
Indications for Imaging dons in the thumb. It is usually associated with repetitive use
Radiographs (posteroanterior, posteroanterior oblique, and of the thumb but can also be associated with odier conditions,
lateral views) should be obtained in all patients with wrist or including pregnancy, rheumatoid ardiritis, and calcium apatite
hand pain with both a history of" trauma and localized ten deposition disease. The typical presentation is of" pain on the
derness to palpation to exclude fracture. Radiographs are also radial aspect of the wrist that occurs when die thumb is used
helpful in patients with suspected osteoarthritis. to pinch or grasp. Examination findings include localized ten
derness over the distal portion of"die radial styloid process and
Carpal Tunnel Syndrome pain widi resisted thumb abduction and extension. Patients
also report pain with the Finkelstein test, in which die patient
Carpel tunnel syndrome is caused by compression of the
median nerve at the level of the wrist. Risk factors include is asked to make a fist over die fully flexed thumb and then to
female sex, obesity, pregnancy, diabetes, hypothyroidism, ulnar deviate the hand. This test is positive when pain is pre
connective tissue disorders, and rheumatoid arthritis. sent on the radial side of the thumb. Acute dierapy consists of
The most common clinical presentation is of pain and applying ice and taping or splinting die thumb to prevent
movement. Persistence of symptoms may require injection
paresthesias in the distribution of die median nerve (see
widi a corticosteroid and local anesdietic or use of a short-arm
Figure 11). Patients may also report paresthesias involving all
five fingers. Pain is usually worse at night and may radiate into cast with thumb spica. Surgery should be considered in
the proximal arm. If" left untreated, thenar muscle atrophy patients with persistent symptoms after a repeat injection.
Ganglion cysts, swellings that overlie either joints or ten
may occur. Symptoms may be unilateral or bilateral, with
dons typically on the dorsal surface, arise as a result of chronic
bilateral symptoms occurring in up to 65% of patients at the
time of* presentation. irritation of the wrist. If" the cyst is not painful, no interven
The most useful history and examination findings in tion is required. Treatment options for painful cysts include
surgical resection or injection with either a crystalline corti
diagnosing carpal tunnel syndrome are use of a hand symp
costeroid (preceded by aspiration of the contents) or with
tom diagram, thumb abduction and opposition weakness,
and hypalgesia in the territory of die median nerve. The hyaluronic acid (followed by aspiration).
Osteoarthritis of the hand is common, often involving the
Phalen maneuver and Tinel sign are frequently used but have
first carpometacarpal joint and die proximal and distal intcr-
poor test characteristics. Nerve conduction studies are widely
considered to be the diagnostic standard for carpal tunnel phalangeal joints. Osteoarthritis of"die wrist (radiocarpal arthri
tis) is uncommon and is almost always post-traumatic. Clinical
syndrome (sensitivity >85%, specificity >95%) and are helpful manifestations of osteoarthritis of the hands and wrists include
to obtain when the diagnosis is uncertain.
decreased range of motion, pain that worsens with activity and
First-line therapies include avoidance of repetitive
motions involving the wrist and hand and nocturnal splinting improves with rest, and swelling. Onset is usually insidious. On
examination, one may note bony enlargement of the involved
of the wrist at a neutral angle. Corticosteroid injection of" die
joints. A combination of pharmacologic and nonpharmacologic
carpal tunnel appears to provide short-term benefit in patients treatments, individualized to die patient, is recommended. It is
with mild to moderate disease. A 2-week course of oral cor
important to educate patients about joint protection and refer
ticosteroids appears to be effective on at least a short-term
to physical therapy or provide instructions on hand exercises.
basis. NSAIDs, diuretics, and pyridoxine have all been used,
Application of heat and ultrasound have been found to be help
but evidence of their efficacy is lacking.
ful. For mild to moderate pain and when only a few joints are
Surgery is indicated in patients with at least moderately affected, local or topical treatments (topical NSAIDs, capsaicin)
severe disease with persistent symptoms (6 or more months), are often more effective than systemic treatments.
severe motor impairment, and nerve conduction studies that
confirm the diagnosis. KEY POINTS

Radiographs should be obtained in all patients widi


Other Causes of Wrist and Hand Pain wrist or hand pain with bodi a history of trauma and
Fracture should be suspected when there is a history of direct localized tenderness to palpation.
trauma and localized tenderness to palpation on examination. First-line therapies for carpal tunnel syndrome
Distal radius and scaphoid fractures most commonly result include avoidance of" repetitive motions involving the
from a fall on an outstretched hand. \f clinical suspicion for a wrist and hand and nocturnal splinting of the wrist at
scaphoid fracture is high, treatment should not be delayed a neutral angle.
even if radiographs are normal, as lack of" treatment can lead If clinical suspicion for a scaphoid fracture is high,
to avascular necrosis. Hamate fractures can result from direct
treatment should not be delayed even if radiographs
trauma or from repetitive trauma, such as from swinging a are normal.
golf club or baseball bat.

57
Musculoskeletal Pain

Hip Pain common in the setting of*corticosteroid or excessive alcohol


use), fracture (suspect when trauma is present), septic arthri
Diagnosis and Evaluation tis, and acute synovitis. The examination findings for each of
Because hip pain can have nonmusculoskeletal causes, it is these conditions are similar to those seen with osteoarthritis.
essential to inquire about symptoms related to the gastroin An extra-articular cause of anterior hip pain should be sus
testinal, gynecologic, and genitourinary systems. On exami pected when hip range of motion is normal and when pain is
nation, one should observe gait, inspect both the affected and not elicited by movement of" the hip. Extra-articular causes
unaffected hips, palpate the sacroiliac joints and bony struc include inguinal hernia, lower abdominal pathology, and lum
tures of" the hip, including the greater trochanter, and assess bar disk disease at the Ll level.
range of motion. The FABER test (Figure 12) is used to Lateral hip pain is most often caused by trochanteric bur
assess Flexion, ABduction, and External Rotation of the hip.
sitis, usually resulting from gait abnormalities. Patients with
It is essential to also perform abdominal, genital, back, and trochanteric bursitis report pain when lying on the affected
knee examinations, as pain can be referred to the hip from side in the area of greater trochanter. On examination, there
each of these sites. Neurologic and vascular examination of is point tenderness to palpation approximately 2.5 cm (1 in)
the lower extremities also may help to reveal a cause.
posterior and superior to the greater trochanter. Treatment
Radiography should be obtained in all patients present consists ofcorrection of"die underlying etiology, heat, stretch
ing with acute hip pain to evaluate for fracture. MRI is help ing, and corticosteroid injection. Another cause of"lateral hip
ful to evaluate for fracture when radiography is negative but
pain is entrapment of the lateral femoral cutaneous nerve
clinical suspicion is high and also to evaluate for avascular
(meralgia paresthetica), frequently presenting as an oval-
necrosis, infection, and tumor, if concern exists.
shaped area of burning, numbness, or tingling on the distal
lateral thigh. Lumbar disk disease at the L4-L5 level may also
Specific Causes of Hip Pain
present with pain in the region of" the lateral hip and thigh.
The location of the pain helps to focus the broad differential Posterior hip pain can result from sacroiliitis, lumbosacral
diagnosis. Anterior (groin) pain is most often caused by disk disease, and, rarely, pathology affecting the hip joint.
osteoarthritis of*the hip. The pain is usually chronic, insidious Sacroiliac pathology can lead to pain referred to die gluteal
in onset, and worsens with activity. Early morning stiffness
region and is characterized by tenderness to palpation of" the
may also be present but usually improves with activity. sacroiliac joint. Spinal disease often causes pain accompanied by
Examination is usually remarkable for decreased and painful
paresthesias and back pain. Piriformis syndrome is caused by
internal rotation. Pain may also be produced by logrolling
compression of the sciatic nerve by the piriformis muscle, lead
(rocking thigh back and forth with both hands while the ing to pain in the buttocks and the distribution of* the sciatic
patient is supine). When anterior hip pain is acute in onset, nerve. Piriformis syndrome should be considered when sciatica
the differential diagnosis includes osteonecrosis (most is present without clear evidence of"lumbosacral disk disease.

In the evaluation of hip pain, it is essential to perform


abdominal, genital, back, and knee examinations, as
pain can be referred to the hip from each of these
sites.

Knee Pain
Diagnosis and Evaluation
Determining the chronicity of pain helps to focus the broad
differential diagnosis. The most common cause of" chronic
knee pain is osteoarthritis. The presence of multijoint involve
ment along with systemic symptoms raises concern for an
inflammatory condition, such as rheumatoid arthritis. The
differential diagnosis for acute knee pain includes trauma,
FIGURE 12. The FABER test assesses Flexion, ABduction, and External overuse, spontaneous meniscal tear, infection, crystalline dis
Rotation of the hip. With the leg in a figure-four position, the normal leg should
ease, and inflammatory arthritis. Septic arthritis must be
attain a parallel plane with the table. Gentle downward pressure on the knee in
treated without delay.
this position simultaneously places stress on the ipsilateral sacroiliac joint.
On examination, both knees should be fully exposed,
Reproduced with permission from Davis MF, Davis PF. Ross DS. ACP Expert Guide to Sports
Medicine. Philadelphia, PA: American College of Physicians. 2005:360. Copyright 2005. and the affected knee compared with the unaffected knee.
American College of Physicians.
Any asymmetry or swelling should be noted, as should any

58
Musculoskeletal Pain

erythema, ecchymoses, or muscle atrophy. The presence of of"the medial collateral ligament (Figure 14). The McMurray
swelling, especially if the pain is acute and the overlying skin test (see Figure 13) can be used to detect medial and lateral
is erythematous, should be considered an indication to per meniscal injuries, but its low sensitivity increases the likelihood
form arthrocentesis to evaluate for both septic arthritis and a of false-negative results. The medial-lateral grind test is consid
crystalline disorder. ered the most sensitive (69%) and specific (86%) test for assess
All knee structures should be palpated, and range of ing the intactness of the menisci. It is performed with die
motion should be evaluated. The integrity of"die anterior cru patient in the supine position. The examiner places the patient's
ciate ligament can be assessed by performing the anterior calf in one hand and places the thumb and index finger of* the
drawer and Lachman tests (Figure 13). The posterior drawer opposite hand over the joint line and then applies varus and val
test is used to assess the posterior cruciate ligament; with the gus stresses to the tibia during both extension and flexion. The
knee at 90 degrees of" flexion and the hip at 45 degrees of flex test is considered to be positive when the examiner detects a
ion, posterior force is applied to the proximal tibia and both the grinding sensation over the joint line with these maneuvers.
extent of" movement and the firmness of* the endpoint arc Radiographs should be obtained if there is a history of"
assessed. A varus stress test assesses the integrity of" the lateral trauma. MRI is useful if concern exists for ligamentous or
collateral ligament, and a valgus stress test assesses die integrity meniscal injury.

Degenerative Joint Disease


Osteoarthritis most commonly involves the medial compart
Anterior Drawer Tcit ment of the knee. Pain typically worsens with use and
improves with rest. Although morning stiffness may be pre
sent, it usually resolves with activity and there are no systemic
symptoms. Examination may reveal joint line tenderness,
decreased range of* motion, crepitus, effusion, and palpable-
bony changes. Radiographs may aid in determining severity
(see MKSAP 16 Rheumatology).

Trauma
Ligamentous injuries of the knee are common. Injury to the
anterior cruciate ligament frequently occurs when the patient
plants one foot and quickly turns in the opposite direction.
Often, a "pop" is heard, swelling quickly develops, and the
patient is not able to complete the activity that was being per
formed when the injury occurred. Medial collateral ligament
injuries occur when the knee experiences a valgus force, and
FIGURE 13. Tests for assessing integrity of knee ligaments and injury to lateral collateral ligament injuries occur when the knee expe
knee menisci. (Top left) Anterior drawer test (anterior cruciate ligament): Place
riences a varus force. Medial collateral ligament injuries are
patient supine, flex the hip to 45 degrees and the knee to 90 degrees. Sit on
the dorsum of the foot, wrap your hands around the hamstrings (ensuring that much more common than lateral collateral ligament injuries.
these muscles are relaxed), then pull and push the proximal part of the leg, Meniscal injuries occur with sudden twisting motions of the
testing the movement of the tibia on the femur. Do these maneuvers in three knee, although "spontaneous"* meniscal tears without evi
positions of tibial rotation: neutral, 30 degrees externally, and 30 degrees dence of significant trauma may occur, primarily in older
internally rotated. A normal test result is no more than 6 to 8 mm of laxity.
(Top right) Lachman test (anterior cruciate ligament): Place patient supine on patients owing to chronic degeneration of" the meniscus.
examining table, leg at the examiner's side, slightly externally rotated and Although painful, the patient is able to immediately bear
flexed (20-30 degrees). Stabilize the femur with one hand and apply pressure weight. Swelling may occur.
to the back of the knee with the other hand with the thumb of the hand exert
Appropriate treatment of knee trauma is highly depend
ing pressure placed on the joint line. A positive test result is movement of the ent on the nature and extent of" injury, die degree of dys
knee with a soft or mushy end point. (Bottom left) Pivot test (anterior cruciate
ligament): Fully extend the knee, rotate the foot internally. Apply a valgus
function, and the overall medical status of the patient. Many
stress while progressively flexing the knee, watching and feeling for transla traumatic knee injuries may be managed without surgical
tion of the tibia on the femur. (Bottom right) McMurray test (meniscus): Flex intervention; orthopedic input may be helpful in establishing
the hip and knee maximally. Apply a valgus (abduction) force to the knee while an optimal therapeutic approach.
externally rotating the foot and passively extending the knee. An audible or
palpable snap during extension suggests a tear of the medial meniscus. For the
lateral meniscus, apply a varus (adduction) stress during internal rotation of Patellofemoral Pain Syndrome
the foot and passive extension of the knee. Patellofemoral pain syndrome, an overuse syndrome, typically
Reproduced with permission from Jackson JL. O'Malley PG. Kroenke K Evaluation of acute manifests as anterior knee pain that is worse with prolonged
knee pain in primary care. Ann Intern Med. 2003;139(7):580. (PMID: 14S30229] Copyright
2003, American College of Physicians. sitting and with walking up and down stairs. It is commonly

59
Musculoskeletal Pain

Varus stress test


Valgus stress test

FIGURE 14. Varus and valgus forces applied to knee.

iSSSSS^J^vlmSam^^* M' ***** ' Pa,ienU ***** ^ knee Pain: Part L HiS'0rV' PhySiCa' Xamina"0n' ">(i'9'aPhs. and laboratory tests. Am Fam

seen in runners but also occurs in nonrunners, and it affects


crossing the legs may be helpful, as may the application of ice.
women more than men. On examination, the pain may be Oral NSAIDs have limited usefulness owing to the lack of"
reproduced by applying pressure to the surface of the patella penetration to the bursal space.
(patellofemoral compression test) with the knee in extension Prepatellar bursitis, inflammation of" the bursa lying
and moving the patella laterally or medially. Radiographs are between the patella and the overlying skin, frequently results
unnecessary as the diagnosis is clinical. from repeated trauma ("housemaid's knee") but can also be
Treatment can be divided into acute (first week) and caused by infection or gout. Patients with prepatellar bursitis
recovery phases. Treatment during the acute phase focuses on present with pain in the anterior aspect of the knee.
pain control and consists of*activity modification, short-term Erythema, swelling, and tenderness to palpation may be
use of NSAIDs, and other conservative measures, such as observed near the lower pole of the patella. Treatment con
icing. Treatment during the recovery phase should consist of" sists of" rest and avoidance of kneeling.
a rehabilitation program that focuses on strengthening the
hip abductor and quadriceps muscles and stretching, Iliotibial Band Syndrome
although there is no single accepted program. Iliotibial band syndrome presents as lateral knee pain diat is
worse widi walking up or down steps. It is caused by friction
Bursitis between the iliotibial band and the lateral femoral condyle and
Pes anserine bursitis manifests as pain located near the antero is frequently seen in runners and cyclists. Tenderness is present
medial aspect of" the proximal tibia and is most commonly at the lateral femoral epicondyle (approximately 3 cm [1.2 in]
caused by overuse or direct trauma. On examination, tender
proximal to die joint line). A Noble test is performed by having
ness is elicited at die level of the tibial tuberosity (approxi the patient, in the supine position, repeatedly flex and extend the
mately 4 cm [ 1.5 in] below the level of the medial joint line). knee with the clinician's thumb placed over the lateral femoral
Swelling may be present at the insertion of" the medial ham epicondyle. A positive Noble test reproduces the patient's pain.
string muscles. Treatment consists of avoiding squatting and Treatment is conservative and typically consists of* rest,
limiting repetitive bending and direct pressure on the bursa. stretching, NSAIDs, and addressing contributing factors
Placing a pillow between the legs at night and avoiding (such as running).

60
Musculoskeletal Pain

Baker Cyst Lateral view Medial view


Patients with a popliteal (Baker) cyst report the insidious Malleolar zone
A B
onset of posterior knee pain and may also note a sense of Poslorior edge L / Midfoot zone Posterior edge
or tip of. or tip of
posterior fullness. It can represent either a true cyst or mav laterial malleolus medial malleolus
be an extension of an intra-articular knee effusion. (6-cm length) (6-cm length)
Examination reveals palpable fullness in rhe posteromedial
knee. Treatment is usually aimed at the underlying condition
but may require aspiration and, in a small number of
patients, surgical resection.
KEY POINT FIGURE 15. Ottawa ankle and foot rules. An ankle radiographic series is
The medial-lateral grind test is the most sensitive and indicated if a patient has pain in the malleolar zone and any of these findings:
bone tenderness at A, bone tenderness at B, or inability to bear weight imme
specific test for assessing the intactness of* the menisci. diately and in the emergency department (or physician's office). A foot radi
ographic series is indicated if a patient has pain in the midfoot zone and any
of these findings: bone tenderness at C, bone tenderness at D, or inability to
Ankle and Foot Pain bear weight immediately and in the emergency department (or physician's
office).
Ankle Sprains Reproduced with permission from Davis MF, Davis PF, Ross DS. ACP Expert Guide to Sports
Medicine. Philadelphia. PA: American College of Physicians. 2005:404. Copyright 2005.
Ankle sprains usually involve a combination of plantar flexion American College of Physicians.
and inversion leading to damage to the lateral ligaments. The
anterior talofibular ligament is the most commonly injured.
Patients with ankle sprains initially present with pain, swelling, Early management of uncomplicated ankle sprains (grade
and diminished proprioception. Although typically a self-lim I) includes controlling swelling with ice, compression, and
ited condition, residual symptoms, including pain, instability, elevation. NSAIDs can be used to help with pain control and
and stiffness, are common and may persist for years. Ankle to reduce inflammation. Range-of-motion exercises should
sprains can be graded by severity: ligamentous stretching with be started promptly. Further rehabilitation, including propri
no instability and mild pain (grade I); partial ligamentous tear oceptive training, is essential to prevent chronic instability.
with moderate tenderness to palpation and pain with ambu Management of" grade II sprains is similar but may require
lation (grade II); and complete ligamentous tear with insta limiting wcightbearing in addition to stabilization during
bility, severe pain, and inability to ambulate (grade III). recovery. Grade III sprains may require surgical intervention
Evaluation should begin with a determination of the cir given the presence of" complete tearing and loss of ankle sta
cumstances under which the ankle was injured and whether bility, particularly with failure to respond to more conserva
the ankle was inverted or everted. The inability to bear weight tive therapies.
immediately after injury suggests a possible fracture or a more
serious sprain. A popping sound when the injury occurred Hind Foot Pain
suggests ligament rupture. Plantar fasciitis, the most common cause of heel pain, is char
Examination should include inspection, palpation, acterized by intense, sharp pain in the heel with the first steps
range-of-inotion testing, assessment of'weightbearing status, after long periods of inactivity (such as in the morning or after
and focused testing of"the ankle. Palpation should include the resting) and initially improves with walking but subsequently
bony structures of the ankle and the entire length of the tibia worsens. Risk factors include pes planus, running, occupa
and fibula. The Achilles tendon should also be palpated along tions requiring prolonged standing, and obesity. Examination
its entire course. Specific tests allow for assessment of the is notable for tenderness to palpation at the anteromedial
integrity of the structures of the ankle. Compression of" the aspect of the heel. Radiographs have limited utility but fre
fibula and tibia at the midcalf ("squeeze test") should be per quently reveal heel spurs and can help to rule out stress frac
formed, as pain with this maneuver in the area of the distal tures of* the calcaneus. Cornerstones of therapy include
tibia and fibular syndesmosis indicates a syndesmosis sprain weight loss when appropriate, exercises that stretch the plan
("high ankle sprain"). The talar tilt test, used to assess the cal- tar fascia and calf, appropriate footwear, activity modification,
caneofibular ligament, is performed by holding the distal end and analgesics. Orthotics or night splints may be helpful for
of"the tibia and fibula in one hand while using the other hand some patients. Corticosteroid injections appear to provide
to invert die ankle. It is considered positive if"the tilt is 5% to short-term improvement in symptoms, although long-term
10% greater on the affected side as compared with the unaf efficacy has not been established. Most cases (>80%) resolve
fected side. The Ottawa ankle and foot rules are a highly sen within a year of onset of" symptoms, regardless of therapy.
sitive tool to help the clinician determine when it is appropri Achilles tendinopathy is caused by recurrent micro
ate to obtain radiographic studies in patients presenting with trauma and can be either acute or chronic. Acute Achilles ten
a history of"trauma (Figure 15). don pain most commonly results from an abrupt increase in

61
Dyslipidemia

activity level whereas chronic pain (lasting >3 months) results there is tenderness to palpation of" the affected interdigital
from poor mechanics (pes planus, pes cavus, limb length dis space. Treatment consists of"avoiding high heels and wearing
crepancy) and improper footwear. Risk factors include male cushioning under the forefoot. NSAIDs are not usually help
sex, increasing age, history of fluoroquinolone or corticos ful, but corticosteroid injection is helpful in select patients.
teroid use, and obesity. Treatment includes avoidance of
KEY POINT
potentially aggravating activities, using a heel lift, and a 7- to
The Ottawa ankle and foot rules are highly sensitive
10-day course of" NSAIDs.
for ruling out fracture.
Rupture of the Achilles tendon commonly occurs when
the patient is pushing off" with the foot. Patients may hear a
popping sound and usually experience severe pain at the time
of" injury, although up to one-third of"patients do not experi
ence pain. On examination, clinicians can perform the Dyslipidemia
Thompson test (sensitivity, 96%; specificity, 93%) by squeez
ing die patient's calf" while the patient kneels with the feet Screening
The U.S. Preventive Services Task Force (USPSTF) strongly
hanging over the edge of" the examining table. Absence of
resultant plantar flexion supports the presence of" an Achilles recommends initiating screening in men at age 35 years, and
tendon rupture. Diagnostic ultrasound can bc obtained when in women who are at increased risk of cardiovascular disease
the diagnosis is in question. Partial Achilles tendon rupture at age 45 years; screening may also be considered in bodi men
can be managed similarly to chronic Achilles tendinopathy; and women beginning at age 20 years who have increased
cardiovascular risk. A repeat screening interval of 5 years is
surgical intervention is required for complete tendon rupture.
considered appropriate in low-risk individuals, widi a shorter
Midfoot Pain interval in those with borderline results, and a longer interval
Tarsal tunnel syndrome is due to compression of" the tibial in patients widi consistently normal testing. The National
nerve at die level of the ankle and is typically manifested as Cholesterol Education Program Adult Treatment Panel III
pain and paresdiesias in die midfoot. The most common (ATP III) guidelines recommend initiating screening for lipid
cause of" tarsal tunnel syndrome is from scar tissue or loose disorders at the age of 20 years for all adults, widi screening
at 5-year intervals for low-risk individuals. More frequent
bony or cartilaginous fragments from fracture of the calca
neus, medial malleolus, or talus. Symptoms also commonly measurements are required for persons widi multiple risk fac
occur in the heel and toes and are worse at night and widi tors or in those with zero or one cardiovascular risk factors
and an LDL cholesterol level only slightly lower dian die goal
prolonged standing. Examination findings can include the
level. The preferred screening test is a fasting lipoprotein pro
presence of a Tinel sign posterior to the medial malleolus (an
electric shot sensation upon percussion of the posterior tibial file including measurements of total cholesterol, HDL cho
nerve) and sensory loss on the plantar surface of" die foot. If lesterol, and triglycerides in order to calculate the LDL cho
severe, atrophy of"the intrinsic muscles of"die foot may result. lesterol level. (LDL cholesterol = total cholesterol - HDL
Treatment options include NSAIDS, orthotics, and modifi cholesterol - [triglycerides + 5]) The ATP III classification of
cations to footwear. lipid levels is shown in Table 24.
Although uncommon, stress fractures involving die KEY POINTS
bones of the midfoot can affect runners. The U.S. Preventive Services Task Force strongly rec
ommends initiating lipid screening in men at age 35
Forefoot Pain
years and in women who are at increased cardiovascu
In hallux valgus deformity (bunion), the first toe deviates lat
lar risk at age 45 years, and possibly earlier in both
erally, and a bony deformity develops on the medial aspect of men and women at increased risk for cardiovascular
the first metatarsophalangeal (MTP) joint. This is typically
disease.
accompanied by the inflammation of a bursa on the medial The National Cholesterol Education Program Adult
aspect of" the first MTP joint, especially with tight-fitting
shoes. Treatment is usually conservative and includes NSAIDs Treatment Panel III recommends initiating lipid
and accommodating footwear. When these interventions fail, screening in all adults at age 20 years.
orthotics, bracing, and surgical intervention may be required.
Morton neuroma, a common cause of" metatarsalgia,
results in burning pain in the space between the third and
Evaluation of Lipid Levels
fourth toes or, less commonly, between the second and third LDL Cholesterol
toes. It results from entrapment of one of the common digi LDL cholesterol comprises 60% to 70% of serum cholesterol
tal nerves. Pain is usually worse with standing and is most and is the most atherogenic of the lipoproteins. Cholesterol
common in women who wear high heels. On examination, treatment guidelines focus on reducing LDL cholesterol as

62
Dyslipidemia

1 TABLE 24. Adult Treatment Panel III Classification of | CHD, the risk for a cardiovascular event over die next 10
Lipid Levels (mg/dL) years may be estimated using die Framingham risk calculator
LDL CholesterolPrimary Target of Therapy (http://hp2010.nhlbihin.net/atpiii/calculator.asp?user
type=prof). Patients are dius stratified into three different car
<100 mg/dL (2.59 mmol/L) Optimal diac risk categories (<10%, 10%-20%, >20% risk over 10 years)
100-129 mg/dL (2.59-3.34 mmol/L) Near optimal/above and treatment goals vary accordingly. Certain medications
optimal and medical conditions can cause abnormal LDL cholesterol
130-159 mg/dL (3.36-4.11 mmol/L) Borderline high levels, including diabetes, hypothyroidism, and kidney dis
160-189 mg/dL (4.14-4.89 mmol/L) High ease. Suggestive history and physical examination findings
>190 mg/dL (4.92 mmol/L) Very high should prompt evaluation for these disorders before initiating
LDL cholesterol-lowering therapy.
Total Cholesterol

<200 mg/dL (5.18 mmol/L) Desirable Triglycerides


200-239 mg/dL (5.18-6.19 mmol/L) Borderline high Elevated triglyceride levels reflect the presence of atherogenic
>240 mg/dL (6.21 mmol/L) High VLDL remnants and are a marker for low HDL cholesterol
levels, increased insulin resistance, and other components of
HDL Cholesterol
the metabolic syndrome. It is controversial whether serum
<40 mg/dL (1.03 mmol/L) Low triglyceride levels themselves are an independent predictor of
>60 mg/dL (1.55 mmol/L) High cardiac risk. However, as triglyceride-rich VLDL remnants
have been associated with cardiovascular disease, die ATP III
Reprinted from ATP III Guidelines At-A-Glance Quick Desk Reference. National Heart.
Lung, and Blood Institute. NIH Publication 01-3305. May 2001. Accessed August 8, guidelines designate non-HDL cholesterol, which comprises
2011. at www.nhlbi.nih.gov/guidelines/cholesterol/atglance.pdf. LDL cholesterol together widi VLDL cholesterol, as a sec
ondary target for Iipid-lowering therapv' when triglyceride lev
the primary target of therapy. LDL cholesterol levels have els are greater than 200 mg/dL (2.26 mmol/L). The goal for
been linked to new and recurrent coronary heart disease non-HDL cholesterol (total cholesterol - HDL cholesterol)
(CHD) in numerous trials; higher levels of" LDL cholesterol is 30 mg/dL (0.34 mmol/L) higher than die goal for LDL
are associated with a greater cardiovascular risk, and a 1% cholesterol.
reduction in LDL cholesterol reduces risk by 1%. As outlined The American Heart Association (AHA) suggests that
in the ATP III guidelines, a patient's goal LDL cholesterol the "optimal" triglyceride level is less dian 100 mg/dL (1.13
level is determined by counting the number of cardiac risk mmol/L), but studies have not shown benefit for using med
factors and assessing the short-term risk for an acute coronary ication to reach this target. Because lifestyle factors, including
syndrome (Table 25). For patients vvidiout diabetes or physical inactivity and a high-carbohydrate diet, have been

table 25. Adult Treatment Panel III Recommendations for LDL Cholesterol Level Management
Risk Category LDL Cholesterol Goal Initiate TLC Consider Drug Therapy3

<100 mg/dL (2.59 mmol/L) >100 mg/dL (2.59 mmol/L) >130 mg/dL (3.37 mmol/L) 100
High risk: CHD or CHD
to 129 mg/dL (2.59 to 3.34
equivalentsb (10-year (optional goal: <70 mg/dL
risk >20%) [1.81 mmol/L])c mmol/L): drug optional
<130 mg/dL (3.37 mmol/L) >130 mg/dL (3.37 mmol/L) >130 mg/dL (3.37 mmol/L)
Moderately high risk: >2 risk
factors (10-year risk 10%
to 20%)
Moderate risk: >2 risk factors <130 mg/dL (3.37 mmol/L) >130 mg/dL (3.37 mmol/L) >160 mg/dL (4.14 mmol/L)
(10-year risk <10%)
Lower risk: 0 to 1 risk factor <160 mg/dL (4.14 mmol/L) >160 mg/dL (4.14 mmol/L) >190 mg/dL (4.92 mmol/L);
160 to 189 mg/dL (4.14 to
4.90 mmol/L): drug optional

CHD = coronary heart disease; TLC = therapeutic lifestyle changes

"When LDL cholesterol-lowering drug therapy is employed, intensity of therapy should be sufficient to achieve at least a 30% to 40% reduction in LDL cholesterol levels.

"CHD risk equivalents include peripheral arterial disease, abdominal aortic aneurysm, carotid artery disease, transient ischemic attacks or stroke of carotid origin or 50% obstruction of a
carotid artery, diabetes, and 10-year risk for CHD >20%.

CATP III Update 2004: Implications of recent clinical trials for the ATP III Guidelines. Available at www.nhlbi.nih.gov/guidelines/cholesterol/atp3upd04.htm.

alf a high-risk person has high triglyceride or low HDL cholesterol levels, combining a fibrate or nicotinic acid with an LDL cholesterol-lowering drug can be considered.

Data from the National Heart Lung and Blood Institute. National Cholesterol Education Program. Third Report of the Expert Panel on Detection, Evaluation, and Treatment of High Blood
Cholesterol in Adults (Adult Treatment Panel 111): Executive Summary, www.nhlbi.nih.gov/guidelines/cholesterol/atp_iii.htm. Published May 2001. Accessed July 14, 2009.

63
Dyslipidemia

linked to elevated triglyceride levels, weight loss, exercise, and combining these changes with exercise can result in an LDL
decreased consumption of carbohydrates and fructose are rec cholesterol reduction of up to 15%. Referral to a nutrition
ommended for patients with borderline (150-199 mg/dL professional is also helpful. If goal LDL cholesterol levels are
[1.69-2.24 mmol/L]) and high (200-499 mg/dL [2.26- not reached after 6 weeks, the addition of 2 g/d of dietary
5.63 mmol/L]) triglyceride levels. In accordance with ATP plant sterols and stanols, found in certain fortified margarines
III guidelines, pharmacotherapy is recommended if"the non- and spreads, can enhance LDL cholesterol lowering by
HDL cholesterol level is above goal, or if" triglyceride levels approximately 10%. Similarly, intake of viscous fiber, found in
are greater than 500 mg/dL (5.65 mmol/L), because of the oatmeal, fruits, legumes, and certain vegetables, results in a
increased risk for pancreatitis. Additionally, treating chronic modest LDL cholesterol reduction.
diseases (diabetes, obesity, kidney disease) or eliminating
medications diat are associated with high triglyceride levels Drug Therapy
(corticosteroids, protease inhibitors, f$-blockers, estrogens) Providers should consider initiating drug therapy in patients
may be beneficial. who have not achieved LDL cholesterol goals after 3 months
of therapeutic lifestyle changes, and simultaneously widi
HDL Cholesterol lifestyle changes in patients with CHD or CHD risk equiva
HDL cholesterol makes up about 20% to 30% of serum cho lents who are unlikely to achieve LDL cholesterol goals with
lesterol, and low levels have been linked to increased mortal dietary therapy alone. The ATP III guidelines for LDL cho
ity from cardiovascular disease; a 1% decrease in serum HDL lesterol goals and treatment strategies are listed in Table 25.
cholesterol level is associated widi a 2% to 3% increase in car Several medications are effective for reducing LDL choles
diovascular risk. Reflecting this inverse relationship, the pres terol (Table 26).
ence of a high HDL cholesterol level, defined as greater than Statins are considered first-line therapy, as they are the
60 mg/dL (1.55 mmol/L), subtracts one risk factor when most effective for LDL cholesterol lowering and also reduce
counting risk factors for LDL cholesterol treatment goals. the risk for CHD outcomes in both primary and secondary
Although HDL cholesterol level is the lipid risk factor most prevention. Additionally, statins have been shown to reduce
tightly correlated widi cardiovascular risk, this may be related markers ofinflammation, including high-sensitivity C-rcactive
in part to die close association among low HDL cholesterol,
protein levels, and this effect may contribute to their clinical
small, dense LDL particles, and high triglyceride levels. Thus, benefits. Several studies have shown that statins reduce total
the updated ATP III guidelines do not designate HDL cho
mortality by 20% to 30% in patients widi established CHD
lesterol level as a specific target for lipid-lovvering therapy.
disease, but a recent meta-analysis of statin treatment for pri
Causes of low HDL cholesterol level include elevated triglyc
mary prevention in high-risk patients (which included data
eride levels, obesity, physical inactivity, smoking, high carbo
from die JUPITER trial) failed to show a mortality benefit.
hydrate diets, type 2 diabetes, p-blockers, anabolic steroid The updated ATP III guidelines recommend a goal LDL
use, progestational medications, and genetic factors. cholesterol level of less than 100 mg/dL (2.59 mmol/L) in
high-risk patients and an optional goal of less dian 70 mg/dL
Nonstandard Lipid Risk Factors
(1.81 mmol/L) in very high-risk patients, such as those with
Several nontraditional risk factors may be related to cardio
established cardiovascular disease plus diabetes.
vascular outcomes, including levels of Lp(a) lipoprotein,
Physicians should consider potencies, drug interactions,
small LDL particles, HDL subspecies, apolipoproteins B and
and metabolism when choosing among die currently available
A-l, and the total cholesterol/HDL cholesterol ratio. statins. The benefits of statin therapy are generally class spe
However, die USPSTF and updated ATP III guidelines do cific, and there is no compelling evidence that newer agents
not recommend measuring or treating any of* these risk fac
are more effective than established statin medications, which
tors when managing lipid levels.
may be more cost effective. The ATP III guidelines recom
mend titrating the statin dose to reduce LDL cholesterol lev
els by 30% to 40%; each doubling of" the statin dose reduces
Management of Dyslipidemias LDL cholesterol level by a mean of approximately 6%.
Therapeutic Lifestyle Changes However, higher doses of statins are associated with an
Therapeutic lifestyle changes are the cornerstone of'lipid-low- increased risk for myopathy. The FDA has recendy recom
ering therapy, and the ATP III guidelines recommend them mended that an 80-mg dosage of simvastatin be prescribed
for all patients with abnormal lipid levels. Sattirated fat intake only to those who have been tolerating it well for 12 months
should be reduced to less than 7% of total calories (about 19 g or more. Other risk factors for statin-induced myopathy
of saturated fat for a 2500-calorie diet) and dietary cholesterol include increased age, multiple medical problems, female sex,
consumption decreased to less than 200 mg/d. and cotreatment with certain medications. Inhibitors of" the
Incorporating diese changes alone into a standard Western cytochrome P-450 3A4 (CYP3A4) isoenzyme, including
diet can reduce LDL cholesterol levels by 9% to 12%;
cyclosporin, amiodarone, fibrates, and protease inhibitors.

64
Dyslipidemia

table 26. Medications for Treating Abnormal Lipid Levels

Agent Changes in Lipid Values Evidence Notes

Statins LDL cholesterol i 18%-55% Reduced CHD in 1 and 2 Most effective agents for reducing LDL
HDL cholesterol t 5%-15% prevention. cholesterol. See Table 27 for adverse effects.
Higher doses increase risk for adverse
TGs 17%-30% Mortality benefit in 2 events. Contraindicated in pregnancy.
prevention.
Bile acid LDL cholesterol i 15%-30% Reduced CHD in 1 prevention. Avoid in patients with high TGs (>200 mg/dL
sequestra nts [2.26 mmol/L]). Constipation, abdominal
HDL cholesterol T3%-5%
pain, and nausea are common side effects,
No effect or possible t in TGs but are seen less frequently with
colesevelam.
Fibrates LDL cholesterol I 5%-20% Reduced CHD in 1 and 2 Most effective agents for reducing TGs, but
(T in patients with elevated TGs) prevention. may raise LDL cholesterol. Combination
HDL cholesterol T 10%-35% therapy with statins reduces overall
cholesterol profile but may increase risk for
TGs i 20%-50% myopathy. Avoid in patients with gallstones
or kidney disease.
Nicotinic acid LDL cholesterol I 5%-25% Reduced CHD and Most effective agent for increasing HDL
atherosclerotic progression in cholesterol. Flushing is common, but may be
HDL cholesterol T 15%-35% less frequent with sustained-release
2 prevention.
TGs I 20%-50% formulations or with prior administration of
aspirin. Other adverse events include
hepatotoxicity, gout, and hyperglycemia.
Ezetimibe LDL cholesterol 118% No evidence for cardiovascular Can reduce LDL cholesterol by an additional
benefit in 1 or 2 prevention. 19% when added to statin therapy. May be
associated with myopathy.

Omega-3 fatty TGs I 30%-50% Mixed evidence for reduction of Used as an alternative to fibrates or
acids CHD in 2 prevention. nicotinic acid for lowering TGs. Therapeutic
doses range from 3-12 g/d.

1 = primary; 2 = secondary; CHD = coronary heart disease; TG = triglycerides.

should be used cautiously or not at all in patients taking would treat similar numbers of patients with statins, but the
CYP3A substrates, such as simvastatin, lovastatin, and ator tailored treatment approach prevented more cardiac morbid
vastatin. In contrast, pravastatin is metabolized renally, and ity and mortality.
fluvastatin and rosuvastatin are metabolized by CYP2C9, Bile acid scquestrants are most effective when used in
potentially making these better choices in patients treated combination with statins, but may be used as monodierapy in
with multiple medications. Additional adverse events associ patients who are considering pregnancy or need only modest
ated widi statin use and potential management strategies are LDL cholesterol reduction. Ezetimibe monotherapy can
listed in Table 27. Patients should be monitored for adverse reduce LDL cholesterol by 18%, but evidence of improve
events by obtaining serum creatine kinase and aminotrans ment in cardiovascular outcomes with this medication is lack
ferase levels before initiating statin therapv'. Because the risk of ing; it may be considered in patients who are intolerant of
statin therapy.
significant liver or muscle damage is very low, routine follow-
up testing is not indicated and should be performed based on
die development of symptoms or other clinical findings while Combination Drug Therapy
on therapy. Combination drug therapy may be required to achieve LDL
Critics of" the ATP III "treat to target" approach, out cholesterol goals in select patients. Bile acid scquestrants are
lined above, note that many patients do not achieve target particularly effective at lowering LDL cholesterol when com
LDL cholesterol levels, possibly because of the complexity of bined with statins and can produce an additional 12% to 16%
the treatment guidelines and the potential adverse effects LDL cholesterol reduction. This combination is efficacious
associated with high-dose statins. A recent decision analysis with low to moderate doses of statins, and should be consid
compared the "treat to target" approach with a "tailored" ered early in therapy for patients with very high LDL choles
terol levels and in patients who may not tolerate high doses
approach, which eliminates the process of" adjusting statin
dosing according to achieved LDL cholesterol levels, and of statins. If combined statin-sequcstrant therapy is unsuc
instead uses a fixed dose of statin based on the calculated 5- cessful at achieving the target LDL cholesterol level, nicotinic
year cardiovascular risk. In this analysis, both approaches acid may be added as a third agent.

65
Dyslipidemia

table 27. Major Adverse Events Associated with Statin Use


Adverse Event D e fi n i t i o n Incidence Management strategy

Myalgia Muscle ache or weakness 5%-10% Investigate for other causes of muscle pain, including vitamin D
without an increase in CK deficiency, thyroid disease, fibromyalgia, medications, exercise,
and strenuous work.
Follow symptoms and CK levels weekly; discontinue statin or
decrease dose if symptoms worsen or CK levels increase.
Once symptoms resolve and CK levels return to baseline, consider
use of a statin or alternative LDL cholesterol-lowering
medications associated with less risk for myopathy: f luvastatin,
rosuvastatin, pravastatin, ezetimibe, bile acid sequestrants.
Consider supplementation with coenzyme Q10.
Myositis Muscle ache or weakness NA Same as for myalgia.
with an increase in CK less
than 10x ULN

Rhabdomyolysis Muscle ache or weakness 0.09% Discontinue statin therapy immediately. Monitor symptoms and
with an increase in CK levels. Statin may be restarted, preferably at a lower dose,
CK > 10x ULN and creatinine once symptoms resolve completely and CK levels normalize.
elevation, accompanied by
myoglobinuria
Elevated Incidental asymptomatic 3% Continue statin therapy. Elevation is typically transient and occurs
aminotransferases elevation of serum during the first 12 weeks of therapy. Recheck liver chemistry tests
aminotransferase levels to only if clinically indicated.
less than 3x ULN with no
associated histopathologic
changes
Hepatotoxicity Alanine aminotransferase 1% Discontinue statin therapy and recheck liver chemistry test results.
> 3x ULN with total bilirubin
If liver chemistry test results normalize, consider rechallenge with
levels > 2x ULN
the same statin at a lower dose or a different statin.
If liver chemistry test results remain elevated, continue to
withhold statin, screen for underlying liver disease, and consider
drug interactions.
CK = creatine kinase; NA = not available; ULN = upper limit of normal.

Statins and fibrates can be used together cautiously to excessive carbohydrate intake. In patients with above-goal
target both LDL and non-HDL cholesterol goals. LDL cholesterol and triglyceride levels, treatment with a
Gemfibrozil raises the serum concentrations of statins by two statin, which reduces triglycerides by 20% to 40%, is first-line
fold or more, thereby increasing the risk for myopathy;
dierapy. Patients with hypertriglyceridemia and elevated non-
fenofibrate is dius preferred for combination therapy. HDL cholesterol (>30 mg/dL [0.77 mmol/L] above LDL
The ARBITER 6-HALTS study compared the effective cholesterol goal) and normal LDL cholesterol levels may be
ness of" niacin or ezetimibe, when added to statin dicrapy, for treated with nicotinic acid or a fibrate initially according to
reducing carotid intima-media thickness and altering lipid lev ATP III guidelines. Nicotinic acid reduces triglyceride levels
els. HDL cholesterol levels in the niacin group increased by
by 30% to 50% and increases HDL cholesterol levels by 20%
18%, and LDL cholesterol levels in the ezetimibe group to 30% and may be the most efficacious monotherapy in
decreased by 19%; an improvement in carotid intima-media
patients with elevated non-HDL cholesterol levels. Fibrates
thickness was noted only for die niacin group. In two recent are also effective for reducing triglycerides and increasing
trials, combination treatment with simvastatin and fenofibrate HDL cholesterol levels (changes of 40%-60% and 15%-25%,
(the ACCORD-Lipid trial) and simvastatin and niacin (the respectively), but diey can raise LDL cholesterol levels by up
AIM-High trial) as compared with simvastatin monotherapy to 30%. Thus, fibrate monodierapy may be ineffective for
did not reduce cardiovascular risk.
achieving non-HDL cholesterol goals. Therapeutic doses of
eicosapentaenoic acid (3 to 9 g/d), which is found in fish oil
Management of Hypertriglyceridemia supplements, can lower triglyceride levels by up to 50%, and
Lifestyle changes are the primary dierapy for treatment of may be an alternative to nicotinic acid or fibrate dierapy.
hypertriglyceridemia, and patients should be encouraged to Evidence is lacking, however, diat treatment widi fish oil sup
lose weight, exercise regularly, and restrict alcohol use and
plementation reduces mortality or cardiovascular events. In a

66
Dyslipidemia

recent randomized controlled trial among patients widi estab


Metabolic Syndrome
lished cardiovascular disease, investigators failed to show a
reduction in cardiovascular events with low-dose daily sup Epidemiology and Pathophysiology
plementation of marine fatty acids, including eicosapen- The metabolic syndrome consists of a group of" risk factors for
taenoic acid and docosahexaenoic acid. cardiovascular disease and type 2 diabetes. Various criteria
exist for making this diagnosis; die ATP III criteria are pre
Management of Low HDL Cholesterol sented in Table 28. The designation of metabolic syndrome
Low levels of HDL cholesterol are associated with increased as a "syndrome" remains controversial. Some experts in the
cardiac risk, and treatment with medications diat increase fields of"diabetes, dyslipidemia, and obesity feel that focusing
HDL cholesterol levels, such as nicotinic acid and fibrates, on interventions aimed at improving the individual con
have been shown to reduce cardiovascular outcomes in sev stituents of the metabolic syndrome are more important than
eral large studies. However, diis effect may be mediated by lumping diose metabolic abnormalities into a syndrome.
treatment modification of die lipoprotein derangements that Others, however, feel diat grouping the risk factors into a syn
are associated with low HDL cholesterol levels, including drome creates a lower threshold for education and interven
tion and diat attaching a name may also help motivate
small, dense LDL particles and elevated triglycerides. Thus,
evidence of" a direct relationship between raising HDL cho patients to exercise, consume healthier diets, and lose weight.
lesterol levels and decreased cardiac risk is lacking. The epidemic of obesity and insulin resistance in the
In patients with CHD or CHD risk equivalents, treat United States has led to a sharply rising prevalence of meta
ment of abnormal LDL cholesterol levels should be the bolic syndrome, from an estimated 50 million cases in 1990
to 64 million by the year 2000. Metabolic syndrome is asso
primary target of" therapy. Once the goal LDL cholesterol
level has been achieved, the addition of nicotinic acid or a ciated with a 7-fold increased risk for the future development
fibrate may raise HDL cholesterol levels. In the VA-HIT of type 2 diabetes. It is also associated with a 2-fold increase
trial, patients with established cardiovascular disease, low in cardiovascular events and a 1.5-fold increase in all-cause
LDL cholesterol levels, and low HDL cholesterol levels mortality. The cardiovascular risk is present before die devel
were treated with gemfibrozil for 5 years. Gemfibrozil opment of diabetes.
increased HDL cholesterol and decreased triglyceride lev The clinical features and cardiovascular risks associated
with this syndrome appear to be related to the dysregulation
els, and this was associated with a 22% reduction in car
diovascular events. of adipose tissue. Cytokines secreted by adipose cells lead to
a proinflammatory state believed to accelerate atherosclerosis,
Therapeutic lifestyle changes are the primary therapy for
patients with low cardiac risk and abnormal HDL cholesterol plaque rupture, and atherothrombosis. Insulin-like growth
levels. Weight reduction and increased physical activity can factor is also believed to play a role in metabolic syndrome
increase HDL cholesterol levels by up to 30%; smoking ces and increased cardiovascular risk.
sation can also increase HDL cholesterol slightly. There is no
clear evidence, however, diat initiating drug therapy to raise Management of Metabolic Syndrome
HDL cholesterol levels in low-risk patients is beneficial. The "ABCDE" approach has been advocated as a reasonable
Moreover, a trial attempting to raise HDL cholesterol levels approach to management of metabolic syndrome (Table 29).
using the cholesteryl-ester transfer protein inhibitor torce-
trapib was terminated early because of excess deaths in the
table 28. Criteria for Metabolic Syndrome
torcetrapib group.
Any Three of the Following:
KEY POINTS
Risk Factor Defining Level
Therapeutic lifestyle changes are the cornerstone of
Abdominal obesity >40 in (>102 cm) in men; >35 in
lipid-lowering therapy and are recommended for all
(waist circumference) (>88 cm) in women
patients with abnormal lipid levels.
Triglyceride level3 >150 mg/dL (1.70 mmol/L)
Lipid-lowering drug therapy should be considered in
HDL cholesterol <40 mg/dL (1.04 mmol/L) in men;
patients who have not achieved LDL cholesterol <50 mg/dL (1.30 mmol/L) in women
goals after 3 months of therapeutic lifestyle changes, >130/>85 mm Hg
Blood pressure
and simultaneously widi lifestyle changes in patients
Fasting glucose >110 mg/dL (6.11 mmol/L)
with coronary heart disease or risk equivalents who
are unlikely to achieve LDL cholesterol goals with 'Triglyceride level 150 mg/dL (1.70 mmol/L) as a single factor correlates highly with
presence of metabolic syndrome.
dietary therapy alone.
Data from the National Heart Lung and Blood Institute. National Cholesterol Education
Baseline serum creatine kinase and aminotransferase Program. Third Report of the Expert Panel on Detection, Evaluation, and Treatment of
levels should be obtained before initiating statin ther High Blood Cholesterol in Adults (Adult Treatment Panel III): Executive Summary.
www.nhlbi.nih.gov/guidelines/cholesterol/atp_iii.htm. Published May 2001. Accessed
apy; routine follow-up testing is not indicated. July 14. 2009.

67
Dyslipidemia

table 29. "ABCDE" Approach to Management of Metabolic Syndrome

Assessment, aspirin Diagnose metabolic syndrome, calculate Framingham risk score


Begin aspirin if >6% 10-year Framingham risk
Blood pressure control Goal <130/80 mm Hg if intermediate Framingham risk (> 6% 10-year risk)
Cholesterol management LDL cholesterol: Statin therapy preferred; LDL cholesterol goal <100 mg/dL (2.59 mmol/L) for
high-risk patients, <130 mg/dL (3.36 mmol/L) for intermediate-risk patients (per ATP III
guidelines; see Table 25)
Non-HDL cholesterol: Statin intensification and fenofibrate therapy; non-HDL cholesterol goal
<130 mg/dL (3.36 mmol/L) for high-risk patients, <160 mg/dL (4.14 mmol/L) for intermediate-
risk patients (per ATP III guidelines)
Niacin for increasing HDL cholesterol controversial
Diabetes prevention, diet Intensive lifestyle modification
Metformin when indicated

Weight loss
Low glycemic diet
Consider Mediterranean diet
Exercise Daily vigorous activity
Recommend use of pedometer; goal >10,000 steps/d
ATP III = National Cholesterol Education Program Adult Treatment Panel III.

Adapted with permission from Blaha MJ. Bansal S. Rouf R. Golden SH, Blumenthal RS. Defilippis AP. A practical "ABCDE" approach to the metabolic syndrome Mayo Clin Proc
2008;83(8):932-941. [PMID: 18674478] Copyright 2008. Elsevier.

The first step is assessment and diagnosis of" metabolic syn in intermediate- and high-risk patients with metabolic syn
drome. The Framingham risk calculator can then be used to drome and elevated triglyceride levels.
calculate the patient's 10-ycar risk of" myocardial infarction Lifestyle modification, increased physical activity, weight
or death due to cardiovascular disease. As patients with loss, and diet should be emphasized in all metabolic syndrome
metabolic syndrome are at increased risk of thrombosis interventions. Weight loss reduces oxidative stress and
owing to increased platelet aggregation, aspirin (81 mg/d) improves all of die components of" metabolic syndrome. A
should be started in patients with intermediate or high diet with a high glycemic load leads to more insulin resistance;
Framingham risk (without contraindications to aspirin) to therefore, a low-glycemic diet should be emphasized. Cardiac
decrease platelet aggregation. The use of aspirin in low-risk and respirator)' fitness is associated widi increased insulin sen
patients is controversial owing to conflicting results in vari sitivity', decreased incidence of metabolic syndrome and
ous studies. decreased cardiovascular mortality.
Data are not clear about target blood pressures in meta KEY POINT
bolic syndrome, but some authors suggest a goal blood pres Aspirin should be started in patients with metabolic
sure of" below 130/80 mm Hg in patients with intermediate
syndrome without contraindications who have an
or high Framingham 10-year risk. Owing to mounting evi intermediate or high Framingham cardiovascular risk.
dence that P-blockers and thiazide diuretics worsen glucose
tolerance, the American Heart Association and others have
removed these drugs as recommended first-line agents to
Dyslipidemia Management in
treat blood pressure that is above goal in patients with meta Older Patients
bolic syndrome. ACE inhibitors and angiotensin receptor
Older patients (ages 65-80 years) with established cardiovas
blockers to treat high blood pressure may be better choices in
cular disease derive similar benefit as younger patients from
these patients because they have consistently been shown to
statin therapy and tolerate these medications well. Primary
improve glycemic control.
prevention with statins in this age group, however, is contro
Although LDL cholesterol is not considered in diag versial. Cardiovascular risk factor assessment may be less reli
nosing the metabolic syndrome, it is considered to be the able in older patients, and older age confers an increased risk
principal lipoprotein determinant of" atherosclerosis. It is for statin-associated myopathy. Thus, risks and benefits of"pri
generally recommended that in intermediate- and high-risk mary prevention with statin therapy must be weighed care
patients, statins be used. Fibrate therapy should be instituted fully in this age group.

68
Obesity

KEY POINT cardiovascular death or myocardial infarction. Again, the


Risks and benefits of"primary prevention widi statin USPSTF recommends weighing the potential harm (gas
trointestinal bleeding) against the benefit. The data are
therapy must be weighed carefully in older patients, as
unclear regarding harm versus benefit of" aspirin in men and
cardiovascular risk factor assessment is less reliable and
women older than 80 years. Table 30 shows die 10-year
adverse effects are more frequent in this age group.
CHD risk levels for men and stroke risk for women at which
the benefit of" aspirin exceeds harms. The USPSTF empha
sizes an individualized approach based on shared decision
Dyslipidemia Management and making between physicians and patients.
Stroke Prevention The optimal dose of aspirin for primary prevention is
The American Heart Association (AHA) guidelines on stroke unclear, as dosages used in studies have varied from 50 mg/d
prevention highlight the importance of lipid-lowering ther to 1500 mg/d. Given no evidence of improved outcomes and
apy. For prevention of recurrent transient ischemic attack or the increased risk of bleeding with higher doses, die USPSTF
stroke in patients with a history of CHD or elevated choles recommends the use of 75 mg/d aspirin when indicated for
terol levels, treatment should be aimed at lipid lowering this purpose.
according to ATP III guidelines. Lipid lowering for second The risk of gastrointestinal bleeding increases with age.
A history of peptic ulcer disease increases the risk of serious
ary stroke prevention in patients without a history of" cardiac
disease should aim for an LDL cholesterol level below 70 bleeding 2 to 3 fold. Other risk factors for serious gastroin
testinal bleeding include male sex (twice the risk of"women)
mg/dL (1.81 mmol/L) or an LDL cholesterol reduction of
at least 50%. Secondary stroke prevention may also include and NSAID therapy (4 times the risk compared with aspirin
treatment with niacin or gemfibrozil to increase low HDL alone).
cholesterol levels. KEY POINT
The U.S. Preventive Services Task Force recommends
aspirin in men ages 45-79 years for primary preven
Aspirin as an Adjunct to tion of cardiovascular disease and in women ages 55-
Dyslipidemia Management 79 years for primary prevention of stroke if the bleed
Aspirin reduces cardiovascular risk by irreversibly inhibiting ing risks do not outweigh benefit.
platelets by inactivating cyclooxygenase-1, thus blocking the
formation of thromboxane A2. Although well established for
die secondary prevention of cardiovascular disease, die USPSTF
recommends against die routine use of aspirin in primary pre Obesity
vention in men aged 44 years and younger as well as in
women aged 54 years and younger. Because the risk of first Definition and Epidemiology
myocardial infarction is reduced by aspirin use in men ages 45 Nearly 34% of adults and 17% of children in the United States
to 79 years, they recommend its use if the harm of gastroin are obese, defined as a BMI of" 30 or greater. These are dou
testinal bleeding does not outweigh the benefit. Calculating ble and triple the rates, respectively, of a quarter century ago,
Framingham risk scores can be used to help determine CHD and these numbers continue to rise. The highest rates are in
risk in this patient population. Similarly, in women ages 55 black women (50%) and Hispanic women (43%). The
to 79 years treated with aspirin, there is a statistically signif Institute of Medicine identifies obesity as an "emerging pri
icant decreased risk of ischemic stroke but not a decrease in ority area" for national action.

table 30. CHD Risk Levels at Which Cardiovascular Disease Events Prevented is Closely Balanced to Serious Bleeding Events
in Patients Taking Aspirin for Primary Prevention
Men Women

10-Year CHD Risk, % Age 10-Year Stroke Risk, %


Age
>4 55-59 y >3
45-59 y
>9 60-69y >8
60-69y
70-79 y >12 70-79y >11

CHD = coronary heart disease.

Reprinted from US Preventive Services Task Force. Aspirin for the prevention of cardiovascular disease: U.S. Preventive Services Task Force recommendation statement. Ann Intern Med.
2009;150<6):396-404. [PMID: 19293072]

69
Obesity

In adults, obesity is defined by BMI (body mass divided TABLE 32. Medications That Cause Weight Gain
by the square of" the height), and online BMI calculators are Drug Category Medications
widely available (www.nhlbisupport.com/bmi/). Categories
of obesity and overweight based on BMI proposed by the Atypical antipsychotic Clozapine, olanzapine, quetiapine,
World Healdi Organization and National Institutes of"Health agents risperidone
Antidiabetic drugs Sulfonylureas, thiazolidinediones,
(NIH) are listed in Table 31, along with die most recent
insulin
prevalence estimates for U.S. men and women.
Obese and overweight persons are at increased risk for Corticosteroids Glucocorticoids (e.g., prednisone)
heart disease, hypertension, dyslipidemia, type 2 diabetes mel i Tricyclic antidepressants Amitriptyline, imipramine, doxepin
litus, stroke, osteoardiritis, sleep apnea, gallbladder disease, Selective serotonin Paroxetine
certain cancers (endometrial, breast, colon), and overall mor reuptake inhibitors
tality. These risks increase progressively with rising BMI. Anticonvulsant agents Valproic acid, carbamazepine
Obesity also is associated with reduced quality oflifc, societal P-Blockers Propranolol, atenolol, metoprolol
discrimination, and increased health care costs. It is estimated
that obesity added S147 billion to health care costs in 2008.
In persons aged 65 years and older, obesity is associated with
impaired physical functioning, including difficulty with activ be challenging owing to excessive intervening adipose tissue,
ities of" daily living. and formal instruction in specific maneuvers in the physical
examination receives limited attention in standard textbooks.
However, practical suggestions to augment accuracy of the
Screening and Evaluation examination have been published (http://jama.jamanet
The NIH and the U.S. Preventive Services Task Force work.com/article.aspx?doi=10.1001/jama.2010.1950).
(USPSTF) recommend screening all adults for obesity. The Assessment for comorbidities associated widi obesity
USPSTF recommends screening by calculating the BMI. The should be considered. The USPSTF does not recommend
NIH recommends screening both widi BMI and waist cir specific laboratory tests based on the presence of" obesity
cumference measurement. Waist circumference is measured alone, but the American Diabetes Association (ADA) recom
widi a measuring tape placed around die abdomen at die level mends screening for diabetes in patients with a BMI of 25 or
of the iliac crest. Central adiposity (waist circumference in greater who have other risk factors for diabetes. Although the
men >102 cm [40 in]; in women >88 cm [35 in]) is associ major cause of obesity is imbalance between caloric intake and
ated with an increased risk for type 2 diabetes, dyslipidemia, energy expenditure, physicians should consider less common
hypertension, and heart disease, not only in obese persons, causes, including hypothyroidism and Cushing syndrome.
but also in those who are overweight. KEY POINTS
The assessment of die obese patient begins with the history
All adults should be screened for obesity' using BMI
and physical examination. The history should specifically elicit
calculation.
chronology of weight gain, family history of obesity, medica
tions that can promote weight gain (Table 32), exercise, eating Central adiposity' is associated with an increased risk
patterns, and symptoms and risk factors for cardiovascular dis for type 2 diabetes, dyslipidemia, hypertension, and
ease. The general physical examination of the obese patient can heart disease in both overweight and obese persons.

table 31. Classification of Obesity

2007-2008 U.S. Prevalence of Obesity8


Category Adult BMI Men >20 y (%) Women >20 y (%)
Underweight <18.5
Normal weight 18.5-24.9

Overweight 25.0-29.9 30 29
Obese >30.o 32 36
Obesity class I 30.0-34.9 21 18
Obesity class II 35.0-39.9 7 11
Obesity class III >40.0 4 7
Summary (obese plus overweight) >25.0 62 65
-Data from U.S. Health and Human Services Centers for Disease Control and Prevention. Health United States. 2010. Available at www.cdc.gov/nchs/data/hus/hus10.pdf. Accessed January 4.

70
Obesity

Treatment goal is to achieve a reduction in body weight of" 10% at a rate


of" 0.5 to 1 kg (1-2 lb) per week during a 6-month period.
After the initial assessment of the obese patient, the physician This degree of" weight loss will reduce the risk of" many med
should determine die patient's level of motivation for weight ical complications of obesity. In addition to lifestyle modifi
loss, and an individualized treatment plan should follow. cation (behavioral changes, exercise, and diet), medication
Success can be enhanced by a combined team of health pro and surgery have become increasingly popular options. The
fessionals, including a dietitian, a behavioral therapist, and an overall management of" the obese patient is presented in
exercise therapist. Adherence to strategies that decrease Figure 16.
caloric input and increase energy expenditure will result in
KEY POINT
weight loss. A combined approach of diet and physical activ
A recommended weight loss goal is a reduction in
ity is likely to be more successful than an individual one, but
body weight of 10% at a rate of 0.5 to 1 kg (1-2 lb)
strategies that focus on decreasing caloric input are more suc
cessful dian those diat only focus on energy expenditure. The per week during a 6-month period.

Obese patient (BMI >30)

Assess comorbid conditions


Determine goals (weight loss,
time frame, comorbidity
risk reduction)
Recommend weight loss,
diet, exercise, lifestyle changes

/ X Ye s Continue weight and lifestyle


( P a t i e n t m e e t s g o a l ? ) '
management with diet and exercise
No

Reassess comorbid conditions


Reassess weight loss goal

c Is weight loss sufficient?

W
o
Yes Continue weight and lifestyle
management with diet and exercise

For patients who desire adjunctive


therapy, discuss risks and benefits,
lack of long-term safety data

Consider adjunctive drug


therapy. Options include
Patient accepts risks orlistat (long-term),
and benefits of drug therapy? phentermine, phentermine/
topiramate, diethylpropion
No (short-term), or lorcaserin.

Continue weight and lifestyle


management with diet and exercise.
For patients with BMI >40 or
BMI 35-40 with obesity-related
complications, consider referral
for bariatric surgery.

FIGURE 16. Management of the obese patient.


Adapted with permission from Snow V, Barry P, Fitterman N, Qaseem A, Weiss K; Clinical Efficacy Assessment Subcommittee of the American College of Physicians. Pharmacologic and surgical
management of obesity in primary care: a clinical practice guideline from the American College of Physicians. Ann Intern Med. 2005; 142(7):526. [PMID: 15809464] Copyright 2005. American
College of Physicians.

71
Obesity

Lifestyle Modification was better in the short term for the low-carbohydrate diet,
The USPSTF recommends that clinicians offer all obese there was no difference at 1 year. A randomized trial com
patients intensive counseling and behavioral interventions. paring outcomes of an intensive behavioral intervention
This includes counseling in diet and exercise. Lifestyle modi combined with either a low-fat or a low-protein diet resulted
fication has been shown to be the most effective intervention in an 11% weight loss at 1 year and a 7% weight loss in 2
to prevent type 2 diabetes, and die ADA recommends lifestyle years in both groups. The low-carbohydrate diet group
modification for diabetes prevention. demonstrated a 23% increase in HDL cholesterol levels at 2
years. Exercise can enhance weight loss, especially when
Behavioral Therapy begun during the early phases, but caloric restriction is the
Behavioral therapv' focuses the patient's attention on his or key aspect to continued loss of weight. Recent evidence sug
her personal maladaptive eating patterns and assists die gests that inadequate sleep (<7-9 hours) also compromises
patient to control or modify food intake, increase exercise, the success rates of" typical dietary interventions.
and avoid stimuli that trigger eating. Behavioral therapy is
KEY POINTS
best when accomplished by a therapist trained in this tech
All obese patients should be offered intensive coun
nique and in combination widi other modalities but can be
initiated by the primary care physician. Principles of behav seling and behavioral interventions to encourage
ioral therapy using motivational interviewing are discussed in weight loss, including counseling in diet and exercise.
Routine Care of"the Healthy Patient. In a systematic evidence No one diet has been demonstrated superior to oth
review for the USPSTF on the effectiveness of" primary ers in achieving long-term weight loss.
care-relevant treatment for obesity in adults, 21 studies that
used behavioral interventions resulted in a weight loss of 1.5 Pharmacologic Therapy
to 5.0 kg (3.3-11 lb) at 12 to 18 months, compared with lit With the rising prevalence of obesity, drug therapy has
tle or no weight loss in the control groups. Behavioral ther emerged as an attractive option for weight loss in obese
apy is time intensive, and regain of" weight is common. patients, especially when lifestyle modification is ineffective.
However, in a recent randomized controlled trial of patients However, bodi short-term and long-term safety and efficacy
who successfully completed an initial behavioral weight loss may limit use in many patients. Current FDA-approved
program, monthly brief"personal contact follow-up was asso options for drug therapy in the United States include sympa
ciated with sustained weight loss. The use of technology- thomimetic drugs that suppress appetite (phentermine,
based interventions (Web-based interactive interventions)
diethylpropion) and drugs that alter tat absorption (orlistat).
provided early but transient benefit. Several appetite suppressant medications, including sibu-
tramine, have been removed from the market over the years
Exercise owing to safety concerns.
Exercise as monotherapy is likely not adequate for significant Sympathomimetic drugs are approved only for short-term
weight loss. In a 12-month study of" moderate to vigorous use as an adjunct to other weight loss programs. Aldiough
exercise for 1 hour daily, 6 days per week, women lost only studies have documented weight loss with continuous and
1.4 kg (3.1 lb) and men lost 1.8 kg (4 lb). However, exercise intermittent use (net loss, 3.6 kg [7.9 lb]), most users regain
at a level of" walking 60 to 90 minutes per day is effective in weight upon discontinuation. Significant increases in blood
maintaining weight that is lost and, therefore, is a useful pressure and arrhythmias can occur with phentermine; caution
adjunct to any weight loss program. Exercise has other health is indicated in patients with hypertension and cardiovascular
benefits, including improving cardiovascular health and disease. However, in a recent study of low-dose, controllcd-
decreasing waist circumference. release phentermine plus topiramate combined widi office-
based lifestyle intervention, modest weight loss was achieved
Consistent Dietary Caloric Restriction (8.1 kg [17.8 lb] at 56 weeks compared with 1.4 kg [3.1 lb]
Consistent dietary caloric restriction leads to successful in the placebo group). Significant improvement was noted in
weight loss. Obese patients can lose approximately 0.45 kg waist circumference, blood pressure, and lipid levels in die
(1 lb) weekly by decreasing their intake by 500 to 1000 treatment group. Combination phentermine/topiramate has
kcal/d below what is needed to maintain current weight. been FDA-approved for die treatment of obesity.
Total calories should not be restricted to less than 800 Orlistat, now available over-the-counter, is a lipase
kcal/d as "very-lovv-calorie" diets are no more effective than inhibitor that leads to fat malabsorption. In a recent meta
low-calorie diets for successful long-term weight loss and analysis on the pharmacologic treatment of obesity, die mean
have higher adverse consequences. No one diet has consis weight loss in patients treated with orlistat was 2.9 kg (6.4 lb)
tently been shown to be superior to others when long-term at 12 months. Secondary benefits included reductions in LDL
outcomes are measured. A meta-analysis comparing low-fat cholesterol level and blood pressure and, in patients with dia
to low-carbohydrate diets found that whereas weight loss betes, improvement in glycemic control. Approximately 15%

72
Obesity

to 30% of"patients experience gastrointestinal side effects (fla


tus, abdominal cramps, fecal incontinence, oily spottage),
especially while consuming high-fat diets. Orlistat has not been
associated with serious cardiovascular side effects. However, a
recently completed review by die FDA noted rare reports of
severe liver injury with orlistat. Malabsorption of fat-soluble
vitamins A, D, and E has been reported, and vitamin supple
mentation is advisable while taking the medication.
In 2012, lorcaserin, a brain serotonin 2C receptor ago
nist, was FDA approved for adults with BMI greater than 30
or greater than 27 with obesity-related complications. In
Roux-en-Y gastric bypass Vertical banded gastroplasty
conjunction with a reduced-calorie diet and exercise coun
seling, lorcaserin was associated with an average weight loss
of 3% at 1 year. Lorcaserin should be used with caution in
patients who are on medications that increase serotonin lev
els. It has not been studied in patients with significant valvu
lar heart disease.

KEY POINT
Pharmacologic options for weight loss include orlis
tat for long-term use, the appetite suppressants
phentermine and diethylpropion for short-term use,
and lorcaserin.

Surgery
For class II or III obese patients (BMI >35) in whom diet,
exercise, and/or medication have failed, especially those with Adjustable gastric band Biliopancreatic diversion
with duodenal switch
significant obesity-related comorbidities, weight loss surgery
should be considered. The NIH Consensus Development
FIGURE 17. Surgical procedures for obesity.
Conference Statement suggested the following criteria for
Reprinted with permission from Maggard MA. Shugarman LR, Suttorp M. et al. Meta-analy
considering a patient for bariatric surgery: sis: surgical treatment of obesity. Ann Intern Med. 2005;142<7):547-559. [PMID: 158094661
Copyright 2005, American College of Physicians.
1. Patients should be well-informed, motivated, able to par
ticipate in treatment and long-term follow-up, and have
acceptable operative risks.
2. Patients should have a BMI that exceeds 40. Randomized controlled and cohort studies comparing
bariatric surgery widi nonsurgical interventions have found
3. Patients with a BMI between 35 and 40 with obesity-
that surgery results in more dramatic and sustained weight
related comorbidities, such as severe sleep apnea, dia
loss and leads to improvement in obesity-related comorbidi
betes, or severe joint disease, should be considered.
ties. In a meta-analysis comprising 16,944 patients with a
4. Patients should be evaluated by a multidisciplinary team
mean BMI of47 (range 32-69) who underwent some type of"
with medical, surgical, psychiatric, and nutritional
bariatric procedure, die average weight loss at 12 mondis was
expertise.
43.46 kg (95.6 lb) for Roux-en-Y gastric bypass, 32.16 kg
Surgical therapies involve restriction of stomach size and (70.8 lb) for vertical band gastroplasty, 30.19 kg (66.4 lb) for
malabsorption of ingested calories as their mechanisms of
adjustable gastric banding, and 51.93 kg (114.2 lb) for bil
action. According to worldwide survey data, the most com
mon procedures are Roux-en-Y gastric bypass (65.1%), iopancreatic diversion. Diabetes resolved in 76.8% of patients,
hypertension resolved in 61.7%, obstructive sleep apnea
laparoscopic adjustable band procedures (24%), vertical
resolved in 85.7%, and hyperlipidemia improved in 70%.
banded gastroplasties (5.4%), and biliopancreatic diversion
Another meta-analysis reached similar conclusions. In a large
(4.8%) (Figure 17). The laparoscopic adjustable band proce
dure is increasing in popularity among patients and physi retrospective cohort study, all-cause mortality decreased by
40% in the surgery group compared with the control group
cians, although the weight loss with this procedure, and
restrictive surgeries in general, is less robust compared with at a mean follow-up of'7.1 years. However, in a recent subset
of obese, high-risk, primarily male patients, bariatric surgery
gastric bypass. The two most common procedures are com
pared in Table 33. was not significantly associated with survival during a mean of

73
Men's Health

TABLE 33. Comparison of Commonly Used Bariatric Procedures


Characteristic Laparoscopic Adjustable Roux-en-Y Gastric Bypass
Gastric Banding

Mechanism of weight loss Restrictive Restrictive and malabsorptive


Technique Adjustable silicone ring is placed around 7-10 mL gastric pouch is separated from the rest of the
the top part of the stomach, creating a stomach and connected to the small intestine, bypassing
15-30 mL pouch. the rest of the stomach and duodenum.
Dietary program <800 kcal/d for 18-36 months; 1000-1200 <800 kcal/d for 12-18 months; 1000-1200 kcal/d thereafter
kcal/d thereafter. in three small, high-protein meals per day.
Certain foods can get "stuck" (rice, bread, Avoid sugar and fats to prevent "dumping syndrome."
meats, nuts), causing pain and vomiting.
Hospital length of stay <1 day 3-4 days
Weight loss at 12 months 30.19 kg (66.4 lb) 43.46 kg (95.6 lb)
30-day mortality 0.1% 0.5%
Incidence of postoperative 10% 15%
complications
Nonoperative Nausea, vomiting, reflux Cholelithiasis, nutritional deficiencies (vitamin B12, other
complications B vitamins, iron, calcium, folic acid, vitamin D; rare:
magnesium, copper, zinc, vitamin A, vitamin C)

6.7 years of follow-up. Some studies document short-term to die attention of their healdi care providers. Sexual dys
improvements in quality of life. function can be categorized as either erectile dysfunction
Bariatric surgery carries procedure-specific short-term (ED), ejaculatory dysfunction, or decreased libido.
and long-term risks. Surgical mortality rates are low (<1%)
and appear to be reduced widi the laparoscopic approach and Erectile Dysfunction
with surgeons who perform a high volume of the procedures. ED, the most common form of"sexual dysfunction in men, is
Complications of" bariatric surgery are discussed in MKSAP the inability' to maintain an erection sufficient for satisfactory
16 Gastroenterology and Hepatology. Long-term multidisci- sexual performance. Up to one in three men experience ED
plinary follow-up of"the patient after bariatric surgery is essen at some point in their lives.
tial. An average of"20% to 25% of lost weight is regained in 10
years. Dietary counseling, increased physical activity (at least Risk Factors
150 minutes weekly), and behavioral modification are rec The prevalence of" ED increases with age, widi up to 30% of*
ommended. For patients who have undergone a Roux-en-Y men older than 70 years being affected. Obese men have
procedure, recent guidelines suggest twice yearly monitoring close to a two-fold greater risk of developing ED dian men of
of vitamin D, calcium, phosphorus, parathyroid hormone, normal weight. Risk is similarly increased in men who smoke
and alkaline phosphatase levels. To assess nutritional deficien cigarettes compared with nonsmokers. Other modifiable risk
cies after a malabsorptive procedure, ferritin, vitamin Bp, factors include sedentary lifestyle and use of" a variety of" sub
folate, vitamin D, and calcium levels should be assessed every stances, including alcohol, barbiturates, cocaine, heroin, mar
6 months for the first 2 years and annually thereafter. Bone ijuana, and methamphetamines. Not only do ED and cardio
mineral density testing is recommended yearly until stable. vascular disease share many risk factors (diabetes niellitus,
KEY POINT hyperlipidemia, and hypertension), ED is itself a cardiac risk
factor diat independendy predicts mortality' and confers a risk
Bariatric surgery may be considered for patients with similar to that of" moderate smoking. Hormonal disorders
a BMI above 40 or with a BMI above 35 with obe associated widi ED include hypogonadism, hypothyroidism,
sity-related complications. and hyperprolactinemia. The risk of ED increases widi a his
tory of previous pelvic irradiation, surgery (including, but not
limited to, prostate surgeries), or trauma. Neurologic condi
tions associated with an increased risk of" ED include demen
Men's Health tia, multiple sclerosis, prior cerebrovascular accident, and
quadri- or paraplegia.
Male Sexual Dysfunction ED is a common side effect of medications. Up to one-
Male sexual dysfunction is common and causes significant dis quarter of"all cases of ED are diought to be due to medications.
tress. Many men, however, hesitate to bring such symptoms Table 34 provides a list of medications commonly associated

74
Men's Health

TABLE 34. Medications Commonly Associated with important to inquire about lifestyle factors and die sexual
Erectile Dysfunction function of" the patient's partners.
During the physical examination, die patient's BMI, blood
Antidepressants pressure, and heart rate should be determined. A genital, digi
Monoamine oxidase inhibitors tal rectal, and screening neurologic examination in addition to
Selective serotonin reuptake inhibitors an assessment of lower extremity pulses and secondary sexual
Tricyclic antidepressants characteristics should also be performed. Because many
Benzodiazepines episodes of ED are transient, an extensive laboratory evaluation
Opioids is not mandatory at presentation vvidiout symptoms or findings
Anticonvulsants suggestive of an underlying systemic disorder or before imple
Phenytoin menting lifestyle modification or counseling dierapy. If" indi
Phenobarbital cated for suspected abnormalities or persistent symptoms, a rea
Antihypertensives sonable initial laboratory investigation consists of obtaining a
a-B lockers fasting serum glucose level, lipid panel, thyroid-stimulating hor
P-Blockers mone level, and early morning total testosterone level.
Calcium channel blockers Additional laboratory tests should only be obtained on die basis
Clonidine of"findings in the history and physical examination.
Spironolactone
Thiazide diuretics Treatment
5a-Reductase inhibitors The first step in successfully treating ED is to attempt to
Dutasteride identify an underlying etiology. If possible, treatment should
Finasteride be directed toward the underlying condition. Regardless of"
etiology, first-line therapies for ED consist of both lifestyle
modifications and pharmacotherapy widi phosphodiesterase
with ED. Various psychiatric conditions are associated widi
type 5 (PDE-5) inhibitors. In one study, nearly one third of"
ED, including anxiety, depression, and prior sexual abuse. obese men had improved ED symptoms simply by exercis
Relationship discord and stress can also lead to ED.
ing regularly and losing weight. Attempts should therefore
be made to lower the patient's BMI below 30. Patients
Diagnosis and Evaluation should be counseled to stop smoking and consideration
The initial evaluation of a man reporting ED consists of"
should be given to stopping (if possible) any medications
obtaining medical, psychosocial, and sexual histories in addi that are associated with ED. Psychosexual therapy per
tion to performing a focused physical examination and a lim
formed by a sex therapist has been found to improve ED in
ited laboratory investigation. During the medical history, it is
40% to 80% of patients.
important to first obtain an accurate description of what the PDE-5 inhibitors function by increasing cyclic guano-
patient is experiencing, as some patients confuse premature
sine monophosphate (cGMP) levels, which leads to smooth
ejaculation with ED. Clinicians should seek to determine the
muscle relaxation and engorgement. Improvement in ED
presence or absence of" medical conditions, medications, and
other risk factors associated with ED. A psychosocial history appears to be dose related for both sildenafil (up to 50 mg)
should be performed with the purpose of" identifying any and vardenafil (all doses) but not tadalafil. All PDE-5
inhibitors improve erections and sexual intercourse success
underlying psychological conditions that may be contribu
compared with placebo and possess approximately equiva
tory. Sexual history includes libido (loss is seen widi androgen
lent efficacy. The efficacy of PDE-5 inhibitors is further
deficient', depression, and as a side effect of medication use);
ability to achieve and maintain erections, including circum improved when combined with other ED therapies such as
stances (die sudden onset of die loss of" ability' to achieve an psychotherapy and attempts at weight loss. Because ED
erection suggests a psychogenic origin but can also occur fol shares many risk factors with atherosclerotic disease and itself"
is an independent risk factor, an assessment of the patient's
lowing radical prostatectomy and after trauma); ability to
achieve orgasm, including quality and timing (if situational, cardiovascular risk should be made before prescribing a
PDE-5 inhibitor. This can be accomplished using guidelines
suggests psychogenic origin); and penile curvature (its pres
ence, along with penile pain, suggests Peyronie disease); and established by the Second Princeton Consensus Conference
presence of early morning erections (presence suggests intact (Table 35).
corpus cavernosae blood flow and intact neurogenic reflexes). PDE-5 inhibitors are generally well tolerated.
The severity of" ED symptoms can be assessed using the Headache, the most common side effect, occurs in approx
International Index of Erectile Function-5 (IIEF-5), a vali imately 10% of patients. Dizziness, dyspepsia, flushing,
dated, five-item questionnaire (available at WAVw.hiv.va.gov/ rhinitis, syncope, and visual disturbances occur less com
provider/manual-primary-care/urology-tool2.asp). It is also monly. Nonarteritic anterior optic neuropathy, although

75
Men's Health

table 35. Second Princeton Consensus Conference Guidelines for Treatment of Erectile Dysfunction in Patients with
Cardiovascular Disease or Cardiac Risk Factors
Risk Level Treatment Recommendation
Low risk

Asymptomatic and <3 major cardiac risk factors1' Can initiate or resume sexual activity or treat for ED with PDE-5
inhibitor (if not using nitrates)
Controlled hypertension
Mild stable angina
Post successful coronary revascularization
Ml (>6-8 weeks before)
Mild valvular disease
Left ventricular dysfunction (NYHA functional class I)

Intermediate/indeterminate risk

Asymptomatic and >3 major cardiac risk factors-' Further cardiac evaluation (stress test or cardiology
Moderate stable angina consultation) and restratification before resumption of sexual
activity or treatment for ED
Recent Ml (2-6 weeks)
Left ventricular dysfunction (ejection fraction <40%) or
heart failure (NYHA class II)
Noncardiac atherosclerotic disease (clinically evident PAD,
history of stroke/TIA)

High risk

Unstable or refractory angina Defer sexual activity or ED treatment until cardiac condition is
stabilized and reassessed
Uncontrolled hypertension
Moderate to severe heart failure (NYHA class III-IV)
Recent Ml (<2 weeks)
High-risk arrhythmia
Obstructive hypertrophic cardiomyopathy
Moderate to severe valvular disease (particularly aortic stenosis)

ED = erectile dysfunction; Ml = myocardial infarction; NYHA = New York Heart Association; PAD = peripheral arterial disease; PDE = phosphodiesterase; TIA = transient ischemic attack.

'Major cardiac risk factors are: age, hypertension, diabetes mellitus, smoking, dyslipidemia, sedentary lifestyle, family history of premature coronary artery disease. (Male sex is excluded.)

Recommendations from Kostis JB, Jackson G, Rosen R. et al. Sexual dysfunction and cardiac risk (the Second Princeton Consensus Conference). Am J Cardiol. 2005;96(2):313-321 [PMID:
16018863]

extremely rare, has been documented and is caused by placement of" penile prostheses. Testosterone dierapy appears to
crossover inhibition of PDE-6 receptors. The use of" PDE- be effective in men with ED and androgen deficiency (see below).
5 inhibitors is contraindicated in patients taking nitrates
owing to the potential for profound hypotension. Caution Premature Ejaculation
should be exercised when using PDE-5 inhibitors in Premature ejaculation, ejaculation that occurs sooner than
patients concomitantly taking drugs that inhibit the desired, is diagnosed by obtaining a thorough sexual history.
cytochrome P-450 3A4 pathway, such as protease Treatment should be individualized and include a discussion
inhibitors, erythromycin, and ketoconazole. A comprehen with the patient of risks and benefits of each option.
sive list of interacting medications can be accessed at Pharmacologic dierapies can be either oral or topical. Oral
http://medicine.iupui.edu/clinpharm/ddis/ClinicalTable therapies function by causing anorgasmia and include several
.aspx. selective serotonin reuptake inhibitors (fluoxetine, paroxetine,
Second-line dierapies for ED include prostaglandin E, and sertraline) and clomipramine, a tricyclic antidepressant.
(alprostadil) administered either via intracavcrnosal or intrau Topical anesthetic agents, such as lidocaine or prilocaine
rethral routes, widi die former being more effective and better cream, function by reducing stimulation and can be used widi
tolerated. Additional therapies include use of" penile pumps and or without a condom.

76
Men's Health

Decreased Libido TABLE 36. Symptoms and Signs Suggestive of


Decreased libido, a decreased desire for sexual activity, is less Androgen Deficiency in Men
common in men than in women but often is more disabling. More specific symptoms and signs
When decreased libido is associated widi marked distress and
Incomplete or delayed sexual development, eunuchoidism
interferes with relationships, the term hypoactive sexual desire
Reduced sexual desire (libido) and activity
disorder is used. Causes include hypogonadism, hyperpro
lactinemia, medications, relationship difficulties, and psychi Decreased spontaneous erections
atric disorders (most commonly depression). Decreased Breast discomfort, gynecomastia
libido also is more common as men age. Treatment is fre Loss of body (axillary and pubic) hair, reduced shaving
quently directed toward the underlying cause, if one is iden Very small (especially <5 mL) or shrinking testes
tifiable. It is also important to explore the quality of the rela
Inability to father children, low or zero sperm count
tionship with the patient's sexual partner, as relationship
difficulties may be die underlying issue rather dian decreased Height loss, low-trauma fracture, low bone mineral density
libido. Hot flushes, sweats

KEY POINTS Less specific symptoms and signs

First-line therapy for erectile dysfunction consists of Decreased energy, motivation, initiative, and self-confidence
lifestyle modifications, psychosexual counseling, and Feeling sad or blue, depressed mood, dysthymia
pharmacotherapy with a phosphodiesterase type 5 Poor concentration and memory
inhibitor.
Sleep disturbance, increased sleepiness
Men with erectile dysfunction (ED) and three or
Mild anemia (normochromic, normocytic, in the female range)
more cardiac risk factors require cardiac evaluation
Reduced muscle bulk and strength
before initiating treatment for ED or resuming sexual
Increased body fat, BMI
activity.
Diminished physical or work performance
The use of phosphodiesterase type 5 inhibitors is con
traindicated in patients taking nitrates owing to the Adapted with permission from Bhasin S, Cunningham GR. Hayes FJ, et al; Task Force,
Endocrine Society. Testosterone therapy in men with androgen deficiency syndromes: an
potential for profound hypotension. Endocrine Society clinical practice guideline. J Clin Endocrinol Metab. 2010;95(6):2537.
[PMID: 20525905I Copyright 2010, The Endocrine Society.

Androgen Deficiency
MKSAP 16 Endocrinology and Metabolism). Testosterone
Androgen deficiency in the setting of male hypogonadism is
dierapy for men is recommended with symptomatic androgen
suggested by the signs and symptoms listed in Table 36.
Men with specific signs and symptoms of" androgen defi deficiency to induce and maintain secondary sex characteris
tics and to improve sexual function, sense of"well-being, mus
ciency should be evaluated by measuring morning total
cle mass and strength, and bone mineral density. Because of
testosterone level as the initial diagnostic test. However,
because the less specific symptoms of potential androgen potential adverse effects of" testosterone therapy, the
Endocrine Society recommends against its use in patients
deficiency overlap considerably with many other common
with breast or prostate cancer, a palpable prostate nodule or
symptoms, testing in men with these symptoms should be
induration, a prostate-specific antigen (PSA) level greater
approached judiciously, particularly those without more
dian 4 ng/mL (or >3 ng/mL in men at high risk for prostate
definitive findings on history or examination that suggest
the diagnosis. The assessment of" men for androgen defi cancer), a hematocrit greater than 50%, untreated severe
obstructive sleep apnea, severe lower urinary tract symptoms,
ciency should include a general health evaluation to exclude
or uncontrolled or poorly controlled heart failure.
systemic illness and a review of" medications (including opi
oids and high-dose corticosteroid therapy) and recreational Management of patients on testosterone replacement dierapy
is addressed in more detail in MKSAP 16 Endocrinology and
drugs (such as marijuana) that affect testosterone produc
Metabolism.
tion or metabolism. Eating disorders and excessive exercise
can transiently lower testosterone levels. Clinical guidelines KEY POINTS
published by the Endocrine Society recommend against Men with symptoms of" androgen deficiency and low-
screening of asymptomatic men in the general population normal testosterone levels should have confirmatory
for androgen deficiency, regardless of age.
testing before initiating testosterone therapy.
Men with low-normal testosterone levels should have
Guidelines recommend against screening of asympto
confirmatory testing before initiating testosterone therapy, matic men in die general population for androgen
and further evaluation of the cause of" hypogonadism should
deficiency, regardless of age.
be pursued before treatment is started, if" indicated (see

77
Men's Health

Benign Prostatic Hyperplasia Peripheral a-blockers and 5a-reductase inhibitors (5-


ARIs) are the two most common drugs used for BPH, either
Benign prostatic hyperplasia (BPH) is a common cause of
alone or in combination, a-Blockers, effective in 70% of
lower urinary tract symptoms (LUTS) in men. LUTS can be
classified as related to overactive bladder symptoms versus blad patients, have more immediate clinical effect, whereas 5-ARIs
der oudet obstruction (BOO). Overactive bladder symptoms, may lead to clinical shrinkage of" the enlarged prostate gland.
caused by BPH and detrusor muscle hyperactivity, include noc Coexisting overactive bladder symptoms can be treated with
turia and urinary frequency and urgency. BOO symptoms, combination a-blocker and anticholinergic therapy (toltero
caused by physical obstruction, include decreased urinary dine, oxybutynin). Anticholinergic drugs should be avoided in
those with post-void residual volumes greater dian 250 mL.
stream, urinary retention, incomplete bladder emptying, and
incontinence. The differential diagnosis of'BPH includes neu When BPH coexists with BOO symptoms, combination
a-blocker and 5-ARI dierapy has shown die highest efficacy
rologic conditions (stroke, multiple sclerosis, spinal cord con
ditions) as well as effects of certain medications, such as diuret compared with other treatments when die prostate is
ics. Guidelines from die American Urological Association enlarged. Data from two trials indicate that combination ther
(AUA) recommend performing a digital rectal examination apy may be better in the short term than treatment with either
and obtaining a baseline AUA symptom index score (ques drug individually, aldiough 5-year outcomes remain similar.
tionnaire available at http://www2.niddk.nih.gov/NR/ Although commonly used by patients, complementary or
alternative medications, such as saw palmetto, have not been
rdonlyres/8E99FCF4-8A92-43EE-8E47-5B70D634938A/
found to have benefit in recent clinical trials.
O/AUABPH.pdf) to track progression and effectiveness of
treatment. A urinalysis should be obtained when evaluating The PDE-5 inhibitor tadalafil has been FDA-approved
BPH symptoms to rule out underlying infection. for treatment of lower urinary tract symptoms in BPH. No
Treatment modalities include watchful waiting and con long-term studies have compared outcomes widi tadalafil to
servative measures for mild symptoms, drug therapy (Table those with a-blockers or 5-ARJs.
37), and surgical interventions for severe or refractory symp Indications for surgical interventions in patients with
toms. Conservative measures include reduced fluid intake, BPH include refractory lower urinary tract symptoms, recur
timed voiding, limiting caffeine and alcohol, discontinuing rent urinary retention with or without urinary tract infection,
exacerbating medications, and improving mobility. bladder stones, and kidney failure widi hydronephrosis. Open
prostatectomy is indicated for a severely enlarged prostate;
however, diose widi less severe enlargement may benefit from
table 37. Medical Therapies for Patients with Moderate less invasive techniques. Transurethral resection of the
to Severe Symptoms of Benign Prostatic Hyperplasia prostate (TURP) has been the usual surgery of choice, with
a-Blockers studies showing symptom improvement in 70% of" men and
average time to recurrent treatment of 3 years. Although
Alfuzosin
long-term results from trials of" minimally invasive surgical
Doxazosin
interventions, such as transurethral needle ablation (TUNA)
Tamsulosin and transurethral microwave thermotherapy (TUMT) show
Terazosin good initial results, the time to recurrent intervention is
Silodosin shorter, and the choice of" procedure should balance die risk
of recurrence with die decreased invasiveness and fewer side
5</-Reductase inhibitors
effects associated with these procedures. Studies are not suf
Dutasteride ficient to recommend any particular intervention.
Finasteride KEY POINT
Anticholinergic agents Overactive bladder symptoms related to benign prosta
Oxybutynin tic hyperplasia include nocturia and urinary frequency
and urgency; bladder oudet obstruction symptoms
Tolterodine
include decreased urinary stream, urinary retention,
Combination therapy
incomplete bladder emptying, and incontinence.
a-Blocker and 5a-reductase inhibitor
a-Blocker and anticholinergic agents
Acute Testicular and Scrotal Pain Q]
Phosphodiesterase type 5 inhibitor
Diagnosis of acute scrotal pain can often be made on the basis
Tadalafil of history and physical examination. The main sources of such
Information adapted from AUA Guidelines: www.auanet.org/content/guidelines-and- symptoms are testicular torsion and epididymitis. Orchitis is
quality-care/clinical-guidelmes.cfm?sub=bph. Accessed August 3, 2012. painful inflammation of the testicle itself and can be virallv

78
Men's Health

induced (mumps) or an extension of bacterial infections of Disease). Older men and men who practice anal intercourse
the urinary tract or epididymis. Other causes of scrotal pain are more susceptible to Escherichia coli, Enterobacteriaceae,
may include referred pain from abdominal aneurysm, inguinal and pseudomona! infection and should be treated with cef
hernia widi strangulation of bowel or omentum, nephrolithi triaxone and a fluoroquinolone. A 21-day course of a fluoro
asis, lumbosacral nerve impingement, and retroperitoneal quinolone antibiotic is appropriate for most odier causes of"
inflammation. infectious epididymitis.
The patient should be asked about the onset (sudden or Noninfectious epididymitis is caused by reflux of urine
gradual), quality and severity of" pain, history of trauma, and into the epididymis, which causes inflammation. Treatment is
recent sexual activity, as well as lower urinary tract symptoms conservative, with scrotal support, application of ice, and
such as urinary frequency, urgency, and dysuria. Physical NSAIDs.
examination should include inspection and palpation and KEY POINTS
transillumination of the testes. The cremasteric reflex Urinalysis is indicated in the evaluation of acute scro
(obtained by stroking the upper inner thigh and observing a tal pain to assess for infection.
rise in the ipsilateral testicle) is absent in most patients with
testicular torsion. Urinalysis is indicated in the evaluation of Testicular torsion, characterized by sudden pain and
acute scrotal pain to assess for infection. Doppler ultrasonog an elevated, high-riding, abnormally oriented testis, is
a surgical emergency.
raphy to assess blood flow can be a useful adjunct aiding in
diagnosis and in one study was 82% sensitive and 100% spe
cific for torsion.
Hydrocele and Varicocele
Testicular Torsion One percent of adult males have a hydrocele, which is a fluid
Testicular torsion constitutes an emergency and occurs when collection between die layers of" the tunica vaginalis. A com
the testes twist on die spermatic cord, leading to decreased municating hydrocele occurs when a patent processus vagi
blood flow and ischemia. It is more common in children and nalis allows fluid to pass from the peritoneal space into the
in men younger than 30 years. Pain is usually sudden in onset scrotum, whereas a simple hydrocele has no such connection.
and often accompanied by nausea and vomiting. Physical Hydroceles vary in size and often develop gradually. Although
examination often reveals an elevated, high-riding testis with most are asymptomatic, larger hydroceles can become painful.
longitudinal access abnormally oriented transversely. Physical examination usually reveals a tense, smooth, scrotal
Considerable edema may bc present. Elevation of the testis mass diat is easily transilluminated. Transillumination can help
hurts more in torsion, whereas epididymal pain may be distinguish a hydrocele from a hernia, varicocele, or solid
relieved by diis maneuver. Treatment of"torsion includes rapid mass. If a question remains, ultrasonography is the preferred
surgical decompression to resume blood flow. In the absence modality for evaluation.
of rapid access to surgery, manual decompression may be In adults, hydroceles diat are small and do not bother die
attempted. patient do not require intervention. Large or painful hydro
celes can be treated either surgically or widi aspiration and
Epididymitis sclerotherapy. Communicating hydroceles usually require
Infection or inflammation of the epididymis often causes pain correction.
localized to the superior and posterior aspect of" the testicle.
The onset may be acute, subacute, or chronic, and pain may Epididymal Cyst
occur more gradually compared with torsion. Pain may bc Epididymal cysts are often palpated at die head of" the epi
accompanied by lower urinary tract symptoms of" dysuria, didymis. Cysts larger than 2 cm are considered spermatoce
urgency, or frequency. Epididymitis may be infectious or non les. Cysts can be palpated separately from the testis and usu
infectious, with infectious epididymitis more likely to be ally are not painful. Ultrasonography is the study of choice for
acute. Patients with acute infectious epididymitis may be quite definitive diagnosis. Surgical repair is recommended only in
sick, widi high fevers and leukocytosis. There may be con rare cases of" chronic pain related to the cyst.
comitant prostatitis. Subacute presentations are more com
mon and may not bc accompanied by lower urinary tract Varicocele
symptoms. A varicocele is a dilation of the testicular vein and pampini
Risk factors for epididymitis include recent sexual activ form plexus within die scrotum; 90% occur on the left side.
ity, heavy exertion, and bicycle riding. Distribution is Varicoceles are one of" die most commonly identified scrotal
bimodal, with occurrences highest in those younger than 35 abnormalities, found in 15% of"adult men and in 40% of"men
years and older than 55 years. In younger patients, sexually with infertility. The classic description is a scrotal mass on die
transmitted diseases such as chlamydial infection and gonor left side with a "bag of worms" consistency diat increases in
rhea are the most likely causes (see MKSAP 16 Infectious size with standing and decreases while supine. Varicoceles are

79
Men's Health

often painless and discovered during adolescence by self- masses, hemorrhoids, and neurologic diseases of the bladder.
examination. Ultrasonography is the test of choice if the diag Urinalysis and urine culture should be performed, although
nosis is in question. die results are usually negative. Leukocytes may be found in
Varicoceles are a leading cause of infertility. Repair may post-prostatic massage urine, expressed prostatic secretions,
be warranted in men with abnormal sperm counts who or semen, but the presence of leukocytes is not correlated
desire children, although a Cochrane review did not confirm with treatment response.
increased fertility with repair. Conservative care is adequate A four-stage classification system was developed by the
for men who do not desire children and are otherwise NIH to describe prostate syndromes (Table 38). Patients
asymptomatic. with acute bacterial prostatitis (class I) present with fever and
Surgical repair techniques include open inguinal varico urinary symptoms and often have an exquisitely tender
celectomy, laparoscopic varicocelectomy, and subinguinal prostate gland upon digital rectal examination. These infec
microscopic varicocelectomy. All of diese techniques are tions generally respond well to standard antibiotic treat
effective at reversing abnormal sperm parameters but not nec ment (fluoroquinolones, trimethoprim-sulfamethoxazole).
essarily fertility. Sclerotherapy is a less invasive option that is Chronic bacterial prostatitis (class II) presents with pain and
associated with fewer complications but may be less effective urinary symptoms with recurrent bacterial infection; an
in reducing symptoms. extended course (1 month) of a fluoroquinolone is typically
KEY POINT the first-line treatment.
In adult men, asymptomatic hydroceles and varico Treatment of class III syndromes, which present with
celes can usually be diagnosed clinically and generally pain or LUTS, is difficult and often refractory to interven
tion, a-Blockers may help control urinary symptoms but
do not require advanced imaging or treatment.
have less of"an effect on pain. Antibiotics are often used in
the treatment of inflammatory chronic pelvic pain syn
drome (CPPS) despite studies that fail to show long-term
Acute and Chronic Prostatitis
efficacy. For inflammatory CPPS, AUA guidelines recom
and Pelvic Pain mend one course of antibiotics but not recurrent courses
Acute and chronic prostatitis and pelvic pain are common, if the first is ineffective. NSAIDs have also been recom
with up to 2 million physician visits annually attributed to mended based on a trial of rofecoxib, which provided some
these syndromes. Symptoms fall into two categories: pain reduction in symptoms.
Symptoms, which may include the perineum, testes, penis, or Other agents that have been studied for the treatment of"
suprapubic area; and urinary symptoms, including dysuria, CPPS include muscle relaxants and finasteride. Agents used in
urinary frequency, and incomplete bladder emptying. The the treatment of"neuropathic pain, such as gabapentin, have
NIH's Chronic Prostatitis Symptom Index can aid in diag been recommended when primary therapies are ineffective.
nosis and can bc used to follow symptoms objectively over Complementary therapies that may be helpful in some
time (www.prostate.net/wp-content/uploads/pdf/chronic patients include cernilton, a pollen extract; quercetin, a natu
-prostatitis-symptom-test.pdf). ral bioflavonoid; and saw palmetto extract, but the efficacy of
The diagnosis of chronic prostatitis and pelvic pain is one these agents has not been determined. Surgical interventions
of clinical exclusion. Examination should rule out active ure for CPPS should be avoided unless there is a structural cause
thritis, urethral stricture, testicular sources of pain, rectal for symptoms, such as urethral stricture or BOO.

table 38. Classification of Prostatitis

Category Definition
I. Acute bacterial prostatitis Acute infection of the prostate
II. Chronic bacterial prostatitis Recurrent infection of the prostate
III. Chronic abacterial prostatitis/chronic pelvic No demonstrable infection
pain syndrome (CPPS)
IMA. Inflammatory CPPS Leukocytes in semen, expressed prostatic secretions, or post-prostatic massage urine
1MB. Noninflammatory CPPS No leukocytes in semen, expressed prostatic secretions, or post-prostatic massage
urine
IV. Asymptomatic inflammatory prostatitis No symptoms; detected either by prostate biopsy, or the presence of leukocytes in
semen samples during evaluation for other disorders

Adapted from Litwin MS, McNaughton-Collins M. Fowler FJ Jr., et al. The National Institutes of Health chronic prostatitis symptom index: development and validation of a new outcome meas-
ure. Chronic Prostatitis Collaborative Research Network. J Urol. 1999 162(2)369-375. [PMID: 10411041) with permission from American Urological Association. Copyright 1999. American
Urological Association.

80
Women's Health

KEY POINTS
Women's Health
Acute (class I) and chronic bacterial (class II) prosta
titis are treated with antibiotic therapy. Female Sexual Dysfunction
Chronic prostatitis/pelvic pain syndromes (class III)
Approach to the Patient
are often refractory to intervention; treatment
Female sexual dysfunction, defined as sexual difficulties diat are
options include a-blockers, a single course of anti
persistent and personally distressing to the patient, affects up to
biotics, and NSAIDs. 35% of"sexually active women, peaking in middle-age. Because
women are often uncomfortable initiating discussions about
sexual problems, it is incumbent upon the practitioner to rou
Hernia tinely ask about sexual function, even during brief office visits.
A hernia occurs when an organ bulges through an area of* Asking if a patient is sexually active and if there are any related
weakness in muscle or connective tissue. They most com problems, including pain with intercourse, is appropriate. For
monly occur in die groin, and the incidence is higher in men individuals with potential issues, a complete sexual history
than women. Hernias also may occur in die abdomen (ven should be obtained, including a thorough review of medical
tral and umbilical) and at incision sites. Hernias may be problems, psychiatric disorders, and reproductive surgeries that
caused by acquired or congenital weakness of connective or can contribute to sexual dysfunction. Patients should be queried
muscle tissue and by increased intra-abdominal pressure about die use of" medications that may interfere widi sexual
owing to straining, heavy lifting, or cough. function. Table 39 lists additional questions that aid in identi
There are two types of inguinal hernia, direct and indirect. fying problems with desire, arousal, orgasm, or pain, which may
Direct inguinal hernias occur when intra-abdominal contents help in determining cause and potential treatment strategies. A
herniate through a weak spot in the fascia between die rectus pelvic examination is important and may be helpful in identify
abdominis and die inguinal ligament. Indirect inguinal hernias ing specific areas of" pain, vaginal atrophy, inadequate lubrica
occur when intra-abdominal contents protrude through the tion, and vaginismus. Laboratory testing is recommended only
internal inguinal ring. The less common femoral hernia occurs if" there is suspicion for a particular diagnosis, such as a pro
when intra-abdominal contents protrude dirough the femoral lactinoma, thyroid abnormalities, or adrenal disease.
canal. The vast majority of inguinal hernias arc indirect.
Clinical presentations of inguinal hernias can vary from an Classification of Female Sexual Disorders
asymptomatic bulge, to a feeling of groin or abdominal pres The DSM-IV identifies six disorders diat correspond to
sure, to severe pain owing to movement of abdominal contents abnormalities in the female sexual response cycle: hypoactive
into the inguinal canal and occasionally into die scrotum.
Hernias may be complicated by incarceration or strangulation, table 39. Essential Questions to Include in an
which can also lead to bowel obstruction. Direct hernias tend Assessment for Female Sexual Dysfunction
to present widi a low abdominal bulge and cause less pain and
How does the patient describe the problem?
less frequently develop incarceration, whereas femoral hernias
How long has the problem been present?
more commonly develop these complications.
Was the onset sudden or gradual?
Diagnosis is made by physical observation and digital
examination of the inguinal canal, widi a bulge diat is often vis Is the problem specific to a situation/partner or is it
ible or palpable and is usually more prominent upon standing. generalized?
Were there likely precipitating events (biologic or situational)?
Asymptomatic inguinal hernias may be monitored,
whereas symptomatic hernias may require surgical repair. Are there any problems in the woman's primary sexual
There is evidence that watchful waiting in asymptomatic relationship (or any relationship in which the sexual problem
is occurring)?
patients ma)' be acceptable, but patients should be warned of"
Are there current life stressors that might be contributing to
potential complications, such as incarceration and strangula sexual problems?
tion. The optimal type of surgery (laparoscopic versus open
Is there guilt, depression, or anger that is not being directly
surgical repair) remains controversial, although reviews indi acknowledged?
cate lower recurrence rates when polypropylene mesh is used. Are there physical problems such as pain?
Potential complications of repair include early recurrence,
Are there problems in desire, arousal, or orgasm?
infection, seroma, hematoma, chronic neuropathic pain, and
Is there a history of physical, emotional, or sexual abuse?
late recurrence.
Does the partner have any sexual problems?
KEY POINT
With kind permission from Springer Science+Business Media: Int Urogynecol J Pelvic
Asymptomatic inguinal hernias may be monitored; Floor Dysfunct. Evaluation and treatment of female sexual disorders. 2009;20 Suppl
1:S33-S43, Kingsberg S. Althof SE. [PMID: 19440781] Copyright 2009. Springer
symptomatic hernias require surgical repair. Science+Business Media.

81
Women's Health

sexual desire disorder and sexual aversion disorder (sexual Orgasmic Disorder
desire disorders), sexual arousal disorder, orgasmic disorder, Female orgasmic disorder is the persistent or recurrent delav
and dyspareunia and vaginismus (sexual pain disorders). or absence of" orgasm following a normal excitement phase.
The Female Sexual Function Index (wwvv.fsfi-question- Cognitive-behavioral therapy is most effective for teaching a
naire.com) is a validated 19-item, patient-completed ques woman to be comfortable, minimizing negative attitudes, and
tionnaire that assesses each of these areas of sexual function decreasing anxiety.
ing and can be used as an adjunct in the clinical assessment.
Women who score less than 26 are considered to be at risk for Sexual Arousal Disorder
sexual dysftinction. Female sexual arousal disorder is the inability to complete sex
ual activity with adequate hibrication and absent or impaired
Sexual Desire Disorders genital responsiveness to sexual stimulation. Cognitive-behav
Hypoactive sexual desire disorder (HSDD) is diagnosed if a ioral therapy widi sensate focus and training to improve partner
woman reports a persistent lack of sexual thoughts or desire communication are effective strategies. Systemic or local estro
for or receptiveness to sexual activity. HSDD is the most gen therapy can increase lubrication in postmenopausal women;
common female sexual disorder, affecting 12% to 19% of in premenopausal women, vaginal moisturizers can be helpful.
U.S. women. Desire encompasses diree separate compo
nentsdrive, cognition, and motivation. Drive, perceived as Sexual Pain Disorders
spontaneous sexual interest, is biological and is based on Dyspareunia is persistent urogenital pain diat occurs around
neuroendocrine functions. Cognition refers to the belief and intercourse and is not related exclusively to inadequate lubrica
value framework that a patient has regarding sex. Motivation tion or vaginismus. Therapy is aimed at identifying and treating
is the willingness to engage in sexual activity and may be die underlying cause, which may include vtilvodynia, interstitial
influenced by the quality of the relationship and several psy cystitis, pelvic adhesions, infections, endometriosis, and pelvic
chosocial factors. venous congestion. Coexisting vaginal atrophy and inadequate
Testosterone is essential for normal sex drive and may lubrication may worsen die pain syndrome and usually can be
also play a role in sexual motivation. Levels of testosterone diagnosed on physical examination. Vaginal estrogen frequently
decline progressively with age in women; however, meas improves atrophy, whereas systemic estrogen or estrogen-prog
ured testosterone levels have not been shown to correlate esterone therapy can increase vaginal blood flow and lubrica
meaningfully with sexual functioning. Despite this, treat tion. Successful treatment strategies must address the complex
ment with testosterone has been shown to increase sexual psychological and behavioral changes that accompany diis syn
function scores and the number of satisfying sexual episodes drome, as well as die desire and arousal disorders diat often
but is associated with adverse effects, including excess hair develop as a result of the painful sexual experience.
grovvdi and acne, and altered lipoprotein levels with oral Vaginismus is the involuntary, recurrent, and persistent
therapy. In 2006, the Endocrine Society acknowledged the spasm of die outer third of the vagina, preventing desired
short-term efficacy of testosterone therapy for the treat
vaginal penetration. Episodes may be situation-specific, for
ment of" sexual dysftinction in women but recommended example, pelvic examinations or intercourse. The anticipation
against generalized use because of the lack of" long-term of" pain with vaginal entry underpins this diagnosis and may
safety data. The low estradiol levels that characterize surgi result in sexual avoidance. Treatment involves cognitive-
cal and natural menopause may also be linked to lack of behavioral therapy to help the patient feel safe, calm, and in
desire and sexual responsiveness. Although study results are control of the encounter, diereby reducing her anticipatory
inconsistent, systemic estrogen or estrogen-progesterone response. Systematic desensitization teaches deep muscle
therapy (EPT) may increase sexual desire, enjoyment, and relaxation and uses objects of" increasing diameter, such as
orgasmic frequency. dilators, to achieve gradual vaginal tolerance.
To date, die FDA has not approved any medication for
KEY POINT
the treatment of female HSDD. Individual and couples sex
therapy or psychotherapy may be beneficial. Laboratory testing is recommended in the evaluation
Sexual aversion disorder is defined as a persistent or of female sexual dysfunction only if there is suspicion
recurrent aversion to any genital contact with a sexual part for a particular diagnosis.
ner. Women with sexual aversion usually experience feelings
of abhorrence and revulsion; panic may accompany specific
sexual situations. Avoidance of sexual behavior typically rein Evaluation of a Breast Mass
forces the aversion, so treatment often involves graduated re- Breast symptoms in women are common. In a retrospective
introduction of" sexual behavior and relaxation exercises, study of 2400 primary care patients followed for 10 years,
which may be augmented by therapy with a selective sero 16% of women ages 40 to 79 years presented with a "breast
tonin reuptake inhibitor. lump" or "lumpiness." Whether discovered by the patient or

82
Women's Health

detected by a physician, a palpable breast mass requires a sys table 40. Differential Diagnosis of Breast Mass
tematic and logical evaluation. The performance of a careful
Cysts
history and physical examination may lead to a likely diagno
sis. However, in most cases, further testing (imaging, tissue Simple

sampling) is necessary. Complex


Infection
Clinical Presentation
Abscess
A breast mass or lump is a discrete, firm, three-dimensional
Mastitis
abnormality that is different from the surrounding breast tis
sue. The differential diagnosis of"a palpable breast mass is out Tuberculosis
lined in Table 40. The critical challenge in evaluating a breast Trauma
mass is distinguishing benign from malignant disease. Risk fac
Fat necrosis
tors for breast cancer should be determined (see MKSAP 16
Hematoma
Hematology and Oncology). History and physical examina
tion features that favor a benign lesion include younger age, Benign tumors
absence of" breast cancer risk factors, normal overlying skin, a
Fibroadenoma
milky (vs. bloody) nipple discharge, change in size during men
Hamartoma
strual cycles, and a mass diat is round, mobile, and soft.
Whereas pain is more likely to be associated with a benign Papilloma
lesion, the presence of pain does not rule out malignancy. Phyllodes tumor (benign variant)
Although most breast masses (up to 90%) are benign Lipoma
cysts or fibroadenomas, neither the history nor the physical Adenoma (tubular or lactating)
examination can definitively rule in or rule out underlying Granular cell tumor
malignancy with sufficient accuracy. In a study of" 201 refer Neurofibroma
ral patients with palpable solid breast masses, the sensitivity of
the physical examination for the detection of breast cancer Hemangioma
was 88%. The positive and negative predictive values for Malignant tumors
breast cancer were 73% and 87%, respectively. Therefore, a Ductal carcinoma in situ
palpable breast mass requires further evaluation, including Invasive ductal or lobular carcinoma
imaging and/or tissue sampling, for definitive diagnosis.
Phyllodes tumor (malignant variant)
Physicians should pursue all breast findings until resolution.
Squamous cell carcinoma
Evaluation Sarcoma

Mammography and ultrasonography arc the best imaging Angiosarcoma


modalities for evaluating a palpable breast mass. The choice Leukemia
of ultrasonography versus diagnostic mammography in eval Lymphoma
uating a breast mass depends in part on the patient's age. Other
The increased density of" breast tissue in women younger
than 30 to 35 years may limit the utility of mammography, Diabetic mastopathy
making ultrasound a better first choice. Ultrasonography Sarcoidosis
may also be a better choice in the pregnant patient as ultra Granulomatosis with polyangiitis (also known as Wegener
sonography avoids potentially harmful radiation exposure. granulomatosis)
The main utility of" ultrasonography is its ability to differen Idiopathic granulomatous mastitis
tiate cystic from solid lesions. A simple cyst with symmetric Gynecomastia (in men)
round borders and no internal echoes that, with aspiration,
Reproduced with permission from Ganschow PS, et al. Breast Health and Common
drains nonbloody fluid and disappears completely, is likely to Breast Problems. American College of Physicians. Philadelphia, PA. 2004:159. Copyright
be benign. If" bloody fluid is obtained from cyst aspiration, 2004, American College of Physicians.

it should be sent for cytology. A solid lesion with uniform


borders and uniformly sized internal echoes is most likely a
benign fibroadenoma. Fibroadenomas should be followed In women older than 30 to 35 years, the test of choice
to assure that they spontaneously regress or decrease in size. is diagnostic mammography. An irregular mass with micro
Alternatively, depending on the preference of the patient, calcifications and spiculations is suspicious for malignant dis
they can be surgically removed. ease, and biopsy is mandatory in these cases (see below).

83
Women's Health

Approximately 10% to 20% of palpable breast cancers are Treatment


undetected by ultrasonograph}' or mammography. Therefore, Women with cyclical mastalgia benefit from education and
the evaluation of a palpable breast mass suspicious for malig reassurance, as most will experience spontaneous resolution of"
nancy should continue with biopsy even if the mammogram their symptoms. Medical treatment may be considered for
or ultrasound is unrevealing. women with severe and persistent pain that interferes with
After palpation and imaging, definitive diagnosis is
quality of"life. Danazol (100 mg twice daily) is die only ther
obtained through tissue sampling. Tissue sampling proce
apy that has been FDA-approved for the treatment of"cyclical
dures include fine needle aspiration, core needle biopsy with
mastalgia, but side effects, including menorrhagia and weight
or vvidiout stereotactic or ultrasound guidance, or excisional
gain, often limit its use. Patients with cyclical mastalgia expe
biopsy. Fine needle aspiration, generally reserved for cystic rience benefit with tamoxifen (10 mg/d), and treatment-
lesions, is operator-dependent and requires an experienced associated hot flushes and menstrual irregularities are rela
cytopathologist for interpretation. Core needle biopsy,
tively infrequent.
although more costly and with a greater risk for postproce
dure hematoma, provides better tissue sampling for patho
logic examination and hormone receptor status (if positive Contraception
for cancer) and can distinguish in situ versus invasive cancer.
Core needle biopsy is the test of choice for most solid Nearly half of pregnancies in the United States are unin
tended. Adherence to contraception may be improved by
lesions. Excisional biopsy is used when core needle biopsy is
counseling patients at risk for unintended pregnancy regard
nondiagnostic or when biopsy and imaging studies are not
in agreement. Further management of abnormal pathology ing appropriate contraceptive options, especially patients with
serious medical conditions in whom the risk of adverse events
requires consultation with a breast surgeon and oncologist. of* unintended pregnancy is high. Contraceptive options arc-
KEY POINT
compared in Table 41.
When imaging a palpable breast mass, ultrasonogra
phy is preferred for pregnant women or women Oral Contraceptive Pills
younger than 30 to 35 years; mammography is pre Oral contraceptive pills (OCPs) include combination estro
ferred for women older than 30 to 35 years. gen-progesterone products and progesterone-only pills.
Differences in combination preparations lie in the type and
strength of" estrogen and in the androgenicity of their prog
Breast Pain esterone component. Combinations with lower estrogen
Clinical Presentation dose are as effective with fewer side effects. The mecha
Breast pain may be characterized as cyclical, noncyclical, or nisms of" action of combination pills include inhibition of"
extramammary. Although most women experience mild ovulation, alteration of the cervical mucus to an environ
breast discomfort with the onset of menses, cyclical mastalgia ment less conducive to sperm migration, and inhibition of"
endometrial proliferation. Most hormonal methods are
typically lasts for several days and is moderate to severe in
intensity. Nonq'clical mastalgia has no relationship to die highly effective; annual failure rates are 0.3% with "perfect"
menstrual cycle and may be caused by pregnancy, medications use, 8% with typical use. Combined products are also avail
(nicotine, hormone therapy), stretching of Cooper ligaments able as a patch and a vaginal ring. Contraindications to
(secondary to large breasts), or cancer. Extramammary breast combination products include history of dirombosis, liver
pain may be caused by musculoskeletal, cardiac, gastroin disease, breast cancer, migraine with aura, and uncontrolled
testinal, or spinal disorders. Chest wall inflammation, a com hypertension. Women older than 35 years who smoke
mon cause of extramammary pain, typically presents with uni more than 15 cigarettes a day should not be prescribed
lateral, localized, burning discomfort. estrogen-containing preparations. Progesterone-only pills,
also called the "mini-pill," may be used by women with
Evaluation contraindications to estrogen.
Breast cancer may be associated with mastalgia; therefore, a Medications that induce the CYP3A class of liver
thorough physical examination is essential in all women pre enzymes may reduce die effectiveness of" hormonal contra
senting widi breast pain. Women widi a palpable breast mass ceptives. Common drugs with this effect include barbiturates,
should be referred for diagnostic imaging. Chest wall pain is carbamazepine, many antiseizure medications, rifampin, and
typically reproduced by palpation or by examination maneu certain antiretroviral agents. In addition, some antibiotics and
vers that place stress on the inflamed musculoskeletal struc some other neurologic medications may alter efficacy of"com
tures. All women who are evaluated for mastalgia should be bination preparations via other mechanisms. Additional con
up to date on routine mammographic screening, according to traception or barrier mediods should be used concurrently
age and personal risk factors for breast cancer. and for 4 weeks after discontinuing such drugs.

84
Women's Health

TA B L E 41. Comparison of Contraceptive Options 1

Agent Failure Rate3 Advantages Disadvantages


(perfect use/
typical useb)

Combination estrogen- Decreased incidence of Increased risk of myocardial infarction,


progesterone preparations endometrial, ovarian cancers ischemic stroke, VTE, hypertension
Decreased dysmenorrhea, Increased risk of cancers of the cervix, liver,
menorrhagia, symptomatic breast
ovarian cysts
Breakthrough bleeding.
Less iron-deficiency anemia

Oral 0.3/8 May worsen acne, exacerbate migraine


Patch 0.3/8 Easier compliance Local skin reaction
Increased estrogen dose, thus higher VTE risk

Vaginal ring 0.3/8 Easier compliance Requires self-insertion


Lowest level of systemic estrogen

Progesterone-only Use when estrogen is May worsen acne


preparations contraindicated

Mini-pill Must maintain precise daily dosing schedule

Long-acting preparations Irregular bleeding, amenorrhea, decreased


bone mineral density (especially in
adolescents)

Depot medroxyprogesterone 0.3/8 Administered every 3 months Delayed return to ovulation (10 months)
acetate (IM or SQ)
Decreased risk of endometrial
cancer, PID
Improves endometriosis
Progesterone implants 0.8-1/ Effective up to 3 years Delayed return to ovulation (6 months)
Intrauterine devices Least dependence on user Bleeding, pain, expulsion (rare); no protection
from STIs

Copper 0.6/0.8 Nonhormonal


Effective up to 10 years

Levonorgestrel 0.1/0.1 Decreased blood loss, decreased anemia


Effective up to 5 years
Barrier methods Only use when needed Most user-dependent

Cervical cap 9-26/16-32 Requires spermicide


Diaphragm 6/16 Requires spermicide
Male condom 2/15 Protection from STIs
Female condom 5/21 Protection from STIs

Vaginal sponge 6-9/16-32


Sterilization
Female (tubal ligation) 2% in 10 years May reduce ovarian cancer risk Surgical complications
Regret
Increased risk of ectopic if pregnancy occurs

Male (vasectomy) <1% in 10 years Lower costs, fewer complications, Surgical complications
and more effective than tubal
ligation

PID = pelvic inflammatory disease; SQ = subcutaneous; STI = sexually transmitted infection; VTE = venous thromboembo sm.

Per 100 women/year.

"Perfect use implies correct and consistent use exactly as directed/intended. Typical use reflects rates in actual practice with patients.

85
Women's Health

Long-Acting Contraceptives greater efficacy and fewer side effects (nausea, vomiting,
Long-acting progesterone compounds include depot dizziness, headache). If" vomiting occurs within 2 hours of
medroxyprogesterone acetate (DMPA) injections, subcuta taking emergency contraception, the dose should be
neous progesterone implants, and progesterone-containing repeated, given vaginally, or given after an antiemetic has been
intrauterine devices (IUDs). DMPA may be administered used. These mediods work by delaying or preventing ovula
either intramuscularly or subcutaneously following a negative tion, and are not abortifacients. They may be taken up to 5
pregnancy test. Etonogestrel implants are small rods inserted days (120 hours) post-intercourse. The copper IUD is
subdermally in the upper inner arm and are effective for 3 inserted up to 5 days after intercourse. It functions primarily
years. These long-acting progesterone methods are less reliant by stopping fertilization and interfering with implantation. It
on user compliance, and therefore may be particularly effec may remain in place as long-term contraception.
tive in patients who may have difficulty with daily treatment Mifepristone, a progesterone-receptor modulator, is
or barrier mediods. They are not recommended for women approved by the FDA as an abortifacient, but has been used
desiring pregnancy in the subsequent 2 years after cessation outside the United States as emergency contraception.
of" treatment because of potential contraceptive-associated Ulipristal, another progesterone-receptor modulator with
delayed fertility. agonist and antagonist effects, has been approved for use as
emergency contraception in the United States.
Intrauterine Devices
IUDs are the most commonly used form of" reversible Sterilization
contraception worldwide. The copper IUD and the Sterilization is highly effective, widi an annual failure rate of
levonorgestrel-containing IUD may be placed at any time 1 in 1000. Female sterilization (tubal ligation) incurs more
(except during pregnancy) in an office setting without need risk and cost than vasectomy and is less effective. Incidence of"
for anesthesia. IUDs combine the highest efficacy (typical fail regret and requests for reversal of tubal ligation are signifi
ure rate <1%) with die lowest cost. The copper IUD may be cantly higher in women younger than 30 years and in diose
implanted within 7 days of unprotected intercourse for use as who are postpartum or postabortion at the time of" the pro
emergency contraception. cedure.
KEY POINTS
Barrier Methods
Women older than 35 years who smoke more than
Barrier mediods (see Table 41) allow for "as-needed" con
15 cigarettes a day should not be prescribed estro
traception but are much less reliable than hormonal methods.
All barrier methods are more effective when used widi sper gen-containing contraceptives.
Intrauterine devices are highly effective at preventing
micides; spermicides alone are not considered reliable. All bar
rier methods reduce risk of sexually transmitted infection pregnancy and are low cost.
(STI); evidence is strongest for prevention of HIV infection
with male condoms. Limited evidence suggests barrier meth
ods protect future fertility, perhaps by reducing STIs. Preconception Counseling
Combining a barrier method with a hormonal method is rec Adequate preconception care can significandy reduce the risk
ommended when pregnancy must be avoided (for example, for preterm birth and birdi anomalies. Each health care visit
during use of teratogenic agents) and in adolescents, in whom with a reproductive-age woman represents an opportunity for
risk of"both unintended pregnancy and STI is very high. preconception counseling; providers should routinely ask if"a
patient is either considering pregnancy or could possibly
Emergency Contraception become pregnant. For patients considering pregnancy, pre
Emergency contraception refers to contraception adminis conception risk should be assessed (Table 42) and brief inter
tered after intercourse but before implantation. Efficacy for all ventions undertaken to optimize reproductive health, includ
products increases the earlier they are used. Four methods are ing encouraging a healthy lifestyle; stressing the importance
available in the United States: levonorgestrel tablets, com of tobacco, alcohol, and drug cessation; and referring under
bined contraceptive tablets, die copper IUD, and proges weight or overweight patients for formal nutritional evalua
terone modulators. tion. Prescription medications should be changed, if possible,
The lcvonorgestrel-only, single-dose (1.5 mg) regimen is to minimize exposure to potential teratogens (Table 43).
the most widely used and most effective option, and is avail Consultation with specialists for co-management of" specific
able over-the-counter (by prescription for adolescents diseases in pregnancy, such as diabetes mellitus and epilepsy,
younger than 17 years). Other options include two-dose lev may be indicated.
onorgestrel (0.75 mg taken 12 hours apart), or two tablets of A physical examination and focused laboratory testing
a high-dose combination OCP (100 micrograms ethinyl can provide additional information regarding a patient's
estradiol, taken twice, 12 hours apart). Levonorgestrel has reproductive healdi and preconception risk. Measurements of"

86
Women's Health

table 42. Preconception Risk Assessment


Risk Category Specific Items to Assess

Reproductive awareness Desire for pregnancy, number and timing of desired pregnancies, age-related changes in
fertility, sexuality, contraception
Environmental hazards and toxins Exposure to radiation, lead, mercury
Nutrition and folic acid consumption Healthy diet, daily consumption of folic acid, restricting consumption of shark, swordfish, king
mackerel, and tilefish to fewer than 2 servings weekly (owing to high mercury content)
Genetics Family history of inherited genetic disorders
Substance abuse Use of tobacco, alcohol, illicit drugs
Medical conditions Seizure disorder, diabetes mellitus, hypertension, thyroid disease, asthma, HIV infection,
systemic lupus erythematosus
Medications Over-the-counter and prescription medications, potential teratogens
Infectious diseases and vaccinations Immunity to varicella, rubella, pertussis, tetanus; risk for hepatitis B
Psychosocial concerns Depression, interpersonal/family relationships, risk for abuse (physical, sexual, emotional)
Based on Johnson K, Posner SF. Biermann J, et al.; CDC/ATSDR Preconception Care Work Group; Select Panel on Preconception Care. Recommendations to improve preconception health and
health careUnited States. A report of the CDC/ATSDR Preconception Care Work Group and the Select Panel on Preconception Care. MMWR Recomm Rep. 2006;55(RR-6):1-23. [PMID:
16617292)

TA B L E 4 3 . Te r a t o g e n i c M e d i c a t i o n s C o m m o n l y in combination with tetanus and diphdieria, is recommended


Prescribed by Internists for all persons who have not been immunized as adults.
Supplementation with folic acid (400 micrograms/d) reduces
ACE inhibitors
the risk of" neural tube defects. Because these defects occur
Androgens, testosterone derivatives very early in gestation, when a woman might not even know
Carbamazepine she is pregnant, folic acid supplementation is generally rec
Fluoxetine, paroxetine ommended for all women who are of reproductive age.
Folic acid antagonists Higher doses of" folic acid may be appropriate in women who
Lithium have prior children with a neural tube defect, take antiseizure
medications, or who are obese.
Phenytoin
Primidone KEY POINT

Statins Preconception counseling should include risk assess


ment in the following areas: reproductive awareness,
Tetracycline
environmental hazards and toxins, nutrition and folic
Valproic acid
acid supplementation, genetics, substance abuse,
Vitamin A derivatives: isotretinoin, retinoids, etretinate
medical conditions, medications, infectious diseases
Warfarin and vaccinations, and psychosocial concerns.
Adapted from Berghella V, Buchanan E, Pereira L. Baxter JK. Preconception Care. Obstet
Gynecol Surv. 2010;65(2); 119-131. [PMID: 20100361)

Menopause
Overview
BMI and blood pressure are essential, and the pelvic exami
nation may include collecting specimens for cervical cytology Menopause is defined as the permanent cessation of menses
and chlamydial testing (if indicated per screening recommen and is diagnosed retrospectively, after a woman has experi
dations for these tests). All women who are considering preg enced amenorrhea for 12 months. Menopause occurs

nancy should be routinely assessed for immunity to varicella through a series of stages characterized according to varia
and rubella, and screening for HIV may be considered. tions in the menstrual cycle and alterations in follicle-stimu
Certain interventions optimize pregnancy outcomes in all lating hormone (FSH) levels. Women typically begin to expe
rience irregular menses in early perimenopause, with cycles of
reproductive-age women, including appropriate vaccinations,
variable length, and then progress into late perimenopause,
daily folic acid, and proper diet and exercise. Vaccination for
rubella and varicella to nonimmune women should be admin with intervals of amenorrhea lasting more than 60 days and
istered at least 4 weeks before conception to minimize risks to two or more skipped cycles. FSH levels begin to rise in this
die fetus. Influenza vaccination is appropriate for those who perimenopausal transition but may fluctuate significantly
will be pregnant during flu season, and pertussis vaccination, depending on the frequency of" anovulation. After 1 year of"

87
Women's Health

amenorrhea (menopause), a woman is considered to be early All women widi an intact uterus who are treated with
postmenopausal, and FSH levels become very elevated (FSH hormone therapy must receive progesterone to avoid estro
>40 units/L, estradiol <20 ng/mL). Late postmenopause gen-induced endometrial hyperplasia and cancer. Several
starts 5 years after the last menstrual period. On average, preparations are available and may be given continuously or
women in the United States experience menopause at the age cyclically. Women treated with cyclic progesterone may have
of 51 years. occasional withdrawal bleeding and should be counseled
The hallmark symptoms of the menopausal transition are regarding this effect. Absolute contraindications to hormone
hot flushes and night sweats (vasomotor symptoms) and vagi therapy include pregnancy; unexplained vaginal bleeding; a
nal dryness and dyspareunia (urogenital symptoms). A hot history of coronary artery disease, stroke, thromboembolic
flush is characterized by die sudden onset of" intense warmth disease, or breast or endometrial cancer; hypertriglyceridemia;
that starts in die face or chest and then spreads diroughout recent vascular thrombosis or cardiovascular event; and
die body; it is usually associated with sweating and palpita immobilization.
tions. Night sweats arc hot flushes that occur during sleep and Table 44 lists a stepwise approach for initiating hormone
often result in nocturnal awakenings. Up to 50% of women therapy in women aged 50-59 years who have experienced
experience hot flushes during the menopausal transition, but menopause at the median age (51 years). In general, women
symptoms typically resolve spontaneously within a few years should be treated with the lowest possible dose of" estrogen
of onset. Conversely, vaginal dryness and dyspareunia, which for the shortest amount of time necessary; treatment for
result from progressive estrogen depletion and subsequent longer than 5 years is not advised. Systemic hormone therapy
urogenital atrophy, become much more prevalent in late post- can be discontinued abruptly or tapered gradually; there is no
menopause. Mood changes, cognitive difficulties, and pri difference in the rates of"vasomotor symptom recurrence. The
North American Menopause Society (NAMS) suggests diat
mary sleep disorders may be more common in menopausal
bone density be measured if hormone therapy is discontinued
women, but these symptoms have not been definitely linked
to changes in hormone levels. The differential diagnosis of after several years of treatment.

menopausal symptoms includes diyroid disease, elevated pro


lactin levels, and pregnancy, and testing for these conditions Benefits and Risks of Systemic Hormone Therapy
The Women's Healdi Initiative (WHI) demonstrated that
may be considered in selected patients.
combination estrogen and progesterone therapy (EPT)
decreased the risk of osteoporotic fractures and colorectal
Management of Vasomotor Symptoms
cancer but increased the risk of" coronary heart disease
Women should be reassured that hot flushes and night sweats
are common in the menopausal transition and that sponta (CHD), venous thromboembolism, invasive breast cancer,
and stroke. In contrast, estrogen-only therapy increased the
neous resolution typically occurs within a few years. As smok
risk for stroke, but no other outcomes. Notably, the absolute
ing, obesity, and sedentary lifestyle are all risk factors associ benefits and risks associated widi treatment were quite small;
ated with hot flushes, behavioral changes may result in
among 10,000 women treated widi EPT, there were five
symptom improvement. fewer hip fractures and eight additional strokes.
Additional analyses of data from the WHI suggest diat
Systemic Hormone Therapy while the overall risk of CHD in postmenopausal women on
Systemic hormone therapy is the most effective treatment for hormone therapy is increased, the greatest risk is in older
moderate to severe vasomotor symptoms, and has FDA
women (ages 70-79 years) and is minimal in younger women
approval for diis indication. It may be considered as a treat
(ages 50-59 years) treated earlier in menopause. This has led
ment option for women who have a thorough understanding
NAMS to recommend that hormone therapy not be used as
of" the risks and benefits associated with therapy. Several for
primary prophylaxis for CHD among women at any age.
mulations of estrogen are currently available and include con
Hormone therapy may be used to treat menopausal symp
jugated equine estrogen, micronized 17 (3-estradiol, and toms in women ages 50 to 59 years or those within 10 years
transdermal 17 ^-estradiol. Most experts recommend starting of" menopause as there is minimal CHD risk; hormone ther
widi the lowest dose of estrogen that effectively relieves symp
apy should not be initiated in women older than 60 years.
toms and titrating up as necessary to achieve maximal relief. In contrast to CHD risk, die increased risk of" stroke
Although supporting data are limited, transdermal estrogen, observed with hormone therapy does not vary according to
as compared with oral estrogen, may be associated widi less
length of time since menopause onset. Notably, the risk of"
thromboembolic risk as it avoids the hepatic first-pass effect. stroke with hormone therapy use among women aged 50 to
Moreover, transdermal and oral estrogen seem to be equally 59 years did not differ significantly from that among women
effective for treating vasomotor symptoms. Common adverse older than 60 years.
effects of systemic estrogen therapy are breast tenderness and In a follow-up study of more than 12,000 women who
uterine bleeding. participated in the WHI, investigators found that EPT, as

88
Women's Health

table 44. Initiating Systemic Hormone Therapy in Women Ages 50-59 Years3

Step 1: Confirm that hot flushes/night sweats are moderate-severe in intensity and/or vaginal symptoms are moderate-severe in
intensity and have been refractory to local therapies.
Step 2: Assess for absolute contraindications to systemic hormone therapy.
Step 3: Assess the patient's baseline risk for stroke, cardiovascular disease, and breast cancer (consider using the Framingham stroke
risk score, Framingham CHD risk score, and Gail risk score to quantify this risk). If the Framingham stroke or CHD risk score is >10% or
Gail risk score is elevated, consider avoiding systemic hormone therapy.b'c
Step 4: Use the lowest dose of estrogen that relieves menopausal symptoms.
Step 5: Add systemic progesterone therapy to estrogen therapy in women who have an intact uterus.
Step 6: Assess symptoms and side effects within 4-6 weeks of initiating therapy. Increase the dose of estrogen if symptoms are
persistent.
Step 7: Reassess symptoms and risk factors for cardiovascular disease, stroke, and breast cancer regularly. Ideally, treatment with
systemic hormone therapy should continue for no more than 5 years.
Step 8: Discontinue systemic hormone therapy if the risks of treatment outweigh the benefits, if symptoms resolve spontaneously, or
according to patient preference. Therapy does not need to be tapered.
CHD = coronary heart disease.

'According to North American Menopause Society, systemic hormone therapy should be avoided in women older than 60 years who have experienced menopause at the median age (51
years). If a woman has experienced menopause later than the median age, these guidelines apply within the first 10 years of menopause.

bSome authors indicate that systemic hormone therapy is safe in women who have experienced menopause within the last 5 years and have a Framingham CHD risk score of 10%-20%.

The majority of participants in the Women's Health Initiative had a Gail risk score of less than 2%.

compared with placebo, increases the risk of node-positive atrophy. Clinically, women may report vaginal dryness, itch
invasive breast cancer and breast cancer mortality. The timing ing, dyspareunia, dysuria, and frequent urinary tract infec
of hormone therapy initiation may be influential: exposure to tions. Mild to moderate symptoms can be effectively treated
EPT during the early postmenopause is associated with an with vaginal moisturizers, and lubrication may case pain with
increased incidence of breast cancer, whereas exposure during intercourse. The FDA has approved the use of vaginal estro
late postmenopause is not. The increased breast cancer risk gen therapy for women who do not respond to these meas
associated with therapy (8 excess cases per 10,000 person ures or who have moderate to severe urogenital symptoms.
years) becomes apparent after 3 to 5 years of exposure to hor Several preparations are currendy available and include con
mone therapv-. Moreover, EPT increases breast density and jugated estrogen vaginal cream, estradiol vaginal cream,
may impede the interpretation of mammograms. Post-hoc vaginal estradiol tablets, and an estradiol vaginal ring.
analysis of die WHI data suggests that EPT may promote Although each of" these treatments is equally effective in
lung cancer among older smokers, but these results need fur relieving symptoms, low-dose vaginal estradiol tablets (10-
ther validation. 25 micrograms) and the estradiol vaginal ring (8-9 micro
The low absolute risk of adverse events supports the grams) are preferred over vaginal estrogen creams, as diey
option to prescribe hormone therapy for women with mod result in minimal systemic estrogen absorption. According
erate to severe vasomotor or urogenital symptoms who are at to the NAMS guidelines, progesterone therapy is typically
low risk for CHD, stroke, and invasive breast cancer. not indicated when low-dose local estrogen therapy is used
to treat vaginal atrophy.
Nonhormonal Therapy
KEY POINTS
Several nonhormonal therapies are available for women who
have contraindications to hormone dierapy or want to avoid Women with vasomotor menopausal symptoms diat
its attendant risks. Certain antidepressants (selective serotonin warrant hormone dierapy should be treated with die
lowest possible dose of estrogen for the shortest
reuptake inhibitors and serotonin-norepinephrine reuptake
inhibitors), gabapentin, and clonidine may reduce hot flush amount of time necessary.
frequency by one or two per day as compared with placebo. To minimize cardiovascular risk, hormone therapy
Red clover extract and black cohosh are ineffective for treat should not be initiated in women older than 60 years
ing hot flushes, and study results suggest mixed benefits with who have not previously received hormone dierapy
soy isoflavone extracts. and who experienced menopause at the median age.
Hormone therapy does not increase the risk for coro
Management of Urogenital Symptoms nary heart disease among women aged 50-59 years
Progressive estrogen depletion during the menopausal tran who have experienced menopause at the median age.
sition leads to thinning of the vaginal epithelium and vaginal

89
Women's Health

Abnormal Uterine Bleeding should be performed in women widi structural abnormalities


noted on examination, abnormal bleeding despite evidence of
Clinical Presentation and Evaluation
ovulatory cycles, or new-onset intermenstrual bleeding.
Abnormal uterine bleeding refers to bleeding diat is excessive In women with prolonged anovulation, exposure to
in frequency, duration, or amount, and is often described by
unopposed estrogen increases risk for endometrial carci
die pattern of occurrence and degree of flow. Polymcnorrhagia noma. Therefore, pelvic ultrasonography is indicated in
is bleeding occurring more than once every 24 days, die lower women older dian 35 years to assess endometrial stripe thick
limit of the average menstrual cycle. Oligomenorrhea is bleed
ness; an endometrial stripe 5 mm or thicker warrants endome
ing less frequently than ever)' 35 days, aldiough women are trial biopsy before initiation of medical therapy.
more likely to complain of too frequent than infrequent
Ultrasonography is also indicated in younger women in
menses. Metrorrhagia is irregular or intermenstrual bleeding. whom empiric medical dierapy has failed to control bleeding.
Menorrhagia refers to regular cycles widi excessive bleeding,
defined as mondily menstrual loss of more than 80 mL or
Management
bleeding for more than 7 days. Menomctrorrhagia is bleeding
Management of abnormal uterine bleeding should be
at irregular intervals with excessive flow.
directed toward die underlying cause. Anatomic or structural
Abnormal bleeding is usually characterized as ovulatory
abnormalities causing ovulatory bleeding generally require
or anovulatory. Ovulatory bleeding is cyclical and may be directed therapy.
caused by anatomic abnormalities (polyps, leiomyomas),
Management of"anovulatory bleeding is directed toward
bleeding disorders, medications that interfere with hemosta
restoring the hormonal balance and stabilizing the
sis, or uterine cancer. Anovulatory bleeding is usually unpre endometrium. Treatment depends on the contraceptive plans
dictable and of variable flow and duration owing to loss of of the patient. For women desiring fertility, a progestin such
cyclical hormonal influences on the endometrium; if" no as medroxyprogesterone acetate may be used for the last 2
anatomic or medical cause is identified for bleeding, anovula
weeks of each cycle to promote wididrawal bleeding. For
tory bleeding may be referred to as dysfunctional uterine women not desiring pregnancy, OCPs may be used, as well as
bleeding (DUB). Anovulatory cycles arc characterized by other contraceptive methods, including depot medroxyprog
estrogen-mediated endometrial proliferation without the sta esterone, die vaginal contraceptive ring, and levonorgestrel
bilizing effects of progesterone, resulting in endometrial IUD. For severe bleeding, short courses of" gonadotropin-
desquamation and erratic bleeding. Anovulation has many releasing hormone (GnRH) agonists or intravenous high-
potential causes, but also commonly occurs in women with dose estrogens may bc used. For patients who do not respond
out underlying medical or anatomic issues. to medical treatment, evaluation for alternative causes or sur
These terms apply only to women of reproductive age; in
gical treatment is appropriate.
postmenopausal women (absence of menses for 1 year), any For patients widi chronic diseases not odierwise amenable
uterine bleeding is abnormal and warrants fiirdier evaluation, to dierapy (for example, chronic kidney disease), treatment is
especially for endometrial cancer. Pcrimenarchal and peri-
symptomatic. NSAIDs can decrease uterine bleeding by up to
menopausal stages are often characterized by abnormal bleed 40% owing to the high concentrations of prostaglandins in die
ing patterns and are usually not cause for concern. endometrium. Danazol (200-400 mg/d) is approved for die
Initial evaluation includes history with attention to stres treatment of heavy or irregular bleeding.
sors, diet, exercise, weight changes, trauma, medications, and
substance abuse. Important historical factors for estimating KEY POINT

changes in blood loss include change in pattern widi flow In postmenopausal women, any uterine bleeding is
excessive for die patient, increase in number of pads or tam abnormal and warrants further evaluation, especially
pons used, leaking despite use of pads or tampons, and for endometrial cancer.
presence of clots. Additional history may give clues to an
underlying endocrine or bleeding disorder, chronic liver or
kidney disease, or STI. Physical examination should include a Dysmenorrhea
complete pelvic examination widi attention to pelvic pathol Dysmenorrhea is complicated or painful menstruation. It is
ogy; screening for cervical malignancy should be up-to-date. usually seen in adolescents and young adults and is the most
Pregnancy testing should be performed in all women with common gynecologic symptom in diis age group. In 90% of
abnormal uterine bleeding as pregnanq' is the most common
cases, dysmenorrhea is associated with normal ovulatory
cause of" a divergence from a normal bleeding pattern. An cycles and no pelvic padiology (primary dysmenorrhea). In
assessment for chronic diseases, including liver, kidney, die remaining 10% of cases, a secondary cause, such as
thyroid, autoimmune, and bleeding diathesis, should be per endometriosis, fibroids, or uterine padiology may be found.
formed as indicated. In adolescents and young adults, coag
Symptoms of dysmenorrhea include abdominal cramps,
ulation disorders are most common. Pelvic ultrasonography headache, nausea, and vomiting. Symptoms coincide with

90
Women's Health

onset of menses and last 1 to 2 days. Severity typically corre the anatomic pelvis, anterior abdominal wall at or below
lates widi die amount and duration of menstrual blood flow. the umbilicus, the lumbosacral back, or the buttocks, and
Initial evaluation includes a thorough history, with par is of sufficient severity to cause functional disability or
ticular attention to sexual activity and risks for abuse or infec necessitate medical care. Approximately 15% to 20% of
tion. Unless pelvic pathology is suspected (previous radiation, reproductive-age women have experienced CPP for more
trauma, infection, foreign body), treatment may begin with than 1 year. The most common causes of CPP are
out further evaluation. Effective treatments include NSAIDs endometriosis, pelvic adhesions, pelvic varices, interstitial
and cyclooxygenase-2 inhibitors. These agents block cystitis, and irritable bowel syndrome (IBS); many women
prostaglandins in the endometrial lining, inhibiting the have more than one diagnosis. Fifty percent to 80% of
inflammation, vasoconstriction, and uterine ischemia thought women with CPP have symptoms consistent with IBS. Risk
to be die etiology. For patients with incomplete relief of factors for CPP include physical and sexual abuse, pelvic
symptoms, use of"combined contraceptive therapy is effective; inflammatory disease, a difficult obstetric delivery, a his
pills, patches, or the vaginal ring may be used. Extended-cycle tory of abdominopelvic surgery, and other chronic pain
combined OCPs may be particularly useful for this indication. syndromes, such as fibromyalgia. Endometriosis, intersti
Other treatment options include long-acting progesterones; tial cystitis, and IBS all tend to worsen during the men
these options should be used widi caution in adolescents strual cycle; additional historical clues and tests that aid in
owing to die risk for osteoporosis from estrogen deficiency. diagnosis are listed in Table 45. Laboratory studies should
Women not responding to therapy should be suspected include a complete blood count, serum chemistries, ery
of" having a secondary cause. The most common ofdiese is
throcyte sedimentation rate, urinalysis and urine culture,
endometriosis, which causes both cyclic and noncyclic pain. and vaginal/endocervical swabs for culture. Transvaginal
Treatment options are similar; additionally, GnRH agonists
and aromatase inhibitors have been used but require calcium ultrasonography is helpful for identifying anatomic pelvic
and vitamin D supplementation. If pelvic padiology is sus pathology, and a normal study aids in providing reassur
ance. Laparoscopy may be indicated for evaluation of
pected or if symptoms are refractory, gynecologic referral may
be appropriate. severe symptoms of unclear etiology or identified pathol
ogy on examination or ultrasonography.
NSAIDs can be used as first-line short-term therapy for
Chronic Pelvic Pain most women with moderate CPP. Although evidence to sup
Chronic pelvic pain (CPP) in women is defined as non- port die use of antidepressants is limited, psychodierapy and
cyclic pain of" at least 6 months' duration that localizes to writing about the stress of"pelvic pain may be helpful.

table 45. Diagnosis and Treatment Strategies for the Most Common Causes of Chronic Pelvic Pain3
Characteristic History Diagnostic Strategies Treatment Strategies Prevalence
Etiology

Endometriosis Pain that may worsen with Empiric treatment with GnRH GnRH agonist therapy; Up to two-thirds
combination OCPs for women of women with
menstruation, dyspareunia, agonist therapy; diagnostic
with significant dysmenorrhea; CPP have
dysmenorrhea laparoscopy
laparoscopic surgical destruction endometriosis
of endometriosis lesions (stages
l-lll endometriosis)

Pelvic adhesions History of PID, endometriosis, Diagnostic laparoscopy Surgical adhesiolysis for severe 25%-50% of
abdominal or pelvic surgery adhesions women with CPP
have adhesions,
although causality
is usually not clear

Pelvic varices Dull, chronic pain worsened Combined transabdominal Medroxyprogesterone acetate May cause pain
with prolonged standing, and transvaginal ultrasound in up to 42% of
women with
improved by lying down and
elevating legs CPP

Interstitial Dysuria, urgency, frequency, Interstitial cystitis symptom Pentosan polysulfate sodium; 38%-85% of
index; cystoscopy tricyclic antidepressants; GnRH women
cystitis repeatedly negative urine
cultures agonist therapy may be helpful presenting to
gynecologists with
CPP may have this
diagnosis

CPP = chronic pelvic pain; GnRH = gonadotropin-releasing hormonec OCP = oral contraceptive pill; PID = pelvic inflammatory disease.

lrritable bowel syndrome, a common cause of CPP, is discussed in MKSAP 16 Gastroenterology and Hepatology.

91
Women's Health

Condition-specific treatment strategies are listed in Table infection, and genital herpes. Symptomatic patients may
45. GnRH agonists are effective in treating women with CPP report a thin, white or gray homogeneous discharge that has
associated with endometriosis or IBS and may improve pain a "fishy" smell.
control in women with interstitial cystitis. The American BV can be diagnosed if at least diree of the four Amsel
College of" Obstetricians and Gynecologists (ACOG) guide criteria are present: homogenous, thin, white discharge; vagi
lines recommend empiric treatment with GnRH agonists in nal pH greater than 4.5; fishy odor before or after the addi
women widi undiagnosed CPP; benefit may obviate the need tion of 10% potassium hydroxide to vaginal secretions
for diagnostic laparoscopy. Add-back therapy with estrogen or ("whiff test); and the presence of clue cells on saline
progesterone can mitigate the detrimental effects that GnRH microscopy. Clue cells are squamous vaginal epithelial cells
agonist dierapy has on bone density without affecting treat with a large number of coceobacillary organisms densely
ment efficacy. attached to their surface, giving them a granular appearance
Surgical options for treatment include laparoscopic exci (Figure 18).The Amsel criteria have a sensitivity of 92% and
sion of endometriotic implants, adhesiolysis, sacral nerve stim a specificity of" 77% for diagnosis of" BV. Odier diagnostic
ulation, presacral neurectomy, and uterine nerve ablation. options include a point-of-care test card that detects proline
Hysterectomy is an effective treatment for CPP, although a aminopeptidase and has a sensitivity of 90% and a specificity
substantial proportion of" young women with normal pelvic of 97%. Culture for G. vaginalis is not recommended owing
anatomy at surgery will continue to experience pain. Except to low specificity.
for pathology-directed interventions (such as for endometrio Treatment is widi either oral (7 days) or topical (5-7
sis), surgical treatment is generally pursued only after failure days) metronidazole or clindamycin (oral or topical).
of medical therapy. Metronidazole is safe in pregnancy, but topical clindamycin
should be avoided as it may increase the risk of" adverse out
comes. Approximately 30% of"women will experience a recur
Vaginitis rence of BV within 3 months.
Clinical Presentation and Evaluation
Trichomoniasis
Vaginitis comprises a spectrum of infectious and nonin
fectious conditions that produce characteristic vulvovagi Women who have multiple sexual partners, exchange sex for
nal symptoms, including vaginal discharge, vulvar itching, payment, or use injected drugs are at particularly high risk for
burning, and irritation. The most common causes of trichomoniasis, an STI caused by infection with the proto
zoan Trichomonas vaginalis. Although some infected
vaginitis are bacterial vaginosis, trichomoniasis, and vul
women are asymptomatic, many experience a profuse mal
vovaginal candidiasis. Less frequently, vaginal irritation
can result from atrophy, allergic reactions, or certain der odorous yellow-green discharge with vulvar itching, burning,
and postcoital bleeding. Trichomoniasis is associated with a
matologic conditions.
All patients should be queried about the duration of vaginal pH greater than 4.5 and die presence of motile tri
chomonads on saline microscopy, but the specificity of an
symptoms; relationship to the menstrual cycle, douching, and
sexual activity; discharge odor; and associated vulvar itching, abnormal vaginal pH and the sensitivity of saline microscopy
for identifying motile trichomonads are both low. If point-of-
irritation, burning, and swelling. The vulva and vagina should
be inspected carefully for erythema, excoriations, and tender care tests are unavailable, vaginal secretions may be sent for
ness, and vaginal wall secretions should be collected for pH, culture.
amine testing, and saline and 10% potassium hydroxide
microscopy. Additional point-of-care testing may aid in die
diagnosis.

Bacterial Vaginosis
Bacterial vaginosis (BV) is a polymicrobial infection charac
terized by an overgrowth of multiple anaerobic bacteria
(Gardncrella vaginalis; Urcaplasma, Mycoplasma, and
Bacteroides species, among others), resulting in a reduction
*
of the normal vaginal hydrogen peroxide-producing
Lactobacillus species. Risk factors include douching, lack of
condom use, and multiple or new sexual partners (although
0 . * *
BV may also be diagnosed in women who have never been *
sexually active). BV infection increases the risk of complica
tions in pregnancy and obstetric surgery, and affected
women are more likely to acquire HIV, gonorrhea, chlamydial
FIGURE 18. Clue cells.

92
Eye Disorders

Oral single-dose metronidazole therapy is associated KEY POINTS


with a high rate of"cure and should be provided to all symp
Bacterial vaginosis can be diagnosed by the presence
tomatic women; in addition, treatment of the sexual partner of" three of the four Amsel criteria: abnormal dis
is essential for preventing reinfection. Metronidazole may be
charge that is thin, gray/white, and homogeneous;
safely given at any stage in pregnancy. Inadequate response
vaginal pH greater than 4.5; positive "whiff" test;
to treatment may be caused by reinfection (up to 17% of and the presence of clue cells on saline microscopy.
treated women are reinfected within 3 months) or dimin
In the diagnosis of trichomoniasis, the specificity of
ished responsiveness to metronidazole. If the latter is sus
an abnormal vaginal pH and the sensitivity of* saline
pected, a 7-day course of metronidazole (500 mg twice
microscopy are both low; diagnostic accuracy is
daily) often results in clinical resolution. Subsequent diag
nostic testing to confirm treatment success should be con improved by the use of point-of-care tests.
sidered for women who are HIV-positive as they may be less
responsive to single-dose metronidazole. Persistent coinfec
tion widi T vaginalis can increase HIV shedding and viral
transmission. Eye Disorders
Red Eye
Vulvovaginal Candidiasis
Most women experience at least one episode of vulvovaginal Clinical Evaluation
candidiasis (WC) in their lifetime, and more than one third Red eye is the most common eye condition seen in primary
will have two or more infections. Pregnancy, diabetes, and care. Most cases of red eye can be safely managed by the pri
treatment with antibiotics or corticosteroids increase the risk mary care physician, but it is important to know when to refer
for WC. Aldiough WC produces classic symptoms, such as to an ophthalmologist. During the initial evaluation, die his
vulvar pruritus, external dysuria, and a thick, "cottage- tory should focus on the acuity of" onset of symptoms,
cheese" discharge, studies have shown that die history alone whether one or both eyes are involved, history of trauma or
is insufficient for reliably establishing the diagnosis. On exam risk factors for foreign bodies or abrasion, other accompany
ination, vulvar edema, erythema, and excoriations are sug ing symptoms (nausea, vomiting, photophobia, headache),
gestive of disease, but WC is confirmed only if symptoms are presence of"systemic disease, and whether the vision is intact.
accompanied by the presence of yeasts, hyphac, or pseudo The physical examination should begin with assessment of
hyphae on microscopy or Gram stain, or if a vaginal culture is visual acuity, location and distribution of redness, presence or
positive for yeast. Vaginal pH is normal in WC. As the sen absence of discharge, and pupillary shape. Funduscopy has lit
sitivity of"microscopy may be as low as 65%, empiric treatment tle diagnostic yield in evaluating the red eye.
for WC can be considered if vaginal culture is unavailable
and symptoms are accompanied by characteristic physical Conjunctivitis
examination findings. Culture for Candida in the absence of" Conjunctivitis, inflammation of" the thin, usually transparent
symptoms is not recommended, as 10% to 20% of women are outermost lining of" the eye, is the most common cause of red
colonized. eye. When inflamed, the conjunctiva appears pink or red and
Once die diagnosis of" WC is made, infection is charac the conjunctival blood vessels may be seen. Conjunctivitis is
terized as uncomplicated or complicated and treatment is pre caused by viruses, bacteria, allergies, or contact lens wear. The
scribed accordingly. Uncomplicated WC responds well to history is usually helpful in distinguishing these etiologies.
therapy widi over-the-counter topical imidazoles or a single Viral conjunctivitis (Figure 19) is usually caused by an aden
dose of oral fluconazole. Complicated WC, which is diag ovirus; a history of a preceding upper respiratory tract infec
nosed in women who are severely symptomatic, pregnant, tion and recent exposure to a person with conjunctivitis are
immunosuppressed, or diabetic, necessitates more aggressive clues to this diagnosis. Onset is usually acute, with unilateral
treatment, including topical imidazole treatment for up to 14 redness and watery discharge. Other symptoms include itch
days or two doses of oral fluconazole given 3 days apart. In ing, mild photophobia, a diffuse foreign body sensation, and
pregnant women, WC is treated with a 7-day course of"top crusting of"the eyelids after sleep. Persons with viral conjunc
ical imidazole. Recurrent WC is defined as more dian four tivitis are contagious for up to 2 weeks after the second eye
symptomatic episodes per year. Recurrence is treated initially becomes involved and, therefore, should perform frequent
with a 7- to 14-day course of topical imidazole therapy or oral hand washing and avoid sharing personal items. Food han
fluconazole every third day for a total of diree doses, followed dlers and health care providers should not return to work
by oral fluconazole weekly for 6 mondis. Twenty percent of until eye discharge ceases. The treatment for viral conjunc
cases of recurrent WC are caused by Candida species other tivitis is largely supportive, including cold compresses and
than C. albicans, and these may be treated with a 2-week artificial tears. There is no role for antibiotic eye drops in the
course of intravaginal boric acid. treatment of viral conjunctivitis.

93
Eye Disorders

should be treated systemically for N. gonorrhoeae as well as


Chlamydia trachomatis infection, as one-third of patients
with gonorrhea are cointccted with C. trachomatis.
Bacterial cultures and Gram staining are not generally per
formed in die evaluation of bacterial conjunctivitis. Exceptions
include when gonococcal infection is suspected, die patient is
immunocompromised, wears contact lenses, has hyperacute
bacterial conjunctivitis, or has failed to improve after 1 week of
therapy. Broad-spectrum antibiotic eyedrops should be pre
scribed for use for 5 to 7 days. Bacterial conjunctivitis that per
sists for at least 4 weeks is considered chronic; these patients
should be evaluated by an ophdialmologist.
Allergic conjunctivitis is a clinical diagnosis. It may resem
ble viral conjunctivitis in terms of" clear discharge; however, it
tends to be associated with seasonal allergies, widi itching in
one or bodi eyes as die predominant symptom. Treatment
FIGURE 19. Viral conjunctivitis.
includes oral antihistamines, topical antihistamines, and artifi
Eye with Viral Conjunctivitis. Digital image. Wikimedia Commons. 1 Feb. 2010. Web. 16 May
2012. <http://commons.wikimedia.org/wiki/File.An_eye_with_viral_conjunaivitis.jpg>. cial tears; antibiotic treatment is not indicated.

Subconjunctival Hematoma
Bacterial conjunctivitis (Figure 20) is also highly conta Subconjunctival hematoma (Figure 21) occurs when subcon
gious and is categorized as hyperacute, acute, and chronic. junctival vessels bleed into the subconjunctival space. This can
Acute bacterial conjunctivitis, die most common form of bac occur spontaneously or result from trauma, a Valsalva maneu
terial conjunctivitis seen in the outpatient setting, is usually ver, cough, or antiplatelet or antithrombotic agents.
caused by Staphylococcus aureus; but Streptococcus pneumo Subconjunctival hematomas are painless and do not affect
niae and Haemophilus influenzae can also be culprit organ vision but are sometimes frightening for die patient. Patients
isms. Bacterial conjunctivitis is usually distinguished from viral should be reassured and informed that resolution occurs grad
conjunctivitis by the presence of mucopurulent discharge that ually over several weeks. Aldiough most subconjunctival hem
is worse when waking in die morning and when dried may orrhages are benign, other causes of this finding include endo
form a significant crust, whereas viral conjunctivitis has clearer carditis, bleeding disorders, and medications (such as imatinib
discharge and may be preceded by an upper respiratory tract mesylate); these causes should be considered if"appropriate.
infection. Hyperacute bacterial conjunctivitis is associated
with a copious purulent discharge, pain, and diminished Corneal Conditions
vision. In sexually active adults, hyperacute bacterial conjunc Corneal abrasions cause a foreign body sensation, photopho
tivitis may be caused by Neisseria gonorrhoeae. Gonococcal bia, tearing, and pain. They often result from trauma caused
conjunctivitis is sudden in onset and can rapidly progress to by foreign bodies or fingernails. To examine the cornea, flu
corneal perforation. Patients with gonococcal conjunctivitis orescein dye should be instilled into the eye and the eye

FIGURE 21. Subconjunctival hematoma. A well-localized superficial col


lection of extravasated blood is visible; the sclera and conjunctiva are not
FIGURE 20. Bacterial conjunctivitis. involved.
Swollen Eye with Conjunctivitis. Digital image. Wikimedia Commons. 8 Feb. 2008. Web. 16 Subconjunctival Hemorrhage. Digital image. Wikimedia Commons. 28 Jun. 2011. Web. 16
May 2012. <http7/commons.wikimedia.org/wiki/File:Swollen_eye_with_conjunctivitis.jpg>. May 2012. <http://commons.wikimedia.0rg/wiki/F1le:Subconjunctival_hemorrhage_eye.jpg>.

94
Eye Disorders

examined under a Wood lamp or with a slit lamp. The area with posterior scleritis. Patients with scleritis should be
underneath the upper lid should be examined to rule out a referred emergently to an ophthalmologist as this can be a
foreign body, which should be removed if present. Aldiough sight-threatening condition.
there is litde supporting evidence that topical antibiotics The history is important for differentiating scleritis (pain,
improve outcomes, many practitioners still prescribe a short vision impairment) from episcleritis (less pain, vision unaf
course (48-72 hours). Eye patches have not been shown to fected); however, it is challenging to differentiate the two
speed healing or improve patient symptoms. Most abrasions entities by physical examination. If there is uncertainty
improve markedly within 24 to 48 hours owing to rapid whether die patient has scleritis or episcleritis, an urgent refer
epidielial cell proliferation. ral should be made to an ophthalmologist for clarification.
Corneal ulcers can result from bacterial infection, herpes Approximately 50% of" patients with scleritis have an
simplex virus infection, contact lens wear, or trauma. The underlying systemic disease, such as rheumatoid arthritis, or
ulcer can be detected with fluorescein dye; herpes infection an infectious disease, such as tuberculosis or syphilis.
classically causes a dendritic-appearing ulcer. Corneal ulcers Treatment depends upon the underlying cause but may
can erode and cause corneal perforation and permanent visual include NSAIDs for mild scleritis and systemic corticosteroids
loss. Patients should be seen by an ophthalmologist early for or tumor necrosis factor inhibitors for severe disease.
antimicrobial therapy and consideration of" corneal scraping.
Uveitis
Episcleritis and Scleritis Uveitis, inflammation of"die uvea, commonly presents as a red
Episcleritis (Figure 22) is inflammation of the superficial ves eye with pain, photophobia, and blurred vision. Anterior
sels of the episclera, the vascular membrane that underlies the uveitis, or iridocyclitis, is inflammation of the iris and ciliary
conjunctiva. The inflammation tends to be more localized body and is more common than posterior uveitis. It is char
than widi conjunctivitis, which tends to be diffuse. Its etiol acterized by circumferential redness (ciliary flush) at the
ogy is unclear. It typically is not associated with pain, visual corneal limbus (junction of" the cornea and sclera). Vision is
changes, or tearing, and it usually resolves without specific usually normal in anterior uveitis. The classic finding upon slit
treatment. lamp examination is the presence of inflammatory "flare cells"
Scleritis is inflammation of" the fibrous layer of die eye in the anterior chamber. The differential diagnosis for causes
underlying die conjunctiva and episclera. Anterior scleritis, of uveitis includes infection (syphilis, tuberculosis), autoim
involving die superficial sclera and the deep vessels within the mune disorders, sarcoidosis, and malignancy, although no
episclera, is the most common form. Posterior scleritis cause is identified in more than 50% of* patients. Anterior
involves the deeper structures of the eye. Patients presenting uveitis is usually idiopathic but can be associated widi herpes
with diis disorder usually have severe, dull pain that may have simplex virus infection, trauma, or the presence of HLA-B27
awoken diem from sleep. There may be visual loss, particularly antigen. Patients with uveitis should be urgently referred to
an ophdialmologist for treatment.

Blepharitis
Blepharitis is inflammation of" die eyelid margins. It is usually
caused by 5. aureus infection or seborrheic dermatitis, and
occasionally by rosacea. It is treated with warm compresses
and cleansing of the eyelid margins with diluted nontearing
shampoo using a cotton tip applicator. Topical antibiotic oint
ment may also be used for staphylococcal blepharitis. Oral
tetracyclines may be prescribed when blepharitis is associated
with rosacea.
KEY POINTS

Conjunctivitis, most often caused by a viral infection,


is die most common cause of" red eye; antibiotic ther
apy is not indicated for viral conjunctivitis.
Both viral and bacterial conjunctivitis arc highly con
tagious, and hand hygiene and odier measures should
be emphasized to minimize transmission.
Corneal ulceration, scleritis, and uveitis should be
FIGURE 22. Episcleritis. Superficial dilated blood vessels are seen, with
white sclera visible between the blood vessels.
evaluated by an ophthalmologist if diagnosed or if
tiiere is diagnostic uncertainty.
Image courtesy of Linda Lippa, MD, University of California, Irvine.

95
Eye Disorders

Macular Degeneration phototherapy for dry AMD and drusen is not recommended
as it may increase die incidence of" neovascularization.
Pathophysiology and Clinical Presentation Several inhibitors of vascular endothelial growth factor
Age-related macular degeneration (AMD) is a leading cause (VEGF) have been used to slow the neovascularization of wet
of" visual loss, particularly in the elderly. Progression of disease AMD. Laser photocoagulation therapy is no longer routinely
can lead to difficulty driving, reading, and performing activi recommended for wet AMD owing to potential complica
ties of daily living, and may increase the risk of falling. tions outweighing possible benefits, except in patients with
There are two types of AMD, dry (atrophic) and wet extrafovcal lesions.
(neovascular) (Figure 23). In dry AMD (approximately 85% KEY POINT
of" cases), soft drusen (deposits of extracellular material) form
in die area of" the macula. Dry AMD may present in one or Quitting smoking reduces die risk of" developing age-
bodi eyes. It may be asymptomatic in the early stages and sub- related macular degeneration.
sequcndy progress, with the gradual loss of"central vision. Wet
AMD is usually more aggressive and sight-threatening dian
dry AMD. It is caused by neovascularization of" the macula
Glaucoma
with subsequent bleeding or scar formation. Visual loss may
Primary Open Angle Glaucoma
be more sudden (over a period of" weeks) and is often more
Primary open angle glaucoma (POAG) is a progressive optic
severe. It most frequently presents in one eye.
neuropathy associated with increased Intraocular pressure
The strongest risk factors for AMD are age, smoking,
(IOP) vvidiout an identifiable blockage of"die normal drainage
family history, and cardiovascular disease. Quitting smoking pathways of die aqueous humor. It is die most common form
reduces die risk of developing AMD. No antioxidant or odier of" glaucoma and is the leading cause of irreversible blindness
supplement has been proved conclusively to prevent AMD; in die world. POAG is characterized by painless, gradual loss
however, some studies report that diets high in antioxidants of" peripheral vision in both eyes, which may be unnoticed by
may be protective. die patient. It is often asymmetric. In later stages, it may
The diagnosis of either wet or dry AMD is made with
progress to involve central visual acuity. Clinical findings
dilated ftinduscopic examination and fluorescein angiography include an increased optic cup:disc ratio (>0.5), disc hemor
performed by an ophthalmologist. rhages, and vertical extension of die central cup. Risk factors
include age older than 40 years, race (incidence in blacks is four
Treatment times higher dian in whites), and positive family history.
There are no proven treatments for dry AMD. Patients widi Lowering IOP has been shown to delay or prevent the
advanced dry AMD may benefit from antioxidant agents (vit
progression of" POAG symptoms over time. Pharmacologic
amin C, vitamin E, (3-carotene) and zinc; smokers should agents are die mainstay of glaucoma treatment, but adverse
avoid P-carotene owing to risk of increased lung cancer. Laser effects can be significant (Table 46). Laser therapy to increase

FIGURE 23. Age-related macular degeneration. The dry form (left), is characterized by distinct yellow-white lesions (drusen) surrounding the macular region
and areas of pigment mottling. The wet form (right), is characterized by clumps of hyperpigmentation, hypopigmentation, and evidence of subretinal hemorrhage.

Images courtesy of Edward A. Jaeger, MD. Jefferson Medical College, Wills Eye Institute, Philadelphia, PA.

96
Eye Disorders

TABLE 46. Drug Treatment for Primary Open Angle Glaucoma

Agent Mechanism of Action Systemic Side Effects

p-Blockers (timolol) Decreases inflow Bradycardia, heart block, bronchospasm, decreased libido, central nervous
system depression, mood swings
Nonselective adrenergic Decreases inflow and Hypertension, headaches, extrasystole
agonists (epinephrine) increases outflow
Selective o^-adrenergic Decreases inflow and Hypotension, vasovagal attack, dry mouth, fatigue, insomnia, depression,
agonists (brimonidine) increases outflow syncope, dizziness, anxiety
Parasympathomimetic Increases outflow Increased salivation, increased gastric secretion, abdominal cramps, urinary
agents (pilocarpine, frequency, shock
echothiophate iodide)
Oral carbonic anhydrase Decreases inflow Acidosis, depression, malaise, hirsutism, paresthesias, numbness, blood
inhibitors (acetazolamide) dyscrasias, diarrhea, weight loss, kidney stones, loss of libido, bone marrow
suppression, hypokalemia, bad taste, increased serum urate level
Topical carbonic anhydrase Decreases inflow Lower incidence of systemic effects compared with oral carbonic anhydrase
inhibitors (dorzolamide) inhibitors

Prostaglandin analogues Increases outflow Flu-like symptoms, joint and muscle pain
(latanoprost)
Hyperosmotic agents Reduces vitreous and Headache, heart failure, expansion of blood volume, nausea, vomiting,
(mannitol) aqueous volume diarrhea, electrolyte disturbance, kidney failure

Adapted from Smith OU, Seligsohn AL, Khan SJ, Spaeth GL. Primary open angle glaucoma, http://pier.acponline.org/physicians/diseases/d602/tables/d602-tables.html (login required). In PIER
(online database). Philadelphia: American College of Physicians, 2009. Accessed July 7, 2009.

aqueous outflow, although able to lower the IOP, may lose painless decreased visual acuity, decreased night vision, glare,
efficacy over time. Surgical options, such as trabeculectomy or and diplopia. On ophthalmoscopic examination, there is a
iridectomy, have attendant risks of* complications, including decrease or absence of the red reflex, and opacification of die
blindness. lens can be visualized.
Treatment is surgical removal of" the cataract. Surgery is
Acute Angle Closure Glaucoma indicated if symptoms from die cataract interfere widi the
Acute angle closure glaucoma results from blocked drainage patient's ability to meet his or her needs of daily living; there are
of the aqueous humor. Patients present widi red eye widi no criteria based upon the level of visual acuity. The posterior
severe pain and headache and occasionally nausea and vomit capsule is left intact, and a corrective replacement intraocular
ing and visual halos. Visual acuity is usually reduced. It is lens may be inserted to restore normal vision. Perioperative con
caused by increased IOP owing to blocked drainage of" the sultation is not required for those undergoing a procedure
under local anesthesia and sedation as the surgery is considered
aqueous humor. Physical examination reveals a semidilated,
nonreactive pupil widi an IOP greater than 50 mm Hg. Acute low risk; aspirin and warfarin can be continued.
angle closure glaucoma is an immediate threat to vision, and Complications from surgery are rare but include inflam
urgent referral to an ophthalmologist is important to avert mation, corneal edema, and macular edema. Rarely, infective
optic nerve atrophy. Q endophthalmitis develops (see Eye Emergencies). A common
late-stage complication is opacification of the posterior cap
KEY POINT
sule, leading to decreased visual acuity; this may bc managed
Pharmacologic agents are the mainstay of primary surgically.
open angle glaucoma treatment, but adverse effects
can be significant.
Dry Eye
Dry eye (keratoconjunctivitis sicca) causes symptoms of gritty
Cataract irritation, dryness, and burning of"the eyes. Symptoms usually
A cataract is an opacity of" the lens. Cataracts are the leading occur gradually and may worsen over die course of the day.
cause of blindness and low vision globally, and the prevalence Symptoms can be caused by any process that disrupts the tear
in die United States approaches 20% in those older than 40 film, either by decreasing tear production and secretion or by
years. Risk factors include older age, ultraviolet B radiation increasing tear evaporation. Symptoms may be aggravated by
exposure, smoking, diabetes mellitus, a family history of environmental irritants (smoke, allergens, low humidity) and
cataracts, and systemic corticosteroid use. Symptoms include relieved widi eve closing and increased humidity.

97
Eye Disorders

Decreased tear secretion is usually caused by inflamma appearing as a black curtain, which then progresses across
tion of the lacrimal gland, which may bc a localized process die visual field. Funduscopic examination usually visualizes
or associated with a systemic disease, such as Sjogren syn the detachment (Figure 24). The treatment is surgical.
drome or rheumatoid arthritis. Additionally, decreased Prognosis depends upon the extent of the detachment or
corneal sensation, which may occur with diabetes, hard con tear and time to surgery, so early recognition and emergent
tact lens wearing, herpes zoster infection, or laser-assisted in referral is crucial.
situ keratomileusis (LASIK) eye surgery, causes reduced tear Posterior vitreous detachment (PVD), in which contrac
secretion. Increased tear evaporation may be caused by tion of* the vitreous places traction on die retina, is the most
increased size of"the palpebral fissure (as seen in Graves oph common type of" retinal detachment and typically occurs in
thalmopathy) or by meibomian gland dysfunction, which persons aged 50 to 75 years. Patients with PVD can be
reduces the protective lipid layer of the tear film. Other risk asymptomatic but most often present with floaters or flashes
factors for dry eyes include increasing age, female sex, of" light (photopsias). Most photopsias are benign and idio
decreased androgen levels, and certain medications, includ pathic, becoming more common as age progresses.
ing anticholinergic agents, antihistamines, selective sero Symptoms of PVD usually progress over a period of 1
tonin reuptake inhibitors, nicotinic acid, and isotretinoin. week to 3 months and can progress to visual loss. Patients
Patients with persistent or severe dry eye, as with Sjogren with uncomplicated PVD have less than a 5% chance of devel
syndrome, Graves disease, or Bell palsy, may suffer corneal oping a full retinal tear within 6 weeks, and symptomatic
damage. patients should simply be reassured and educated about
Treatment of dry eye is directed at decreasing inflamma potential symptoms of"visual loss. Symptoms in patients vvidi
tion and addressing any lid pathology disrupting the tear film. out full tears usually resolve over 3 to 12 months. No specific
Artificial tears are helpful for lubrication. Warm compresses limits on activity are necessary. For those patients with PVD
may also bc helpful. Lid inflammation ma}' be treated with who develop retinal tears, one half go on to full retinal
oral tetracyclines. Meibomianitis may respond to gentle detachment. A full retinal detachment may present widi cob
scrubs with mild nontearing shampoo. Punctal or canalicular web-like floaters with acute visual loss or with a monocular
plugs may lower tear film osmolality and thus decrease evap decreased visual field; these patients should be evaluated
oration. Topical corticosteroid drops (short-term use only) acutely. Those with full detachments will require ophdialmo-
and topical cyclosporinc have also been used in patients widi logic intervention.
dry eye caused by systemic illness. KEY POINT

Patients with cobweb-like floaters with acute visual


loss or who are found to have a monocular decreased
Excessive Tearing
visual field may have a full retinal detachment and
Excessive tearing results from either overproduction of" tears should be seen urgently by an ophthalmologist.
(lacrimation) or impaired drainage through the lacrimal duct
system (epiphora). Lacrimation is often bilateral and painless,
whereas epiphora may be unilateral and can be painful, par
ticularly if infection of the ductal system is involved (dacry
ocystitis). The eyes should be examined for evidence of" for
eign body or acute conjunctivitis. The lids should be
examined for ectropion and adequate lid closure and the
punctum examined for blockage, inflammation, or purulence.
Treatment is focused on the underlying cause, with oph-
dialmologic referral for mechanical intervention for patients
with lacrimal drainage problems.

Retinal Detachment
Retinal detachment occurs when the neurosensory layer of
the retina separates from the retinal pigment epithelial layer
and choroid. This can result from either fluid accumulation
behind or vitreous traction on the retina. Separation can
cause ischemia in the retina or a tear that may lead to visual
FIGURE 24. Retinal detachment, with characteristic folding and tearing
loss. Retinal detachment occurs predominantly in myopic
of the retina.
patients. Patients may report floaters, squiggly lines, flashes
Image courtesy of Edward A. Jaeger, MD. Jefferson Medical College. Wills Eye Institute,
of light, and then a sudden visual defect that is peripheral, Philadelphia. PA.

98
(A
?. . vj o c p __; .2 "2
0) | o G
o '3
o O
0
a 56 a
60 "_J
3 o
o
aq " > o p
n
0 -r.
a (-A u
= _o
0 o
C/5
u T3 a.
So _ -rr t "73
2/-o
\ f >
, y - t/3 __5 B 1-

i
3
a
s a
t .2 = Ci, ^
U U 3 _ d 3 4-1 1>
3JC3
-a ts o y
c
J-C -8o E Er q B
11
v_ s_

j T3 p 5 Q p -a S2 -a to _, n p o _ o
.o rt o 'J u
p
O
a
J= > C J_ I
S.
a. aS 1 "3 a
3 o r.
(A u
bO O
p
p
<
5
3 cu G
- j o .h n to c 5
O 3 a. 5 -c
c .-- H IS
'J
to 0
j

Ph
5 u u
O
o c SJ 5 ^6bt oo S
3 * 2 '5
c O to >>
( j in 5 c
p<^^-_; -s
g -s h -a * _ _

e
d to OJ
_> _o 1 = 3 E CD

o u -. g u
u
u d u u u

Q in 3 O
to
a_
r.
u u
D- rt 4 -_> O p
__
to u ___ guo a, u

o ~* ~, u
"C __.
a
t J ? a, s 5J
5 u
'J
u u ra o
aJ+
Q. VI
o '> t o ^__
1 O
3 u "33 i- o vfl
r
o rt rt
o
H 3 u (J a _'> J ro
E
g | d
J
_^ o ed
c o
a
ar
u
'
4J Q. pin
H ai
a.
u
t/J
j

to o -r. jO- So 0 0) D
^ c O o
3 3 "t33
C)
*- c
ro >
-I
o _c ^c: d S
"5 ro

9
f -'2 of
JO T 3 w C
U OJ QJ

< OJ | | bO U u u a X u c o
O -a "0
a p c
c5 ss
3
3 3 t* 3 rt "3 j ol u u

3
UJ 1.1 V5 'J
13
c E
u
t>
d
_5
5 H
H O
CI c3
5 u <L < u U LU

> 'J
I J W5
_ _ Bl C 2 -^ R g
5 3 _o
'J -2 ___ -b 2a ? 0 g
_i -* rt
'Eo-S I 3 o
2 & 11 o o f a _ g ^ > ">
pi I __. | gj | ^
^ t; o .a "_j *Q o -g 2 __: 'J
u u5

g

c ^ 5> f
" Q
^.
u
o y JD
S
o
oj ra c to e^
C a. 3 0 2 ;| -
^ '! "0 > -a - o is :y a
3 . - t ? c x y -5 -^
w = 8 .a .3 rt rt _2 3
c 5 -q y
^ o _, __, 112 _5 O C
3 c rt
(Q c "g *g y ts 73
O 4-1

3
O -^
3 > a e E "3
to g = o y to
ij> +.
c y. J _s o 1
U d -*- rt i
D- .H 34 o
1 *- -3
I3 gw 3
us ra S o l
O -3 o " a u S ^
2- ffl ^ * 3 C >
. 2 s rt s to -

J Si
cc
3 cu a to t o b 3
a
U-
i:
-Q "O Si
i$
K uss 1:
^ C3 r^
O r - - .
to "3 <
Ear, Nose, Mouth, and Throat Disorders

of acoustic vibrations from external sources to the cochlea


and may originate in the ear canal, tympanic membrane, or
the ossicles of the middle ear. Sensorineural hearing loss
involves the perception and transmission of acoustic vibra
tions and may reflect a cochlear problem or an issue with the
acoustic nerve.
If hearing loss is unilateral, the Weber and Rinne tests can
help to differentiate between conductive and sensorineural
hearing loss. A 256 Hz tuning fork is commonly used for the
Weber test and a 512 Hz fork for the Rinne test, aldiough a
512 Hz fork may be used for both. In the Weber test, the
vibrating tuning fork is placed in contact with die forehead or
scalp in the midline equidistant from both cars. In conductive
hearing loss, the fork will be heard more loudly in die affected
ear, and in sensorineural hearing loss, it will be louder in the
unaffected ear. In the Rinne test, die vibrating tuning fork is
held against the mastoid process of the affected ear until it can
no longer be heard; it is then removed and held outside the
ear. If the tuning fork can be heard after removal, die con
ductive system is likely intact, favoring a sensorineural cause
of hearing loss. If the tuning fork is better heard in contact
FIGURE 26. Herpes zoster ophthalmicus. with the mastoid bone, conductive hearing loss is more likely.
In practice, patients who report hearing loss and lack an
with herpes zoster involving die tip of die nose (Hutchinson obvious cause on physical examination (such as cerumen
CONT.
sign) should be seen by an ophthalmologist to rule out herpes impaction or middle ear effusion) should bc referred for for
zoster ophthalmicus (Figure 26). The Hutchinson sign has a mal audiometry, which is better able to discriminate between
fairly high correlation widi die presence of corneal involve conductive and sensorineural hearing loss, and help deter
ment, which can cause permanent visual loss. mine which patients will likely benefit from amplification.
Optic neuritis occurs prcdominandy in middle-aged, white Imaging of the central nervous system is not considered
persons, and affects women more dian men. Patients present part of a routine evaluation for hearing loss. A clinical setting
widi pain widi eye movement, blurred vision, visual field deficits, in which imaging should be considered is the progressive
and change in color perception. The optic disc usually appears onset of" unilateral sensorineural hearing loss; a small but sig
normal. Optic neuritis is usually self-limited, but an ophthal nificant number of these patients have retrocochlcar pathol
mology consultation and a brain MRI should be obtained. Optic ogy as a cause of their hearing loss, such as an acoustic neu
neuritis can be an early harbinger of multiple sclerosis. roma. If" imaging is pursued, MRI is the study of choice to
Intravenous corticosteroids may help speed recovery. CJ evaluate the posterior fossa and internal auditor}' canal.
KEY POINTS Sudden sensorineural hearing loss, defined as a 30 dB
Any patient with acute vision loss should be seen loss over a period of 3 days or less, is a discrete and poorly
understood clinical syndrome. Viral infection, bacterial
emergendy by an ophthalmologist.
Globe trauma, chemical injury, orbital cellulitis, meningitis, Lyme disease, migraine, Meniere disease,
acoustic neuroma, head injury, drug reactions, and neu-
endophthalmitis, and optic neuritis are ocular emer rosarcoidosis can all cause sudden hearing loss; however, a
gencies, and urgent ophthalmologic evaluation cause is identified in only approximately 10% of these
should be obtained.
patients. Most cases are idiopathic and unilateral and oto
scopy is normal; acute otorhinolaryngologic evaluation is
appropriate. Limited data suggest that corticosteroid ther
Ear, Nose, Mouth, and apy may improve the likelihood of hearing recovery.
Fortunately, more than half of" patients recover completely
Throat Disorders within 2 weeks.

Evaluation of Hearing Loss KEY POINT

Causes of hearing loss are generally categorized as conduc Patients who report hearing loss and lack an obvious
tive or sensorineural (Table 48). Conductive hearing loss is cause on physical examination should be referred for
caused by a mechanical problem preventing the transmission formal audiometry.

100
Ear, Nose, Mouth, and Throat Disorders

table 48. Common Causes of Hearing Loss


Disease Notes
Conductive

Cerumen impaction Cerumen may completely obstruct ear canal, causing conductive hearing loss. Impacted cerumen can be
removed with gentle irrigation or an ear curette. No removal needed if asymptomatic.
Otosclerosis Bony overgrowth of the stapes footplate with eventual fixation. Family history of otosclerosis is common.
Treatment is stapedectomy or stapedotomy. Hearing aid may be helpful.

Tympanic membrane Often heals without intervention. Ear should be kept dry. Refer for possible repair if associated with
perforation significant hearing loss or possible middle ear pathology.
Cholesteatoma An expanding mass composed of keratinizing squamous epithelial cells that may contain cholesterol
crystals. Although histologically benign, it may erode extensively into local structures, including the cochlea,
ossicles, tympanic membrane, and facial nerve. Treatment is surgical removal.
Sensorineural

Presbycusis Age-related hearing loss; typically symmetric high-frequency hearing loss. Hearing aids are mainstay of
treatment.
Sudden sensorineural Unclear etiology; presents as a sudden loss of hearing that is sensorineural in nature. Rapid treatment with
hearing loss corticosteroids may improve outcome, although one study showed little difference whether treatment was
initiated within 24 hours or 7 days.
Meniere disease Classically presents as a triad of sensorineural hearing loss, tinnitus, and vertigo, although all three are not
necessarily present in each patient. Symptoms may fluctuate, and attacks are often precipitated by high salt
intake.
Vestibular schwannoma Benign neoplasm, usually causing sensorineural hearing loss, tinnitus, and sometimes vertigo. A family or
(acoustic neuroma) personal history of neurofibromatosis 2 puts patients at high risk for these tumors, often bilaterally.
Noise-induced History of chronic noise exposure or sudden, short exposure to noise blast. Prevention is mainstay of
treatment; hearing aids if condition is already advanced.
Drug-induced History of ototoxic medication use (aminoglycosides, chemotherapeutic agents [irreversible], aspirin and
NSAIDs [partially reversible], antimalarials [reversible], loop diuretics [sometimes irreversible]).

Both conductive
and sensorineural

Infection Middle ear infection may impair movement of the tympanic membrane or ossicles, producing reversible
conductive hearing loss. Viral cochleitis may cause reversible sensorineural hearing loss. Chronic ear
infection may lead to conductive hearing loss.
Head trauma May produce a conductive hearing loss from ossicular disruption and hemotympanum or a sensorineural
hearing loss from cochlear fracture or auditory nerve injury.

tinnitus includes auscultation for bruits over the ear and eye as
Tinnitus well as die neck. Pulsatile tinnitus of indeterminate etiology
Tinnitus is the perception of sound that is not audible. should be evaluated by an otorhinolaryngologist.
Patients frequently characterize it as "ringing," "buzzing," or Absent a treatment for the underlying cause, treatment
"whistling," although it may manifest as other sounds. It may for tinnitus is challenging. Medications are largely ineffective,
be unilateral or bilateral, depending on the nature of the and die mainstay of therapy is neurocognitive interventions to
underlying disorder. Although tinnitus is part of the defining help patients cope with the problem and diminish dysfunc
triad for Meniere disease, it is not specific to Meniere disease tional cognitive processes associated with the experience of
and can occur with virtually any cause of" sensorineural hear tinnitus. External noise generators are sometimes used to
ing loss, such as presbycusis, noise exposure, or acoustic neu mask die sound of"tinnitus, but evidence supporting their effi
roma. It can also occur with conductive hearing loss, such as
cacy is sparse.
from eustachian tube dysfunction or cerumen impaction,
KEY POINTS
although this is less common.
Pulsatile tinnitus (tinnitus synchronized with the patient's Tinnitus can occur with virtually any cause of sen
pulse) and objective tinnitus (tinnitus that can be heard by an sorineural hearing loss, including Meniere disease,
external observer, for example with a stethoscope) may be indi presbycusis, noise exposure, or acoustic neuroma.
cators of an underlying vascular etiology such as arteriovenous Pulsatile tinnitus and objective tinnitus may be indi
malformation, atherosclerosis, carotid artery disease, or a para cators of an underlying vascular etiology.
ganglioma (glomus tumor). Physical examination for pulsatile

101
Ear, Nose, Mouth, and Throat Disorders

Medications are largely ineffective in the treatment by bacteria. The chronic form, which lasts 3 or more months,
of tinnitus; the mainstay of therapy is neurocogni- is usually caused by fungal infection, allergy, or a systemic der
tive interventions to help patients cope with the matitis. Manifestations of otitis externa include pain, itching,
and erythema early in the course but may progress to include
problem.
edema, otorrhea and conductive hearing loss. Risk factors for
the development of"otitis externa include increased moisture,
Otitis Media use of cotton-tipped swabs or bobby pins to clean the ear
canal, use of hearing aids, and decreased production of" ceru
Acute otitis media is inflammation of the middle ear. It
occurs predominantly in children. Despite the fact that it men, which contains an antimicrobial lysozyme and has a pH
of 6.9, helping to inhibit bacterial growth.
is diagnosed more than 5 million times each year in the
Acute otitis externa can range from mild inflammation of
United States, there is a paucity of quality studies of the
the external canal to a severe and life-threatening infection if
disorder in children and remarkably little substantive liter
the temporal bone becomes involved. Onset is usually over a
ature about acute otitis media in adults. In some countries,
few days to a week and is initially manifested by pruritus, ery
such as the Netherlands, management is frequently obser
vation only, in contrast to the United States, where this thema, and discomfort, which progresses to edema of the
canal, serous or purulent secretions, and pain with tugging on
diagnosis is the leading cause of antibiotic prescriptions for
the pinna or moving die tragus. Approximately 50% of bac
children.
Otitis media usually follows an upper respiratory tract terial cases are caused by Pseudomonas aeruginosa and the
infection and is frequently accompanied by ear pain for 24 remainder by Staphylococcus aureus and other aerobic and
to 48 hours. Fever, although less common dian ear pain, anaerobic bacteria. Fungi cause 10% or fewer cases of acute
increases the likelihood of otitis media. In the pediatric lit otitis externa; other etiologies, such as herpes zoster, cause
fewer than 5% of cases. Hospital admission is usually not
erature, tympanic membrane bulging has the highest likeli
hood ratio for acute otitis media, followed by tympanic required, but is indicated if disease progresses to necrosis of
membrane cloudiness and immobility. Other signs include die ear canal and osteomyelitis of"the underlying bone (malig
nant otitis externa) or to involve the temporal or mastoid
erythema of the tympanic membrane, with the likelihood
ratio increasing for greater degrees of" redness. bones. These patients usually have disproportionate pain

Complications of acute otitis media include hearing loss, along with fever higher than 39.0 C (102.2 F); skin necro
tympanic membrane perforation, mastoiditis, and meningi sis, facial paralysis, vertigo, and meningeal signs also may be
tis. However, suppurative complications may occur in as few present.
as 0.12% of"untreated children. Chronic otitis externa is frequently caused by allergic
There are no adequate trials of"treatment in adults, but contact dermatitis from earrings, cosmetics, soaps, shampoos,
in children, the American Academy of Pediatrics and the or the plastic components in hearing aids. Ironically, some
American Academy of" Family Practice currently recom patients initially treated for acute otitis externa develop a type
mend observation along with analgesic therapy in cases in IV delayed hypersensitivity reaction to the otic solutions used
which there is diagnostic uncertainty or in milder cases of to treat die acute infection. Fungal infections causing otitis
otitis media. In patients in whom a decision is made to externa usually can be identified by the presence of white, cot
treat, amoxicillin should be used in patients who are not ton-like strands (Candida) with or without white or black
allergic to penicillin-containing antibiotics, with escalation fungal balls (Aspergillus).
to amoxicillin-clavulanate in 48 to 72 hours if the patient Management of" acute otitis externa involves clearing
fails to improve or worsens. A macrolide antibiotic should the canal of as much debris as possible to optimize penetra
be used as initial therapy in penicillin-allergic patients. An tion of ototopical agents as well as to visualize the tympanic
anti-inflammatory agent should be recommended for pain membrane to ensure it is intact before initiating an ototopi
relief. cal agent. Most forms of otitis externa may be effectively
treated with topical therapy. Mild otitis externa can be
KEY POINT
treated with a dilute acetic acid solution, although many
Complications of otitis media include hearing loss, practitioners still choose to prescribe a 7- to 10-day course
tympanic membrane perforation, mastoiditis, and of an ototopical agent containing neomycin, polymyxin B,
meningitis. and hydrocortisone for both mild and more severe forms.
However, ototoxicity can occur from topical aminoglyco
sides when use is prolonged or the tympanic membrane is
Otitis Externa not intact; in addition, these agents can cause contact der
Otitis externa, a diffuse inflammation of the outer ear canal, matitis in 5% to 18% of patients. Topical fluoroquinolones
may be acute or chronic. The acute form, which accounts for are more expensive but are approved for use when the tym
approximately 90% of cases of otitis externa, is usually caused panic membrane is not intact. Oral antibiotics should be

102
Ear, Nose, Mouth, and Throat Disorders

considered for extra-canal involvement, for older patients,


Upper Respiratory Tract
for those who have not responded to topical treatment, and
for immunocompromised or diabetic patients. Patients with
Infections
malignant otitis externa or disease involving the temporal or Sinusitis
mastoid bones should be hospitalized and given intravenous Acute sinusitis is a challenging clinical domain because the
antibiotics. correlation between symptoms, pathophysiology, and treat
Fungal infections are usually treated with 2% acetic ment is tenuous. Evidence about diagnosis of sinusitis is lim
acid or isopropyl alcohol. Severe fungal infections may ited. A history of cigarette smoking, allergic rhinitis, or previ
require treatment with topical antifungal agents, such as ous episodes of sinusitis may indicate patients at increased risk.
1% clotrimazole. Common symptoms associated with sinusitis such as
KEY POINTS headache, facial pain and pressure that increases when bend
ing forward, fever, and toothache have not been well assessed
Management of otitis externa involves clearing the in comparison with gold standard tests such as sinus aspiration
canal of as much debris as possible before initiating
or radiography. Physical examination findings that have been
an ototopical agent.
shown to add diagnostic value include purulent rhinorrhea
Patients with malignant otitis externa or involvement widi unilateral predominance, local pain with unilateral pre
of the temporal or mastoid bones usually have dispro dominance, bilateral purulent rhinorrhea, and pus in the nasal
portionate pain and fever higher than 39.0 C cavity. The presence of three or more of these symptoms has
(102.2 F); hospital admission and intravenous anti a positive likelihood ratio of 6.75. Imaging is rarely necessary
biotics are indicated. in an average-risk patient, but should be considered in
immunocompromised patients at risk for unusual organisms,
such as fungal or pseudomonal sinusitis.
Cerumen Impaction Initial treatment of"patients with symptoms suggestive of
Cerumen cleans, protects, and lubricates the external audi acute sinusitis is largely symptomatic. Systemic antihista
tory canal. Cerumen is normally eliminated by a mecha mines, intranasal corticosteroids, and topical decongestants
nism involving jaw motion, causing it to migrate out of" the have all been shown to be helpful. Topical decongestants
ear canal. Accumulation of cerumen occurs when this self- should be limited to a few days of use to avoid rebound rhini
cleaning mechanism is inadequate. Excess cerumen is pre tis (rhinitis medicamentosa). Evidence in acute sinusitis sug
sent in 1 in 20 adults and one third of geriatric patients and gests a small increase in die number of patients whose symp
is a common cause of"visits to physicians for ear symptoms. toms resolve if antibiotics are used in patients with symptoms
Cerumen impaction refers to a collection of cerumen suf that have been present for at least 7 days. However, the cure
ficiently large that it is either symptomatic or blocks visu rate is high in placebo-treated patients (80%); therefore, the
alization of the ear canal or tympanic membrane. number needed to treat is also high: between 8 and 15
Cerumen-related symptoms include pain, itching, tinnitus, patients would need to be treated with antibiotics to produce
odor, drainage, vertigo, and hearing loss. Hearing loss can one additional cure. Because of this, some guidelines and cli
range from 5 to 40 dB, depending on the degree of nicians recommend initial symptomatic treatment with initi
impaction. ation of antibiotics only in patients with 3 to 4 days of severe
Treatment options for cerumen impaction include obser symptoms (such as fever >39.0 C [102.2 F], purulent
vation without specific treatment, use of ceruminolytic drainage, and facial pain), worsening of symptoms that were
agents, manual removal of cerumen, and irrigation. Evidence- initially improving following a typical upper respiratory tract
based guidelines indicate that it is not necessary to remove die infection, or failure to improve after 10 days. If"antibiotics are
cerumen unless the patient is symptomatic or die canal or used, there is no evidence of" superiority for any particular
tympanic membrane needs to be evaluated. Patients should antibiotic in patients without risk for infection with resistant
be reassured that cerumen is naturally occurring and has ben organisms. Amoxicillin-clavulanate and doxycycline are both
eficial effects, and that specific efforts to cleanse the ear canals
appropriate first-line agents.
are not needed. Complications of treatment, which occur in
approximately 1 in 1000 treated patients, include tympanic Allergic Rhinitis
membrane perforation, ear canal laceration, infection, and
Allergic rhinitis should be strongly considered in a patient
hearing loss. with rhinitis symptoms (sneezing, congestion, rhinorrhea)
KEY POINT associated with a particular season, environment (for example,
It is not necessary to remove cerumen from the ear home or work), or exposure (such as pets). A detailed history
canal unless the patient is symptomatic or the canal or of when and where symptoms occur can often point to a

tympanic membrane needs to be evaluated. probable allergen. Although not a part of routine evaluation,
either in vitro specific IgE antibody testing or skin testing can

103
Ear, Nose, Mouth, and Throat Disorders

confirm an allergy to specific antigens if needed. Such testing Pharyngitis


is most valuable when considering allergen immunotherapy Most cases of acute pharyngitis are viral and require no spe
but should also be considered if the patient is considering cific intervention odier than symptomatic therapy. Because of
expensive or difficult lifestyle changes to reduce allergen the association between group A streptococcal (GAS) pharyn
exposure. gitis and rheumatic fever, however, GAS pharyngitis should
Initial therapy should include an assessment of the be identified and treated to prevent rheumatic heart disease.
options to reduce exposure to allergens; for example, closing However, empiric antimicrobial treatment not based on a
windows and doors and using an air filter at home to mitigate clinical decision tool incorporating specific clinical informa
pollen exposure. These measures can be cumbersome, how tion is discouraged as this has been shown to lead to signifi
ever, and patients, especially those with mild or transient cant inappropriate antibiotic use. The four-point Centor cri
symptoms, may prefer pharmacotherapy. Treatment options teria are widely used to identify patients at risk of GAS
include intranasal corticosteroids, oral antihistamines, pharyngitis: (1) fever (subjective or measured >38.1 C
intranasal antihistamines, oral leukotriene inhibitors, and [100.5 F]); (2) absence of cough; (3) tonsillar exudates; and
intranasal cromolyn. Intranasal corticosteroids are considered (4) tender anterior cervical lymphadenopathy. Patients widi
first line dierapy because of superior effectiveness in con zero or one Centor criterion are low-risk and do not need
trolled trials. Cost and frequency of dosing are also impor additional testing. Patients with two or three criteria should
tant to consider in developing a treatment plan. Combining have a confirmatory test (either a rapid GAS antigen test or
intranasal corticosteroids and oral antihistamines may be throat culture; bodi studies are not necessary) and be treated
helpful in patients who have persistent symptoms with a based on die result. Patients meeting all four criteria should
single agent. Patients with specific allergies confirmed by also have a confirmatory test but may be treated empirically
testing may be candidates for allergen immunotherapy. while awaiting results.
Immunotherapy reduces symptoms and medication use but is Penicillin is the treatment of choice for GAS pharyngitis:
expensive, requires multiple injections or sublingual adminis options include a 10-day course of" oral penicillin or a single
trations over a long period of time, and carries a small risk of intramuscular injection of penicillin G. The latter is a partic
systemic reactions with each injection. ularly good option in high-risk situations (for example, sec
ondary prevention of rheumatic fever) or if there is concern
about die patient completing a full course of oral therapy. A
Nonallergic Rhinitis
Patients widi chronic rhinitis symptoms without an associated 10-day course of erythromycin or a 5-day course of
exposure may have nonallergic or vasomotor rhinitis. azithromycin are alternatives in patients who arc allergic to
penicillin.
Nonallergic rhinitis may also have specific triggers, such as
odors, spicy foods, and changes in temperature. The diagno Group C and group G streptococci can cause a clinical
sis is often evident by clinical history but may be confirmed syndrome similar to GAS pharyngitis, although often less
severe. Aldiough they have been associated widi glomeru
by allergy testing, which reveals either no specific allergies or
lonephritis and reactive arthritis, group C and G streptococci
allergies that correlate poorly with the patient's symptoms. arc not known to cause acute rheumatic fever. Treatment widi
Treatments for nonallergic rhinitis include intranasal corti
antibiotics may shorten the duration of symptoms, but con
costeroids, antihistamines, and anticholinergic agents; oral trolled trials are lacking.
medications are less effective for nonallergic rhinitis. Nasal
saline irrigation may also be helpful.
Lemierre Syndrome
Rhinitis medicamentosa is the syndrome of chronic
Acute pharyngitis is rarely complicated by septic thrombosis
rhinitis resulting from long-term use of topical nasal decon of the internal jugular vein (Lemierre syndrome). This is most
gestants. Withdrawal of the offending medication is the only often caused by Fusobactcrium nccrophorum, an anaerobic
effective therapy, but management of the resulting rebound
gram-negative rod that is part of die normal oropharyngeal
rhinitis is notoriously difficult. Intranasal corticosteroids may flora. Infection is thought to occur by contiguous spread
be helpful.
through die tissues of the pharynx. The diagnosis should be
Many anatomic conditions and systemic illnesses should suspected in anyone with antecedent pharyngitis and persist
be considered in the differential diagnosis of" chronic rhinitis. ent fever despite antimicrobial therapy. Anterior neck pain and
Anatomic conditions include deviated septum, nasal polyps, tenderness are frequently but not universally present. Soft tis
hypertrophic turbinates, and cerebrospinal fluid leak; systemic sue CT of the neck widi contrast typically shows a jugular vein
illnesses include sarcoidosis, granulomatosis with polyangiitis thrombus with surrounding tissue enhancement. Empiric
(also known as Wegener granulomatosis), cystic fibrosis, and therapy should be directed at bodi streptococci and anaerobes
hypothyroidism. Pregnancy also may be associated with and should be active against [3-lactamasc-producing organ
chronic rhinorrhea and congestion. isms. Therapy typically lasts at least 4 weeks.

104
Ear, Nose, Mouth, and Throat Disorders

KEY POINTS studies are usually not necessary. If these studies are obtained,
a complete blood count and coagulation panel should be
Initial treatment of patients with symptoms sugges
ordered. These studies, however, are normal in nearly 80% of
tive of acute sinusitis is largely symptomatic, includ
patients, including those widi systemic conditions that pre
ing systemic antihistamines, intranasal corticosteroids,
and topical decongestants. dispose to a coagulopathy. Imaging is also not required in
most patients with epistaxis; in patients with recurrent unilat
First-line therapy for the treatment of allergic rhinitis eral epistaxis, particularly in those who smoke, a malignant or
is an intranasal corticosteroid.
benign sinonasal neoplasm should be considered as a diag
Patients widi pharyngitis widi zero or one of the nostic possibility and radiographic imaging should be
Centor criteria (fever, absence of cough, tonsillar exu obtained.
dates, tender anterior cervical lymphadenopadiy) have a Most anterior nosebleeds do not require any medical
low risk of group A streptococcal infection, and do not treatment and respond to compression of the nasal ala
need additional testing or antibiotic dierapy. against the septum by the patient for at least 15 minutes.
The head position while applying pressure can be either for
ward or backward depending on which is more comfortable.
Epistaxis Vasoconstrictor topical agents may also be useful. If bleed
Epistaxis is a common occurrence, affecting 60% of the pop ing fails to resolve, cautery can be carried out with chemi
ulation some time in their lifetime; approximately 6% seek cals such as silver nitrate or electrical cautery by an otorhi-
medical treatment. More than 90% of"nosebleeds occur along nolaryngologist. Anterior nasal packing (which is extremely
die anterior portion of" die nasal septum (the Kiesselbach uncomfortable for patients) for 1 to 3 days can be performed
area). This area receives its blood supply from branches of with cither nondegradable products or absorbable or
both the internal and external carotid arteries. Most episodes biodegradable products. These maneuvers work in 60% to
of anterior epistaxis can be controlled with patient-exerted 80% of patients in whom pressure and vasoconstrictor agents
direct pressure. The other 10% of epistaxis episodes occur do not control bleeding. Patients widi posterior epistaxis
posteriorly, along the nasal septum and lateral wall. This area's should be seen by an otorhinolaryngologist who may need
blood supply is from the external carotid artery via the to insert posterior packing which will stop 70% of posterior
sphenopalatine branch of die maxillary artery. Posterior epis epistaxis. Patients for whom these maneuvers arc unsuccess
taxis is more common in older patients. ful may require surgical ligation or embolization of persist
The most common cause of epistaxis in children is nose ently bleeding nasal arteries.
picking; other causes in adults include the administration of Many physicians prescribe topical or oral antibiotics
intranasal medications such as corticosteroids or deconges while packing is in place in eidier the anterior or posterior
tants, dry nasal mucosa during winter months, viral or bacte locations to prevent toxic shock syndrome. The true risk of"
rial rhinosinusitis, and neoplasms. Systemic diseases associated developing toxic shock syndrome from nasal packing, how
with epistaxis include hematologic malignancies, hemophilia, ever, is unknown.
and acquired bleeding disorders from liver or kidney disease. KEY POINT
Iatrogenic causes associated widi epistaxis include anticoagu Most anterior nosebleeds do not require any medical
lant and antiplatelet medications and possibly Ginkgo biloba
treatment and respond to compression of the nasal
and ginseng supplements. Patients with epistaxis frequently
ala against die septum for at least 15 minutes.
have hypertension upon presenting to a health care provider;
however, it is unclear whether chronic hypertension is associ
ated with epistaxis or whether the hypertension is simply the
result of"anxiety owing to die epistaxis. Nosebleeds are rarely
Oral Health
life threatening. Various oral conditions can impact die overall health of the
Evaluation of epistaxis begins widi a history focused on patient. Dental disease can cause pain and discomfort and
result in nutritional compromise if" the ability to chew is
frequency, location, whether the nosebleed is unilateral or
bilateral, and evaluation of risk factors that may be associated impaired. Periodontal disease has been associated with
with the bleeding. Ideally, the site of die bleed should be visu increased prevalence of coronary heart disease. Mucosal
alized in order to localize the epistaxis as anterior or posterior. lesions can cause pain and discomfort, and oral cancers pose
Anesthetic or vasoconstrictor topical sprays (lidocaine with a direct threat to healdi. Xerostomia, common in elderly
oxymetazoline, for example) may be needed both to control patients, is frequendy exacerbated by medication adverse
bleeding and allow visual localization of" the bleeding site. effects. Gingival hyperplasia is also strongly associated with
Unless the patient has severe epistaxis or has an associ exposure to medications: anticonvulsant agents (especially
ated systemic disease such as a coagulopathy or hematologic phenytoin), cyclosporinc*, and nifedipine are most frequendy
malignancy or is on medications diat affect clotting, laboratory implicated.

105
Ear, Nose, Mouth, and Throat Disorders

The U.S. Preventive Services Task Force concluded in advanced kidney failure, and advanced liver disease, can pro
2004 that the evidence is insufficient to recommend for or duce characteristic breath odors. Esophageal diverticula and
against screening adults for oral cancer. Some patients have chronic pulmonary infections (abscess, bronchiectasis) may
poor access to dental care, however, and for these patients, and also cause halitosis. A clinician can sometimes differentiate
for patients at high risk of" oral disease (tobacco, alcohol, or oral, nasal, and odier causes of halitosis by positioning his or
methamphetaniine use; bulimia; family history oforal cancer), her nose in front of die patient's face and asking the patient
some groups recommend diat screening oral examinations be to exhale several breaths through the mouth, dien several
performed. This examination should include inspection of die through the nose. Oral halitosis should have a stronger odor
teedi and oral mucosa, looking for evidence of" caries, peri coming from die moudi, and nasal from the nose; if" both are
odontal disease, and leukoplakia or ulcerative lesions of" die equal it raises concern of" a systemic, esophageal, or pul
mucosa. Palpation of the neck and submandibular area for monary cause of halitosis. Treatment for bad breath should
masses or lymphadenopathy is also appropriate. focus on oral hygiene, particularly flossing between the teedi
and cleaning (and possible scraping) the posterior tongue,
Oral Infections and Ulcers bodi common sites of" origin of" halitosis. Patients with pri
Oral lesions can be caused by infection, neoplasm, and sys mary dental disorders (caries, abscesses, periodontal disease)
temic conditions. Common infectious causes of mucosal should be referred for appropriate dental care. Chlorhexidine
lesions include Candida, herpes simplex virus, coxsack mouthwashes may reduce odor.
ievirus, HIV (bacillary angiomatosis, Kaposi sarcoma, hairy
leukoplakia), and syphilis. The most common malignancy in Tongue Syndromes
the oral cavity is squamous cell carcinoma (particularly in
Geographic tongue manifests as patchy areas of" atrophy of" die
tobacco users); melanoma can also occur. Systemic diseases filiform papillae, leading to erythematous patches on die
associated with oral findings include lichen planus, bullous
tongue with white borders. Lesions typically recur and regress
pemphigoid and pemphigus vulgaris, erythema multiforme in various areas over time. Patients are usually asymptomatic,
and Stevens-Johnson syndrome, and Behcet syndrome. More
although tongue discomfort is occasionally reported.
information about oral mucosal lesions is provided in MKSAP
Atrophic glossitis, a bright red, smooth, sometimes ten
16 Dermatology. der tongue widiout visible taste buds, can be seen in patients
with vitamin B12 deficiency and has been reported in those
Dental Infection with iron deficiency and widi celiac disease. Treatment should
Dental infections may involve either the tooth and underly be directed at the underlying disorder.
ing bony structures or the gingiva and periodontal tissues.
Infections of" the tooth structure are typically asymptomatic Burning Mouth Syndrome
until they involve the pulp cavity, at which point the patient
Burning mouth syndrome is characterized by a burning sen
develops a toothache. Infection frequendy extends into die sation in die moudi or tongue in the absence of an explana
underlying bone, forming a periapical abscess. Definitive tory diagnosis. Patients frequendy have other oral symptoms
treatment of"these types of"infection requires either endodon such as dryness or taste alterations. It appears to be most
tic removal of diseased pulp (root canal) or extraction of die
prevalent in postmenopausal women. It is typically managed
infected tooth. In patients without cellulitis or symptoms of
by addressing xerostomia, if present, and excluding odier pos
systemic infection, antibiotic dierapy is not necessary if" den sible causes, such as atrophic candidiasis, herpes virus infec
tal intervention can be performed within several davs.
tion, post-herpetic neuralgia, local reaction to dental prod
Periodontal disease involves the gum, connective tissue,
ucts, and nutritional deficiencies. Medications directed at
and bone supporting the teeth. Most periodontal disease is a
neuropathic pain, such as anticonvulsant agents and tricyclic
chronic, indolent condition that poses a long-term risk of antidepressants, may be helpful.
tooth loss. Periodontal disease has also been associated with
atherosclerotic cardiovascular disease. Oral hygiene (including
toothbrushing and flossing) and removal of" plaque are the Temporomandibular Disorders
mainstay of therapy. Oral antimicrobial rinses may also be Temporomandibular disorders are a group of pain problems
helpful in preventing progression of" disease, aldiough evi that involve die temporomandibular joint (TMJ) and associ
dence is limited. ated structures. Typical features include jaw pain and
headache and clicking, grinding, or grating at the TMJ.
Halitosis Although temporomandibular symptoms are common, most
Eighty percent to 90% of cases of" halitosis (bad breadi) orig are self-limiting; fewer than 5% of adults with temporo
inate in the mouth. The remainder are attributed to other mandibular symptoms develop chronic symptoms.
conditions, including chronic sinusitis, nasal polyps, and ton Temporomandibular disorders are classified into articular dis
sillar stones. Some systemic conditions, such as ketoacidosis, orders, which include derangements of the intra-articular disk

106
Anorectal Disorders

that normally sits between the mandibular condyle and its and itching. Weight loss may suggest underlying inflamma
articulation in the glenoid fossa, and masticatory muscle dis tory bowel disease or malignancy. Fever may be associated
orders, which include myofascial pain syndrome. with abscess. Hard stools can contribute to die formation of"
Diagnostic evaluation of temporomandibular disorders hemorrhoids and anal fissures, and history of" sharp pain fol
should focus on eliminating alternative diagnoses, such as lowing instrumentation or an unusually hard bowel movement
dental pain, otitis and mastoiditis, salivary gland disorders, should increase suspicion of anal fissure. Pain with defecation
temporal arteritis, trigeminal neuralgia, and herpes zoster. is typical of anal fissures and anorectal abscesses but can also
Palpation of"the TMJ by applying mild anterior pressure widi occur widi hemorrhoids. A palpable mass on defecation sug
a finger placed posteriorly to each tragus may reveal tender gests a prolapsed internal hemorrhoid or overt anal prolapse.
ness, clicking, or crepitus. Side-to-side jaw movement with Physical examination should include inspection for external
the fingers palpating die TMJ may also show similar abnor masses, thrombosed external hemorrhoids, skin lesions, and
malities or asymmetry in findings. The jaw muscles should excoriations. Having die patient bear down during die exami
also be palpated for evidence of" asymmetry or tenderness. nation can sometimes reveal prolapsed internal hemorrhoids.
Although not indicated in most patients, diagnostic imaging Digital rectal examination is valuable to exclude perirectal abscess
and anorectal neoplasia. Anoscopy allows for visualization of
may be helpful in excluding dental disease if" suspected and
internal hemorrhoids, anal fissures, and cancers of die anal canal.
assessing TMJ anatomy to distinguish articular disorders
from muscle disorders in complex cases. CT is particularly
helpful in the diagnosis of" osteoarthritis of the TMJ, whereas Hemorrhoids and Rectal
MRI provides additional information about the soft tissues,
vascularization, and cartilaginous structures. In general, the Bleeding
therapeutic value of imaging is modest, and most patients are Hemorrhoids are dilated veins in the hemorrhoidal plexus
managed conservatively regardless of the underlying etiology. surrounding the anal canal. They are classified as internal or
Initial treatment of" temporomandibular disorders external depending on whether they are above or below the
focuses on noninterventional, nonpharmacologic strategies. dentate line. Internal hemorrhoids are more frequently asso
Jaw relaxation, heat, and therapeutic exercises may be help- ciated with painless bleeding, whereas external hemorrhoids
Mil. For patients with chronic temporomandibular disorders, more often cause pain; thrombosed external hemorrhoids are
cognitive-behavioral therapy has been shown to reduce pain, particularly painftil. Internal hemorrhoids may prolapse and
depression, and interference with activities. Biofeedback may only be externally visible or palpable during a bowel move
also be of*value. Jaw appliances and occlusal splints have been ment or Valsalva maneuver.
a prominent part of"temporomandibular disorder therapy for Hemorrhoids are a common cause of bright red blood from
die rectum, and furdier evaluation should be based on risk for
years despite questionable evidence of" benefit. Evidence is
limited on the benefit of pharmacotherapy, but NSAIDs and colon cancer and odier gastrointestinal diseases. Visualizing a
hemorrhoid or odier source of bleeding in a low-risk patient
tricyclic antidepressants are sometimes used.
Patients with anatomic abnormalities of the TMJ, such as younger dian 40 years widiout odier symptoms to suggest
osteoarthritis or disk derangements, may benefit from intra inflammatory bowel disease or colon cancer may spare die
articular injections of corticosteroids or hyaluronic acid, patient Mirdier endoscopic evaluation as the risk of malignancy is
low in diis age group. Patients 40 to 50 years old widi typical
although neither is recommended for long-term use.
hemorrhoidal symptoms but at low risk for colon cancer should
Arthrocentesis, arthroscopy, and joint replacement may be
probably have at least sigmoidoscopy. In patients older dian 50
helpful, but controlled trials are few and of"poor quality.
years, rectal bleeding should not be considered hemorrhoidal
KEY POINT widiout additional investigation. These patients should undergo
Initial treatment of temporomandibular disorders colonoscopy to evaluate die source of" bleeding provided that
focuses on noninterventional, nonpharmacologic routine screening has not been recendy performed,
strategies, such as jaw relaxation, heat, and therapeu Initial treatment of" hemorrhoids should focus on inter
tic exercises. ventions to soften bowel movements, sitz baths, and topical
anesthetics or topical corticosteroids to relieve pain and itch
ing. Recurrent hemorrhoids that fail to respond to conserva
tive therapy can be treated widi sclerotherapy, banding, pho
Anorectal Disorders tocoagulation, or surgical resection.
KEY POINT
Approach to the Patient with In patients older than 50 years, rectal bleeding should
Anorectal Disorders not be considered hemorrhoidal without additional
Patients with anorectal disorders should be questioned
investigation.
regarding bowel frequency and consistency, bleeding, pain,

107
Mental and Behavioral Health

Anal Fissure implicated in pruritus ani; dietary modification is frequently


Anal fissures are tears in the anal skin distal to the dentate recommended, although its effectiveness has not been
established.
line, and may therefore be exquisitely painful, particularly
with defecation. They are most often caused by local trauma For some patients, anal itching becomes a primary and
such as hard stools or anal instrumentation. High-fiber diets self-perpetuating problem. Patients should have a careful
and other measures that soften stools may help to reduce the external and digital rectal examination; endoscopic evalu
risk of anal fissures, aldiough the specific causes are poorly ation should be considered in patients with a history of
rectal bleeding or change in bowel movements. Patients
understood, particularly of fissures that are persistent or
recurrent. Anoscopy typically reveals a small mucosal tear, without an apparent underlying cause should be counseled
most often in the posterior midline. on proper anal hygiene (fecal soiling can cause irritation),
Acute anal fissures often heal spontaneously; treatment including the avoidance of excessive cleaning or astringent
should start with warm sitz baths for symptom relief, cleansers. Protective ointments such as zinc oxide and
increased dietary fiber to soften stools, and topical anesthet limited short-term use of topical corticosteroids may be
ics to decrease pain widi bowel movements. Chronic anal fis helpful.
sures are more challenging to treat. Topical nitroglycerin has
the most evidence of" benefit but must be compounded at a
lower concentration for anorectal use (0.2% instead of"2%, the
standard concentration). Topical calcium channel blockers
Mental and Behavioral
(also compounded) and botulinum toxin injections may also Health
be helpful, but evidence is limited. Internal anal sphinctero
tomy is occasionally used for refractory anal fissures but car Depression
ries an increased risk of incontinence. The prevalence of depression in the United States is 15%, and
depression is the second most common cause of"primary care
visits. Patients widi chronic medical disorders, such as dia
Anorectal Abscess betes mellitus, heart disease, stroke, and cancer, have an
Anorectal abscesses typically originate in one of" the anal crypt increased risk of developing depression, and depression can
glands surrounding the anal canal at die dentate line. Pain and
negatively influence morbidity and mortality outcomes in
tenderness to palpation are typical, and fluctuance may be these patients.
noted either on palpation of the external anus or internally on
Despite its high prevalence and severe negative effects,
digital rectal examination. depression often goes undiagnosed in the primary care set
Definitive management involves surgical drainage, which
can often be accomplished in an outpatient setting with local ting. Brief screening tools have been validated, with the sim
anesthesia. Antibiotics are generally unnecessary unless there plest being a two-question instrument (see Routine Care of
the Healthy Patient). It is imperative to assess die presence
are specific risk factors, such as extensive cellulitis, diabetes
of suicidal ideation and level of functional impairment,
mellitus, or the patient is immunocompromised. which may guide treatment decisions. Patients with active
suicidal plans warrant urgent referral to a psychiatrist or
Chronic Anorectal Pain emergency hospitalization in order to immediately treat
their depression.
Chronic anorectal pain syndromes include chronic proctalgia
(levator ani syndrome), characterized by chronic pain or
Diagnosis of Depressive Disorders
aching, with episodes lasting more dian 20 minutes, and
A major depressive episode is diagnosed according to the
proctalgia Migax, characterized by sudden severe pain that dis
DSM-IV by die presence of" five or more of the following
appears completely within seconds or minutes. Neither con
dition is well understood. The approach to chronic proctal symptoms during the same 2-week period, at least one of
which is cither (1) depressed mood or (2) loss of interest or
gia is similar to other chronic pain syndromes. Proctalgia
Migax rarely requires specific treatment because of" the brief pleasure:
nature of symptoms. 1. Depressed mood most of the day, nearly ever)' day
2. Loss of interest or pleasure in all or almost all activities
most of the day, nearly every day
Pruritus Ani
3. Significant weight loss when not dieting, or weight gain;
Many conditions, including infection, primary skin diseases,
local irritants, and malignancy, can cause anal itching. or decrease or increase in appetite nearly every day
Dietary factors, including caffeine, alcohol, spices, citrus 4. Insomnia or hypersomnia nearly every day
foods, milk products, tomatoes, and peanuts, have been 5. Psychomotor agitation or retardation nearly every day

108
Mental and Behavioral Health

6. Fatigue or loss of energy nearly every day psychotherapy, which is usually performed by a trained
7. Feelings of worthlessness or inappropriate guilt nearly psychotherapist.
every day There are several classes of" antidepressant agents with
proven efficacy in relieving depressive symptoms (Table 49).
8. Diminished ability to think or concentrate nearly every
The choice of"agent is primarily based on side effect profile,
da)'
cost, prominent symptoms, and patient preference.
9. Recurrent thoughts of death, recurrent suicidal ideation Treatment should be to full remission of symptoms, and
with or without a specific plan; suicide attempt
symptoms should be monitored regularly. The American
The nine-item Patient Health Questionnaire (PHQ-9) is College of Physicians recommends that treatment for a first
a validated instrument for identifying and assessing severity of episode continue for 4 to 9 months after full remission of"
depression, with a sensitivity of" 80% and specificity of" 92%. symptoms, and that patients with recurrent depression con
Each of"the nine symptoms is scored from 0 (not bothered by sider lifelong therapy.
the symptom at all) to 3 (bothered by the symptom nearly Antidepressants have been associated with a risk of" pre
every day), for a maximum score of 27. A score of 10 or cipitating suicidal ideation in children, adolescents, and young
greater indicates the diagnosis of depression. adults; therefore, close monitoring is required. The risk of sui
Minor (subsyndromal) depression is characterized by the cide in untreated depression, however, is likely much greater.
presence in the preceding 2 weeks of two to four depressive The most commonly prescribed antidepressants are cur
symptoms associated with impaired social functioning, men rendy the selective serotonin reuptake inhibitors (SSRis),
tal health, and healdi perceptions. Dysthymia has a similar which have good efficacy and also treat anxiety syndromes.
level of symptoms as minor depression but symptoms must be SSRIs have an excellent safety profile compared with tricyclic
present most of the time for a duration of 2 years or more. antidepressants, but sexual side effects are common.
Depressive symptoms arc common after the loss of a loved Scrotonin-norepinephrine reuptake inhibitors (SNRIs) may
one; treatment should be considered if the bereaved person be especially helpful in patients with concomitant pain syn
meets die criteria for major depression 2 months after the loss. dromes. Bupropion may be a good alternative for those with
Seasonal affective disorder is a cyclical depression usually sexual side effects on SSRI or SNRI therapy but may lower
occurring in the fall and winter months with improvement in seizure threshold in higher doses and thus is contraindicated
spring and summer. Lack of exposure to sunlight seems to be in patients with seizure disorders. Because of potential inter
the triggering factor. actions with other medications, monoamine oxidase
Premenstrual dysphoric disorder occurs in 3% to 5% of inhibitors (MAOIs) are not frequently used relative to other
menstruating women. It is characterized by recurrent symp agents. However, a transdermal preparation of selegiline is
toms of" depression, anxiety, or emotional lability within 1 available that is the only nonoral antidepressant available. All
week of menstruation and resolving within 1 week after men SSRIs, SNRIs, and MAOIs can cause serotonin syndrome,
struation. Postpartum depression occurs in up to 15% of which is characterized by mental stattis changes, neuromus
women widiin 6 months of" giving birth and can lead to sig cular hyperactivity, and autonomic instability and is poten
nificant negative outcomes in both mother and child, such as tially lethal.
decreased effectiveness at home and work, increased risk of Patients refractory to a single SSRI may respond to a
maternal suicide, and poorer infant-mother bonding. change in therapy, which may include replacement with
another antidepressant, either from the same or a different
Management of Depression class; addition of a second antidepressant; or a psychothera
Most patients with mild to moderate depression can be diag peutic intervention. Patients with severe or refractory depres
nosed and treated by primary care physicians. Before initiat sive symptoms (suicidality, poor cognitive functioning, inter
ing treatment the physician should rule out underlying med ference with activities of daily living) should be referred to
ical illnesses causing depressive symptoms and evaluate any psychiatric providers, who may employ electroconvulsive and
comorbid conditions. Treatment can be multimodal and may other multimodality interventions.
include medication, psychotherapy, or a combination of both. The treatment of seasonal affective disorder involves
Psychiatric referral is recommended for any patient with (1) ftill-spectrum light therapy in addition to antidepressants
suicidal or homicidal ideation, (2) bipolar disorder, (3) psy and CBT.
chotic symptoms, or (4) symptoms refractory to at least two Treatment of premenstrual and postpartum depression is
medications. similar to that of other forms of depression. There are no con
Response rates with cognitive-behavioral therapy traindications to breastfeeding while on antidepressants; how
(CBT), interpersonal therapy, or psychodynamic therapy ever, SSRIs and SNRIs are FDA pregnancy category C, which
are similar to those with medication alone; the best out warrants consideration of risks and benefits of treating mater
comes occur with the combination of medication and formal nal depression versus theoretical risks to the fetus.

109
Mental and Behavioral Health

table 49. Characteristics of Selected Antidepressants

Drug Advantages Disadvantages


SSRIs

Citalopram Few drug interactions Gastrointestinal, sexual side effects


Escitalopram Few drug interactions Gastrointestinal, sexual side effects
Fluoxetine Long half-life reduces risk of Long half-life can lead to accumulation, drug interactions
withdrawal syndrome; effective for common (cytochrome P-450 inhibitor)
anxiety disorders, OCD, PMDD
Paroxetine Effective for anxiety disorders, panic High risk in pregnancy (class D), drug interactions (cytochrome P-
disorder, PTSD, OCD 450 inhibitor), weight gain; high risk for withdrawal syndrome
Sertraline Few drug interactions; effective for
panic disorder, PTSD, OCD, PMDD
SNRIs

Venlafaxine Effective in anxiety disorders Nausea, can increase blood pressure


Desvenlafaxine Effective in anxiety disorders Nausea, can increase blood pressure
Duloxetine Effective in pain conditions, generalized Nausea, urinary retention
anxiety disorder

Tricyclic antidepressants
I
Nortriptyline Drug level monitoring possible, Cardiac toxicity with overdose, anticholinergic effects
analgesic effect
Amitriptyline Analgesic effect, sedating effect Cardiac toxicity with overdose, anticholinergic effects, sedation,
weight gain
Serotonin antagonist/norepinephrine agonist

Mirtazapine Sedating, increased appetite; available Weight gain, sedation


as orally disintegrating tablet

Norepinephrine and dopamine reuptake inhibitor

Bupropion Fewer sexual side effects than SSRIs, Seizure risk


improved concentration, less weight gain
MAOIs Oral formulation can cause hypertensive crisis, serotonin syndrome

Tranylcypromine Good for atypical symptoms Dietary restrictions, hypertensive crisis, hypotension, serotonin
syndrome
Selegiline Fewer dietary restrictions than other Hypotension, serotonin syndrome
transdermal system MAOIs, transdermal

GAD = generalized anxiety disorder; MAOI = monoamine oxidase inhibitor; OCD = obsessive-compulsive disorder; PMDD = premenstrual dysphoric disorder; PTSD = posttraumatic stress disor
der; SNRI = serotonin-norepinephrine reuptake inhibitor; SSRI = selective serotonin reuptake inhibitor.

KEY POINTS anxiety disorder (GAD) is the most common, with a preva
lence of approximately 4% to 6%. GAD is characterized by
Psychiatric referral is recommended for any patient
with depression associated with suicidal or homicidal excessive anxiety and worry about various events or activities
on most days for at least 6 mondis, with difficulty controlling
ideation, bipolar disorder, psychotic symptoms, or
refractory symptoms. worrying. Associated symptoms include fatigue, irritability,
resdessness, insomnia, and difficulty concentrating. Patients
In the treatment of depression, the best outcomes with GAD often have comorbid anxiety disorders, depression,
are achieved with a combination of medication and or substance abuse. Patients with GAD often have somato
psychotherapy. form symptoms, which can make them high utilizers of"health
care resources. Most patients widi GAD or panic disorder
present to their primary care physician or the emergency
Anxiety Disorders department rather than a mental healdi professional. The his
Anxiety disorders are among the most common psychiatric tory should include inquiries into underlying medical condi
disorders in die general population; of these, generalized tions diat can cause anxiety symptoms as well as any history of

11 0
Mental and Behavioral Health

comorbid psychiatric disorders, substance abuse, recent stres well (see MKSAP 16 Neurology). A study of"returning U.S.
sors, coping skills, and family history of psychiatric disorders. veterans reported that patients with mild TBI had a 6-fold
Panic disorder is a syndrome characterized by sudden greater prevalence of PTSD versus those without such an
panic attacks with the sudden onset of" somatic symptoms, injur)'. Patients with PTSD have a higher incidence of marital
which may include chest pain, palpitations, sweating, nausea, and occupational problems and a higher incidence of" suicide
dizziness, dyspnea, and numbness. These symptoms usually dian die general population.
last from 5 to 60 minutes. The diagnosis of panic disorder
requires diat an attack be followed by at least 1 mondi of Treatment
worry about a recurrence of an attack. The incidence of panic PTSD symptoms are complex and often require multimodal
disorder is twice as high in women as in men. About one half" treatments. Early intervention may prevent chronicity of
of patients with panic disorder also have associated agora symptoms. No advantage has been identified for cither psy
phobia, with fears of being in crowds or in places from which chotherapy or pharmacotherapy, although, for patients with
escape would be difficult. Studies have shown higher suicide refractory PTSD, combination therapy may be most useful.
attempt rates in those with panic disorder compared with the Trauma-focused CBT focuses on cognitively refraining dis
general population. Although not common, medical disor torted thinking patterns while gradually re-exposing the
ders potentially presenting widi anxiety-like symptoms need
patient to die traumatic experience to allow desensitization of
to be considered, including cardiac disease, thyroid disease, or
triggered symptoms. Simple stress management interventions
pheochromocytoma. can also be helpful in alleviating some symptoms. The most
Treatment options for GAD and panic disorder include effective medications for PTSD are SSRIs, with positive
medication and psychotherapy. CBT has been shown to be
symptoms (hyperarousal, flashbacks) responding best.
die most effective psychotherapeutic intervention in con
Tricyclic antidepressants may also be used. Benzodiazepines
trolled trials, appears to be equal in efficacy to pharmacologic have not been shown to be effective despite their anxiolytic
interventions, and has a lower relapse rate. SSRIs and SNRIs effect. Finally, die a-blocker prazosin has been demonstrated
have been shown to bc effective for both GAD and panic
to reduce the incidence and severity of" nightmares but not
disorder. Buspirone is another pharmacologic option,
other associated symptoms.
although it may take several weeks to show clinical effect.
Benzodiazepines are frequently used, either alone or in con Social Anxiety Disorder
junction with other treatments, although they carry a risk of
Social anxiety disorder is one of the most common anxiety
dependence and should not be used in those with a history of
substance abuse. Panic disorder diat is severe or refractory disorders, with a lifetime prevalence estimated at 2.4%. It is
characterized by a severe, persistent fear of social or perform
appears to be most amenable to the combination of"CBT and
ance situations, such as public speaking, test taking, or parties.
pharmacotherapy compared with eidier treatment alone.
In these situations, autonomic symptoms of anxiety occur,
Posttraumatic Stress Disorder including blushing, dyspnea, palpitations, and emotional dis
tress. Social anxiety disorder can be generalized or specific to
Clinical Presentation
a single activity. Patients generally realize their fear is excessive
Posttraumatic stress disorder (PTSD) occurs in response to
and often avoid trigger situations, which may lead to impair
exposure to a traumatic event that involves serious threat to ment in social function. Effective treatments include CBT as
oneself or others. PTSD is characterized by at least 1 month
well as SSRI pharmacotherapy. Pharmacotherapy, particularly
of symptoms that include intrusive thoughts about the
with an SSRI, has been demonstrated effective for both short-
trauma, nightmares or flashbacks, avoidance of" reminders of and long-term use. MAOIs have shown efficacy, although to
the event, and hypervigilance with sleep disturbance. To meet
a lesser extent.
DSM-IV criteria, the symptoms must be in each of three
areas: re-experiencing die event, avoiding reminders of the
Obsessive-Compulsive Disorder
event, and heightened arousal. Risk factors for PTSD include
lower socioeconomic status, parental neglect, a family or per Patients with obsessive-compulsive disorder (OCD) report
sonal history of" a psychiatric condition, poor social support, recurrent obsessions or compulsions sufficiently severe to
and initial severity of reaction to the traumatic event. occupy 1 hour daily or result in marked distress or impaired
Common events precipitating PTSD include military combat, social function. Obsessions are persistent ideas, thoughts,
sexual assault, mass displacement or disaster, and severe phys impulses, or images experienced as intrusive and are associ
ical illness. PTSD most commonly presents within 1 month ated widi significant anxiety or distress. Examples include
of the traumatic event, but symptoms can be delayed for more fears of" having left doors unlocked and fears of" germ con
than 6 months. Comorbid psychiatric conditions may include tamination. Compulsions are repetitive behaviors, such as
depression, anxiety, and substance abuse. Traumatic brain injur)' handwashing, checking, ordering, or counting, diat are
(TBI) and postconcussion syndrome are often coexistent as repeated to decrease die anxiety related to the obsessions.

111
Mental and Behavioral Health

CBT, with exposure and response prevention interven Treatment of bipolar disorder is more complex than
tions, is the preferred primary treatment for OCD. unipolar depression and is optimally managed in conjunc
Pharmacodierapy is used in conjunction with CBT in patients tion with a psychiatrist. Treatment is mainly with mood-sta
with severe symptoms or in those with an incomplete bilizing drugs (lithium, valproate, carbamazepine) or lamot
response to CBT alone. SSRIs in higher doses are the primary rigine and is vital given the high recurrence of both
pharmacodierapy, aldiough clomipramine may bc effective; depressive and manic symptoms in the absence of treatment.
some antipsychotic agents may also be useful as adjunctive Although lithium has long been the mainstay of mainte
dierapy in severe cases. nance therapy, it has a narrow therapeutic window with
long-term negative effects on the thyroid and kidneys; in
KEY POINTS
addition, it is teratogenic. For acute manic episodes, the
Cognitive-behavioral dierapy is die most effective combination of either lidiium or valproate with an atypical
nonpharmacologic intervention for generalized anxi antipsychotic agent such as olanzapine, quetiapine or arip
ety disorder and panic disorder and may be equal in iprazole appears to be more effective than either mood sta
efficacy to pharmacologic interventions and have bilizer alone. Adjunctive psychotherapy may help patients
lower relapse rates. adjust to having this chronic illness and may enhance com
Cognitive-behavioral dierapy, widi exposure and pliance with maintenance pharmacotherapy. It is important
response prevention interventions, is the preferred that the history be assessed for manic symptoms in any
primary treatment for obsessive-compulsive disorder. patient being considered for initiation of pharmacologic
treatment for depression, both to identify the presence of
the disorder and to avoid triggering a manic episode, which
Intermittent Explosive Disorder may occur with treatment with SSRIs.
Intermittent explosive disorder (IED), an impulse control
disorder, may affect 0.3% of die U.S. population. It is char Somatoform Disorders
acterized by repeated episodes of aggressive violent behav
iors grossly out of proportion to the situation. Examples Clinical Presentation and Evaluation
include road rage, severe temper tantrums, and domestic Somatization refers to die presence of medically unexplained
abuse. Explosive episodes may be accompanied by feelings of symptoms (MUSs). MUSs are seen in all populations,
irritability or rage and by physical symptoms of tingling, although diey are more common in women, minorities, and
those widi less education or lower socioeconomic status.
tremors, palpitations, or head pressure. Outbursts may result
in injury. Persons with IED often later express remorse or MUSs are relatively common in primary care offices, account
embarrassment. Episodes may occur in clusters or be sepa ing for up to 50% of all symptom-related visits. Aldiough 10%
to 20% of patients in primary care practices have four or more
rated in time.
Treatment is achieved through both CBT and pharma unexplained symptoms, only a small number of these patients
have a true somatoform disorder. True somatoform disorders
cotherapy. Mood stabilizers and anticonvulsant agents (such are psychiatric diseases diat involve persistent medically unex
as carbamazepine, phenytoin, and lidiium) may be effective in
plained symptoms or symptoms diat are out of proportion to
decreasing aggressive behavior. SSRIs are helpful in treating medically expected findings and significandy affect a patient's
comorbid depression in diese patients.
ability to function. Somatoform disorders can cause great dis
tress to patients, families, and physicians; in addition, patients
Bipolar Disorder suffering from these diseases are often high utilizers of die
medical system, subjecting diem to batteries of cosdy and
Bipolar disorder is characterized by manic or hypomanic potentially dangerous diagnostic tests.
mood episodes and depressive episodes and affect up to 2% of Somatization disorder requires that multiple MUSs bc
the general population. A manic episode is marked by a per
present for years in multiple organ systems, although each
sistent period of elevated mood, irritability, lack of" need for
symptom can wax and wane and all need not be present
sleep, racing dioughts, high energy levels, increased talkative simultaneously. To fulfill the diagnostic criteria, symptoms
ness, spending sprees, hypersexuality, and increased self-con need to have started before age 30 years and include gas
fidence, widi possible delusions of grandeur or psychosis. trointestinal, pain, pseudoneurologic, and sexual symptoms.
Bipolar disorder is a leading cause of suicide, and is highly Undifferentiated somatoform disorder requires only a single
associated with substance abuse problems. Bipolar disorder is somatic symptom that is present for at least 6 months (for
thought to be underdiagnosed; many patients presenting example, nausea). Patients with conversion disorder have a
with symptoms of" major depression are not asked about single pseudoneurologic symptom diat is not explained by a
manic symptoms and are therefore not recognized as having medical evaluation and often follows lay understanding of
bipolar disease. neurology (for example, hemiparesis that does not follow

112
Mental and Behavioral Health

crossed corticospinal tracts). In hypochondriasis, patients mis should focus on functioning with symptoms rather than elim
interpret normal bodily sensations and are afraid these symp ination of the symptoms.
toms are manifestations of serious illness. Patients with body No therapy has been shown to be consistently helpful in
dysmorphic disorder are preoccupied with a real (usual!)' treating somatoform disorders. Reassurance can be helpful;
minor) or imagined physical finding (for example, swelling of" however, it rarely leads to a resolution of" symptoms.
the face when examination reveals no edema). Although antidepressants have shown benefit in patients with
The evaluation of possible somatoform disorders medically unexplained symptoms, they have not been consis
requires the consideration of" both a medical and psychiatric tently shown to benefit patients with somatoform disorders.
differential diagnosis. Even when somatoform disorders are Several studies have shown a benefit from CBT, and psychi
strongly suspected, clinicians should perform a thorough his atric evaluation is often appropriate. Such referrals must be
tory and physical examination to construct a differential diag handled widi care because patients with somatoform disor
nosis that could explain a patient's symptoms medically. ders can be sensitive to feelings of abandonment.
Laboratory and other testing should be ordered logically to KEY POINT
evaluate plausible medical diagnoses; extensive and elaborate
In patients with somatoform disorders, primary care
testing to explore unsupported or very unlikely diagnoses
visits should focus on the evaluation of" new or
should be avoided despite a patient's concerns and requests.
changed symptoms and functioning with somatic
Testing should not be ordered to reassure a patient, as tests,
even those with normal results, rare!)' help patients improve symptoms rather than elimination of" the symptoms.
their level of function and increase the risk of iatrogenic com
plications. It is also important to screen for psychiatric comor
bidities, including depression, panic disorder, and substance Eating Disorders
abuse. In addition, clinicians must carefully rule out malin Types of Eating Disorders
gering and factitious disorders. In these conditions, a patient Anorexia nervosa is defined by four diagnostic criteria: an
purposely adopts a physical symptom. Patients with malin abnormally low body weight (<85% of expected) in associa
gering do this for external gain (such as avoidance of work), tion with an intense fear of gaining weight, an overemphasis
whereas those with a factitious disorder do so in order to of body weight on self-evaluation, and, in women and girls,
remain in the sick role. amenorrhea for at least three consecutive menstrual cycles. It
is classified as either restricting (with regular caloric restric
Management tion) or binge-eating/purging (binge eating that may or may
Patients with a somatoform disorder are psychologically not be associated with self-induced vomiting or the misuse of
dependent on potential illness, and unlike conversion or laxatives, diuretics, or enemas). The lifetime prevalence of
malingering, the symptoms expressed are not consciously fab anorexia nervosa is 1.0% to 3.7%, with adolescent girls and
ricated. Treatment for somatoform disorders starts with an young women disproportionately affected. A family history of
honest discussion of the diagnosis. Comorbid medical or psy eating disorders or a diagnosis of childhood anxiety or OCD
chiatric diseases should be treated. Regular office visits should increases the risk for developing anorexia nervosa. The
be scheduled at frequent intervals. Some patients will need SCOFF questionnaire is a brief five-question instrument that
limits on visits or between-visit communications. Visits should can be used in the primary care setting to screen for anorexia
focus on new or changed symptoms. These should be thor nervosa (Table 50).
oughly evaluated to exclude new medical problems, although Physical signs of anorexia nervosa may be obvious on
the disorder may require an increased reliance on physical signs general inspection, as patients may present with parotid gland
rather than symptoms. Otherwise, primary care encounters hyperplasia, dry and brittle hair, lanugo, yellowing skin, and

table so. SCOFF Questionnaire for Screening for Eating Disorders

S Do you make yourself Sick because you feel uncomfortably full?


C Do you worry that you have lost Control over how much you eat?
O Have you recently lost more than One stone (6.4 kg [14 lb]) in a 3-month period?
F Do you believe yourself to be Fat when others say you are too thin?
F Would you say Food dominates your life?

Score one point for each yes. A score of two or more points indicates a likely diagnosis of anorexia nervosa or bulimia.

Adapted with permission from Morgan JF, Reid F, Lacey JH. The SCOFF questionnaire: assessment of a new screening tool for eating disorders. BMJ. 1999:319(7223): 1467-1468. [PMID:
10582927] Copyright 1999. BMJ Publishing Group Ltd.

11 3
Mental and Behavioral Health

xerosis. Other signs include a low BMI (usually <18.5), abnormalities are noted. The hypocstrogenic state that is
bradycardia, orthostatic hypotension, and hypothermia. characteristic of anorexia nervosa results in amenorrhea and
Cognitive impairment and a depressed or anxious mood are low bone mineral density, and current guidelines recommend
common in patients widi anorexia nervosa, and diese patients obtaining a dual-energy x-ray absorptiometry scan if menses
are also at high risk for suicide. are absent for more than 6 months. The most worrisome
In a study reviewing die prognosis of 5590 patients complication of" anorexia nervosa is refeeding syndrome,
treated for anorexia nervosa, the mortality rate was 5%. which occurs when a severely malnourished patient receives
Among survivors, approximately 47% experienced a full aggressive oral, enteral, or parenteral nutritional repletion.
recovery, whereas 33% improved and 20% remained chroni This syndrome is associated widi large volume shifts and sud
cally ill. Younger age (<19 years) and treatment within 3 years den changes in electrolyte levels, resulting in edema, hypo
of diagnosis ma)' be associated with improved recovery rates. magnesemia, hypophosphatemia, hypokalemia, and in rare
Although bulimia nervosa is more common than cases, death. Close monitoring and repletion of electrolytes
anorexia nervosa, affecting approximately 1% to 1.5% of" are essential for avoiding this complication.
women, it may be more difficult to diagnose. This disorder is
characterized by recurrent episodes of binge eating with com Treatment of Eating Disorders
pensatory behavior aimed at preventing weight gain, includ There is limited evidence to indicate the optimal thera
ing self-induced vomiting and misuse of" medications (purg peutic approach in patients with anorexia nervosa.
ing type) or fasting and excessive exercise (nonpurging type). Individual psychotherapy may be most beneficial during
Additional diagnostic criteria include the presence of diese acute refeeding, and diere is weak evidence that CBT is
behaviors at least twice a week for 3 months and die excessive efficacious in preventing relapse after weight has been
influence of body weight and shape on die patient's self-per restored. There is no role for monotherapy with psy
ception. Patients with bulimia nervosa are usually ashamed of chotropic medications in anorexia nervosa, although they
and secretive about their abnormal eating patterns and may are indicated to treat comorbid psychiatric disorders and
not seek medical treatment; nor may bulimia be suspected in may have an adjunctive role in severely ill patients. As
typically normal-weight patients. The SCOFF screening tool, underweight patients are at increased risk for adverse
as well as subtle physical findings, such as dental caries and effects, specific agents should be avoided, including bupro
enlarged salivary glands, may aid in the diagnosis of bulimia pion, tricyclic antidepressants, and MAOIs.
nervosa. Scarring on the dorsum of die hand (Russell sign) In patients widi bulimia nervosa and binge-eating disor
caused by repeated abrasions during self-induced vomiting, is der, in contrast to those with anorexia nervosa, diere is strong
highly suggestive. As widi anorexia nervosa, additional psy evidence to support the use of CBT. Pharmacodierapy is a
chiatric illnesses may accompany bulimia nervosa, including useful adjunct to CBT, and SSRI antidepressants have been
anxiety and depression. shown to be effective and safe for treatment. Fluoxetine has
Eating disorder not odierwise specified (EDNOS) is been FDA-approved for die treatment of bulimia nervosa,
diagnosed in patients who have disordered eating but do not and sertraline may also be effective. Topiramate has been
fulfill the diagnostic criteria for anorexia nervosa or bulimia shown to reduce binge eating and promote weight loss.
nervosa. The most common subtype of EDNOS is binge-eat
KEY POINTS
ing disorder, which has a prevalence of about 2% to 3% in die
general population; approximately one diird of diagnosed Psychotherapy and nutritional support are the pri
patients are male. Characteristic behavior includes consump mary treatments for acutely ill patients widi anorexia
tion of large quantities of food in a 2-hour period with an nervosa.
associated sense of lack of control. Although the binge Cognitive-behavioral therapy is an effective treatment
episodes often occur in secret and patients feels disgust after for bulimia nervosa and binge-eating disorder.
ward, diere is no compensatory behavior after die binge.
Characteristic physical findings include an increased BMI;
some patients may be severely obese. The Eating Attitudes Schizophrenia
Test (www.eat-26.com) is a useful screen for binge-eating dis Schizophrenia has a prevalence of 1% in the general popula
order in the primary care setting. tion and affects women and men equally. It usually begins in
the teenage years or in die early 20s; the strongest risk factor
Medical Complications of Eating Disorders for its development is family history. First-degree relatives of

Hypokalemia, hypomagnesemia, and metabolic alkalosis may persons with schizophrenia have a 6% to 17% lifetime inci
be seen in patients widi anorexia nervosa or bulimia nervosa. dence of die disease. It is manifested by positive and negative
A patient with severe anorexia nervosa may experience signif symptoms. Positive symptoms include paranoid delusions,
icant cardiovascular complications, including bradycardia and hearing voices, and hallucinations; negative symptoms include
orthostatic hypotension; hospitalization is necessary if" these flat affect, social withdrawal, and lack of interest or enjoyment

114
Mental and Behavioral Health

in life. Thought tends to be disorganized, with confused Diagnosing ADHD in an adult patient can be challenging but
speech and rapid shifts in topic. No symptom or sign is should be considered in a patient with a history of inattention
pathognomonic for schizophrenia. Diagnosis is based on die or impulsive behavior and significant functional impairment
presence of signs and symptoms for at least 1 month in dura beginning in early childhood. About 10% to 20% of adults
tion, with some manifestations of" the disease present for at with substance abuse or mood disorders have ADHD; it is
least 6 months. The onset of schizophrenia can be abrupt or therefore important to review the patient's history for evi
insidious. Schizophrenia is thought to have several inciting dence of ADHD in childhood.
factors, but genetics appears to play the biggest role in its The first-line treatment for childhood ADHD is stimu
development. It is important to consider other psychiatric lants, such as amphetamine and methylphenidate. These
diagnoses (depression with psychotic features, bipolar disor agents may be effective in adults but must be used more cau
der) or medical diseases (substance abuse and withdrawal, tiously, particularly in patients with hypertension or cardio
delirium, central nervous system tumors) in the differential vascular disease. The increased risk of substance use disorders
diagnosis, as 22% of diose initially diagnosed with schizo is also of concern in adults with ADHD, as stimulants are
phrenia have their diagnosis changed during subsequent hos controlled substances with high potential for abuse or diver
pitalizations. Patients with schizophrenia should be co-man sion. Atomoxetine is a selective norepinephrine reuptake
aged widi psychiatrists and other mental health professionals inhibitor specifically approved for treatment of ADHD in
whenever possible. adults and has shown benefit. Bupropion and tricyclic anti
Atypical antipsychotic agents, such as clozapine and depressants may also be beneficial. Because the natural history
olanzapine, have less risk of extrapyramidal side effects than of ADHD usually shows gradual improvement with age, it is
traditional antipsychotic agents, such as haloperidol and important to regularly verily the ongoing need for medication
chlorpromazine. However, adverse effects of these newer with periodic "drug holidays" in well controlled patients.
agents confer a higher risk of diabetes, lipid disorders, and CBT has been reported to be effective for ADHD but may
weight gain. There is little consensus about the frequency of be most helpful as an adjunct to medications.
how often these metabolic parameters should be monitored,
KEY POINT
but most recommendations suggest monitoring periodically.
Because attention-deficit/hyperactivity disorder usu
KEY POINT
ally shows gradual improvement with age, it is impor
It is important to consider odier psychiatric diagnoses tant to regularly verify the need for medication in
and medical diseases in the differential diagnosis of adults.
schizophrenia, as 22% of those initially diagnosed
with schizophrenia have their diagnosis changed dur
ing subsequent hospitalizations. Autism Spectrum Disorders
Autism spectrum disorders are characterized by difficulties in
social interactions and repetitive behaviors or narrow inter
Attention-Deficit/Hyperactivity ests. Severe autism accompanied by developmental delay is
Disorder evident in early childhood, while those widi milder symptoms
Attention-deficit/hyperactivity disorder (ADHD) first mani and no cognitive deficits may not be diagnosed until later in
fests in childhood and is characterized by inattention, hyper life. Approximate!)' 0.9% of children have an autism spectrum
activity, and impulsivity accompanied by functional impair disorder, according to Centers for Disease Control and
ment in at least two settings (home, work, school). Although Prevention estimates, and autism spectrum disorders are
many children with ADHD show improvement as they age, thought to be equally prevalent in adults.
symptoms of inattention may persist into adulthood. DSM-IV-TR diagnostic criteria for autistic disorder are
Prevalence in adults is estimated at 4%. ADHD is more likely based on childhood behavior and include impairment in
in first-degree relatives of persons with ADHD. It may be social interaction; impairment in communication; and repet
associated widi neurodevelopmental disorders (cerebral palsy, itive patterns of" behavior, interests, or activities, with onset
autism, learning disabilities) and with psychiatric disorders before age 3 years. Patients with these symptoms without
(substance abuse, mood disorders). significant delay in language or cognitive development may
There are no diagnostic tests for ADHD. The DSM-IV- meet criteria for Asperger disorder. The current draft version
TR includes criteria for ADHD in children but may be appli of the DSM-V, however, has eliminated Asperger disorder in
cable to adults if adapted. For example, functional impair favor of a broader definition of autism spectrum disorder
ment manifests differendy in adult patients, who can have that no longer requires the presence of developmental delay
problems at bodi work and school. Adults with ADHD are but includes impairment of everyday functioning. The new
more likely to have traffic infractions, vehicular accidents, and definition also specifies that symptoms must be present in
spousal separation and divorce than the general population. early childhood but may not become fully manifest until

11 5
Geriatric Medicine

later in life, when social demands exceed capacities. Persons It is vital for a physician to identify the underlying emo
with Asperger disorder frequently come to medical attention tion that is creating difficult)' in order to best address it.
later than autistic children because of their lesser develop Frequent negative emotions in providers include anger, fear
mental deficits. Making the formal diagnosis of an autism of losing control, fear of displeasing, and fear of" harming die
spectrum disorder in adults is difficult and frequently patient, as well as those stemming from unique personal
requires a specialized clinical evaluation using a number of issues, such as fear of incompetence, fear of death, or
diagnostic tools. reminders of illnesses in the provider's personal life.
Managing adults with autism spectrum disorders is One of the first steps in improving a difficult patient
highly influenced by individual patient characteristics. For interaction is recognizing the underlying emotion at work in
diose with difficulty with physical contact, physical examina the situation. Often, being open with the patient about any
tion should be limited only to essential maneuvers, performed patient behaviors that are eliciting the negative reaction can
slowly, and explained in detail before proceeding. Efforts be helpful in alleviating the underlying tension. It is important
should be made to communicate with patients who are that the physician take responsibility for managing his or her
unable to speak as they frequendy are able to comprehend own negative emotion rather than expecting the patient to
and participate in decision making; alternative methods of respond, however, as the physician may be better able to over
communication (pen and paper, pictures, assistive devices) come the inherent conflicts in the situation.
should be considered. Repetitive and stereotyped behaviors Depending on the clinical situation, it may be important
are calming for man)' patients with autism and should not be for the physician to specifically address certain behaviors, such
as limit setting with borderline patients or setting expectations
interrupted unless they are harmful or disruptive. The assis
tance of a caregiver or colleague who is familiar with the indi for frequency of visits with high utilizers. Being observant of
vidual patient and how best to interact with him or her is and compassionate about the underlying emotions that drive
invaluable. patients to become "difficult" is one of the best ways to rem
edy unpleasant interactions. If a complete lack of trust devel
KEY POINT
ops or it becomes obvious that a therapeutic relationship is
Because patients with autism generally have difficulty not possible, the physician should consider transferring care
with physical contact, physical examination should be of the patient to another provider.
limited only to essential maneuvers, performed KEY POINT
slowly, and explained in detail before proceeding.
Both patient and physician factors contribute to diffi
cult patient encounters.
Difficult Patient Encounters
Several studies indicate that physicians experience about 15%
of patient encounters as difficult. Difficult encounters often Geriatric Medicine
involve patients who have a depressive or anxiety disorder,
poorer functional status, unmet expectations, reduced satis Functional Assessment
faction, and greater use of" health care services. Physicians In older patients, functional status is as important as medical
often report the most difficult)' with patients by whom they illness in determining overall well-being. Function in geri
feel manipulated or frustrated, patients who are time con atric patients is a predictor of independent living as well as
suming and have unrealistic demands, patients who do not the development of future medical illnesses. Therefore,
follow recommendations, patients who express anger, and functional assessment and treatment of" disorders associated
those who interrupt physician routine and make extra work. with functional decline must be incorporated into the eval
Patients who have somatization disorder, chronic pain, sub uation and care of elderly patients in both office and hospi
stance abuse, or an undiagnosable medical problem are often tal settings.
labeled as difficult. Patients with identifiable personality dis Aids to functional assessment, beyond history and physi
orders, such as borderline, dependent, histrionic, obsessive, or cal examination, may include screening instruments and a mul-
antisocial personality disorders, also are frequently labeled as
tidisciplinary assessment involving occupational and physical
difficult. therapists as well as physicians trained in geriatric medicine.
Physician characteristics also play a role in generating a Activities of daily living (ADLs) are basic self-care activities,
difficult patient encounter. Physicians who are fatigued or including bathing, dressing, and feeding, diat help determine
harried or develop a dislike for a patient are more likely to a patient's required level of support (Table 51). Instrumental
consider a patient difficult. Additional physician characteris activities of daily living (IADLs) are activities diat are associated
tics include having less experience, having a higher perceived with independent living, including shopping for food, admin
workload, and having poor communication skills. istering one's own medications, and handling finances. To

11 6
Geriatric Medicine

table 51. Indices to Assess Basic and Instrumental | Fall Prevention and Home Safety
Activities of Daily Living Falls are a serious problem for older adults and occur in
Index (time to complete) Functional Activities approximately one third of" community-dwelling adults who
Assessed are 65 years or older and half of" similarly aged hospitalized
adults. Falling is most often related to patient and environ
Katz Index of Independence Bathing
in Activities of Daily Living0 mental factors. Those at greatest risk include persons widi gait
Dressing
(5-10 min) imbalance, stroke, or dementia, or who use an assistive device
Toileting to walk. Chronic conditions associated with falls include
Transferring arthritis, depression, orthostatic hypotension, and visual
Continence deficits. Leg muscle weakness, Parkinson disease, and periph
Feeding eral neuropathy also can increase risk. Falls are associated with
Barthel lndexb increased morbidity and mortality, a decline in functional abil
Feeding
(5-10 min) Bathing ities, and a loss of"independence.
Because of the high prevalence of falls, as well as die asso
Grooming
ciated burden of illness and cost and the potential to improve
Dressing
outcomes, the Institute ofMedicine has identified prevention
Bowels of falls as a priority area. Screening for fall risk and institution
Bladder of" fall prevention strategies has been proposed as a way to
Toilet use limit the impact of falls in the geriatric population. The
Transfers American Geriatrics Society recommends that physicians ask
adults aged 75 years and older if" they have difficulty widi
Mobility
Stairs walking and balance and if the)' have fallen in die past year.
For patients who have fallen, the history should include a
Lawton and Brody Instrumental Ability to use a telephone
Activities of Daily Living Scalec description of the circumstances of the tall as well as a review
Shopping of current medications. Psychotropic medications increase the
(5-10 min) Food preparation risk of falling, and their gradual withdrawal reduces the rate
Housekeeping of falls. Educating physicians about modifying medications in
Laundry the elderly has been shown to reduce the risk of falling.
Mode of transportation Physical examination for patients with a history of falling
in the past year should include assessment of" gait and mobil
Responsibility for own
medications ity; this can be facilitated by using the timed "Up & Go"
Ability to handle finances (TUG) test. The patient is asked to rise from a chair, walk 10
Direct Assessment of Time orientation feet, turn around, walk back, and sit down again in the same
Functional Statusd chair. The physician observes the patient for ease of perform
Communication
(30-35 min) ance, speed, and balance. Patients should perform the task
Transportation
using any routine assistive devices, such as a cane or walker.
Finance The average health)- adult can complete the task in less than
Shopping 10 seconds; those completing the task in more than 14 sec
Eating onds are considered to be at high risk for subsequent falls. A
Dressing and grooming recent pooled analysis of nine cohort studies including nearly

The Katz index is scored by assigning a score of 1 to each activity if it can be completed
35,000 community-dwelling adults with a mean age of"73.5
independently, which is defined as having no supervision, direction, or personal assis years found that increased gait speed was associated with
tance; scores are then added for a range of 0 to 6.
increased survival. In these studies, patients were asked to
''Each item on the Barthel index is assigned a 0 to 10 value, resulting in a score of 0 to
100, with higher scores reflecting independence. walk at their usual pace. The walk distance varied from 2.4 m
The Lawton and Brody scale results in a score of 0 to 8, with a score of 8 representing (8 ft) to 6 m (20 ft), and speed was calculated as meters per
independence and 0 representing total dependence for activities of daily living. second. For men between 75 and 84 years old, mean 10-year
The Direct Assessment of Functional Status is a more complex 85-item instrument that survival was 15% for gait speeds slower than 0.4 m/s and 50%
evaluates individual tasks for each of the skills using direct observation.
for gait speeds 1.4 m/s or faster. For women the same age,
the corresponding 10-year survival rates were 35% and 92%,
ensure accuracy and objectivity, assessment of" these activities respectively. This model, using age, sex, and gait speed, per
should include direct observations of" patients by health care formed as well in predicting survival as more complex mod
personnel when possible. Assessing ADLs and IADLs in the els based on several factors, including chronic conditions,
hospital can facilitate discharge planning. smoking, and blood pressure.

117
Geriatric Medicine

Physical examination should also include assessment for Mild Cognitive Impairment and Dementia
visual deficits and lower extremity joint examination to detect Mild cognitive impairment (MCI), or cognitive impairment
arthritis. Cardiovascular evaluation should include assess without dementia, is associated with decreased quality of life
ment for ordiostatic hypotension, arrhythmia, and, when syn and functional status, loss of" independence, and increased
cope or presyncope is suspected, for carotid sinus hypersensi health care costs. MCI is present in about one fifth of patients
tivity. Neurologic examination should include evaluation of older dian 70 years and may be present in nearly one third of"
cognitive function, lower extremity weakness, and peripheral hospitalized patients older than 75 years. The most common
neuropathy. subtypes include prodromal Alzheimer disease, vascular cog
Finally, the home environment should be evaluated for nitive impairment, and MCI due to medical conditions and
falling hazards. Environmental hazards are responsible for stroke. (In vascular cognitive impairment, cardiovascular or
25% to 66% of falls in the home, and hazards can be found cerebrovascular disease is present, but the cognitive impair
in two thirds or more of homes. Common environmental ment is not temporally linked to a single stroke). MCI may be
hazards include throw rugs; low toilet seats; no grab bar for more common in older patients with depression, neurologic

bathtub, shower, or toilet; no nightlight in the bedroom, conditions, alcohol abuse, and low baseline intellect. Patients
who develop essential tremor after age 65 years may be more
kitchen, or living room; uneven lighting of stairway or
poorly visible step edges; and difficult-to-reach kitchen stor likely to have MCI than either those without essential tremor
or those who develop tremor before age 65 years. Progression
age. Odier hazards include slippery bathing areas, general
from MCI to dementia is about 12% annually but is higher
clutter, difficult-to-reach light switches, and unsafe handrails.
(17%-20%) in those widi stroke and prodromal Alzheimer
Recommendations to improve home safety' include installing
disease. Researchers are evaluating cerebrospinal fluid mark
nonslip stripping to noncarpeted steps, removing all uneven- ers and genetic markers as predictors for progression to
ness in floors (thresholds) and changing chair seat height so
Alzheimer disease. A recent study found that a genetic varia
that the patient's upper and lower legs are at 90-degree angles
tion in the caspase-1 gene predicted accelerated progression
when sitting. If the physician or family member has a concern from MCI to Alzheimer disease over a 2-year period, but the
about the patient's living environment, a home nurse evalua clinical utility of"this genetic test is unknown at this time.
tion for home safety should be arranged. The USPSTF, how The Mini-Mental State Examination (MMSE) has been
ever, was not able to find sufficient evidence for or against the standard screening instrument for cognitive function,
home hazard modification. with a sensitivity of 76% and specificity of 88% for detecting
Persons with a history of falls should undergo the assess cognitive impairment. Scores of 24 to 25 out of 30 suggest
ment detailed above followed by individually tailored inter mild impairment, scores of 19 to 24 suggest mild dementia,
ventions to reduce the risk for future falling. Evidence-based and scores of 10 to 19 suggest moderate dementia. The
interventions include prescribing gait, exercise, and balance MMSE may under-diagnose those with high intellect and
training. A Cochrane Database review of interventions to pre may over-diagnose those with low intellect or with delirium.
vent falls in elderly persons living in the community also A briefer screening tool, the Mini-Cog (sensitivity 76%, speci
found evidence for use of anti-slip shoes in icy conditions, ficity 89%), performs comparably to the MMSE and employs
a three-item recall test followed by the clock-drawing test if
pacemaker installation in patients with carotid sinus hyper
sensitivity, and cataract surgery in die first eye affected. any one of the diree items is missed. If" all three items are
A recent systematic review and meta-analysis recom recalled, no further testing is necessary. The Sweet 16 (sensi
mends prescribing vitamin D at a dose of 800 IU daily for eld tivity 80%, specificity 70%) is a newer instrument that is eas
ier to administer than the MMSE and can be completed in 2
erly patients who have vitamin D deficiency, for those resid
to 3 minutes. The 16 items include eight orientation items,
ing in long-term care facilities, and potentially for all elderly
three immediate-recall and three delayed-recall items, and
adults at increased risk for falling. The Institute of Medicine
two backward digit span items. When screening for demen
recommends a vitamin D intake of 600 units/d for all men
tia, it is also extremely important to ask a family member or
and women aged 51 to 70 years old and 800 units/d for men
and women older than 70 years. In a meta-analysis of adults caregiver about memory loss, personality change, word-find
ing difficulties, changes in activity level, getting lost, and dif
aged 60 years or older, vitamin D supplementation for at least
ficulty in performing ADLs to place testing results into a clin
6 mondis resulted in a 14% relative risk reduction for falls ical context.
(number needed to treat = 15). The proposed mechanism of Although there is no widely accepted treatment for MCI,
action of vitamin D is its beneficial effect on muscle strength
cognitive rehabilitation has been shown to have some effec
and function and on gait. The USPSTF recommends exercise tiveness in improving functioning in some patients. Cognitive
or physical therapy and vitamin D supplementation to prevent rehabilitation is performed by neuropsychologists and occu
falls in community-dwelling adults aged 65 years or older pational therapists and involves using external memorv aids as
who are at increased risk for falls. well as teaching patients organizational and attention skills. A

118
Geriatric Medicine

randomized trial of cognitive rehabilitation for patients with Hearing


MCI widi a mean age of 79 years demonstrated improved Hearing loss is present in one diird of patients aged 65 years
memory at 2 weeks and 4 months. and older and in 80% of those 80 years or older. The most
common cause of" hearing loss is presbycusis, or age-related
Depression hearing loss. Presbycusis results in high-frequency hearing
Depression is common in older adults, with major depressive loss, which typically impairs sound localization and hearing
disorder being present in 6% to 9% of patients older dian 60 the spoken voice (particularly in noisy environments).
years presenting to physicians' offices and some form of Hearing impairment can lead to depression, limited activity,
depression in 25% of persons older than age 60 years. and social isolation. Among die available screening tests, the
Depression in older adults may present with somatic and veg whispered voice test, in which the examiner stands 2 feet
etative symptoms radier than dysphoria. Depression is more behind a seated patient and assesses the ability of" the patient
common in patients residing in institutions and in diose with to repeat a whispered combination of numbers and letters, or
acute or chronic illnesses, including cardiovascular and cere a single question about whether the patient has hearing diffi
brovascular disease, cognitive decline and dementia, and culty seem to be nearly as accurate as hand-held audiometry
bereavement. Medical conditions associated with depression or a detailed hearing loss questionnaire. The USPSTF con
include hypothyroidism, hyperthyroidism ("apathetic hyper cludes that evidence is insufficient to weigh the benefits and
thyroidism"), chronic pain, Parkinson disease, cancer, dia harms of screening for hearing loss in older adults. The
betes mellitus, vitamin Bp deficiency, alcohol abuse, and use Canadian Task Force on the Periodic Health Examination has
of corticosteroids or interferon. Depressed patients present recommended screening older adults for hearing impairment
ing with cognitive decline (pseudodementia) may display using single-question screening, die whispered voice test, or
delayed responses to cognitive test questions compared with audiometry. The use of hearing aids does not result in normal
patients widi true dementia. Treatment of the depression hearing but can improve communication abilities. Only 20%
improves cognitive function in these patients. Because depres of patients who could potentially benefit from using a hear
sion in older adults is often perceived as an expected conse ing aid actually use one. Cochlear implants can be considered
quence of chronic illness, it is frequently undiagnosed and for patients who are not able to distinguish more than 50% of
untreated. words in a test sentence using the worst ear widi a hearing aid
The PHQ-9 (see Mental and Behavioral Health) can be in place.
used to identify and assess severity of" depression in older
adults. When tested in adults aged 65 years and older, its sen Vision
sitivity and specificity for diagnosing major depressive disor Visual impairment, defined as best corrected vision worse
der were 100% and 77%, respectively. The Geriatric than 20/40, is present in 1% of persons aged 65 to 69 years
Depression Scale consists of 15 questions, and a score of 5 or and increases to 17% in those older than 80 years. The most
greater indicates depression. Its sensitivity is 80% to 90% and common causes of" visual impairment in older persons are
its specificity is 70% to 85%. refractive errors, cataracts, and age-related macular degener
The USPSTF has found good evidence that treatment of ation (AMD). Diabetic retinopathy and glaucoma are also
depression in older adults who are identified through screen important causes of visual impairment.
ing in primary care settings decreases clinical morbidity. The USPSTF found insufficient evidence to recommend
Treatment may include antidepressants, psychotherapy, or for or against screening adults for glaucoma and also con
both. Relapse may occur more frequently in older compared cluded that the current evidence is insufficient to recommend
with younger adults. Patients older than 70 years receiving for or against screening for visual acuity in older adults.
selective serotonin reuptake inhibitor (SSRI) dierapy may Although evidence was adequate that early treatment of
have fewer recurrences of" depression if treated for 2 years. refractive error, cataracts, and AMD improves or prevents loss
Evidence is fair diat SSRI use is associated with an increased of visual acuity, evidence that these improvements would
risk for upper gastrointestinal bleeding in older patients, with enhance functional outcomes was inadequate. The American
risk increasing widi age. SSRIs can also cause the syndrome Academy of Ophthalmology recommends comprehensive
of inappropriate antidiuretic hormone secretion (SIADH). eye examinations every 1 to 2 years for persons 65 years or
The use of stimulants can bc considered for some older older who have no risk factors. When screening is performed
patients with apathetic major depressive illness. For elderly by primary care physicians, use of"a visual acuity test (such as
patients with insomnia and weight loss, mirtazapine may be the Snellen eye chart) is recommended.
preferred because of its beneficial effect on these symptoms. Some evidence suggests diat the use of multifocal lenses
For patients with refractory severe depression, electroconvul may increase risk for falls in older adults, probably because
sive dierapy (ECT) can be considered in medically stable looking down through the reading segment of the lens causes
patients. the ground to be out of focus.

119
Geriatric Medicine

The Older Driver (sometimes called subacute rehabilitation); long-term acute


The risk for automobile accidents is increased among older care hospitals (LTACHs), which provide long-term complex
drivers, and both patients and physicians have a responsibility care following hospital discharge, including ventilator care
to reduce diis risk. Although patients often self-restrict their and weaning; advanced healdi care services provided in the
driving as they become aware of driving difficulties, "low home (home health care); and hospice or palliative care.
mileage" drivers may be at the greatest risk. Medical condi Long-term care options include supportive home care, which
tions most likely to cause problems include those affecting includes assistance in performing ADLs; assisted living, which
vision, motor function, and cognition. Specific medical con provides institution-based care in semi-independent units
ditions that increase risk include cataract, arthritis, dizziness, widi variable levels of assistance available; nursing homes,
history of falls, arrhythmia, and seizure disorders, as well as which provide ongoing nursing-level care; and adult day care,
substance abuse and use of sedating medications. Dementia in which care services are provided during the day. It is not
is associated with a two-fold risk for driving accidents. uncommon, particularly in elderly patients being discharged
Although some studies have shown impaired driving in older from the hospital, for patients to receive postacute care but
adults widi MCI, there are not enough data to make clear rec ultimately require a higher level of chronic care dian before
ommendations regarding assessing or restricting drivers with admission. It is, therefore, imperative that physicians have a
MCI. The American Medical Association (AMA) recom basic understanding of" various care options so that they can
mends that physicians assess patients for physical or mental help patients or their representatives make the best possible
impairments diat might adversely affect driving abilities. In decisions for both long-term and transitional care.
addition, older drivers can be evaluated by driver rehabilita Pressures to discharge patients as soon as medical condi
tion specialists (associated with hospital occupational therapy tions requiring hospitalization are resolved may interfere with
departments) who can also make recommendations for safer the process of"determining the best long-term living situation
driving. State-specific physician reporting requirements, for a patient. Primary care physicians are particularly impor
along with tools for patients and physicians, can be found in tant advocates for their patients and can provide valuable
the AMA's Physician's Guide to Assessing and Counseling
input to hospitalists to ensure optimal post-discharge plan
Older Drivers (available at www.ama-assn.org/ama/
ning. Care managers, funded either privately or through pub
pub/physician-resources/public-health/promoting-healthy lic agencies, can also be valuable assets in configuring the best
-lifestyles/geriatric-health/older-driver-safety/assessing living situation for a patient.
-counseling-older-drivers.pagc).
KEY POINTS

Adults aged 75 years and older should be assessed for Polypharmacy


fall risk by asking if they have difficulty with walking Polypharmacy refers to the use of many medications together,
and it tends to be a term used almost exclusively in the con
and balance and if they have fallen in the past year.
text of elderly patients. Ninety percent of noninstitutionalized
Evidence-based interventions to reduce falls in older
patients older than 65 years take at least one medication, and
adults include supplemental vitamin D; modification
approximately 50% take five or more medications each week.
of risk factors in the home; reducing or eliminating
Twelve percent of patients older than 65 years take ten or
psychoactive medications; and prescribing gait, exer more medications each week. As more medications are pre
cise, and balance training to patients with abnormal
scribed, rates of adverse drug reactions and medication errors
gait or balance. rise. More drugs being taken together increases the risk of
Depression in older adults may present with somatic
drug-drug interactions.
and vegetative symptoms rather dian dysphoria. Many drugs used in the elderly have been studied pri
marily in younger patients with significantly longer life
expectancies, and their safety and efficacy in older patients arc-
Levels of Care not well established. Drug metabolism may be altered in the

For patients who cannot live independently at home, either elderly owing to decreased glomerular filtration or underlying
following a hospitalization or as a result of progressive illness as well as to altered pharmacokinetics related to aging.
decline, various care options may be available. Levels of care A recent study found that four medications were responsible
can be divided into postacute and long-term care. Postacute for two thirds of emergency hospitalizations for adverse drug
care options following hospitalization include inpatient reha events. Hospitalizations involving three of them (warfarin,
bilitation (for patients widi stable medical issues able to par insulin, and oral hypoglycemic agents) were related to unin
ticipate in >3 hours/d of therapy); skilled nursing facilities, tentional overdose. Warfarin was implicated most frequently,
for patients requiring care diat must be administered by accounting for one diird of emergency hospitalizations. The
trained nursing personnel or needing rehabilitation services fourth class of drugs, oral antiplatelet agents, were implicated
but are unable to participate in at least 3 hours/day of therapy by acting alone or by interacting with warfarin.

120
Geriatric Medicine

Functional incontinence, defined as simply not getting to die


Frequent, routine review to verify need for medication
and appropriate dosing is an important aspect of optimal geri toilet quickly enough, may occur in patients with significant
atric care. Man)' strategies have been studied to monitor and mobility and cognitive impairments. Determining die type
reduce polypharmacy. Biannual review (or more frequently (or types) of"incontinence guides management.
for higher numbers of" medications taken) of medication lists Because patients may not report incontinence sponta
helps to prevent duplication of medication classes. The neously, the Agency for Healthcare Research and Quality
"Good Palliative-Geriatric Practice" (GP-GP) algorithm for recommends routine screening for all frail older men and
drug discontinuation (available at http://archinte.jamanet women; some groups also recommend screening women
work.com/article.aspx?doi= 10.1001/archinternmed aged 65 years and older for incontinence. Standardized
.2010.355) has been shown to be effective in reducing questionnaires can distinguish urge from stress inconti
polypharmacy and improving mortality and morbidity in nence, including the 3 Incontinence Questions (3IQ)
nursing home inpatients and has been studied in smaller pop (Figure 27). For urge incontinence, die 3IQ has a sensitiv
ulations of community-dwelling outpatients and found to be ity of 75%, specificity of 77%, positive likelihood ratio of
effective. 3.29, and negative likelihood ratio of 0.32. The 3IQ's met
rics for stress incontinence are similar (86%, 60%, 2.13, and
KEY POINTS
0.24, respectively).
Altered pharmacodynamics, multiple medications, The evaluation should include a targeted history,
and increased susceptibility to adverse effects make
including surgeries, instrumentations, and other relevant
polypharmacy a major problem in geriatric patients. interventions; medication review; and physical examination,
Frequent, routine review to verify need for medica including a pelvic examination in women and a digital rec
tion and appropriate dosing is an important aspect of tal examination in men. Men should be asked about prostate
optimal geriatric care. symptoms. Reversible causes should be noted, including
delirium, urinary tract infection, atrophic vaginitis, medica
\
tions, depression, hyperglycemia, impaired mobility, and
Urinary Incontinence fecal impaction. Urinalysis should be performed. Unless
there is high clinical suspicion for neurologic disease or blad
Epidemiology
der outlet obstruction, a post-void residual urine volume
Urinary incontinence, or involuntary urine leakage, affects
determination is not necessary.
one third of middle-aged and older women and 20% of older
men. These numbers likely underestimate its true preva
Treatment
lence, however, as many patients do not report incontinence
to their physician. In addition to female sex, risk factors General recommendations for all patients with urinary incon
include age, diabetes mellitus, obesity, history of vaginal tinence include caffeine restriction and, if overweight or
childbirth, history of gynecologic surgery, pelvic floor mus obese, weight reduction. Excess fluid intake should be
cle weakness, high caffeine intake, tobacco use, menopause, avoided, especially at nighttime, but not at the expense of
and impairment in cognition or mobility. Urinary inconti adequate hydration. Any underlying causes should be
nence in men may result from benign prostatic hyperplasia addressed. Further treatment depends on the type of urinary
incontinence.
(overflow incontinence) or from surgery or radiation ther
apy for prostate cancer.
Behavioral Therapy
Urinary incontinence is associated with excess health care
Pelvic floor muscle training (PFMT, or Kegel exercises) and
expenditures. It increases the risk of falls and may lead to
social isolation, embarrassment, decreased quality of life, bladder training/urge suppression techniques are the two
functional decline, and admission to a nursing home. most effective behavioral therapies. PFMT is considered first-
Effective treatment is available and improves quality of life. line therapy for patients with stress incontinence and is of
Although much of the evidence regarding management of likely benefit in patients with mixed urge and stress inconti
incontinence comes from studies enrolling only women, nence. PFMT exercises, if performed correctly and diligently,
most principles can be generalized to men. strengthen the pubococcygeus muscles that form the pelvic
floor and enhance urinary retention by increasing the tone of
Evaluation the supporting structures of the urethra. The patient is
instructed to contract the pelvic muscles as if trying to inter
Urinary incontinence is categorized as (1) urge incontinence
(loss of urine accompanied by sense of urgency; caused by rupt urination. In women, correct technique may bc assessed
detrusor overreactivity); (2) stress incontinence (loss of"urine by inserting a finger into die vagina and feeling the circum-
with effort, coughing, or sneezing; caused by sphincter vaginal muscles tighten and the pelvic floor move upward.
incompetence); (3) mixed urge and stress incontinence; and Patients should work up to three or four sets often contrac
(4) overflow incontinence (caused by outlet obstruction). tions daily, with contractions lasting 10 seconds. Bladder

121
Geriatric Medicine

1. During the last 3 months, have you leaked urine (even a small amount)?

Yes No
I
Questionnaire completed.

2. During the last 3 months, did you leak urine:


(Check all that apply.)

a. When you were performing some physical activity, such as coughing, sneezing, lifting, or exercise?
b. When you had the urge or the feeling that you needed to empty your bladder, but you could not get to the toilet fast enough?
U c. Without physical activity and without a sense of urgency?

3. During the last 3 months, did you leak urine most often:
(Check only one.)

a. When you were performing some physical activity, such as coughing, sneezing, lifting, or exercise?
b. When you had the urge or the feeling that you needed to empty your bladder, but you could not get to the toilet fast enouqh?
c. Without physical activity and without a sense of urgency?
d. About equally as often with physical activity as with a sense of urgency?

Definitions of type of urinary incontinence are based on responses to question 3:

Response to Question 3 Type of Incontinence


a. Most often with physical activity Stress only or stress predominant
b. Most often with the urge to empty the bladder
Urge only or urge predominant
c. Without physical activity or sense of urgency Other cause only or other cause predominant
d. About equally with physical activity and sense of urgency Mixed

FIGURE 27. The 3 Incontinence Questions (3IQ) for evaluation of urinary incontinence.

iStfi
guish^ between
permissin,,rom B,ownJS-
urge and Bradley
stress urinary CS; SubakAnn
incontinence. LL-Intern
al; Diagnostic Aspects of Incontinence
Med. 2006;144:715-723. Study (DAISy)
[PMID: 16702587) Research
Copyright Group. The
2006, American sensitivity
College and specificity of a simple test to distin
of Physicians

training and suppressive therapy arc indicated for urge and Pharmacologic Therapy
mixed incontinence. Patients are instructed to void regularly Effective pharmacologic therapy is available for treatment of
throughout the day, regardless of urge, and progressively incontinence. Data on long-term continence rates are lacking,
increase the interval between voids. Urge to void outside of
however, and adverse effects may limit long-term tolerability.
the schedule is managed by suppression techniques. The In patients with stress incontinence for whom PFMT has
patient is instructed to contract pelvic floor muscles quickly not been successful, duloxetine, a serotonin and norepineph
three or four times, use a distraction technique (counting rine reuptake inhibitor, is an option. In a systematic review of
backwards from 100), and, when the urge passes, walk to the randomized, controlled trials, duloxetine improved inconti
bathroom to urinate. nence rates and quality of life but did not cure incontinence.
A systematic review of 96 randomized controlled trials For urge incontinence, anticholinergic antimuscarinic
of nonsurgical treatments for urinary incontinence con medications are first-line therapy. Options include oxybu
cluded that PFMT alone improved stress incontinence com
tynin, tolterodine, fesoterodine, darifenacin, solifenacin, and
pared with usual care. The pooled relative risk ratio for con trospium. High-quality head-to-head trials comparing indi
tinence was 7.1. Individual therapy, biofeedback, and use of vidual antimuscarinics are limited. However, all of these
skilled therapists improved outcomes. Bladder training alone
drugs appear to provide similar small benefits in continence
improved symptoms but not continence rates. In studies of rates widiout clear superiority of" newer agents relative to
stress and urge incontinence, PFMT coupled with bladder older medications, except for a slightly lower incidence of
training increased continence rates (pooled relative risk anticholinergic side effects. Anticholinergic agents are con
ratio, 13). traindicated in patients with angle-closure glaucoma.
Prompted voiding (periodical!)' asking the patient about Treatment of prostate-related lower urinary tract symp
incontinence, reminding and assisting the patient to go to die toms is addressed in further detail in Men's Health.
toilet, and providing positive reinforcement for continence) is Medications that have been found to be ineffective for
effective in elderly nursing home residents with functional incontinence are pscudoephedrine (an cc-agonist), oral estro
incontinence. gens (may worsen incontinence), and transdermal and

122
Geriatric Medicine

Pressure ulcers most commonly occur on bony promi


vaginal estrogens. Conclusive long-term data for imipramine,
a tricyclic antidepressant with ct-agonist and anticholinergic nences, usually on the hips and lower extremities. One study
properties, are lacking. reported that 36% of patients widi hip fractures developed
Combined use of behavioral therapy with pharmaco pressure ulcers. It is important to differentiate pressure ulcers
from ulcers related to diabetic neuropathy or venous or arte
logic therapy has not been shown to bc more effective than
medication alone in women with urge incontinence. rial insufficiency, as treatments for these conditions vary.

Although behavioral therapy may reduce incontinence fre


Prevention and Management
quency during active treatment, it does not improve ability
to stop the medication. Preventive interventions are much more cost effective than
the prolonged and intensive efforts required to treat pressure
ulcers. Patients should be assessed at each hospital or nursing
Devices, Injectable Bulking Agents, and Surgery
For stress incontinence, medical devices (pessaries, intravagi- home admission for risks of development of ulcers. Skin
nal devices, urethral plugs, vaginal cones) are available. inspection of those at high risk should be conducted regularly.
Preventive efforts should focus on avoiding friction, shear,
Pessaries are most effective but require fitting by experienced
and heavy moisture, which are often a result of wound
practitioners. The most effective surgeries for stress inconti
nence are sling procedures for intrinsic sphincter deficiency drainage or incontinence. Various types of support surfaces,
including beds, mattresses and overlays, have been studied to
(success rate, 80%-90%), retropubic suspension, needle blad
evaluate their ability to prevent pressure ulcers. Foam mat
der neck suspension, and anterior vaginal repair with plication
tresses that distribute pressure over a larger area (relative risk
of the bladder neck (success rates 79%, 74%, and 65%, respec
[RR], 0.40; 95% CI, 0.21 to 0.74) and medical grade sheep
tively). Although increasingly used, results of" studies of peri skin (RR, 0.56; 95% CI, 0.32 to 0.97) are beneficial in pre
urethral injection of bulking agents (collagen, porcine dermal
venting ulcers in hospitalized patients compared with stan
implant, myoblasts, fibroblasts, and dextran) for treatment of" dard mattresses. The effectiveness of alternating and constant
stress incontinence have been variable.
low-pressure mattresses and overlays is not clear. Scientific evi
For urge incontinence, studies of intradetrusor botu
dence for nutritional supplements, seat cushions, and lotions
linum toxin injection report complete continence in 32% to
is minimal.
86% of patients, widi mean duration of 6 months. Variable Treatment of existing ulcers can be informed by staging
cure rates are seen with sacral nerve stimulation and aug
(Table 52). Treatment should be provided by interdiscipli
mentation cystoplasty.
nary teams, with a focus on addressing predisposing and exac
Condom or indwelling (Foley) urinary catheters increase
erbating factors that initially led to the ulcer. In general, a
risk of urinary tract infections. Their use, except as palliative wide variety of dressings, barriers, and gels may be used, with
or temporizing measures, is not advised. a focus on maintaining a clean and moist wound environment
KEY POINTS and managing exudates. Higher-stage ulcers with eschar may
Pelvic floor muscle training and bladder require surgical or nonsurgical debridement. Attention to
superinfection, frank cellulitis, and potential underlying
training/urge suppression techniques are effective
behavioral therapies for stress urinary incontinence. osteomyelitis is vital. The presence ofinfectious complications
is an indication for systemic antibiotic dierapy.
Pharmacologic options for urinary incontinence treat
Stage I ulcers can generally be treated with transparent
ment include oxybutynin or tolterodine for urge films and do not require debriding. Emphasis should be
incontinence and duloxetine for stress incontinence.
placed on prevention of further progression. Stage II ulcers
can bc treated using an occlusive dressing to keep the area
moist. Wet-to-dry dressings should be avoided because
m Pressure Ulcers debridement is usually unnecessary at diis stage. Stage III and
Clinical Presentation IV ulcers generally require surgical or nonsurgical debride
Pressure ulcers are a common condition, particularly in ment, treatment of wound infection, and appropriate dress
persons who are acutely hospitalized, admitted to nursing ings based on the wound environment. For nonhealing
homes, or receiving home health care. More than 2.5 mil wounds that are stage III or higher, imaging to rule out
lion pressure ulcers are reported nationally in acute care underlying osteomyelitis is indicated. Surgical wound inter
settings alone. Annual treatment costs are estimated at SI 1 vention may be necessary in severe or nonhealing ulcers if
billion. They result from continuous pressure, friction, and conservative measures fail.
shearing forces to the skin. Risk factors include increased Although often used, there is no evidence that supple
age, reduced mobility, reduced level of consciousness, mentation with either vitamin C or zinc is helpful in die
malnutrition, peripheral vascular disease, incontinence, absence of"deficiency. Adequate nutrition to maintain a posi
and poor skin condition. tive anabolic balance should be provided. Negative-pressure

123
Perioperative Medicine

table 52. Classification of Pressure Ulcers

Stage Description

Suspected deep Purple or maroon localized area of discolored, intact skin or blood-filled blister due to damage of
tissue injury underlying soft tissue from pressure and/or shear. May be difficult to detect in persons with dark skin tones.
Stage I Intact skin with nonblanchable redness of a localized area, usually over a bony prominence Darkly
pigmented skin may not have visible blanching; its color may differ from the surrounding area.
Stage II Partial-thickness loss of dermis presenting as a shallow open ulcer with a red-pink wound bed, without
slough. May also present as an intact or open/ruptured serum-filled blister.
Stage III Full-thickness tissue loss. Subcutaneous fat may be visible but bone, tendon, or muscle is not exposed
Slough may be present but does not obscure the depth of tissue loss. May include undermining and
tunneling. Depth varies by anatomic location and may be extremely deep in areas of significant adiposity.
Stage IV Full-thickness tissue loss with exposed bone, tendon, or muscle. Slough or eschar may be present on some
parts of the wound bed. Often includes undermining and tunneling.
Unstageable Full-thickness tissue loss in which the base of the ulcer is covered by slough (yellow, tan, gray, green or
brown) and/or eschar (tan, brown, or black) in the wound bed.

C^a?^.fSlS^^AdVto,y PaPeL PfeSSUfe Ukef S,a96S reViSCd by NPUAR hPV/npuap.org/pr2.hUn. Publ.shed February 2007. Accessed October 6. 201,.

I wound therapy using a vacuum-assisted closure device has


* been shown to enhance patient comfort and is less labor However, preoperative and in-hospital management should
ideally be linked dirough close communication, particularly in
intensive dian standard dierapy, but in three controlled trials,
complex, high-risk patients, in whom coordination and con
it has not been shown to improve objective measures of
sistency of care may lead to avoidance of" unexpected prob
wound healing. lems and improved transition from inpatient to outpatient
KEY POINTS settings.
Skin inspection should be conducted regularly in
Perioperative Te s t i n g p-i
patients at high risk of pressure ulcer.
Patients undergoing even minor surgeries are often asked to *
Foam alternatives to standard mattresses and medical
obtain a comprehensive battery of" laboratory and other
grade sheepskin support surfaces are effective in pre
testing. However, results of up to 5% of tests can be abnor
venting pressure ulcers. mal given chance alone, and this prevalence is increased in
Staging of existing pressure ulcers is helpful in guid older, medically ill patients who often undergo surgery.
ing appropriate therapy. Thus, this approach uncovers a relatively high number of
abnormal test results that are of no clinical significance or
are falsely positive. Each abnormal test typically requires
further testing, which delays surgery, adds expense, rarely
Perioperative Medicine influences perioperative care, and may lead to potentially
dangerous and low-value care. For example, comprehensive
General Recommendations preoperative testing has not been shown to bc helpful in
The preoperative risk assessment allows the internist to cataract surgery and is not endorsed by any major specialty
identify and mitigate, if possible, complications from sur society or payer.
gery for which the patient is at higher-than-baseline risk. Preoperative tests should be based on known or sus
In addition, the preoperative risk assessment can serve an
pected comorbidities and should only be ordered when a
important patient safety role by aiding with medication result will alter management. For example, pregnancy test
reconciliation, noting baseline abnormalities in physical ing should be conducted in women of reproductive age, cer
examination, and, if appropriate, outlining goals of care. vical spine radiographs should be ordered to assess for odon
Although there is no set time to perform a preoperative toid-axial stability in patients with rheumatoid ardiritis,
risk assessment, scheduling a visit 3 to 4 weeks before a serum creatinine and calculated glomerular filtration rate-
proposed procedure allows time to complete a diagnostic should bc measured in patients with or at risk for chronic
evaluation and implement preoperative management
kidney disease, and serum potassium level should be
without delaying surgery. obtained in patients taking diuretics. In each case, the test is
Internists are also consulted to assist in the postoperative performed to evaluate for evidence of" a condition that can
care of patients; these internists may be hospitalists or other have a silent presentation or assess an important physiologic
consultants who did not perform the preoperative evaluation. variable likely to be abnormal.

124
Perioperative Medicine

Perioperative Medication Management the most widely used. This risk score was developed in 1999
The preoperative evaluation should explicitly address periop based on a cohort of patients undergoing noncardiac surgery.
erative medication management. The internist should collect Clinicians can quickly calculate a RCRI score based on history
a comprehensive medication list that includes herbal and and easily available laboratory testing (Table 54, page 128).
over-the-counter medications. This list is essential for bodi The RCRI is considered to be the best available prognostic
perioperative management and medication reconciliation. scoring system to distinguish low- versus high-risk patients.
Medication reconciliation, or ensuring continuity of Recommendations from the American College of
medications from the preoperative period until full recovery, Cardiology/American Heart Association (ACC/AHA)
is an important component of perioperative management. regarding perioperative cardiovascular evaluation for noncar
Alterations in medication associated with surgery or other diac surgery are presented in Figure 28 (page 127).
interventions are a source of potential medication errors but Regardless of their cardiac risk, patients who need emergent
also represent an opportunity to review a patient's medication singer)' should not delay surgery for preoperative testing.
regimen for need and for appropriate dosing. Consultation for these patients will necessarily focus on man
There is a relative paucity of either evidence or consen aging postoperative complications. Conversely, patients with
sus regarding perioperative medication use. Specific recom "active cardiac conditions" (see Figure 28) are at extremely
mendations exist for the management of aspirin, clopidogrel, high risk for perioperative cardiac complications. Elective sur
anticoagulants, antihypertensive agents, oral hypoglycemic gery for these patients, including minor surgery, should be
agents, and short-acting insulins (see individual sections and deferred until these conditions have been treated.
Table 53). For other medications, management is usually ACC/AHA recommendations highlight three groups of
based on an individualized assessment of the need for the patients in whom the risk of" perioperative cardiac complica
treatment and the potential consequences of"either continu tions is low enough that preoperative cardiac evaluation is
ing, stopping, or adjusting the dose of the medication in the rarely, if" ever, warranted: patients undergoing low-risk sur
perioperative period. gery, those who have at least moderate exercise capacity (>4
Patients are routinely advised to fast on the morning of metabolic equivalents [METs]), and diose who have low
surgery. The true risk of perioperative aspiration in nontast- exercise capacity but no RCRI risk factors. However, preop
ing patients, however, is unclear, and medications deemed erative noninvasive cardiac testing should be considered in
necessary can be taken widi a small amount of water on the patients with poor or unknown exercise tolerance based on
day of surgery. ED the presence and number of" RCRI risk factors and the nature
of the surgery, provided that the results will impact manage
KEY POINTS
ment. For patients in whom cardiac testing is pursued, exer
Preoperative imaging and laboratory tests should be
cise stress testing is preferred. In general, exercise stress tests
ordered only when an abnormal result is suspected
provide additional information, specifically exercise tolerance,
on the basis of known or suspected comorbidities and
compared with pharmacologic stress tests. However, many
when such a result would alter management.
patients requiring preoperative stress testing have limited
As part of the preoperative evaluation, a list of all exercise tolerance and thus require a pharmacologic stress
medications taken by the patient should be gener test. Although both nuclear stress tests and dobutamine stress
ated; those for which risk outweighs benefit during echocardiograms can predict perioperative cardiac events,
the perioperative period should be temporarily dis data from meta-analyses suggest that dobutamine stress
continued.
echocardiography is superior.
Coronary revascularization is not routinely recom
mended for patients with CAD before noncardiac surgery.
f7| Cardiovascular Perioperative Coronary artery bypass grafting has considerable risks
Management (which may be more than the proposed surgery) and should
Unexpected perioperative myocardial infarction in patients be reserved for patients who already meet criteria for revas
undergoing noncardiac surgery is a much feared complica cularization, including those with stable angina with high-
tion, although it impacts less than 5% of patients. Searching risk lesions, such as left main or three-vessel disease.
for undiagnosed coronary artery disease (CAD) in unsclectcd Similarly, percutaneous coronary intervention (PCI) gener
patients, therefore, is a costly and low-yield strategy that ally is reserved for patients already meeting criteria for PCI,
places patients at risk for delayed surgery and complications such as those with unstable angina, and mostly has not been
from diagnostic procedures. Prevention of perioperative shown to be helpful preoperatively in patients without such
myocardial infarction now emphasizes cardiac evaluation in conditions. If PCI is performed preoperatively, the choice of
patients at highest risk for these complications. intervention (balloon angioplasty, bare metal stent, drug-
Although a number of risk stratification schemas have eluting stent) should take into account both the nature of
been developed, the Revised Cardiac Risk Index (RCRI) is the atherosclerotic lesion and urgency of the surgery, as

125
Perioperative Medicine

table 53. Suggested Perioperative Medication Management


Medication Class Recommendation Comments

Anticoagulant Continue for minor surgery. Discontinue before Bridging with heparin indicated for high-risk patients
major surgery: 6 h for intravenous heparin; and possibly moderate-risk patients (see text, Table 57,
12-24 h for LMWH; 3-5 d for warfarin; 1-2 d and Table 58).
(normal kidney function), 3-5 d (creatinine
clearance <50 mL/min) for dabigatran.
Antiplatelet Clopidogrel: discontinue 5-7 d before surgery; Aspirin and clopidogrel use in patients with cardiac
patients with cardiac stent may require continuation. stent and/or at high risk is controversial.
Aspirin: continue if minor surgery. Continue if Aspirin should be started before CABG.
indication is recent myocardial infarction (up to
6 months), cardiac stent, or high risk for coronary
event; otherwise, discontinue 7-10 d before major
surgery (other than CABG)
Cardiovascular Continue p-blockers, calcium channel blockers, ACEIs and ARBs can promote intraoperative
nitrates, antiarrhythmia agents. ACEIs and ARBs hypotension, especially in patients with hypovolemia;
should be used with caution. Diuretics optional perioperative use, especially in persons with left
(usually withheld). ventricular dysfunction, is controversial.
Lipid lowering Continue statins; hold cholestyramine.
Pulmonary Continue controller and rescue inhalers as well as
systemic corticosteroids (if used). Probably continue
leukotriene antagonists and lipoxygenase inhibitors.
Gastrointestinal Continue H2 receptor blockers and proton pump
inhibitors.

Hypoglycemic agents Oral hypoglycemic agents: discontinue 12-72 h Hypoglycemia is more dangerous than hyperglycemia;
before surgery depending upon half-life of the caution to always have some basal insulin in patients
drug and risk of hypoglycemia. with type 1 diabetes.
Short-acting insulin: hold morning of surgery; may
need dose reduction preoperatively if modified diet
(e.g., gastrointestinal surgery).
Long-acting insulin: reduce dose, typically to one-half
to two-thirds of usual dose.

Thyroid Continue thyroid replacement,


propylthiouracil, methimazole.
Corticosteroids Continue; increase to stress doses if indicated. Stress-dose corticosteroids for patients taking >10 mg/d
prednisone for >3 weeks.
Estrogen Discontinue several weeks before surgery. May
continue oral contraceptives and increase level of
deep venous thrombosis prophylaxis.
Psychiatric Discontinue MAOIs 10-14 d before surgery; SSRIs and Paucity of evidence, although most agents confer at
TCAs can either be continued or tapered 2-3 weeks least some theoretical risk. Risk of serotonin syndrome
before surgery. Continue antipsychotic medications. with some anesthetic agents. Must weigh risks of
Can continue lithium, although some experts taper continuing vs. stopping. May wish to consult with
and discontinue several days before surgery. psychiatrist.
Neurologic Continue anticonvulsants. May continue
antiparkinsonian agents, although some
experts may discontinue the night before surgery.
Discontinue Alzheimer drugs.
Herbal Discontinue up to 1 week before surgery.
Analgesic NSAIDs and COX-2 inhibitors are usually discontinued
7 d before surgery. Long-acting narcotics continued
or dose reduced.
Immunomodulators Methotrexate should be continued; other agents Paucity of data; risk of disease flare balanced against
have not been studied but are usually continued. risk of adverse reaction from medication.
ACEI = angiotensin-converting enzyme inhibitor; ARB = angiotensin receptor blocker; CABG = coronary artery bypass grafting; COX-2 = cyclooxygenase-2; LMWH = low-molecular-weight
heparin; MAOI = monoamine oxidase inhibitor; SSRI = selective serotonin reuptake inhibitor; TCA = tricyclic antidepressant.

126
Perioperative Medicine

Yes Proceed with surgery


Stepl iMeea Tor emergency surgery r (Optimize medical therapy
as time permits)

No
''
Yes Delay surgery for further
Step 2 Active cardiac conditions3? evaluation/treatment

No

b-, Yes
Step 3 Proceed with surgery

No

Adequate functional capacity Yes


Step 4 Proceed with surgery0
(>4 METs) without symptoms?

None
No or Proceed with surgery
unknown

1-2 Proceed with surgery; consider


How many clinical perioperative heart rate control
Step5 risk factors ? and/or noninvasive testing
>3; (if it will change management)
intermediate-
ns k surgery5

Consider noninvasive testing if



it will change management
>3; vascular
surgery

FIGURE 28. Perioperative cardiac evaluation and care for noncardiac surgery. MET = metabolic equivalent.
Active cardiac conditions: unstable coronary syndromes (myocardial infarction <30 days ago. unstable or severe angina), decompensated heart failure, significant arrhythmia, severe valvular
disease.

tow-risk surgical procedure examples: endoscopic, superficial, breast, cataract, ambulatory.

'Consider noninvasive testing in patients undergoing vascular surgery with >l-2 risk factors if it will change management.

dClinical risk factors: (1) history of heart disease, (2) history of compensated or prior heart failure, (3) history of cerebrovascular disease, (4) kidney insufficiency, (5) diabetes mellitus. Unlike the
Revised Cardiac Risk Index (RCRI), the ACC/AHA guidelines do not limit diabetes to insulin-requiring to be considered a risk factor.

elntermediate-risk surgery: intraperitoneal, intrathoracic, endovascular aortic aneurysm repair, carotid endarterectomy, head/neck, orthopedic, prostate.

Vascular surgery: aortic or other major vascular surgery; peripheral vascular surgery.

Recommendations from Fleisher LA, Beckman JA, Brown KA, et al. 2009 ACCF/AHA focused update on perioperative beta blockade incorporated into the ACC/AHA 2007 guidelines on periopera
tive cardiovascular evaluation and care for noncardiac surgery: a report of the American College of Cardiology Foundation/American Heart Association Task Force on Practice Guidelines.
Circulation. 2009; 120(2 l):e 180. [PMID: 198844731

stenting requires postprocedural antiplatelet therapy that weeks before surgery at a low dose, and then titrated to con
will likely delay surgery (see Hematologic Perioperative trol heart rate and blood pressure without hypotension or
C0NT.
Management, below). bradycardia. Although there are no recommendations on
P-Blockers have been studied extensively in the context the duration of P-blocker therapy started in the perioperative
of perioperative care, with sometimes conflicting results. period, most of these patients have other reasons to be on
According to recommendations from the ACC/AHA, P-blockers and should be continued on these medications
patients who are already using a ^-blocker should not stop indefinitely.
this medication preoperatively. The ACC/AHA recom Preoperative heart failure is an independent risk factor for
mends p-blocker use for patients with CAD who are under perioperative cardiac morbidity even in the absence of CAD.
going vascular surgery. It also recommends ^-blockers in In general, patients with decompensated heart failure should
patients undergoing intermediate or higher risk surgeries, have surgery deferred if possible. Beyond this there are no
including vascular surgeries, who have more than one RCRI specific consensus recommendations for die perioperative
risk factor. If used, P-blockers should ideally be started a few management of patients with heart failure. Q

127
Perioperative Medicine

table 54. Revised Cardiac Risk Index (RCRI) table 55. Risk factors for Perioperative Pulmonary
Assign 1 point each for: Complications
Patient-Related Surgery-Related
History of ischemic heart disease
Major Major
Compensated or prior chronic heart failure
Diabetes mellitus requiring insulin Older age Intrathoracic, intra-abdominal
COPD surgeries
Chronic kidney disease (creatinine >2.0 mg/dL [176.8 pmol/L])
Chronic heart failure Surgeries lasting >3 hours
History of cerebrovascular disease
Poor general health status Emergency surgery
Higher risk surgery (intrathoracic, intraperitoneal, supra-
and/or functional
inguinal vascular)
dependence
Risk by RCRI Score3
Smoking
Number of Points Risk Low serum albumin level
0 0.4% Kidney dysfunction
1 1.0% Minor or Possible Minor or Possible
2 2.4%
Obesity General anesthesia
>3 5.4%
Sleep apnea
Risk of perioperative cardiac death, nonfatal myocardial infarction, or nonfatal cardiac
arrest.

Adapted from Devereaux PJ, Goldman L, Cook DJ. Gilbert K, Leslie K, Guyatt GH.
Perioperative cardiac events in patients undergoing noncardiac surgery: a review of the small and not apparent in every study, and the risk of pul
magnitude of the problem, the pathophysiology of the events and methods to estimate
and communicate risk. CMAJ. 2005;173(6):627-634. [PMID: 16157727) monary complications should not drive the selection of
type of anesthesia. There is no correlation between FEV,
or FVC and perioperative pulmonary complications.
Therefore, preoperative spirometry should be reserved for
KEY POINTS patients with unexplained dyspnea.
Preoperative cardiac stress testing should be reserved Few interventions have been shown to reduce the risk
for patients with reduced exercise tolerance and mul of perioperative pulmonary complications. Pre- and postop
tiple cardiovascular risk factors. erative lung volume expansion, either via deep breathing or
incentive spirometry, reduces the risk for pulmonary com
In patients not already on a P-blocker, perioperative
plications. In patients with poorly controlled asdima, sys
p-blocker use is recommended only for those with temic corticosteroids can be used perioperatively. Patients
coronary artery disease who are undergoing vascular with suboptimally controlled COPD should be treated
surgery and for those undergoing other intermediate
or higher risk surgery who have more than one car aggressively with inhaled bronchodilators and probably sys
temic corticosteroids. Patients should have adequate pain
diac risk factor.
control postoperatively to minimize atelectasis. Smoking is
clearly associated with postoperative complications, includ
1 Pulmonary Perioperative ing both pulmonary and nonpulmonary complications.
However, there is mixed evidence regarding the benefit of
Management smoking cessation less than 2 months before surgery, with
Perioperative pulmonary complications, including prolonged some studies suggesting recent quitters may have higher
and unexpected intubation, occur after 3% to 13% of surgical rates of perioperative pulmonary complications. Smoking
procedures. Many risk factors are associated with periopera cessation at least 8 weeks before surgery, however, is clearly
tive pulmonary complications (Table 55). Patients with beneficial. In patients at highest risk for perioperative pul
COPD are two to five times more likely to experience such monary complications, the internist, surgeon, and anesthe
complications, and patients with more severe COPD are likely siologist should collaborate closely to assess the risk/benefit
at higher risk. ratio for surgery and consider alternative surgical or nonsur
The type of surgery is an extremely important predic gical interventions for the patient.
tor of perioperative pulmonary complications. The high Obstructive sleep apnea (OSA) is associated with dif
est-risk surgeries are thoracic surgeries, abdominal aortic ficult intubation and an increased risk of postoperative
aneurysm repairs, and abdominal surgeries, especially open apnea. The importance of OSA in the perioperative period
procedures near the diaphragm. General anesthesia may may be underappreciated, in part because of the high
slightly increase the risk of pulmonary complications com prevalence of undiagnosed OSA in the community. The
pared with spinal anesthesia. However, the added risk is STOP-BANG questionnaire, a series of eight yes-or-no

128
Perioperative Medicine

questions, is an easy-to-use screening tool for OSA (Table 56). to clinical risk factors and proposed surgery (Table 57).
Three or more positive responses have a sensitivity of 92% Preoperative evaluation should focus on assessing for clinical
CONT.
and specificity of 63% for predicting perioperative pul risk factors.
monary complications. In patients with known or sus Early ambulation is often sufficient postoperative pro
pected OSA, this information should be made known to phylaxis in patients with a low risk of VTE. In patients with
the anesthesiologist preoperatively and the patient moni a high risk of bleeding or for whom postoperative bleed
tored closely postoperatively, possibly with extended ing could be catastrophic (for example, neurosurgery
observation in an intensive care or step-down unit. These patients), postoperative mechanical prophylaxis with pneu
patients may also require continuous or bilevel positive matic compression devices can be used alone; elastic grad
airway pressure. Q uated compression stockings are not recommended owing
to lack of clear efficacy and risk for lower extremity skin
KEY POINTS
damage. For the remainder of patients, drug prophylaxis
Preoperative spirometry should be reserved for
for VTE with low-molecular-weight heparin (LMWH),
patients with unexplained dyspnea. fondaparinux, subcutaneous unfractionated heparin
Pre- and postoperative lung volume expansion, either (UFH), or warfarin should be provided unless the assessed
via deep breathing or incentive spirometry, reduces risk of bleeding outweighs the likely benefits (see MKSAP
the risk for pulmonary complications. 16 Hematology and Oncology). Prophylactic anticoagu
Obstructive sleep apnea (OSA) is associated with lants should be withheld until the risk of postoperative
perioperative pulmonary complications, and preoper bleeding is low, at least 12 hours after surgery. In general,
ative screening for OSA is warranted because of the prophylaxis should be continued until discharge. Patients
with orthopedic surgeries, abdominal or gynecologic sur
high prevalence of undiagnosed OSA in the commu
nity and its potential for causing perioperative pul gery for malignancy, or previous VTE should receive pro
monary complications. phylaxis for up to 5 weeks after surgery.
Inferior vena cava (IVC) filters have been used for VTE
prophylaxis in selected patients at high risk of VTE, especially
F1 Hematologic Perioperative those who have had a VIT within 1 to 3 months before sur
gery and for whom anticoagulants will be contraindicated
Management postoperatively. Removable IVC filters are reasonable for
Venous Thromboembolism Prophylaxis these patients. There is no evidence supporting the use of
Perioperative venous thromboembolism (VTE) is a major
prophylactic IVC filters in other populations, including
preventable surgical complication. The risk of VTE is related
patients undergoing bariatric surgery, and their routine use is
discouraged.
TABLE 56. STOP-BANG Screening Tool for Obstructive
Sleep Apnea I Perioperative Management of Warfarin
Parameter Question Therapy
Do you snore loudly (louder than talking or Depending on the patient's thrombotic risk and the type
Snoring
loud enough to be heard through closed of procedure, perioperative management options for
doors)? patients using warfarin are to stop the warfarin during the
Tired Do you often feel tired, fatigued, or sleepy perioperative period, stop the warfarin and bridge with a
during daytime? parenteral anticoagulant, or continue the warfarin
Observed Has anyone observed you stop breathing throughout the perioperative period. Patients undergoing
during your sleep? procedures considered low risk for bleeding, including
Blood pressure Do you have or are you being treated for cataract surgery, dental procedures, and biopsy of non-
high blood pressure?
major organs, should continue warfarin during the peri
BMI BMI more than 35?
operative period, with a target INR of 1.3 to 1.5 at the
Age Age over 50 years? time of surgery. In patients using warfarin who are under
Neck Neck circumference greater than 40 cm? going other surgeries, management of anticoagulant ther
circumference apy is guided by the patient's risk of perioperative throm
Gender Gender male? bosis (Tabic 58). Patients at low risk for thrombosis
should usually discontinue warfarin 4 to 5 days preopera
Scoring: High risk of obstructive sleep apnea: answering yes to three or more items; low
risk of OSA: answering yes to less than three items. tively without bridging therapy. In patients at intermedi
Adapted with permission from Chung f, Yegneswaran B, Liao P, et al. STOP question ate or high risk for perioperative thrombosis, warfarin also
naire: a tool to screen patients for obstructive sleep apnea. Anesthesiology.
2008; 108(5):812-821. [PMID: 18431116) Copyright 2008. Lippincott Williams & should be discontinued 4 to 5 days preoperatively; inter
Wilkins.
mediate-risk patients should be considered for bridging,

129
Perioperative Medicine

TABLE 57. Surgical Risk Stratification and Associated VTE Incidence

Patient Risk R i s k F a c t o r S t r a t i fi c a t i o n VTE Incidence Without


Prophylaxis
Low Minor surgery in patients younger than 40 years with no additional risk factors3 CalfveinDVT:2%
Proximal vein DVT: 0.4%
Clinical PE: 0.2%
Fatal PE: 0.002%
Moderate Nonmajor surgery in patients aged 40-60 years with no clinical risk factors Calf vein DVT: 10%-20%
Minor surgery lasting <30 min in patients with clinical risk factors Proximal DVT: 2%-4%
Major surgery in patients younger than 40 years with no clinical risk factors Clinical PE: 1 %-2%
Fatal PE: 0.1 %-0.4%
High Nonmajor surgery in patients older than 60 years or with clinical risk factors Calf vein DVT: 20%-40%
Major surgery in patients older than 40 years or with clinical risk factors Proximal DVT: 4%-8%
Clinical PE: 2%-4%
Fatal PE: 0.4%-1 %
Very high Major surgery in patients older than 40 years with history of VTE, cancer, Calf vein DVT: 40%-80%
or certain hypercoagulable states
Proximal DVT: 10%-20%
Hip or knee arthroplasty
Clinical PE: 4%-10%
Hip fracture surgery
Fatal PE: 0.2%-5%
Major trauma
Spinal cord injury
DVT = deep venous thrombosis; PE = pulmonary embolism; VTE = venous thromboembolism.

Potential risk factors include heart failure, nephrotic syndrome, pregnancy, estrogen use, use of general anesthesia, acute respiratory failure, active cancer, stroke with paresis, history of VTE,
acute infectious illness, age >60 years, thrombophilia, acute rheumatic disease, inflammatory bowel disease, obesity, trauma, institutionalization, and immobility.

Adapted from PIER. Venous thromboembolism prophylaxis in the surgical patient, http://pier.acponline.org/physicians/diseases/periopr830/periopr830.html. Accessed 8 December 2011.

and high-risk patients should be bridged with either Perioperative Management of Antiplatelet
LMWH at therapeutic doses or UFH. Bridging should be Medications, Coagulopathies, and
C0NT.
started 1 to 2 days after the last dose of warfarin. For Thrombocytopenia
patients at intermediate thrombotic risk, LMWH at pro Patients undergoing surgeries with low risk for bleeding can
phylactic doses is an option, especially for patients with continue aspirin and/or clopidogrel therapy. Patients receiv
intermediate risk of thrombosis. The decision of which ing antiplatelet therapy for recent myocardial infarction or
agent to use and at what dosage should be based on cardiac stent placement should not have this therapy inter
patient-specific risks and benefits. rupted for 6 weeks for a bare metal stent or myocardial infarc
Bridging therapy with LMWH should be stopped 24 tion or for 1 year for a drug-eluting stent. Low-risk surgery
hours preoperatively and UFH 4 to 6 hours preopera can usually be performed while the patient is on antiplatelet
tively. Patients with an intermediate thrombotic risk who therapy, whereas most other surgeries should bc delayed if
are receiving LMWH for bridging can bc given a half dose possible. Antiplatelet dierapy is usually avoided in patients
for the last injection before surgery to minimize the risk undergoing neurosurgery or other surgeries in which the
of intraoperative bleeding. LMWH or UFH can be sequelae of postoperative hemorrhage could be catastrophic.
resumed as early as 12 to 24 hours after surgery, although Conversely, aspirin therapy is recommended before coronary
the exact timing should depend on the patient's indication artery bypass grafting (CABG), although other antiplatelet
for anticoagulation and bleeding risk. Patients at interme agents should be discontinued before elective CABG. There
diate risk for thrombosis can restart LMWH at prophy is little evidence or consensus regarding antiplatelet therapy in
lactic doses; those at high risk should be treated at thera other instances, and decisions regarding clinical care should
peutic doses. Warfarin should be restarted once the be based on a patient's individualized risks and benefits.
patient is at low risk for bleeding, and bridging therapy is Aspirin therapy may be safe in patients undergoing other sur
stopped once the INR is therapeutic. geries, although it is often stopped 7 to 10 days before

130
Perioperative Medicine

TABLE 58. Perioperative Bridging Strategies for Patients Using Warfarin


Risk Conditions Bridging Strategy

Low VTE >12 months ago and no additional risk factors Discontinue warfarin; no bridging
Atrial fibrillation with CHADS2 score <2 without
prior CVA
Intermediate VTE within 3-12 months or history of recurrent VTE Discontinue warfarin; consider bridging with LMWH
(therapeutic or prophylactic dose) or UFH based on
Active malignancy (treated within 6 months or assessment of individual patient and surgery-related
palliatively) factors
Known hypercoagulable state
Atrial fibrillation with CHADS2 score of 3 or 4
without prior CVA
Newer mechanical heart valve in nonmitral position

High VTE <3 months ago or VTE with thrombophilia Discontinue warfarin; bridge with LMWH (therapeutic
dose) or UFH
CVA <6 months ago
Atrial fibrillation with CHADS2 score >4 or prior CVA
Mechanical heart valve in mitral position or older
aortic prosthesis

CVA = cerebrovascular accident; LMWH = low-molecular-weight heparin; UFH = unfractionated heparin; VTE = venous thromboembolism.

elective surgery. Antiplatelet therapy is usually restarted 24 often helpful in the perioperative management of these
hours alter surgery unless there is evidence of or elevated risk patients.
CONT.
for postoperative bleeding.
All patients should be carefully screened preoperatively Perioperative Management of Anemia
for signs or symptoms of underlying bleeding disorders and Patients with a preoperative hemoglobin concentration below
anemia. Laboratory testing should be reserved for patients in 6 g/dL (60 g/L) or a postoperative hemoglobin concentra
whom there is a reasonable probability of an abnormal test, tion below 7 g/dL (70 g/L) have worse outcomes than those
and is not required as a routine component of preoperative with higher hemoglobin levels. Patients with either known
evaluation. A complete blood count, and possibly coagulation anemia or anemia discovered as part of the preoperative eval
uation should be evaluated for an underlying cause. Patients
studies, can be obtained in patients undergoing surgery with
a high risk for significant bleeding as well as in patients with who require iron, vitamin B,,, or folic acid supplements
known or suspected hepatic dysfunction. A preoperative pro should receive these therapies perioperatively. Patients with
anemia of kidney disease or other anemias that are potentially
thrombin time may also be reasonable in Ashkenazi Jews,
responsive to exogenous erythropoietin should be treated
approximately 3% of whom have low factor XI levels from a
preoperatively only if blood losses are expected to be large and
hereditary deficiency.
transfusion is contraindicated.
In patients with known quantitative platelet deficiencies,
Transfusion is reserved for patients with symptomatic
the platelet count should be increased to above 50,000/pX
anemia, a preoperative hemoglobin concentration below 6
(50 x 109/L) before elective surgery, if possible. Patients with
g/dL (60 g/L), postoperative hemoglobin concentration
immune thrombocytopenic purpura should receive corticos
below 7 g/dL (70 g/L), or patients with symptomatic car
teroids, intravenous immune globulin, or Rlio(D) immune diovascular disease and hemoglobin concentrations between
globulin preoperatively. Patients with thrombotic thrombo 6 and 10 g/dL (60 and 100 g/L). Transfusing asymptomatic
cytopenic purpura (TTP) should receive plasmapheresis. patients with known CAD to achieve a hemoglobin level of
Platelet transfusion is unlikely to be helpful in patients with
greater than 10 g/dL (100 g/L) is frequently done, although
TTP; however, other patients with persistent thrombocy the data supporting this practice remain controversial.
topenia are typically transfused with platelets just before sur Patients with sickle cell disease are at high risk for periopera
gery. Patients with qualitative platelet dysfunction (for exam tive sickle cell crisis, which can be reduced by reducing the
ple, from kidney or liver disease) can be given desmopressin percentage of sickle-cell variant erythrocytes in total blood
acetate 1 hour preoperatively for lower-risk procedures for volume. Consultation with an experienced hematologist is
bleeding. Patients undergoing higher-risk procedures should advisable to determine die extent of transfusion needed for
receive a platelet transfusion. Hematology consultation is these patients.

131
Perioperative Medicine

KEY POINTS to raise glucose levels. Given the complexities of preop


Patients on chronic warfarin undergoing low-risk erative medication adjustment, a patient's plasma glucose-
level should be measured the morning of surgery, and
procedures, including cataract surgery, dental pro
insulin or dextrose may be needed to keep the patient
cedures, and biopsy of nonmajor organs, can con
tinue warfarin, with a target INR of 1.3 to 1.5 euglycemic.
Oral agents are challenging to use perioperatively
perioperatively.
Patients on chronic warfarin undergoing intermedi because of risk of hypoglycemia given their long half-lives and
variable oral intake in the postoperative patient. In addition,
ate- or high-risk procedures should discontinue war
metformin confers a risk of lactic acidosis while thiazolidine
farin 4 to 5 days before surgery; patients with a low
diones promote fluid retention. Thus, many patients with dia
thrombotic risk typically do not require bridging
betes, even those who have previously been well controlled
anticoagulant therapy, whereas those with an inter with diet or oral medications, may need insulin during the
mediate or high thrombotic risk should receive
perioperative period. Intensive insulin therapy (HT) using
heparin bridging therapy perioperatively. continuous insulin infusions has been extensively investigated
Aspirin and/or clopidogrel should be continued in in the postoperative setting with mixed results. Current evi
patients undergoing low-risk surgery and in dence does not support IIT as part of routine postoperative
patients with recent cardiac stent placement or care. Approaches to glycemic management in hospitalized
myocardial infarction; in most other patients, patients are discussed in MKSAP 16 Endocrinology and
antiplatelet therapy can be safely discontinued 7 to Metabolism.
10 days preoperatively.
Thyroid Disease
Patients with thyroid disease who have not had recent thy
1 Perioperative Management of roid function testing and those in whom thyroid disease is
Endocrine Diseases suspected should have thyroid function tested preopera
Diabetes Mellitus tively. Hyperthyroidism can predispose patients to perioper
Patients with diabetes mellitus are at higher risk for peri ative complications, including atrial fibrillation, cardiomy
operative complications, although many of these are opathy, respiratory failure, and thrombocytopenia. These
related to end-organ damage from the diabetes rather risks are highest in patients undergoing thyroid surgery.
than hypo- or hyperglycemia. Patients with type 1 dia Patients with hyperthyroidism should be made euthyroid
betes mellitus will need to have some insulin continued at prior to surgery, typically with antithyroid drugs. In patients
all times to prevent the development of diabetic ketoaci whose surgery cannot be delayed, ^-blockers, iodine, and
dosis; this warning should be emphasized in preoperative corticosteroids can be added, although the risks of these
consultations and in communications with the surgeon medications should be balanced by the degree of thyroid
and anesthesiologist. dysfunction and nature of the surgery. In contrast, for
Hyperglycemia increases the risk of infectious and other patients with hypothyroidism, surgery should be delayed
postoperative complications. Hemoglobin A,c and kidney only for severe hypothyroidism; mild or subclinical hypothy
function should be measured preoperatively in patients with roidism does not significantly increase the risk of periopera
diabetes if these have not been measured recently. Patients tive complications.
found to have poorly controlled diabetes should usually have
elective surgeries delayed until their glycemic control is Adrenal Insufficiency
improved, ideally to a fasting plasma glucose level below 220 The incidence of acute adrenal insufficiency is relatively
mg/dL (12.21 mmol/L). low in the perioperative period, even in patients treated
Diabetes medications require adjustment on the with chronic corticosteroids. Patients using chronic low-
morning of surgery. Metformin is usually discontinued at dose corticosteroids (prednisone <10 mg/d), short-term
least the night before surgery; other oral hypoglycemic corticosteroids, or inhaled corticosteroids have a low risk
medications should be withheld on the morning of sur for adrenal crisis and do not need preoperative testing or
gery. In general, short-acting insulins should be withheld treatment. Patients who chronically use higher doses of
the morning of surgery. Long-acting insulins are usually corticosteroids (>10 mg/d prednisone for at least 3 weeks)
dose adjusted, although the exact amount depends on the or who have known adrenal insufficiency should take their
patient's baseline control and regimen. Typically one-half usual daily dose on the day of surgery. These patients
to two-thirds of the usual dose of long-acting insulin is should be well hydrated and can receive supplemental
given in the 12 hours before surgery, with a lesser reduc hydrocortisone on the day of surgery, with low doses (for
tion in patients who are poorly controlled at baseline or example, 25 mg) for minor surgery and higher doses (up
who are taking corticosteroids or other agents expected to 150 mg) for major surgery. These doses should be

132
Perioperative Medicine

tapered rapidly back to baseline over the first 2 postopera Perioperative Management of
tive days unless signs or symptoms of adrenal crisis develop.
CONT. Liver Disease
Adrenocorticotropic hormone stimulation testing should
Both acute hepatitis and chronic liver disease with cirrhosis
bc reserved for patients in whom primary adrenal insuffi
are associated with an elevated risk of perioperative complica
ciency is suspected or in whom empiric supplemental cor
ticosteroids could be harmful. tions and death. Surgery can worsen liver disease because of
hepatic ischemia and hepatic hypoxemia. The half-life of
KEY POINTS
many medications, including sedatives, opioids, and anes
Many patients with diabetes niellitus, even those who thetic agents, may be prolonged in patients with hepatic dys
have previously been well controlled with diet or oral ftinction, causing further hypoxia and hypotension. Poor liver
medications, need insulin during the perioperative function predisposes to hepatic failure, bleeding, poor wound
period. healing, and infections. The internist should be alert for signs
For patients with hypothyroidism, surgery should be and symptoms of hepatic dysftinction, especially in patients
who have risk factors for liver disease. Patients whose history
delayed only for severe hypothyroidism; mild or sub
clinical hypothyroidism does not significantly increase and physical examination suggest ongoing liver disease should
the risk of perioperative complications. receive preoperative liver chemistry tests.
Elective surgery should be deferred to evaluate newly
diagnosed liver disease and postponed in patients with acute
hepatitis from any cause. Patients with chronic liver disease
F] Perioperative Management of without evidence of cirrhosis appear to be at no excess peri
Kidney Disease operative risk. Patients with hemochromatosis should be care
Patients with chronic kidney disease (CKD) have a higher
fully evaluated for cardiomyopathy. In patients being treated
incidence of perioperative complications than those without, for hepatitis B, die danger of temporary cessation of anti
although many of these complications are attributable to car retroviral therapy causing a disease flare should be emphasized
diovascular disease. Preoperative evaluation of patients with to the surgeon.
CKD should include laboratory testing to assess the stage of In patients with chronic liver disease with evidence of cir
kidney disease. Patients with significant anemia may benefit rhosis, surgical risk is based on both the extent of liver disease
from preoperative exogenous erythropoietin if more than and the type of surgery. The Child-Turcotte-Pugh (CTP)
minimal blood loss is expected as these patients are unable to score has been used to assess risk: patients with CTP class A
physiologically respond to acute blood loss. ACE inhibitors liver disease have a 10% risk of death after general surgery;
and angiotensin receptor blockers, commonly used in patients those with class B disease have a 30% risk; and those with class
with CKD, can cause intraoperative hypotension and may be C disease have a 75% to 80% risk. The Model for End-stage
withheld on the day of surgery. Liver Disease (MELD) score, American Society of Anesthesia
No agents have been shown to be protective against (ASA) risk score, and patient age are all independent predic
intraoperative acute kidney injury. In general, patients with tors of mortality. These variables have been linked into a sin
CKD should be adequately hydrated perioperatively with
gle calculator that can be used to estimate postoperative mor
anesthetic and other agents selected specifically to avoid tality risk (www.niayoclinic.org/nield/mayomodel9.html).
hypotension. Intraoperative urine output is not a good pre Intraabdominal surgeries, especially hepatic surgery and
dictor of postoperative acute kidney injury, and fluid intake- gallbladder surgery, are high risk in patients with cirrhotic
should not be titrated to urine output. liver disease associated with portal hypertension. The combi
Postoperatively, patients with CKD are at risk for volume nation of coagulopathy and internal varices makes these sur
overload, electrolyte imbalance, and acute kidney injury. geries technically challenging. Estimating operative risk in bil
NSAIDs and other nephrotoxic agents should be avoided.
iary surgery is especially difficult as the jaundice in these
Care should be taken that all medications are adjusted for the patients is multifactorial, confounding die risk prediction
patient's glomerular filtration rate. Patients with uremia are at models. Coronary artery bypass grafting is also higher risk in
risk for platelet dysftinction and postoperative bleeding. In patients with cirrhotic liver disease. Herniorrhaphy, in con
patients on dialysis, the timing of dialysis perioperatively trast, is average risk.
should be coordinated between anesthesiologist, surgeon, Preoperative management of cirrhotic liver disease
and nephrologist. [_] involves optimizing liver function. Patients with coagulopa
KEY POINT thy are typically treated with oral vitamin K to reverse any
nutritional deficiencies. Vitamin K, however, does not typi
In patients with chronic kidney disease, care should
be taken that all medications are adjusted for the cally normalize these patients' coagulopathy, and they may
need fresh frozen plasma, cryoprecipitate, or activated factor
patient's glomerular filtration rate. Vila. Ascites predisposes to perioperative pulmonary

133
Perioperative Medicine

prt complications, poor wound healing of abdominal surgeries, Perioperative management of patients who use short-acting
and excess death. Ascites should primarily be managed with
dopamine agonists is challenging, as abrupt withdrawal of
diuretics, as large-volume paracentesis is effective only for a these agents can produce severe muscle rigidity that can inter
short period of time. Placement of a transjugular intrahep fere with ventilation and mimic neuroleptic malignant syn
atic portosystemic shunt (TIPS) may be indicated preoper drome. Patients with Parkinson disease are also at risk for aspi
atively in patients with refractory ascites. TIPS may also be ration secondary to hypersalivation and poor respiratory
appropriate for patients with known esophageal varices mechanics.
undergoing major surgery. Perioperative stroke impacts 0.3% to 0.8% of postsurgi
Perioperative management should focus on preventing cal patients and is further discussed in MKSAP 16 Neurology.
complications of chronic liver disease. Blood pressure and vol Risk factors for delirium in the postoperative setting are sim
ume status should be carefully monitored to minimize a renal ilar to those La medically hospitalized patients and include
insult that could precipitate a hepatorenal syndrome. pain, immobilization, anemia, hypoxemia, and sedative and
Sedatives and narcotics that are hcpatically metabolized opioid use. Early consultation by an internist or geriatrician
should be dosed carefully. Patients with ileus or other causes for patients at risk for postoperative delirium may be helpful.
of slowed intestinal transit should be observed closely for Delirium is further discussed in MKSAP 16 Pulmonary and
signs of hepatic encephalopathy. If encephalopathy develops, Critical Care Medicine.
the patient should receive lactulose orally or, if on NPO sta
tus (nothing by mouth), via retention enema. Patients with Bibliography
advanced liver disease are at risk for hypoglycemia postoper Interpretation of the Medical Literature
atively because of impaired gluconeogenesis. Guyatt G, Rennie D, Meade MO, Cook DJ, eds. Users' Guides to the
Medical Literature: A Manual for Evidence-Based Clinical Practice.
KEY POINT
2nd ed. New York, NY: McGraw-Hill; 2008.
Patients with examination findings suggesting ongo Ho PM, Peterson PN, Masoudi FA. Evaluating the evidence: is there a
ing liver disease should receive preoperative liver rigid hierarchy? Circulation. 2008;118(16):1675-1684. [PMID:
18852378]
chemistry tests; elective surgery should be deferred to I.in KW, Slawson DC. Identifying and using good practice guidelines.
evaluate newly diagnosed liver disease. Am Fam Physician. 2009;8()(f):67-70. [PMID: 19621847)
Sox HC, Greenfield S. Comparative effectiveness research: a report from
the Institute of Medicine. Ann Intern Med. 2009;151(3):203-205.
[PMID: 19567618]
ffl Perioperative Management of
Routine Care of the Healthy Patient
Neurologic Disease 2008 PHS Guideline Update Panel, Liasons, and Staff. Treating tobacco
Patients with neurologic diseases present specific challenges use and dependence: 2008 update U.S. Public Health Service Clinical
in the perioperative period. Patients with neuromuscular dis Practice Guideline executive summary. Respir Care.
2008;53(9):1217-1222. [PMID: 18807274]
eases, including amyotrophic lateral sclerosis, muscular dys
trophy, and myasthenia gravis, are at high risk for respiratory Berg AO, Baird MA, Botkin JR, et al. National Institutes of Health
State-of-the-Science Conference Statement: Family History and
failure. Preoperative pulmonary function testing is some
Improving Health. Ann Intern Med. 2009;151(12):872-877.
times done in these patients, although there are no specific [PMID: 19884615]
risk prediction models for respiratory failure. Close consul Fiorc MC, Baker TB. Clinical practice. Treating smokers in the health
care setting. N Engl J Med. 2011;365(13):1222-1231. [PMID:
tation among surgeon, anesthesiologist, internist, and neu
21991895]
rologist is essential in safely managing patients with neuro Heshka JT, Palleschi C, Howley H, Wilson B, Wells PS. A systematic
muscular diseases. review of perceived risks, psychological and behavioral impacts of
Patients with a seizure disorder should have their therapy genetic testing. Genet Med. 2008;10(1): 19-32. [PMID: 18197053]
optimized preoperatively, and anticonvulsant drug levels Kahn JA. HPV vaccination for the prevention of cervical intraepithelial
should be therapeutic. Patients should continue the anticon neoplasia. New Engl J Med. 2009;361(3):271-278. [PMID:
19605832]
vulsant drugs through the morning of surgery. The presence
Lambert LC, Fauci AS. Influenza vaccines for the future. New End J
of a seizure disorder will usually influence an anesthesiolo Med. 2010;363(21):2036-2044. [PMID: 21083388]
gist's choice of agents. Postoperatively, patients with a seizure Lin JS, O'Connor E, Whitlock EP, Bcil TL. Behavioral counseling to
disorder may require parenteral medication. promote physical activity and a healthful diet to prevent cardiovascu
lar disease in adults: a systematic review for the U.S. Preventive
Patients with multiple sclerosis may experience an acute Services Task Force. Ann Intern Med. 2010;153(ll):736-750.
exacerbation after anesthesia. Interferon is typically continued [PMID: 21135297]
for these patients perioperatively, while baclofen, which is only Long FT, Brandeau ML, Owens DK. The cost-effectiveness and popu
available orally, may need to be transitioned to a benzodi lation outcomes of expanded HIV screening and antiretroviral treat
ment in the United States. Ann Intern Med. 2010;153(12):778-789.
azepine to prevent postoperative withdrawal seizures. [PMID: 21173412]
Patients with Parkinson disease should continue Porter ME. What is value in health care? N Engl J Med.
antiparkinsonian agents through the morning of surgery. 2010;363(26):2477-2481. [PMID: 21142528]

134
Bibliography

Powers BJ, Olsen MK, Smith VA, Woolson RF, Boswoth HB, Oddonc Manual: sixth edition. Ann Intern Med. 2012;156(1 Pt 2):73-104.
EZ. Measuring blood pressure for decision making and quality [PMID: 22213573]
reporting: where and how many measures? Ann Intern Med. Palliative Care
2011;154(12):781-788. [PMID: 21690592]
Bakitas M, Lyons KD, Hegel MT, et al. Effects of a palliative care inter
Rollnick S, Buder CC, Kinnerslev P, Gregory J, Mash B. Motivational vention on clinical outcomes in patients with advanced cancer: the
interviewing. BMJ. 2010;340:cl900. [PMID: 204239571 Project ENABLE II randomized controlled trial. JAMA.
Saitz R, Alford DP, Bernstein J, Cheng DM, Samet J, Palfai T Screening 2009;302(7):741-749. [PMID: 19690306]
and brief intervention for unhealthy drug use in primary care settings:
Lorenz KA, Lynn J, Dy SM, et al. Evidence for improving palliative care
randomized clinical trials are needed. J Addict Med. 2010;4(3):123-
at the end of life: a systematic review. Ann Intern Med.
130. [PMID: 20936079]
2008;148(2):147-159. [PMID: 18195339]
Schousboe J, Kerlikowski K, Loh A, Cummings SR. Personalizing mam Morrison RS, Penrod JD, Cassel JB, et al. Cost savings associated with
mography by breast density and other risk factors for breast cancer: US hospital palliative care consultation programs. Arch Intern Med.
analvsis of health benefits and cost-effectiveness. Ann Intern Med.
2008;168(16):17S3-1790. [PMID: 18779466]
2011;155(l):10-20. [PMID: 21727289)
Revillc B, Axelrod D, Maury R. Palliative care for the cancer patient.
Smidi PC, Schmidt SM, Allensworth-Davics D, Saitz R. A single-ques
Prim Care. 2009;36(4):781-810. [PMID: 19913186]
tion screening test for drug use in primary care. Arch Intern Med.
2010;170(13):1155-1159. [PMID: 20625025] Swetz KM, Kamal AH. Palliative care. Ann Intern Med.
2012;156(3):ITC21. [PMID: 22312158]
Thompson SG, Ashton HA, Gao L, Scott RA; Multicentre Aneurysm
Screening Study Group. Screening men for abdominal aortic Temel JS, Greer JA, Muzikansky, et al. Early palliative care for patients
aneurysm: 10 year mortality and cost effectiveness results from the with metastatic non-small-cell lung cancer. N Engl J Med.
randomised Multicentre Aneurysm Screening Study. BMJ. 2010;363(8):733-742. [PMID: 20818875]
2009;338:b2307. [PMID: 19553269]
Common Symptoms
U.S. Preventive Services Task Force. Behavioral counseling to prevent
American Geriatrics Society Panel on the Pharmacologic Management of
sexually transmitted infections: U.S. Preventative Services Task Force Persistent Pain in Older persons. Pharmacologic management of per
recommendation statement. Ann Intern Med. 2008;149(7):491-496,
sistent pain in older persons. J Am Geriatr Soc. 2009;57(8):1331-
W95. [PMID: 18838729]
1346. [PMID: 19573219)
Zolotor AJ, Denham AC, Weil A. Intimate partner violence. Prim Care.
Anderson JL, Adams CD, Antman EM, et al; American College of
2009;36(l):167-179,x. [PMID: 19231608]
Cardiology; American Flcart Association Task Force on Practice
Patient Safety Guidelines (Writing Committee to Revise the 2002 Guidelines for the
Chassin MR, Loeb JM, Schmaltz SP, Wachter RM. Accountability meas Management of Patients With Unstable Angina/Non-ST-Elevation
Myocardial Infarction); American College of Emergency Physicians;
uresusing measurement to promote quality improvement. N Engl
J Med. 2010;363(7):683-688. (PMID: 20573915] Society for Cardiovascular Angiography and Interventions; Society of
Thoracic Surgeons; American Association of Cardiovascular and
Schiff GD, Bates DW. Can electronic clinical documentation help pre Pulmonary Rehabilitation; Society for Academic Emergency
vent diagnostic errors? N Engl J Med. 2010;362(12):1066-1069. Medicine. ACC/AHA 2007 guidelines for the management of
[PMID: 20335582] patients with unstable angina/non-ST-Elevation myocardial infarc
Swensen, SJ, Meyer GS, Nelson EC, et al. Cottage industry to postin- tion: a report of the American College of Cardiology/American
dustrial carethe revolution in health care delivery. N Engl J Med. Heart Association Task Force on Practice Guidelines (Writing
2010;362(5):el2. [PxMID: 20089956] Committee to Revise the 2002 Guidelines for the Management of
Patients With Unstable Angina/Non-ST-Elevation Myocardial
Professionalism and Ethics
Infarction) developed in collaboration with the American College of
Braddock CH 3rd, Edwards KA, Hasenberg NM, Laidley TL, Levinson Emergency Physicians, the Society for Cardiovascular Angiography
W. Informed decision making in outpatient practice: time to get back and Interventions, and the Society of Thoracic Surgeons endorsed by
to basics. JAMA. 1999;282(24):2313-2320. [PMID: 10612318] the American Association of Cardiovascular and Pulmonary
Castillo LS, Williams BA, Hooper SM, Sabatino CP, Weithorn LA, Rehabilitation and the Society for Academic Emergency Medicine. J
Sudore RL. Lost in translation: die unintended consequences of Am Coll Cardiol. 2007;50(7):el-el57. [PMID: 17692738]
advance directive law on clinical care. Ann Intern Med. Bittner ML, Marcus DA, Tenzer P, Romito K. Using Opioids in the
2011;154(2):121-128. [PMID: 21242368] Management of Chronic Pain Patients: Challenges and Future
DcsRoches CM, Rao SR, Fromson JA, et al. Physicians' perceptions, Options. American Academy of Family Physicians. 2010.
preparedness for reporting, and experiences related to impaired and Bohnert AS, Valenstein M, Bair MJ, et al. Association between opioid
incompetent colleagues. JAMA. 2010;304(2):187-193. [PMID: prescribing patterns and opioid overdose-related deaths. JAMA.
20628132] 2011;305(13):1315-1321.]PMID: 21467284]
Gallagher TH, Studdcrt D, Levinson W. Disclosing harmful medical Cooper PN, Westby M, Pitcher DW, Bullock I. Synopsis of the National
errors to patients. N Engl J Med. 2007;356(26):2713-2719. [PMID: Institute for Health and Clinical Excellence Guideline for manage
17596606] ment of transient loss of consciousness. Ann Intern Med.
Localio AR. Patient compensation without litigation: a promising devel 2011;155(8):543-549. [PMID: 21930835]
opment. Ann Intern Med. 2010;153(4):266-267. [PMID: Dunn KM, Saunders KW, Rutter CM, et al. Opioid prescriptions for
20713794] chronic pain and overdose: a cohort study. Ann Intern Med.
Owens DK, Qaseem A, Chou R, Shekelle P; Clinical Guidelines 2010;152(2):85-92. [PMID: 20083827]
Committee of the American College of Physicians. Fligh-value, cost- EcclestOQ C, Williams AC, Morley S. Psychological therapies for the
conscious health care: concepts for clinicians to evaluate the benefits,
management of chronic pain (excluding headache) in adults.
harms, and costs of medical interventions. Ann Intern Med. Cochrane Database Syst Rev. 2009;(2):CD007407. [PMID:
2011;154(3):174-180. [PMID: 21282697] 19370688]
Quill T, Arnold RM. Evaluating requests for hastened death #156. J Fife TD, Iverson DJ, Lempert T, et al; Quality Standards Subcommittee,
PalliatMed. 2008;11(8):1151-1152. [PMID: 18980457] American Academy of Neurology. Practice parameter: dierapies for
Snyder L; American College of Physicians Ethics, Professionalism, and benign paroxysmal positional vertigo (an evidence based review):
Human Rights Committee. American College of Physicians Ethics report of the Quality Standards Subcommittee of the American

135
Bibliography

Academy of Neurology. Neurology. 2008;70(22):2067-2074. systematic review of randomised controlled trials. Lancet.
[PMID: 18505980] 2010;376(9754):1751-1767. [PMID: 20970844]
Gilron I, Bailey JM, Tu D, Holden RR, Jackson AC, Houlden RL.
Gaujoux-Viala C, Dougados M, Gossec L. Efficacy and safety of steroid
Nortriptyline and gabapentin, alone and in combination for neuro injections for shoulder and elbow tendonitis: a meta-analysis of ran
pathic pain: a double-blind, randomised controlled crossover trial. domised controlled trials. Ann Rheum Dis. 2009;68(12):1S43-1849.
Lancet. 2009;374(9697):1252-1261. [PMID: 19796802] [PMID: 19054817]
Gonzales R, Bartlett JG, Besser RE, et al; American Academy of Family Jarvik JG, Comstock BA, Kliot M, et al. Surgery versus non-surgical
Physicians; American College of Physicians-American Society of
Internal Medicine; Centers for Disease Control; Infectious Diseases therapy for carpal tunnel syndrome: a randomised parallel-group trial.
Lancet. 2009;374(9695):1074-10S1. [PMID: 19782873]
Society of America. Principles of appropriate antibiotic use for treat
ment of uncomplicated acute bronchitis: background. Ann Intern Kuijper B, Tans JTJ, Beelen A, Nollet F, de Visser M. Cervical collar or
Mcd.2001;134(6):521-529. [PMID: 11255532] physiotherapy versus wait and see policy for recent onset cervical
Hess EP, Dipti A, Chandra S, et al. Diagnostic accuracy of the TIMI risk radiculopathy: randomised trial. BMJ. 2009;339:b3883. [PMID:
score in patients with chest pain the emergency department: a meta 19812130]
analysis. CMAJ. 2010;182(10):1039-1044. [PMID: 20530163] Nikolaidis I, Fouyas IP, Sandercock PA, Statham PF. Surgery for cervi
Flotson JR, Baloh RW. Acute vestibular syndrome. N Engl J Med. cal radiculopathy or myelopathy. Cochrane Database Svst Rev.
1998;339(10):680-685. [PMID: 9725927] 2010;(1):CD001466. |PMID: 20091520]
Irwin RS, Baumann MH, Bolser DC, et al; American College of Chest Ottenheijm RP, Jansen MJ, Staal JB, et al. Accuracy of diagnostic ultra
sound in patients with suspected subacromial disorders: a systematic
Physicians (ACCP). Diagnosis and management of cough executive
review and meta-analysis. Arch Phvs Med Rehabil.
summary: ACCP evidence-based clinical practice muidelines. Chest.
2006;129(1 suppl):lS-23S. [PMID: 16428686] 2010;91(10):1616-1625. [PMID: 20875523]
Moore RA, Straube S, Wiffen PJ, Derry S, McQuay HJ. Pregabalin for Seida JC, LeBlanc C, Schouten JR, et al. Systematic review: nonopera-
acute and chronic pain in adults. Cochrane Database Syst Rev. tivc and operative treatments for rotator cuff tears. Ann Intern Med.
2009;(3):CD007076. [PMID: 19588419] 2010;153(4):246-255. [PMID: 20621893]
Moya A, Sutton R, Ammirati F, et al; Task Force for the Diagnosis and Young C. In the clinic. Plantar fasciitis. Ann Intern Med. 2012;156( 1 Pt
Management of Syncope; European Society of Cardiology (ESC); 1):ITC1-1-ITC1-16. [PMID: 22213510]
European Heart Rhythm Association (EHRA); Heart Failure
Association (HFA); Heart Rhythm Society (HRS). Guidelines for die- Dyslipidemia
diagnosis and management of syncope (version 2009). Eur Heart J. Ginsberg HN, Elam MB, Lovato MC, et al; ACCORD Study Group.
2009;30(21 ):2631-2671. [PMID: 19713422] Effects of combination lipid therapy in type 2 diabetes mellitus. N
Nijroldcr I, van der Horst H, van der Windt D. Prognosis of fatigue: a Engl J Med. 2010;362(17):1563-1574. [PMID: 20228404]
systematic review. J Psvchosom Res. 2008;64(4):335-349. [PMID: Blaha MJ, Bansal S, Rouf R, Golden SH, Blumenthal RS, Defilippis AP.
18374732] A practical "ABCDE" approach to the metabolic syndrome. Mavo
Clin Proc. 2008;83(8):932-941. [PMID: 18674478]
Ouyang H, Quinn J. Diagnosis and evaluation of syncope in the emer
gency department. Emerg Med Clin North Am. 2010;28(3):471- Calderon RM, Cubeddu LX, Goldberg RB, Schiff ER. Statins in the
485. [PMID: 20709239] treatment of dyslipidemia in the presence of elevated liver amino
Parry SW, Tan MP. An approach to the evaluation and management of transferase levels: a therapeutic dilemma. Mavo Clin Proc.
syncope in adults. BMJ. 2010;340:c880. [PMID: 20172928] 2010;85(4):349-356. [PMID: 20360293]
Schroeder K, Fahey T Over-the-counter medications for acute cough in Egan A, Colman E. Weighing die benefits of high-dose simvastatin
children and adults in ambulatory settings. Cochrane Database Syst against the risk of myopathy. N EngkJ Med. 2011;365(4):285-287.
Rev. 2004;(4):CD001831. [PMID: 15495019] | PMID: 21675881]
Schutte-Rodin S, Broch L, Buysse D, Dorsey C, Sateia M. Clinical Hayward IU, Krumholz HM, Zulman DM, Timbie JW, Vijan S.
guideline for the evaluation and management of chronic insomnia in Optimizing statin treatment for primary prevention of coronary
adults. J Clin Sleep Med. 2008;4(5):487-504. [PMID: 18853708] artery disease. Ann Intern Med. 2010;152(2):69-77. [PMID:
Sloane PD, Coeytaux RR, Beck RS, Dallara J. Dizziness: state of the sci 20083825]
ence. Ann Intern Med. 2001;134(9 Pt 2):823-832. [PMID: Joy TR, Hegele RA. Narrative review: statin-related mvopathv. Ann
11346317] Intern Med. 2009;150( 12):858-868. [PMID: 19528564]
Swap CJ, Nagumey JT Value and limitations of chest pain history in the Kromhout D, Giltay EJ, Geleijnse JM; Alpha Omega Trial Group, ii-3
evaluation of patients with suspected acute coronary syndromes. fatty acids and cardiovascular events after myocardial infarction. N
JAMA. 2005;294(20):2623-2629. [PMID: 16304077] Engl J Med. 2010;363(21):2015-2026. [PMID: 20929341]
Wilson JF. In the clinic. Insomnia. Ann Intern Med. Mottillo S, Filion KB, Genest J, et al. The metabolic syndrome and car
2008;148(1):ITC13-1-ITC13-16. [PMID: 18166757] diovascular risk a systematic review and meta-analysis. J Am Coll
Musculoskeletal Pain Cardiol. 2010;56(14):1113-1132. [PMID: 20863953]
Chou R, Qaseem A, Snow V, et al; Clinical Efficacy Assessment Ray KK, Seshasai SRK, Erqou S, et al. Statins and all-cause mortality in
Subcommittee of the American College of Physicians; American high-risk primary prevention: a meta-analysis of 11 randomized con
College of Physicians; American Pain Society Low Back Pain trolled trials involving 65,229 participants. Arch Intern Med.
Guidelines Panel. Diagnosis and treatment of low back pain: a joint 2010;170(12):1024-1031. [PMID: 20585067]
clinical practice guideline from the American College of Physicians Taylor AJ, Villines TC, Stanek EJ, et al. Extended-release niacin or eze
and the American Pain Society. Ann Intern Med. 2007;147(7):478- timibe and carotid intima-media thickness. N Engl J Med.
491. [PMID: 17909209] 2()09;361(22):2113-2122. [PMID: 19915217]
Chou R, Shekelle P. Will this patient develop persistent disabling low
Tota-Maharaj R, Defilippis AP, Blumenthal RS, Blaha MJ. A practical
back pain? JAMA. 2010;303(13):1295-1302. [PMID: 20371789]
approach to the metabolic syndrome: review of current concepts and
Coombes BK, Bisset L, Vicenzino B. Efficacy and safety of corticosteroid management. Curr Opin Cardiol. 2010;25(5):502-512. [PMID:
injections and other injections for management of lendinopathy: a 20644468|

136
Bibliography

Chlcbowski RT, Andersonn GL, Gass M, et al; WHI Investigators.


Obesity
Adams TD, Gress RE, Smith SC, et al. Long-term mortality after gastric Estrogen plus progestin and breast cancer incidence and mortality in
postmenopausal women. JAMA. 2010;304(15):1684-1692. [PMID:
bypass surgery. N Engl J Med. 2007;357(8):753-761. [PMID: 20959578]
17715409]'
Harel Z. Dysmenorrhea in adolescents and young adults: from patho
Foster GD, Wyatt HR, Hill JO, et al. Weight and metabolic outcomes
after 2 years on a low-carbohydrate versus low-fat diet: a randomized physiology to pharmacological treatments and management strate
trial. Ann Intern Med. 2010;153(3):147-157. [PMID: 20679559] gies. Expert Opin on Pharmacother. 2008;9(15):2661-2672.
[PMID: 18803452]
Lang IA, Llewellyn DJ, Alexander K, Melzer D. Obesity, physical func Kingsberg S, Althof SE. Evaluation and treatment of female sexual dis
tion, and mortality in older adults. J Am Geriatr Soc. 2008;56:1474- orders. Int Urogvnecol J Pelvic Floor Dysftmct. 2009;(20)(suppl
1478. [PMID: 18662211]
1):S33-S43. [PMID: 19440781]
Leblanc ES, O'Connor E, Whitlock EP, Patnode CD, Kapka T. Lanston A. Emergency contraception: update and review. Semin Reprod
Effectiveness of primary care-relevant treatments for obesity in adults:
Med. 2010;28(2):95-102. [PMID: 20352558]
a systematic evidence review for the U.S. Preventive Services Task
Force. Ann Intern Med. 2011;155(7):434-477. [PMID: 21969342] Martin KA, Manson JE. Approach to the patient with menopausal symp
toms. J Clin Endocrinol Metab. 2008;93(12):4567-4575. [PMID:
Li Z, Maglione M, Tu W, et al. Meta-analysis: pharmacologic treatment
19056840]
of obesity. Ann Intern xVIed. 2005;142(7):532-546. [PMID:
15809465] Nappi RE, Martini E, Terreno E, et al. Management of hypoactive sex
ual desire disorder in women: current and emerging therapies. Int J
Maciejewski ML, Livingston EH, Smith VA, et al. Survival among high- Womens Health. 2010;2:167-175. [PMID: 21072309]
risk patients after bariatric surgery. JAMA. 2011 ;305(23):2419-2526.
[PMID: 21666276] Rossouw JE, Prentice RL, Manson JE, et al. Postmenopausal hormone
therapy and risk for cardiovascular disease by age and years since
Maggard MA, Shugarman LR, Suttorp M, et al. Meta-analysis: surgical
treatment of obesity. Ann Intern Med. 2005;142(7):547-559. menopause. JAMA. 2007;297( 13): 1465-1477. [PMID: 17405972]
[PMID: 15809466] Rungruang B, Kelley JL 3rd. Benign breast diseases: epidemiology, eval
uation, and management. Clin Obstet Gynecol. 2011;54(1):110-
Silk AW, McTitcuic KM. Reexamining the physical examination for obese 124. [PMID: 21278510]
patients. JAMA. 2011;305(2):193-194. [PMID: 21191004]
Santen RJ, Mansel R. Benign breast disorders. N Engl J Med.
Men's Health 2005;353(3):275-285. [PMID: 16034013]
Greco KA, McVary KT. The role of combination medical therapy in Vercellini P, Somigliana E, Vigano P, Abbiati A, Barbara G, Fcdele L.
benign prostatic hyperplasia. Int J Impot Res. 2008;20(suppl 3):S33- Chronic pelvic pain in women: etiology, pathogenesis, and diagnostic
S43. [PMID: 19002123] approach. Gynecol Endocrinol. io09;25(3):149-158. [PMID:
Gupta BP, Murad MFI, Clifton MM, Prokop L, Nehra A, Kopecky SL. 19347704]
The effect of lifestyle modification and cardiovascular risk factor Wilson, JF. In the clinic. Vaginitis and cervicitis. Ann Intern Med.
reduction on erectile dysfunction: a systematic review and meta-analy 2009;151(5):ITC3-l-ITC3-15;QuizITC3-16. [PMID: 19721016]
sis. Arch Intern Med. 2011;171(20):1797-1803. [PMID:
21911624] Eye Disorders
Qaseem A, Snow V, Denberg TD, et al; Clinical Efficacy Assessment Ahmed R, Foroozan R. Transient monocular visual loss. Neurol Clin.
Subcommittee of the American College of Physicians. Hormonal test 2010;28(3):619-629. [PMID: 20637992]
ing and pharmacologic treatment of erectile dysfunction: a clinical Arroyo JG. A 76-vear-old man with macular degeneration. JAMA.
practice guideline from the American College of Physicians. Ann 2(i()6;295(20):2394-2406. [PMID: 16720825]
Intern Med. 2009;151(9):639-649. [PMID: 19884625] Asbell PA, Dualan I, Mindel J, Brocks D, Ahmad M, Epstein S. Age-
Rosen RC, Cappelleri JC, Smith MD, et al. Development and evaluation related cataract. Lancet. 2005;365(9459):599-609. [PMID:
of an abridged, 5-item version of the International Index of Erectile 15708105]
Function (IIEF-5) as a diagnostic tool for erectile dysfunction. Int J Cronau H, Kankanala RR, Manger T. Diagnosis and management of red
Impot Res. 1999;ll(6):319-326. [PMID: 10637462] eye in primary care. Am Fam Physician. 2010;81(2):137-144.
Shah NR, Mikami DJ, Cook C, et al. A comparison of outcomes [PMID: 20082509]
between open and laparoscopic surgical repair of recurrent inguinal
Hollands H, Johnson D, Brox AC, Almeida D, Simel DL, Sharma S.
hernias. Surg Endosc. 2011;25(7):2330-2337. [PMID: 21298523] Acute-onset floaters and flashes: is this patient at risk for retinal
Touma NJ, Nickel JC. Prostatitis and chronic pelvic pain in men. Med detachment? JAMA. 2009;302(20):2243-2249. [PMID: 19934426]
Clin North Am. 201 l;95(l):75-86. [PMID: 21095412]
Magauran B. Conditions requiring emergency ophthalmologic consul
Traish AM, Miner MM, Morgentaler A, Zitzmann M. Testosterone defi tation. Emerg Med Clin North Am. 2008;26(l):233-238, viii.
ciency. Am J Med. 2011;124(7):578-587. [PMID: 21683825] [PMID: 18249265]
Tsertsvadze A, Fink HA, Yazdi F, et al. Oral phosphodiesterase-5 Weinreb RN, Khaw PT. Primary open-angle glaucoma. Lancet.
inhibitors and hormonal treatments for erectile dysftinction: a sys 2004;363(9422):1711-1720. [PMID: 15158634]
tematic review and meta-analysis. Ann Intern Med. 20()9;151(9):650-
661. [PMID: 19884626] Ear, Nose, Mouth, and Throat Disorders
WamplerSM, Llancs M. Common scrotal and testicular problems. Prim Calderon MA, Alves B, Jacobson M, Hurwitz B, Sheikh A, Durham S.
Care. 2010;37(3):613-626. [PMID: 20705202] Allergen injection immunotherapy for seasonal allergic rhinitis.
Cochrane Database Syst Rev. 2007;( 1 ):CD001936. [PMID:
Women's Health 17253469]
Amy JJ, Tripathi V. Contraception for women: an evidence based Chow AW, Benninger MS, Brook I, et al. IDSA clinical practice guide
overview. BMJ. 2009:339:b2895. [PMID: 19666684] line for acute bacterial rhinosinusitis in children and adults. Clin Infect
Berghella V, Buchanan E, Pereira L, Baxter JK. Preconception care. Dis. 2012;54(8):c72-cll2. [PMID: 22438350]
Obstet Gynecol Surv. 2010;65(2):119-131. [PMID: 20100361] Coco A, Vernacchio L, Horst M, Anderson A. Management of acute oti
Casablanca Y. Management of dysfunctional uterine bleeding. Obstet tis media after publication of the 2004 AAP and AAFP clinical prac
Gynecol Clin North Am. 2008;35(2):219-234. [PMID: 184868381 tice guideline. Pediatrics. 2010;125(2):214-220. [PMID: 20100746]

137
Bibliography

Douglass AB, Gonsalves W, Maier R, et al. Smiles for Life: A National Gartlehner G, Flansen RA, Morgan LC, et al. Second-Generation
Oral Health Curriculum for Family Medicine. A model for curriculum Antidepressants in the Pharmacologic Treatment of Adult Depression:
development bv STFM groups. Earn Med. 2007;39(2):88-90 An Update of the 2007 Comparative Effectiveness Review [Internet].
] PMID: 17273948] Rockville (MD): Agency for Healthcare Research and Quality (US);
Friedewald VE, Kornman KS, Beck JD, et al; American Journal of 2011 Dec. (Comparative Effectiveness Reviews, No. 46.) Peer
Cardiology; Journal of Pcriodontology. The American Journal of Reviewers. Available at www.effectivchealthcare.ahrq.gov/index
Cardiology and Journal of Pcriodontology Editors' Consensus: peri .cfm/search-for-guides-reviews-and-reports/?pageaction
odontitis and atherosclerotic cardiovascular disease. Am J Cardiol. =displayproduct&productid=862. Accessed July 9, 2012.
2009;104(l):59-68. [PMID: 19576322] Jones RM, Arlidge J, Gillham R, Reagu S, van den Bree M, Taylor PJ.
Hobson J, Chisholm E, El Refaie A. Sound therapy (masking) in die Efficacy of mood stabilisers in the treatment of impulsive or repetitive
management of tinnitus in adults. Cochrane Database Syst Rev. aggression: systematic review and meta-analysis. Br J Psychiatry.
2010;(12):CD006371. [PMID: 21154366] 2011;198(2):93-98. [PMID: 21282779]
Osguthorpe DJ, Nielsen DR. Otitis externa: Review and clinical update. Kroenke K, Spitzer RL, Williams JB, Monahan PO, Lowe B. Anxiety dis
Am Fam Physician. 2006;74(9):1510-1516. [PMID: 17111889] orders in primary care: prevalence, impairment, comorbidity, and
Powers JH. Diagnosis and treatment of acute otitis media: evaluating the detection. Ann Intern Med. 2007;146(5):317-325. [PMID:
evidence. Infect Dis Clin North Am. 2007;21(2):409-426, vi. 17339617]
[PMID: 17561076] Kroenke K. Unburdening die difficult clinical encounter. Arch Intern
Roland PS, Smith TL, Schwartz SR, et al. Clinical practice guideline: Med. 2009;169(4):333-334. [PMTD: 19237715]
Cerumen impaction. Otolavngol Head Neck Sur. Schultz SH, North SW, Shields CG. Schizophrenia: a review. Am Fam
2008;139(3)(suppl 2):S1-S21. [PMID: 18707628] Physician. 2007;75(12):1S21-1829. [PMID: 17619525]
Schlosser RJ. Clinical practice. Epistaxis. N Engl J Med. Sim LA, McAlpine DE, Grothe KB, Himes SM, Cockerill RG, Clark
2009;360(8):784-789. [PMID: 19228621] MM. Identification and treatment of eating disorders in the primary
Schreiber BE, Agrup C, Haskard DO, Luxon LM. Sudden sensorineural care setting. Mavo Clin Proc. 2010;85(8):746-751. [PMID-
hearing loss. Lancet. 2010;375(9721):1203-1211. [PMID: 20605951]
20362815] Stein MB, Coin MK, Pollack MH, et al. Practice guideline for the treat
Scrivani SJ, Keith DA, Kaban LB. Temporomandibular disorders. N ment of patients with panic disorder. Arlington, VA: American
Engl J Med. 2008;359(25):2693-2705. [PMID: 19092154] Psychiatric Association, 2009.
Stachler RJ, Chandrasekhar SS, Archer SM, et al; American Academy of Wilens TE, Faraone SV, Biederman J. Attention-dcficit/hvpcractivity
disorder in adults. JAMA. 2004;292(5):619-623! [PMID:
Otolaryngology-Hcad and Neck Surgery. Clinical practice guideline:
sudden hearing loss. Otolaryngol Head Neck Surg. 2012;146(3 1529208S]
suppl):Sl-35. |PMID: 22383545] Geriatric Medicine
Young J, De Sutter A, Merenstein D, et al. Antibiotics for adults with Budnitz DS, Lovcgrove MC, Shehab N, Richards CL. Emergency hos
clinically diagnosed acute rhinosinusitis: a meta-analysis of individual pitalizations for adverse drug events in older Americans. N Enid J
patient data. Lancet. 2008;371(9616):908-914. [PMID: 18342685] Med. 2011;365:2002-2012. [PMID: 22111719]
Anorectal Disorders Burgio KL, Kraus SR, Menefee S, et al; Urinary Incontinence Treatment
Schubert MC, Sridhar S, Schade RR, Wcxner SD. What every gastroen- Network. Behavioral therapy to enable women with urge inconti
nence to discontinue drug treatment: a randomized trial. Ann Intern
terologist needs to know about common anorectal disorders. World
J Gastroenterol. 2009;15(26):3201-3209. [PMID: 19598294] Med. 2008;149(3):161-169. [PMID: 18678843]
Carr DB, Ott BR. The older adult driver with cognitive impairment:
Mental and Behavioral Health "It's a very frustrating life". JAMA. 2010;303(16):1632-1641.
American Psychiatric Association Work Group on Eating Disorders. [Erramm in: JAMA. 2010;303(23):2357]. [PMID: 20424254]
Practice guideline for the treatment of patients with eating disorders
Pong TG, Jones RN, Rudolph JL, et al. Development and validation of
(revision). Am J Psychiatry. 2000;157(1 suppl):l-39. [PMID: a brief cognitive assessment tool: the sweet 16. Arch Intern Med.
10642782] 2011;171(5):432-427. [PMID: 21059967]
Belmaker RH. Treatment of bipolar depression. N Engl J Med. Garfinkel D, Mangin D. Feasibility study of a systematic approach for
2007;356( 17): 1771-1773. [PMTD: 17392296] discontinuation of multiple medications in older adults: addressing
Brugha TS, McManus S, Bankart J, et al. Epidemiology of autism spec polypharmacy. Arch Intern Med. 2010;170(18):1648-1654. [PMID:
trum disorders in adults in the community in England. Arch Gen 20937924]
Psychiatry. 201 l;6S(5):459-465. [PMID: 21536975] Gillespie LD, Robertson MC, Gillespie WJ, et al. Interventions for pre
Bulik CM, Berkman ND, Brownley KA, Sedway JA, Lohr KN. Anorexia venting falls in older people living in the community. Cochrane
nervosa treatment: a systematic review of randomized controlled tri Database Syst Rev. 2009;(2):CD007146. [PMID: 19370674]
als. Int J Eat Disord. 2007;40(4):310-320. [PMID: 17370290]
Hayes BD, Klein-Schwartz W, Barrueto F. Polypharmacy and the geri
Fancher TL, Kravitz RL. In the clinic. Depression. Ann Intern Med. atric patient. Clin Geriatr Med. 2007;23:371-390. [PMID:
2010;152(9):ITC51-15;quiz ITC5-16. [PMID: 20439571 ] 17462523]
Franklin ME, Foa EB. Treatment of obsessive compulsive disorder. Annu Kalyani RR, Stein B, Valiyil R, Manno R, Maynard JW, Crews DC.
Rev Clin Psychol. 2011;7:229-243. [PMID: 21443448] Vitamin D treatment for the prevention of falls in older adults: sys
Gammicchia C, Johnson C. Autism Information for Paramedics and tematic review and meta-analysis. J Am Geriatr Soc.
Emergency Room Staff. Autism Society. Available at www.autism- 2010;58(7):1299-1310. [PMID: 20579169]
society.org/about-us/publications/resource-materials.html. Kane RL. Finding the right level of posthospital care: "We didn't realize-
Accessed July 9, 2012. there was anv other option for him". JAMA. 2011;305(3):284-293.
Gartlehner G, Hansen RA, Morgan LC, et al. Comparative benefits and | PMID: 21245184]
harms of second-generation antidepressants for treating major depres Kinsella GJ, Mullaly E, Rand E, et al. Early intervention for mild cogni
sive disorder: an updated meta-analysis. Ann Intern Med. tive impairment: a randomised controlled trial. J Neurol Neurosuru;
2011;155(ll):772-785. [PMID: 22147715) Psychiatry. 2009;80(7):730-736. [ PMID: 19332424]

138
Bibliography

Li C, Friedman B, Conwell Y, Fiscella K. Validity of the Patient Health Douketis JD, Spyropoulos AC, Spencer FA, et al; American College of
Chest Physicians. Perioperative management of antithrombotic ther
Questionnaire 2 (PHQ-2) in identifying major depression in older
people. J Am Geriatr Soc. 2007;55(4):596-602. [PMID: 17397440] apy: Antithrombotic Therapy and Prevention of Thrombosis, 9th ed:
American College of Chest Physicians Evidence-Based Clinical
Mclnnes E, Dumville JC, Jammali-Blasi A, Bell-Syer SE. Support sur Practice Guidelines. Chest. 2012;141(2 suppl):c326S-e350S.
faces for treating pressure ulcers. Cochrane Database Syst Rev. [PMID: 22315266]
2011;(12):CD009490. [PMID: 22161450] Eilers H, Liu KD, Gruber A, Niemann CU. Chronic kidney disease:
Mclnnes E, Jammali-Blasi A, Bell-Syer SE, Dumville JC, Cullum N. implications for the perioperative period. Minerva Anestesiol.
Support surfaces for pressure ulcer prevention. Cochrane Database 2010;76(9):725-736. [PMID: 20820151]
Syst Rev. 2011;(4):CD001735. [PMID: 21491384] Ford MK, Beattie WS, Wijeysundera DN. Systematic review: prediction
Panel on Prevention of Falls in Older Persons; American Geriatrics of perioperative cardiac complications and mortality by die revised
Society and British Geriatrics Society. Summary of the Updated cardiac risk index. Ann Intern Med. 2010;152(l):26-35. [PMID:
American Geriatrics Society/British Geriatrics Society clinical practice 20048269]
guideline for prevention of falls in older persons. J Am Geriatr Soc. Hepner DL. The role of testing in the preoperative evaluation. Clevc
2011;59(1):148-157. [PMID: 21226685] Clin J Med. 2009;76(suppl 4):S22-S27. [PMID: 19880831 ]
Plassman BL, Langa KM, Fisher GG, et al. Prevalence of cognitive Lieb K, Selim M. Preoperative evaluation of patients with neurological
impairment without dementia in the United States. Ann Intern Med. disease. Semin Neurol. 2008;28(5):603-610. [PMID: 19115168]
20()8;14S(6):427-434. [PMID: 18347351]
Lipshutz AK, Cropper MA. Perioperative glycemic control: an evidence-
Rcddy M, Gill SS, Kalkar SR, Wu W, Anderson PJ, Rochon PA. based review. Anesthesiology. 2009;110(2):408-421. [PMID:
Treatment of pressure ulcers: a systematic review. JAMA. 19194167]
2008;300(22):2647-2662. [PMID: 19066385] Marik PE, Varon J. Requirement of perioperative stress doses of corti
Shamliyan T, Wyman JF, Ramakrishnan R, Sainfort F, Kane RL. costeroids: a systematic review of the literature. Arch Surg.
Systematic Review: Benefits and Harms of Pharmacologic Treatment 2008;143(12):1222-1226. [PMID: 19075176]
for Urinary Incontinence in Women. Ann Intern Med. 2012 Apr 9 O'Learv JG, Yachimski PS, Friedman LS. Surgery in the patient with
[Epub ahead of print] [PMID: 22492633] liver' disease. Clin Liver Dis. 20()9;13(2):211-231. [PMID:
Studenski S, Pcrera S, Patel K, et al. Gait speed and survival in older 19442915]
adults. JAMA. 2011:305(l):50-58. [PMID: 21205966] Patel MS, Carson JL. Anemia in the preoperative patient. Med Clin
North Am. 2009;93(5):1095-1104. [PMID: 19665622]
Wyman JF, Croghan CF, Nachreiner NM, et al. Effectiveness of educa
tion and individualized counseling in reducing environmental hazards Qaseem A, Humphrey LL, Chou R, Snow V, Shekelle P; Clinical
in the homes of community-dwelling older women. J Am Geriatr Soc. Guidelines Committee of the American College of Physicians. Use of
2007;55( 10): 1548-1556. [PMID: 17908058] intensive insulin therapy for the management of glycemic control in
hospitalized patients: a clinical practice guideline from the American
Perioperative Medicine College of Physicians. Ann Intern Med. 2011;154(4):260-267.
Carson JL, Grossman BJ, Kleinman S, et al; for the Clinical Transfusion [PMID: 21320941]
Medicine Committee of the AABB. Red Blood Cell Transtiision: A Smetana GW, Lawrence VA, Cornell JE; American College of Physicians.
Clinical Practice Guideline From the AABB*. Ann Intern Med. Preoperative pulmonary risk stratification for noncardiothoracic sur
2012;157(l):49-58. [PMID: 22751760] gery: systematic review for the American College of Physicians. Ann
Douketis JD, Berger PB, Dunn AS, et al; American College of Chest IntemMed. 2006;144(8):581-595. [PMID: 16618956]
Physicians. The perioperative management of antithrombotic therapy: Vasu TS, Doghramji K, Cavallazzi R, et al. Obstructive sleep apnea syn
American College of Chest Physicians Evidence-Based Clinical drome and postoperative complications: clinical use of the STOP-
Practice Guidelines (8th Edition).' Chest. 2008;133(6 suppl):299S- BANG questionnaire. Arch Otolaryngol Head Neck Surg.
339S. [PMID: 18574269] 2010;136(10):1020-1024. [PMID: 20956>51]

139
General Internal Medicine
Self-Assessment Test
This self-assessment test contains one-best-answer multiple-choice questions. Please read these directions carefully
before answering die questions. Answers, critiques, and bibliographies immediately follow these multiple-choice
questions. The American College of Physicians is accredited by the Accreditation Council for Continuing Medical
Education (ACCME) to provide continuing medical education for physicians.

The American College of Physicians designates MKSAP 16 General Internal Medicine for a maximum of 24 AMA
PRA Category 1 Credits. Physicians should claim only die credit commensurate with the extent of their partic
ipation in the activity.

Earn "Same-Day" CME Credits Online


For the first time, print subscribers can enter their answers online to earn CME credits in 24 hours or less. You can
submit your answers using online answer sheets that are provided at mksap.acponlinc.org, where a record of your
MKSAP 16 credits will be available. To earn CME credits, you need to answer all of the questions in a test and
earn a score of at least 50% correct (number of correct answers divided by the total number of questions). Take
any of the following approaches:
> Use the printed answer sheet at the back of this book to record your answers. Go to mksap.acponline.org,
access the appropriate online answer sheet, transcribe your answers, and submit your test for same-day
CME credits. There is no additional fee for this service.
> Go to mksap.acponline.org, access the appropriate online answer sheet, directly enter your answers, and
submit your test for same-day CME credits. There is no additional fee for this service.
>* Pay a $10 processing fee per answer sheet and submit the printed answer sheet at the back of this book by
mail or fax, as instructed on the answer sheet. Make sure you calculate your score and fax the answer sheet
to 215-351-2799 or mail the answer sheet to Member and Customer Service, American College of Physi
cians, 190 N. Independence Mall West, Philadelphia, PA 19106-1572, using the courtesy envelope pro
vided in your MKSAP 16 slipcase. You will need your 10-digit order number and 8-digit ACP ID number,
which are printed on your packing slip. Please allow 4 to 6 weeks for your score report to be emailed back
to you. Be sure to include your email address for a response.

If you do not have a 10-digit order number and 8-digit ACP ID number or if you need help creating a username and
password to access the MKSAP 16 online answer sheets, go to mksap.acponline.org or email custserv@acponline.org.

CME credit is available from the publication date of December 31,2012, until December 31,2015. You may sub
mit your answer sheets at any time during this period.

141
Directions
Each of the numbered items is followed by lettered answers. Select the ONE lettered answer that is BEST in each case.

Item 1 Item 3
A 32-year-old man is evaluated during a routine examina A 40-year-old woman is evaluated for having difficulty at
tion. He is in good healdi, has no concerning symptoms, work. She is a nurse, and over the past 6 months she has
and takes no medications. He does not smoke, seldom become very zealous about avoiding infection. She washes
drinks alcohol, and exercises 30 minutes daily 5 days per her hands six or seven times before entering patients' rooms
week. He ingests a heart-healthy diet. All of his immuniza and then again afterwards. She is having difficult}' complet
tions are up to date. The patient has no symptoms or health ing tasks on time. She showers multiple times daily and has
problems. scrubbed her skin raw in several areas. She recognizes that
diese actions are unreasonable but she no control over
Which of the following is the most reasonable next
them. She has no history of psychiatric disease, bipolar dis
screening step for genetic disease in this patient?
order, or schizophrenia.
(A) Obtain a family history of disease On physical examination, vital signs are normal. Both
(B) Obtain a three-generation pedigree hands are raw and there are several areas of denuded skin on
her arms and legs.
(C) Refer for genetic counseling
(D) Screen for common genetic mutations In addition to cognitive-behavioral therapy, which of
the following is the most appropriate pharmacologic
treatment?
Item 2
A 58-vear-old woman is evaluated for a 7-week history of (A) Fluoxetine
tingling pain involving die first, second, and third digits of (B) Haloperidol
the left hand. The pain is worse at night and radiates into (C) Lorazepam
the thenar eminence. The pain does not radiate into the (D) Quetiapine
proximal forearm. She has hypothyroidism and her only
current medication is levothyroxine.
On physical examination, die patient reports pain with Item 4
plantar flexion at the wrist widi the elbow extended. She A 24-year-old man is evaluated for a 2-hour history of epis
also reports pain with percussion over the median nerve at
taxis, which began after blowing his nose. The bleeding is
the level of the wrist. There is no thenar or hypothenar emi controlled by placing pressure on the anterior portion of the
nence atrophy. Strength is 5/5 with thumb opposition. A nose for 3 minutes but then recurs. The bleeding is from the
hand diagram is completed (shown) demonstrating the left nostril only. He has severe seasonal rhinitis diat has been
location of the patient's paresthesia. active recently. He has no history of bleeding, bruising, or
clotting, and there is no family history of bleeding disorders.
Current medications are loratadine and an intranasal
corticosteroid.
On physical examination, he is afebrile, blood pressure
is 138/88 mm Hg, and pulse rate is 82/min. Blood pres
sure and pulse are without significant change from supine
to standing positions. He is holding a tissue against his nose.
Examination of the left naris with a nasal speculum after the
removal of dried blood reveals a small oozing vessel in the
septum in die Kiesselbach area. The right naris is clear of
blood, and a skin examination demonstrates no petechiae or
bruises.
Which of die following is the most appropriate man
agement of this patient?
(A) Arterial embolization
(B) Cauterization and nasal packing
(C) Complete blood count and coagulation studies
In addition to avoidance of repetitive wrist motions,
which of the following is the most appropriate initial (D) Uninterrupted nasal pressure for 15 to 30 minutes
treatment? (E) Urgent otorhinolaryngology evaluation

(A) Local corticosteroid injection


(B) Oral ibuprofen Item 5
(C) Surgical intervention A 54-year-old woman is evaluated during a routine exam
(D) Wrist splinting ination. She is very concerned by her lack of interest in

143
Self-Assessment Test

sexual intercourse. The patient feels like she "just doesn't began after that. She has undergone several comprehensive
want to be touched." She used to enjoy intercourse and medical evaluations with no explanation for her symptoms.
does not know why she feels this way now, but she Recent records from previous physicians reveal normal
acknowledges that it is causing tremendous stress in her complete blood count, comprehensive metabolic panel, and
marriage. She has been menopausal for the past 2 years. thyroid function studies.
She uses lubrication for intercourse, which is successful in On physical examination, vital signs are normal. BMI
reducing discomfort. She has no previous history of men is 24. There is no lymphadenopathy, but the patient notes
strual irregularities, pelvic surgeries, sexual trauma, or sex tenderness when lymph node areas are palpated. There is
ually transmitted infections. She currently takes calcium tenderness to joint movement but no evidence of synovitis
and vitamin D supplements daily. or restricted movement. There is generalized tenderness to
On physical examination, external genitalia are nor muscle palpation; strength is normal. The remainder of the
mal, with no pain with vulvar palpation or with speculum physical examination is unremarkable. Mini-Mental State
insertion. The vaginal walls are pale with decreased rugae Examination score is 29/30; a two-question depression
and petechial hemorrhages. Decreased vaginal lubrication screen is negative.
is noted. The remainder of the physical examination is
normal. Which of die following should be done next to help
diagnose diis patient's symptoms?
Which of die following is die most likely diagnosis?
(A) Epstein-Barr virus titer
(A) Dyspareunia
(B) Erythrocyte sedimentation rate
(B) Hypoactive sexual desire disorder
(C) Parvovirus B19 titer
(C) Sexual aversion disorder
(D) No additional testing
(D) Vaginismus

Item 8
Item 6
A 29-year-old woman is evaluated preoperatively before
An 88-year-old man in hospice care is evaluated for dyspnea. elective breast reduction and liposuction. She feels well and
He has advanced dementia, severe COPD, and coronary has no symptoms or pertinent medical history. She exercises
artery disease. Based on prior discussions with his family regularly. Her last menstrual period was 3 weeks ago. She
regarding the goals of care, it was decided that his treatment drinks alcohol socially and does not smoke cigarettes. She
should consist of comfort care measures only. All of his has no family history of premature heart disease or abnor
medications except as-needed albuterol and iprau-opium mal bleeding. The procedure will be done under general
have been discontinued.
anesthesia. She takes no medications or supplements.
On physical examination, he is afebrile, blood pressure Results of the physical examination are normal.
is 108/76 mm Hg, pulse rate is 110/min, and respiration
rate is 26/min. Oxygen saturation is satisfactory. He is Which of the following is the most appropriate preop
cachectic and tachypneic and is disoriented and in moder erative test to perform next?
ate respiratory distress. Fleart sounds are distant and tachy
cardic but an S3 is not present. Chest examination reveals (A) Chest radiography
decreased breath sounds as well as diffuse, fine inspiratorv (B) Coagulation studies
crackles consistent with prior examinations. Extremities are (C) Electrocardiography
warm and dry. (D) Pregnancy testing
In addition to continuing his bronchodilator therapy, (E) No further testing
which of die following is the most appropriate next
step in die treatment of this patient?
Item 9
(A) Ceftriaxone and azithromycin A 66-year-old woman is evaluated for several months of a
(B) Lorazepam "whistling" or "swishing" sound in her right ear. She notes
(C) Methylprednisolone that it gets faster and louder when she exercises and thinks
(D) Morphine it is timed to her heartbeat. She does not notice any hear
ing loss, dizziness, or vertigo.
On physical examination, temperature is 37.4 C
Item 7 (99.3 F), blood pressure is 138/84 mm Hg, and pulse rate
A 55-year-old woman is evaluated for fatigue for the past 9 is 84/min. Auditory acuity to normal conversation appears
months. She used to be an avid runner, but now can only normal, and otoscopic examination is unremarkable bilat
walk 1 mile before experiencing severe muscle aches, joint erally. Neurologic examination is normal.
pain and fatigue for the next several days. She reports no Which of die following is the most appropriate next
insomnia but describes her 8 hours of nightly sleep as unre-
step in the management of diis patient?
freshing. She does not smoke, drink, or use illicit drugs. She
describes tender lymph nodes in her neck and axillae. She (A) Audiometry
reports having "the flu" last winter and believes her fatigue (B) Auscultation over the right ear, eye, and neck

144
Self-Assessment Test

(C) Trial of a sound-masking device On physical examination, he is afebrile. Blood pres


sure is 100/62 mm Hg and pulse rate is 104/niin. Res
(D) Trial of a nasal corticosteroid spray
piration rate is 16/min. The abdomen is slightly dis
tended but nontender. There is no focal tenderness over
Item 10 the hip or pelvis. Active and passive range of motion test
A 46-year-old woman is evaluated during a routine exami ing of his right hip does not exacerbate his groin pain.
The patient is reassured that a groin muscle strain is the
nation. Her 72-year-old mother was just diagnosed with
likely cause of his discomfort. He is advised to take more
lung cancer, so the patient asks you for help with quitting fluids and rest.
smoking. She has a 27-pack-year smoking history. She made 12 hours later he is brought to the emergency depart
one previous quit attempt several years ago using over-the- ment because of diffuse severe abdominal pain. A CT scan
counter nicotine gum, but she was unable to quit for more of die abdomen shows a rupturing 8.2-cm abdominal aor
than a few days. Medical history is significant for seizure dis tic aneurysm.
order. Review of systems discloses mild shortness of breath
with exertion and occasional wheezing. Medications are a Which of die following categories of diagnostic error
multivitamin and phenytoin. is responsible for missing the correct diagnosis at the
On physical examination, vital signs are normal. Lung follow-up examination?
examination reveals occasional wheezing and a prolonged (A) Anchoring heuristic
expiratory phase. The rest of the examination is normal. (B) Availability heuristic
In addition to counseling regarding tobacco use, (C) No-fault error
which of the following is an appropriate adjunct to (D) Representativeness heuristic
increase her likelihood of successful smoking cessation?

(A) A benzodiazepine Item 13


(B) Bupropion A 32-year-old man is evaluated for a 3-day history of pro
(C) Electronic smokeless cigarette use ductive cough, sore throat, coryza, rhinorrhea, nasal con
(D) Nicotine replacement therapy gestion, generalized myalgia, and fatigue. His sputum is
slightly yellow. His two children (ages 3 years and 1 year)
had similar symptoms 1 week ago. He is a nonsmoker and
Item 11 has no history of asthma.
A 23-year-old man is evaluated for a 3-day history of red On physical examination, temperature is 37.5 C
ness and itchiness of the right eye. He had an upper respi (99.4 F), blood pressure is 128/76 mm Hg, pulse rate is
ratory tract infection 3 days before the eye symptoms 92/min, and respiration rate is 14/min. There is bilateral
began. Each morning he has awoken with crusting over the conjunctival injection. The nasal mucosa is boggy, with clear
lids. He is otherwise healthy, with no ocular trauma or drainage. The oropharynx is erythematous without tonsil
lar enlargement or exudates. The tympanic membranes and
recent medical problems.
external auditory canals are normal. Lungs are clear to aus
On physical examination, he is afebrile, blood pressure
cultation. There is no rash or lymphadenopathy.
is 122/72 mm Hg, pulse rate is 66/min, and respiration
rate is 16/min. Right eye conjunctival injection is present, Which of die following is the most appropriate treat
with some crusting at the lids. Bilateral vision is 20/20. ment?
Pupils are equally round and reactive to light.
(A) Albuterol
Which of the following is die most appropriate man (B) Amoxicillin
agement of diis patient? (C) Chlorpheniramine
(A) Cool compresses to the affected eye (D) Codeine
(B) Oral antihistamine
(C) Topical antibiotics Item 14
(D) Topical corticosteroids A 48-year-old man is evaluated for a 2-day history of
episodic dizziness with nausea. He noted the onset abruptly
and compares the feeling to "being on a roller coaster." His
Item 12 most severe episodes occurred while arising from bed and
A 67-year-old man is evaluated for right groin pain. The when parallel parking his car. The symptoms lasted 30 to 40
pain began spontaneously 2 days ago. Yesterday, he was seconds and were followed by two episodes of emesis. He
evaluated in the emergency department and was diagnosed has no recent fever, headache, tinnitus, hearing loss, double
with groin strain and discharged home. He is reevaluated vision, dysarthria, weakness, or difficulty walking. He had a
today because the groin pain has continued without similar episode 5 years ago. Medical history is significant for
improvement. He has well-controlled hypertension and a depression. His only medication is citalopram.
30-pack-year smoking history. Medications are hydro On physical examination, vital signs are normal. Results
chlorothiazide and aspirin. of cardiac and neurologic examinations are normal. The

145
Self-Assessment Test

Dix-Hallpike maneuver precipitates severe horizontal nys Item 17


tagmus after about 20 seconds. With repeated maneuvers, An 85-year-old man is admitted to a nursing home. He has
the nystagmus is less severe. diabetes mellitus, coronary artery disease, chronic heart fail
Which of die following is die most likely diagnosis? ure, and dementia. On physical examination, vital signs are
normal. He has a full thickness 5x8 cm pressure ulcer on
(A) Benign paroxysmal positional vertigo his left buttock covered with a thick eschar. There is visible
(B) Cerebellar infarction subcutaneous fat beneath the eschar; no bone or tendon is
(C) Meniere disease exposed. His skin is dry and diere is evidence of mild dehy
dration and malnutrition. He has urinary but not fecal
(D) Vestibular neuronitis incontinence. His current medications are lisinopril, met
formin, hydrochlorodiiazide, glipizide, and carvedilol.
Item 15 Which of the following is the most appropriate man
A 47-year-old woman is evaluated for an abnormal com agement of diis patient's ulcer?
plete blood count obtained as part of a life insurance phys (A) Debridement
ical examination. She has no active medical problems. She
underwent Roux-en-Y gasttic bypass surgery 10 years ago (B) Hyperbaric oxygen therapy
and has successfully kept off the weight she lost after die (C) Negative-pressure wound vacuum therapy
surgery. She has not seen a physician for more dian 7 years. (D) Oral vitamin C and zinc supplementation
She has no family history of hematologic disorders or colon (E) Surgical flap therapy
cancer.
On physical examination, temperature is normal, blood
pressure is 124/80 mm Hg, and pulse rate is 78/min. BMI
Item 18
is 24. Cardiovascular, pulmonary, and neurologic examina An 80-year-old man is evaluated for a 1-year history of
tions are all normal. There is no thyromegaly. Abdominal progressive urinary symptoms including weak stream, hes
examination shows a well-healed surgical scar. The remain itancy, and nocturia four times nightly. He has coronary
der of the examination is unremarkable. A stool sample is artery disease and chronic heart failure. His current med
guaiac-negative. The laboratory results obtained by die ications are lisinopril, isosorbide dinitrate, aspirin, and
patient are shown. metoprolol.
On physical examination, vital signs are normal. He has
Laboratory studies: mild suprapubic tenderness and a symmetrically enlarged
Leukocyte count 4200/uL (4.2 x 109/L)
prostate without nodules or tenderness. The remainder of
Hemoglobin 10.9 g/dL (109 g/L) the physical examination is normal.
Mean corpuscular volume 107 fL
Platelet count 122,000/pL (122 x 109/L) Which of the following is the most appropriate diag
Reticulocyte count 1.5% nostic test to perform next?

Which of the following is die most appropriate test to (A) Postvoid residual urinary volume measurement
establish the diagnosis? (B) Plasma glucose level
(A) Bone marrow biopsy (C) Prostate-specific antigen testing
(B) Colonoscopy (D) Transrectal ulttasound
(C) Serum thyroid-stimulating hormone level (E) Urinalysis
(D) Serum vitamin B12 level
Item 19
Item 16 A 58-year-old man recently diagnosed with multiple

A 28-year-old woman is evaluated during a routine myeloma with bony metastases is evaluated before hospital
examination. Her 52-year-old mother was recently diag discharge. He is being discharged today on hospice care
after receiving melphalan and prednisone and having a poor
nosed with Huntington disease. The patient has no
response to dierapy. He is currently on intravenous mor
symptoms. She is planning on starting a family. Physical phine as needed for pain and has requested 10 mg every 4
examination, including complete neurologic examina hours over die last 24 hours. On physical examination, vital
tion, is normal. signs are normal.
Which of the following should be done next? Which of die following is the most appropriate treat
ment of diis patient's pain?
(A) Obtain genetic testing for Huntington disease
(B) Order brain MRI (A) Fentanyl transdermal patch
(C) Reassure the patient diat she is unlikely to develop (B) Methadone
Huntington disease (C) Short-acting hydromorphone
(D) Refer for genetic counseling (D) Sustained-release morphine

146
Self-Assessment Test

Item 20 indentations in her skin with prolonged standing. She has


no periorbital or upper extremity edema, chest pain, short
A 29-year-old man is evaluated for the gradual onset of
ness of breath, paroxysmal nocturnal dyspnea, orthopnea,
right-sided hearing loss. He reports a continuous high-
pitched ringing in his right car that has been present for 3 change in urinary habits, or abdominal fullness. She has no
to 4 mondis. history of malignancy, immobility, or hormone replace
ment dierapy.
On physical examination, vital signs are normal. When
On physical examination, vital signs are normal. BMI
a vibrating 512 Hz tuning fork is placed on the top of his
is 34. Results of die cardiovascular examination and abdom
head, it is louder in the left ear. When placed adjacent to his inal examination are normal. The lungs are clear. There is
right ear, it is heard better when outside the ear canal dian no inguinal lymphadenopathy, lower extremity rash, or ery-
when touching the mastoid bone. Otoscopic examination is
tiiema. There is 1+ pitting ankle edema bilaterally.
normal bilaterally. Neurologic examination is normal other
than right-sided hearing loss. Laboratory studies:
Creatinine 0.8 mg/dL (70.7 pmol/L)
Which of the following is die most appropriate man Albumin 4.1 g/dL (41 g/L)
agement of this patient? Alanine aminotransferase 28 units/L
(A) Biofeedback therapy Aspartate aminotransferase 24 units/L
Bilirubin 1.1 mg/dL (18.8 pmol/L)
(B) Immediate treatment with oral corticosteroids Sodium 140 meq/L (140 mmol/L)
(C) MRI of the posterior fossa and internal auditory Potassium 3.9 meq/L (3.9 mmol/L)
canal
Results of thyroid function testing are normal. Urinal
(D) Otolith repositioning maneuver
ysis shows no protein or blood.
Which of die following is die most appropriate next
Item 21 step in management?
A 72-year-old woman is evaluated in the emergency depart
ment after an episode of syncope. While watching a movie, (A) Compression stockings
die patient felt palpitations; the next thing she remembers (B) CT scan of abdomen and pelvis
is being on the floor. She experienced a similar episode (C) Furosemide
about 1 month ago. History is significant for hypertension, (D) Lower extremity venous duplex ultrasonography
hypodiyroidism, osteoporosis, and chronic kidney disease.
Medications are amlodipine, lisinopril, levothyroxine, and
calcium supplements. She currently feels well. Item 23
On physical examination, temperature is normal, blood A 29-year-old man is evaluated for a 1-day history ofTcft
pressure is 148/78 mm Hg, pulse rate is 84/min and reg shoulder pain. He was throwing a football approximately 30
ular, and respiration rate is 12/min. Oxygen saturation on yards when the pain began. The pain is located over die left
ambient air is normal. There is no thyromegaly, carotid lateral deltoid muscle. He notes weakness with abduction.
upsuokes are +2 without bruits, and there is no jugular He has no previous history of shoulder problems, no history
venous distention. Cardiac auscultation reveals a grade of trauma, and no paresthesia. He has been taking ibupro
2/6, early peaking, crescendo-decrescendo systolic murmur fen as needed for pain.
at the right upper sternal border with occasional extra beats. On physical examination, he is afebrile, blood pressure is
The remainder of the physical examination is normal. 126/80 mm Hg, and pulse rate is 96/min. There is pain in
A resting electrocardiogram and rhythm strip show a die left shoulder with active abduction beginning at approxi
sinus rate of 85/min with occasional premature ventricular mately 60 degrees, and he has difficulty actively abducting the
contractions but no sustained arrhythmia, normal axis and left arm beyond 60 degrees. The patient is unable to slowly
intervals, and no ischemic changes. lower his left arm to his waist (positive drop-arm test). He has
no pain widi his left arm in full flexion (negative Neer test).
Which of the following is the most likely cause of this
When the patient is asked to hold the arm extended anteriorly
patient's syncope? at 90 degrees widi the forearm bent to 90 degrees (at 12
(A) Aortic stenosis o'clock), he does not have pain with die arm internally rotated
to cross in front of the body (negative Hawkins test). There
(B) Cardiac arrhythmia
is no pain with forward elevation of the left arm to 90 degrees
(C) Myocardial ischemia
widi active adduction of die arm (negative cross-arm test).
(D) Transient ischemic attack Strength (odier than during abduction) is intact.
Which of the following is the most appropriate next
Item 22 step in management?
A 56-year-old woman is evaluated for a 6-month history of
(A) MRI of the left shoulder
symmetric bilateral lower extremity edema. She notices no
(B) NSAID dierapy
leg swelling upon arising in the morning; her symptoms
appear by midday and worsen thereafter. She does not have (C) Physical therapy
pain but notes her legs feel heavy and her shoes leave (D) Subacromial corticosteroid injection

147
Self-Assessment Test

Item 24 and external range of motion. There is pain with palpation


A 70-year-old man is admitted to the hospital for peritoni of the acromioclavicular joint. He has no pain with his left
tis. He has Alzheimer disease, hypertension, type 2 diabetes arm in full flexion (negative Neer test). He is able to slowly
lower his right arm to his waist (negative drop-arm test).
niellitus, cirrhosis, and ascites. He has been falling fre
quendy at home. His current medications are metformin, Which of the following is the most likely diagnosis?
donepezil, lisinopril, and propranolol.
On physical examination, temperature is 38.5 C (A) Acromioclavicular joint degeneration
(101.3 F), blood pressure is 130/70 mm Hg, pulse rate is (B) Adhesive capsulitis
80/min, and respiration rate is 12/min. He is oriented to (C) Rotator cuff tear
person and is in no acute distress. There is shifting dullness (D) Rotator cuff tendinitis
on abdominal examination and diffuse abdominal tender
ness widi palpation. There are areas of blanching erythema
on die lower back and buttocks. There is no ulceration or Item 27
skin breakdown. Laboratory studies show serum albumin
A 59-year-old man is evaluated during a follow-up exami
level of 2.8 g/dL (28 g/L).
nation. He has COPD and hypertension. He has an 80-
Which of the following interventions is most appro pack-year history of cigarette tise, but has recently decreased
priate for preventing pressure ulcers in this patient? his smoking to a half pack of cigarettes daily. Medications
are ipratropium and amlodipine.
(A) Doughnut-type device On physical examination, temperature is 37.3 C
(B) Free ambulation (99.2 F), blood pressure is 138/92 mm Hg, pulse rate is
(C) Indwelling urinary catheter 96/min, and respiration rate is 22/min. BMI is 29. He is
(D) Pressure-dismbuting mattress barrel-chested with diffuse wheezing on lung examination.
The remainder of the physical examination is normal.
Which of the following is die most appropriate man
Item 25
agement regarding diis patient's tobacco use?
A 56-year-old woman is evaluated for severe vaginal itching
and discomfort. Her symptoms have progressively wors (A) Assess his interest in smoking cessation
ened for die past 4 months. There is no associated vaginal (B) Prescribe bupropion
discharge or vaginal odor. She is experiencing significant (C) Prescribe nicotine replacement therapv
vaginal dryness and intercourse has become painful despite (D) Refer for smoking cessation counseling
the use of lubricants. She has been menopausal since age 53
years. Her only medications are calcium and vitamin D.
On physical examination, vital signs are normal. BMI Item 28
is 29. She has pale, dry vaginal walls with decreased rugae A 45-year-old woman is evaluated in die hospital after rad
and petechial hemorrhages. There is scant vaginal dis ical hysterectomy for cervical carcinoma. Aside from post
charge. Vaginal pH is 6.0. Wet mount shows occasional operative pain, she has no symptoms. She has no history of
leukocytes. "Whiff" test is negative. There are no clue cells venous thromboembolism or excessive bleeding. Her onlv
and no hyphae on potassium hydroxide preparation. current medication is morphine as needed.
Which of the following is the most appropriate man On physical examination, temperature is normal, blood
agement of this patient? pressure is 110/72 mm Hg, and pulse rate is 84/min.
There is trace edema in the legs. Prothrombin time, acti
(A) Oral conjugated estrogen with medroxyprogesterone vated partial thromboplastin time, and INR are normal.
acetate
In addition to early ambulation, which of die follow
(B) Oral metronidazole
ing interventions is the most appropriate in this
(C) Vaginal clotrimazole patient for thromboembolism prophylaxis?
(D) Vaginal estradiol
(A) Enoxaparin for 5 weeks
(B) Inferior vena cava filter placement
Item 26 (C) Unfractionated heparin until discharge
A 58-year-old man is evaluated for an 8-month history of (D) Warfarin for 3 months
slowly progressive right shoulder pain. The pain is located
over the anterior shoulder and is worse with moving his arm
across his chest and also when he fully abducts his arm. His Item 29
only medication is acetaminophen. A 55-year-old man is evaluated for a 1-day history of seeing
On physical examination, vital signs arc normal. He has flashing lights, "squiggly" lines, and floating objects in his
pain when he forward elevates his right arm to 90 degrees left eye followed by loss of vision at the outer periphery of
and actively adducts his arm across his chest wall (positive the eye shortly after having breakfast diis morning. He now
cross-arm test). There is pain with shoulder abduction describes seeing what looks like a curtain coming down in
beyond 120 degrees. He exhibits normal shoulder internal that location. He has myopia requiring prescription glasses.

148
Self-Assessment Test

On physical examination, vital signs are normal. Vision Item 31


in the right eye is 20/100 uncorrected and 20/40 widi A 30-year-old woman is evaluated during a follow-up exam
glasses. Vision in the left eye is 20/100 uncorrected and ination. She has had recurrent episodes of presyncope and
20/40 with glasses. Pupils are equally reactive to light and svneope over the past few months. She continues to have an
accommodation. There is no conjunctival injection. Find
episode every 3 to 4 weeks, with no discernible pattern or
ings on funduscopic examination are shown. trigger. She reports becoming light-headed and feeling
faint, without other associated symptoms, followed by tran
sient loss of consciousness for several seconds followed by
spontaneous recovery without residual symptoms. On pre
vious evaluation, an electrocardiogram (ECG) and echocar
diogram were normal. Results of 24-hour continuous
ambulatory ECG monitoring were unremarkable, and a
cardiac event recorder showed no arrhythmia associated
with presyncopal symptoms. History is significant for anxi
ety and intermittent insomnia; the patient takes no medica
tions for these conditions. There is no history of prior head
trauma. She docs not use drugs or alcohol.
On physical examination, temperature is normal.
Blood pressure is 122/68 mm Hg and pulse rate is 72/min
while supine. After three minutes of standing, blood pres
sure is 112/84 mm Hg and pulse rate is 88/min, without
reproduction of syncope or symptoms. The remainder of
the examination is normal. Serum electrolytes, kidney func
tion, and thyroid function studies are normal.
Which of the following is die most likely diagnosis?
Which of the following is the most appropriate next
(A) Central retinal artery occlusion step in the evaluation of this patient?
(B) Central retinal vein occlusion (A) Electroencephalography
(C) Ocular migraine (B) Exercise cardiac stress test
(D) Retinal detachment (C) Signal-averaged electrocardiogram
(E) Temporal arteritis (D) Tilt-table testing

Item 30 Item 32
A 73-year-old woman is admitted to the hospital for drug-
A 32-year-old woman is evaluated as a new patient. She is
related hypersensitivity syndrome. She was hospitalized 2
weeks ago for a right ankle fracture and subsequendy planning to attempt conception with her partner. She has a
underwent open reduction and internal fixation. On the day history of systemic lupus erythematosus complicated by
chronic kidney disease that has been inactive for several
of discharge she was noted to have a urinary tract infection
and was prescribed trimethoprim-sulfamethoxazole despite years off of treatment. She has had borderline blood pres
a previously documented allergy to this agent in her sure elevations since the diagnosis of kidney disease. She was
internist's office chart, which was paper based and not also diagnosed with impaired fasting glucose and mild
linked to the hospital's electronic order entry system and hyperlipidemia 2 years ago, both of which have been treated
widi dietary changes. Her current medications are calcium
drug allergy alert system.
and vitamin D supplements.
After admitting the patient to the hospital and stop On physical examination, blood pressure is 156/92
ping her antibiotic, which of die following is die most mm Hg and her vital signs are otherwise normal. BMI is 26.
appropriate immediate next step to reduce die likeli The remainder of the physical examination, including a
hood of future similar errors?
gynecologic examination, is normal.
(A) Discuss with the patient's internist the need to Laboratory studies:
emphasize to patients the importance of communi Electrolytes Normal
cating medication allergies with other caregivers Blood urea nitrogen 12 mg/dL (4.2 mmol/L)
(B) Emphasize to the patient the importance of knowing Creatinine 1.2 mg/dL (106.0 pmol/L)
and communicating her known allergies widi care Total cholesterol 250 mg/dL (6.4 mmol/L)
givers LDL cholesterol 160 mg/dL (4.1 mmol/L)
(C) Encourage hospital administration to consider imple HDL cholesterol 34 mg/dL (0.8 mmol/L)
mentation of an electronic health record Triglycerides 200 mg/dL (2.26 mmol/L)
Spot urine albumin/ 300 mg/g
(D) Plan an intervention to improve communication of
medication allergies from outpatient to inpatient creatinine ratio
records Hemoglobin Alc 7.5%

149
Self-Assessment Test

In addition to a daily prenatal vitamin, which of the activities of daily living, planning his day, or managing his
following is the most appropriate treatment? finances. He is frustrated but not depressed and is still able
to enjoy life. He has hypertension and hyperlipidemia, con
(A) Aspirin trolled with hydrochlorodiiazide and simvastatin.
(B) Lisinopril On physical examination, he is afebrile, blood pressure-
(C) Metformin is 140/82 mm Hg, and pulse rate is 78/min. Mini-Mental
(D) Simvastatin State Examination score is 25. The lungs are clear. The
heart is without murmur. Neurologic, motor, and sensory
examinations are normal.
Item 33
Which of die following is the most likely diagnosis?
A 78-year-old woman living in a nursing home is evalu
ated for incontinence. Over the past year, she has had pro (A) Alzheimer disease
gressive decline in her cognitive status and now spends (B) Mild cognitive impairment
most of the day in bed. She requires coaxing to join the
other residents in their communal meals and requires (C) Pseudodementia
assistance for eating and bathing. When accompanied by (D) Vascular dementia
an aide or family member, she is able to walk slowly to the
bathroom without leakage and to urinate. Medical history
is significant for dementia and depression treated with Item 36
citalopram. A 76-year-old woman was evaluated 10 days ago for weight
She is a frail, elderly woman in no acute distress. On loss and occasional hemoptysis. Non-small cell lung cancer
physical examination, temperature is normal, blood pressure was subsequently diagnosed. In addition to a 4 x 5 cm sin
is 132/88 mm Hg, and pulse rate is 68/min. BMI is 23. gle lesion in the left upper lobe of her lung, diere arc
Her score on die Mini-Mental State Examination is 14/30. metastatic lesions in the left humerus, as well as a single
Her gait is slow and she requires assistance. Abdominal lesion in the left lobe of the liver. She does all of her own
examination is without suprapubic fullness. Rectal exami activities of daily living and is able to walk 1 mile before
nation reveals normal sphincter tone. Results of urinalysis stopping because of fatigue. Her only medications arc a
are normal. daily multivitamin and a calcium supplement.
On physical examination, vital signs are normal. She
Which of the following is die most appropriate man
appears comfortable and in no distress and is interested in
agement of this patient?
life-prolonging therapy.
(A) Cystoscopy Which of the following is the most appropriate time to
(B) Indwelling Foley catheter
begin palliative care discussions with this patient?
(C) Pelvic floor muscle training
(A) After she develops symptoms
(D) Prompted voiding
(B) At the current visit
(E) Tolterodine
(C) When admitted to hospice care
(D) When she no longer desires active treatment
Item 34
A 31-year-old woman is evaluated during a routine office
visit. She is married and in a monogamous relationship widi Item 37
her husband of 10 years. She has been getting annual Pap A 58-year-old woman is evaluated for a 2-day history of
smears since die age of 20 years, all of which have been burning, stinging pain in her posterior neck that radiates
within normal limits, including the most recent, 1 year ago. down her left arm and began after she spent several hours
She has no family history of cervical cancer. On physical painting the ceiling of her home. She reports no trauma or
examination, vital signs are normal. other symptoms. She feels mild numbness and paresthesias
on the back of her left hand.
Which of the following is the most appropriate man
On physical examination, vital signs are normal. Neck
agement of this patient?
range of motion is limited by pain, especially in extension.
(A) Obtain human papillomavirus DNA testing Axial loading of the neck increases pain. There is slighdy
(B) Obtain Pap smear in 2 years diminished sensation on the back of the left hand. There arc-
no motor deficits. Reflexes are symmetric.
(C) Obtain Pap smear now
(D) Discontinue Pap smears Which of die following is the most appropriate man
agement of this patient?
Item 35 (A) Analgesics and avoidance of provocative activities
A 78-year-old man is evaluated for a 1-year history of for- (B) Cervical traction
getfiilness and not being able to remember names. He is a (C) Electromyography/nerve conduction study
retired attorney. He reports no problems with performing (D) MRI of cervical spine

150
Self-Assessment Test

Item 38 88/min, and respiration rate is 17/min. BMI is 32. She is


uncomfortable lying down and prefers to sit forward for the
A 55-year-old man is evaluated during a follow-up examination. Cardiac auscultation demonstrates normal
appointment for a 6-month history of nonproductive heart sounds with no murmur or rub. The remainder of the
cough. The cough predictably comes after meals, at bed examination is normal.
time, or any time he lies down, but it can occur at other
times as well. He experiences heartburn throughout the Electrocardiogram is shown. Chest radiograph is normal.
day. He has no shortness of breath, dyspnea on exertion, Which of die following is the most likely diagnosis?
fever, chills, postnasal drip, recent upper respiratory tract
infection, or wheezing. Omeprazole was prescribed 2 (A) Acute coronary syndrome
weeks ago but he reports no change in his cough or heart (B) Acute pericarditis
burn symptoms. He is a nonsmoker, does not drink alco (C) Acute pleuritis
hol, and takes no other medications. (D) Acute pulmonary embolism
Vital signs are normal, as is the remainder of die phys
ical examination. Chest radiograph is normal.
Item 40
Which of the following is the most appropriate treatment?
A 60-year-old man is evaluated for a 1-year history of gen
(A) Amoxicillin-clavulanatc eralized fatigue and lack of energy. He has had erectile dys
(B) Continue omeprazole ftinction for the past 9 months. He has chronic low back
(C) Inhaled albuterol pain and hypertension. Current medications are metopro
lol, hydrochlorothiazide, hydrocodone, and naproxen.
(D) Loratadine with pseudoephedrine On physical examination, vital signs are normal. Cardiac,
lung, and thyroid examinations are all normal. Laboratory
Item 39 studies show a morning total serum testosterone level of'180
ng/dL (6.2 nmol/L). Complete blood count, metabolic
A 42-year-old woman is evaluated for chest pain diat started
panel, and diyroid-stimulating hormone level are all normal.
a few days ago. It is midstcmal, sharp, constant, and worsens
with deep inspiration and recumbency. It does not radiate to Which of the following is the most appropriate next
the back and does not worsen widi physical activity. The pain step in die management of this patient?
has increased slighdy over die past day. She has no recent (A) Discontinue hydrocodone
fevers or chills, cough, joint pain, or rash. Two weeks ago, she
had symptoms consistent with acute tracheobronchitis. (B) Discontinue metoprolol
On physical examination, temperature is 37.5 C (C) Recheck testosterone level
(99.5 F), blood pressure is 122/80 mm Hg, pulse rate is (D) Start testosterone replacement therapy

151
Self-Assessment Test

Item 41 previously underwent lobectomy, now with adrenal metas


A 67-year-old man is evaluated during a routine examina tases. He has remained anuric since admission. This morn
tion. He has hypertension and obesity. He also has a history ing his serum potassium level was 7.2 meq/L (7.2
of gout, but has not had an attack in more than 1 year. His mmol/L) with electrocardiographic changes. It is clear that
current medications are lisinopril and a daily aspirin. dialysis is indicated. The patient is unable to give consent,
On physical examination, blood pressure is 140/82 and his wife is his surrogate decision maker. She says that he
mm Hg; vital signs are otherwise normal. BMI is 32. His was aware of the poor prognosis from his lung cancer and
waist circumference is 107 cm (42 in). There is no expressed a desire not to be kept alive on machines for a
hepatomegaly. long period of time. However, he was looking forward to
his great-grandson's graduation from college in 3 weeks and
Laboratory studies:
To t a l c h o l e s t e r o l 1 9 2 m g / d L ( 4 . 9 7 m m o l / L ) hoped that he could be able to attend. The wife is willing
to consent to dialysis.
HDL cholesterol 27 mg/dL (0.70 mmol/L)
On physical examination, temperature is 38.1 C
LDL cholesterol 68 mg/dL (1.76 mmol/L)
(100.5 F), blood pressure is 110/64 mm Hg, pulse rate is
(directly measured) 112/min, and respiration rate is 28/min.
Triglycerides 554 mg/dL (6.26 mmol/L)
Glucose 100 mg/dL (5.5 mmol/L) Which of the following is the most appropriate man
Creatinine 1.1 mg/dL (97.2 pmol/L) agement of this patient?
In addition to recommending weight loss and exercise, (A) Start long-term hemodialysis
which of the following is the most appropriate treat (B) Start temporary hemodialysis
ment for his lipid abnormalities?
(C) Withdraw all life-sustaining treatment
(A) Colesevelam (D) Withhold dialysis and continue medical treatment
(B) Extended-release nicotinic acid
(C) Fenofibrate Item 44
(D) Omega-3 fatty acids
A 38-year-old woman is evaluated for gritty, burning eyes
that worsen over the course of the day. She reports her eyes
Item 42 are often dry and are worse on windy days. She also reports
A 65-year-old man is evaluated for a 3-day history of scro dry mouth with difficulty salivating at times. She has no
other .symptoms.
tal pain. He notes some pain with urination and tenderness
On physical examination, vital signs are normal. The
in the left testicular region. He has felt febrile at home widi
some nausea and generalized weakness. He has not had sim conjunctiva is irritated. She has normal vision by Snellen
chart. Fundi are normal. Decreased tear production is doc
ilar symptoms before and has not had any trauma. He has
umented with the Schirmer test. The remainder of the phys
no nocturia, urinary frequency, or weak urinary stream. He
ical examination is normal.
takes no medications.
The antinuclear antibody test, rheumatoid factor, and
On physical examination, temperature is 38.7 C
anti-Ro/SSA and anti-La/SSB tests are positive.
(101.6 F), blood pressure is 140/80 mm Hg, pulse rate is
90/min, and respiration rate is 14/min. He is in moderate Which of the following is the most likely diagnosis?
distress. The left scrotum shows erythema with mild full
ness. The testicle itself is nontender, but there is fullness (A) Meibomianitis
superior to it that is extremely tender to palpation, widi (B) Primary Sjogren syndrome
some discomfort to palpation over the posterior aspect of (C) Rheumatoid arthritis
the testicle. The left testicle is lower in the scrotum than the (D) Systemic lupus erythematosus
right testicle. The prostate is normal in size and nontender.
Leukocyte count is 14,000/pL (14 x 109/L) with 18%
band forms. Item 45
Which of the following is the most likely diagnosis? A 28-year-old woman is evaluated for a painful lump in her
left breast of 6 weeks' duration. There is neither discharge
(A) Acute prostatitis from the nipple nor skin changes over the area. Her last nor
(B) Epididymitis mal menstrual period was 3 weeks ago and she thinks that
(C) Indirect hernia the lump became slightly larger right before and during her
menses. She is on low-dose oral contraceptives. She has no
(D) Orchitis
history of breast disease or breast biopsy. Menarche was at
(E) Testicular torsion age 12 years. She has never been pregnant. A maternal aunt
had breast cancer.
On physical examination, vital signs are normal. BMI
Item 43 is 24. There is a 1.5-cm mobile, soft, slightly tender mass
An 82-year-old man was admitted to the hospital 2 days ago in the lower mid quadrant of the left breast. There is no
with pneumonia, sepsis, and acute kidney injury. Medical nipple discharge and no abnormalities of the overlying
history is significant for recurrent lung cancer, for which he skin. The right breast has no masses. There is no axillary

152
Self-Assessment Test

lymphadenopathy. The remainder of the examination is Item 47


unremarkable. A 60-year-old man is admitted to the hospital with a trau
Which of die following is the most appropriate man matic hip fracture. A total hip arthroplasty under general
anesthesia is planned. He has COPD and reports that he is
agement of this patient?
significantly limited in his exercise tolerance because of dysp
(A) Core needle biopsy nea, although his functional capacity has remained stable-
(B) Mammography over die past 4 to 6 months. He has a cough with occasional
(C) Repeat clinical breast examination in 6 months white sputum, unchanged for the past 6 months. He has no
other acute respiratory symptoms. He smokes 1 pack per
(D) Ultrasonography
day of cigarettes. Current medications are tiotropium,
albuterol, and fluticasone/salnieterol.
Item 46 On physical examination, temperature is normal, blood
A 94-year-old woman is brought to the emergency depart pressure is 108/72 mm Hg, pulse rate is 78/min, and res
ment by her daughter for a 5-day history of progressive piration rate is 18/min. Oxygen saturation is 96% on ambi
weakness, anorexia, dizziness, and mild confusion. She was ent air. Pulmonary examination demonstrates scattered
hospitalized 2 weeks ago for an acute exacerbation of crackles and wheezing, unchanged from his baseline find
chronic heart failure that was treated with intravenous ings. Cardiac examination shows regular rate and rhythm
diuretics and an increase in her daily oral diuretic dose. She and a normal S, and S2.
initially did well following discharge, and a follow-up Which of the following should be recommended
appointment with her primary care physician is scheduled before surgery?
for next week. She has a history of chronic atrial fibrillation,
upper gastrointestinal bleeding owing to a duodenal ulcer (A) Nocturnal continuous positive airway pressure
18 months ago, COPD, hypertension, post-hcrpctic neu
(B) Chest radiograph
ralgia, chronic kidney disease, depression, anxiety, and sea
sonal rhinitis. Medications are furosemide, potassium chlo (C) Incentive spirometry
ride, aspirin, omeprazole, ipratropium and albuterol (D) Pulmonary function testing
inhalers, metoprolol, gabapentin, loratadine, and as-nccded
lorazepam.
On physical examination, she is a pleasant but frail- Item 48
appearing woman who is arousable but mildly confused. A 46-year-old man is evaluated for a 3-week history of occa
Temperature is 37.3 C (99.1 F), blood pressure is sional painless bright red rectal bleeding. He has no fatigue,
108/56 mm Hg, pulse rate is 95/min, and respiration rate lightheadedness, weight loss, or abdominal pain. His stools
is 16/min. Oxygen saturation is 94% on ambient air. The are frequendy firm, occasionally hard, and there is no
mucous membranes are dry. The pupils are symmetric and change in the frequency or consistency of his bowel move
reactive. Heart examination is significant for an irregularly ments. He has never been screened for colorectal cancer.
irregular rate and a grade 3/6 crcscendo-decrescendo On physical examination, temperature is 37.2 C
murmur at the right upper sternal border. The lungs are (98.9 F), blood pressure is 132/78 mm Hg, and pulse rate
clear to auscultation. The abdomen is scaphoid without is 84/min. Digital rectal examination yields a stool sample
hepatosplenomegaly. There is no peripheral edema. Her diat is positive for occult blood; the examination is other
neurologic examination is nonfocal except for her cogni wise normal. Anoscopy reveals a few internal hemorrhoids
tive deficits. without active bleeding. Laboratory studies show a blood
Laboratory studies show normal serum electrolytes and hemoglobin level of 14 g/dL (140 g/L).
a plasma glucose level of 110 mg/dL (6.1 mmol/L). Her
serum creatinine level is 1.4 mg/dL (123.8 pmol/L), Which of the following is the most appropriate man
increased from 1.2 mg/dL (106.1 pmol/L) at die time of agement of this patient?
hospital discharge. Her complete blood count reveals a (A) Banding of hemorrhoids
leukocyte count of 7500/pL (7.5 x 10<;/L) with a normal
differential, a hematocrit of 35%, and normal platelet count. (B) Colonoscopy
A urinalysis shows trace ketones but no cells. A chest radio (C) Fiber supplementation without further evaluation
graph is significant for severe kyphoscoliosis and changes (D) Home fecal occult blood testing
consistent with emphysema, but not pneumonia or heart
failure.
Which of the following is the most likely cause of die
Item 49
A 52-ycar-old man is evaluated during a periodic health
patient's clinical presentation?
examination. He has benign prostatic hyperplasia, and his
(A) Acute kidney injury father died of prostate cancer at the age of 74 years. His only
(B) Medication effect current medication is tamsulosin. He has no urinary symp
(C) Occult infection toms. Vital signs are normal, as is the remainder of the phys
(D) Recent stroke- ical examination.

153
Self-Assessment Test

Which of the following is the most appropriate man (C) Orlistat


agement? (D) Reduce current caloric intake by 500-1000 kcal/d
(A) Discuss the risks and benefits of prostate cancer
screening
Item 52
(B) Obtain a prostate-specific antigen level
A 75-year-old woman is evaluated during a follow-up
(C) Perform a digital rectal examination
examination for recently diagnosed symptomatic periph
(D) Perform a digital rectal examination and obtain a eral arterial disease. The patient has hypothyroidism,
prostate-specific antigen level hypertension, atrial fibrillation, and smokes cigarettes
(30-pack-year history). Her current medications are dil
Item 50 tiazem, warfarin, hydrochlorothiazide, levothyroxine,
A 72-year-old man is evaluated in the emergency depart
calcium, and vitamin D.
On physical examination, she is afebrile, blood pressure
ment for a 12-hour episode of* dizziness, described as a is 140/82 mm Hg, pulse rate is 66/min, and respiration
"spinning sensation" when he opens his eyes. He has nau rate is 12/min. BMI is 21. Posterior tibialis and dorsalis
sea without vomiting, has had no loss of consciousness, no
pedis pulses are diminished bilaterally (1+); die skin on the
palpitations, and no other neurologic symptoms. He- anterior aspect of die lower legs is shiny and hairless. Heart
requires assistance to walk. He prefers to keep his eyes rhythm is irregularly irregular and without murmurs. Neu
closed but has no diplopia. He has hypertension, hyperlipid
rologic and musculoskeletal examinations are normal.
emia, and type 2 diabetes mellitus. He had an upper respi
ratory tract infection 2 weeks ago. Medications are Laboratory studies:
To t a l c h o l e s t e r o l 2 3 8 m g / d L ( 6 . 1 6 m m o l / L )
hydrochlorothiazide, lisinopril, simvastatin, and metformin.
On physical examination, vital signs are normal. There HDL cholesterol 36 mg/dL (0.93 mmol/L)
are no orthostatic changes. Results of a cardiovascular exam LDL cholesterol 165 mg/dL (4.27 mmol/L)
ination are normal. He has no focal weakness. He cannot Triglycerides 205 mg/dL (2.32 mmol/L)
stand widiout assistance. Vertical nystagmus occurs immedi Serum creatinine 0.9 mg/dL (79.6 pmol/L)
ately widi die Dix-Hallpike maneuver. It persists for 90 sec In addition to strongly recommending smoking cessa
onds and does not fatigue. Electrocardiogram is consistent tion, which of the following is the safest treatment for
with left ventricular hypertrophy and shows no acute changes. diis patient?
Which of die following is the most appropriate next
(A) Atorvastatin
step in management?
(B) Pravastatin
(A) CT scan of die head without contrast (C) Rosuvastatin
(B) MRI widi angiography of the brain (D) Simvastatin
(C) Otolith repositioning
(D) Trial of vestibular suppressant medication
Item 53
A 42-year-old woman is evaluated for a 6-month history of
Item 51 heavy menstrual bleeding. She has been menstruating for
A 48-year-old woman is evaluated during a routine exami the last 8 days and is still going dirough 10 pads or more
nation. She is concerned about her gradual weight gain over daily with frequent clots. She has fatigue but no dizziness.
die years and requests counseling on how she can most Previous evaluation for diis problem has included normal
effectively lose weight. thyroid function and prolactin testing. She has no other
Over 8 years, she has gained approximately 18 kg (40 medical problems and takes no medications. Pelvic ultra
lb). With several commercial diets, she has lost weight but sonography has demonstrated a large posterior submucosal
fibroid. A surgical treatment is planned in 2 weeks.
always gains it back. She has a sedentary job, and often skips
breakfast or eats dinner on the run. She states she cannot fit On physical examination, vital signs are normal.
exercise into her busy day. She takes no medications and has Abdominal examination is benign, and the pelvic examina
no allergies. tion reveals a moderate amount of blood in the vaginal
On physical examination, temperature is normal, blood vault.
pressure is 132/70 mm Hg, pulse rate is 80/min, and res Hemoglobin level is 10.5 g/dL (105 g/L). Pregnancy
piration rate is 12/min. BMI is 32. There is no thy test is negative.
romegaly. The abdomen is obese, soft, nontender, and Which of the following is the most appropriate next
without striae. Fasting plasma glucose level is 106 mg/dL
management step?
(5.9 mmol/L) and thyroid function test results are normal.
Which of the following is die most appropriate next step (A) Estrogen/progesterone multiphasic oral contracep
tive
to help this patient achieve long-term weight reduction?
(B) Intravenous estrogen
(A) Exercise 15-30 minutes 5 days/week (C) Oral medroxyprogesterone acetate
(B) Laparoscopic adjustable band surgery (D) Reevaluation in 1 week

154
Self-Assessment Test

Item 54 spending sprees, lack of need of sleep, or previous psychiatric


A 31-year-old woman is evaluated for a 4-week history of problems. She is not interested in psychotherapy at diis time.
anterior knee pain. It developed insidiously and has pro Which of the following is the most appropriate next
gressively worsened. The pain worsens with prolonged sit step in treatment?
ting and with walking up and down stairs. There is no
morning stiffness. She has no history of trauma. She is tak (A) Add buspirone
ing acetaminophen as needed for the pain. (B) Continue bupropion at current dose for an additional
On physical examination, vital signs are normal. The 8 weeks
pain is reproduced by applying pressure to the surface of the (C) Discontinue bupropion, begin sertraline
patella with die knee in extension and moving the patella (D) Refer for electroconvulsive therapy
bodi laterally and medially. There is no effusion, swelling, or
warmth. Range of knee motion is normal, without crepitus
or pain. Item 57
Which of the following is die most likely diagnosis? A 44-year-old man is evaluated during a routine examina
tion. He is concerned about his general health and risk of
(A) Knee osteoarthritis diabetes niellitus. He has no medical problems. Both par
(B) Patellofemoral pain syndrome ents and his sister have type 2 diabetes mellitus.
On physical examination, temperature is normal, blood
(C) Pes anserine bursitis
pressure is 130/79 mm Hg, pulse rate is 66/min, and res
(D) Prepatellar bursitis
piration rate is 14/min. BMI is 28. The remainder of die
physical examination is normal.
Item 55 Laboratory studies:
Glucose (fasting) 104 mg/dL (5.8 mmol/L)
An 87-year-old woman is evaluated for dizziness of 1 year's
Total cholesterol 247 mg/dL (6.40 mmol/L)
duration. She describes feeling lightheaded and unsteady
when she walks but has not fallen. She denies vertigo, tin HDL cholesterol 50 mg/dL (1.30 mmol/L)
LDL cholesterol 177 mg/dL (4.58 mmol/L)
nitus, headache, loss of consciousness, chest pain, palpita
Triglycerides 100 mg/dL (1.13 mmol/L)
tions, or focal weakness. Medical history is significant for
hypertension, glaucoma, and left eye cataract. Current Which of the following interventions is the most
medications are lisinopril and latanoprost ophthalmic drops.
appropriate initial strategy to decrease diis patient's
On physical examination, blood pressure is 142/72 chance of developing type 2 diabetes mellitus?
mm Hg supine and 136/66 mm Hg standing, pulse rate is
72/min supine and 76/min standing. BMI is 22. On neu (A) Acarbose
rologic examination, she has 20/50 vision, decreased audi (B) Metformin
tory acuity, and 4+/5 motor strength throughout. Vibra (C) Pioglitazone
tion and position sensation are normal. There is no tremor,
(D) Weight loss and exercise-
cogwheeling, or bradykinesia, and her gait is not ataxic,
aldiough she feels safer holding on to the wall. There is no
nystagmus. The lungs are clear. Cardiovascular examination Item 58
is normal.
A physician is asked to advise the Pharmacy and Therapeu
Laboratory studies, including a metabolic profile and tics Committee of the hospital regarding a new drug to pre
complete blood count, are normal. vent deep venous thrombosis (DVT), drug "Z." The physi
Which of the following is die most appropriate man cian reviews a recent randomized controlled trial of 5000
agement of this patient? patients that compared drug Z with drug C, which is com
monly used and is on the hospital's formulary. The follow
(A) Physical therapy with gait evaluation ing data are abstracted from the trial:
(B) Replace latanoprost with timolol
Study results:
(C) Replace lisinopril with losartan Drug DVT Cases
(D) Vestibular rehabilitation therapy Drug Z (n = 2500) 25
Drug C (n = 2500) 50
Item 56 Based on these data, how many patients need to be
treated (number needed to treat, NNT) with drug Z,
A 50-year-old woman is evaluated during a follow-up
compared widi drug C, to prevent one extra case of DVT?
appointment for moderate depression. Eight weeks ago, she
was started on bupropion; 4 weeks ago, the dose was (A) 1
increased to the maximal dose. At this time, her PHQ-9 (B) 2
score has not improved over baseline, and she confirms that
her symptoms have not improved. She has no suicidal (C)25
ideation and does not have hallucinations or other psychotic (D) 100
features. She has no previous episodes of high energy, (E) 167

155
Self-Assessment Test

Item 59
A 21 -year-old man is evaluated in the emergency depart
ment for left ankle pain diat began 6 hours ago when he
inverted his left ankle while playing soccer. He was unable
to bear weight immediately after the injury, but is now able
to bear weight with some difficulty.
There is ecchymosis and swelling around the entire
ankle joint, with tenderness to palpation of the anterior
talofibular ligament. He is able to bear weight but finds it
painful to do so. There is no tenderness to palpation of bonv
structures (lateral and medial malleolus, base of fifth
metatarsal) or the Achilles tendon. There is no ankle insta
bility. Compression of die distal tibia and fibula does not
cause any discomfort (negative squeeze test).
Which of the following is the most appropriate man
agement for this patient?
(A) Ankle joint corticosteroid injection
(B) Ankle MRI Which of the following is die most likely diagnosis?
(C) Ankle radiograph
(A) Acute angle closure glaucoma
(D) Ankle splinting
(B) Central retinal artery occlusion
(E) Urgent surgical evaluation
(C) Central retinal vein occlusion
(D) Retinal detachment
Item 60
A 61-year-old woman is evaluated for hot flushes, which
have been persistent for the last 10 years. They occur at least Item 62
7 times per day, last for approximately 60 seconds, and are A 62-year-old man is evaluated before elective total hip
associated with severe sweating, palpitations, and occasional arthroplasty. He reports no prior medical problems aside
nausea. She is awakened several times per night. She has from hip osteoarthritis. His only medications are ibuprofen
tried herbal medications, including soy and black cohosh, and oxycodone. He drinks 1 pint of liquor daily.
but has not experienced any benefit. She has hypertension, On physical examination, temperature is normal, blood
type 2 diabetes mellitus, and hyperlipidemia. Five years ago, pressure is 100/62 mm Hg, and pulse rate is 92/min. He
she developed deep venous thrombosis after hip replace is alert and oriented. He has gynecomastia and multiple spi
ment surgery. Her current medications are ramipril, met der angiomata. He is jaundiced. There is ascites but no
formin, atorvastatin, calcium, and vitamin D. hepatomegaly or splenomegaly.
On physical examination, vital signs are normal. BMI Laboratory studies:
is 29. The remainder of the examination is normal. Platelet count 52,000/uL(52xl09/L)
Bilirubin (total) 2.3 mg/dL (39.3 pmol/L)
Which of die following is die most appropriate treatment?
Alanine aminotransferase 68 units/L
(A) Citalopram Aspartate aminotransferase 90 units/L
(B) Oral estrogen therapv' INR 1.8
(C) Oral estrogen/progesterone therapv Abdominal ultrasound shows a cirrhotic liver and
(D) Topical (vaginal) estrogen ascites. The patient's Cliild-Turcotte-Pugh (CTP) score is
class C.
(E) Venlafaxine
Which of the following is the best management of this
patient?
Item 61
A 60-year-old man is evaluated for new-onset monocular (A) Administer prednisolone for 1 week prior to surgery
cloudy vision of the left eye diat began 4 hours ago. He has (B) Administer vitamin K for 3 days prior to surgery
type 2 diabetes mellitus and coronary artery disease. His (C) Nonoperative management
current medications are aspirin, simvastatin, lisinopril, (D) Proceed with surgery
metoprolol, and metformin.
On physical examination, vital signs are normal. When
a light is shined into his left eye it is not reactive, but shin Item 63
ing a light in his right eye causes his left pupil to contract A 52-year-old man is evaluated during a routine examina
(left afferent pupillary defect). The visual acuity of the right tion. He is asymptomatic but is concerned about his weight.
eye is 20/30, that of the left eye is 20/120. Retinal findings Medical history is significant for prediabetes and elevated
are shown. The remainder of the examination is normal. cholesterol levels. He smokes one or two cigars a week. He

156
Self-Assessment Test

drinks one or two alcoholic beverages a few nights each Which of the following is the most appropriate treat
week. He does not get any regular exercise. ment?
On physical examination, vital signs are normal. BMI
is 33. The examination is otherwise unremarkable. The (A) Gabapentin
patient indicates he is ready to make important lifestyle (B) Ibuprofen
changes to improve his health. (C) Meperidine, orally
Which of the following is the best initial management? (D) Morphine, intramuscularly

(A) Assess the patient's confidence in making lifestyle


changes Item 66
(B) Determine which lifestyle change the patient believes A 52-year-old man presents for routine care. Several years
is most important ago he was told that his cholesterol level was borderline. He
is a vegetarian and a marathon runner, does not smoke, and
(C) Inform the patient he needs to lose weight
drinks alcohol only socially. He takes a daily multivitamin.
(D) Provide advice on smoking cessation
His father had a myocardial infarction at die age of 42 years.
On physical examination, vital signs are normal. BMI
Item 64 is 22. The remainder of the examination is normal.
A 56-year-old woman is evaluated during a follow-up visit Laboratory studies:
after presenting as a new patient 2 weeks ago. At that time, Total cholesterol 325 mg/dL (8.42 mmol/L)
her blood pressure was found to be elevated (156/88 mm HDL cholesterol 50 mg/dL (1.30 mmol/L)
Hg) and follow-up laboratory tests were ordered. She has LDL cholesterol 196 mg/dL (5.08 mmol/L)
had no major illnesses. Her father had type 2 diabetes mel Triglycerides 185 mg/dL (2.09 mmol/L)
litus and died at age 52 years of a myocardial infarction. She Glucose 72 mg/dL (3.9 mmol/L)
is currently taking no medications. Thyroid-stimulating 0.52 uU/mL (0.52 mU/L)
On physical examination, blood pressure is 156/92 hormone
mm Hg in the left arm and 160/90 mm Hg in the right
Which of the following is the most appropriate man
arm. Pulse rate is 86/min and respiration rate is 16/min.
BMI is 34. Waist circumference is 39 in (99 cm). Results of agement?
a funduscopic examination arc normal. (A) Calculate the non-HDL cholesterol level
Laboratory studies: (B) Measure high-sensitivity C-reactive protein
Blood urea nitrogen 16 mg/dL (5.7 mmol/L)
(C) Initiate fibrate therapy
Creatinine 0.9 mg/dL (79.6 pmol/L)
162 mg/dL (4.19 mmol/L) (D) Initiate statin therapy
LDL cholesterol (fasting)
HDL cholesterol (fasting) 32 mg/dL (0.83 mmol/L)
Triglycerides (tasting) 148 mg/dL (1.67 mmol/L) Item 67
Glucose (tasting) 98 mg/dL (5.4 mmol/L)
Trace protein, no glucose- A 72-year-old woman is evaluated for a fall three nights ago.
Urinalysis She lives in a single-floor apartment. At about 2 AM, she got
In addition to hypertension and obesity, which of the up to go to the bathroom and fell after bumping into a wall.
following is the most likely diagnosis? She had no lightheadedness or loss of consciousness and has
not fallen before. Her home has no rugs and no thresholds
(A) Hypertriglyceridemia between rooms. She normally has no problems walking and
(B) Impaired fasting glucose docs not use an assistive device.
(C) Metabolic syndrome On physical examination, vital signs are normal. BMI
(D) No additional diagnoses is 23. She is alert and oriented. There arc no orthostatic
blood pressure changes or pulse changes. Visual testing
using a Snellen chart reveals 20/20 distance vision in both
Item 65
eyes. The remainder of the physical examination, including
An 85-year-old man is evaluated following a recent diag a motor examination, is normal. In the Timed "Up & Go"
nosis of non-small cell lung cancer with metastatic disease test, she walks 10 feet in 10 seconds (normal, <14 sec).
to the liver, spine, multiple ribs, and sternum. He has
declined treatment. The patient describes pain in his ribs Which of the following is the most appropriate next
that is present throughout the day and wakes him from step in this patient's management?
sleep. He rates his pain as a 2 or 3 on a 10-point pain scale. (A) Ask about use of night lights
His only medication is acetaminophen, 1000 mg every 6
(B) Begin an individualized exercise program
hours, but this is not entirely effective in relieving his pain.
Palpation of the right anterior chest and sternum repro (C) Ophthalmology evaluation
duces his pain. (D) Provide patient with a walker

157
Self-Assessment Test

Item 68 beginning to interfere with his quality of life, particularly


A 28-year-old woman is evaluated after being brought to the the urinary frequency and nocturia. His only other med
office by her boyfriend. He reports that she has been hearing ical problem is hypertension, for which he takes lisinopril
voices and exhibiting increasingly paranoid behavior, believ and metoprolol.
On physical examination, he is afebrile, blood pressure
ing that the mailman is trying to poison her. She has not
is 140/85 mm Fig, pulse rate is 70/min, and respiration
gone to work in 4 weeks and spends most of her day alone
in her bedroom wearing head phones and listening to heavy rate is 14/min. BMI is 25. He has a symmetric moderately
metal rock music. Her boyfriend reports that she has had sev enlarged prostate gland with no prostate nodules or areas of
eral odier episodes of paranoid behavior over the past 8 tenderness. A urinalysis is normal.
mondis but none as bad as the current one. The patient is Which of die following is most appropriate next step
minimally interactive. She previously drank two beers daily in treatment of this patient's benign prostatic hyperplasia?
(none recently) and smoked marijuana at parties. Her fadier
was diagnosed widi schizophrenia at age IS years. Her (A) Add finasteride
mother died of breast cancer 2 years ago after a long illness. (B) Change doxazosin to finasteride
On physical examination, vital signs are normal. She (C) Change doxazosin to tamsulosin
appears disheveled and withdrawn. She is a thin woman, (D) Prescribe a fluoroquinolone antibiotic for 4 weeks
staring straight ahead and not making eye contact. She
declines to talk to the physician or undress to be examined.
A urine toxicology screen is positive for cannabinoids. A Item 71
complete blood count and basic chemistry panel are normal. A 30-year-old woman is evaluated during a routine examina
Which of the following is the most likely diagnosis? tion in November. She received a routine tetanus, diphtheria,
and acellular pertussis (Tdap) booster 5 years ago. She is sex
(A) Bipolar disorder ually active with a single lifetime sexual partner. She has had
(B) Cannabis abuse with psychosis no history of sexually transmitted infection. She was born in
(C) Major depressive disorder with psychotic features die United States and reports getting "routine shots" in child
hood. She has had regular Pap smears without any abnormal
(D) Schizophrenia
results; her most recent was 3 years ago. She does not smoke
cigarettes. She works as an attorney in a large corporate law
Item 69 firm. Findings on physical examination are unremarkable.
A 20-year-old woman presents for a gynecologic examina Which of the following vaccinations should be admin
tion and discussion of contraception. She has been sexually istered?
active for the past 3 years and has been using condoms.
(A) Hepatitis B vaccine series
However, she finds condom use inconvenient, aldiough she
admits she is bad about remembering to take pills. She has (B) Human papillomavirus vaccine series
had four partners in the past, currendy has a new partner, (C) Influenza vaccine
has no history of sexually transmitted infection, and has (D) Tetanus and diphtheria (Td) vaccine-
never been pregnant. Medical history and family history are
noncontiibutory. She drinks 2 to 4 beers on the weekends
and does not smoke cigarettes. Item 72
Physical examination, including pelvic examination, is A 32-year-old woman is evaluated following a diagnosis of
normal. chronic fatigue syndrome. She has a several-year history of
Which of the following is the most appropriate con chronic disabling fatigue, unrefreshing sleep, muscle and joint
traceptive recommendation for this patient? pain, and headache. A comprehensive evaluation has not iden
tified any other medical condition, and a screen for depression
(A) Condom use with combination estrogen-proges is normal. Her only medications are multiple vitamins and
terone pills dietary supplements. Physical examination is normal.
(B) Condom use with subcutaneous progesterone Which of die following is the most appropriate man
implants agement for this patient's symptoms?
(C) Depot medroxyprogesterone acetate
(A) Acyclovir
(D) Estrogen-progesterone vaginal ring
(B) Evening primrose oil
(C) Graded exercise program
Item 70 (D) Growth hormone
A 68-year-old man is evaluated for continuing urinary fre (E) Sertraline
quency and nocturia. His symptoms have been slowly pro
gressive over the past 1 to 2 years widi a weak urinary stream
and hesitancy. He was started on doxazosin 6 mondis ago, Item 73
which he tolerates well and initially provided some improve A 36-year-old woman comes for her fifth visit in the past 3
ment. However, his symptoms have continued and are months. She has a history of chronic abdominal pain and

158
Self-Assessment Test

reports continued excruciating, diffuse, chronic abdominal On physical examination, vital signs and the remainder
pain and bloating. She has intermittent diarrhea and con of the physical examination are normal.
stipation, but reports no weight loss or other localizing Laboratory studies:
symptoms. She is able to carry out routine activities of daily Hemoglobin A|C 8.1%
living. She has tried multiple over-the-counter medications Total cholesterol 223 mg/dL (5.78 mmol/L)
as well as previous prescriptions for omeprazole, psyllium 67 mg/dL (1.74 mmol/L)
HDL cholesterol
fiber supplements, dicyclomine, loperamide, and NSAIDs,
LDL cholesterol 140 mg/dL (3.63 mmol/L)
all of which she states "do not touch" her pain. She states
that she tried her friend's acetaminophen-oxycodone and Triglycerides 90 mg/dL (1.02 mmol/L)
had good relief. A previous workup (including complete In addition to recommending therapeutic lifestyle
blood count, comprehensive metabolic panel, amylase,
changes, which of die following is die most appropri
lipase, anti-transgltitaminase antibodies, and abdominal CT ate management of this patient's lipid levels?
scan) was negative. She reports several episodes of abuse as
a child and has been in a number of difficult and disruptive (A) Colesevelam
relationships as an adult. Although she smokes cigarettes, (B) Ezetimibe
she denies any past or present alcohol or drug use. She is
(C) Gemfibrozil
currendy on no medications.
Results of the physical examination are normal. When (D) Simvastatin
her request for acetaminophen-oxycodone is denied, she
becomes angry and upset, stating that all she needs is a med
icine that works. Item 76

A 67-year-old woman is admitted to die hospital with short
Which of the following is the most appropriate
ness of breath and is found to have a pulmonary embolus.
approach to diis patient? She is begun on low-molecular-weight heparin. Upon fur
(A) Initiate an ongoing discussion of the causes and sig ther evaluation, a large left breast mass is found along with
nificance of her pain a malignant left-sided pleural effusion. Biopsy of the breast
(B) Prescribe a limited number of acetaminophen- mass reveals poorly differentiated adenocarcinoma.
oxycodone tablets Although she has a limited social support system and mini
(C) Refer to a gastroenterologist mal understanding of her disease, she is interested in evalu
(D) Request that her care be transferred to another physi ating possible treatment options for her condition. She con
cian tinues to have mild shortness of breath and marked anxiety
related to her newly diagnosed condition, but otherwise
feels well.
Item 74
In addition to oncology and surgery consultations,
A 58-year-old man is evaluated as a new patient. A review which of die following is die most appropriate next
of his previous records shows he received a pneumococcal
vaccination 6 years ago when he was admitted to the hos step in this patient's care?
pital with community-acquired pneumonia. He feels well (A) Antidepressant therapy
with no acute symptoms. He has type 2 diabetes mellitus,
(B) Hospice care referral
hypertension, and hyperlipidemia. Medications are insulin
(C) Long-acting morphine
glargine, metformin, lisinopril, and simvastatin. Results of
the physical examination are unremarkable. (D) Palliative care consultation
When should this patient receive an additional pneu
mococcal vaccination? Item 77
(A) Today A 32-vear-old woman is evaluated for a 6-month history of
(B) Today and repeat every 5 years nonproductive cough. She has no history of recurrent upper
(C) Today and at age 65 years respiratory tract infections and has never smoked cigarettes.
She has no fever, dyspnea on exertion, hemoptysis, heart
(D) At age 65 years
burn, or wheezing. She has worked in the same office for 7
(E) No further pneumococcal vaccinations are required
years and has lived in the same house for the past 20 years.
She has not traveled out of the area for more than 2 years.
Item 75 She has no pets at home, no occupational or other exposure
to toxic chemicals, and no family history of pulmonary dis
A 30-year-old woman is evaluated for hyperlipidemia. Med
ease. She takes no medications.
ical history is significant for type 1 diabetes mellitus,
The vital signs and results of the physical examination
hypothyroidism, and hypertension. She is planning preg
are normal.
nancy. Her father was diagnosed widi coronary artery dis
ease at die age of 47 years. Her current medications are A complete blood count with differential is normal.
levothyroxine, hydrochlorothiazide, insulin glargine, and Chest radiograph is normal. Pulmonary function tests are
insulin aspart. normal and a methacholine challenge test is negative.

159
Self-Assessment Test

Which of the following is die most appropriate diag symptom resolution at that time. Her only medication is a
nostic test to perform next? prenatal vitamin.
On physical examination, vital signs arc normal. There-
(A) Bronchoscopy is vulvar edema, erythema, and excoriations with a thick,
(B) 24-hour esophageal pH manometry white, "cottage cheese" discharge present in the vaginal
(C) Sinus imaging vault. There is no cervical motion or adnexal tenderness.
(D) Sputum testing for eosinophils Vaginal pH is 4.5; potassium hydroxide preparation shows
yeast and hyphae. There are no clue cells or motile tri
chomonads on saline microscopy.
Item 78
Which of the following is die most appropriate treatment?
A 78-year-old woman is evaluated after she tripped while
carrying a garbage bag to die trash bin in her kitchen. She (A) Boric acid, topically
remembers falling but did not injure herself. She has had no (B) Clotrimazole, topically
previous falls. She reports no loss of consciousness, light (C) Fluconazole, orally
headedness, or dizziness. She has no history of seizures. She
lives in a one-floor apartment with no steps, no loose rugs, (D) Voriconazole, orally
and good lighting. She has a history of" hypertension. Her
daughter heard the fall and immediately came into the Item 81
kitchen; when she entered, her mother was already getting
back up and was not confused. Her only current medication A 56-year-old woman is evaluated for an 8-week history of per
is lisinopril. sistent nonproductive cough. The cough is paroxysmal and is
On physical examination, blood pressure is 138/85 preceded by a tickle in die back of her throat. She has no short
mm Hg, without orthostatic changes. There are no ecchy ness of breadi, hemoptysis, fever, chills, sore diroat, myalgia,
moses or tenderness over the hips and no pain on ambula otalgia, wheezing, or rhinorrhea. Approximately 3 months
tion. The physical examination is otherwise normal. ago, she was diagnosed widi type 2 diabetes niellitus and
hvpertension and was started on metformin, hydrochlorothi
Which of the following is die most appropriate man azide, lisinopril, and atorvastatin. She has a l()-pack-year his
agement of this patient? tory of tobacco use, but stopped smoking 5 years ago.
On physical examination, vital signs are normal. There
(A) Assess gait and mobility is no conjunctival injection, oropharyngeal erythema, or
(B) Discontinue lisinopril cobble-stoning. The lungs are clear and cardiovascular exam
(C) Prescribe an exercise program ination is unremarkable. Chest radiograph is normal.
(D) Provide a standard walker Which of the following is the most appropriate treat
ment?
Item 79 (A) Albuterol inhaler
A 38-year-old woman is evaluated for left knee pain. The (B) Discontinue lisinopril
pain has been present for the past 3 weeks. Before onset, she (C) Loratadine
had been preparing for a 5-kilometer race by running
(D) Omeprazole
approximately 2 miles per day, 6 days per week, for die past
6 months. Walking up steps makes the pain worse; she also
notes pain at night. She has never had this pain before. Item 82
On physical examination, vital signs are normal. There
A 70-year-old man is evaluated before elective cataract
is tenderness to palpation located near the anteromedial
surgery. Aside from decreased vision he has no symptoms.
aspect of the proximal tibia. A small amount of swelling is He has coronary artery disease and a seizure disorder. He
present at the insertion of the medial hamstring muscle. does not have chest pain, dyspnea, or recent seizures. He-
There is no medial or lateral joint line tenderness.
had a myocardial infarction 6 months ago and was treated
Which of the following is the most likely diagnosis? widi a drug-eluting stent placed in the left anterior descend
ing coronary artery. Current medications are aspirin, clopid
(A) Iliotibial band syndrome ogrel, simvastatin, metoprolol, and phenytoin.
(B) Patellofemoral pain syndrome On physical examination, temperature is normal, blood
(C) Pes anserine bursitis pressure is 142/88 mm Hg, and pulse rate is 64/min. The
(D) Prepatellar bursitis remainder of the examination is normal.
Which of the following is the best perioperative man
Item 80 agement of this patient's medications?

A 32-year-old woman at 30 weeks' gestation is evaluated for (A) Continue all medications
a 1-week history of thick, white vaginal discharge as well as (B) Discontinue clopidogrel 1 week before surgery
severe vaginal itching and discomfort. She was diagnosed (C) Hold metoprolol the morning of surgery
with vulvovaginal candidiasis 8 weeks ago, treated, and had (D) Hold phenytoin on the morning of surgery

160
Self-Assessment Test

Item 83 Which of the following is the most appropriate man


A 54-year-old man is evaluated for a long-standing history agement?
of COPD. Although he had previously done well, his lung
(A) Cognitive-behavioral therapy
function has progressively declined ewer the past year. He is
(B) Diazepam
oxygen dependent and is unable to perform even minor
(C) MRI of the abdomen
physical activity without severe dyspnea. He is not a trans
plant candidate and is unhappy with his quality of life and (D) Olanzapine
prognosis. He requests a prescription that he can take that
will cause him to die at the time of his choosing.
Item 86
Which of the following is die most appropriate next A 42-year-old woman is evaluated in the emergency depart
step in management of this patient's request? ment after fainting earlier in the evening. She was at a din
(A) Assess the adequacy of his current treatment ner party and reports having two glasses of wine. After
(B) Consult legal counsel about state law in such cases standing for approximately 35 minutes, she felt warm,
diaphoretic, and anxious; as she moved toward a chair, she
(C) Decline the request lost consciousness. She recovered spontaneously within 2
(D) Prescribe sedating medication that could ensure a minutes and has been completely lucid ever since. Medical
comfortable deadi
history is significant only for hypothyroidism and perennial
allergies; medications are levothyroxine and fexofenadine.
On physical examination, she is alert and oriented.
Item 84
Vital signs are normal without orthostatic changes. Thyroid
A 37-year-old woman is evaluated for right forefoot pain on is normal. The remainder of the physical examination is nor
the plantar surface. She describes the pain as burning in mal. A 12-lead electrocardiogram is normal.
character and worsening with standing. She feels as if she is
"walking on a marble." The pain began 2 to 3 months ago. Which of the following is the most appropriate next
She has never had this problem before. She frequently wears step in the management of this patient?
high heels. (A) Admit to hospital for observation and telemetry
On physical examination, there is tenderness to palpa
tion on the plantar surface of the foot in the space between (B) Head CT scan
the diird and fourth toes. There is no tenderness to palpa (C) Obtain echocardiography
tion of the plantar surface of the metatarsal head, no ten (D) Perform tilt-table testing
derness to palpation of the metatarsophalangeal joint, and
(E) No further testing
no dorsal metatarsal tenderness to palpation.
Which of the following is die most likely diagnosis?
Item 87
(A) Hammer toe A 78-year-old-woman is evaluated in the emergency depart
(B) Metatarsal stress fracture ment after she fell at home last night. She has long-stand
(C) Morton neuroma ing sleeping difficulties and last night got out of bed and fell
(D) Tarsal tunnel syndrome- in her hallway. She had no loss of consciousness and notes
left hip pain. She has hypertension, hyperlipidemia, and
gastroesophageal reflux disease. Her current medications
Item 85 are lisinopril, simvastatin, and omeprazole.
A 25-year-old woman presents as a new patient for re-eval On physical examination, she is afebrile. Blood pres
uation of an abdominal mass. She reports finding a right sure is 142/82 mm Hg supine and 138/76 mm Hg stand
lower quadrant mass 9 months ago. She reports that the ing, and pulse rate is 76/min supine and 78/min standing.
mass has been stable in size, does not vary with meals or a She appears frail with generalized weakness. There is mild
Valsalva maneuver, and is not tender. She has had no tenderness in the left lateral hip and weakness of the
change in bowel habits. A review of her chart reveals that quadriceps muscles bilaterally. There arc no ecchymoses in
she has been seen by two internists, a gastroenterologist, the left hip area. She is slow getting up from a chair and
and a general surgeon since her initial presentation. All has a slow walking speed but no ataxia. Distance vision
reported normal physical examination findings. She has no using glasses without bifocal lenses evaluated with a
history of colorectal cancer. She has been unable to work Snellen chart is normal. There is mild difficulty with near
because of the mass. vision evaluated using a near-vision testing card. Lungs are
On physical examination, her affect is normal, with no clear. The heart rhythm is regular with no murmur. There
evidence of delusional thinking or hallucinations. Vital is no focal neurologic deficit. Radiograph of the left hip
signs are normal. Abdominal examination shows no masses and femur reveals no fracture.
or hernia. Results of a metabolic panel are normal. An Acetaminophen is prescribed for pain. Arrangements
abdominal ultrasound 5 months ago and a CT scan of the are made for home physical therapy and for a visiting nurse
abdomen and pelvis 3 months ago both were normal. to perform a home safety evaluation.

161
Self-Assessment Test

Which of the following is the most appropriate addi Item 90


tional management of this patient? A 62-year-old woman is evaluated for a 3-month history of
(A) Discontinue lisinopril a palpable nonpainful breast mass. She has no nipple dis
(B) Prescribe vitamin D charge. She underwent menarche at age 14 years and
menopause at age 55 years. She has no history of previous
(C) Prescribe Zolpidem at bedtime breast biopsies and no family history of breast, ovarian, or
(D) Refer for prescription glasses with bifocal lenses colorectal cancer. Her current medications are calcium and
vitamin D. She took hormone replacement dierapy for 1
Item 88 year after menopause because of vasomotor symptoms.
On physical examination, temperature is 37.4 C
A 69-year-old woman is evaluated for involuntary leakage of
(99.3 F), blood pressure is 135/80 mm Hg, pulse rate is
urine widi coughing, sneezing, laughing, or when lifting 80/min, and respiration rate is 14/min. There is a firm,
heavy boxes at work. She has no dysuria, frequency, or nontender mass in die upper outer quadrant of the right
urgency and she has no mobility problems. She is gravida 4, breast, approximately 2 cm at its largest dimension. There
para 4, and underwent a total abdominal hysterectomy 20 is no nipple discharge or change in or fixation to the over
years ago for uterine fibroids. She has type 2 diabetes mel lying skin. There is no axillary lymphadenopathy. A diag
litus. Medications are metformin and lisinopril. She has no nostic mammogram obtained 2 days before the visit
known drug allergies. revealed no masses or calcifications.
On physical examination, vital signs are normal. BMI
is 31. There is bulging of the anterior vaginal wall when the Which of the following is the most appropriate man
patient is asked to cough, accompanied by leakage of urine. agement of this patient?
Bimanual examination is unremarkable. The remainder of
her examination is normal. (A) Breast MRI
Laboratory studies show fasting plasma glucose level of (B) Breast ultrasonography
89 mg/dL (5.0 mmol/L) widi hemoglobin Alc of 6.5%. (C) Core needle biopsy
Urinalysis is normal. (D) Reassurance-
Which of the following is the most appropriate treat
ment?
Item 91
(A) Pelvic floor muscle training A 44-year-old man is evaluated for low back pain. Five days
(B) Prompted voiding ago he was playing racquetball when he felt a popping sen
(C) Pubovaginal sling sation in his back and felt a shooting pain down his leg. The
(D) Tolterodine pain worsened over die next 2 to 3 days, causing some dif
ficulty with sleeping. He started taking ibuprofen on day 2,
and has improved slightly since then. He currendy rates his
Item 89 pain as 5 or 6 out of 10. He has no numbness, weakness, or
A 97-year-old woman was hospitalized with jaundice, bladder/bowel incontinence.
On physical examination, vital signs are normal. BMI
abdominal pain, weight loss, nausea, and intermittent vom
is 31. Straight leg raise test on bodi the left and right sides
iting 1 week ago. She was found to have poorly differenti
ated metastatic pancreatic adenocarcinoma. She lives with reproduces pain in die left leg. The ankle reflex is dimin
ished on the left side compared widi the right side. He is
her daughter. Current medications are morphine, a stool
able to walk with some discomfort. No motor or sensory
softener, and a laxative. On physical examination, vital signs
are normal. She is a depressed-appearing woman in no dis deficits are observed. Saddle anesdiesia is not present. Rec
tress who appears cachectic but comfortable. tal tone is normal.
During bedside discussions, the patient has deferred all Which of the following is the most appropriate man
medical decision-making to her family. They have asked diat agement of this patient?
"everything be done" and have declined to place die patient
on do-not-resuscitate status. They have requested that a sur (A) Analgesics and mobilization as tolerated
geon be consulted to remove the cancer and that an oncol (B) Complete blood count and erythrocyte sedimenta
ogist be consulted for initiation of chemotherapy. The tion rate
health care team has arranged a family meeting to address (C) Epidural corticosteroid injection
end-of-life care.
(D) Lumbar spine MRI
Which of the following is the best initial communica (E) Lumbar spine radiograph
tion strategy for die family meeting?

(A) Ask the patient's opinion about an advanced directive Item 92


(B) Explain that curative therapy is futile A 75-year-old man is hospitalized with sepsis leading to
(C) Explain the diagnosis and the prognosis multi-organ failure. A meeting with family members is
(D) Explore the family's understanding about the convened to discuss goals of care for the patient. The treat
patient's condition ment team, including infectious disease and critical care

162
Self-Assessment Test

consultants, has indicated that the patient is deteriorating is no longer able to live independently as she had before
despite optimized therapy, and the prognosis is poor. The hospitalization. She asks that the patient be transferred
daughter brings an Internet printout of a trial of a new med back to the hospital or emergency department if she devel
ication for sepsis. The abstract states "We gave drug 'X' to ops more acute medical issues following discharge, and
100 consecutive patients with refractory sepsis in our five requests that the patient receive everything short of
intensive care units located in the same geographic region. aggressive resuscitation with cardiopulmonary resuscita
Eight percent were alive at 30 days." Although drug "X" is tion and intubation if this situation were to arise. The
marketed in the United States, it is not FDA-approved for treating internist meets separately with the patient and she
treatment of sepsis. A quick literature search reveals no other agrees that these are her preferences.
studies of drug "X" in the treatment of sepsis.
Based on the patient's medical status and the wishes of
Which of the following is the main reason that it is dif the patient and her family, which of the following
ficult to determine the effectiveness of drug "X" based postdischarge care options is most appropriate?
on the published study?
(A) Inpatient rehabilitation facility
(A) No comparison group (B) Long term acute care hospital
(B) Outcome assessment not blinded (C) Residential hospice facility
(C) Patients not randomly assigned to treatment (D) Skilled nursing facility
(D) Small study size

Item 95
Item 93 A 76-year-old woman is evaluated for a 1-day history of
A 37-year-old man is seen as a new patient. He requests a headache, left eye pain, nausea and vomiting, seeing halos
refill of dexamphetamine, which he takes for attention- around lights, and decreased visual acuity of the left eye. She
deficit/hyperactivity disorder (ADHD). He was diag has type 2 diabetes mellitus, hypertension, and atrial fibril
nosed in childhood when he had difficulty in school and lation. Medications are metformin, digoxin, metoprolol,
has been on the medication ever since. His symptoms are hydrochlorothiazide, and warfarin.
generally well controlled with occasional impulsive behav On physical examination, temperature is 36.8 C
ior (traffic ticket 3 years ago, confrontation with his boss (98.2 F), blood pressure is 148/88 mm Hg, pulse rate
5 years ago). He has had no problems over the past 2 is 104/min, and respiration rate is 16/min. Visual acuity
years. He is otherwise healthy and drinks three or four wearing glasses is 20/40 (right eye) and 20/100 (left
beers per week. He smokes socially, less than one pack per eye). The left eye has conjunctival erythema. The right
week. He does not use illicit drugs. His only medication pupil is reactive to light, the left pupil is sluggish and con
is dexamphetamine. stricts in response to light from 6 mm to 4 mm. On pal
On physical examination, temperature is 36.6 C pation of the ocular globe, the left globe feels firm as com
(97.8 F), blood pressure is 149/92 mm Hg, pulse rate is pared with the right.
96/min, and respiration rate is 14/min. BMI is 23. The Which of die following is the most likely diagnosis?
remainder of the physical examination is normal.
Which of the following is the most appropriate man (A) Acute angle-closure glaucoma
agement? (B) Central retinal artery occlusion
(C) Ocular migraine
(A) Continue dexamphetamine
(D) Temporal arteritis
(B) Switch to atomoxetine
(C) Switch to fluoxetine
(D) Switch to methylphenidate Item 96
(E) Stop medications and reassess A 28-year-old man is evaluated for pain on die radial aspect
of die right wrist that occurs with use of the thumb. The
pain has been present for 2 weeks. He has never had diis
Item 94 pain before and has not had any trauma. He works as a com
An 85-year-old woman is evaluated before hospital dis puter programmer and plays video games for 3 to 4 hours
charge after a 2-week hospitalization for a traumatic right each night when he gets home from work.
hip fracture treated with open reduction and internal fixa On physical examination, vital signs are normal.
tion complicated by a pulmonary embolism, catheter-asso Localized tenderness to palpation is present over the dis
ciated urosepsis, and acute delirium. She has improved tal radial styloid; pain is present with resisted thumb
steadily but continues to require low-level supplemental abduction and extension, and the patient has pain on the
oxygen, remains significantly debilitated, and is able to par radial side of the thumb when he is asked to make a fist
ticipate in only 30 minutes of physical therapy daily. Med over the frilly flexed thumb and then to ulnar deviate the
ical history is significant for type 2 diabetes mellitus, hand (positive Finkelstein test). There are no palpable
hypertension, depression, and obesity. Her daughter meets masses; there is no joint pain, bogginess, or swelling; sen
widi die treating internist to discuss discharge planning. sation is intact throughout the wrist and hand, strength is
She feels her mother has been failing for several years and 5/5 throughout.

163
Self-Assessment Test

Which of the following is the most likely diagnosis? Item 99


(A) Carpometacarpal arthritis An 84-year-old man who resides in a skilled nursing facility
is brought to the office by his daughter, who reports that he
(B) de Quervain tenosynovitis
has become less active in the past few months. The patient
(C) Flexor carpi radialis ganglion cyst does not have a change in mood. He has multiple somatic
(D) Scaphoid fracture symptoms including headache, scalp pain, and constipation,
all of which are long-standing, intermittent, of brief dura
tion, and not associated with any positive physical examina
Item 97 tion or laboratory test findings. There has been no weight
A 32-year-old woman is evaluated for a 3-month history of loss. He has osteoarthritis but has no difficulty with ambu
left-sided jaw pain and clicking below her left ear when she lation and has had no falls. His wife died 4 months ago. His
chews. She reports no joint problems elsewhere, no visual only medication is acetaminophen.
changes, and no headache other than the jaw pain. She has On physical examination, he is afebrile, blood pressure
some trouble sleeping at night, but the pain is more likely is 148/92 mm Hg, and pulse rate is 68/min. He is slow to
to affect her at work. Once or twice a month she goes home- respond but answers questions appropriately. He is alert and
early because of jaw pain. oriented. His gait is slow but otherwise unremarkable. The
On physical examination, temperature is 37.3 C remainder of the physical examination is normal.
(99.1 F), blood pressure is 118/72 mm Hg, and pulse rate Complete blood count, comprehensive metabolic
is 60/min. There is mild tenderness and palpable crepitus
profile, and serum thyroid-stimulating hormone level are
over die left temporomandibular joint. Otoscopy is normal normal.
bilaterally. She has no lymphadenopathy, rash, or salivary
Which of the following is the most appropriate diag
gland masses. Oropharynx is normal. Thyroid is normal. She
has no tooth pain when teeth are tapped with a tongue blade. nostic test to perform next?

Which of the following is die most appropriate next (A) Dix-Hallpike maneuver
step in management? (B) Mini-Mental State Examination
(C) PHQ-9 depression assessment
(A) Fluoxetine
(B) Ibuprofen (D) Timed "Up & Go" test
(C) law MRI
(D) law relaxation, heat, and therapeutic exercises Item 100
(E) Radiography of the teeth A 60-year-old woman is evaluated for increased irritabilitv
and anxiety. She was in an automobile accident 3 mondis
ago in which she was rear-ended by a car at a stop light.
Item 98 Since that time she has nightmares about the incident and
A 49-year-old woman is evaluated for vertigo of 1 week's states she has not returned to driving for fear of being in
duration. She was seen 1 week ago in the emergency depart another accident. Her sleep is poor and her husband states
ment. During that visit, she described severe vertigo that she is becoming more socially isolated since she has stopped
predictably occurred while abruptly turning her head to the driving. She has continued to perform her usual hobbies at
right and lasted less than 1 minute. She had no antecedent home. She has no suicidal thoughts. On physical examina
viral illness, headache, hearing loss, tinnitus, diplopia, tion, all vital signs are normal.
dysarthria, dysphagia, or weakness. She was diagnosed widi
Which of the following is the most likely diagnosis?
benign paroxysmal positional vertigo and given instructions
for head tilting exercises (Epley maneuver). Her symptoms (A) Generalized anxiety disorder
improved but have not abated. She is afraid to drive because (B) Major depressive disorder
of the symptoms. She has no history of hypertension, dia
betes mellitus, hyperlipidemia, or tobacco use. (C) Obsessive-compulsive disorder
On physical examination, vital signs are normal. With (D) Posttraumatic stress disorder
the Dix-Hallpike maneuver, she develops horizontal nys
tagmus and nausea after 15 seconds. The nystagmus lasts Item 101
approximately 1 minute. The Epley maneuver is unsuccess
ful in relieving symptoms. The remainder of the examina A 59-year-old woman is evaluated in the emergency depart
tion is normal, including the neurologic examination. ment for midsternal chest pain. The pain began several hours
ago as a vague ache in her left upper sternal region that pro
Which of the following is the most appropriate man
gressed in intensity and severity. The pain abated sponta
agement? neously after approximately 45 minutes. She had no further
chest pain until several hours later, when it recurred unpro
(A) Brain MRI
voked by exertion. She has no shoruiess of breath, nausea or
(B) Hydrochlorothiazide
vomiting, syncope, previous history of chest pain, or known
(C) Meclizine cardiac disease or risk factors for venous thromboem
(D) Vestibular rehabilitation bolism. Medical history is significant for hyperlipidemia and

164
Self-Assessment Test

hvpertension. She does not smoke cigarettes. Medications Which of die following is the recommended interval
are simvastatin, aspirin, lisinopril, and hydrochlorothiazide. for follow-up screening for hypertension in this
On physical examination, she is afebrile, blood pressure- patient?
is 110/70 mm Hg, pulse rate is 68/min, and respiration
rate is 22/min. BMI is 28. Oxygen saturation on ambient (A) 1 year
air is 97%. Estimated central venous presstire is 8 cm H20 (B) 2 years
and carotid pulses are without bruits. Lungs arc clear. Heart (C) 3 years
sounds are normal. There is a grade 2/6 holosystolic mur (D) 4 years
mur at the left sternal border with radiation to the apex.
(E) 5 years
There is no lower extremity edema. The remainder of the
examination is normal.
Electrocardiogram is shown. Chest radiograph is normal. Item 103
Which of the following is die most appropriate initial A 72-year-old woman is evaluated for short-term memory
management of this patient? loss. She has trouble remembering names, where she placed
certain items such as her keys, and occasionally, what she did
(A) Adenosine stress test earlier in the day. She avoids some social situations and feels
(B) Admit to the coronary care unit lower self esteem because of memory problems and
(C) CT pulmonary angiography decreased social contact, but notes no depression, low
(D) Ibuprofen administration energy, or sleep disturbance, and she still enjoys playing
cards with her husband. She does not need help with eat
ing, dressing, or bathing. She has hypertension, well con
Item 102 trolled with hydrochlorothiazide. She has no history of
A 36-year-old man is evaluated during a routine health stroke. She is concerned about her condition and wants to
examination. He has no family history of hypertension, know if anything can be done about it.
On physical examination, temperature is 37.2 C
hyperlipidemia, or cardiovascular disease. Fie has never used
tobacco, drinks approximately two beers each week, and (98.9 F), blood pressure is 135/84 mm Hg, and pulse rate
docs not use illicit drugs. He is fairly sedentary but feels well is 72/min. She is conversant, with a normal range of affect.
and is without cardiovascular or other symptoms. Neurologic examination is without focal deficit. The
On physical examination, he is afebrile, blood pressure remainder of the physical examination is normal.
is 112/70 mm Hg, and pulse rate is 76/min. BMI is 25. Mini-Mental State Examination score is 26.

Laboratory studies: Which of the following is the most appropriate man


To t a l c h o l e s t e r o l 1 4 8 m g / d L ( 3 . 8 3 m m o l / L ) agement of this patient?
LDL cholesterol 96 mg/dL (2.48 mmol/L)
HDL cholesterol 44 mg/dL (1.14 mmol/L) (A) Anticholinesterase inhibitor
Triglycerides 88 mg/dL (0.99 mmol/L) (B) Cognitive rehabilitation

4-JU_>
i P- i jlj-i | ; : I ' . v i fl .; .;.' I: <

l--i^i^^i:___^l.r^r_A-':^^.">'>C_^(^-r^--:^ : /v_^, A rt i A^-A|_AJ| _/*&*

P i i ^ W W 11 I Itrl-KTlTiB-
MiBi^iit^

;; uk" |i323tff;6!i-"-^-SJvrpd-:2V;n>ni7TCB';;:|;iiii!mhV;Tbi;iiiw/niV"i " Chtjrrr) ri-rem/TJiy.:,;!: ,,.;,/,,:,,,!,.., .,


ITEM 101

165
Self-Assessment Test

(C) PET scan Item 106


(D) Reassurance that progression to dementia is unlikely A 28-year-old man is evaluated for a 6-month history of
pelvic pain, urinary frequency, and painful ejaculation. He
has been treated with antibiotics for urinary tract infections
Item 104
three times in the past 6 months, each time widi temporary
A 70-year-old man is evaluated for sharp left-sided pleuritic relief of symptoms but recurrence shordy after completion
chest pain and shortness of breadi that began suddenly 24 of antibiotics.
hours ago. The pain has been persistent over the past 24 On physical examination, vital signs arc normal. There
hours and does not worsen or improve widi exertion or is minimal suprapubic tenderness with palpation. The
position. History is significant for severe COPD, hyperten
prostate is of normal size with minimal tenderness and no
sion, and hyperlipidemia. He is a current smoker with a 52- nodules. Urinalysis shows multiple leukocytes, bacteria,
pack-year history of smoking. Medications are ipratropium, and no erythrocytes.
albuterol, lisinopril, simvastatin, and aspirin.
On physical examination, temperauire is normal, blood Which of the following is the most appropriate treat
pressure is 128/80 mm Hg, pulse rate is 88/min, and res ment of this patient?
piration rate is 18/min. BMI is 24. Oxygen saturation on
ambient air is 89%. Cardiac examination reveals distant (A) 1-week course of trimethoprim-sulfamethoxazole
heart sounds but no S3. Lung examination reveals hyper- (B) 1 -month course of ciprofloxacin
resonance, decreased chest wall expansion, and decreased (C) Cognitive-behavioral therapy
breath sounds on die left. The trachea is midline.
(D) Finasteride
Which of the following is the most appropriate diag
nostic test to perform next?
Item 107
(A) Chest CT A 28-year-old woman is evaluated for headache, purulent
(B) Chest radiography nasal discharge, and left unilateral facial and maxillary tooth
(C) Echocardiography pain present for 4 days.
(D) Electrocardiography On physical examination, temperature is 37.3 C
(99.1 F); vital signs are odierwise normal. There is mild
tenderness to palpation over the maxillary sinus on die left.
Item 105 Nasal examination shows inflamed turbinates bilaterally
A 28-year-old man is evaluated for a 6-week history of with a small amount of purulent discharge. Maxillary trans
intractable nausea. He states he is nauseated all day illumination is darker on die left dian on die right. Oto-
although he is able to eat if he forces himself. He has had scopic examination is normal bilaterally. There is no lym
no vomiting, weight loss, or change in stool pattern. He
phadenopathy in the head or neck.
went to an emergency department last week because of his
symptoms, and the results of laboratory testing at diat time Which of the following is the most appropriate next
were normal. step in management?
He reports a 1-year history of intermittent episodes of
(A) Amoxicillin
severe right upper quadrant abdominal pain and bloating,
as well as separate episodes of intermittent numbness of the (B) Chlorpheniramine
right side of his face and body. He says that he has had neck (C) Nasal culture
and back pain, dysuria, intermittent odynophagia, and loss (D) Sinus CT
of libido for die past 3 years. He does not have sleep dis
(E) Systemic corticosteroids
turbance, anhedonia, or crying spells. Prior laboratory test
ing, upper endoscopy, colonoscopy, CT scan of the
abdomen and pelvis, and MRI of the cervical and lumbar Item 108
spine were normal. Current medications are aceta A 72-year-old woman is evaluated for sudden hearing loss
minophen, odansetron, and tramadol. He is currendy in die left ear with moderate ringing that started yesterday.
unemployed. She has no vertigo or dizziness.
On physical examination, vital signs are normal. The
On physical examination, vital signs are normal.
abdomen shows diffuse tenderness to palpation but is oth
erwise normal. Neurologic examination is normal. A two- Otoscopic examination is initially obscured by cerumen
question depression screen is normal. bilaterally. Once cerumen is removed, the tympanic
membranes appear normal and there is some redness in
Which of the following is the most likely diagnosis? the canals bilaterally. When a 512 Hz tuning fork is

(A) Celiac disease placed on top of the head, it is louder in the right ear.
When placed adjacent to the left ear, it is heard better
(B) Malingering when outside the ear canal than when touching the mas
(C) Multiple sclerosis toid bone. Neurologic examination is normal other than
(D) Somatization disorder left-sided hearing loss.

166
Self-Assessment Test

Which of die following is the most appropriate man Item 111


agement of this patient? A 25-year-old woman is evaluated for a 1-week history of
malodorous vaginal discharge associated with vulvar itching
(A) Acyclovir
and burning. She is sexually active and has had three part
(B) Neomycin, polymyxin B, and hydrocortisone ear ners in the past 6 months. She has no history of sexually
drops transmitted infection.
(C) Triethanolamine ear drops On physical examination, vital signs are normal. BMI
(D) Urgent audiometry and referral is 22. There is a thin, gray, homogeneous discharge coating
the vaginal walls. There are no external genital lesions and
no vulvar erythema or excoriations. The cervix appears nor
Item 109 mal. There is no cervical motion tenderness or adnexal ten
A 78-year-old man is brought to the emergency department derness. Vaginal pH is 6.0, "whiff" test is positive. Results
with a 1-hour history of vomiting bright red blood. Despite of saline microscopy are shown. Microscopy after the addi
profuse hematemesis, he clearly states that he does not want tion of potassium hydroxide does not show hyphae or
a blood transfusion for religious reasons. Four minutes after pseudohyphae. A urine pregnancy test is negative.
he arrives, he starts to have new severe substernal chest pain
and 2 minutes later loses consciousness. His wife, who he-
appointed his agent with durable power of attorney for
health care, confirms his long-standing religious beliefs
against transfusion. Medical history is significant for coronary
artery disease, hypertension, and hyperlipidemia. There is no
history of cognitive decline or impaired judgment. His cur
rent medications are aspirin, simvastatin, and amlodipine.
On physical examination, temperature is 36.8 C !
(98.2 F), blood pressure is 80/40 mm Hg, pulse rate is
156/min, and respiration rate is 24/min. His skin is pale,
clammy, and cool to touch. The chest is clear to ausculta
tion. Cardiac examination reveals tachycardia but is other
wise normal. The abdomen is soft and nondistended. 0
Complete blood count shows a hemoglobin level of 6
g/dL (60 g/L) and hematocrit of 18%. Electrocardiogram Unstained, 400x
shows 2- to 3-mm ST-segment depression in leads V3
through V6.
Which of die following is die most appropriate treat
Which of the following is the most appropriate man
ment?
agement?
(A) Immediate blood transtiision (A) Clotrimazole cream, 7-day topical regimen
(B) Fluconazole, single-dose oral regimen
(B) Obtain an emergency court-appointed guardian
(C) Seek permission from the patient's wife to transfuse (C) Metronidazole, 7-day oral regimen
(D) Metronidazole, single-dose oral regimen
(D) Treat without transfusion

Item 110 Item 112


A 24-year-old man is evaluated in the emergency depart A 56-vear-old man is evaluated for a 7-month history of dif
ment for a 6-hour history of acute scrotal pain. The pain ficulty maintaining erections. He has difficulty with sexual
intercourse and achieving orgasm. He reports that sexual
occurred suddenly while mowing the lawn. The patient is
desire is good. He does not have penile curvature, genital
not sexually active and has no recent trauma, history of
pain, depressed mood, or anxiety. He has not tried anything
penile discharge, urinary urgency, frequency, or dysuria. for this problem. He is active, jogging for 30 minutes 4 to
On physical examination, he is afebrile, blood pressure is
5 times per week. He has type 2 diabetes mellitus, which is
160/100 mm Hg, pulse rate is 100/min, and respiration rate- diet controlled, and hyperlipidemia. He does not use alco
is 12/min. The right testicle rides high in the scrotum and is
hol, tobacco, or illicit drugs. Current medications are
exquisitely tender. The cremasteric reflex is absent on the right
side. There is no abnormal mass in the scrotum or inguinal aspirin and simvastatin.
On physical examination, he is afebrile, blood pressure
area. There is no penile discharge. A urinalysis is normal.
is 124/70 mm Hg, and pulse rate is 76/min. BMI is 24.
Which of die following is the most likely diagnosis? The testes are bilaterally descended and normal in size and
consistency, the penis is normal in appearance, and there is
(A) Epididymitis normal distribution of male secondary hair. Rectal exami
(B) Strangulated inguinal hernia nation reveals a normal-sized prostate. Posterior tibialis and
(C) Orchitis dorsalis pedis pulses are palpable bilaterally and the neuro
(D) Testicular torsion logic examination is normal.

167
Self-Assessment Test

Serum thyroid-stimulating hormone level is 2.95 82/min, and respiration rate is 16/min. He has nasal con
pU/mL (2.95 mU/L) and early morning serum total gestion and has an occasional cough. There is mild clear
testosterone level is 320 ng/dL (11 nmol/L). nasal discharge with no sinus tenderness. The oropharynx
Which of die following is die most appropriate treat is without injection or exudate. There is no lymphadenopa
ment? thy. External auditory canals are normal. The tympanic
membranes are dull bilaterally but widiout injection. A
(A) Intracavernosal alprostadil small left middle ear effusion is noted.
(B) Penile pump Which of the following is the most appropriate man
(C) Sildenafil agement?
(D) Testosterone therapy
(A) Amoxicillin
(B) Erythromycin
Item 113 (C) Referral to an otorhinolaryngologist
A 78-year-old woman was recendy admitted to die hospital (D) Reassurance and observation
for an acute exacerbation of chronic heart failure. She has
coronary artery disease and hypertension. During that
admission she was treated with intravenous diuretics, widi Item 115
reduction of her weight to slightly below her established A 32-year-old man is evaluated for daytime fatigue of 9
optimal weight goal and resolution of her heart failure months' duration. He has never fallen asleep at the wheel,
symptoms. Upon discharge her dosages of lisinopril and but falls asleep at other times during the day. He does not
oral furosemide were increased from their preadmission
think he snores, but his wife is unavailable to confirm this.
level, and spironolactone was started. She was scheduled for He reports no leg symptoms. He has no significant medical
a follow-up appointment with her internist in 1 week.
Four days after discharge she presents to the emergency history and takes no medications. He does not smoke. He
drinks two or three beers on Friday and Saturday nights. He
department because of worsening shortness of breath since does not exercise regularly, and has gained 9.1 kg (20 lb)
hospital discharge and is readmitted for further treatment. since getting married 18 months ago.
She has no chest pain but has noticed increased swelling in
On physical examination, temperature is normal, blood
her lower extremities. She states diat she has been taking her
medications as directed. pressure is 128/76 mm Hg, and pulse rate is 82/min. BMI
On physical examination, temperature is 37.4 C is 32. Neck circumference is 43 cm (17 in). Pharynx is nor
mal. The thyroid is difficult to palpate owing to die patient's
(99.3 F), btood pressure is 115/78 mm Hg, and respira
tion rate is 18/min. Oxygen saturation on ambient air is large neck size. The lungs are clear, and the cardiovascular
89%. The chest examination reveals mild bilateral crackles at and neurologic examinations are normal.
the lung bases. Heart examination shows a regular rate In addition to counseling regarding sleep hygiene and
without murmur. There is trace lower extremity edema.
weight loss, which of die following is the most appro
The remainder of the examination is unremarkable.
priate management for this patient?
Laboratory studies are significant for normal serum
electrolyte levels and a serum creatinine level of 1.2 mg/dL (A) Advise alcohol abstinence
(106 pmol/L) (unchanged from discharge). A chest radio (B) Initiate therapy with Zolpidem
graph shows bilateral hilar infiltrates consistent widi pul (C) Order iron studies
monary edema. (D) Refer for polysomnography
Which of the following is the most likely cause of her
readmission?
Item 116
(A) Diuretic resistance A 46-year-old man is evaluated for dull, aching, right groin
(B) Inadequate hospital follow-up pain. He says it has been present for the past 3 months. The
(C) Medication nonadherence pain was initially severe, improved for a few weeks, but
(D) Spironolactone intolerance- slowly has been worsening since that time. He is now walk
ing with a limp. He has no history of trauma. He drinks six
beers a day and has done so for the past 25 years. He is a
Item 114 current smoker, with a 30-pack-year history.
An 84-year-old man is evaluated for a 5-day history of rhini On physical examination, vital signs are normal. He has
tis, nasal congestion, sneezing, and nonproductive cough. limited internal and external rotation of die right hip. Inter
The symptoms began with a sore throat, which resolved nal rotation is limited to a greater degree dian external rota
after 24 hours. He has mild ear pain when blowing his nose tion and pain is present widi rotating die diigh medially and
or coughing. He has a history of coronary artery disease and laterally (log-rolling). Radiographs of the right hip are normal.
hypertension. Medications are aspirin, metoprolol, and Which of die following is the most likely diagnosis?
hydrochlorothiazide.
On physical examination, temperature is 36.5 C (A) Hip osteoarthritis
(97.7 F), blood pressure is 130/72 mm Hg, pulse rate is (B) Ll radiculopathy

168
Self-Assessment Test

(C) Osteonecrosis of the hip times per week and are accompanied by palpitations and
(D) Septic arthritis of the hip sweating, as well as a "sense of doom." They resolve spon
taneously after approximately 30 minutes. His symptoms
are unrelieved with antacids, can occur at rest or widi exer
Item 117 tion, and are nonpositional. There are no specific precipi
A 33-year-old woman is evaluated for chronic pain. She has tating factors. He is a nonsmoker. His lipid levels, which
were checked recendy at a school wellness fair, arc normal.
a history of fibromyalgia and reports widespread muscu He has no personal or family history of coronary artery dis
loskeletal pain involving her hips, knees, back, and neck with
ease, diabetes mellitus, hyperlipidemia, or hypertension. He
significant worsening in recent weeks. The pain in her neck is not taking any medications.
and back is 7/10 in intensity and is constant. She partici On physical examination, vital signs are normal. He has
pated in a 3-month course of physical therapy with litde no cardiac murmurs and no abdominal pain. Complete
improvement in her symptoms. The patient reports that she blood count, serum thyroid-stimulating hormone level, and
is sleeping about 3 to 4 hours per night because her pain is
electrocardiogram are all normal.
keeping her awake. Previously she had been able to work,
but now she has difficulty getting out of bed every day. Her Which of die following is the most appropriate man
current medications are gabapentin and duloxetine. agement of this patient?
On physical examination, vital signs are normal. BMI
is 28. She has several tender trigger points. Her neck has (A) Cardiac event monitor
limited range of motion secondary to pain. There is ten (B) Cardiac stress test
derness to palpation over the paravertebral muscles of the (C) Empiric trial of proton pump inhibitor
lower lumbar spine. Strength is 5/5 in all extremities, (D) Selective serotonin reuptake inhibitor
reflexes are 2+ and symmetric, sensation to light touch and
pinprick is normal throughout, and a straight leg raise test
is negative. Her affect is flat. Item 120
A 47-year-old woman is evaluated during a follow-up
Which of the following is the most appropriate man
appointment. She had been hospitalized for acute pancre
agement? atitis secondary to alcohol abuse. She has had no previous
(A) Electromyogram/nerve conduction velocity study primary medical care. She takes no medications and is cur
(B) Evaluate for intimate partner violence rently unemployed. She has been arrested one time for dri
ving while intoxicated, but no longer drives. She smokes
(C) High-dose ibuprofen one pack of cigarettes daily and does not currendy use illicit
(D) Oxycodone/acetaminopheii drugs; she did use marijuana when she was younger. She
tells you she only drinks when she can afford it, which may
be once a week or less, and usually has 6 or 7 drinks when
Item 118 she does. She has never had withdrawal symptoms.
A 78-year-old man is evaluated for routine follow up of Physical examination reveals a thin woman in no appar
hypertension. He reports that he is able to perform all activ ent distress; vital signs are normal. Sclera and skin are
ities of daily living and that he only drives in the neighbor anicteric. The liver edge is felt 4 cm below the right costal
hood and to nearby stores. He has used a cane while walk margin. There is mild epigastric tenderness but no ascites or
ing since he fell 3 mondis ago. His only medication is other stigmata of chronic liver disease.
chlordialidone.
Which of the following is die most appropriate man
On physical examination, vital signs are normal. His
corrected vision with glasses is 20/20 in both eyes. His gait agement for diis patient?
is somewhat slow and he needs some assistance getting on (A) Connect her drinking habits with the negative
to die examination table. The remainder of the physical exam consequences
ination is normal. He scores 25/30 on the Mini-Mental State-
(B) Identify that she is an alcoholic and needs to abstain
Examination. from drinking
Which of the following is the most appropriate man (C) Initiate therapy with disulfirani
agement of this patient? (D) Initiate therapy with naltrexone
(A) Advise the patient that he should no longer drive
(B) Advise the patient to continue to drive only locally Item 121 |y|
(C) Ask about any driving difficulties A 54-year-old man is evaluated after urgent transfer to die
(D) Report die patient to the state department of trans intensive care unit following an elective hemicolectomy. His
portation surgery was performed under general anesthesia and he was
extubated before leaving the operating room. After several
hours of observation in the postanesthesia care unit, he was
Item 119 reported by nursing staff to be drowsy but awake and
A 24-year-old man is evaluated for a 6-month history of responsive with stable vital signs, and was nansferrcd to a
episodic substernal chest pain. Episodes occur four to seven general suntical ward.

169
Self-Assessment Test

history of illicit drug use. He has had two lifetime female


A few hours
tress, later he was
was intubated, noted on
and placed to mechanical
be in respiratory dis
ventilation. sexual partners and is sexually active in a monogamous rela
cont. pjc js currently avVake and alert while being ventilated. His
tionship with a woman for the past 5 years. His father is 58
wife reports that he is a nonsmoker, has hypertension but years old and has hypertension; his mother is 57 years old
no known lung or heart problems, and had been doing well and has hyperlipidemia. Results of the physical examination,
prior to surgery except for chronic tiredness. including vital signs, are normal. Results of a fasting lipid
On physical examination, temperature is normal, blood panel 4 years ago were normal.
pressure is 150/90 mm Hg, pulse rate is 98/min, and res
Which of the following is the most appropriate screen
piration rate is 12/min. BMI is 39. Oxygen saturation by
pulse oximetry is 99% on 40% Fio,. Heart and lung exam ing test to obtain?
inations are normal. There is no lower extremity edema.
(A) Fasting lipid panel
Laboratory studies performed just before intubation
included an arterial blood gas measurement widi a pH of (B) Fasting plasma glucose
7.24 and a Pco, of 75 mm Hg (10 kPa). Serum bicarbo (C) HIV enzyme immunoassay antibody testing
nate level was 28 meq/L (28 mmol/L). CT angiogram (D) Thyroid-stimulating hormone level
shows no pulmonary embolism and normal lungs. Electro
cardiogram shows sinus tachycardia with no ST- or T-wave
changes. Item 124 ir
Which of the following is the most likely cause of his A physician unexpectedly encounters a colleague at a pub "
on a Friday night. They enjoy four or five drinks together
respiratory failure? while watching the first hour of a baseball game. Suddenly,
(A) Myocardial ischemia die colleague's pager goes off. Annoyed, she looks at it, and
(B) Obstructive sleep apnea says, slurring her words, "It's the hospital. I have to go
admit a patient." When he tries to stop her, she gets angry
(C) Premature extubation
and shakes him off.
(D) Sepsis
Which of the following is the most appropriate course of
action for this physician to take regarding his colleague?
Item 122
A 42-year-old woman is evaluated for a 10-day history of (A) Contact the hospital chief of staff and report his con
cerns immediately
right shoulder pain, located posteriorly and superiorly, that
becomes worse with overhead activities. She has no history (B) Report his concerns to state authorities on Monday
of trauma. She recently painted her basement ceiling. She (C) Take his colleague aside later and discuss his concerns
has no weakness or paresthesia of her right arm and has with her
never had diis problem before. She has been taking ibupro (D) No obligation to intervene-
fen as needed for the pain.
On physical examination, vital signs are normal. There
is no shoulder asymmetry and no tenderness to palpation of Item 125
bony structures or soft tissue structures. There is frill range A 55-year-old man is evaluated before scheduled endo
of motion (other than with internal rotation, which is lim
scopic sinus surgery. In addition to symptoms of chronic
ited by pain) and strength is 5/5 throughout die right arm,
sinusitis, he has chronic knee pain that limits his activity to
with sensation intact. She is able to slowly lower her below 4 metabolic equivalents (METs). He has no chest
extended arm from over her head to her side (negative
pain, dyspnea, or lower extremity edema. He has type 2 dia
drop-arm test). There is pain with abduction of die right betes mellitus, coronary artery disease, chronic kidney dis
arm between 60 and 120 degrees. The patient is asked to
ease, hypertension, hyperlipidemia, osteoarthritis, and
hold the arm extended anteriorly at 90 degrees with die chronic sinusitis. He had a myocardial infarction 5 years ago
forearm bent to 90 degrees (at 12 o'clock), as if holding a and was treated widi a single bare metal stent. Medications
shield. When the arm is internally rotated to cross in front are insulin glargine, metoprolol, aspirin, simvastatin, lisino
of the body, the patient feels pain in die shoulder (positive
pril, and acetaminophen.
Hawkins test). On physical examination, temperature is normal, blood
Which of the following is the most likely diagnosis? pressure is 142/84 mm Hg, and pulse rate is 56/min. The
electrocardiogram is consistent with a previous lateral
(A) Acromioclavicular joint degeneration myocardial infarction and is unchanged from 1 year ago.
(B) Adhesive capsulitis Serum creatinine level is 2.1 mg/dL (186 pmol/L).
(C) Rotator cuff impingement Which of the following is die most appropriate preop
(D) Rotator cuff tear erative management of diis patient?

(A) Discontinue metoprolol


Item 123 (B) Obtain dobutamine stress echocardiogram
A 30-year-old man is evaluated during a routine examina (C) Refer for cardiac catheterization
tion. He is asymptomatic. He is a nonsmoker and has no
(D) Proceed to surgery without further testing

170
Self-Assessment Test

Item 126 contexts. He is prone to emotional outbursts, and fre


A 32-year-old woman presents for emergency department quendy repeats back what is said to him many times over.
Currendy he spends most of his time in his room, and is
follow-up. She was seen 1 week ago for a facial laceration.
She tells you she cannot remember the incident. She had a unwilling to help with routine chores at home. However, he
is fascinated by trains, and when he is engaged will recite
humeral fracture 1 year ago, and has had bruising on her
numerous details about trains.
arms and legs on several visits. History is significant for
depression and recurrent urinary tract infection. Her only Which of the following is the most likely diagnosis?
current medication is citalopram. Upon questioning about
intimate partner violence, die patient admits that her hus (A) Autism spectrum disorder
band often beats her. (B) Obsessive-compulsive disorder
Physical examination is significant for normal vital (C) Schizophrenia
signs; a healing, sutured, 4-cm laceration across the left (D) Social anxiety disorder
zygomatic arch; and several 5- to 6-cm ecchymoses on her
upper extremities.
Item 129
Which of the following is the most appropriate next
A 25-year-old woman is evaluated in October before start
step in management?
ing a certified nursing assistant degree program. She was
(A) Advise the patient to leave her current living situation diagnosed with HIV infection 3 months ago. She reports
immediately having received "all my shots as a child," and specifically
(B) Ask her to bring her husband to her next recalls having chickenpox as a child. Before starting school
appointment she needs to provide proof of her immunization status. Her
(C) Assess her immediate safety and develop a safety plan only medication is an oral contraceptive. Findings on phys
ical examination are unremarkable.
(D) Report the husband to the police
(E) Request psychiatry consultation Which of the following is the most appropriate next
step in this patient's immunization management?

Item 127 (A) Administer a single measles, mumps, and rubella


booster now
A 50-year-old woman is evaluated for nonischemic car
(B) Administer live, attenuated, intranasal influenza vac
diomyopathy. Her exercise tolerance is not limited. She cine now
takes an ACE inhibitor daily. She took a (3-blockcr briefly
but discontinued because of fatigue. Results of the physical (C) Begin hepatitis B immunization series
examination are normal. (D) Certify her as immune to varicella given her clinical
The patient inquires whether she should receive drug history
"H". Drug H was studied in 2000 patients ages 40 to 80 (E) Obtain a CD4 cell count
years (mean age 63 years) with New York Heart Association
functional class III or IV heart failure. Patients were ran
domized to receive drug H or a placebo in addition to usual Item 130
medications. Eighty percent of patients in the trial also took A 38-year-old man is evaluated in die emergenq' depart
a p-blocker and 70% an ACE inhibitor. At the end of 3 ment for a 2-week history of nonpruritic, sharp, anterior
years, patients taking drug H had a significantly reduced chest pain. Each episode of pain lasts 3 to 10 hours. He
rate of a composite outcome of death or heart failure exac states that the pain is to the left of die sternum but at times
erbations. Approximately 5% of die patients taking drug H it radiates across the entire chest. It does not radiate to die
had serious adverse events, compared with 2% in the shoulders, arms, or back. The pain can be present at rest. It
placebo group. is worsened widi lateral movement of the trunk. He does
not notice any change in intensity widi walking or odier
Which of the following is the main reason why diis
activity. He has no other symptoms and no odier medical
patient should not be treated widi drug H?
problems. He does not use drugs and takes no medications.
(A) Her heart failure is too mild On physical examination, temperature is 37.0 C
(B) She is too young (98.6 F), blood pressure is 132/70 mm Hg, pulse rate is
90/min, and respiration rate is 14/min. BMI is 26. There
(C) She should be treated with a (3-blocker first is reproducible point tenderness along the sternum. The
(D) The drug's adverse event rate is too high remainder of the examination, including the cardiovascular
examination, is normal.
Item 128 Which of the following is the most likely diagnosis?
A 16-year-old patient is brought to the office by his mother
(A) Acute pericarditis
for an evaluation. His mother notes that he has been
(B) Aortic dissection
"strange" since a very young age. He did not start speaking
until age 3 years. Since diat time, he has been home- (C) Costochondritis
schooled but has avoided other children and adults in social (D) Unstable angina

171
Self-Assessment Test

Item 131 (B) Informed consent is only needed for invasive


A 19-year-old woman is evaluated for painful menses. She procedures
usually misses one or two days of school each month owing (C) Informed consent only applies when signing a con
to diese symptoms, which include cramps and nausea. sent form
Menarche occurred at age 12 years. Menses have been reg (D) Informed consent was not properly obtained
ular for the past 2 years, occurring every 29 days. The
patient is not sexually active, and her medical history is non-
contributory. She takes no medications. Item 134
Physical examination, including external pelvic exami A 47-year-old man is evaluated for follow-up of left-sided
nation, is normal. cervical radiculopathy. He presented 2 weeks ago with
Which of the following is the most appropriate man severe arm and hand pain that developed shortly after doing
yard work; no trauma was noted. He was treated with
agement option for this patient? NSAIDs and rest. Although his pain symptoms have
(A) Combined estrogen-progesterone contraceptive improved, he has developed progressive difficulty opening
(B) Depot medroxyprogesterone acetate jars because he has trouble holding onto the lid. On two
occasions over the past several days, he has dropped a cof
(C) Ibuprofen
fee cup without intending to.
(D) Measurement of follicle-stimulating hormone and On physical examination, vital signs are normal. There
luteinizing hormone levels is normal bulk and tone of the trapezius muscle on die left.
(E) Pelvic ultrasound Triceps strength is normal on die left, although die biceps
muscle group shows 4/5 strength relative to die right. There-
are also diminished biceps and brachioradialis reflexes on the
Item 132 left. The remainder of the examination is unremarkable.
A 42-year-old man is evaluated for a 4-month history of left An MRI of the cervical spine shows herniation of the
elbow pain. The pain radiates to his hand and is worse at C5-C6 disk with compression of the left C6 nerve root and
night, with flexion of the arm at the elbow, and with wrist increased signal in the area of compression.
flexion. The pain is accompanied by an intermittent tingling
Which of the following is the most appropriate next
sensation in the fourth and fifth fingers. He has no weakness
and has never had this problem before. step in management?
On physical examination, vital signs are normal. Pain is (A) Continued analgesics and rest
elicited in the left elbow with flexion of die arm at the (B) Epidural corticosteroid injection
elbow. There is decreased light touch sensation involving
(C) Physical therapy for strengthening
both palmar and dorsal surfaces of the fourth and fifth fin
(D) Surgical evaluation
gers to die level of the wrist. No tenderness to palpation of
any of the structures of the elbow is elicited.
Which of the following is the most likely diagnosis? Item 135
A 33-year-old woman is evaluated for chronic lower pelvic
(A) Lateral epicondylitis
pain. It has been persistent for the past year but has wors
(B) Medial epicondylitis ened in recent months. She describes it as a constant, aching
(C) Olecranon bursitis discomfort centered over her lower pelvis that persists dur
(D) Ulnar nerve entrapment ing her menstrual cycle and has prevented her from being
sexually active with her partner. She also reports a 4-month
history of urinary urgency and frequency. She has been
Item 133 empirically treated twice for urinary tract infections, but her
A 48-year-old man is evaluated for watery bowel move urinary symptoms improve for only a few days and then
ments. The patient is having 8 to 12 watery bowel move recur. She has no history of pelvic surgeries or pelvic infec
tions and has never been pregnant. She has no associated
ments daily. Several days before die diarrhea began, rhe
patient went to an urgent care clinic with a sinus headache. constipation, diarrhea, abdominal distention, or flank pain.
She currently takes ibuprofen as needed for pain.
He was diagnosed with bacterial sinusitis and given a 7-day
On physical examination, vital signs are normal. BMI
course of antibiotics. He was asked to return to die urgent
is 24. There is mild tenderness to palpation over rhe pelvic
care center if he experienced any problems or did not
floor muscles with significant tenderness over the anterior
improve. He says there was no discussion of adverse effects
or alternatives to antibiotic therapy. Stool assay is positive for vaginal wall. External genitalia are normal in appearance;
there is no tenderness to palpation over the vulva. There is
Clostridium difficile toxin. no cervical motion tenderness, adnexal tenderness, or dis
Which of the following can be concluded about comfort with palpation of the uterus.
informed consent in diis case? Laboratory studies show normal electrolytes, kidney
function, and a complete blood count. Erythrocyte sedi
(A) Acceptance of the antibiotics fulfilled requirements mentation rate is 4 mm/h. Urinalysis is without erythrocytes
for informed consent or leukocytes and is negative for nitrite and leukocyte

172
Self-Assessment Test

esterase. Urine culture is negative. Tests for chlamydial infec out of bed in the morning. The patient's wife died of can
tion and gonorrhea are negative. cer 9 months ago. He says that he has been seeing her face
Transvaginal/transabdominal ultrasonography is neg at night when he closes his eyes and frequently awakes at
ative for endometrial or ovarian masses and no abnormali night thinking that she is next to him in bed. Medical his
ties arc noted. tory is significant for hypertension and hyperlipidemia.
Medications are hydrochlorothiazide, atorvastatin, and
Which of die following is the most likely diagnosis?
diphenhydramine at bedtime as needed for sleep. Results of
(A) Endometriosis the physical examination are normal.
(B) Interstitial cystitis Chemistry panel and complete blood count are nor
mal. Electrocardiogram reveals normal sinus rhythm with
(C) Irritable bowel syndrome left ventricular hypertrophy without ischemic changes.
(D) Pelvic adhesions Chest radiograph is normal. Exercise treadmill test is nega
tive for cardiac ischemia.

Item 136 Which of die following is the most likely diagnosis?


A 66-vear-old man with diabetic neuropath)' is evaluated for (A) Anticholinergic drug side effect
increasing pain in his lower extremities. The pain is 7/10 in (B) Complicated grief
intensity, constant, and keeps him awake at night. He has
been treated with trials of NSAIDs, amitriptyline, prega (C) Generalized anxiety disorder
balin, gabapentin. and duloxetine, both individually and in (D) Major depression with psychotic features
various combinations, without significant control of his
symptoms. He presented to an outside emergency depart Item 138
ment 3 weeks ago and was prescribed oxycodone-aceta-
minophen 5 mg/325 mg. He has been taking 2 tablets A 34-year-old woman is evaluated for a 6-month history of
every 6 hours as recommended and has experienced good allergies that have not responded to treatment. She nor
relief of his pain, although his symptoms worsen with late mally has hay fever in the spring, but it typically subsides
or missed doses. Medical history is also significant for hyper after 1 to 2 months and the symptoms can be controlled
tension, hyperlipidemia, and ischemic cardiomyopathy. He with antihistamines and decongestants. Now she reports
has no personal or family history of drug or alcohol abuse. intractable nasal congestion and rhinorrhea present for
Current medications are gabapentin, amitriptyline, insulin months. She does not have sinus pain or dental pain. She
glargine, insulin aspart, lisinopril, aspirin, simvastatin, lives in a 10-year old house with no known mold problems
carvedilol, and oxycodonc-acetaminophcn. and has no pets. She does not think her symptoms are worse
On physical examination, temperature is 36.7 C or better when she leaves the house or travels. She notes no
(98.2 F), blood pressure is 122/76 mm Hg, pulse rate is sneezing or itchy eyes. Current medications are oxymeta-
64/min, and respiration rate is 12/min. BMI is 28. He has zoline nasal spray, pseudoephedrine, cetirizine, and diphen
decreased sensation to light touch and pinprick in a stock hydramine at bedtime. She has no other medical problems.
ing-glove distribution, and reflexes are 1+ and symmetric. On physical examination, temperature is 36.8 C
Romberg test is positive. There is no hepatomegaly, scleral (98.2 F), blood pressure is 124/72 mm Hg, pulse rate is
icterus, or jaundice. 96/niin, and respiration rate is 16/min. BMI is 23. Nasal
Laboratory studies are significant for normal elec mucosa is hyperemic and slightly edematous with clear nasal
trolytes, liver chemistry studies, vitamin B__ level, and com drainage. The pharynx is normal without tonsillar enlarge
plete blood count. The scrum creatinine level is 1.3 mg/dL ment or cobblcstoiiing.
(115 pmol/L). Which of the following is the most likely diagnosis?
A previous resting electrocardiogram showed first-
degree atrioventricular block and small Q waves in an (A) Chronic rhinosinusitis
anterolateral distribution without active ischemia. (B) Chronic vasomotor rhinitis
Which of the following is the most appropriate treat (C) Granulomatosis with polyangiitis (Wegener
ment for this patient's neuropathic pain? granulomatosis)
(D) Rhinitis medicamentosa
(A) Continue oxycodonc-acetaminophcn at the current
dose
(B) Discontinue oxycodone-acctaminophen and start Item 139
tramadol
A 34-year-old woman is evaluated for significant breast pain.
(C) Transition to methadone She says her breasts are generally very "sensitive" but she
(D) Transition to sustained-release morphine develops more pronounced discomfort shortly before she has
her menstrual period, when both breasts seem to ache and
throb. She has not noticed any associated skin discoloration,
Item 137 nipple discharge, or breast masses. She is obese and has
An 81-year-old man is evaluated for a 3-week history of gastroesophageal reflux disease. She does not use tobacco or
shortness of breath, chest pain, palpitations, difficulty sleep alcohol. She otherwise feels well and has increased her exer
ing, early morning awakening, and lack of interest in getting cise level to help her lose weight. She has no shortness of

173
Self-Assessment Test

breath, nausea, vomiting, difficulty swallowing, or recent Additional laboratory results reveal normal kidney and
rashes. Her menses are regular and occur every 28 days. Her liver function.
mother was diagnosed widi breast cancer at age 53 years and
her sister had a benign breast biopsy. Her current medica In addition to recommending diet and exercise ther
tions are omeprazole and a daily multivitamin. apy, which of die following is the most appropriate
On physical examination, vital signs are normal. BMI management?
is 32. Her breasts are symmetric in shape and appearance.
There are no palpable masses in either breast, and there is (A) Add gemfibrozil
no supraclavicular or axillary lymphadenopathy. There is no (B) Increase simvastatin to 80 mg/d
nipple discharge or skin discoloration or dimpling. There is (C) Measure hemoglobin Alc level
diffuse tenderness to palpation over both breasts, most
(D) Measure thyroid-stimulating hormone level
prominent in the upper outer quadrants. Cardiovascular,
pulmonary, and abdominal examinations are normal. There
is no tenderness to palpation over the chest wall.
Item 142
Which of the following is the most appropriate man A 54-year-old woman is evaluated before an elective chole
agement? cystectomy. Medical history is significant for atrial fibrilla
(A) Danazol tion, type 2 diabetes mellitus, chronic heart failure, hyper
(B) Diagnostic mammography tension, and a transient ischemic attack 2 months ago.
(C) Support bra Medications are warfarin, insulin glargine, insulin lispro,
(D) Tamoxifen metoprolol, lisinopril, furosemide, and simvastatin.
On physical examination, temperature is normal, blood
pressure is 142/88 mm Hg, and pulse rate is 88/min and
Item 140 irregularly irregular. The remainder of the physical exami
A 26-year-old man is evaluated for a 2-mondi history of nation is normal. INR is 2.5.
depressed mood, lack of energy, and increased sleep. He has Which of the following is the most appropriate treat
had less interest in his usual hobby of woodworking and has
ment?
found it more difficult to perform well at his job at a law
firm. In the past, he has had periods of high energy requir
(A) Administer half the usual dose of warfarin for 5 days
ing little sleep without getting tired. During these periods before surgery
he recalls going on spending sprees and having many sexual
partners. He does not have any suicidal ideation. His (B) Continue warfarin
mother has a history of alcohol abuse. The physical exami (C) Discontinue warfarin 5 days before surgery
nation, including the mental status examination, is normal. (D) Discontinue warfarin 5 days before surgery and
Which of die following is the most appropriate treat administer enoxaparin until the morning of surgery
ment?

(A) Duloxetine Item 143


(B) Lamotrigine A 22-year-old woman is evaluated during a routine exami
(C) Lorazepam nation. She has a history of anorexia nervosa but has been
(D) Sertraline in remission for die past 2 years. She has rccendy been
under significant stress related to her parents' divorce, and
has started resuicting her caloric intake because she is wor
Item 141 ried about gaining weight. She has not had a menstrual
A 55-year-old woman is evaluated during a follow-up
period for 4 months (previously she had regular menses)
appointment. She has hypertension and hyperlipidemia. She and she has lost 13.6 kg (30 lb) in die last 12 weeks. Med
does not use alcohol. Review of systems is notable for ical history is significant for depression and osteopenia. Cur
fatigue and occasional constipation. She is menopausal. Her rent medications are calcium and vitamin D.
family history is noncontributory. Her medications are sim On physical examination, temperature is 36.4 C
vastatin (40 mg/d), aspirin, and lisinopril.
On physical examination, she is afebrile, blood pressure (97.6 F), blood pressure is 100/60 mm Hg, pulse rate is
is 140/82 mm Hg, pulse rate is 66/min, and respiration 60/min, and respiration rate is 12/min. There is no
rate is 12/min. BMI is 25. She has mildly dry skin. There ordiostasis. BMI is 16. There is fine, soft hair covering the
is no evidence of xanthomas and no hepatomegaly. arms, chest, and abdomen. The heart rate is slow with a reg
ular rhydim. The abdomen is scaphoid. The skin is pale and
Laboratory studies:
Total cholesterol 284 mg/dL (7.36 mmol/L) diere is no edema. Pulmonary, abdominal, and muscu
LDL cholesterol 231 mg/dL (5.98 mmol/L) loskeletal examinations are normal. Blood chemistry stud
HDL cholesterol 55 mg/dL (1.42 mmol/L) ies are normal except for a sodium level of 132 meq/L (132
T r i g l y c e r i d e s 11 3 m g / d L ( 1 . 2 8 m m o l / L ) mmol/L) and a phosphorus level of 2.5 mg/dL (0.81
Glucose (fasting) 100 mg/dL (5.5 mmol/L) mmol/L). A pregnancy test is negative.

174
Self-Assessment Test

In addition to referring the patient for nutritional (C) Hepatitis B virus infection
rehabilitation, which of the following is the most (D) Osteoporosis
appropriate treatment?
(A) Amitriptyline Item 146
(B) Bupropion
A 38-year-old woman is evaluated for a 2-year history of irri
(C) Cognitive-behavioral therapy
tability and frequent headaches, accompanied by nausea and
(D) Megestrol acetate sweating. She is a housecleaner and has had increasing diffi
(E) Oral contraceptive pills culty concentrating at work over die past year, and it takes her
much longer to clean houses lately. She has a difficult time get
ting to sleep and frequendy arises after 2 to 3 hours of fitful
Item 144 sleep in bed. Her mood is good. She worries frequendy about
A 75-year-old man is evaluated for low back pain. The pain her ability to pay her bills and what she will do for retirement.
began several mondis ago and is getting worse. Its intensity She has cut back on activities widi friends and does not like to
is now 6 or 7 (on a scale ofl-10) most days. It worsens widi go out in social situations anymore. She has asthma, and her
activity and improves with sitting. He has been unable to only current medication is albuterol as needed.
walk or exercise because of die pain and has gained 9.0 kg On physical examination, she is afebrile, blood pressure
(20 lb) in the past year. He has not experienced any bowel is 130/72 mm Hg, pulse rate is 98/min, and respiration rate
or bladder incontinence or lower extremity numbness or is 14/min. BMI is 22. Serum thyroid-stimulating hormone
tingling, and has not had fevers or chills. Medical history is level, complete blood count, and urinalysis are normal.
significant for hypertension, hyperlipidemia, and peptic Which of the following is the most likely diagnosis?
ulcer disease. He does not use tobacco or alcohol. His cur
rent medications are hydrochlorothiazide, lisinopril, sim (A) Attention-deficit/hyperactivity disorder
vastatin, omeprazole, and aspirin. He has been taking addi (B) Bipolar disorder
tional aspirin, which has been providing modest pain relief.
On physical examination, vital signs are normal. BMI is (C) Generalized anxiety disorder
34. There is mild tenderness to palpation over the lower lum (D) Major depressive disorder
bar paravertebral muscles. Patellar and ankle jerk reflexes are
1+ and symmetric. Sensation to light touch and pinprick is
Item 147
symmetric, and strengdi is 5/5 in die lower extremities.
Complete blood count and metabolic panel are normal. A 56-year-old woman is evaluated during a routine exami
A radiograph of the lumbar spine shows degeneration nation. On review of systems, she reports a 2-year history of
of the L4 and L5 vertebrae, osteophyte formation, and facet decreased interest in sexual intercourse. Her lack of interest
arthropathy widi narrowing of the spinal canal. is gradually worsening and starting to cause problems in her
Which of the following is the most appropriate treat marriage. She has no history of sexual trauma, sexually
transmitted infection, or pelvic surgery. Her last menstrual
ment for this patient's symptoms?
period was 2 years ago.
(A) Acetaminophen On physical examination, vital signs are normal. Gyne
(B) Amitriptyline cologic examination shows pink, well-lubricated vaginal
walls; there is no cervical motion or adnexal tenderness and
(C) Cyclobenzaprine no discomfort with speculum insertion.
(D) Ibuprofen
Which of the following is the most appropriate treat
ment of this patient?
Item 145
(A) Sex therapy
A 60-year-old woman is evaluated during a routine exami
nation. She has hyperlipidemia. She has a 5-pack-year smok (B) Sildenafil
ing history but is not actively using tobacco and has no his (C) Systemic estrogen and progesterone dierapy
tory of illicit drug use. She is married and in a monogamous (D) Systemic testosterone therapy
relationship widi her husband of 25 years. She has no fam
ily history of breast, colon, or cervical cancer. Her only
medication is simvastatin. Item 148
On physical examination, she is afebrile, blood pressure- A 70-year-old woman is evaluated for a 3-mondi history of
is 118/76 mm Hg, and pulse rate is 74/min. BMI is 25. vision problems. She reports that objects may appear blurry
Pap smear and mammography, both performed 11 mondis or distorted, particularly in the central field. She has diffi
ago, were within normal limits. culty reading and recognizing faces. She has no eye pain or
Which of the following conditions should also be recent eye trauma. She is a smoker. Her only medication is
screened for in this patient? tiotropium.
On physical examination, vital signs are normal. Fun-
(A) Abdominal aortic aneurysm duscopic findings are shown (sec next page). The remain
(B) Depression der of the eye examination is normal.

175
Self-Assessment Test

On physical examination, temperature is normal, blood


pressure is 130/80 mm Hg, pulse rate is 80/niin, and respi
ration rate is 14/min. BMI is 48. Waist circumference is
121.9 cm (48 in). There is no thyromegaly. Heart sounds are
normal with no murmur. There is no lower extremity edema.
Results of complete blood count, thyroid studies, and
urinalysis are unremarkable.
Which of the following is the most appropriate man
agement of this patient?
(A) Bariatric surgery evaluation
(B) Prescribe phentermine
(C) Reduce caloric intake to below 800 kcal/d
(D) Refer to an exercise program

Item 151
ITEM 148 A 66-year-old man is evaluated during a routine examina
tion. He is asymptomatic and walks for 2 miles on the tread
Which of the following is the most likely diagnosis? mill three times a week. He has hypertension. He drinks
three or four beers three times per week and has done so for
(A) Age-related macular degeneration
the past 30 years. He is a former smoker, smoking one pack
(B) Cataracts
per day for 2 years between the ages of 20 and 25 years.
(C) Primary open angle glaucoma Current medications are hydrochlorothiazide and aspirin.
(D) Retinal detachment On physical examination, he is afebrile, blood pressure
is 124/76 mm Hg, and pulse rate is 72/min. BMI is 23.
The examination is otherwise unremarkable. A fasting lipid
Item 149 profile last year was within normal limits. His most recent
A 70-year-old man is evaluated preoperatively before elective- colonoscopy was performed 5 years ago and was negative
total hip arthroplasty. He is able to ambulate on a level sur for any polyps.
face slowly but is unable to carry laundry or groceries up
Which of the following is the most appropriate screen
stairs because of his hip pain. He has had no recent chest pain
or pressure. He has no orthopnea. Odier than his hip pain, ing test?
he feels well. He has hypertension and hyperlipidemia. Cur (A) Abdominal ultrasonograph)'
rent medications arc metoprolol, losartan, and simvastatin. (B) Chest radiography
On physical examination, temperature is 36.8 C
(C) Coronary artery calcium score determination
(98.3 F), blood pressure is 132/78 mm Hg, pulse rate is
64/min, and respiration rate is 14/min. Results of the (D) Fasting lipid profile
remainder of the examination are normal. Serum creatinine (E) No additional testing
level is 1.3 mg/dL (115 pmol/L). An electrocardiogram
shows sinus rhythm with no ST- or T-wave abnormalities.
Item 152
Which of the following is the most appropriate diag
A 44-ycar-old man is evaluated for chronic back pain. Six
nostic test to perform next?
months ago he underwent decompressive spinal laminectomy
(A) Cardiac catheterization for treaunent of refractor)- pain secondary to spinal stenosis.
He was discharged on oxycodone-acetaminophen, 5 mg/325
(B) CT coronarv' angiography
mg, and despite taking 2 tablets every 6 hours, he continues
(C) Dobutamine stress echocardiography to report severe discomfort. He has been taking ibuprofen
(D) No further testing needed before surgery and cyciobenzaprinc to help with the pain and has been using
hot compresses intermittently. His surgeon has been satisfied
with his operative treatment and has recommended that he
Item 150 start a physical therapv' program, but die patient reports diat
A 42-year-old-man is evaluated for obesity. His weight has he is in too much pain to exercise. Medical history is signifi
gradually increased over the past two decades and is cur cant for mild depression and tobacco use. His father abused
rently 168.2 kg (370 lb). Five years ago, he was diagnosed alcohol and his mother has COPD secondary to smoking.
with type 2 diabetes niellitus, hypertension, and hyperlipid On physical examination, vital signs are normal. BMI
emia. Over the past 6 months, he has unsuccessfully tried is 32. There is mild tenderness to palpation over the lum
diet and exercise therapy for his obesity. He tried over-the- bar paravertebral muscles. A complete neurologic examina
counter orlistat but could not tolerate the gastrointestinal tion of die lower extremities and a straight leg raise test can
side effects. Medications are metformin, lisinopril, and sim not be performed because of patient discomfort, although
vastatin. His total weight loss goal is 45.4 kg (100 lb). there are no abnormalities noted on limited testing.

176
Self-Assessment Test

In addition to starting die patient on a good bowel Which of die following is die most likely diagnosis?
regimen, which of the following is die most appropri
ate management? (A) Anorexia nervosa, binge-eating/purging subtype
(B) Binge-eating disorder
(A) Amitriptyline
(C) Bulimia nervosa, purging subtype
(B) Change analgesic to extended-release morphine
(D) Night-eating syndrome-
(C) Evaluate for opioid dependency
(D) MRI of the lumbar spine-
Item 155
A 48-year-old man is evaluated for pain located on his lat
Item 153 eral thigh. He describes the pain as a burning sensation that
A 42-year-old man is evaluated for difficulty sleeping the has been present for 3 weeks. He has never had this pain
past several months. He reports trouble both falling asleep before and has no associated leg weakness or back pain.
and staying asleep. He has not tried any over-the-counter On physical examination, vital signs are normal. BMI
medications. He drinks two or three beers on the weekends is 34. Dysesthesia is present in the anterolateral thigh. There-
only and this has not changed; he also drinks two cups of is no tenderness to palpation of the lateral femoral epi
coffee in the mornings. His wife, who is present, has not
condyle. Knee and hip examinations are normal. A straight
heard any snoring, gasping, or other breathing problems at
leg raising test is negative bilaterally and strength is 5/5 in
night. He reports no leg symptoms. They have recently both extremities.
moved to a new apartment; he reports that the bedroom
may be hotter than the previous one, although his wife Which of die following is the most likely diagnosis?
reports feeling comfortable.
Results of the physical examination are unremarkable. (A) Greater trochanteric bursitis
Vital signs are normal, BMI is 26, and mood and mental (B) Iliotibial band syndrome
status are normal. (C) L5 radiculopathy
Which of the following is die best initial management (D) Melalgia paresthetica
for this patient?

(A) Advise alcohol abstinence Item 156 m


(B) Benzodiazepine A 78-ycar-old man is evaluated in the emergency depart
(C) Counseling regarding sleep hygiene ment after a witnessed episode of syncope. The patient
(D) Over-the-counter antihistamine reports diat, while eating dinner, he experienced a pound
ing in his chest and then fell to die floor. His wife estimates
he was unconscious for approximately 30 seconds, had no
Item 154 head trauma, and was oriented and alert upon regaining
A 28-year-old woman is evaluated for a 3-month history of consciousness. He has not experienced any similar episodes
in the past, although he has felt the pounding previously.
fatigue and muscle cramps. She states that she is eating well,
drinking plenty of fluids, and exercising regularly, but the Medical history is significant for hypertension, COPD,
fatigue is starting to interfere with her ability to complete osteoarthritis, and benign prostatic hyperplasia. Medica
her daily 3-mile run. She has a previous history of anorexia tions are chlordialidone, lisinopril, celecoxib, ipratropium-
nervosa diagnosed at age 16 years requiring two inpatient albuterol inhaler, and tamsulosin. He currently feels well
hospitalizations. She has had a normal weight and men except for pain in his right thigh where he fell.
strual cycle for the last 4 years. She currently takes daily cal On physical examination, temperature is normal.
cium and vitamin D supplements. Blood pressure is 138/88 mm Hg and pulse rate is
On physical examination, temperature is 36.6 C 82/min, without orthostatic changes. Respiration rate is
(98.0 F), blood pressure is 106/64 mm Hg, pulse rate is 16/min. Oxygen saturation on ambient air is normal.
66/min, and respiration rate is 12/min. BMI is 22. She has Carotid upstrokes are +2 without bruits, and diere is no
poor dentition with multiple dental caries. The remainder of jugular venous distention. Cardiac examination is normal,
the examination is normal. Pregnancy test is negative. widi die exception of occasional extra beats. The remainder
Laboratory studies: of the examination, including neurologic examination, is
Complete blood count Normal normal. 12-Lead electrocardiogram shows a few premature
Electrolytes ventricular contractions widiout evidence of ischemia.
Sodium 132 meq/L (132 mmol/L)
Potassium 3.2 meq/L (3.2 mmol/L) Which of the following is the most appropriate next
Chloride 95 meq/L (95 mmol/L) step in this patient's management?
Bicarbonate 31 meq/L (31 mmol/L)
(A) Carotid Doppler ultrasonograph)'
Blood urea nitrogen 6 mg/dL (2.1 mmol/L)
Creatinine 0.8 mg/dL (70.7 pmol/L) (B) Echocardiography
Thyroid-stimulating 2.5 pU/mL (2.5 mU/L) (C) Inpatient cardiac monitoring
hormone (D) Noncontrast CT of head

177
Self-Assessment Test

Item 157 Laboratory studies:


A 26-year-old woman is evaluated in the emergency depart 12 Weeks Ago Current
ment for an 8-day history of sore throat, fever, and neck Alanine aminotransferase 28 units/L 76 units/L
pain. She has severe pain on the left side of her neck with Aspartate 21 units/L 63 units/L
swallowing. She has had fevers for the past week with rigors aminotransferase
To t a l b i l i r u b i n 0 . 8 m g / d L 1 m g / d L
starting today. Over the past 3 to 4 days she has had increas
ing cough. She was previously healthy and takes no med (13.6 pmol/L) (17.1 pmol/L)
ications. Which of die following is the most appropriate man
On physical examination, temperature is 39.1 C
agement?
(102.3 F), blood pressure is 108/68 mm Hg, pulse rate is
116/min, and respiration rate is 20/min. BMI is 19. She is (A) Discontinue simvastatin
toxic-appearing. The neck is tender to palpation along the (B) Measure serum antibodies to hepatitis B and C
left side without lymphadenopathy. The pharynx is erythe
(C) Order liver ultrasonography
matous with tonsillar enlargement and no exudates. The
chest is clear to auscultation. Other than tachycardia, the (D) No change in management
cardiac examination is normal.
Chest radiograph is shown. Leukocyte count is
Item 159 If
18,400/uL (18.4 x 109/L) with 17% band forms. Serum
creatinine level is 0.8 mg/dL (70.7 umol/L). A 30-year-old man was admitted to the hospital 2 days ago with
an acute exacerbation of asthma. He was placed on a regimen
of corticosteroids and inhaled (3-agonists. He has responded
well and is read)' for discharge today with plans for a short
course of oral corticosteroids as well as inhaled corticosteroids
and (3-agonists. He reports receiving "all his shots" while grow
ing up. He does not smoke cigarettes. He is in a monogamous
relationship with his wife and reports no illicit drug use.
Which of the following vaccines should diis patient
receive?

(A) Hepatitis B
(B) Human papillomavirus
(C) Meningococcal
(D) Pneumococcal

Item 160 n
A 66-year-old woman is evaluated in the hospital 12 hours "*
after a total knee arthroplasty. She is experiencing anorexia
and some nausea and is not eating. She has a history of type
2 diabetes mellitus treated with glimepiride.
On physical examination, vital signs are normal. BMI
Which of die following tests is most likely to establish is 35. Her preoperative hemoglobin Alc concentration was
the diagnosis?
6.9%, and plasma glucose level 6 hours postoperatively is
(A) CT of the chest with contrast 250 mg/dL (13.9 mmol/L).
(B) CT of the neck with contrast Which of die following is the most appropriate treat
(C) Radiography of the pharyngeal soft tissues ment of her diabetes?
(D) Transthoracic echocardiography
(A) Glimepiride
(B) Long- and short-acting insulin
Item 158 (C) Sliding scale insulin
A 56-year-old man presents for evaluation of elevated liver (D) No treatment at this time-
chemistry test results that were obtained during an applica
tion for life insurance. At an office visit 12 weeks ago, he was
started on simvastatin for dyslipidemia. He has not experi Item 161
enced any side effects with this medication and specifically A 19-year-old man is evaluated for a 2-day history of sore
does not have nausea, vomiting, or abdominal pain. throat, cough, fever, and chills. On physical examination, tem
On physical examination today, blood pressure is perature is 38.9 C (102.0 F), blood pressure is 122/82 mm
140/80 mm Hg; vital signs arc otherwise normal. BMI is Hg, pulse rate is 88/min, and respiration rate is 14/min. The
29. There is no scleral icterus, hepatomegaly, or abdominal pharynx is erythematous with tonsillar enlargement and exu
tenderness. dates bilaterally. There is no cervical lymphadenopathy.

178
Self-Assessment Test

Which of die following is the most appropriate man die left tympanic membrane appears normal. There is no
agement? preauricular or cervical lymphadenopathy.
(A) Obtain throat culture and start penicillin therapv' Which of the following is die most likely diagnosis?
(B) Perform rapid antigen detection testing
(A) Acute otitis externa
(C) Start penicillin therapy
(B) Delayed-type hypersensitivity reaction to ear plugs
(D) No further testing or treatment indicated
(C) Malignant otitis externa
(D) Otitis media
Item 162
A 42-year-old man is seen for follow-up of hepatitis C.
Item 164
Three years ago, serologic tests were positive for hepatitis A
virus and negative for hepatitis B virus. He received a single- A 38-year-old woman is evaluated during a routine exam
dose of hepatitis B vaccine but was lost to follow-up. He ination. She has recently divorced and is interested in some
feels well today vvidiout specific symptoms. Findings on form of hormonal contraception, as her ex-husband had a
physical examination are unremarkable. vasectomy after their last child, and she does not want to
use condoms. She has no history of deep venous throm
Which of the following is the most appropriate man bosis, hypertension, or heart disease. She drinks one glass
agement of this patient's hepatitis B vaccination? of wine 4 or 5 nights per week, and smokes a pack of cig
(A) Complete the hepatitis B vaccine series arettes daily. Family history is significant for stroke in her
mother at age 72 years. All previous Pap smears have been
(B) Measure hepatitis B surface antibody
negative.
(C) Restart the hepatitis B vaccine scries
Physical examination, including pelvic examination, is
(D) No further vaccination or serologic testing for normal. A Pap smear is obtained.
hepatitis B is needed
Which of die following is die most appropriate hor
monal contraceptive option for this patient?
Item 163
A 19-year-old woman is evaluated for a 1-week history of (A) Estrogen patch
left ear canal pruritus, redness, and pain. She swims 1 mile (B) Esu-ogcn-progesteronc combination
each day and has recendy started wearing plastic car plugs (C) Progesterone contraceptive
to keep water out of her ears while swimming. (D) No hormonal-based method
On physical examination, she is afebrile, blood pressure
is 98/66 mm Hg, pulse rate is 62/min, and respiration rate-
is 16/min. She appears healthy and in no distress. There is Item 165
pain with tugging on the pinna and compression or move A 70-year-old man is evaluated for a 6-month history of low
ment of die nagus. The left ear canal is shown. With irrigation,
energy and decreased libido. He is not in a depressed mood
and is still interested in daily activities. He has glaucoma and
hypertension. Over the past year his vision has decreased
and his ophthalmologist has adjusted his medications
repeatedly. His current medications are timolol drops,
latanoprost drops (a prostaglandin analogue), dorzolamide
drops (a topical carbonic anhydrase inhibitor), lisinopril,
and amlodipine.
On physical examination, temperature is 37.6 C
(99.7 F), blood pressure is 138/84 mm Hg, pulse rate is
48/min and regular, and respiration rate is 12/min. BMI is
28. Other than bradycardia, the results of die physical exam
ination are normal. An electrocardiogram shows only sinus
bradycardia.
Which of this patient's medications should be discon
tinued?

(A) Amlodipine
(B) Dorzolamide
(C) Latanoprost
(D) Lisinopril
(E) Timolol

179
Self-Assessment Test

Item 166 On physical examination, there is tenderness to pal


A 65-year-old man is evaluated for a 6-month history of pation of the plantar medial calcaneal tuberosity and 1 to
2 cm distally along the plantar fascia. Passive dorsiflexion
inability to achieve a successful erection. He is otherwise
of the toes increases the patient's pain. Pes planus is pre
asymptomatic. He has coronary artery disease and hyper sent. There is no ecchymosis, tenderness, or swelling over
lipidemia. A bare metal stent was placed 5 years ago to his the plantar fascia and no tenderness of the calcaneus with
mid-left anterior descending coronarv' artery after he expe
medial or lateral pressure. The calcaneal tuberosity is nei
rienced exertional chest pain. Currendy, he exercises in the
ther prominent nor tender. Sensation is intact on the plan
form of brisk walking 3 to 4 times per week. He does not
tar surface of the foot.
smoke. His current medications are aspirin, metoprolol, and
simvastatin. He has no family history of early coronary Which of the following is the most appropriate treat
artery disease. ment?
On physical examination, he is afebrile, blood pressure
(A) Arch supports
is 114/80 mm Hg, pulse rate is 84/min, and respiration
rate is 16/min. BMI is 29. Results of the cardiac exami (B) Corticosteroid injection
nation are normal. He has no gynecomastia, the testes arc (C) Extracorporeal shock wave therapv'
normal in size, and sensation is intact in both lower (D) Plantar fascia release surgery
extremities. The dorsalis pedis and posterior tibialis pulses
are palpable bilaterally.
An electrocardiogram is normal. Laboratory investiga Item 168
tion reveals a serum thyroid-stimulating hormone level of A 60-year-old woman is admitted to the hospital for
2.75 pU/mL (2.75 mU/L) and an 8 AM total testosterone abdominal pain. A subsequent evaluation has revealed a
level of 425 ng/dL (15 nmol/L). large cecal mass, and hemicolectomy is planned. She is able
to walk her dog at a brisk pace, actively garden, and shovel
Which of the following is the most appropriate man
light snow. She has had no chest pain or dyspnea on exer
agement for this patient? tion. She has type 2 diabetes mellitus, hypertension, hyper
(A) Begin sildenafil lipidemia, and chronic kidney disease. Current medications
are insulin, lisinopril, and simvastatin.
(B) Begin testosterone replacement therapy On physical examination, temperature is normal, blood
(C) Perform exercise stress testing pressure is 132/88 mm Hg, and pulse rate is 78/min.
(D) Stop metoprolol Heart rate and rhythm are regular with no murmurs or gal
lops. Laboratory studies show a serum creatinine level of
2.4 mg/dL (212 pmol/L). Electrocardiogram shows nor
Item 167 mal sinus rhythm without ST- or T-wave abnormalities.
A 22-year-old woman is evaluated during a follow-up Which of the following is the most appropriate diag
appointment for plantar fasciitis. She has been doing nostic test to perform prior to surgery?
appropriate stretching exercises for the past 8 weeks and
has been using acetaminophen, 1000 mg every 8 hours, (A) Adenosine thallium stress test
with some improvement in pain. The pain is still present, (B) Cardiac catheterization
however, and interferes with her job. She has never had (C) Exercise stress test
pain like this before. (D) Proceed to surgery without further testing

180
Answers and Critiques

Item 1 Answer: A Bibliography


Educational Objective: Identify appropriate Berg AO, Baird MA, Botkin JR, et al. National Institute of Health
State-of-the-Scienee Conference Statement: Family History and
genetic counseling strategies. Improving Health. Ann Intern Med. 2009; 15l'(12):872-877.
[PMID: 19884615]
The most appropriate screening step for genetic diseases in this
patient is to inquire about any diseases diat "run in the family"
and, specifically, to inquire about family history of the more com Item 2 Answer: D
mon and important inherited diseases, including breast, ovarian,
Educational Objective: Treat carpal tunnel
prostate, and colon cancer, as well as early cardiovascular disease.
A detailed family history should follow for diose a ondirk >ns iden syndrome.
tified through diis preliminary questioning. Patient-reported This patient has carpal tunnel syndrome, and die most
family histories for fust-degree relatives have been shown to be appropriate initial treatment, in addition to the avoidance
accurate and valuable for breast and colon cancer, but a negative of repetitive wrist motions, is wrist splinting. Wrist splint
family history for ovarian and endometrial cancer is less accurate. ing appears to be most effective when done in the neutral
Aldiough few diseases follow strict Mendelian genetics that allow position compared with 20 degrees of extension. In one
for a relatively certain prediction of disease, knowledge regard prospective study, full-time splinting was superior to noc
ing die frequency of disease occurrence in a given family cohort turnal splinting at 6 weeks in terms of nerve latencies,
is helpful in assessing individual risk for developing specific dis although there was significant cross-over of patients
orders widi a genetic predisposition. between groups in this study. Nocturnal splinting has the
Genetic counseling with die option for testing should be
advantage of being more convenient to patients in com
offered when: (1) the patient has a personal or family history
parison with full-time splinting.
suggestive of a genetic susceptibility condition; (2) the genetic Local corticosteroid injection has been shown to pro
test can bc adequately interpreted; and (3) the test results will vide short-term (up to 3 months) pain relief, although the
aid in diagnosis or influence die medical or surgical manage effect does not appear to be durable. Contraindications to
ment of the patient or family at hereditary risk. It is premature local corticosteroid injections include thenar weakness
to refer for genetic counseling vvidiout first determining if and atrophy, profound sensory loss, and acute carpal tun
there is concern for a genetic disorder. nel syndrome.
The process of taking a family history typically
Owing to possible adverse effects, drug therapy should
employed by medical genetics professionals is both labor- be reserved for patients in whom wrist splinting has failed.
and time-intensive. It typically involves a three-generation
Although NSAIDs are frequendy used as first-line therapy,
pedigree and may require hours to complete. Given the evidence is lacking as to their effectiveness.
multiple demands on the internist during the clinical Surgical intervention should be reserved for patients in
encounter, this degree of detail is not feasible. whom both nonpharmacologic and pharmacologic conser
Aldiough it is standard of care to perform genetic testing vative therapies have failed. Other indications include pro
for certain mutations in unselccted preconception, prenatal,
gressive sensory or motor deficits and moderate to severe
and newborn populations, and direct-to-consumer genomic
findings on elcctrodiagnostic studies. In one randomized
kits arc commercially available, at least three important issues conn-oiled trial of 116 patients, those widi carpal tunnel
make it unwise to perform genetic testing in unselccted pop
syndrome who underwent surgical intervention had better
ulations seen by internists: (1) die clinical validity of such a test
outcomes (function and symptoms) than those who under
may be lacking, (2) diere may be a high likelihood of false- went nonsurgical management, although clinically, the
positive tests, and (3) die harms of performing a genetic test benefit was modest.
may outweigh any benefits.
KEY POINT
KEY POINT
Initial therapy for carpal tunnel syndrome is
The most appropriate first screening step for
wrist splinting and avoidance of repetitive wrist
genetic diseases is to inquire about any diseases
motions.
that "run in the family" and to inquire specifi
cally about family history of the more common
and important inherited diseases, including Bibliography
Jarvik JG, Comstock BA, Kliot M, et al. Surgery versus non-surgi
breast, ovarian, prostate, and colon cancer, as cal therapv for carpal tunnel syndrome: a randomised parallel-
well as early cardiovascular disease. group trial. Lancet. 2009;374(9695):1074-1081. [PMID:
19782873]

181
Answers and Critiques

Item 3 Answer: A Item 4 Answer: D


Educational Objective: Treat obsessive-compul Educational Objective: Manage epistaxis.
sive disorder.
This patient should be told to apply uninterrupted nasal
In addition to cognitive-behavioral therapy (CBT), the pressure for 15 to 30 minutes, to not remove die clot or
pharmacologic treatment of choice for this patient is a blow his nose, and to temporarily discontinue nasal cortico
selective serotonin reuptake inhibitor (SSRI) such as flu steroids. More than 90% of epistaxis cases occur at die ante
oxetine. She has the hallmark features of obsessive-com rior nasal septum in the Kicsselbach area, the anteroinferior
pulsive disorder (OCD), including recurrent obsessions aspect of the nasal septum where multiple arteries anasto
or compulsions sufficiently severe to occupy 1 hour daily mose to form a plexus. These episodes of bleeding almost
or result in marked distress or impaired social function. always stop with consistent pressure for at least 15 minutes.
Obsessions are persistent ideas, thoughts, impulses, or Direct causes of epistaxis include nose picking, dry air dur
images experienced as intrusive and are associated with ing winter months, intranasal corticosteroids and decon
significant anxiety or distress. Examples include fears of gestants, bacterial or viral rhinosinusitis, and less commonlv
having left doors unlocked and fears of germ contami tumors.
nation. Compulsions are repetitive behaviors, such as Nasal arterial embolization is reserved for severe refrac
handwashing, checking, ordering, or counting, that are tory epistaxis, which this patient does not have.
repeated to decrease the anxiety related to the obses The patient would not require cauterization and ante
sions. Cognitive-behavioral therapy (CBT) with an rior nasal packing unless the bleeding fails to resolve with
exposure therapy element is the treatment of choice for at least 15 to 30 minutes of pressure. Nasal packing widi or
OCD. SSRIs are the most effective pharmacotherapy without nasal constrictive agents is effective (60%-80%) but
and should be used in patients who are resistant or only uncomfortable for the patient given that the packing is left
partially responsive to CBT and in those with more in for 1 to 3 days.
severe OCD or in whom a rapid response is critical. Based upon the patient's history, it is unlikely that he
Higher SSRI doses are often needed to treat OCD as has a bleeding disorder as a systemic cause of his epistaxis
compared with depression, and the dose may be esca and is unlikely to have developed a significant anemia from
lated at 2- to 4-week intervals. Adjunctive use of antipsy 2 hours of epistaxis. Laboratory studies are rarely helpful in
chotics has some evidence of benefit. healthy patients with epistaxis. Even in patients with known
Haloperidol, a typical antipsychotic agent, has been bleeding disorders, laboratory studies and coagulation
shown to be effective in combination widi an SSRI for studies are normal in 80% of patients.
OCD in patients who are refractory to initial treatment. It is not necessary to consult an otorhinolaryngologist
Haloperidol has many serious adverse effects, including because the patient reports that the bleeding stops widi
QT-interval prolongation and extrapyramidal symptoms pressure and the examiner has identified an oozing vessel in
that further limit its use as a first-line agent. the anterior nasal septum. Posterior bleeds warrant referral
Benzodiazepines, such as lorazepam, are not effective because they can be difficult to control by application of
for treating OCD and should be avoided given their lack of direct external pressure.
efficacy and high risk for dependence.
KEY POINT
Quetiapine is an atypical antipsychotic agent that has
not been shown to improve OCD symptoms in patients Anterior nosebleeds almost always stop with
who are refractory to serotonin reuptake inhibitor consistent pressure for at least 15 minutes.
monotherapy.
Bibliography
KEY POINT Schlosser R. Epistaxis. N Engl J Med. 2009;360(8):784-789. [PMID:
192286211
Cognitive-behavioral dierapy (CBT) with an
exposure element is the treatment of choice for
obsessive-compulsive disorder (OCD); high-
dose selective serotonin reuptake inhibitors Item 5 Answer: B
should be used in patients who are resistant or Educational Objective: Diagnose hypoactive sex
only partially responsive to CBT and in those ual desire disorder.
with more severe OCD or in whom a rapid The most likely diagnosis is hypoactive sexual desire disorder
response is critical. (HSDD). Female sexual dysftinction, defined as sexual diffi
culties that are persistent and personally distressing to the
Bibliography
Soomro GM, Altman D, Rajagopal S, Oakley-Browne M. Selective patient, affects up to 35% of sexually active women and is
serotonin re-uptake inhibitors (SSRIs) versus placebo for obsessive common among middle-aged women. HSDD is defined as
compulsive disorder (OCD). Cochrane Database Syst Rev. a persistent lack of desire for or receptiveness to sexual activ
2008;( 1 ):CD001765. | PMID: 18253995] ity or a persistent lack of sexual thoughts. HSDD is one of

182
Answers and Critiques

the most common causes of female sexual dysfunction, and palliative care. It is most often the result of direct cardio
prevalence ranges from 12% to 19%. Natural and surgical thoracic pathology, such as pleural effusion, heart failure,
menopause may contribute to the development of HSDD, as COPD, pulmonary embolism, pneumonia, or lung metas
the associated decline in testosterone levels may decrease sex tases. Dyspnea can also be caused by systemic conditions,
ual motivation and desire. There is no FDA-approved med such as anemia, muscle weakness, or conditions causing
ication for die treatment of female HSDD; individual and abdominal distention. Patients with underlying lung disease
couples sex therapy or psychodierapy may be beneficial. on bronchodilator therapy should have this therapy con
Dyspareunia is persistent urogenital pain diat occurs tinued to maintain comfort. Opioids are effective in reduc
around intercourse and is not related exclusively to inade ing dyspnea in patients with underlying cardiopulmonary
quate lubrication or vaginismus. Several conditions may disease and malignancy. In patients already receiving opi
cause dyspareunia, including interstitial cystitis, pelvic adhe oids, using the breakthrough pain dose for dyspnea and
sions, infections, endometriosis, pelvic venous congestion, increasing diis dose by 25% if not fully effective may be
and vulvodynia. Treatment is aimed at correcting die under helpful. A 5-mg dose of oral morphine given four times
lying abnormality. This patient's absence of sexual pain, his daily has been shown to help relieve dyspnea in patients
tory of previously normal sexual intercourse, and lack of widi end-stage heart failure. Low-dose (20 mg) extended-
symptoms and signs associated widi any of die aforemen release morphine given daily has been used to relieve dysp
tioned conditions (no urinary symptoms, no history of pelvic nea in patients with advanced COPD.
surgeries or sexually transmitted infections) make dyspareu Antibiotics and corticosteroids are appropriately used
nia an unlikely etiology for her current sexual problems. in patients widi exacerbations of severe COPD. However,
Sexual aversion disorder is a persistent or recurrent neither would be expected to provide immediate relief of
aversive response to any genital contact with a sexual part the patient's respiratory distress and would also be incon
ner. Physiologic responses often accompany these feelings, sistent with care focusing primarily on comfort measures at
with associated nausea and shortness of breath. Frequendy die end of life.
there is a history of a painful or traumatic sexual event. In contrast to opioids, benzodiazepines have not
Although this patient is avoiding intercourse, this is related demonstrated consistent benefit in treating dyspnea. How
to low sexual desire and motivation. Patients with sexual ever, they may be useful in specific patients who have sig
aversion disorder avoid intercourse because of feelings of nificant anxiety associated with their dyspnea.
revulsion and disgust.
KEY POINT
Vaginismus is involuntary and recurrent spasm of the
outer third of the vaginal musculature that interferes with Opioids are effective in reducing dyspnea in
vaginal penetration. Pain may accompany this involuntary patients with underlying cardiopulmonary dis
spasm, and there is often associated avoidance and antici ease and malignancy.
patory fear of penetration. Prevalence ranges between 1%
and 6%. On examination, this patient easily tolerated inser Bibliography
tion of the vaginal speculum without any evidence of Swetz KM, Kamal AH. Palliative care. Ann Intern Med.
muscular spasm, although it should be noted that some- 2012;156(3):ITC21. [PMID: 22312158]
women experience vaginismus only during sexual activity
(situation -specific). Item 7 Answer: D
KEY POINT
Educational Objective: Diagnose chronic fatigue
Hypoactive sexual desire disorder, a common syndrome.
cause of female sexual dysfunction, is defined as
a persistent lack of desire for or reccptiveness to This patient requires no additional testing. Chronic fatigue
sexual activity or a persistent lack of sexual syndrome (CFS) is defined as medically unexplained fatigue
diat persists for 6 months or more, accompanied by at least
thoughts.
four of die following symptoms: subjective memory impair
Bibliography ment, sore throat, tender lymph nodes, muscle or joint
Kingsbcrg S, Althof SE. Evaluation and treatment of female sexual dis pain, headache, unrefreshing sleep, and postexertional
orders. Int Urogvnecol J Pelvic Eloor Dysfunct. 2009;20(suppl malaise lasting longer than 24 hours.
1):S33-S43. [PMJD: 19440781] There is no diagnostic test for CFS. An appropriate his
tory and physical examination should include an assessment
for common causes of fatigue, including a sleep history; diose
Item 6 Answer: D
with a history of loud snoring, apneic spells, or frequent limb
Educational Objective: Treat dyspnea at the end movements during sleep should undergo a sleep study to
of life. evaluate for sleep apnea or resdess legs syndrome. All patients
This patient on comfort care should be given morphine. Dys with fatigue should be assessed for depression and anxiety
pnea is one of the most common symptoms encountered in disorders. Patients widi hypothyroidism may present widi

183
Answers and Critiques

fatigue in the absence of odier findings; it is reasonable to pretest probability of abnormality and should not be
order a thyroid-stimulating hormone level. If significant ordered.
weight loss, lymphadenopathy, or fever is found, assessment In the absence of a personal or family history of abnor
for malignancy and chronic infections should ensue. A com mal bleeding, liver disease, significant alcohol use, malab
plete blood count should be obtained to rule out anemia and sorption, or anticoagulation therapy, the likelihood of a
to look for evidence of lymphoma or leukemia. A metabolic
bleeding disorder is low, and no further preoperative test
panel is reasonable to rule out diabetes mellitus, kidney dis ing is required.
ease, and liver disease. An erythrocyte sedimentation rate can This patient is young, has no signs or symptoms of car
help assess for an active inflammatory process. Additional diac disease, and no risk factors for early or silent myocar
studies may be warranted in selected patients based on die- dial infarction. Preoperative electrocardiography should be
history and physical examination findings. This patient, how limited to patients in whom a silent or previously unrecog
ever, has no findings on physical examination that warrant nized myocardial infarction is possible.
additional testing and her previous laboratory evaluation was
KEY POINT
appropriate and normal; repeating previously normal labora
tory tests is not indicated. Perioperative testing should be limited to tests
Although CFS has been associated widi certain viruses, that have a reasonable pretest probability of
such as Epstein-Barr virus and parvovirus B19, these asso being abnormal, and whose abnormal result
ciations have not been reproduced consistently in studies. would direcdy impact perioperative care.
Thus, obtaining titers for these viruses would neither defi
nitely identify etiology nor result in a treatable diagnosis. Bibliography
KEY POINT Hepner DL. The role of testing in the preoperative evaluation. Cleve
Clin J Med. 2009;76 (suppl 4):S22-7. [PMID: 19880831]
There is no test that is diagnostic for chronic
fatigue syndrome; in addition to a complete
history and physical examination, reasonable Item 9 Answer: B
tests to consider are a complete blood count, Educational Objective: Evaluate pulsatile
comprehensive metabolic panel, and diyroid- tinnitus.
stimulating hormone level.
Tinnitus that is timed with the patient's pulse (pulsatile tin
Bibliography nitus) is concerning for an intracranial vascular anomaly,
Holgate ST, Komaroff AI., Mangan D, Wcsscly S. Chronic fatigue syn such as stenosis, arteriovenous malformation, or glomus
drome: understanding a complex illness. Nat Rev Neurosci. tumor, although patients with middle ear congestion may
2011;12(9):539-544. |PMID: 21792218]
report a pulsatile sensation as part of their symptom com
plex. It is valuable to listen over die patient's ears, eyes, and
Item 8 Answer: D neck, because if a vascular abnormality is present, the bruit
causing tinnitus may sometimes be detected externally
Educational Objective: Manage preoperative test
(objective tinnitus). If a bruit is present, or there is other
ing in a patient without comorbidities.
significant concern for a vascular cause of tinnitus, imaging
The only diagnostic test appropriate to perform preopera of die cranial vasculature (Doppler ultrasonography or
tively in diis young woman is a pregnancy test. A preopera magnetic resonance angiography) is the definitive study to
tive pregnane)' test is recommended in all women for whom rule out these diagnoses.
pregnancy is still possible. Any additional preoperative test Audiometry would be appropriate if the patient had
ing should be selective, based on the likelihood of finding an decreased auditory acuity, but this patient's physical exam
abnormality and, more importandy, diat die result will ination findings do not suggest hearing loss.
change management. The results of most screening tests will External noise generators to mask the sound of tinni
be normal, and any abnormalities found usually do not affect tus are helpful for some patients and may be reasonable to
management. Additionally, patients with previously normal consider in this patient if no reversible cause is found. Evi
laboratory studies in the 4 months prior to surgery and no dence supporting their efficacy is sparse, however. The
change in their clinical condition rarely warrant repeat test mainstay of therapy for tinnitus is neurocognitive interven
ing. However, most hospital policies continue to require pre tions to help patients cope with the problem and diminish
operative screening tests despite the evidence to die contrary. dysfunctional cognitive processes associated with the expe
This approach is not endorsed by any major physician groups rience of tinnitus.
and, owing to chance alone, will yield 1 abnormal result for Nasal corticosteroid sprays can be helpful in patients
every 20 tests ordered, which are typically either false-posi with eustachian tube dysfunction, although this is an
tive results or of no clinical significance. uncommon cause of tinnitus associated with conductive
This patient has no symptoms or signs of pulmonary
hearing loss. This patient does not demonstrate conductive
disease. In the absence of this, chest radiography has a low hearing loss and has no evidence of middle ear congestion

184
Answers and Critiques

on examination. Thus, nasal corticosteroids are not indi Clinical Practice Guideline executive summary. Respir Care.
2008;53(9):1217-1222. [PMID: 18807274]
cated as die next step in management.
KEY POINT
Patients with pulsatile tinnitus should be evalu Item 11 Answer: A
ated for the possible presence of an intracranial Educational Objective: Manage viral conjunctivitis.
vascular anomaly, such as stenosis, arteriove
The most appropriate management is the application of cool
nous malformation, or glomus tumor as a cause
compresses to the affected eye. This patient has symptoms
of their symptoms. and signs most consistent with viral conjunctivitis. Onset is
usually acute, with unilateral redness, watery discharge, itch
Bibliography
ing, crusting, a diffuse foreign body sensation, and mild pho
Liyanagc SH, Singh A, Savundra P, Kalan A. Pulsatile tinnitus. J Laryn-
gol Otol. 2006;120(2):93-97. [PMID: 16359136] tophobia. This patient's preceding upper respiratory tract
infection, normal vision, and unilateral eye involvement are
supportive of this diagnosis. Viral conjunctivitis is managed
Item 10 Answer: D conservatively widi cool compresses. The patient should be
Educational Objective: Counsel a patient regard told not to share towels or other personal items widi family
members and should wash his hands frequendy throughout
ing methods for smoking cessation.
the day. He should also be warned that the infection may
Aldiough both counseling and pharmacodierapy are effective spread to the odier eye before resolving.
strategies for smoking cessation, the combination of coun Allergic conjunctivitis may be recurrent and seasonal
seling with medication use is more effective than either inter and presents with itching, conjunctival edema, and cobble-
vention alone. Nicotine replacement is effective for smoking stoning under the upper lid. It usually responds to topical
cessation; its availability in multiple formulations (gum, antihistamines, short-course topical NSAIDs (3 days max
lozenge, patch, aerosol) allows for alternative options in imum), and compresses. Oral antihistamines have no role in
patients who have not benefited from one type of dierapy, as the treatment of viral conjunctivitis.
in diis patient. Although cenually acting agents (bupropion, Bacterial conjunctivitis usually has a mucopurulent dis
varenicline) are also effective treatment options, bupropion charge, in contrast to the clear, watery discharge seen in
would be contraindicated in this patient with an underlying viral conjunctivitis. Topical antibiotics are not efficacious for
seizure disorder. The choice of cessation method is less viral conjunctivitis and can be associated with adverse
important than that an effective method is used correctly by effects, including the development of contact dermatitis
die patient; the array of treatment options allows for individ and antibiotic resistance. If a lubricant is required, non-
ualization based on patient preference, previous experience, antibacterial lubricating agents may be used.
cost, and potential side effects. Counseling may be brief or Topical corticosteroids are not indicated despite die
intensive; the two most effective counseling components patient's discomfort and should rarely, if ever, be used by
include practical problem-solving skills and social support. physicians other than ophthalmologists. If used inappro
Many smokers indicate that stress reduction is a pri priately for herpes simplex, fungal, or bacterial conjunctivi
mary reason for their tobacco use. Although selected indi tis, topical corticosteroids can lead to corneal scarring,
viduals widi true anxiety disorders may benefit from anxi melting, and perforation.
olytic dicrapy, the use of benzodiazepines as a smoking
KEY POINT
cessation medication has not been documented.
Electronic smokeless cigarettes deliver a warmed Viral conjunctivitis, characterized by acute
aerosol through a cigarette-like device that bears the onset and unilateral redness, watery discharge,
appearance, physical sensation, and possibly the taste of itching, crusting, a diffuse foreign body sensa
tobacco smoke, with the intention of helping smokers tion, and mild photophobia, is managed con
maintain the activities associated with smoking but without servatively with cool compresses.
the harmful effects. However, their use in smoking cessa
tion has not been established. Bibliography
Galor A, Jeng BH. Red eve for the internist: when to treat, when to
KEY POINT refer. Clev Clin J Mcd.'2008;75(2): 137-144. [PMID: 18290357]
Smoking cessation is achieved more effectively
with a combination of counseling and anti-
Item 12 Answer: A
smoking medication use than with eidier inter
vention alone. Educational Objective: Identify anchoring as a
source of diagnostic error.
Bibliography
This case illustrates an error resulting from an anchoring
2008 PHS Guideline Update Panel, Liasons, and Staff. Treating
tobacco use and dependence: 2008 update U.S. Public Health Service heuristic. Heuristics are cognitive shortcuts diat clinicians

185
Answers and Critiques

employ in an attempt to efficiendy reach a diagnosis. Anchor effects on cough suppression. Second-generation nonse
ing occurs when a diagnostician latches onto a diagnosis and dating antihistamines are generally ineffective for rhinosi
fails to consider other possibilities for die presenting symp nusitis symptoms.
toms. The physician in diis case settled on a diagnosis Albuterol does not relieve symptoms of rhinosinusitis
because another physician previously diagnosed die patient. unless wheezing is present. The patient did not report
Despite the fact that there are other pieces of'data that do not wheezing or shortness of breath, and wheezes were not
fit the diagnosistachycardia, relative hypotension, abdom heard on examination.
inal distention, and lack of reproducibility of pain with mus Because rhinosinusitis is caused by viruses, routine
culoskeletal examination contrary to die diagnosis of "groin antibiotic treatment in immunocompetent hosts is not rec
strain"die clinician remains anchored to die diagnosis ommended. Antibiotics do not improve symptoms, illness
established in the emergency department. This type of error duration, or patient satisfaction with medical care. Contrary
can be avoided by explicidy looking for findings diat do not to common belief, purulent sputum does not reliably predict
fit the diagnosis as well as by using a "worst case scenario" bacterial infection or superinfection. Therefore, sputum
approach to patient careconsidering the diagnoses that purulence should not be used as a criterion for antibiotic
would be most life-threatening to die patient, such as an administration. Evidence-based guidelines from die Infec
abdominal aortic aneurysm in a patient with risk factors for tious Diseases Society of America suggest that if bacterial rhi
this condition. nosinusitis is highly suspected, based on the presence of per
The availability heuristic is used when a clinician sistent symptoms or signs lasting more than 10 days widiout
attempts to make a diagnosis based upon what is available evidence of clinical improvement, onset widi severe symp
in the physician's mind rather dian what is most probable. toms (fever >39.0 C [102.2 F]), or onset widi worsening
For example, if the physician had seen a similar patient with
symptoms or signs (new fever, headache, or upper respiratory
a groin strain from physical activity die previous week and tract infection symptoms that were initially improving), the
decided that this was the most likely diagnosis, the clinician antibiotic of choice is amoxicillin-clavulanate.
would be missing the diagnosis because of die availability
Multiple studies have found little if any improvement
heuristic. in acute cough associated with acute upper respiratory tract
A no-fault error is one in which die presentation is mis infections by using codeine, dextromethorphan, or mogu-
leading and the clinician has little opportunity to pick up isteine antitussive therapy. The American College of Chest
clues based on any data that there is an underlying problem.
Physicians does not recommend treatment with these med
The representativeness heuristic is used when die clin ications. Codeine may be effective in patients with chronic
ician applies pattern recognition in making a diagnosis. This
cough; however, it is not indicated in diis patient widi acute
source of diagnostic error occurs when the patient's clini rhinosinusitis.
cal presentation appears to fit a typical case widi similar fea Other treatments diat may relieve symptoms of rhi
tures but the clinician fails to consider other disease nosinusitis include intranasal ipratropium (rhinorrhea and
processes that may also present in diis manner. sneezing), intranasal cromolyn (rhinorrhea, cough, throat
KEY POINT pain), and short-term topical nasal decongestants (nasal
obstruction). Consistent high-quality data on the use of
Anchoring heuristic errors occur when a clini
cian holds to an initial impression, such as zinc, echinacea, and vitamin C do not support the use of
these over-die-counter products for the treatment or pre
might occur when a referring physician has
vention of rhinosinusitis.
provided a diagnosis that is then accepted at
face value. KEY POINT
Antibiotics are not recommended for the treat
Bibliography ment of acute rhinosinusitis.
Trowbridge RL. Twelve tips for teaching avoidance of diagnostic
errors. Med Teach. 2008;30(5):496-500. [PMID: 18576188]
Bibliography
Siamasek M, Blandino DA. Treatment of the common cold. Am Fam
Physician. 2007;75(4):515-520. [PMID: 17323712]
Item 13 Answer: C
Educational Objective: Treat acute rhinosinusitis.
Item 14 Answer: A
Treatment with chlorpheniramine may be considered for
this patient. The common cold, or rhinosinusitis, presents Educational Objective: Diagnose benign paroxys
with acute cough, nasal congestion, rhinorrhea, and occa mal positional vertigo.
sionally, low-grade fever. Targeted treatment is aimed at This patient has benign paroxysmal positional vertigo
symptom relief. Antihistamines, such as chlorpheniramine, (BPPV). The first step in the evaluation of the patient with
and antihistamine-decongestant combinations have been dizziness is to distinguish among its major causes: vertigo,
shown to decrease congestion and rhinorrhea with variable
presyncope, dysequilibrium, and other nonspecific causes.

186
Answers and Critiques

This patient describes classic vertigo, a sensation that his Item 15 Answer: D
stationary environment is spinning around him (or that one- Educational Objective: Diagnose vitamin B12
is spinning around in one's environment). deficiency in a patient who has undergone gastric
There are two major categories of vertigo, peripheral bypass surgery.
and central. Peripheral causes of vertigo include BPPV,
vestibular neuronitis, Meniere disease, aminoglycoside tox This patient's serum vitamin Bl2 level shotdd be measured.
icity, and herpes zoster. Patients with all causes of periph Macrocytic anemia, thrombocytopenia, mild neuttopenia, and
eral vertigo have similar findings on the Dix-Hallpike an inappropriately low reticulocyte count are die hallmark
maneuver, in which the examiner supports the patient's hematologic findings in vitamin B,2 deficiency. Vitamin B12
head in 20 degrees of extension while sitting, then assists deficiency is one die most common nonoperativc complications
of Roux-en-Y gastric bypass surgery. It results from decreased
the patient rapidly to die supine position with the head
turned to one side and hanging over the edge of the exam absorption of vitamin B,,, mainly through lack of intrinsic fac
tor production from the bypassed gastric mucosa. It is essential
ining table; the test is repeated with the head turned to the for patients who undergo a roux-en-Y gastric bypass procedure
odier side. A positive test results in horizontal nystagmus
to participate in postoperative monitoring of vitamin B12 levels
with latency of 2 to 40 seconds and duration less than 1
and to maintain lifelong adequate vitamin Bp supplementation
minute, with reproduction of symptoms that will fatigue
and habituate. Patients with BPPV describe episodes of ver (500-1000 micrograms/d orally or 1000 micrograms intra
muscularly monthly). Recent guidelines recommend that
tigo lasting less than 1 minute that occur with a rapid serum vitamin B12 levels, along with ferritin, folate, vitamin D,
change in head position, as in this patient. Nervous system and calcium, be monitored twice yearly for die first 2 years after
imaging or other testing is not required for diagnosis in Roux-en-Y gastric b)yjass surgery and yearly diereafter.
patients with classic findings and an otherwise normal neu A bone marrow biopsy, a relatively invasive and cosdy
rologic examination. test, is premature at diis point. Bone marrow biopsy is indi
Central vertigo may result from ischemia, infarction, or cated for unexplained anemia, leukopenia, thrombocytopenia,
hemorrhage of die brainstem or cerebellum. It is accompa or pancytopenia. The patient has a high likelihood of vitamin
nied by diplopia, dysarthria, dysphagia, focal weakness,
Bp deficiency that can explain the patient's hematologic find
numbness, or gait abnormalities, depending on the
involved area of the brain. On the Dix-Hallpike maneuver, ings, and a performing a bone marrow biopsy prior to mea
suring die serum vitamin B,2 level would be inappropriate.
the induced nystagmus has no latency, lasts more than 1 With no gastrointestinal symptoms, no family history
minute, and can have a vertical direction. The Dix-Hallpike of colorectal cancer, a benign abdominal examination,
maneuver findings in diis patient are consistent with periph
guaiac-negative stool, and a macrocytic (not microcytic)
eral, not central, vertigo. As he has neither cardiovascular anemia with neutropenia and thrombocytopenia, the sus
risk factors nor focal neurologic abnormalities, ccrebellar
picion for gastrointestinal malignancy or other lower
infarction is unlikely.
gastrointestinal tract disease is very low. Therefore, a
The classic triad of Meniere disease is vertigo, unilat colonoscopy is not indicated at this point.
eral hearing loss, and tinnitus. The vertigo in Meniere dis Based on history and physical examination, diere is no
ease is usually not positional, as in this patient, and he does
strong clinical suspicion for thyroid disease. Although
not have hearing loss or tinnitus. hypothyroidism can be associated with mild macrocytosis,
Vestibular neuronitis is frequently associated with a the presence of neutropenia and thrombocytopenia is
viral infection. Symptoms generally last much longer than inconsistent with this diagnosis. Therefore, measuring the
they do in BPPV (days) but can be more severe. Although scrum thyroid-stimulating hormone level is not indicated.
symptoms typically resolve within 1 week, residual dizziness
can last for months. The recurrent, intense, and brief nature KEY POINT

of this patient's episodes of vertigo makes BPPV a more- Vitamin B12 deficiency is common after Roux-
likely diagnosis. en-Y gastric bypass surgery.
KEY POINT
Bibliography
Benign paroxysmal positional vertigo is charac Buchwald H, Ikramuddin S, Dorman RB, Schonc IL, Dixon JB. Man
terized by brief episodes of severe vertigo agement of the metabolic/bariatric surgery patient. Am J Med.
2011;124(12):1099-1105. [PMID: 22014789]
brought on by a rapid change in head position
and Dix-Hallpike maneuver findings of delayed
horizontal nystagmus widi a rotatory compo Item 16 Answer: D
nent concurrent with symptoms of vertigo.
Educational Objective: Manage a patient's
Bibliography request for genetic testing.
Bhattacharyya N, Baugh RF, Orvidas L, et al. Clinical practice guide This patient should bc referred for genetic counseling.
line: benign paroxvsmal positional vertigo. Otolaryngol Head Neck
Surg. 2008;139(5)(suppl 4):S47-81. [PMID: 18973840] Huntington disease is an autosomal dominant disorder

187
Answers and Critiques

caused by a CAG repeat within the gene on chromosome when cellulitis is present. The possibility of underlying
4. Because of the potential harms from genetic test infor osteomyelitis should be considered. Surgical or nonsurgi
mation and the need for patients and their families to cal debridement of eschar and nonviable tissue may be
receive appropriate information for decision-making, needed. Wet-to-dry dressings may aid in debridement but
patients with possible inherited diseases should undergo caution must be used to avoid removing excessive viable-
genetic testing only in the context of genetic counseling. tissue with dressing changes.
Genetic counseling should include discussion of possible Cochrane reviews do not support a role for electro
risks and benefits of early detection and prevention
magnetic therapy, ultrasound therapy, or hyperbaric oxygen
modalities. Genetic counseling with the option for testing therapv' in pressure ulcer treatment.
should be offered when: (1) the patient has a personal or Negative-pressure wound vacuum healing has been
family history suggestive of a genetic susceptibility condi used for stage TV ulcers. However, three clinical trials have
tion; (2) the genetic test can be adequately interpreted; not shown superiority to standard dicrapy. Wound vacuum
and (3) the test results will aid in diagnosis or influence
therapy may be more convenient due to less frequent dress
the medical or surgical management of the patient or fam ing changes but is also very cosdy. It is not die recom
ily at hereditary risk. mended first line therapv'.
Brain MRI in patients with well-defined findings of
Although frequendy utilized, vitamin C and zinc oral
Huntington disease demonstrates caudate atrophy. Such supplements have not been shown to aid in ulcer healing.
imaging is unlikely to be helpful in an asymptomatic patient Referral for surgical flap and repair may be necessary
and is not preferred to genetic counseling in estimating the for refractory pressure ulcers but is usually reserved for
likelihood of disease. patients in whom conservative treatment has failed.
Symptoms of Huntington disease typically begin in die
KEY POINT
fourth and fifth decades, but 10% of patients have symp
toms in the second decade. It is premature to reassure the Stage m pressure ulcers, defined by full-thick
patient, considering her young age and absence of genetic ness tissue loss but without exposure of bone,
test data. tendon, or muscle, generally require debride
ment, proper dressing selection, and treatment
KEY POINT
of infection, if present.
Patients with possible inherited diseases should
be referred for genetic testing only in the con Bibliography
text of genetic counseling. Reddy M, Cill SS, Kalkar SR, et al. Treatment of pressure ulcers: a sys
tematic review. fAMA. 2008;300(22):2647-2662. [PMID-
I9066385]
Bibliography
Berg AO, Baird MA, Botkin JR, et al. National Institute of Health
State-of-the-Science Conference Statement: Family History and
Improving Health. Ann Intern Med. 2009; 151(12)872-877 Item 18 Answer: E
[PMID: 19884615] Educational Objective: Diagnose benign prosta
tic hyperplasia.
Item 17 Answer: A The most important diagnostic test to perform next in
Educational Objective: Manage a pressure ulcer. this patient is urinalysis. Benign prostatic hyperplasia
(BPH) is an extremely common cause of lower urinary
This patient has a stage III or IV pressure ulcer, and tract symptoms in men, such as nocturia, urinary fre
debridement is the most appropriate management. Both
quency and urgency, decreased urinary stream, urinary
stage III and stage IV pressure ulcers include full-thick retention, incomplete bladder emptying, and inconti
ness tissue loss. In stage III ulcers, subcutaneous fat may nence. Symptoms may worsen with increasing size of the
be visible but bone, tendon, and muscle are not
prostate. The American Urological Association (AUA)
exposed, whereas in stage IV ulcers, bone, tendon, or recommends screening and following the AUA symptom
muscle is exposed and undermining and tunneling are score to assess need for and effectiveness of treatment.
often present. The eschar covering this patient's wound
Diagnostic work-up for BPH includes the necessity to
precludes the definitive differentiation of a stage III rule out underlying infection. Although this patient does
from a stage IV ulcer. not have specific symptoms or clinical findings suggestive
Treatment of pressure ulcers is best managed with an of a urinary tract infection, abnormal bladder emptying
interdisciplinary team approach, widi a care plan directed increases the risk for infection, and even subclinical infec
toward addressing the factors diat predisposed to the tion may exacerbate BPH symptoms. Therefore, urinaly
development of the ulcer. Dressings should be chosen to sis (and if positive, a subsequent urine culture) is indicated
maintain a moist wound environment and manage exu in this patient.
dates. When present, infection should be controlled with Postvoid residual urinary volumes are measured via in-
topical therapies and the addition of systemic antibiotics and-out catheterization or by ultrasonograph)' after a

188
Answers and Critiques

patient spontaneously voids. This test is not indicated for challenging to dose-adjust. It has also been linked to QT-
routine management of BPH but is useful in evaluation of interval prolongation and other arrhythmias. It should be
overflow incontinence or neurogenic bladder. As neither of used with caution, with monitoring of the QT interval. For
these conditions is present in this patient, postvoid residual these reasons, methadone is generally a less ideal choice for
urinary volume measurement is not indicated. long-term pain control in cancer patients.
Elevated plasma glucose levels may cause urinary fre Short-acting hydromorphonc has a rapid onset of
quency and nocturia due to osmotic diuresis but are action and is dosed every 4 to 6 hours. This would not be
unlikely to cause weak stream or urinary hesitancy. Addi optimal in this patient as he requires sustained relief from
tionally, urinary incontinence due to neurogenic bladder his pain and would require repeated doses for adequate
can be seen in late-stage diabetes mellitus with neuropathic control, including during hours of sleep. Extended-release
complications; however, this patient has no history of dia hydromorphonc preparations are now available, and in con
betes mellitus and is not incontinent. junction with short-acting hydromorphone for break
The U.S. Preventive Services Task Force has con through pain, these formulations would also be a reason
cluded that the harms of screening for prostate cancer out able treatment option.
weigh the benefits in men of any age regardless of risk fac
KEY POINT
tors. Other guidelines, including those of the AUA,
recommend offering periodic screening to men older than Long-acting morphine is indicated in cancer
50 years who have a life expectancy of at least 10 years. The patients who develop persistent pain through
presence of BPH symptoms does not affect indications for out the day or beyond 24 hours of treatment
prostate cancer screening. with shorter-acting opioids; die initial dose is
Transrectal ultrasound of the prostate is most com usually 30% to 50% of a patient's current 24-
monly used for evaluation of prostate cancer. Prostate can hour usage of short-acting opioid.
cer itself rarely causes lower urinary tract symptoms as seen
in this patient. Bibliography
Svvetz KM, Kama! AH. Palliative care. Ann Intern Med.
KEY POINT 2012;156(3):ITC2-1-TC2-16. [PMID: 22312158]
Diagnostic work-up for benign prostatic hyper
plasia (BPH) includes urinalysis to rule out
Item 20 Answer: C
underlying infection; neither post-void residual
urinary volume measurement nor prostate-spe Educational Objective: Manage unilateral sen
cific antigen testing is indicated in die routine sorineural hearing loss.
evaluation of BPH.
This patient should undergo Mill of the posterior fossa
and internal auditory canal. His sensorineural hearing loss
Bibliography
is confirmed on physical examination, which demon
McVary KT, Rochrborn CG, Avins AL, et al. Update on AUA guide
line on die manacement of benign prostatic hyperplasia. J Urol. strates lateralization of the Weber test to the left ear and
2011;185(5):1793-1803. [PMID: 21420124) demonstration of better hearing with air conduction than
bone conduction with the Rinne test. The Rinne test is
about 80% sensitive in diagnosing sensorineural hearing
Item 19 Answer: D loss. In patients with asymmetric sensorineural hearing
Educational Objective: Treat cancer-related pain. loss that is not clearly due to Meniere disease, contrast-
enhanced MlU of the posterior fossa and internal auditory
Sustained-release morphine should be started to better con
trol this patient's cancer-related pain. Long-acting mor canal should be considered to exclude acoustic neuroma
and meningioma.
phine is indicated in cancer patients who develop persistent
Continuous tinnitus most often originates within the
pain throughout the day or beyond 24 hours of treatment
with shorter-acting opioids. The initial dose is usually 30% auditory system and is usually a consequence of sen
to 50% of a patient's current 24-hour usage of short-acting sorineural hearing loss. A high-pitched continuous tone is
most common. Low-pitched tinnitus may be seen in
opioid. He should also be prescribed short-acting oral mor
phine for breakthrough pain, and his long-acting morphine patients with Meniere disease. Other causes are noise expo
should be gradually titrated upward until his pain is well sure, ototoxic medications, presbycusis, otosclerosis,
controlled. acoustic neuroma, and barotrauma. Treatment of tinnitus
A fentanyl patch is convenient and reduces frequency should first be directed at the underlying disorder. Behav
of dosing but will take 24 hours to begin working. There ioral dierapies include biofeedback, stress reduction, and
fore, used alone it would not be the correct choice for this cognitive-behavioral therapy directed at improving the
patient who is being discharged today. patient's ability to cope with tinnitus. Behavioral dierapy is
Methadone, a long-acting opioid, has an unpredictable not appropriate until other remediable causes of tinnitus are
half-life that varies from patient to patient making it excluded.

189
Answers and Critiques

Sudden sensorineural hearing loss is defined as hearing KEY POINT


loss occurring in 3 days or less. Ninety percent of patients Arrhythmia is a common cause of syncope in
have unilateral hearing loss, and some have tinnitus, car full the elderly; arrhythmogenic syncope is charac
ness, and vertigo. It is considered an otologic emergency, terized by a brief or absent prodrome and pal
and oral corticosteroids are usually given, although ran
pitations immediately preceding the event.
domized trials differ in their conclusions regarding efficacy.
This patient's hearing loss has occurred over a period of Bibliography
mondis, and oral corticosteroids arc not indicated. Moya A, Sutton R, Ammirati F, et al; Task Force for the Diagnosis and
Benign paroxysmal positional vertigo (BPPV) is the Management of Syncope; European Society of Cardiology (ESC);
most common cause of vestibular dizziness. BPPV is European Heart Rhythm Association (EHRA); Heart Failure Asso
ciation (HFA); Heart Rhythm Society (HRS). Guidelines for the
thought to be caused by otolith debris within die semicir
diagnosis and management of syncope (version 2009). Eur Heart
cular canal. Otolith repositioning has been shown helpful in J. 2009;30(21):2631-2671. [PMID: 19713422]
resolving symptoms of BPPV, however this patient does not
have symptoms compatible with BPPV and therefore this
intervention is not warranted. Item 22 Answer: A
KEY POINT Educational Objective: Treat bilateral symmetric
In patients with asymmetric sensorineural hear lower extremity edema due to venous stasis.

ing loss that is not clearly due to Meniere dis The most appropriate treatment for this patient is use of
ease, contrast-enhanced MRI of the posterior compression stockings. This obese woman has dependent
fossa and internal auditory canal should be symmetric bilateral lower extremity edema caused by
considered to exclude acoustic neuroma and venous insufficiency. Her normal cardiovascular and
meningioma. abdominal examinations, along with normal creatinine and
albumin levels, urinalysis, and liver chemistry tests, rules out
Bibliography significant heart, lung, kidney, or liver disease as causative.
McDonald R. Acoustic neuroma: what the evidence says about evalu The most effective treatments for edema due to chronic
ation and treatment. J Fam Pract. 2011;60(6):El-4. [PMID:
venous stasis are weight reduction, sodium restriction, leg
21647465]
elevation, and compression stockings. External compres
sion decreases accumulation of tissue fluid in the lower
Item 21 Answer: B extremities diat is unable to bc removed through the
Educational Objective: Diagnose the cause of a
venous system. Compression stockings are particularly
effective when used for extended periods when in the
syncopal episode.
upright position to prevent dependent pooling of fluid.
In this elderly woman, given the short prodrome, palpita Lower abdominal and pelvic imaging may be useful in
tions, and history of a previous event, a cardiac arrhythmia assessing for lesions obstructing venous and lymphatic
is the most likely cause of syncope. Arrhythmias are the return as a cause of bilateral lower extremity edema. How
most common causes of syncope in the elderly population. ever, this is an uncommon cause of swelling in an otherwise
Patients with an arrhythmogenic cause of syncope usually healthy individual with a typical clinical picture of venous
have had only one or two episodes. A prodrome is usuallv insufficiency.
brief or absent. The patient often experiences palpitations According to expert opinion, diuretics should be
immediately preceding the episode. avoided in patients with chronic venous insufficiency
Aortic stenosis may cause syncope; however, despite a because they do not predictably lead to mobilization of
low-grade systolic murmur, this patient's carotid upstrokes fluid from the interstitial to the vascular space. Instead,
are normal and her episode occurred at rest and not with diuretics may lead to decreased intravascular volume fol
exertion, making aortic stenosis unlikely. lowed by orthostatic hypotension and prerenal azotemia.
Myocardial ischemia is a rare cause of syncope, espe Lower extremity venous duplex ultrasonography is
cially in the absence of typical ischemic symptoms. Myocar useful in evaluating for the presence of venous thrombo
dial ischemia is a consideration in patients with an arrhyth sis. In this patient there is no suspicion of deep vein dirom-
mia leading to presyncope or syncope as ischemic bosis, and imaging is not necessary for a diagnosis of
myocardium may bc arrhythmogenic. However, myocar venous insufficiency without additional concerning signs
dial ischemia as a cause of hypoperfusion in a patient with or symptoms.
out symptoms at rest would likely not account for her clin
KEY POINT
ical presentation.
This patient has several risk factors for a possible tran The most effective treatments for edema due to
sient ischemic attack (TIA). However, TIAs typically pres chronic venous stasis are sodium restriction, leg
ent widi focal neurologic symptoms and findings, and arc elevation, weight reduction, and compression
rarely a cause of syncope. stockings; diuretics should be avoided.

190
Answers and Critiques

Bibliography Item 24 Answer: D ^Tj


O'Brien JG, Chennubhoda SA, Chennubhotla RV. Treatment of Educational Objective: Prevent pressure ulcers in
edema. Am Fam Physician. 2005;71( 11 ):2111-2117. [PMID:
an elderly patient.
15952439]
The most appropriate intervention for preventing pressure
ulcers in this patient is use of a pressure-distributing mat
Item 23 Answer: A tress. He was admitted to the hospital with peritonitis and
Educational Objective: Manage a suspected rota has a very high risk for pressure ulcer given his limited
tor cuff tear. mobility, ascites, and low serum albumin level. The blanch
ing erythematous patches on his lower back and buttocks
This patient should undergo MlU of the left shoulder. must be watched carefully because diey are at-risk areas for
He most likely has a complete left supraspinatus rotator ulceration. A nonblanching erythematous patch is a stage I
cuff tear. The diagnosis is suggested by his difficulty with pressure ulcer. There is evidence that pressure reduction,
abducting the left arm and the positive drop-arm test. accomplished through frequent patient repositioning and
The drop-arm test can be performed by the examiner use of pressure-distributing support surfaces, is effective in
passively abducting the patient's arm and then having reducing the risk of ulcers. The strongest evidence of ben
the patient slowlv lower the arm to the waist. When a efit exists for higher-specification foam mattresses com
complete supraspinatus tear is present, the patient's arm pared with standard hospital mattresses; beds or mattresses
often drops to the waist. Although imaging is not nec that actively alternate pressure by shifting air or water are of
unclear benefit, but may be useful in clinical situations in
essary in most patients with uncomplicated shoulder
which prolonged bedrest is required. Medical-grade sheep
pain, because of the high likelihood of a complete
skin overlays also have some evidence of benefit compared
supraspinatus tear by history and examination, it is
widi standard hospital mattresses. Bed overlays made of
appropriate to obtain an MPJ to confirm the diagnosis.
MRI has a high sensitivity (>90%) and specificity in the foam, gel, or water or air-filled pockets are commonly used,
although their efficacy in preventing ulcer development has
diagnosis of rotator cuff tears. Not all rotator cuff tears
not been established and there is some evidence that diey
require surgical intervention and many respond to con
servative therapy; however, establishing the diagnosis may contribute to additional skin irritation.
Air-filled vinyl boots, water-filled gloves, regular sheep
and obtaining more detailed anatomic information are
skin, and doughnut-type devices are likely to be harmful
necessary in making the decision about whether surgery and should not be used for prevention of pressure ulcers.
would be indicated.
Indwelling urinary catheters increase the risk of urinary
Medication with an NSAID may form a component of tract infection if left in place. Although they are frequently
the initial treadnent plan but a confirmed diagnosis is nec
placed specifically to avoid urine contact with skin areas at
essary to make definitive treatment decisions. high risk of ulceration, other mechanisms to avoid moisture
Aldiough referral to physical therapy is appropriate contact with the area at risk for breakdown are preferred.
for suspected or confirmed incomplete tears, it would not
KEY POINT
be the appropriate first step in this patient with a sus
Pressure-distributing mattresses and medical-
pected complete tear who is young and has no medical
comorbidities. grade sheepskin overlays reduce the incidence
of pressure ulcers in high-risk patients.
Performing a subacromial corticosteroid injection
would not be the most appropriate option in this patient
Bibliography
who is suspected of having a complete supraspinatus tear.
Mclnnes E, Jammali-Blasi A, Bell-Syer SE, Dumville JC, Cullum N.
Subacromial corticosteroid injections have been shown Support surfaces for pressure ulcer prevention. Cochrane Database
to provide pain relief that lasts up to 9 months in patients Syst Rev. 2011;(4):CD0() 1735. [PMID: 21491384]
with rotator cuff tendinitis or an impingement syn
drome, but a significant tear may require surgical inter
Item 25 Answer: D
vention, and this should be determined as an initial step
in management. Educational Objective: Treat menopausal vaginal
symptoms.
KEY PO NT
MRI has a high sensitivity and specificity for The most appropriate management of this patient is vaginal
esuadiol. The clinical history and physical examination are
diagnosing a rotator cuff tear. most helpful for making the diagnosis of vaginal atrophy; pale

Bibliography vaginal walls, decreased rugae, and petechiae are characteris


tic findings. Approximately 10% to 40% of menopausal
Scida JC, LeBlanc C, Schouten JR, et al. Systematic review: nonoper-
ative and operative treaunents for rotator cuff tears. Ann Intern women experience symptoms related to vaginal atrophy, which
Med. 2010;153(4):246-255. [PMID: 20621893] include vulvar itching, vaginal dryness, and dyspareunia. In

191
Answers and Critiques

contrast to menopausal vasomotor symptoms, which may last characteristically occurs with shoulder adduction and
for a few years and resolve spontaneously, vaginal atrophy is abduction above 120 degrees.
frequendy progressive and often requires treatment. Mild to Adhesive capsulitis is caused by thickening of" the cap
moderate symptoms can be treated with vaginal moisturizers sule surrounding the glenohumeral joint. Pain is character
and lubricants, but more severe symptoms, as experienced by istically slow in onset and is located near the insertion of the
this patient, are best treated with vaginal estrogen. Low-dose deltoid muscle, and patients often avoid lying on the
vaginal estradiol tablets (10-25 micrograms) and die estradiol affected side. On examination, there is loss of both active
vaginal ring (8-9 micrograms) are preferred over vaginal and passive range of motion. Although this patient's symp
estrogen creams, as they result in minimal systemic estrogen toms have been insidious in onset, which could be consis
absorption. tent with adhesive capsulitis, he does not have limited range
Although oral estrogen therapy is effective for relieving of motion in all or most planes of motion, which argues
vaginal atrophy symptoms, it has been associated with sev against adhesive capsulitis.
eral adverse outcomes, including increased rates of coro Examination findings of a rotator cuff tear include
nary heart disease, stroke, venous thromboembolism, and supraspinatus weakness, weakness with external rotation,
invasive breast cancer. For that reason, current guidelines evidence of impingement, and a positive drop-arm test
recommend the use of low-dose local, rather than systemic, (inability to slowly and steadily lower the arm completely;
estrogen therapy for the treatment of patients who only the arm drops to die side). The patient has a negative drop-
have vaginal symptoms. arm sign, which argues against rotator cuff tear.
Bacterial vaginosis is characterized by a vaginal dis The patient does not have pain between 60 to 120
charge, an increased vaginal pH, and clue cells on normal degrees of abduction, has normal internal/external rota
saline preparation and a positive "whiff" test; the absence tion, and has a negative Neer and Hawkins sign, all of
of these findings in this patient argues against this diagno which argue against the diagnosis of rotator cuff tendinitis.
sis. Treatment for bacterial vaginosis with metronidazole
KEY POINT
would therefore not bc indicated.
Acromioclavicular joint degeneration is charac
Yeast infections are often accompanied by a thick
white discharge that is potassium hydroxide-positive. This terized by pain to palpation of die acromioclav
patient does not have discharge and no hyphae are seen on icular joint and pain that occurs with shoulder
adduction and abduction above 120 degrees.
potassium hydroxide preparation; therefore, she is unlikely
to respond to vaginal clotrimazole therapy.
Bibliography
KEY POINT House J, Mooradian A. Evaluation and management of shoulder pain
Vaginal estrogen is the recommended therapy in primary care clinics. South Med J. 2010;103(11):1129-1135.
[PMID: 20890250|
for severe symptoms of vaginal atrophy that
have not responded to vaginal moisturizers and
lubricants. Item 27 Answer: A
Educational Objective: Counsel a patient regard
Bibliography
North American Menopause Society. The role of local vaginal estro ing smoking cessation.
gen for treatment of vaginal atrophy in postmenopausal women: Current recommendations are that all clinicians assess
2007 position statement of The North American Menopause Soci
ety. Menopause. 2007;14(3 Pt l):355-369. [PMID: 17438512] tobacco use at every visit, encourage every patient to make
a quit attempt, and counsel patients appropriately. Patients
who exhibit medical illnesses related to smoking present an
Item 26 Answer: A opportunity for clinicians to increase the patient's aware
Educational Objective: Diagnosis acromioclavic ness of the connection between the tin health)' behavior
ular joint degeneration. and its negative consequences. Even if time does not allow
for an in-depth counseling session, all patients should be
This patient's pain is originating from his acromioclavicu asked about their smoking at every visit, and a brief, clear
lar joint. Given his age, his pain is most likely due to message about quitting should be provided to all patients.
osteoarthritis. Acromioclavicular joint degeneration typi A recommended strategy for counseling is to follow the
cally results from trauma (in younger patients) or "five A's": Ask every patient at every visit about their
osteoarthritis (in older patients). Bilateral involvement smoking; Advise all smokers to quit; Assess their current
should raise concern for rheumatoid arthritis. On exami interest in quitting; Assist by offering resources and/or
nation, there is typically pain to palpation of the acromio medications, and Arrange for follow-up.
clavicular joint. Pain on palpation is a very sensitive but not It is not clear yet whether this patient is truly interested
specific sign of acromioclavicular joint disease; absent in quitting. Thus it would be inappropriate to prescribe
pain on palpation makes acromioclavicular joint disease either smoking cessation aids or counseling until the physi
unlikely. Palpable osteophytes may be present. Pain cian has determined that the patient is indeed ready to quit.

192
Answers and Critiques

KEY POINT and would substantially increase the risk of bleeding once
Tobacco use should bc assessed at every visit, the perioperative thromboembolism risk has resolved.
and patients who smoke should be encour KEY POINT
aged to make a quit attempt and counseled Surgical patients at high risk for venous throm
appropriately. boembolism, including those with previous
venous thromboembolism, patients who have
Bibliography
2008 PHS Guideline Update Panel, Liasons, and Staff. Treating undergone ordiopedic surgery, and patients
tobacco use and dependence: 2008 update U.S. Public Health Ser with some cancers (especially gynecologic
vice Clinical Practice Guideline executive summary. Respir Care. malignancy), should receive extended (up to 5
2008;53(9):1217-1222. [PMID: 18807274] weeks) prophylaxis.

Bibliography
item 28 Answer: A Gould MK, Garcia DA, Wren SM, et al; American College of Chest
Educational Objective: Manage postoperative Physicians; Prevention of VTE in nonorthopedic surgical patients:
Antithrombotic Therapy and Prevention of Thrombosis, 9th ed:
venous thromboembolism prophylaxis in a high-risk American College of Chest Physicians Evidence-Based Clinical
patient. Practice Guidelines. Chest. 2012;141(2 suppl):e227S-c77S.

The most appropriate treatment for this patient is venous


thromboembolism (VTE) prophylactic therapy for up to 5 Item 29 Answer: D
weeks with a low-molecular-weight heparin (LMWH), such
Educational Objective: Diagnose acute retinal
as enoxaparin. VTE is a major preventable postoperative
detachment.
complication, and nearly all surgical patients should receive
some VTE prophylaxis postoperatively. Patients at high risk This patient presents with symptoms consistent with a retinal
for VTE, including patients with previous VTE, patients who detachment. Retinal detachment occurs predominantly in
have undergone orthopedic surgery, and patients with some myopic patients and is a separation of the retina from under
cancers (especially gynecologic malignancy) should receive lying retinal epithelium and choroid as fluid from the vitreous
extended (up to 5 weeks) prophylaxis with LMWH. cavity enters a tear in the retina and dissects underneath the
Nonpharmacologic prophylaxis against VTE, such as retina. As with this patient, patients may experience floaters,
early ambulation, should be encouraged in all postsurgical squiggly lines, flashes of light, and then a sudden peripheral
patients. Other nonpharmacologic treatments include elas visual defect, appearing as a black curtain that progresses
tic compression stockings and pneumatic compression across the visual field. Funduscopic examination usually visu
devices. However, these treatments are only suitable as the alizes die tear and folding of the retina. Treatment is surgical.
sole modality when either die risk of VTE is very low (out Prognosis depends upon the extent of the tear and time to
patient surgery) or the morbidity from excess bleeding is surgery, so early recognition and emergent referral are crucial.
unacccptably high (such as in patients undergoing neuro Central retinal artery occlusion is marked by painless
surgery). This patient's surgery for a gynecologic malig loss of vision and occurs most frequently in the elderly. It is
nancy places her in a high-risk category, and pharmacologic usually caused by emboli or thrombosis. Examination
prophviaxis is indicated. demonstrates a pale fundus with a cherry-red fovea (accen
Inferior vena cava (IVC) filters arc sometimes used tuated by the pale background).
perioperatively, especially in patients with known VTE or Central retinal vein occlusion is characterized by
patients with a high risk for VTE who cannot receive pro abrupt monocular visual loss, and may present with tran
phylaxis because of bleeding risk. This patient does not have sient episodes of monocular blindness, which can last 2 to
an excessive bleeding risk, and IVC placement is not indi 4 hours, longer than is typical for transient arterial retinal
cated. Although newer IVC filters are thought to be ischemia. Patients may report cloudiness of vision rather
extractable following a procedure, the retrieval rate is not dian frank visual loss. Examination of the retina will show-
typically 100%, and a filter that cannot be removed post congested, tortuous veins; retinal hemorrhages; and cotton
operatively may complicate ongoing care. wool spots in the area of the vein occlusion.
Unfractionated subcutaneous heparin is an accepted Ocular migraine, with or without headache, can cause-
medication to prevent VTE. However, in diis high-risk floaters and squiggly lines but would not cause a visual field
patient, extended prophylaxis is indicated; therefore, provid defect or a retinal tear and folding seen on funduscopic
ing prophylaxis only until die patient is discharged is incorrect. examination.
Warfarin, in both fixed doses and adjusted doses, has The most common presenting symptom in patients
been studied for VTE prophylaxis, primarily in the ortho with temporal arteritis is a new headache, which this patient
pedic setting, and been found to be effective in preventing does not have. Although visual loss can be sudden and irre
venous thromboembolism in the perioperative period. versible in the setting of temporal arteritis, it is not pre
However, 3 months of prophylaxis would not be indicated ceded by floaters, squiggly lines, or peripheral field defects.

193
Answers and Critiques

KEY POINT KEY POINT

Patients with a retinal detachment may experi The Plan-Do-Study-Act (PDSA) cycle is a qual
ence floaters, squiggly lines, flashes of light, ity improvement approach in which a specific
and then a sudden peripheral visual defect, change is planned and implemented on a lim
appearing as a black curtain that progresses ited scale, the results are observed, and action is
across the visual field. taken based on what is learned.

Bibliography Bibliography
Magauran B. Condirions requiring emergency ophthalmologic con- Institute of Medicine of the National Academies. Crossing the Qual
sultarion. Emcrg Med Clin North Am.'2008;26(l):233-238. ity Chasm: A New Health System for the 21st Century.
| PMID: 182492651 wvvvv.iom.cdu/Reports/2001/Crossing-the-Quality-Chasm-A-
Nc\v-Hcalth-Systcm-for-thc-21st-Centurv.aspx. Published March
1, 2001. Accessed July 12, 2012.
Item 30 Answer: D
Educational Objective: Implement the Plan-Do-
Item 31 Answer: D
Study-Act (PDSA) cycle in quality improvement.
Educational Objective: Evaluate a patient with
A specific plan to improve communication of medication recurrent syncope.
allergies from outpatient to inpatient medical records
should be developed to attempt to avoid subsequent The most appropriate next step in the evaluation of diis
occurrences. The Plan-Do-Study-Act (PDSA) cycle is a patient is tilt-table testing. Tilt-table testing is useful in eval
quality improvement approach in which a specific change uating recurrent syncope in the absence of heart disease, to
is planned and implemented on a limited scale, the results discriminate neurocardiogenic from orthostatic syncope,
are observed, and action is taken based on what is learned. and to evaluate frequent syncope in patients with psychi
The first step in a PDSA cycle in this case would be to plan atric disease. This patient continues to have recurrent syn
an intervention diat would remedy the communication copal episodes despite normal cardiac and metabolic evalu
deficit between the internist's office records and the hos ations without definitive evidence of orthostasis or other
pital's electronic order system and drug allergy alert sys explanation for her symptoms.
tem. The next steps are to institute the planned interven Electroencephalography may be useful in patients in
tion in a limited fashion and then to study the outcome of whom a seizure is suspected as a cause of syncope. How
the intervention. The "act" step involves refining die inter ever, this patient has no risk factors for seizure and her
vention to achieve the ideal outcome based upon what is episodes are without a prodromal aura, evidence of seizure
learned by evaluating the limited intervention. This activity, or postictal symptoms suggestive of seizure activity.
Exercise cardiac stress testing has a low yield for syn
approach to quality' improvement works well in a small-
scale healdi care environment, such as a small office, as well cope in patients at low risk for ischemic heart disease. In this
as in a large-scale environment, such as a hospital or health patient widi a normal electrocardiogram and echocardio
care system. gram, cardiac stress testing would not be expected to con
Most physicians are aware of die importance of patient tribute significant diagnostic information.
engagement in their care, but greater involvement by Signal-averaged electrocardiography is a technique
patients may not be adequate in overcoming issues regard designed to detect altered depolarization through the
ing consistent and reliable communication of key medical myocardium that could lead to reentrant arrhythmias
information across different caregivers in different settings. that may not be evident on surface electrocardiography.
It has been studied primarily in patients following
Although it is important for patients to know, under
stand, and communicate their important medical informa myocardial infarction to assess for risk of developing sus
tion to other caregivers, not all patients are able to do so in tained tachyarrhythmias. Its use in evaluating syncope has
a reliable manner, and diis intervention will not address not been established, however, and its routine use is not
larger systemic issues related to improving quality of care recommended.
and patient safety. KEY POINT
Electronic health records may be of immeasurable Tilt-table testing is useful in evaluating recur
help in improving communication of medical informa rent syncope in the absence of heart disease, to
tion. However, implementing such systems in institutions
discriminate neurocardiogenic from orthostatic
and communities is costly, complex, and not easily accom
syncope, and to evaluate frequent syncope in
plished, and is not the next step in this case. Clear inter
ventions to avoid known patient safety issues should patients with psychiatric disease.
occur as possible within existing systems, with overall sys
Bibliography
tem change to optimize quality of care being the long-
Moya A, Sutton R, Ammirati F, et al; Task Force for the Diagnosis and
term goal. Management of Syncope; European Society of Cardiology (ESC);

194
Answers and Critiques

European Heart Rhythm Association (EHRA); Heart Failure Asso KEY POINT
ciation (HFA); Heart Rhythm Society (HRS). Guidelines for the
Avoidance of potentially teratogenic medica
diagnosis and management of syncope (version 2009). Eur Heart
J.2009;30(21):2631-2671. [PMID: 19713422] tions is important when treating medical condi
tions in reproductive-age women contemplating
pregnancy.
Item 32 Answer: C
Educational Objective: Manage medications in a Bibliography
woman who may become pregnant. Berghella V, Buchanan E, Pereira L, Baxter JK. Preconception care.
Obstet Gynecol Surv. 2010;65(2):119-131. [PMID: 20100361]
The most appropriate treatment for this woman is met
formin to treat her type 2 diabetes mellitus. Each visit
with a reproductive-age woman represents an opportu Item 33 Answer: D
nity for preconception counseling, as adequate precon Educational Objective: Treat functional urinary
ception care can reduce the risks for preterm birth and incontinence.
birth anomalies, particularly in a woman actively con
This patient would be best managed by establishing a
templating pregnancy. If this patient were to become
prompted voiding protocol. Urinary incontinence affects
pregnant, her poorly controlled diabetes and hyperten
more dian 50% of nursing home patients and is associated
sion increase her risk for adverse maternal and fetal out
with significant morbidity and cost. Most of these patients
comes. This patient should be counseled about her risk
have limited mobility or significant cognitive impairment,
factors for potential medical complications of pregnancy,
and she should be referred to a high-risk obstetrician for leading to a high prevalence of functional incontinence,
defined as simply not getting to the toilet quickly enough.
co-management of her medical and gynecologic issues In a systematic review of 14 randomized controlled studies
should she become pregnant. It is essential to avoid pre
involving 1161 nursing home patients, the use of prompted
scribing teratogenic medications to reproductive-age
women who may become pregnant. Metformin is an voiding (periodically asking the patient about incontinence,
FDA pregnancy class B medication and is a reasonable reminding the patient to go to the toilet, and providing
praise for maintaining continence and using the toilet) was
option for controlling this patient's hyperglycemia associated with modest short-term improvement in urinary
before pregnancy. If she were to become pregnant, con
incontinence.
sideration may be given to discontinuing the metformin
and starting insulin therapy, which is the preferred treat History, focused examination, and urinalysis are often
ment of diabetes in pregnancy. adequate to classify urinary incontinence. Postvoid residual
urine volume determination is most useful if overflow
The risk of premature fetal loss is increased in women
incontinence due to outlet obstruction or a flaccid neuro
with systemic lupus erythematosus, particularly in those
genic bladder is suspected. Detailed urologic evaluations,
with die antiphospholipid antibody syndrome. Low-dose such as cystoscopy and urodynamic testing, are unnecessary
aspirin has been used in these patients to attempt to lower in uncomplicated urinary incontinence.
this risk, aldiough the effectiveness of this intervention is
An indwelling Foley catheter is not advised as a first-
not clear. As aspirin may interfere with implantation when line measure to manage urinary incontinence owing to an
used near the time of conception and this patient has no
increased risk of urinary tract infection, resultant antibiotic
clear indication for aspirin therapv' at present, it should not
treatment, and the development of antibiotic complications
be prescribed.
and resistance.
Lisinopril and simvastatin, and all ACE inhibitors Pelvic floor muscle training is effective for stress incon
and statins, are teratogenic medications and can cause
tinence, which ma)' bc coexistent in this patient, but suc
serious fetal anomalies. They arc FDA pregnancy class
cessful implementation requires a cooperative and cogni-
X medications and should not be prescribed to this
tively intact patient who can understand and participate in
patient who is anticipating pregnancy. Additionally, die exercise program.
although this patient might benefit from treatment with Tolterodine, a selective anticholinergic antimuscarinic
an angiotensin receptor blocker because of her diabetes
medication, is primarily indicated for urge incontinence and
and proteinuria, this class of medication is also con is of no benefit in functional incontinence. In addition,
traindicated in pregnancy owing to its potential terato
adverse side effects, such as dry mouth and worsening cog
genic effects. Labetalol or methyldopa is safely used for nitive function, render its use in this patient ill advised.
the treatment of hypertension in pregnant women, and
may be considered for this patient. Bile acid resins, such KEY POINT
as colestipol, are not orally absorbed and are FDA preg Prompted voiding is an effective management
nancy class B medications. They may be a useful adjunct strategy for patients with functional urinary
to diet and lifestyle therapy for managing this patient's incontinence.
dyslipidemia.

195
Answers and Critiques

Bibliography Item 35 Answer: B


Fink HA, Taylor BC, Tacklind JW, Rutks IR, Wilt TJ. Treatment inter Educational Objective: Diagnose mild cognitive
ventions in nursing home residents with urinary incontinence: a sys
tematic review of randomized trials. Mavo Clin Proc. impairment.
200S;83(12):1332-1343. [PMID: L9046552] This patient most likely has mild cognitive impairment
(MCI). Memory is the only cognitive domain that is
Item 34 Answer: B impaired. Impairment of other domains that might stiggest
dementia would include impairment of language, apraxia
Educational Objective: Screen for cervical cancer.
(for example, problems with dressing not related to motor
The patient should have a Pap smear in 2 years. Because she dysfunction), and impaired executive functioning, none of
has had multiple normal consecutive satisfactory Pap which are abnormal in this patient. Patients with MCI have
smears, the most appropriate interval for cervical cancer a single or few areas of cognitive impairment, and this
screening is every 3 years (this patient had a Pap smear 1 patient's deficit is limited to forgctfulness and recalling
year ago and should undergo repeat screening in 2 years). names. His age is typical for MCI and about one-fifth of
Although the U.S. Preventive Services Task Force (USP patients older than age 70 years have diis condition. His
STF), die American College of Obstetrics and Gynecology Mini-Mental State Examination (MMSE) score is within
(ACOG), and the American Cancer Society (ACS) each dif die expected range of 24-25 for MCI and may even be
fer slightly in their specific recommendations, each agrees falsely elevated because of his high intellectual level.
that the screening interval can be extended beyond 1 year Alzheimer disease is less likely in this patient because
for this patient. The USPSTF recommends screening at there are no impairments in other domains, such as activi
least once every 3 years following the initiation of screen ties of daily living and instrumental activities of daily living;
other language difficulties; or personality changes. MMSE
ing. In contrast, the ACOG recommends that screening
occur every 2 years between the ages of 21 and 29 years and scores of 19 to 24 suggest mild dementia, and scores oflO
to 19 suggest moderate dementia. His MMSE score of 25
every 3 years beginning at the age of 30 years if the patient
has had three normal consecutive satisfactory Pap smears suggests MCI rather than dementia.
and has no history of in utero diethylstilbestrol exposure, is Pseudodementia is a condition in which the cognitive
not immunocompromised, is not HIV-positive, and docs impairment is secondary to depression. Treatment of the
not have a history of cervical intraepithelial neoplasia grade depression leads to improvement in cognition. Whereas this
2 or 3. The ACS recommends that screening should be per patient is frustrated with his condition, he is not depressed.
formed every 3 years between the ages of 21 and 29 years. Although he has risk factors for cerebrovascular dis
Between the ages of 30 and 65 years, the preferred method ease, vascular dementia would be less likely with his MMSE
of screening is the combination of a Pap smear and human score of 25 and normal neurologic examination. In addi
papillomavirus (HPV) DNA testing every 5 years. Alterna tion, vascular dementia would not affect memory in isola
tion and would likely affect additional cognitive domains
tively, a Pap smear alone can be performed every 3 years.
and neurologic functioning.
Aldiough medical providers can consider using HPV
DNA testing along with cervical cytology in women aged 30 KEY POINT
years and older to help guide the appropriate screening inter Patients with mild cognitive impairment have a
val, it should not be used alone owing to poor specificity.
Discontinuation of screening for cervical cancer at the single or few areas of cognitive impairment,
and die Mini-Mental State Examination score
age of 31 years would not be appropriate despite the is typically 24 or 25.
patient's having multiple previous normal satisfactory Pap
smears. It is generally agreed that screening should be con
Bibliography
tinued into the seventh decade, although controversy exists Plassman BL, Langa KM, Fisher GG, et al. Prevalence of cognitive
regarding the exact age to stop screening. impairment without dementia in die United States. Ann Intern
Med. 2008;148(6):427-434. [PMID: 18347351]
KEY POINT
In women older than 30 years with no risk fac
tors for cervical cancer or history of abnormal Item 36 Answer: B
Pap smears, the cervical cancer screening inter Educational Objective: Manage palliative care
val can be extended to 3 years with cytology or discussion.
5 years with cytology and human papillo
Palliative care discussions with this patient should begin
mavirus DNA testing.
now. It is important to stress to patients that a palliative care
discussion is not a discussion of withholding or withdrawal
Bibliography
Mover VA. Screening for cervical cancer: U.S. Preventive Services of treatment or patient abandonment. The primary focus of
Task Force recommendation statement. Ann Intern Med. palliative care is to relieve patient suffering and to improve
2012;156(12):880-891. [PMID: 22711081 | the quality of patients' lives and those of their caregivers.

196
Answers and Critiques

Palliative care is often thought of as end-of-life care only, Imaging studies should be directed based on suspi
but palliative care addresses pain, suffering, and quality of cions raised during history and physical examination.
life across all stages of treatment and does not exclude life- Except in cases of acute trauma, mechanical neck pain
prolonging treatment and rehabilitation. Palliative care rarely requires imaging. Patients with weakness, hypore-
may be offered along with curative or life-prolonging ther flexia, or symptoms or signs of spinal cord involvement
apies for patients with complex, life-threatening disorders. should be evaluated with MRI or CT myelography. An
Recent literature suggests that early referral for palliative electromyelogram and nerve conduction studies are most
care improves quality of life and decreases depressive symp helpful in patients with radiculopathy that is poorly
toms as compared with patients who only receive standard defined or in those for whom surgery is being considered
oncologic care. In a study of 151 patients with metastatic to localize the specific area of nerve compression. These
non-small cell lung cancer, patients referred for early pal studies are not indicated in the initial management of cer
liative care had longer median survival than those referred vical spine radiculopathy.
for oncologic care only.
KEY POINT
Waiting until the patient develops symptoms, refuses
active treatment, or is admitted to hospice care does not Conservative treatment of acute cervical radicu
take full advantage of the benefits of early and appropriately lopathy without imaging or further testing is
administered palliative care. appropriate in patients without trauma or evi
dence of weakness or myelopathy.
KEY POINT
Early referral for palliative care in addition to Bibliography
oncologic care improves quality of life and Carette S, Fehlings MG. Clinical practice. Cervical radiculopathy. N
decreases depressive symptoms as compared Engl J Med. 2005;353:392-399. [PMID: 16049211]
with standard oncologic care only.

Item 38 Answer: B
Bibliography
Temcl JS, Greer JA, Muzikansky A, et al. Early palliative care for Educational Objective: Treat chronic cough due
patients with metastatic non-small-cell lung cancer. N Engl I Med. to gastroesophageal reflux disease.
2010;363(8):733-742. [PMID: 20818875]
Omeprazole should be continued in this patient. He pre
sents widi chronic cough (>8 weeks) most likely due to
Item 37 Answer: A gastroesophageal reflux disease (GERD). Although typical
Educational Objective: Manage acute cervical heartburn symptoms are absent in more than one-third of
radiculopathy. patients with GERD-related cough, this patient's clinical
This patient should be treated with analgesics and avoid profile and symptoms of heartburn and cough exacerbated
ance of provocative activities. The initial approach to by the recumbent position are classic for GERD. The treat
ment of chronic cough due to GERD is challenging. If
patients with cervical spine radiculopathy, typically due to
nerve root compression, is assessment for weakness and lifestyle modification (weight loss, elevation of the head of
the bed, avoidance of tobacco and alcohol) is unsuccessful,
possible involvement of the spinal cord (myelopathy).
Weakness due to nerve root compression will be seen in the targeted and prolonged treatment with histamine blockers
or proton pump inhibitors (PPIs) is recommended. In a
muscles in the area of distribution of the affected nerve(s);
recent Cochrane review, patients who were treated with
myelopathy is seen as additional neurologic symptoms at PPIs experienced a significant improvement in cough
and below the affected level of the spinal cord. In patients
scores. There was no significant difference in total resolu
with acute cervical radiculopathy without appreciable
motor deficits, imaging and nerve conduction studies are tion of cough, however (odds ratio [OR] 0.46, 95% CI
not initially necessary, even if there are mild focal sensory 0.19-1.15, intention to treat analysis). The duration of
findings. Management should focus on relief of symptoms, dierapy was 2 to 3 months. As this patient has been on ther
as many patients experience complete resolution without apy for only 2 weeks and his clinical picture is without any
intervention. interim change, continuation for 8 to 12 weeks would be
A variety of nonsurgical treatments are used for man recommended.
The American College of Chest Physicians recom
agement of cervical radiculopathy in addition to nonnar
cotic analgesics, including a short course of systemic mends a symptom-guided, systematic, algorithmic
corticosteroids, hard and soft cervical collars, and cervical approach to chronic cough. There is no evidence of infec
pillows, although evidence that these interventions are tion, and therefore, antibiotics are not indicated. The
effective relative to analgesics and rest alone is of poor qual patient does not present with symptoms or signs of upper
ity. Cervical traction has also not been documented to be airway cough syndrome (postnasal drainage, frequent
effective and is not recommended as initial treatment for diroat clearing, nasal discharge, cobblestone appearance of
acute cervical radiculopathy. the oropharyngeal mucosa, or mucus dripping down the

197
Answers and Critiques

oropharynx). The use of antihistamines and decongestants, Although the chest pain that accompanies a pulmonary
such as loratadine with pseudoephedrine, should be embolism is typically pleuritic, pulmonary embolism is not
reserved until die empiric trial of treatment for GERD is associated with widespread ST-segment elevation or PR-
completed and found to be ineffective. segment depression on ECG.
Cough-variant asdima (cough is the predominant
KEY POINT
symptom) occurs in up to 57% of patients with asdima.
The chest pain of acute pericarditis is typically
Cough-variant asthma is suggested by die presence of air
way hyperresponsiveness and confirmed when cough sharp, pleuritic, retrosternal, worsened by
resolves with asthma medications. The treatment of cough- recumbency, and improved by sitting forward.
variant asdima is the same as asdima in general, but the
maximum symptomatic benefit may not occur for 6 to 8 Bibliography
weeks in cough-variant asdima. This patient does not have Khandaker MH, Espinosa RE, Nishimura RA, et al. Pericardial disease:
asthma and has a reasonable alternative explanation for his diagnosis and management. Mavo Clin Proc. 2010;85(6):572-593.
[PMID: 20511488]
chronic cough; dierefore treatment with an inhaled bron
chodilator such as albuterol is not indicated at diis time.
KEY POINT Item 40 Answer: A
The duration of empiric proton pump inhibitor Educational Objective: Manage androgen
therapy for a patient with gastroesophageal deficiency.
reflux disease-related cough is 8 to 12 weeks. This patient's hydrocodone should be discontinued. Low
testosterone levels can lead to decreased energy and libido,
Bibliography
fatigue, and erectile dysfunction. Once discovered, low
Chang AB, Lasserson Tl, Gaffhey I, Connor FL, Garske LA. Gastro- testosterone levels should be investigated further. Many
oesophageal reflux treatment for prolonged non-specific cough in
children and adults. Cochrane Database Svst Rev. 2011(1)- drugs, including opioids, high-dose corticosteroids, and
CD004823. [PMID: 21249664] hormonal therapies, can lower testosterone levels, and a
review of medications is an important initial step in the eval
uation of men with low testosterone levels. It is also impor
Item 39 Answer: B tant to test morning levels of testosterone as opposed to
Educational Objective: Diagnose acute random levels, as secretion is cyclical. In this patient,
pericarditis. hydrocodone may be decreasing testosterone levels and
This patient most likely has acute pericarditis. Characteristic should be discontinued, and the testosterone level should
findings in acute pericarditis include sharp, pleuritic ret subsequendy be retested before any testosterone replace
rosternal chest pain diat is more prominent in die recumbent ment therapy is given.
Whereas metoprolol and other p-blockers may cause
position, a pericardial friction rub, widespread ST-segment
elevation or PR-segment depression on electrocardiogram erectile dysfunction and fatigue, they do not generally
lower testosterone levels.
(ECG), and new or worsening pericardial effusion. This
While repeat morning testing of testosterone levels is
patient has pleuritic chest pain diat worsens with recumbency
and improves with sitting forward and widespread concave- recommended to confirm low values, it would be more
upward ST-segment elevation, making acute pericarditis die appropriate to first discontinue the potential offending
likely diagnosis. The presence of a friction rub is helpful if agent before retesting the testosterone level.
As testosterone replacement therapy is usually a long-
present but its absence does not exclude the diagnosis. Eti
term treatment intervention, it should only be initiated
ologies of acute pericarditis include infection (especially viral
after definitive confirmation of testosterone deficiency in
infection, as is likely in diis patient), autoimmune disease,
neoplasia, uremia, and trauma. the absence of testosterone-lowering therapies and after
The patient has no risk factors for ischemic heart dis weighing the risks and benefits and discussing the multiple
ease and her description of her chest pain is atypical for that potential delivery options for the hormone with the patient.
of coronary artery disease. The widespread concave-upward KEY POINT
ST- segment ECG changes are more consistent with acute
In men with low testosterone levels, a review of
pericarditis rather than ischemia, in which changes localized
to leads associated with die specific regions of the involved medications should be undertaken; many

myocardium would be expected. drugs, including opioids, high-dose corticoster


Like pericarditis, the pain of acute pleuritis worsens oids, and hormonal therapies, can lower testos
with inspiration, may be positional, and can be accompa terone levels.
nied by dyspnea. However, pleuritic chest pain is not con
fined to the retrosternal area, as it is in pericarditis, and Bibliography
Bhasin S, Cunningham GR, Hayes FJ, et al; Task Force, Endocrine
ECG changes would not be expected with pleuritis.
Society. Testosterone therapy in men with androgen deficiency

198
Answers and Critiques

syndromes: an Endocrine Society clinical practice guideline. J Clin Bibliography


Endocrinol Metab. 2010;95(6):2536-2559. [PMID: 205259051
McCullough PA, Ahmed Al?, Zughaib MT, Glanz ED, Di Loreto MJ.
Treatment of hypertriglyceridemia with fibric acid derivatives:
impact on lipid subtractions and translation into a reduction in car
Item 41 Answer: C diovascular events. Rev Cardiovasc Med. 2011;12(4):173-185.
[PMID: 22249508]
Educational Objective: Treat isolated
hypertriglyceridemia.
Item 42 Answer: B
Among the options listed, fenofibrate is die best treatment
option for this patient. His serum triglyceride level is classi Educational Objective: Diagnose epididymitis.
fied as "very high" (>500 mg/dL [5.65 mmol/L]) accord
This patient most likely has epididymitis. Infection or
ing to the National Cholesterol Education Program Adult inflammation of the epididymis often causes pain localized
Treatment Panel III (ATP III) guidelines. He also has a sig
to die superior and posterior aspect of the testicle. The onset
nificantly low HDL cholesterol level. The non-HDL choles
terol level (calculated as total cholesterol - HDL cholesterol) may be acute, subacute, or chronic, and pain may occur
more gradually compared widi torsion. Pain may bc accom
correlates closely with elevated LDL and VLDL concentra
panied by lower urinary tract symptoms of dysuria, urgency,
tions, and is considered a secondary target of therapy when or frequency. Patients with acute epididymitis may be quite
triglycerides are elevated. In patients with an LDL cholesterol
sick, with high fevers and leukocytosis. Risk factors for epi
goal of 130 mg/dL (3.37 mmol/L) or less, as widi this
didymitis include recent sexual activity, heavy exertion, and
patient, the non-HDL cholesterol goal is 160 mg/dL (4.14
mmol/L [30 points higher than the LDL cholesterol goal]). bicycle riding. Distribution is bimodal, with occurrences
This patient's calculated non-HDL cholesterol level is 165 highest in diose younger than 35 years and older than 55
years. In younger patients, sexually transmitted infections
mg/dL (4.27 mmol/L), and so medical therapy should be
such as chlamydial infection and gonorrhea are die most
initiated. Fenofibrate is very effective for reducing serum
likely causes. Older men and men who engage in receptive
triglyceride levels, widi observed decreases of 20% to 50%.
anal intercourse are more susceptible to Escherichia coli,
Additionally, several trials have demonstrated the benefit of
fibrate therapy in the primary and secondary prevention of Enterobacteriaceae, and pseudomonal infection.
Acute prostatitis usually presents with pelvic pain and
cardiovascular outcomes.
lower urinary tract symptoms, such as dysuria, urgency, and
Colcsevelam, a bile acid resin, would not be indicated
for treatment in this patient with hypertriglyceridemia. Bile- frequency. It may cause fever and leukocytosis, but on
acid resins are effective alone or in combination with statins examination, the prostate gland is usually exquisitely ten
for lowering LDL cholesterol level but may raise serum der, which is not die case with this patient.
An indirect hernia may lead to discomfort and fullness
triglycerides, especially in patients with serum triglyceride
levels greater than 400 mg/dL (4.52 mmol/L). These in the scrotum unilaterally, and also cause palpable changes

agents should be avoided as monotherapy in patients with along die course of the inguinal canal on the affected side.
An indirect hernia is unlikely in this patient given lack of a
triglyceride levels above 200 mg/dL (2.26 mmol/L).
Nicotinic acid will reduce serum triglyceride level by scrotal mass or odier physical findings consistent with a her
20% to 50%, decrease LDL cholesterol level by 5% to 25%, nia; in addition, hernias do not cause fever or leukocytosis.
and increase the HDL cholesterol level by 15% to 35%. Orchitis, or inflammation of the testicle, can present as
Treatment with nicotinic acid has been shown to reduce a febrile illness widi testicular pain but pain would be
cardiovascular outcomes in both primary and secondary expected with direct palpation of the testicle and potentially
testicular enlargement.
prevention trials. However, treatment with nicotinic acid
can precipitate gouty attacks, and so should be avoided in Testicular torsion, which occurs when the testicle twists
on die spermatic cord, usually occurs quite acutely. It leads to
patients, such as this one, with a known history of gout.
Omega-3 fatty acids reduce hepatic secretion of decreased blood flow and ischemia and is a surgical emergency.
triglyceride-rich lipoproteins and thereby lower serum Physical findings usually include severe pain, often accompa
triglyceride levels. However, their effectiveness in reducing nied by nausea and vomiting, and a high-riding testicle. This
cardiovascular risk in high-risk patients has not been estab patient's presentation is not consistent with testicular torsion.
lished, and diese agents thus should be considered alterna KEY POINT
tive therapies for treatment of patients with hypertriglyc
Epididymitis is characterized by pain localized
eridemia who cannot tolerate fibric acids or nicotinic acid.
to the superior and posterior aspect of the testi
KEY POINT
cle; patients may be quite sick, with high fevers
For patients with hypertriglyceridemia, fibric and leukocytosis.
acid derivatives, such as fenofibrate, reduce
triglyceride levels and provide benefit in pri Bibliography
mary and secondary cardiovascular prevention. Wampler SM, Llanes M. Common scrotal and testicular problems.
Prim Care. 2010;37(3):613-626, x. [PMID: 20705202]

199
Answers and Critiques

Item 43 Answer: B Item 44 Answer: B


Educational Objective: Manage life-sustaining Educational Objective: Diagnose primary Sjo
care in a critically ill patient. gren syndrome.
It is unclear from die clinical scenario whether or not the This patient has primary Sjogren syndrome. Sjogren syn
patient will need dialysis for an extended period of time. drome is characterized by keratoconjunctivitis sicca,
However, widi the information given, the best course of which causes xerophthalmia (dry eyes), and xerostomia
action is to dialyze temporarily widi the hope that the (dry mouth). The absence of oral mucosal moisture often
patient will either regain kidney function or improve suffi causes difficulty with mastication and swallowing and
ciently to participate in decision making about long-term increases the risk for dental caries and periodontal disease.
dialysis. Although it may be more difficult and resource Vaginal dryness and parotid gland enlargement are fre
intensive to initiate dialysis now and stop later if the patient quently present, and fatigue and arthralgia are common.
fails to improve, it is die course of action that is most likely Some patients with Sjogren syndrome also may develop
to meet both his short-term goal of seeing his great-grand an inflammatory polyarthritis. Additional systemic fea
son graduate and his long-term goal of not being depen tures of Sjogren syndrome include cutaneous vasculitis,
dent on machines. It is important to recognize diat from an peripheral neuropathy, vasculitis that may be associated
ethical and legal perspective, stopping a life-sustaining dier with mononeuritis multiplex, and interstitial nephritis
apy is no different from not starting it, although evidentiary with associated distal renal tubular acidosis. Pulmonary
standards among states and cultural and religious beliefs involvement may develop in patients with Sjogren syn
regarding withdrawing or withholding treatment may vary. drome and most commonly manifests with interstitial
Interventions should not be withheld for fear they cannot lung disease; however, bronchial and bronchiolar disease-
be withdrawn if necessary. also may occur. Abnormal findings on the Schirmer test,
Placing a long-term dialysis catheter may reflect a rea which measures moisture under the lower eyelids, are-
sonable assessment of this patient's likelihood of regain consistent with Sjogren syndrome. Approximately 50% of
ing normal kidney function, but implies disregard for his patients with this syndrome are antinuclear antibody pos
wish not to be dependent on a machine for a long period itive and 60% to 75% of patients with primary Sjogren syn
of time. drome are anti-Ro/SSA antibody positive, and approxi
Withdrawing all treatment would be in conflict with mately 40% of these patients are anti-La/SSB antibody
the wishes of both the patient and his wife so diis is not an positive. A total of 60% to 80% of patients with this con
appropriate choice at diis time. However, it would be dition are rheumatoid factor positive. The presence of
important to meet widi die wife and family to set realistic xerophthalmia and xerostomia accompanied by anti-
expectations given die patient's wishes, age, comorbidities, Ro/SSA and anti-La/SSB antibody positivity and abnor
and the severity of illness. They need to be informed diat mal findings on the Schirmer test have a 94% sensitivity
his survival to discharge even with maximal medical support and specificity for primary Sjogren syndrome.
and dialysis is highly unlikely. Meibomianitis is caused by dysfunction of die meibo
Withholding dialysis now would honor his wish not mian glands responsible for production of die lipid portion
to be dependent on machines, but he would be unlikely of the tear film. Given that this patient also has dry mouth
to survive. Because it is unclear how long he will need and positive anti-Ro/SSA and anti-La/SSB serology, mei
dialysis, it is difficult to tell if it will conflict with his desire- bomianitis is unlikely.
not to be on machines for "a long period of time." Both rheumatoid arthritis and systemic lupus erythe
Although dialysis will not help his poor prognosis from matosus can be associated with Sjogren syndrome, in which
his cancer, it may help him meet his short-term goal of case, multiple systemic symptoms and findings such as joint
surviving until his great-grandson's graduation, so it is involvement, pleuritis, cerebritis, lung dysfunction, and skin
not futile. changes may all occur. Despite this patient's positive anti
nuclear antibody and rheumatoid factor test results, her lack
KEY POINT
of systemic symptoms and a normal physical examination
From an ethical and legal perspective, stopping
(except for xerophthalmia and xerostomia) argue against
a life-sustaining therapy is no different from rheumatoid arthritis or systemic lupus erythematosus as a
not starting it; interventions should not be cause of secondary Sjogren syndrome.
withheld for fear they cannot be withdrawn if
KEY POINT
necessary.
The presence of xerophthalmia and xerostomia
Bibliography accompanied by anti-Ro/SSA and anti-La/SSB
Snyder L; .American College of Physicians Ethics, Professionalism, and antibody positivity and abnormal findings on
Human Rights Committee. American College of Physicians Ethics die Schirmer test have a 94% sensitivity and
Manual: sixth edition. Ann Intern Med. 2012;! 56( 1 Pt 2):73-104.
[PMID: 22213573] specificity for primary Sjogren syndrome.

200
Answers and Critiques

Bibliography which, in turn, may have led to changes in her kidneys' abil
Latkany R. Dry eves: etiology and management. Curr Opin Ophthal ity to metabolize drugs that are renally cleared.
mol. 2008;19(4):287-291. [PMID: 185450008] Polypharmacy is becoming more common as the pop
ulation ages. Twelve percent of patients in the United
States older than 65 years take 10 or more medications each
Item 45 Answer: D week, and adverse drug reactions in the elderly account for
Educational Objective: Evaluate a breast mass in 10% of emergency department visits and tip to 17% of acute
a young woman. hospitalizations. Numerous medications on this patient's
list could cause adverse reactions. For example, gabapentin
This patient should undergo ultrasonography. A slightly
can cause dizziness and weakness and needs to be dose-
tender, discrete, round, soft, and mobile breast mass, with
adjusted in die setting of kidney disease. A review of every
no nipple discharge and no overlying skin changes, is con
patient's medications, particularly in the elderly, should be
sistent with a fibroadenoma or benign cyst and not breast a part of routine care to avoid polypharmacy and potential
cancer. However, no single clinical factor by history or medication-related adverse events.
physical examination has sufficient accuracy to rule in or Although this patient has an apparent decline in her
rule out underlying malignancy, and diagnostic imaging
kidney function as estimated by her serum creatinine level,
should be performed. Ultrasonography serves to distin there is no clear evidence of acute kidney injury being the
guish cystic from solid masses. A cystic mass should be aspi primary cause of her altered mental status.
rated and the fluid sent for cytologic evaluation if bloody or Infection should always be a primary consideration in
recurrent. A solid mass requires biopsy by fine-needle aspi
elderly patients presenting with mental status changes and
ration, core needle, or excision. A benign biopsy in a failure to thrive. However, in this patient there is no evi
woman widi a normal mammogram still requires close fol dence of infection as a cause of her symptoms.
low-up, as documented by a study in which breast cancer The patient has atrial fibrillation and is not receiv
devcloped in 707 of 9087 women with previous benign ing anticoagulation therapy. She does, therefore, have an
breast biopsies followed for a median of 15 years. increased risk for thromboembolic disease. However,
In general, imaging should precede core needle biopsy. her neurologic examination is nonfocal, which would bc
A core needle biopsy will not be necessary if die mass is less consistent with stroke as the underlying cause of her
definitively cystic by ultrasonography. presentation.
The increased density of breast tissue in women 30 to
KEY POINT
35 years of age and younger may limit die utility of mam
Polypharmacy, particularly in elderly patients
mography, making ultrasonography a better first choice.
with multiple comorbid medical conditions, is a
Ultrasonography can readily distinguish cystic from solid
lesions and guide further evaluation, such as aspiration, if frequent cause of adverse events; ongoing
needed. review of the need and appropriate dosing of
Clinical observation and follow-up in 6 months is not medications should be a part of routine care.

appropriate for a palpable breast mass, which should be


evaluated until diagnosis or resolution. Bibliography
Hayes BD, Klein-Schwartz VV, Barrueto F Ir. Polypharmacy and the
KEY POINT geriatric patient. Clin Geriatr Med. 2007;23(2):371-390. [PMID:
17462523]
A palpable breast mass should be evaluated
until diagnosis or resolution.
Item 47 Answer: C
Bibliography
Educational Objective: Manage preoperative care
Rungruang B, Kclley JL 3rd. Benign breast diseases: epidemiology, of a patient with COPD scheduled for an intermedi
evaluation, and management. Clin Obstet Gynecol.
2011;54(1):110-124. [PMID: 21278510] ate-risk procedure.

This patient with COPD undergoing hip surgery should


Item 46 Answer: B begin performing incentive spirometry preoperatively to
reduce his risk of perioperative pulmonary complications
Educational Objective: Recognize a medication-
(PPCs). Risk factors for PPCs including chronic lung dis
related adverse effect in an elderly patient.
ease, older age, use of spinal or general anesthesia, and
The patient's clinical presentation is likely the result of over- surgerv around the diaphragm. Patients with COPD are-
medication in an elderly patient who has significant medical two to five times more likely to experience PPCs. The only
comorbidities and is taking numerous medications. She was therapy proved effective for reducing the risk of PPCs in the
recently hospitalized and had medication adjustments made immediate postoperative period is pre- and postoperative
in the setting of chronic kidney disease, including an increase- lung volume expansion, either via deep breathing or incen
in her diuretic dose, that have led to volume depletion, tive spirometry.

201
Answers and Critiques

Positive airway pressure can be used postoperatively to agree that the nature of the evaluation is governed by die
minimize atelectasis in patients unable to adequately per patient's risk for colon cancer. In young patients (age <40
CONT.
form incentive spirometry (such as those with muscu years) with typical symptoms of hemorrhoid bleeding and
loskeletal or neuromuscular limitations to full lung expan low risk for colon cancer, additional evaluation of the colon
sion). However, diere is no clear role for perioperative is unnecessary. Because of the increasing incidence of colon
positive airway pressure in those who are able to perform cancer with age, patients 40 to 50 years old with typical hem
incentive spirometry. Additionally, continuous positive air orrhoidal symptoms but at low risk for colon cancer should
way pressure (CPAP) is not indicated during the day, noc- probably have at least sigmoidoscopy. Patients aged 50 years
turnally, or both, in the preoperative management of and older should undergo colonoscopy to evaluate die
patients with COPD unless the)' already have an indication source of bleeding provided that routine screening has not
for treatment or are unable to use an incentive spirometer. been recently performed.
Chest radiographs are often ordered preoperatively. If colon cancer is excluded by colonoscopy, his hem
Only 1% to 3% show a clinically significant abnormality, orrhoids can be treated conservatively. Banding and other
however, of which only approximately 0.1% affect clinical invasive procedures are reserved for patients whose hemor
care. Furthermore, most clinically significant chest radi rhoids do not respond to conservative therapy.
ograph abnormalities can be predicted by history and phys Fiber supplementation is an appropriate treatment for
ical examination. Given this patient's unchanged clinical this patient with hard stools but as the only management
symptoms and stable pulmonary examination, imaging for his hematochezia is inappropriate, as this option
would not be expected to lead to an improved clinical out potentially puts him at risk for a missed diagnosis of colon
come associated with surgery. cancer.
Results of pulmonary function testing do not predict Home fecal occult blood testing would likely be posi
the risk of PPCs, and these tests have no role in routine pre tive, but whether positive or negative, die recommendation
operative evaluation. Pulmonary function testing should be for this 46-year-old patient remains the same; colonoscopy
reserved for the evaluation of dyspnea when the cause is not or sigmoidoscopy based upon the report of bright red rec
known. tal bleeding.

KEY POINT KEY POINT

The only therapy proved effective for reducing Patients older than 40 years with hematochezia
the risk of postoperative pulmonary complica should undergo colon cancer evaluation with
tions in the immediate perioperative period is colonoscopy or sigmoidoscopy.
pre- and postoperative lung volume expansion,
either via deep breathing or incentive spirometry. Bibliography
Schubert MC, Sridhar S, Schade RR, Wexncr SD. What every gas-
Bibliography troenterologist needs to know about common anorectal disorders.
World J Gastroenterol. 2009;15(26):3201-3209. [PMID-
Qaseem A, Snow V, Fitterman N, et al; Clinical Efficacy Assessment 195982941
Subcommittee of the American College of Physicians. Risk assess
ment for and strategies to reduce perioperative pulmonary compli
cations for patients undergoing noncardiothoracic surgery: a guide
line from the American College of Physicians. Ann Intern Med. Item 49 Answer: A
2006;144(8):575-580. [PMID: 16618955] Educational Objective: Manage prostate cancer
screening.
Item 48 Answer: B The most appropriate management is to have an informed
Educational Objective: Manage rectal bleeding. discussion with the patient regarding the risks and benefits
of prostate cancer screening. The European Randomized
In diis average-risk patient, the most appropriate next step
Study of Screening for Prostate Cancer included 162,243
would be to proceed to endoscopic evaluation, such as men aged 55 to 69 years. During a median of 9 years, the
colonoscopy, to rule out colonic neoplasia as a source of rate of diagnosis of prostate cancer was higher in the
bleeding. Typically, patients with hemorrhoidal bleeding prostate-specific antigen (PSA)-screened group (8.2%)
report streaks of bright red blood on the toilet paper or on compared with the control (non-screened) group (4.8%)
die outside of a firm stool. There is usually pain associated and there was an absolute, albeit small, mortality benefit
widi defecation. However, the hemorrhoids themselves are
(1410 men would need to be screened and an additional 48
not painful because diere is no innervation to the colonic men would need to be treated for prostate cancer to pre
mucosa proximal to die dentate line. Hemorrhoids are vent one death from prostate cancer). In contrast, the
unlikely to cause serious bleeding. Although hemorrhoids are Prostate, Lung, Colorectal, and Prostate Cancer Screening
a common cause of bright red blood from the rectum, rectal Trial found no benefit for annual concurrent PSA and dig
bleeding should not be considered hemorrhoidal without ital rectal examination (DRE) after 7 to 10 years of follow
additional investigation in older patients. Most authorities
up. Given the conflicting evidence regarding the benefit of

202
Answers and Critiques

prostate cancer screening, the decision of whether or not to in most cases, can be performed more quickly and with less
screen an individual patient should begin with the clinician cost, it is less sensitive than MRI for acute ischemic stroke
having an informed discussion with the patient regarding and does not provide adequate imaging of the vasculature
the risks and benefits of screening and the limitations of the of the posterior circulation.
methods used to screen. Based on the conflicting results of Otolith repositioning, commonly known as the Epley
diese trials, it is not surprising that there is little consensus maneuver, has been shown helpful in resolving symptoms
in terms of screening recommendations. The American of benign paroxysmal positional vertigo (BPPV). BPPV is
Cancer Society supports the need for men to be involved in the most common cause of vestibular (peripheral) dizzi
the decision of whether or not to be screened. In 2012, the ness. Patients with BPPV describe vertigo lasting minutes
U.S. Preventive Services Task Force (USPSTF) published a in duration, with multiple episodes occurring over weeks
formal recommendation statement based on a review of to months. Tinnitus, ear pain, and hearing loss are absent,
existing evidence advising against PSA testing for prostate- and nausea that is sufficiently severe or prolonged to cause
cancer screening in all men. vomiting is rare. The patient's findings are not consistent
Performing a DRE alone is not recommended for with BPPV.
screening owing to the poor test characteristics (positive like Symptomatic treatment with vestibular suppressants is
lihood ratio, 0.53-1.33; negative likelihood ratio, 0.65-14.9). generallv reserved for peripheral causes. The patient's
Aldiough obtaining a PSA level alone and performing recent upper respirator)' tract infection is a risk factor for
a DRE in combination with obtaining a PSA level are fre vestibular neuritis. However, his history and bedside exam
quendy employed in screening for prostate cancer, neither ination are inconsistent with peripheral vertigo. On the
approach should be performed without first having an Dix-Hallpike maneuver, vertigo of peripheral origin is usu
informed discussion with the patient. ally severe, lasts less than 1 minute, has a latency of 2 to 40
KEY POINT seconds, and fatigues with time. The direction of peripheral
The decision of whether or not to screen for nystagmus is not vertical but is horizontal with a rotational
component.
prostate cancer in an individual man should
begin with an informed discussion regarding KEY PO NT
the risks and benefits of screening and die limi Acute vertigo accompanied by vertical nystag
tations of the methods used to screen. mus and nystagmus that is immediate, pro
longed, and nonfatigable on the Dix-Hallpike
Bibliography maneuver is characteristic of central vertigo.
Chou R, Crosvvell JM, Dana T, et al. Screening for prostate cancer: a
review of the evidence for the U.S. Preventive Services Task Force.
Ann Intern Med. 2011;155(11):762-771. Epub 2011 Oct 7. Bibliography
[PMID: 21984740] Post RE, Dickerson LM. Dizziness: a diagnostic approach. Am Fam
Physician. 2010;82(4):361-368, 369. [PMID: 20704166]

Item 50 Answer: B
Item 51 Answer: D
Educational Objective: Diagnose central vertigo.
Educational Objective: Counsel an obese patient
This patient demonstrates vertigo of central origin, a regarding weight reduction.
medical emergency. He should undergo MRI with
This patient should reduce her current caloric intake by
angiography of the brain. Acute vertigo accompanied by
500 to 1000 kcal/d. Consistent reduction in daily dietary
vertical nystagmus and nystagmus that is immediate, pro
caloric intake is the most successful long-term and safest
longed (>1 min), and nonfatigable on the Dix-Hallpike
maneuver is characteristic of a central origin. Central ver weight loss strategy in obese and overweight patients.
Patients who follow a diet that reduces their caloric intake
tigo may be caused by ischemia or infarct in the brain
stem or cerebellum. The patient's inability to stand with by 500 to 1000 kcal/d as compared with their intake that
out support and the absence of diplopia or dysarthria is currently maintaining weight will lose an average of
0.45 to 0.91 kg (1-2 lb) per week. The initial goal should
likely puts his lesion in the cerebellum. Up to one quar
ter of patients with traditional risk factors for stroke who be a loss of 10% of total body weight. If this patient
adheres to this recommendation, she should lose 9.1 kg
present with severe vertigo and nystagmus and who are
unable to stand without support have ischemia or infarc (20 lb) in 4 to 5 months. This degree of weight loss has
tion of the cerebellum. been shown to decrease the health-related consequences
Diffusion-weighted MRI is recommended over CT of obesity, including diabetes mellitus. Because she
scanning as the first-line evaluation for stroke within 12 already has prediabetes, weight loss is important to her
hours of symptom onset owing to its superior sensitivity. long-term health.
Although CT scanning without contrast has excellent sen- Exercise is an important part of a comprehensive
sitivitv for intracranial hemorrhage, is widely available, and, weight loss program that focuses on lifestyle modification.

203
Answers and Critiques

However, without attention to eating habits and caloric In contrast, atorvastatin, lovastatin, and simvastatin are
restriction, exercise alone is not adequate for weight loss. primarily metabolized through the cytochrome P-450 3A4
Current National Institutes of Health and Veterans isoenzyme, and treatment with these medications in com
Affairs guidelines recommend consideration of bariatric bination with diltiazem can increase serum statin levels,
interventions, such as laparoscopic band surgery, in patients placing the patient at higher risk for statin myopathy. These
with a BMI greater than 40 or patients with a BMI of 35 statins should not be combined with diltiazem in a patient
to 40 with obesity-related comorbidities, such as diabetes who is at already high risk for statin-induced myopathy
niellitus, obstructive sleep apnea, or severe joint disease. based on her age, small body frame, hypothyroidism, and
This patient does not meet these recommendations. multiple medications.
Orlistat is a lipase inhibitor that leads to fat malab Rosuvastatin and fluvastatin are metabolized through
sorption. It is FDA approved when used in conjunction the cytochrome P-450 CYP2C9 isoenzyme, and would
with a reduced caloric diet. It is moderately effective in
effectively lower this patient's LDL cholesterol level. How
weight loss (2.9 kg [6.4 lb] at 12 months) but gastrointes ever, rosuvastatin may affect the metabolism of warfarin,
tinal side effects are common. More serious adverse effects,
leading to an increased INR. Caution is therefore necessary
such as severe liver injury and malabsorption of fat-soluble when combining these medications, and this is not the
vitamins, have been reported. Lifestyle management widi safest choice for this patient.
diet and exercise should bc the first step in any weight loss
KEY POINT
program. Medications can be used in conjunction with, but
not as a substitute for, diet and exercise. In patients who require lipid-lowering therapy

KEY POINT
and who are taking multiple medications, a
statin that is renally metabolized, such as
Consistent reduction in daily dietary caloric
pravastatin, has a lower risk of drug-drug
intake is the most successful long-term and
interactions.
safest weight loss strategy in obese and over
weight patients. Bibliography
Joy TR, Hegele RA. Narrative review: statin-related myopathy. Ann
Bibliography Intern Med. 2009;150(12):858-868. [PMID: 19528564] '
Management of Overweight and Obesity Working Group. VA/DoD
clinical practice guideline for screening and management of over
weight and obesity. Washington (DC): Department of Veterans
Affairs, Department of Defense; 2006. Item 53 Answer: C
Educational Objective: Treat heavy menstrual
bleeding.
Item 52 Answer: B
The most appropriate next management step is oral
Educational Objective: Treat elevated LDL cho
lesterol level. medroxyprogesterone acetate. In patients who present widi
menorrhagia (heavy menstrual bleeding) with a known eti
Pravastatin is the safest choice for lowering LDL choles ology, several therapeutic agents can decrease bleeding. For
terol level in this patient. She has recently diagnosed moderate bleeding that can be managed on an outpatient
peripheral arterial disease, which is associated with athero basis, a progestational agent such as medroxyprogesterone
sclerosis; her goal LDL cholesterol level is, therefore, less acetate can be given for 10 to 21 days. The progesterone
than 100 mg/dL (2.59 mmol/L). Pravastatin is one of the will typically act to stabilize the endometrium and stop
preferred statins in patients who are being treated with uterine blood flow.
multiple medications. Estrogen/progesterone-containing oral contraceptives
Statins should bc considered first-line therapy for may be useful in decreasing menstrual blood loss, although
lowering LDL cholesterol levels in this patient, as stud the doses of both agents in most typical formulations would
ies have shown that older patients (ages 65-80 years) likely be inadequate to control the degree of bleeding in
derive similar benefit as younger patients for secondary this patient, particularly with a multiphasic preparation. If
prevention of cardiovascular disease. However, advanced a specific progestational agent is not available, a monopha
age is also a risk factor for statin-related myopathy, and sic oral contraceptive may be dosed four times daily for 5 to
therapy should be chosen carefully to minimize this risk. 7 days, and subsequently reduced to daily dosing until
Female sex, small body frame, hypothyroidism, statin definitive treatment is undertaken.
dosage, and treatment with multiple medications also If die patient were orthostatic or dizzy from blood
influence the likelihood of developing statin-related loss, intravenous estrogen would be appropriate. Parenteral
myopathy. Pravastatin is metabolized by the kidneys; conjugated estrogens are approximately 70% effective in
therefore, its concentration will be unaffected by stopping the bleeding entirely. Pulmonary embolism and
cytochrome isoenzymes that affect the metabolism of venous thrombosis are complications of intravenous estro
other statins or warfarin.
gen therapy.

204
Answers and Critiques

Monitoring the patient for an additional week of KEY POINT


observation is not appropriate given her significant, ongo Patellofemoral pain syndrome is more com
ing blood loss. mon in women than in men and is character
KEY POINT ized by anterior knee pain that is made worse
Medroxyprogesterone acetate for 10 to 21 days with prolonged sitting and with going up and
down stairs.
is effective treatment for moderate menstrual
bleeding. Bibliography
Collado H, Fredericson M. Patellofemoral pain syndrome. Clin Sports
Bibliography Med. 2010;29(3):379-398. [PMID: 20610028]
Fazio SR, Ship AN. Abnormal uterine bleeding. South Med J.
2007;100(4):376-382; quiz 383, 402. [PMID: 17458397]
Item 55 Answer: A
Item 54 Answer: B Educational Objective: Manage chronic dizziness
in an elderly patient.
Educational Objective: Diagnose patellofemoral
pain syndrome. This elderly woman widi chronic dizziness should receive
physical therapy with gait evaluation. Chronic dizziness is a
This patient most likely has patellofemoral pain syndrome, common problem in the elderly. Although not associated
the most common cause of knee pain in patients younger widi excess mortality, chronic dizziness is associated with
than 45 years. Patellofemoral pain syndrome is a clinical falls, syncope, and self-rated health decline. It is often multi
diagnosis and further diagnostic testing such as radiographs factorial and exhaustive work-ups are costly and often unre
is not necessary. Patellofemoral pain syndrome is more warding, particularly in die absence of suggestive or sup
common in women than in men and is characterized by porting findings on history or physical examination.
anterior knee pain that is made worse with prolonged sit Common contributing factors, including medications, sen
ting and with going up and down stairs. The pain is repro sory impairments, neuropadiy, muscle weakness, decondi-
duced by applying pressure to the patella with the knee in tioning, anxiety, depression, and postural hypotension,
extension and moving the patella both medially and later should be identified and addressed. This patient specifically
ally (patellofemoral compression test). presents widi dysequilibrium, a nonvertiginous feeling of
According to the American College of Rheumatol- imbalance or unsteadiness while standing or walking. She has
ogy's clinical criteria, osteoarthritis of the knee can bc diag impairments in vision and hearing. She is at high risk for falls.
nosed if knee pain is accompanied by at least three of the In addition to evaluation for visual and hearing aids, referral
following features: age greater than 50 years, stiffness last to physical therapy for balance and gait training is an appro
ing less than 30 minutes, crepitus, bony tenderness, bony priate first step. In a recent systematic review and meta-analy
enlargement, and no palpable warmth. These criteria are sis of community- and residential care-dwelling older adults,
95% sensitive and 69% specific but have not been validated programs that included exercise and that focused on exercises
for clinical practice. Crepitus of the knee is common in that challenge balance had the greatest relative effects on fall
patients with osteoarthritis between the patella and the rates (RR = 0.58, 95% CI = 0.48-0.69).
femur. Passive range of motion of the knee often elicits pain Administration of topical ophthalmic medication car
at the extremes of flexion and extension. Palpation of the ries some risk of systemic absorption. This patient is being
knee discloses only mild tenderness. This patient has no treated with latanoprost, a prostaglandin analogue that typ
clinical evidence of knee osteoarthritis. ically has minimal systemic effects. P-Blocker ophthalmic
Pes anserine bursitis characteristically produces pain drops, such as timolol, are associated with systemic absorp
that is located near the anteromedial aspect of the proximal tion, which may worsen her symptoms.
tibia. On examination, tenderness is elicited at the level of Although medication side effects and polypharmacy
the tibial tuberosity' (approximately 3.8 cm [1.5 in] below can contribute to dizziness, this patient is not orthostatic,
the level of the medial joint line). Swelling may be present and changing from an ACE inhibitor to an angiotensin
at the insertion of the medial hamstring muscles. This receptor blocker would not likely relieve her symptoms.
Vestibular rehabilitation therapy is most useful for
patient's presentation is not consistent with pes anserine
bursitis. patients with benign paroxysmal positional vertigo. This
Prepatellar bursitis is often caused by recurrent trauma, patient presents with nonspecific dizziness and not true
such as repeated kneeling ("housemaid's knee") but can vertigo.
also be caused by infection or gout. Although the pain is KEY POINT
located anteriorly, examination reveals swelling, tenderness
to palpation (usually localized near the lower pole of the Physical therapy is an effective intervention
to decrease fall risk in elderly patients with
patella), and erythema, all of which arc lacking in this dysequilibrium.
patient.

205
Answers and Critiques

Bibliography Bibliography
Sherrington C, Whitney ]C, Lord SR, Herbert RD, dimming RG, Fanchcr TL, Kravitz RL. In the clinic. Depression. Ann Intern Med.
Close JC. Effective exercise for the prevention of falls: a systematic 2010;152(9):ITC51-15; quiz ITC5-16. [PMID: 20439571]
review and meta-analysis. I Am Geriatr Soc. 20()8;56(12):2234-
2243. I PMID: 19093923]
Item 57 Answer: D
Item 56 Answer: C Educational Objective: Prevent type 2 diabetes
niellitus in an overweight patient.
Educational Objective: Treat depression.
The most appropriate therapies to prevent type 2 diabetes
The most appropriate next treatment step for this patient is
niellitus in this patient are weight loss and exercise. This
to discontinue bupropion and begin a different antidepres
patient has a strong family history of type 2 diabetes mel
sant, such as sertraline. She has moderate depression diat is litus and impaired fasting glucose, defined as a fasting
refractor)' to initial single-agent treatment. The goal of treat
ment is to achieve complete remission within 6 to 12 weeks plasma glucose level of 100 to 125 mg/dL (5.6 to 7.0
and continue treatment for 4 to 9 mondis thereafter. Patients mmol/L). Based on multiple clinical trials, lifestyle modi
fication has been shown to be die most effective interven
should be assessed 2 and 4 weeks after starting therapv' for
tion to prevent type 2 diabetes and its associated cardio
adherence, adverse drug reactions, and suicide risk, and again vascular consequences. The Finnish Diabetes Prevention
at 6 to 8 weeks for response to dierapy. Using a formal tool
for severity assessment (such as die PHQ-9) helps quantify Study and the U.S. Diabetes Prevention Program
die nature of the response; patients are considered to have at (USDPP) both demonstrated a 58% relative risk reduction
in die progression to diabetes with these methods in gen
least a partial response if a 50% or greater decrease in symp
tom score has occurred. Using die PHQ-9, patients can be erally obese, middle-aged persons with impaired glucose
tolerance. The American Diabetes Association recom
classified as complete rcsponders, partial responders, or non-
mends that lifestyle modifications continue to be the stan
responders. Complete responders should continue die same dard approach in diabetes prevention, with the goal being
therapy modality for an additional 4 to 9 months. Treatment to increase regular physical activity by approximately 30
options for partial responders and nonresponders include minutes on most days of the week and to reduce calories
using a higher dose of die same agent (ineffective in this
(to reduce weight) by 7%.
patient), adding a second agent, switching to a new drug, or The USDPP reported a 31% risk reduction in die
adding psychotherapy (patient not interested). Any change in
development of diabetes in patients treated with met
therapy requires periodic follow-up as outlined above. formin. Acarbose reduced die risk of diabetes by 25% in the
Buspirone is an anxiolytic. Given the patient's lack of
Study to Prevent Non-Insulin-Dependent Diabetes Melli
anxiety symptoms and her total lack of response to bupro tus (STOP-NIDDM) trial but had a high drop-out rate
pion, adding diis medication is not appropriate.
This patient remains unresponsive to treatment with owing to gastrointestinal adverse effects. Other studies have
shown a 62% reduction in progression to diabetes widi
bupropion following a dose escalated to the maximal dose.
rosiglitazone in patients with impaired glucose tolerance or
Waiting an additional 8 weeks is unlikely to change man
impaired fasting glucose and an 82% reduction in the pro
agement and will slow this patient's recovery. Switching to
a new drug, either of die same class (unavailable in diis gression to diabetes with pioglitazone. However, the thia
zolidinediones are associated with significant potential
case) or different class, is indicated. Selective serotonin
adverse effects, and die harm of diese drugs may outweigh
reuptake inhibitors (SSRIs), such as sertraline, are the most the benefit of their use in this patient population. Despite
commonly prescribed class of antidepressants. In general, these findings, currendy no drugs are FDA-approved for
SSRIs are well tolerated with low toxicity; however, sexual
the prevention of diabetes. In patients with impaired fast
side effects are common.
Electroconvulsive therapy is indicated in severely ing glucose and other risk factors (BMI >35, a strong fam
ily history, elevated triglyceride level, reduced HDL cho
depressed patients, such as those with profound suicidal lesterol level, hypertension, hemoglobin Alc >6.0%), some
ideation or psychotic features in whom a rapid response to
clinicians will use metformin if lifestyle modifications have
therapy is particularly desirable. This patient has no indica not been successful.
tion for electroconvulsive therapy.
KEY POINT
The most appropriate therapies to prevent type
Patients refractory to a single antidepressant
2 diabetes mellitus are weight loss and exercise.
may respond to a change in therapy, which may
include replacement with another antidepres
Bibliography
sant, either from the same or a different class; Knowler WC, Fowler SE, Hamman RF, et al; Diabetes Prevention
addition of a second antidepressant; or a psy Program Research Group. 10-year follow-up of diabetes incidence
chotherapeutic intervention. and weight loss in the Diabetes Prevention Program Outcomes
Study. Lancet. 2009;374(9702):1677-1686. [PMID: 19878986]

206
Answers and Critiques

Item 58 Answer: D KEY PO NT

Educational Objective: Evaluate relative versus Treatment study outcomes reported in


absolute risk. absolute terms reflect the frequency of a dis
ease in the study population and allow esti
The number needed to treat (NNT) widi drug Z compared mation of "numbers needed"; outcomes
with drug C to prevent one additional case of deep venous
reported in relative terms tend to appear to
thrombosis (DVT) is 100. be of greater magnitude.
Absolute risk (AR) is the risk of a specific disease based
on its actual occurrence, or its event rate (ER), in a group Bibliography
of patients being studied, and is expressed as: Guvatt GH, Sackett DL, Cook DJ. Users' guides to the medical liter
AR = patients with event in group ature. II. How to use an article about therapy or prevention. B.
What were the results and will diey help me in caring for my
total patients in group
patients? Evidence-Based Medicine Working Group. JAMA.
As seen in the table, in this study, the AR for DVT 1994;271(l):59-63. [PMID: 8258890]
in the group treated with drug Z is 25/2500, or 1%, and
the AR for the group treated with drug C is 50/2500,
Item 59 Answer: D
or 2%.
Often, the event rate of a disease in an experimental Educational Objective: Manage ankle sprain.
group (EER) is compared with the event rate in a control The most appropriate management of this patient's ankle
group (CER). When the risk between groups is reduced, sprain is splinting. This patient most likely has a grade II
this difference is termed the absolute risk reduction (ARR), ankle sprain. Correct grading of ankle sprains is important
or if the outcome is of benefit, the difference is called the both in terms of predicting prognosis and for helping to
absolute benefit index (ABI). In this case, patients treated ensure appropriate treatment. Grade II ankle sprains
with drug Z (EER) appear to benefit from treatment with involve partial tears of one or more ankle ligaments and
a lower risk of DVT than patients in the group treated with manifest clinically with moderate pain and some difficulty
drug C (CER). This is expressed as: with bearing weight, which is occurring in this patient.
ABI = | EER-CER | Mild ankle instability and limited range of motion may also
ABI = |1%-2%|= 1% or 0.01 be present. In addition to conservative therapy (rest, ice,
This means that treatment with drug Z benefits compression, elevation) and NSAIDs, joint stabilization is
patients compared with drug C by lowering the risk of indicated and may be achieved with an elastic bandage and
DVT from 2% to 1%, or an absolute difference of 1 %. ankle splint. Adequate splinting will also support early
Assessing treatment studies using absolute measures ambulation and help protect against repeat injur)'. The opti
also allows determination of "numbers needed," which are mal duration of splinting is unclear, although several weeks
estimates of the clinical magnitude of the differences is reasonable with improvement of symptoms. Formal reha
between treatments. In this case, the NNT indicates the bilitation, including proprioceptive training, may be help
number of patients needed to be treated with drug Z, com ful in preventing chronic instability in grade II sprains.
Corticosteroid injections do not have a role in acute
pared with drug C, to obtain one additional beneficial out
ankle injuries and should not be performed.
come. The NNT is calculated as:
NNT = 1/ABI Although MRI may provide more detailed informa
tion than plain radiographs, diey offer no advantage if acute
NNT= 1+0.01 = 100
This means that 100 patients would need to be treated imaging is indicated. MRIs are reserved primarily for eval
uation of ankle sprains that fail to respond to a course of
with drug Z compared with drug C in order to prevent one-
standard therapv' or complex ankle injuries.
additional case of DVT. An inability to bear weight, although common with
Treatment study results may also be reported as rela severe sprains, should raise concern for the possibility of a
tive measures; these measures compare the ratio of two out fracture. Fractures of the base of the fifth metatarsal are
comes without regard to the actual frequency of the out associated with tenderness to palpation of this area on
come in a given study population. In this case, treatment examination. Lateral malleolar fractures are associated with
with drug Z leads to a 50% reduction in risk of DVT com tenderness to palpation of the lateral malleolus. According
pared widi treatment with drug C (25 compared with 50 to the Ottawa ankle rules, ankle radiographs should be
events), even though the actual frequency of DVT in the obtained only in patients with ankle pain who are unable to
study populations does not exceed 2%. Therefore, out bear weight or who have bony tenderness to palpation at
comes expressed in relative terms usually appear of greater the posterior edge of either the lateral or medial malleoli.
magnitude than when expressed in absolute terms; they One systematic review found this set of criteria to be almost
also do not allow calculations of number needed to estimate 100% sensitive in diagnosing fractures. Thus, it would not
clinical impact. be appropriate to obtain ankle radiographs in this patient.

207
Answers and Critiques

Urgent surgical referral would be appropriate for a of hot flushes, even if they do not receive active treatment.
patient with a grade III ankle sprain, which is characterized In numerous studies, venlafaxine, administered at doses of
by complete rupture of one or more ligaments and presents 37.5 mg/d to 150 mg/d, decreases hot flush severity and
widi severe swelling, ecchymosis, instability, and the inabil frequency in approximate!)' 60% of patients (as compared
ity to bear weight. Urgent surgical referral is not needed in with 30% who experienced benefit with placebo treatment).
a patient widi a grade II ankle sprain. Paroxetine is similarly beneficial; in contrast, few studies
have shown efficacy with fluoxetine or citalopram.
KEY POINT
Ankle radiographs should only be obtained in Gabapentin and clonidine are two additional nonhormonal
treatments that reduce hot flushes, but attendant side-
patients with acute ankle pain who are unable effects may limit their use in some patients.
to bear weight or who have bony tenderness to
Vaginal estrogen therapy is typically used for the iso
palpation at the posterior edge of eitiier the lat lated treatment of vaginal dryness, pruritus, and dyspareu
eral or medial malleoli. nia. Treatment with vaginal estrogen tablets will improve
local vaginal symptoms, but will not improve menopausal
Bibliography vasomotor symptoms.
Seah R, Mani-Babu S. Managing ankle sprains in primary care: what
is best practice? A systematic review of the last 10 years of evidence. KEY POINT
Br Med Bull. 2011;97:105-135. [PMID: 20710025]
Owing to cardiovascular and thromboembolic
risks, systemic hormone therapy is not recom
Item 60 Answer: E mended for treatment of menopausal vasomo
tor symptoms in women older than 60 years
Educational Objective: Treat menopausal vaso
motor symptoms. who experienced menopause at die median age.

This 61-year-old woman with cardiovascular risk factors Bibliography


and a history of deep venous thrombosis should be Nelson HD, Vesco KK, Haney E, et al. Nonhormonal therapies tor
started on a nonhormonal therapy for her hot flushes. menopausal hot Hashes: systematic review and meta-analysis. JAMA.
2006;295(17):2057-2071. [PMID: 16670414]
Certain antidepressants, including serotonin-norepineph-
rine reuptake inhibitors such as venlafaxine, are effective
nonhormonal medications for reducing menopausal vaso Item 61 Answer: C
motor symptoms.
Educational Objective: Diagnose central retinal
Approximately 10% of menopausal women experience vein occlusion.
hot flushes for 7 to 10 years after the cessation of menses.
This patient is continuing to experience frequent and severe This patient most likely has central retinal vein occlusion
hot flushes which have been refractory to conservative ther (CRVO). Patients with CRVO experience acute onset of
apy and are decreasing her quality of life; thus, pharmaco painless blurry vision due to reduced venous outflow and
logic therapy is warranted. Systemic estrogen therapy is die- vascular edema in the eye. CRVO is characterized by optic
most effective treatment for the relief of menopausal hot disc swelling, dilated and tortuous veins, flame-shaped reti
flushes and must be coadministered with progesterone in nal hemorrhages, and cotton-wool spots ("blood and
women widi an intact uterus. However, combined estrogen thunder"). Because the nerve supply to the eye remains
and progesterone therapy has been shown to increase the intact but vision is decreased, an afferent pupillary defect
risk of several adverse outcomes, including coronary heart in the affected eye may be present. CRVO is most com
disease, stroke, invasive breast cancer, and venous throm monly encountered in older patients with hypertension
boembolism. The North American Menopause Society and atherosclerotic vascular disease. Cases have also been
guideline notes that women older than 60 years who expe associated with acute carotid artery dissection and condi
rienced natural menopause at the median age and have tions associated with increased blood viscosity, such as
never used hormone therapy will have elevated baseline polycythemia vera, sickle cell disease, and leukemia. Prog
risks of cardiovascular disease, venous thromboembolism, nosis depends on the degree of visual impairment at the
and breast cancer; hormone therapy, therefore, should not onset of symptoms. There is no generally accepted acute
be initiated in diis population without a compelling indica management, but a thorough investigation into possible-
tion and only after appropriate counseling and attention to etiologies should be undertaken.
cardiovascular risk factors. Moreover, this patient has a his Acute angle closure glaucoma is characterized by nar
tory of deep venous thrombosis, which is an absolute con rowing or closure of die anterior chamber angle, which
traindication to initiating hormone therapv'. impedes the trabecular drainage system in the anterior
Several nonhormonal medications have been found to chamber, resulting in elevated intraocular pressure and
be effective for the treatment of menopausal hot flushes. damage to the optic nerve. Typical signs and symptoms
Notably, there is a significant placebo effect: in most stud include significant pain, diminished visual acuity, seeing
ies, approximately one-third of women will experience relief halos around lights, a red eye, headache, and a dilated pupil.

208
Answers and Critiques

The globe may feel firm owing to increased intraocular points), class B (7-9 points), and class C (>10 points) scores
pressure (often to 30 mm Hg or higher). Retinal examina have postoperative mortality rates of approximately 10%,
tion may be normal or demonstrate optic cupping if 30%, and 80%, respectively. Patients with a CTP class C
chronic narrow-angle glaucoma is present. score are usually advised to avoid elective surgery.
Central retinal artery occlusion (CRAO) classically Patients with acute alcoholic hepatitis may present with
presents in a 50- to 70-year-old patient as a painless, abrupt leukocytosis, jaundice, hepatomegaly, and right upper
blurring or loss of vision diat occurs in the early morning quadrant pain. In addition to abstinence from alcohol and
hoursusually between midnight and 6 AM. It results from nutritional therapy, pharmacologic therapy widi cortico
an embolic or thrombotic event in the ophthalmic artery. steroids may be beneficial in patients widi severe alcoholic
It is typically unilateral. On examination, visual acuity is hepatitis. However, this patient's cirrhosis is a result of
markedly diminished in the affected eye to either finger long-term chronic alcohol use, and there is no indication
for corticosteroid therapy.
counting or light perception. There is an afferent pupillary
defect. On funduscopic examination, the retina appears Patients with cirrhosis are predisposed to bleeding
because of quantitative and qualitative platelet dysftinction
pale, eidier scgmentally or completely. The fovea may
as well as a deficiency of hepatically produced clotting fac
appear as a cherry red spot. Interruption of die venous
blood columns may be recognized with the appearance of tors. The INR elevation in these patients is usually not
caused by vitamin K deficiency. Although patients with
"box-earring"rows of slowly moving corpuscles sepa
rated by clear intervals. These findings are not present in coagulopathy are sometimes treated with oral vitamin K to
this patient. reverse any nutritional deficiencies, administration of vita
min K would only minimally change the inherent risks of
Symptoms of retinal detachment include diminished
vision, photopsia (flashes of light), abrupt onset of multiple- surgery in this patient.
floaters in the vision, and mctamorphopsia (wavy vision). KEY POINT
Retinal detachment may result from trauma or occur spon Patients with a Child-Turcotte-Pugh class C
taneously, particularly in persons with myopia. It is typically score are usually advised to avoid elective
unilateral. On funduscopic examination, retinal detachment
surgery.
is characterized by distortion, folding, and tearing of the
retina, which were not seen in diis patient. Bibliography
KEY POINT O'Leary JG, Yachimski PS, Friedman LS. Surgery in the patient with
liver disease. Clin Liver Dis. 2009;13(2)l211-231. [PMID:
Central retinal vein occlusion is characterized 19442915]
by acute onset of painless blurry vision and
optic disc swelling, dilated and tortuous veins,
Item 63 Answer: B
flame-shaped retinal hemorrhages, and cotton
wool spots ("blood and thunder"). Educational Objective: Counsel a patient using
motivational interviewing.
Bibliography
It would be appropriate to determine which lifestyle change
Ahmed R, Foroozan R. Transient monocular visual loss. Neurol Clin.
this patient believes is most important to implement. Key
2010;28(3):619-629. [PMID: 20637992]
features of motivational interviewing are diat die patient
chooses the agenda, the provider is not in "control" and
Item 62 Answer: C does not tell the patient what he or she should do, and the
Educational Objective: Manage a perioperative provider assesses the patient's sense of the importance of
issues for them and their level of confidence in making these
patient with cirrhosis.
changes, which should be small and incremental. Physicians
This patient has cirrhotic liver disease, placing him at a high can support the patient's choice and provide advice on how-
risk of perioperative complications, and nonoperative man to overcome barriers to implementation.
agement of his hip osteoarthritis should be recommended. The patient has indicated a willingness to make impor
Patients with cirrhosis have an increased risk of periopera tant lifestyle changes; he now needs to be empowered to
tive morbidity and mortality. Surgery and anesthesia in identify those changes he wishes to make and identify bar
these patients are risk factors for fulminant hepatic failure, riers and facilitators to those changes. He does not need to
and cirrhosis itself can predispose to bleeding, infection, be told to lose weight; die patient has already acknowl
adverse reactions to drugs, and poor wound healing. The edged die need to make lifestyle changes. After the patient
risk of perioperative complications can be quantified using identifies the changes diat are important to him, it would
a number of scales, including Child-Turcotte-Pugh (CTP) be appropriate to ascertain his confidence in making those
classification. The CTP score is based on the presence of changes, but to do so prior to this step would be prema
ascites and encephalopathy, as well as serum bilirubin, albu ture. Having the patient control the agenda and select the
min, and INR values. Patients with CTP class A (0-6 changes he believes are most important and identify the

209
Answers and Critiques

associated barriers and facilitators is more likely to be effec KEY POINT


tive dian die physician providing advice on smoking or Metabolic syndrome is diagnosed by the pres
other lifestyle changes that the patient may deem as less ence of three or more of five abnormalities:
important. increased waist circumference, elevated sys
KEY POINT tolic or diastolic blood pressure, decreased
Key features of motivational interviewing are HDL cholesterol level, elevated triglyceride
that the patient chooses the agenda, the level, and elevated fasting plasma glucose
level.
provider is not in "control" and does not tell
the patient what he or she should do, and the
Bibliography
provider assesses the patient's sense of the
Tota-Maharaj R, Defilippis AP, Blumenthal RS, Blaha MI. A practical
importance of issues and level of confidence in approach to die metabolic syndrome: review of current concepts
making changes. and management. Curr Opin Cardiol. 2010;25(5):502-512.
[PMID: 20644468]
Bibliography
Rollnick S, Butler CC, Kinnerslcy P, Gregory J, Mash B. Morivational
interviewing. BMJ. 2010;340:1242-1245. [PMID: 20423957] Item 65 Answer: B
Educational Objective: Manage advanced cancer

Item 64 Answer: C pain.

Educational Objective: Diagnose metabolic The next step in the treatment protocol for this patient
would bc a NSAID, such as ibuprofen. The World Health
syndrome.
Organization analgesic ladder represents a useful frame
Given diis patient's hypertension, lipid profile, and abdom work for pharmacologic treatment of pain. Nonnarcotic
inal obesity, she meets the criteria for metabolic syndrome. treatments, such as aspirin, acetaminophen, or NSAIDs, are
The diagnosis of metabolic syndrome (Adult Treatment used for mild pain (score of 1-3 on die 0-10 pain intensity
Panel III criteria) is made by the presence of three or more scale). Moderate pain (pain score 4-6) is treated with a
of die following five criteria: (1) waist circumference >40 in combination of opioids and nonnarcotic pain relievers. If
(102 cm) in men and >35 in (88 cm) in women; (2) sys these agents are combined in a single pill (such as oxy
tolic blood pressure >130 mm Hg or diastolic blood pres codone and acetaminophen) to reduce polypharmacy, care
sure >85 mm Hg; (3) HDL cholesterol level <40 mg/dL must be taken to avoid inadvertent overdosing of the non
(1.04 mmol/L) in men and <50 mg/dL (1.30 mmol/L) narcotic component when die need for the opioid ingredi
in women; (4) triglyceride level >150 mg/dL (1.70 ent increases. The daily cumulative acetaminophen dose
mmol/L); and (5) fasting plasma glucose level >110 (<4 g) limits the dosing of the opioid in combination med
mg/dL (6.1 mmol/L). ications. Severe pain (pain score 7-10) is mainly treated
The clinical importance of identifying the metabolic syn with opioids. Adjunctive dierapies (antidepressants, anti
drome is the increased risk for cardiovascular disease and type convulsants, corticosteroids, muscle relaxants) can be used
2 diabetes mellitus in those with diis diagnosis. Persons with at all levels of the analgesic ladder. NSAIDs can be especially
the metabolic syndrome should receive aggressive interven effective in patients with bone pain, particularly if diere is
tion focused on lifestyle modification to decrease weight, an inflammatory component.
increase physical activity, and implement a nonathcrogenic The use of gabapentin, an anticonvulsant, is a useful
diet, in addition to treating the significant metabolic abnor adjuvant pain medication and may be particularly helpful in
malities diat define die syndrome. The metabolic syndrome patients widi neuropathic pain but would be a poor choice
is frequendy identified in patients widi polycystic ovary syn for this patient widi bone pain.
drome, and has also been associated with die development of Meperidine is rarely appropriate for oral use owing to
other disorders, including fatty liver disease, obstructive sleep variable oral bioavailability and the accumulation of active
apnea, hyperuricemia, and gout. metabolites with prolonged use at high doses or in kidney
The patient does not meet the criteria for hyper failure. Such accumulation lowers die seizure threshold and
triglyceridemia, although American Heart Association causes central nervous system symptoms, such as tremors,
guidelines recommend an optimal triglyceride level of twitching, and nervousness.
below 100 mg/dL (1.13 mmol/L), and this would be an Oral administration is die preferred route for opioid
appropriate goal for this patient for lifestyle modifications. analgesics because of its convenience, low cost, and ability
Impaired fasting glucose (prediabetes) is defined as a to produce stable opioid blood levels. Intramuscular injec
fasting blood glucose level of 100 to 125 mg/dL (5.6-6.9 tions are not recommended because of the associated pain,
mmol/L). She does not have this diagnosis, and the diag unreliable absorption, and relatively long interval to peak
nosis of the metabolic syndrome does not stricdy require drug concentrations. This patient should be tried on a
abnormalities in glucose metabolism. NSAID prior to an opioid medication.

210
Answers and Critiques

KEY POINT Item 67 Answer: A


Nonopioid agents are the first step in the man Educational Objective: Manage fall risk in an
agement of mild cancer pain and should be ini elderly patient.
tiated before progressing to an opioid agent.
This patient appears to have had a fall without any indica
tion of gait abnormality, visual deficit, or weakness. Her
Bibliography
Swetz KM, Kamal AH. Palliative care. Ann Intern Med. bumping into a wall at night may have caused the fall. It
2012;156(3):ITC21. [PMID: 22312158] would be important to know if she has night lights in each
room, because lack of proper lighting would contribute to
her risk for falls. Installing night lights in each room is a
Item 66 Answer: D simple and inexpensive fall prevention strategy. Addition
Educational Objective: Treat dyslipidemia. ally, an evaluation for home safety is always an appropriate
consideration for patients who have fallen, because hazards
This patient should be started on statin therapy. He has a can be found in two-thirds of homes. Other causes for falls,
very high LDL cholesterol level despite a healthy diet and such as die presence of rugs and diresholds between rooms,
regular vigorous exercise. According to the National Cho have been ruled out. If the patient was already using night
lesterol Education Program (NCEP) Adult Treatment lights, dien a home safety evaluation for other hazards,
Panel III guidelines, persons with two or more cardiovas including general clutter, uneven lighting, and unsafe or
cular risk factors, such as this patient, should have an LDL absent hand rails, would be important.
cholesterol goal of below 130 mg/dL (3.37 mmol/L), and In the Timed "Up & Go" (TUG) test, the patient is
drug therapy should bc considered in those with LDL cho asked to arise from a chair, walk 10 feet, turn around, and
lesterol levels of 160 mg/dL (4.14 mmol/L) or above. sit in die same chair. Those completing the task in more
The non-HDL cholesterol level is calculated by sub than 14 seconds are considered high risk for subsequent
tracting die measured HDL cholesterol from the total cho falling. This patient's gait and TUG test are normal. An
lesterol level. In patients widi elevated triglycerides, the individualized exercise program is probably unnecessary in
non-HDL level correlates widi high concentrations of diis patient with normal gait and motor examination. Addi
atherogenic lipoproteins, including VLDL remnants. The tionally, meta-analyses have shown some harm with exercise
NCEP guidelines designate the non-HDL cholesterol level
programs in the elderly, especially in frail patients.
as a secondary target for therapy, after goal LDL cholesterol This patient's normal gross visual acuity on office test
levels have been achieved. Calculation of die non-HDL ing suggests against an ophthalmologic contribution to her
cholesterol level will not alter this patient's current man fall. Her normal gait and timed Up & Go test indicate that
agement strategy. she does not require a walker.
The U.S. Preventive Services Task Force (USPSTF)
has recendy reviewed die utility of adding several novel car KEY POINT
diac risk markers to traditional methods of cardiac risk strat An evaluation for home safety is important for
ification. According to their guidelines, there is insufficient patients who have fallen and may include the
evidence to recommend for or against the use of high-sen presence of rugs and thresholds between
sitivity C-reactive protein levels for assessing cardiovascular rooms, use of night lights, general clutter,
risk. Measurement of die high-sensitivity C-reactive protein uneven lighting, and unsafe or absent hand
level in this patient is not likely to affect management. rails.
Fibric acid derivatives are effective at lowering triglyc
eride levels, but have a lesser effect on LDL cholesterol Bibliography
level. Although this patient has borderline-high triglyceride Wyman JF, Croghan CF, Nachreiner NM, et al. Effectiveness of edu
cation and individualized counseling in reducing environmental
levels, lowering his LDL cholesterol level would signifi- hazards in the homes of community-dwelling older women. J Am
candy reduce his cardiovascular risk, and some reduction in Geriatr Soc. 2007;55(10):1548-1556. [PMID: 17908058]
his triglyceride levels would be expected with statin therapy.
KEY POINT
Item 68 Answer: D
In moderate-risk patients with LDL cholesterol
Educational Objective: Diagnose schizophrenia.
levels of 160 mg/dL (4.14 mmol/L) or above,
therapeutic lifestyle changes and pharmacologic This patient's negative symptoms of social wididrawal and
lipid-lowering therapy are warranted. flat affect and positive symptoms of paranoia and hearing
voices are consistent widi schizophrenia. Signs and symp
Bibliography toms should be present for at least 1 month and some for
U.S. Preventive Services Task Force. Using nontraditional risk factors at least 6 months, as in this patient. Her family history of
in coronary heart disease risk assessment: U.S. Preventive Services
Task Force recommendation statement. Ann Intern Med. schizophrenia in a first-degree relative puts diis patient's
2009;151(7):474-482. [PMID: 19805770] lifetime risk for developing this disorder between 6% and

211
Answers and Critiques

17%. Positive symptoms of schizophrenia include paranoid KEY POINT


delusions, hearing voices, and hallucinations; negative Contraception with a combination of a barrier
symptoms include flat affect, social withdrawal, and lack of method and a hormonal method is recom
interest or enjoyment in life. Thought tends to be disorga mended in patients in whom the risks of unin
nized, widi confused speech and rapid shifts in topic. No tended pregnancy and STIs are high.
symptom or sign is pathognomonic for schizophrenia.
Women tend to present later in life than males, who tend Bibliography
to present in their teenage years or early twenties. Workowski KA, Berman S; Centers for Disease Control and Preven
Both bipolar disorder and major depressive disorder tion (CDC). Sexually transmitted diseases treatment guidelines,
should be included in the differential diagnosis at this time. 2010. MMWR Rcco'mm Rep. 2010;59(RR-12):1-110. [PMID:
21160459]
Up to 22% of patients diagnosed with schizophrenia will
have their diagnosis changed during subsequent hospital
izations, so it is important to keep other medical and psy Item 70 Answer: A
chiatric causes of paranoia, auditory hallucinations, and
social withdrawal in the differential diagnosis. This patient Educational Objective: Treat benign prostatic

requires urgent psychiatric evaluation and possibly admis hyperplasia.


sion to die hospital for further evaluation. This patient has classic findings of symptomatic benign pro
Aldiough this patient has been using marijuana, there static hyperplasia (BPH), and combination therapy with
is little evidence that Cannabis dependence or abuse leads both an a-blocker and a 5a-reductase inhibitor is indicated.
to psychosis to the degree she is experiencing. The American Urological Association (AUA) guideline on
KEY POINT
treatment of BPH recommends that patients widi an AUA
To make a diagnosis of schizophrenia, signs symptom score greater than 7 (questionnaire available at
and symptoms should be present for at least 1 u-ww2.niddk.nih.gov/NR/rdonlyres/8E99FCF4-8A92-
month and some for at least 6 mondis. 43EE-8E47-5B70D634938A/0/AUABPH.pdf) or who
are bothered by their symptoms receive treatment for
BPH. 5a-Reductase inhibitors (5-ARIs), such as finasteride
Bibliography
Schultz SH, North SW, Shields CG. Schizophrenia: a review. Am Fam and dutasteride, may bc suitable in patients who have failed
Physician. 2007;75(12):1821-1829. [PMID: 17619525] to respond to or do not tolerate a-antagonists and those
with severe symptoms. The Medical Therapy of Prostate
Symptoms Study demonstrated that in the long term,
Item 69 Answer: B among men with larger prostates, combination therapv' is
Educational Objective: Recommend contracep superior to either a-blocker or 5-ARI rherapy in prevent
tion options for a young woman. ing progression and improving symptoms. Similarly, the
ComBAT trial demonstrated that combination therapy
Given this patient's demographic profile and behavior pat resulted in significantly greater improvements than single-
tern, her risks of unintended pregnancy and sexually trans agent therapy. Combination therapy was associated with a
mitted infection (STI) are very high, and a combination of
higher incidence of adverse effects than monotherapy.
barrier and hormonal methods is recommended. Condom 5-APJs decrease die production of dihydrotestos
use with a repository form of progesterone would bodi
terone, thereby arresting prostatic hyperplasia. Because
help prevent STIs and minimize die risk of contraceptive shrinkage is slow, symptoms often do not improve until
failure due to medication noncompliance. after 6 months of therapy. Therefore, these agents are not
Although condom use with combination estrogen- typically used as initial monotherapy for BPH, and switch
progesterone pills would be an effective regimen for both ing this patient from an a-antagonist to a 5-ARI would
prevention of STIs and contraception, it would require not be indicated. Side effects include erectile and ejacula-
daily medication adherence. tory dysfunction, reduced libido, gynecomastia, and
Although long-acting progesterone compounds such breast tenderness.
as depot medroxyprogesterone acetate are recommended
a-Antagonists (terazosin, doxazosin, alftizosin, tamsu
in adolescents and young adults in whom user compliance
losin, silodosin) relax the prostatic smooth muscle in the
may be unreliable, they do not offer protection from STIs. bladder outflow tract, act rapidly (usually within 48 hours),
A contraceptive vaginal ring containing estrogen and and are considered first-line treatment, producing a clinical
progesterone is a method of non-coitally based contracep response in 70% of men. All drugs in this class have similar
tion that many users find more convenient than daily pills.
efficacy and tend to improve symptoms by 30% to 40%.
It is an effective means of contraception, although it does
Although some agents are more selective for prostate-spe
require diat the user be able to properly place die ring. It cific a-receptors and therefore have less effect on systemic
does not, however, confer protection against STIs, and blood pressure, there is not a significant difference in effec
therefore would not be a method of choice in this patient. tiveness in treating BPH. Therefore, there is no benefit in

212
Answers and Critiques

switching between a-antagonists in this patient, as he has between the ages of 11 and 21 years. The vaccine is not
tolerated his current treatment well. Abnormal ejaculation indicated for this 30-year-old woman.
is a side effect and appears similar for all a-antagonists. Current recommendations are that a tetanus and diph
Elderly patients are less likely to discontinue treatment theria (Td) vaccine be routinely administered every 10
because of ejaculatory dysfunction than because of cardio years. Owing to an increased incidence of pertussis,
vascular side effects, such as postural hypotension, dizzi thought in part to be related to waning immunity from
ness, and headaches. childhood vaccination, all adults are recommended to
A 4-week course of a fluoroquinolone antibiotic would receive a single tetanus, diphtheria, and acellular pertussis
be appropriate dierapy for chronic bacterial prostatitis. (Tdap) vaccination regardless of the interval since their last
However, this patient has no symptoms or signs of prosta Td booster (although it may be given in place of a decen
titis on examination and a normal urinalysis, making this nial Td booster if scheduled); this is a particularly important
recommendation for persons aged 65 years or older
diagnosis unlikely.
because of the high burden of associated disease in this
KEY POINT
patient population. In addition, all postpartum women,
In patients with symptomatic benign prostatic healdi care workers, and adults who have close contact with
hyperplasia, the combination of an a-blocker infants younger than 12 months should receive a one-time
and a 5a-reductase inhibitor is associated with Tdap booster if not already given. This patient is not due
greater improvement in symptoms and more for a routine repeat Td booster for another 5 years and has
side effects than treatment with either agent no indications to receive either a Td or Tdap vaccination at
alone. this time.
KEY POINT
Bibliography Annual seasonal influenza vaccination is recom
Juliao AA, Plata M, Kazzazi A, Bostanci Y, Djavan B. American Uro
logical Association and European Association of Urology guidelines mended for all adults, regardless of risk factors.
in the management of benign prostatic hypertrophy: revisited. Curr
Opin Urol. 2012;22(l):34-39. [PMID: 22123290] Bibliography
National Center for Immunization and Respiratory Diseases. General
recommendations on immunization recommendations of the
Item 71 Answer: C Advisory Committee on Immunization Practices (ACIP). MMWR
Recomm Rep. 201 l;60(2):l-64. [PMID: 21293327]
Educational Objective: Manage influenza vacci
nation in a healthy woman.
Item 72 Answer: C
This healthy 30-year-old woman should receive a seasonal
influenza vaccination. The Centers for Disease Control and Educational Objective: Manage chronic fatigue
Prevention currently recommends that all adults be vacci syndrome.
nated annually against influenza, regardless of risk factors. The most appropriate management for this patient is to
Vaccination usually takes place between September and begin a graded exercise program. Chronic fatigue syn
March in the Northern hemisphere. Healthy adults can bc drome (CFS) is defined as medically unexplained fatigue
vaccinated with either an inactivated vaccine injected intra that persists for 6 months or more and is accompanied by
muscularly or a live attenuated intranasal vaccine. at least four of the following symptoms: subjective memory
The hepatitis B vaccine is indicated for all children and impairment, sore diroat, tender lymph nodes, muscle or
adolescents through age 18 years, persons widi HIV or joint pain, headache, unrefreshing sleep, and postexertional
other recent sexually transmitted infections, persons who malaise lasting longer than 24 hours. Management of CFS
are sexually active but not monogamous, workers with is challenging and is geared toward managing symptoms
occupational exposure to blood, clients and staff of institu and maintaining function, radier than seeking cure. A com
tions for the developmental!)' disabled, correctional facility prehensive, individually tailored approach is required, typ
inmates, illicit drug users, persons with diabetes mellitus ically based on nonpharmacologic therapy, such as lifestyle
who are younger dian 60 years, and persons with advanced modification and sleep hygiene. Specific treatment options
chronic kidney disease who are approaching hemodialysis. that have been demonstrated to improve symptoms include
Hepatitis B vaccination is also indicated for diose planning graded exercise programs and cognitive-behavioral therapy
travel to an endemic area and those with an increased risk (CBT). CBT in this setting is targeted in part at breaking
for morbidity related to die disease, as well as for persons the cycle of effort avoidance, decline in physical condition
who request vaccination. This patient has no indication for ing, and increase in fatigue, and can work well in combina
hepatitis B vaccination. tion widi graded exercise in this regard. CBT reduces
The human papillomavirus vaccine is licensed for males fatigue and improves functional status.
and females aged 9 through 26 years and is recommended Although Epstein-Barr virus and a host of other infec
for females between the ages of 11 and 26 years and males tious agents have been considered in the padiogenesis of

213
Answers and Critiques

CFS, none have been borne out by careful study; therefore, difficult situation, it would not address the potentially com
antiviral therapy, including acyclovir, has no role in die plex causes of her symptoms or allow identification of more-
treatment of CFS. A variety of other medications have been appropriate therapeutic interventions.
tried, including corticosteroids, mineralocorticoids, growth Patients with physical complaints that are inconsistent
hormone, and melatonin, but with no clear evidence of or without evidence of underlying pathology are particu
benefit, and are not indicated for this patient. larly difficult to manage, especially in the context of a busy
Current evidence is not sufficiently robust to recom medical practice. These persons are often evaluated by mul
mend dietary supplements, herbal preparations (evening tiple subspecialist physicians and undergo extensive testing,
primrose oil), homeopathy, or even pharmacotherapy. both as an attempt to achieve a diagnosis and because of
Patients widi concomitant depression should be treated time considerations. In this patient, while not inappropri
with antidepressants. Aldiough no specific class of antide ate, gastroenterology evaluation would likely not be of high
pressant is recommended in this setting, tricyclic antide yield given her nonspecific clinical symptoms and examina
pressants are often utilized in patients with CFS and depres tion findings, and would likely not address the issues
sion owing to their adjunct effectiveness in treating muscle
underlying her complaints.
pain. Transferring the care of the patient to anodier physi
KEY PO NT cian would not be desirable. One of die most important
Effective treatment options for chronic fatigue aspects of dealing with patients seen as difficult is to recog
nize the potential psychosocial and emotional factors that
syndrome include graded exercise programs and
may be contributing to a patient's symptoms, even if they
cognitive-behavioral therapy. are not recognized as such by the individual. Equally
Bibliography important is for physicians to understand their own, poten
Reid S, Chalder T, Clcare A, Hotopf M, Wessely S. Chronic fatigue- tially negative, emotions about the patient. Having some
syndrome. Clin Evid (Online). 2011;05:( 1101)1-52. [PMID: insight into these factors will hopefully allow the patient
216159741 and physician to work togedier collaboratively to develop
an appropriate diagnostic and treatment plan.
Item 73 Answer: A KEY POINT

Educational Objective: Manage a difficult clinical When involved in a difficult clinical encounter,
encounter. an understanding of the patient's psychosocial
In this case of a patient with persistent symptoms not attrib and emotional factors potentially contributing
to his or her symptoms as well as the physi
utable to a specific cause and who is angry and demanding,
an appropriate response is to provide empadietic support cian's own feelings about the patient is essential
ive care at relatively frequent intervals. This would be best in establishing and maintaining an effective
accomplished by initiating an ongoing discussion with the therapeutic relationship.
patient to better understand the potential causes and sig
nificance of her pain symptoms. Bibliography
Kroenke K. Unburdening the difficult clinical encounter. Arch Inter
Multiple studies indicate that physicians classify about Med. 2009;169(4):333-334. [PMID: 19237715]
15% of clinical encounters as difficult. Difficult encounters
often involve patients who have a depressive or anxiety dis
order, poorer functional status, unmet expectations, Item 74 Answer: D
reduced satisfaction, and greater use of health care services.
Educational Objective: Appropriately administer
Physicians often report the most difficult)' with patients by the pneumococcal vaccine in a patient who has been
whom they feel manipulated or frustrated, patients who are
time consuming and have unrealistic demands, patients previously vaccinated.
who express anger, patients who do not follow the physi This man should receive a single pneumococcal polysac
cian's recommendations, and diose who interrupt physician charide vaccination at age 65 years. Adults 65 years and
routine and make extra work. Patients who have somatiza older should be immunized against pneumococcal pneu
tion disorder, chronic pain, substance abuse, or have an monia. The vaccine contains 23 antigen types of Strepto
undiagnosable medical problem are often labeled as diffi coccus pneumoniae and protects against 60% of bacteremie
cult. Patients with identifiable personality disorders such as disease. The vaccine is also recommended in some popula
borderline, dependent, histrionic, obsessive, or antisocial tions of younger patients, including Alaskan natives and
personality disorders also are frequently labeled as difficult. certain American Indian populations; residents of long-
It would be inappropriate to treat this patient's non term care facilities; patients who are undergoing radiation
specific pain with opioid medications based only on a therapy or arc on immunosuppressive medication; patients
patient's request without a better understanding of its eti who smoke; and patients with chronic pulmonary disor
ology. Although this is an expedient way of dealing with a ders (including asthma), diabetes mellitus, cardiovascular

214
Answers and Critiques

disease, chronic liver or kidney disease, cochlear implants, Gemfibrozil is a fibric acid that is typically used for the
asplenia, immune disorders, or malignancies. There is no treatment of hypertriglyceridemia. It reduces LDL choles
information on vaccine safety during pregnancy. The vac terol levels by 10% or less; in addition, gemfibrozil
cine is reasonably effective, widi high levels of antibody typ monodierapy in patients with hypertriglyceridemia can
ically found for at least 5 years. Currendy, immunocompe actually raise LDL cholesterol levels. Gemfibrozil is not
tent persons vaccinated after age 65 years are not contraindicated in pregnancy (FDA pregnancy class C) but
recommended to receive a booster. Immunocompetent would not be effective for achieving diis patient's LDL cho
persons vaccinated before age 65 years, such as this patient, lesterol goal.
should receive a single booster vaccination at age 65 years, Although statins (such as simvastatin) are typically the
or 5 years after their first vaccination if they were vaccinated first-line treatment for lowering LDL cholesterol levels,
between the ages of 60 and 64 years. statins are teratogenic (FDA pregnancy class X) and should
Immunocompromised patients (including those with be avoided in women who may be or wish to become preg
HIV infection and kidney disease) as well as patients with nant. Colesevelam, which is a bile acid sequestrant diat low
asplenia should receive a single pneumococcal vaccine ers LDL cholesterol levels by up to 18%, is die best initial
booster 5 years after their first vaccine. This strategy would treatment option for diis patient. Colesevelam is FDA preg
be inappropriate for this patient. nancy class B and so is safe to use in premenopausal women
Current recommendations do not support more than who are sexually active. Bile acid scquestrants have been
a single booster after initial pneumococcal vaccination for shown to reduce the risk of coronary heart disease in pri
any persons. Hence, a strategy of vaccination every 5 years mary prevention trials. The most common side effects asso
would be inappropriate. ciated with bile acid scquestrants include constipation,
All patients vaccinated before age 65 years need a abdominal pain and bloating, and flatulence. Bile acid
booster at some point. Hence, withholding further pneu scquestrants can bind to and reduce the absorption of other
mococcal vaccination is inappropriate. drugs; diis effect can be minimized by administering the
odier drugs 1 hour before or 4 hours after taking a bile acid
KEY POINT
sequestrant.
Immunocompetent persons who received the
KEY POINT
pneumococcal polysaccharide vaccine before age
65 years should receive a single booster vaccina Bile acid sequestrants are an option for reduc
tion at age 65 years, or 5 years after their first ing LDL cholesterol levels in women widi
vaccination if they were vaccinated between the hyperlipidemia who wish to become pregnant.
ages of 60 and 64 years.
Bibliography
Pande RL. Approach to lipid dierapy in the patient with atheroscle
Bibliography rotic vascular disease. Curr Treat Options Cardiovasc Med.
Targonski PV, Poland GA. Pneumococcal vaccination in adults: rec 2012;14(2)177-183. [PxMID: 22270374]
ommendations, trends, and prospects. Cleve Clin J Med.
2007;74(6):401-406,408-410, 413-414. [PMID: 17569198]

Item 76 Answer: D
Item 75 Answer: A Educational Objective: Initiate nonhospice pal
Educational Objective: Treat hyperlipidemia in a liative care in a patient with newly diagnosed cancer.
woman who desires pregnancy. In diis patient widi a newly diagnosed advanced malignancy,
This patient should be started on colesevelam. She has sev palliative care consultation is an important component of her
eral risk factors for cardiovascular disease, including poorly care. Palliative care focuses on improving and maintaining
controlled diabetes mellitus, hypertension, dyslipidemia, the quality of life in individuals with any severe illness. Pallia
and a family history of premature myocardial infarction. tive care is a multidisciplinary, boarded specialty that focuses
Her goal LDL cholesterol level is below 100 mg/dL (2.59 on preventing and relieving suffering and establishing goals
mmol/L). Therapeutic lifestyle changes, which include a of treatment that are consistent widi the patient's wishes.
low-saturated fat diet and at least 120 minutes of aerobic This often involves efforts at pain and symptom control and
exercise weekly, will reduce LDL cholesterol levels by 7% to encouraging and enabling patients to be actively involved in
15%. Thus, this patient needs additional dierapy to achieve the decisions regarding dieir care. Nonhospice palliative care
her LDL cholesterol goal. does not exclude testing, treatment, or hospitalization, but
Ezetimibe is a cholesterol absorption inhibitor that can seeks to ensure diat these interventions arc consistent with
reduce LDL cholesterol levels by up to 19%. Treatment what the patient wants and the expected goals and outcomes
with ezetimibe has not been shown to have beneficial of care. Whereas care in a hospice setting may be palliative in
effects on cardiovascular morbidity or mortality. Moreover, nature, not all palliative care takes place in patients with ter
ezetimibe is FDA pregnancy class X, and should be avoided minal illness. Palliative care input may be particularly valuable
in women who may become pregnant. in assisting this patient, who has a new diagnosis of severe

215
Answers and Critiques

disease, with understanding her illness and making key deci inhaled corticosteroid dierapy as initial therapy, as most
sions regarding her care. Although studies are limited, pal patients with NAEB will respond to this intervention.
CONT.
liative care has been shown to improve overall quality of life Targeted and optimized empiric treatment of common
in the setting of various diseases relative to usual care for causes of chronic cough is generally recommended prior to
severely ill individuals. more invasive or cosdy testing. Treatment with antihista
Although depression may be seen in some patients mines or decongestants should begin first, widiout need for
with severe illness, starting dierapy for depression without sinus radiographs to evaluate for sinus disease in patients
clear evidence the patient is having significant depressive with suspected upper airway cough syndrome. Similarly,
symptoms or that pharmacologic treatment is indicated diet and lifestyle modification plus proton pump inhibitors
would be inappropriate. for 1 to 3 mondis should be prescribed prior to consider
It is not clear diat this patient is either medically or ing 24-hour esophageal pH manometry to evaluate for acid
emotionally ready for hospice care. Although her newly reflux disease.
diagnosed malignancy may carry a poor prognosis, her cur
KEY POINT
rendy stable condition and expressed desire to explore pos
sible treatment options would make a decision to pursue A diagnosis of nonasthmatic eosmophilic bron

hospice care premature widiout further characterization of chitis should be considered in patients with
her disease and discussion of her long-term treatment goals. chronic, nonproductive cough without an
Opioid therapy is commonly used in cancers and par apparent cause, including asthma; sputum
ticularly malignancies involving the respiratory tract to examination for eosinophils is useful in estab
reduce both pain and dyspnea. However, this patient does lishing the diagnosis.
not have significant pain and has only mild shortness of
breadi, which should improve as her pulmonary embolism Bibliography
resolves. Therefore, initiation of ongoing opioid therapy is Desai D, Brighding C. Cough due to asthma, cough-variant asthma
and non-asthmatic eosinophilic bronchitis. Otolaryngol Clin North
not indicated.
Am. 2010;43(1):123-130, x. [PMID: 20172262]
KEY POINT
Palliative care focuses on improving and main
Item 78 Answer: A
taining the quality of life in any patient with
severe illness; it is not limited to those with ter Educational Objective: Evaluate an elderly
minal illness or inpatient settings. patient with a recent fall.
This patient who has recently fallen should be assessed for
Bibliography
gait and mobility. The Timed "Up & Go" (TUG) is a vali
Bakitas M, Lyons KD, Hegel MT, et al. Effects of a palliative care
intervention on clinical outcomes in patients with advanced cancer: dated test for mobility that can be easily performed in the
the Project ENABLE II randomized control trial. JAMA clinic and may be useful in predicting the likelihood of
2009;302(7):741-749. [PMID: 19690306] future falls. The test is performed by asking the patient to
arise from a chair, walk 10 feet, turn around, and sit back
down in the same chair. A time of more dian 14 seconds
Item 77 Answer: D indicates an increased risk for future falls. The sensitivity and
Educational Objective: Diagnose nonasthmatic specificity for one or more falls during the 6 mondis after die
eosinophilic bronchitis. TUG test are 96% and 32%, respective!)'. In patients who
have fallen, it is important to get a detailed history of the cir
This patient's presentation is consistent with nonasthmatic
cumstances of the fall and whether die patient has fallen
eosinophilic bronchitis (NAEB), and the next diagnostic before or has problems widi balance. A complete list of
step would be sputum testing for eosinophils. NAEB is an
medications, particularly psychotropic medications, should
increasingly recognized cause of chronic cough, particularly be obtained. This patient had what sounds like a mechani
in patients such as this one who lack risk factors or findings
cal tall, without any evidence of syncope, lightheadedness, or
for the more common causes of chronic cough (smoking,
dizziness. She remembers the entire event and was not con
cough-variant asthma, gastroesophageal reflux disease, fused after die fall, making a seizure unlikely.
upper airways disease). Patients widi NAEB do not exhibit
Discontinuing lisinopril is not likely to reduce this
symptoms of or pulmonary function testing evidence of air
flow obstruction or hyperresponsivencss, with or widiout patient's risk for falls because her blood pressure is not low,
she has no orthostatic blood pressure changes, and she has
provocation with mediacholine, which differentiates rhis no symptoms of lighdieadedness or dizziness. Discontinu
entity from asthma. The diagnosis is supported by airway
ing antihypertensive medication would be indicated if it
eosinophilia in an induced sputum sample (greater than 3%), were deemed likely that hypotension or orthostatic
bronchial washings, or biopsy. Aldiough bronchial mucosal
hypotension were present and a possible cause for her fall.
biopsies are required to definitively diagnose eosinophilic
Prescribing an exercise program would be indicated if
bronchitis, most experts recommend a therapeutic trial of
physical examination revealed weakness or poor balance.

216
Answers and Critiques

Some studies have shown that there may be harms associ KEY POINT
ated with exercise programs in the frail older adult, so all The pain of pes anserine bursitis is typically
such programs should bc supervised. located along the anteromedial aspect of the
Whereas providing an assistive device such as a walker proximal tibia distal to the joint line of the
would be indicated for patients with weakness, poor bal knee and characteristically worsens with step
ance, or gait disturbance, assessment of gait and mobility climbing and at night.
should be performed first. If the TUG were abnormal, fur
ther assessment by a physical therapist could be performed Bibliography
to determine if an assistive device is indicated and which Schraeder TL, Terek RM, Smith CC. Clinical evaluation of die knee.
assistive device might be most helpful. New Engl J Med. 2010;363(4):e5. [PMID: 20660399]

KEY POINT
The Timed "Up & Go" (TUG) test for evaluat Item 80 Answer: B
ing gait and mobility is performed by asking Educational Objective: Treat vulvovaginal can
the patient to arise from a chair, walk 10 feet, didiasis in a pregnant patient.
turn around and sit back down in the same
The most appropriate treatment for this pregnant woman
chair; a time of more than 14 seconds indicates is a topical imidazole, such as clotrimazole. She has classic
an increased risk for future falls.
symptoms and signs of vulvovaginal candidiasis (WC),
Bibliography including itching, discomfort, and a thick, white vaginal
Nordin E, Lindelof N, Rosendahl E, Jensen J, Lundin-Olsson L. Prog discharge with evidence of vulvar edema and erythema on
nostic validity of the Timed Up-and-Go test, a modified Gct-Up- examination. The vaginal pH is normal, and potassium
and-Go test, Staff's global judgement and fall history in evaluating hydroxide preparation shows evidence of hyphae and yeast,
fall risk in residential care facilities. Age Ageing. 2008;37(4):442-
448. [PMID: 18515291] supporting this diagnosis. Uncomplicated WC is diag
nosed when an otherwise healthy, nonpregnant patient has
mild to moderate symptoms and suspected Candida albi
Item 79 Answer: C cans infection. Complicated WC is diagnosed in women
widi severe symptoms (extensive vulvar erythema, edema,
Educational Objective: Diagnose pes anserine excoriation, and fissure formation), immunosuppression,
bursitis.
multiple recurrences, or diabetes mellitus; and in women
This patient most likely has pes anserine bursitis. Although who are pregnant. As compared with treatment of uncom
pes anserine bursitis most commonly occurs in patients with plicated WC, patients with complicated WC, such as this
medial compartment osteoarthritis, it also occurs in the set pregnant woman, typically require longer and more aggres
ting of overuse, as is the case with diis patient. The pain is sive dierapy. Regimens for treatment of complicated C.
albicans vulvovaginitis include topical imidazole dierapy for
typically located along the anteromedial aspect of proximal
tibia distal to the joint line of die knee. Pain is worse with up to 14 days or two 150-mg doses of oral fluconazole
climbing steps and frequendy worsens at night. given in two sequential doses 72 hours apart (compared
Iliotibial band syndrome is a common cause of knife with single-dose dierapy). Topical imidazole therapy should
like lateral knee pain diat occurs with vigorous flexion- be used preferentially in pregnant women, and a variety of
extension activities of die knee, such as running. It is regimens are available for treatment. Seven days of topical
treated conservatively with rest and stretching exercises. clotrimazole cream is an appropriate option in this patient.
This patient's presentation is not consistent widi iliotibial Oral fluconazole is an appropriate treatment for uncom
band syndrome as die patient's pain is located medially. plicated WC, and is associated widi high treatment success
rates. However, it should not be given in prcgnanq' as the
Also, the pain with iliotibial band syndrome is characteris
effect on die fetus is unknown (FDA pregnane)' category C
tically worsened with walking bodi up and down steps,
which this patient does not report. medication), and topical dierapy is equally efficacious.
The most common cause of knee pain in persons Although 30% of women will experience two or more
younger than 45 years, especially in women, is the episodes of WC, recurrent WC is diagnosed if a patient
patellofemoral syndrome. The pain is peripatellar and exac experiences more than four symptomatic episodes per
erbated by overuse (such as running), descending stairs, or year. Frequently, recurrent WC is associated with non-
'albicans Candida infection. Initial treatment of culture-
prolonged sitting. On examination, pain and apprehension
can often be elicited by applying pressure on die patella (the proven non- albicans WC includes 7 to 14 days of an oral
or topical non-imidazole therapy, such as voriconazole, a
patellofemoral compression test).
Prepatellar bursitis presents with pain in the anterior second-generation triazole antifungal agent; recurrent
WC may be treated with intravaginal boric acid for 2
aspect of the knee. On examination, swelling and tender
ness to palpation are frequently present near die lower pole weeks. Because this patient has had only one previous
of the patella. episode of symptomatic WC, which responded well to

217
Answers and Critiques

treatment, she can be empirically treated for Candida after the drug is discontinued. If this patient's cough per
albicans infection again. sists beyond this time, a systemic approach to treatment
KEY POINT
of chronic cough should ensue.
Treatment of Candida albicans vulvovaginal KEY POINT
candidiasis in pregnant women typically In patients with chronic cough and a normal
requires longer and more aggressive therapy; chest radiograph, smoking cessation and dis
topical imidazole therapy for 7 days is die pre continuation of ACE inhibitors should be rec
ferred treatment. ommended for 4 weeks before additional evalu
ation for the cough.
Bibliography
ACOG Committee on Practice BulletinsGynecology. ACOG Prac
tice Bulletin. Clinical management guidelines for obstetrician- Bibliography
gynecologists, Number 72, May 2006: Vaginitis. Obstet Gvnecol. Dicpinigaitis PV. Angiotensin-converting enzyme inhibitor-induced
2006;107(5):1195-1206. [PMID: 16648432) cough: ACCP evidence-based clinical practice guidelines. Chest.
2006;129(1 suppl):169S-173S. [PMID: 16428706]

Item 81 Answer: B
Item 82 Answer: A
Educational Objective: Treat cough in a patient
Educational Objective: Manage medications
taking an ACE inhibitor.
perioperatively.
In diis patient with a nonproductive cough, the best option
is to discontinue lisinopril. Clinical evaluation of chronic This patient should continue all of his current medications.
cough (>8 weeks in duration) includes a careful history and Antiplatelet dierapy (including aspirin and clopidogrel),
physical examination focusing on the common causes of antihypertensive medications, and anticonvulsant drugs
chronic cough. All patients should undergo chest radiogra can be continued during low-risk surgeries, such as cataract

phy. Smoking cessation and discontinuation of ACE surgery, if the benefit outweighs the risk. In a preoperative
inhibitors should bc recommended for 4 weeks before evaluation each medication should be assessed for its indi
additional workup. Cough is a common side effect of ACE vidual risk and benefit. Those for which risk outweighs ben
inhibitors. Approximately 15% of patients who are pre efit should be temporarily discontinued perioperatively.
scribed these medications will develop a nonproductive Dual antiplatelet dierapy with clopidogrel and aspirin
cough. Reported causative factors include bradykinin and should be continued for at least 1 year in patients who have
substance P, which are metabolized by angiotensin-con received a drug-eluting stent. This patient's drug-eluting
verting enzyme and prostaglandins. The onset may be stent was inserted only 6 months ago; dierefore, the patient
delayed, as in this patient, and may take up to 4 weeks to remains at increased risk for in-stent dirombosis if clopido
resolve upon discontinuation of the drug (rarely, up to 3 grel is discontinued and at no increased risk of bleeding
months). Although the cough is frequendy mild, in some related to the cataract surgery if clopidogrel is continued.
patients it is significant enough to interfere widi quality of The use of p-blockers in the perioperative period is
life, and alternate dierapy needs to be considered. Substi controversial, with well-designed randomized controlled
tution of an angiotensin receptor blocker, such as losartan, trials showing both benefit and harm. Current recom
is a good alternative in this patient; these medications gen mendations are that patients currendy on a P-blocker
erally do not cause cough (incidence is similar to that of should continue taking it. Thus, metoprolol should not be
placebo) and evidence supports dieir renal protective ben discontinued.
efits in patients with diabetes mellitus. Any medication that can produce withdrawal symp
Because this patient's cough has been present for 8 toms, such as benzodiazepines, long-acting narcotics, and
weeks, other causes of chronic cough should be consid antiseizure drugs, are typically continued the morning of
ered. As the clinical picture is most consistent with ACE surgery.
inhibitor-induced cough and there is no symptom pre
KEY POINT
dominance to support bronchospasm (history of asthma,
wheezing and cough with exertion, exposure to allergens In a preoperative evaluation, each medication
or cold air), upper airway cough syndrome (postnasal should be assessed for its individual risk and
drip with frequent nasal discharge, a sensation of liquid benefit, and those for which risk outweighs
dripping into the back of the throat, and frequent throat benefit should be temporarily discontinued
clearing), or gastroesophageal reflux disease (heartburn perioperatively.
or regurgitation), initial empiric treatment with
albuterol, an antihistamine, intranasal corticosteroids, or Bibliography
omeprazole is not indicated at this time. ACE Whinncy C. Perioperative medication management: general principles
and practical applications. Clevc Clin J Med. 2009;76(suppl
inhibitor-induced cough generally abates within 4 weeks 4):S126-32. [PMID: 19880829]

218
Answers and Critiques

Item 83 Answer: A overuse syndromes (such as running) and widi wearing


Educational Objective: Manage a request for tight shoes. It classically presents with burning pain on the
plantar surface in the space between the third and fourdi
physician-assisted suicide. toes but may also occur between the second and third toes.
When approached with a request for assistance in dying, it Women are more commonly affected dian men, and the
is best to respond to the request with empadiy and com wearing of high heels is a recognized risk factor. Treatment
passion, and assess whedier or not the patient is receiving is typically conservative, with the goal of reducing pressure
adequate palliative interventions. Optimizing care inter across die metatarsal heads through die use of padding,
ventions focused on maintaining or improving the quality orthotics, and the removal of likely inciting footwear or
of life may not always occur in die context of treating the activities. If conservative measures fail, a local corticoster
underlying disease process; thus, reviewing the patient's oid injection is usually successful.
overall care to address comfort and functional issues in A hammer toe is characterized by a flexion deformity
severe illness is essential to appropriate management. of the proximal interphalangeal joints with normal distal
Involving physicians trained specifically in palliative care interphalangeal joints and metatarsophalangeal joints. Pre
medicine may also be helpful in such situations. senting symptoms include pain and difficulty wearing shoes
Physician-assisted suicide is a controversial area of because of the resulting toe structure. A com may also
ethics. Most ethicists agree diat it is acceptable to consider develop on the dorsal surface of die proximal interpha
interventions that may hasten the death of a terminally ill langeal joint.
patient if die primary intent is therapeutic (die principle of In metatarsal stress fractures, the examination is
"double effect"). However, physician-assisted suicide using notable for tenderness to palpation of the fracture site. It
prescriptions or interventions with the specific intent to kill would not bc expected to cause pain between toes, as is the
the patient is illegal in most states. The American Medical case in this patient.
Association and the American College of Physicians have Tarsal tunnel syndrome is entrapment of the poste
both taken positions against die practice. rior tibial nerve or one of its branches as it travels behind
Seeking legal counsel may be advisable if one intends the medial malleolus. Although this may cause pain and a
to provide the patient assistance in dying, as states in which burning sensation, symptoms tend to occur on the medial
it is legal have specific protocols that must be followed. plantar aspect of the foot, occasionally mimicking plantar
However, this step would not be appropriate until alterna fasciitis.
tives such as improved palliative care were assessed.
KEY POINT
Categorically refusing to discuss a request for physi
cian-assisted suicide can close the door to a discussion of Morton neuroma is characterized by burning
why the patient is making the request and may jeopardize pain on the plantar surface in the space
the dierapeutic relationship with the patient. between the third and fourth toes.
Writing a prescription for medication to assist a patient
in dying without a detailed assessment of the patient's sit Bibliography
Wu KK. Morton neuroma and metatarsalgia. Curr Opin Rheumatol.
uation and motives would be irresponsible.
2000;12:131-142. [PMID: 10751016]
KEY POINT
When approached with a request for assistance
Item 85 Answer: A
in dying, it is best to respond to the request
with empathy and compassion, and assess Educational Objective: Manage body dysmorphic
whether or not the patient is receiving adequate disorder.
palliative care. Cognitive-behavioral dierapy (CBT) would be appropriate
in the management of diis patient. This patient's presenta
Bibliography tion is consistent with body dysmorphic disorder, a somato
Snyder L, Sulmasy DP; Ethics and Human Rights Committee, Amer
ican College of Physicians-American Society of Internal Medicine. form disorder in which a patient is focused on a single real
Physician-assisted suicide. Ann Intern Med. 2001;135(3):209-216. or imagined symptom. In order to qualify as a psychiatric
[PMID: 11487490] disorder, somatoform symptoms need to be medically
unexplained or out of proportion to medically expected
findings, should persist over time, and cause impairment in
Item 84 Answer: C a patient's ability to function. Although no therapy has
Educational Objective: Diagnose Morton neu been shown to be consistendy helpful in treating somato
roma. form disorders, multiple trials have found benefit in patients
This patient most likely has Morton neuroma, which is who undergo CBT.
thought to be caused by inflammation, edema, and scarring Benzodiazepines such as diazepam have not been
of the small interdigital nerves. It commonly occurs in shown to be of benefit for somatoform disorders, and

219
Answers and Critiques

would likely be a poor choice because of their ability to Neuroimaging, such as CT scanning, is of limited use
induce tolerance and long-term dependence. in evaluating syncope. It has the highest yield in patients
It is unlikely that diis patient would have any additional who are older dian 65 years and have neurologic symp
abnormal findings on further abdominal imaging. As toms, such as headache, neurologic examination abnor
patients with somatoform disorders have a true psychiatric malities, head trauma, or are on anticoagulants. This patient
disease, additional normal diagnostic tests are ineffective in has no features to suggest a neurologic cause of syncope
alleviating symptoms. and a head CT scan is not indicated.
Psychosis encompasses delusions, hallucinations, dis If structural heart disease is suspected, further assess
organized speech, and disorganized or catatonic behavior. ment should include cardiac imaging, usually by echocar
Psychotic features may occur in depression as well as other diography, as the first diagnostic test. There is nothing in
psychiatric or organic disorders, but schizophrenia is a dis the patient's history or physical examination that sug
order in which psychosis is a defining feature. The diagno gests structural heart disease, and echocardiography is
sis requires at least 6 months of symptoms, including 1 not indicated.
month or more of at least two active-phase symptoms, such Tilt-table testing is reserved for patients with suspected
as hallucinations, delusions, disorganized speech, grossly
neurocardiogenic syncope not confirmed by history and
disorganized or catatonic behavior; and negative symp physical examination, those with recurrent episodes, and
toms, such as flattened affect. This patient is not psychotic those with a suspected cardiac cause. As this patient's his
and treatment with an antipsychotic medication such as
tory and physical examination are consistent with vasovagal
olanzapine is not indicted. syncope and there is no history of recurrent episodes, tilt-
KEY POINT table testing is not indicated.
Although no therapy has been shown to be KEY POINT
consistently helpful in treating somatoform dis Persons with uncomplicated faint, situational
orders, multiple trials have found benefit in syncope, or orthostatic hypotension do not
patients who undergo cognitive-behavioral require further investigation if the initial physi
therapy. cal examination and electrocardiogram are
normal.
Bibliography
Kroenke K. Efficacy of treatment for somatoform disorders: a review
of randomized controlled trials. Psvchosom Med. 2007;69(9)S81- Bibliography
888. [PMID: 18040099] Cooper PN, Westby M, Pitcher DW, Bullock I. Synopsis of the
National Institute for Health and Clinical Excellence Guideline for
management of transient loss of" consciousness. Ann Intern Med
2011;155(8):543-549. [PMID: 21930835]
Item 86 Answer: E
Educational Objective: Evaluate vasovagal syn
cope. Item 87 Answer: B
No further testing is required for this patient. Her symp Educational Objective: Manage a fall in an
toms are consistent with the most common form of syn elderly patient.
cope, vasovagal neurocardiogenic syncope, the common In this patient with generalized weakness as well as leg mus
"faint." Suggestive features of vasovagal syncope include cle weakness, slow gait, and a recent fall, it is appropriate to
any of the "3 P's": Posture (occurrence during prolonged prescribe vitamin D. Vitamin D deficiency increases die risk
standing, or similar previous episodes that have been for falls in the elderly and vitamin D supplementation
aborted by lying down); Provoking factors (e.g., pain, a reduces this risk. According to U.S. Preventive Services
medical procedure); and Prodromal symptoms (sweating, Task Force recommendations, vitamin D supplementation
feeling warm before loss ofconsciousness). Persons with an can be prescribed without first obtaining a serum vitamin
uncomplicated faint, situational syncope, or orthostatic D level for patients widi an increased risk of falling. The
hypotension should undergo electrocardiography but do proposed mechanism of action of vitamin D is its beneficial
not otherwise require immediate further investigation or effect on muscle strength and function and on gait.
specialist referral. Although calcium supplementation may have a beneficial
Admission to the hospital for telemetry should be con effect on bone loss, there is no clear benefit to adding cal
sidered in patients with undiagnosed syncope but with cium in reducing falls.
known structural heart disease and at high risk for arrhytii-
Discontinuing lisinopril is not appropriate because she-
mia. Even in this selected group, however, the diagnostic does not demonstrate orthostatic blood pressure changes
yield is low (approximately 16%), and there is no benefit for diat would account for her fall, and discontinuing antihy
patients with symptoms compatible with vasovagal syncope pertensive medication would likely result in elevated blood
in the absence of known heart disease.
pressure.

220
Answers and Critiques

Zolpidem is a nonbenzodiazepine sedative hypnotic approaches, including behavioral or appropriate pharmaco


widi a short half-life diat can be prescribed for a limited logic therapy.
time period for insomnia. Caution must be exercised, how Tolterodine, a selective antimuscarinic anticholinergic
ever, because of adverse effects, including an increased risk medication, is most effective for patients with urge, rather
for falls, especially among older adults. Reviewing sleep than stress, incontinence. This patient does not experience
hygiene would be a better first step in managing her the classic sense of urinary urgency with her incontinence
insomnia. episodes, and, therefore, tolterodine would not be an
Although this patient demonstrates a mild near-vision appropriate first choice.
deficit, it is not likely diat diis deficit contributed signifi- KEY POINT
candy to her fall. Furthermore, bifocal lenses are associated
Pelvic floor muscle training is first-line treat
with an increased risk for falling. If needed, reading glasses
could be obtained. ment for stress urinary incontinence.

KEY POINT Bibliography


Vitamin D supplementation reduces the risk for Shamliyna TA, Kane RL, Wyman J, Wilt TJ. Systematic review: ran
falls in elderly patients, and can be prescribed domized, controlled trials of nonsurgical treatments for urinary
incontinence in women. Ann Intern Med. 2008;148(6):459-473.
without obtaining a serum vitamin D level in [PMID: 18268288]
patients with an increased risk of falling.

Bibliography Item 89 Answer: D


Kalyani RR, Stein B, Valiyil R, et al. Vitamin D treatment for the pre
Educational Objective: Initiate a discussion
vention of falls in older adults: systematic review and meta-analysis.
J Am Geriatr Soc. 2010;58(7):1299-1310. [PMID: 20579169[ about palliative care with the family of a cancer
patient.
The cornerstone of establishing goals of care in die end-of-
Item 88 Answer: A
life setting is to communicate in a patient-centered, open-
Educational Objective: Treat stress urinary ended format. This is true regardless of whether a patient or
incontinence.
patient's family is angry or is requesting inappropriately
This patient has stress urinary incontinence and should aggressive care. The first step in this process in diis case is to
receive pelvic floor muscle training (PFMT). Stress uri ask the family to tell you what diey understand about the
nary incontinence, defined as loss of urine widi physical patient's condition. Active, empathic listening allows the
activity, cough, or sneeze, is caused by sphincter incom caregiver to establish what die patient and family understand
about die diagnosis and prognosis. It also shows respect for
petence. Findings on physical examination include weak
ened anterior or posterior vaginal wall support (cystocele die myriad ways in which loved ones process information
or rectocele, respectively). PFMT is considered first-line about medical conditions and helps to establish trust. The

therapy for urinary stress incontinence. In PFMT, women family should be allowed to vent their frustration and to artic
learn repetitive exercises (Kegel exercises) to strengthen ulate what diey believe the patient's condition and chance of
the voluntary urethral sphincter and levator ani muscles. meaningful recovery to be. Given die feelings of distress
For PFMT to be effective, it is important that the patient about the patient's condition, it is entirely possible diat one
learn to correctly contract her muscles without straining, meeting may not be enough to establish clearly defined goals
which increases abdominal pressure. Each contraction is of care. Asking open-ended questions and being comfortable
held for approximately 10 seconds, followed by an equal with silences are important in building a trusting relationship
relaxation period. The number of repetitions should be with the patient and family.
increased weekly until the patient is performing 8 to 12 The upcoming dialogue with the family is likely to be
repetitions three times daily, every day or at least 3 to 4 emotionally charged, and a series of visits may be needed to
cover all appropriate areas. It would not be appropriate to
days per week. In a systematic review of nonsurgical ther
initiate the discussion with the patient and family about
apy, PFMT improved stress urinary incontinence
advanced directives until it is learned what the family
episodes. Outcomes were even better when PFMT was
combined with biofeedback and when skilled therapists knows about the diagnosis and prognosis.
directed the treatment. It would not be helpful to begin a meeting with a dis
Prompted voiding is indicated in and is effective in traught family or patient by stating curative therapy would
be futile. This approach is likely to further alienate a family
patients widi significant mobility or cognitive impairments
diat may hinder the patient's ability to reach the toilet in struggling with a distressing diagnosis.
time, neidier of which this patient has. Aldiough explaining the diagnosis and prognosis may
be an important goal for a family meeting, it is usually more
Sling procedures are effective for moderate to severe-
stress incontinence, but surgery is usually reserved for effective to begin the meeting with an open-ended question
that allows the physician to better understand die family's
patients who do not benefit from more conservative

221
Answers and Critiques

perspective. Explanations can then be better tailored to what die initial treatment widi nonopioid analgesics and mobiliza
family knows and understands about the patient's condition. tion as tolerated is most appropriate. The overall prognosis
CONT.
for acute musculoskeletal low back pain is excellent; most
KEY POINT
patients without sciatica show substantial improvement
The cornerstone of establishing goals of care in within 2 weeks, and diree quarters of diose widi sciatica are
the end-of-life setting is to communicate in a
substantially better after 3 months; dierefore, dierapeutic
patient-centered, open-ended format. interventions should focus on mitigating symptoms and
maintaining function while the patient recovers.
Bibliography
Swetz KM, Kamal AH. Palliative care. Ann Intern Med. Complete blood count and erythrocyte sedimentation
2012;156(3):ITC21. [PMID: 22312158] rate are helpful in assessing for infection, inflammatory
spondylitis, and malignancy. This patient has no signs or
symptoms suggestive of systemic illness, and specific labo
Item 90 Answer: C ratory testing is dierefore not indicated at this time.
Educational Objective: Evaluate a breast mass in Epidural corticosteroid injection is sometimes consid
a postmenopausal woman. ered in patients with chronic radiculopathy, although the
literature is mixed regarding its value. This patient has an
This patient should undergo core needle biopsy of the mass. excellent prognosis widiout intervention, so invasive treat
She presents with a normal mammogram but widi suspicious ments would be inappropriate as initial therapy.
findings on physical examination for breast cancer. The pal Lumbar spine imaging is not indicated for most
pable mass is nonpainful, persistent, and firm. Although her patients with acute lumbosacral back pain with radiculopa
normal mammogram could be interpreted as reassuring,
thy as it does not add clinically significant information. Sit
approximately 10% to 20% of palpable breast cancers can be uations in which imaging is necessary include patients widi
missed by either ultrasonography or screening mammogra
rapidly progressing neurologic symptoms, evidence of cord
phy. She requires further evaluation to definitively rtde in or
rule out malignancy. Core needle biopsy, with or without compression, cauda equina syndrome, or if infection or
malignancy is a possible cause of the patient's symptoms
ultrasonographic or stereotactic guidance, provides excellent and examination findings.
tissue sampling for padiology and receptor status. It is the
Lumbar spine radiography is helpful to assess for pos
test of choice for most solid lesions.
sible malignancy or compression fracture. In diis younger
Breast MRI would likely better define the breast lesion
patient without evidence of systemic illness, radiography is
radiographically that was not visualized on mammography, not indicated. An MRI would likely demonstrate the disk
but would not replace die need for a tissue diagnosis in this
herniation and nerve root compression that are already evi
patient. dent on physical examination, but the management plan
Breast ultrasonography is particularly useful in defining
would still be analgesics and gende mobilization.
possible cystic lesions identified on examination or mam
mography. However, given die highly suspicious nature of KEY POINT
this patient's breast mass, ultrasonography would not be Therapeutic interventions for most patients
indicated. with acute low back pain should focus on miti
Reassurance is inappropriate as definitive diagnosis of
the mass via tissue sampling is imperative in this post gating symptoms and maintaining function
while the patient recovers.
menopausal woman.
KEY POINT Bibliography
Core needle biopsy is the test of choice for Chou R, Qaseem A, Snow V, et al; Clinical Efficacy Assessment Sub
committee of the American College of Physicians; American Col
most solid breast masses. lege of Physicians; American Pain Society Low Back Pain Guidelines
Panel. Diagnosis and treatment of low back pain: a joint clinical
Bibliography practice guideline from the American College of Physicians and the
Barlow WE, Lehman CD, Zheng Y, et al. Performance of diagnostic American Pain Society. Ann Intern Med. 2007;147(7):478-491
I PMID: 17909209]
mammography for women widi signs or symptoms of breast can
cer. I Natl Cancer Inst. 2002;94(15):1151-1159. [PMID:
12165640]
Item 92 Answer: A

Item 91 Answer: A
Educational Objective: Recognize threats to II
validity in a medical study.
Educational Objective: Manage acute low back
The main reason diat it is difficult to determine the effec
pain.
tiveness of this drug based on the published study is that
In diis patient presenting widi uncomplicated low back there is no comparison, or control, group. When evaluat
pain and examination findings suggesting radiculopathy, ing the medical literature, it is important to consider the

222
Answers and Critiques

quality of die study design. Studies assessing treatment need for ongoing medications; this would be an opportune
effectiveness should always have a control group, which can moment for a drug holiday in diis patient.
receive either an alternative treatment or a placebo. A con Atomoxetine is a selective norepinephrine reuptake
trol group is critical because it tells the investigators what inhibitor that is specifically approved for treatment of
would have happened if die intervention had not been ADHD in adults. It is not associated with hypertension
done. Depending on die study type, patients can be and would be a reasonable option after his drug holiday
assigned to a control group (in an experimental study), or if there is a need for ongoing medications; however, first
be part of a "natural" control (observational study). The it should be ascertained whether he still needs pharmaco
primary threat to validity of this study is the absence of a logic treatment for his ADHD. Atomoxetine carries a
control group; that is, there is no group with which the black box warning of increased suicidal ideation in the
patients taking "drug X" can be compared. pediatric population and has been associated with rare but
In general, it is always best for outcomes in any study serious hepatotoxicity.
to be assessed by an independent evaluator who is unaware Fluoxetine and other selective serotonin reuptake
of treatment assignment ("blinded"). In the case of unam inhibitors have not been shown to be useful for die treat
biguous outcomes such as deadi, however, an unblinded ment of ADHD. Bupropion and tricyclic antidepressants
outcomes assessment is permissible. may be helpful for ADHD, although this is an off-label use
In experimental study designs, investigators often ran of these drugs.
domly assign patients to a dicrapy in order to equalize the Methylphenidate has sympathomimetic properties,
group for measured and unmeasured confounding vari and could contribute to his hypertension. Therefore,
ables. This trial has no comparison group, so randomiza switching to this agent would not be beneficial.
tion would not be possible.
KEY PO NT
Increased numbers of patients in studies generally yield
In adult patients with attention-deficit/hyper-
greater precision in measurement. In diis trial, however, the
key direat to validity is not trial size but absence of a con activity disorder (ADHD), it is important to
trol group. reassess the need for ADHD medications
periodically.
KEY POINT
The primary threat to validity in a case series is Bibliography
the absence of a control group. Okie S. ADHD in Adults. N Engl J Med. 2006;354:2637-2641.
[PMID: 16790695]
Bibliography
Ho PM, Peterson PN, Masoudi FA. Evaluating die evidence: is there
a rigid hierarchy? Circulation. 2008;118(16):1675-1684. [PMID: Item 94 Answer: D ^rl
18852378]
Educational Objective: Manage a transition of care
in an elderly patient undergoing hospital discharge.
Item 93 Answer: E The patient should be evaluated for placement in a skilled
Educational Objective: Manage attention- nursing facility. Although discharge home is preferable if a
deficit/hyperactivity disorder in an adult. patient is safe and medically stable for care in that setting,
this patient is not able to return home to a setting in which
The most appropriate management for diis patient widi
she was not doing well prior to admission. Her complicated
attention-defidt/hyperactivity disorder (ADHD) is to stop
his ADHD medication and reassess his need for treatment. hospitalization has left her significantly debilitated. Skilled
ADHD is characterized by difficulty paying attention, impul nursing facilities provide nursing level services, such as
intravenous medications and medication management,
sivity, and motor resdessness or hyperactivity', with onset wound care, and other medical services in addition to low-
before the age of 7 years. There must also be some impair
level rehabilitation services. With further recovery, her
ment in social, occupational, or academic functioning, and
the symptoms must be manifest in at least two different envi long-term care options may be reassessed and the most
ronments, such as school and home. There are diree sub appropriate type pursued.
types: prcdominandy inattentive, predominandy hyperactive, Inpatient rehabilitation is focused on intensive physical
and a combined type. The presence of symptoms in child and occupational therapy and other forms of rehabilitative
hood is crucial to die diagnosis of ADHD, and this patient treatment as needed. Aldiough patients widi active medical
issues may be candidates for inpatient rehabilitation, diese
gives a history diat is compatible widi ADHD. However, he
also has new-onset h)pertension and a heart rate at die issues need to bc stable, and patients are generally required
to participate in therapy for a minimum of 3 hours daily.
higher end of normal, which could be related to sympath
omimetic drug use. He has had relatively few symptoms Long-term acute care hospitals (LTACHs) provide care
related to ADHD for the past 2 years. Periodic "drug holi similar to diat in an acute hospital setting but for patients
who are considered stable with the need for hospital-based
days" for adult patients with ADHD may be useful to assess

223
Answers and Critiques

testing or interventions and with few anticipated changes in loss of vision that occurs in die early morning hoursusu
the care plan. This setting is overseen by physicians and is ally between midnight and 6 AM and, second most com
CONT.
appropriate for patients who require significant medical monly, between 6 AM and noon. It results from an embolic
monitoring but are expected to have a more prolonged or thrombotic event in the ophthalmic artery. Aldiough
(more dian 25 days) time to recovery. This patient's medical this patient is at risk for CRAO owing to her atrial fibrilla
needs arc minimal and could be appropriately provided in a tion, CRAO would not cause red eye, a firm globe, ocular
skilled nursing setting. pain, nausea, or vomiting.
Although diis patient ma)' have been declining for sev Ocular migraine, also known as retinal migraine, typ
eral years, she does not have a diagnosis or condition indica ically occurs in persons with a family history or personal
tive of a prognosis of less than 6 months. To qualify for hos history of migraine, which this patient does not have.
pice care, her physician must feel that her expected Symptoms include flashing lights, scintillating scotomas,
prognosis is less than 6 months of life remaining. visual blurring, and even total unilateral vision loss.
Establishing appropriate discharge care plans involves Patients with ocular migraine tend to be younger than 40
close coordination widi the physician, patient and family, years, making this diagnosis highly unlikely in this 76-
and the care coordination staff. Understanding the year-old patient.
patient's medical needs is critical in determining the appro Temporal arteritis should be considered in patients
priate options available, and this process should be started older than 50 years presenting with a severe new headache.
as early as possible in the course of hospitalization. Visual loss in temporal arteritis is painless, however, and
would not cause a red eye, nausea, or vomiting.
KEY POINT
Understanding a patient's medical needs in KEY POINT

conjunction with the wishes and resources Acute angle-closure glaucoma is characterized
available to the patient and family is critical in by severe unilateral eye pain, headache, nausea
determining appropriate postiiospital care, and and vomiting, and seeing halos around lights;
this process should be started as early as possi physical examination findings include conjunc
ble in the course of hospitalization. tival erythema; a sluggish or nonreactive, mid-
range dilated pupil; corneal cloudiness; and
Bibliography cupping of the optic nerve.
Kane RL. Finding the right level of postiiospital care: "We didn't real
ize there was any odier option for him". JAMA. 2011;305(3):284-
293. [PiMID: 21245184] Bibliography
Magauran B. Conditions requiring emergency ophdialniologic con
sultation. Emerg Med Clin North Am.'2008;26(l):233-238.
[PMID: 18249265]
Item 95 Answer: A
Educational Objective: Diagnose acute angle-clo
sure glaucoma. Item 96 Answer: B
This patient most likely has acute angle-closure glaucoma. Educational Objective: Diagnose de Quervain
Angle-closure glaucoma is characterized by narrowing or clo tenosynovitis.
sure of die anterior chamber angle, which impedes die tra This patient's presentation is most consistent with de Quer
becular drainage system in the anterior chamber, resulting in vain tenosynovitis, which refers to swelling or stenosis of
elevated intraocular pressure and damage to the optic nerve. the abductor pollicis longus and extensor pollicis brevis ten
Acute angle-closure glaucoma is an ophthalmologic emer don sheaths at the level of the wrist. It is most commonly
gency. Symptoms depend upon die rapidity of die elevation caused by repetitive motion of the thumb but can also be
of intraocular pressure. Typical history of acute angle-closure associated with underlying conditions including pregnancy
glaucoma may include seeing halos around lights, severe uni and rheumatoid arthritis. This condition commonly pre
lateral eye pain, headache, and nausea and vomiting. Occa sents with pain and swelling located over die radial styloid.
sionally, patients may present with only nausea and vomiting The pain occurs with use of the thumb. On examination,
and be mistaken as having cardiac or abdominal pathology. there is localized tenderness of the distal radial styloid. Pain
Physical examination may show conjunctival erythema; a is elicited with both resisted thumb abduction and exten
sluggish or nonreactive, mid-range dilated pupil; corneal sion. The Finkelstein test is frequendy positive (as with this
cloudiness; and, on funduscopic examination, cupping of die patient).
optic nerve. Treatment in this case would be immediate refer Carpometacarpal arthritis presents widi pain at die
ral to an ophthalmologist or emergency department for ini base of the thumb that occurs with thumb gripping and
tiation of topical P-adrenergic antagonists and pilocarpine pinching. This pain may radiate into the distal forearm. On
and carbonic anhydrase inhibitors.
examination, there is tenderness to palpation on bodi the
Central retinal artery occlusion (CRAO) classically dorsal and palmar surfaces of the joint. Compressing the
presents in a 50- to 70-year-old patient as a painless, abrupt joint by applying a longitudinal load frequendy produces

224
Answers and Critiques

pain. In advanced cases, joint stiffness and loss of range of jaw MRI is the imaging procedure of choice but is not indi
motion may be present. The absence of diese findings, as cated in most patients.
well as the patient's young age, argues against car For patients with chronic temporomandibular disor
pometacarpal arthritis as the cause of his pain. ders, cognitive-behavioral therapy has been shown to
A ganglion is a cyst that forms on the tendon sheath reduce pain, depression, and interference with activities.
and results from inflammation, often following trauma. Biofeedback may also be of value. Jaw appliances and
The anatomic location of the patient's pain and the absence occlusal splints have been a prominent part of temporo
of a palpable cystic structure do not support this diagnosis. mandibular disorder therapy for years despite questionable
Patients with a scaphoid fracture usually have a his evidence of benefit.
tory of an injury that involves wrist dorsiflexion. Pain is KEY POINT
located in the anatomic snuffbox (the radial side of the
Initial treatment of temporomandibular disor
wrist between the abductor and long thumb extensor ten
ders focuses on noninterventional, nonpharma
dons just distal to the radial styloid). On examination,
there is significant tenderness to palpation of the anatomic cologic strategies.
snuffbox.
Bibliography
KEY POINT Mujakperuo HR, Watson M, Morrison R, Macfarlane TV. Pharmaco
logical interventions for pain in patients with temporomandibular
de Quervain tenosynovitis is pain that occurs disorders. Cochrane Database Syst Rev. 2010;(10):CD004715.
with thumb use, characterized by pain and [PMID: 20927737]
swelling over the radial styloid that is is elicited
with both resisted thumb abduction and
extension. Item 98 Answer: D
Educational Objective: Manage peripheral
Bibliography vertigo.
Moore JS. De Qucrvain's tenosynovitis. Stcnosing tenosynovitis of the
first dorsal compartment. J Occup Environ Med. 1997;39( 10):990- This patient should be referred for vestibular rehabilitation,
1002. [PMID: 9343764) which involves balance exercises and physical therapy. She
presents with persistent symptoms of benign paroxysmal
positional vertigo (BPPV), the most common cause of ver
Item 97 Answer: D tigo. Patients with BPPV classically report recurrent
Educational Objective: Manage temporo episodes of vertigo with rapid change in head position. The
mandibular joint disorder. Dix-Hallpike maneuver is performed by having the exam
iner support the patient's head in 20 degrees of extension
This patient likely has a temporomandibular disorder. Initial while sitting, then assisting the patient rapidly to the supine-
treatment of temporomandibular disorders focuses on non position with the head turned to one side and hanging over
interventional, nonpharmacologic strategies. Jaw relaxation, the edge of the examining table; die test is repeated with
heat, and therapeutic exercises may be helpful. Temporo die head turned to the other side. A positive test results in
mandibular disorders are more common in women and typ horizontal nystagmus with latency 2 to 40 seconds and
ically present in the third or fourth decade of lite. Patients duration less than 1 minute with reproduction of symptoms
typically report unilateral jaw discomfort in die masticatory that will fatigue and habituate.
muscles, often with radiation to die ear or posterior neck; BPPV is thought to bc caused by movement of debris
chewing almost always makes the pain worse. Some patients in the semicircular canals and resultant perturbation of sen
report a history of clicking with jaw movement. sory receptors. The Epley maneuver is a particle reposi
There is insufficient evidence for or against the effec tioning procedure intended to move die debris within the
tiveness of simple analgesics such as ibuprofen or the use of semicircular canal; it can successfully relieve symptoms in
tricyclic drugs to treat temporomandibular disorders. Evi more dian 60% of patients. In diis patient, however, the
dence for the effectiveness for selective serotonin reuptake maneuver was unsuccessful, and, based on her symptoms,
inhibitors such as fluoxetine is lacking. Furthermore, flu farther treatment is warranted. Vestibular rehabilitation,
oxetine and paroxetine should be avoided as diey can some especially if initiated early, is effective in treating peripheral
times cause bruxism and exacerbate the problem. vertigo. Studies have shown improvement in symptoms,
The diagnosis of temporomandibular disorders is pri balance, and activities of daily living. Referral to a therapist
marily clinical. Radiography is usually not helpful but may trained in vestibular rehabilitation is recommended.
be considered when dental disease is suspected based on With a classic presentation for BPPV and no focal neu
visual inspection or tooth percussion or if the patient does rologic findings, brain imaging is not indicated in this
not respond to conservative therapy. If structural (nonden- patient. The severity of vertiginous symptoms is highly con
tal) changes of the jaw are suspected and the patient has not cerning for many patients, who seek reassurance that their
responded to conservative therapy or jaw locking is present, condition is benign by undergoing imaging. However, such

225
Answers and Critiques

testing is typically of very low yield and does not alter die allowed to lie below the horizontal and turned to one side.
course of clinical management. The maneuver is repeated with the head turned to the other
Meniere disease is characterized by die symptomatic side. The examiner observes for nystagmus and vertigo.
triad of vertigo, tinnitus, and hearing loss, although it can Dizziness is not a symptom in this patient.
present with vertigo alone. The vertigo is usually episodic The Mini-Mental State Examination (MMSE) is a val
and may not be positional. It can be accompanied by a sen idated instrument to test cognition. Although the MMSE
sation of aural fullness. Diuretic therapy (hydrochlorothi is an appropriate screening test in older adults, this patient's
azide) is advocated for patients with this disease by some normal orientation and lack of symptoms regarding cogni
experts, but robust efficacy data are lacking. This patient's tion would make this test less of a priority at this time. Fur-
presentation is most consistent with BPPV, and therapeu diermore, false-positive MMSE results can be seen in
tic interventions should be targeted toward this diagnosis. patients with untreated depression.
In general, medications are ineffective for die treat In the Timed "Up & Go" (TUG) test, the patient is
ment of BPPV. Benzodiazepines and centrally acting anti asked to arise from a chair, walk 10 feet, turn around, and
cholinergic antihistamines (meclizine) may modify the return to sit in the same chair. Those completing the task
intensity of symptoms for some patients, particularly those in more than 14 seconds are considered to be at high risk
with vestibular neuritis. When medications are used, they for subsequent falling. Aldiough the TUG test is appropri
should be prescribed for less than 24 to 48 hours, since ate for assessing fall risk, it would not address the causes of
longer use may prolong symptoms by suppressing vestibu his diminished activity or his somatic symptoms.
lar feedback and central compensation mechanisms.
KEY POINT
KEY POINT Elderly patients with depression may present
Vestibular rehabilitation is effective in treating with somatic and vegetative symptoms rather
peripheral vertigo. than dysphoria.

Bibliography Bibliography
Bhattacharyya N, Baugh RE, Orvidas L, et al. Clinical practice guide Kroenke K, Spitzer RL, Williams IB. The PHQ-9: validity of a brief
line: benign paroxysmal positional vertigo. Otolaryngol Head Neck depression severity measure. } Gen Intern Med. 2001;16(9):606-
Surg.2008;139(5)(suppl4):S47-S81. [PMID: 18973840] 613. [PMID: 11556941]

Item 99 Answer: C Item 100 Answer: D


Educational Objective: Diagnose depression in Educational Objective: Diagnose posttraumatic
an older adult. stress disorder.
This patient should be assessed for depression. Elderly This patient most likely has posttraumatic stress disorder
patients with depression may present with somatic and (PTSD). PTSD occurs in response to exposure to a trau
vegetative symptoms rather than dysphoria. The PHQ-9 matic event that involves serious threat to oneself or oth
is a validated depression assessment tool with a sensitivity ers. PTSD is characterized by at least 1 month of symp
of 80% and specificity of 92% for major depression using toms that include intrusive thoughts about the trauma,
a score 10 or above as a cut off value. Administering the nightmares or flashbacks, avoidance of reminders of the
PHQ-9 in this patient is appropriate because depression in event, and hypervigilance widi sleep disturbance. To meet
older adults is often underdiagnosed as changes in mood DSM-IV criteria, the symptoms must be in each of three
or behavior are often perceived as a consequence of areas: re-experiencing the event, avoiding reminders of
underlying chronic illnesses. This patient's risk factors for the event, and heightened arousal. Comorbid psychiatric
depression include living in an institutional facility, conditions are common, so screening for depression, anx
chronic illness, and the recent death of his wife. The
iety, and substance abuse is essential. PTSD symptoms
PHQ-9 assesses anhcdonia, depressed mood, sleeping dif often require multimodal treatments, and early interven
ficulties, decreased energy, changes in appetite, feelings of tion may prevent chronicity of symptoms. Trauma-
guilt or worthlessness, concentration impairment, psy focused cognitive-behavioral therapy focuses on cogni-
chomotor changes, and suicidal ideation. Alternatively, tively refraining distorted thinking patterns while
the Geriatric Depression Scale, consisting of 15 questions, gradually re-exposing the patient to the traumatic experi
could be administered. Its sensitivity of 80% to 90% and ence. Simple stress management interventions can also be
specificity of 70% to 85% are similar to the sensitivity and helpful in alleviating some symptoms. The most effective
specificity of the PHQ-9. pharmacotherapy is selective serotonin reuptake
The Dix-Hallpike maneuver is often performed in inhibitors. However, no advantage has been identified for
patients with dizziness to assess for benign paroxysmal posi either psychotherapy or pharmacotherapy.
tional vertigo. To perform the maneuver, the patient is Generalized anxiety disorder is characterized by exces
directed to assume the supine position with the head sive anxiety and worry about a variety of events or activities

226
Answers and Critiques

on most days for at least 6 months, with difficulty control category for cardiac-related pain. Coupled with her elec
ling worrying. Physical symptoms of headache, nausea, and trocardiogram (ECG) showing anterolateral ST-segment
tremulousness may occur. In generalized anxiety disorder depression, her clinical picture is consistent widi coronary
there is usually not a link to a particular inciting incident, ischemia as die cause of her chest pain. She dierefore
and interfering nightmares arc not a hallmark. requires emergent treatment for coronary ischemia and
A major depressive episode is diagnosed according to admission to a coronary care unit.
die DSM-IV by die presence of five or more of die nine car Low-risk patients without evidence of myocardial
dinal symptoms of depression during the same 2-week infarction can be evaluated with an exercise or pharmaco
period, at least one of which is either depressed mood or logic stress test. However, a stress test in a patient with
loss of interest or pleasure. Symptoms should represent a probable acute coronary syndrome could provoke an
change from previous functioning and cause clinically sig extension of her myocardial infarction or a life-threatening
nificant distress or impairment in functioning. This patient arrhythmia.
docs not have significant mood disturbance or anhedonia. CT pulmonary angiography would be helpful if diere
Patients widi obsessive-compulsive disorder report were a high probability of acute pulmonary embolism.
recurrent obsessions or compulsions sufficiently severe to Because diis patient has symptoms and ECG findings of
occupy 1 hour daily or result in marked distress or impaired acute coronary syndrome and die probability of pulmonary
social function. Obsessions are persistent ideas, thoughts, embolism is low (she has no risk factors or physical exami
impulses, or images experienced as intrusive and are associ nation findings to support the diagnosis of venous throm
ated widi significant anxiety or distress. Examples include boembolism and her symptoms can be explained by an
fears of having left doors unlocked and fears of germ cont alternative diagnosis), a CT pulmonary angiogram is not
amination. Compulsions are repetitive behaviors, such as indicated.
handwashing, checking, ordering, or counting, that are NSAIDs are indicated for the treatment of acute peri
repeated to decrease the anxiety related to the obsessions. carditis or musculoskeletal chest wall pain. The pain of
This patient's presentation is not consistent with obsessive- pericarditis is characteristically pleuritic in nature. In addi
compulsive disorder. tion, the characteristic ECG finding in pericarditis is ST-
segment elevation throughout the precordial and limb
KEY POINT
leads rather than regional ST-segment depression, as in
Posttraumatic stress disorder is characterized this patient. ECG changes are not present in muscu
by at least 1 month of symptoms that include loskeletal chest pain.
intrusive thoughts about the trauma, night
KEY POINT
mares or flashbacks, avoidance of reminders of
the event, and hypervigilance with sleep distur Assessment of chest pain is based on preexisting
bance; symptoms must be in each of three risks for cardiac or other diseases, elements of
areas: re-experiencing the event, avoiding the history and physical examination, and
reminders of the event, and heightened arousal. appropriate, directed testing based on the likely
cause of chest symptoms.
Bibliography
Ravindran LN, Stein MB. Pharmacotherapy of PTSD: premises, prin Bibliography
ciples, and priorities. Brain Res. 2009;1293:24-39. [PMID: Panju AA, Hemmelgarn BR, Guyatt GH, Simcl DL. The rational clin
19332035] ical examination. Is this patient having a myocardial infarction?
JAMA. 1998;280(14):1256-1263. [PMID: 97863771

Item 101 Answer: B


Item 102 Answer: B
Educational Objective: Manage chest pain due to
an acute coronary syndrome. Educational Objective: Screen for essential
hypertension.
This patient should be admitted to the coronary care unit
for further treatment. Because chest pain is a common clin It is recommended that diis patient be screened again for
ical symptom and may have noncardiac causes, assessment hypertension in 2 years. Aldiough there are not strong data
of the probability that chest pain has a cardiac etiology is on outcomes with different screening intervals, the Seventh
critical to pursuing appropriate diagnosis and treatment. Report of die Joint National Committee on Prevention,
This is done through an understanding of existing cardiac Detection, Evaluation, and Treatment of High Blood Pres
risk factors, die nature of die presenting symptoms, find sure (INC 7) recommends that persons with blood pres
ings on physical examination, and the results of specific ini sure below 120/80 mm Hg should be screened every 2
tial diagnostic studies, such as chest radiography and elec years. The mean of two or more seated clinic measurements
trocardiography. Her cardiac risk factors include her age, should be used to evaluate for possible hypertension.
hypertension, and hyperlipidemia, and the substernal The INC 7 recommends that yearly screening should
nature of her chest pain places her in an intermediate risk be reserved for patients with systolic blood pressures of 120

227
Answers and Critiques

mm Hg to 139 mm Hg and diastolic blood pressures of 80 cognition relative to those with normal cognitive func
mm Hg to 89 mm Hg. tion. The annual incidence rate of dementia in the gen
Benefits of screening for hypertension at intervals eral elderly population is 1% to 3%, whereas the annual
longer than 2 years have not been established. incidence rate of dementia in patients with a diagnosis
of MCI is approximately 12%, suggesting a significant
KEY POINT
increased risk of progression to dementia from their
All adults aged 18 years and older should be baseline level of cognitive impairment.
screened for hypertension; screening every 2
KEY POINT
years is appropriate for those with blood pres
sure below 120/80 mm Hg. Although there is no widely accepted treatment
for mild cognitive impairment, cognitive reha
Bibliography bilitation has been shown to have some effec
Chobanian AV, Bakris GL, Black HR, et al; Joint National Commit tiveness in improving functioning in some
tee on Prevention, Detection, Evaluation, and Treatment of High
Blood Pressure; National Heart, I.ung, and Blood Institute; patients.
National High Blood Pressure Education Program Coordinating
Committee. Seventh report of the Joint National Committee on Bibliography
Prevention, Detection, Evaluation, and Treatment of High Blood Kinsella GJ, Mullaly E, Rand E, et al. Early intervention for mild cog
Pressure. Hypertension. 2003;42(6):1206-1252. [PMID: nitive impairment: a randomised controlled trial. J Neurol Ncuro-
146569571 surg Psychiatry. 2009;80(7):730-736. [PMID: 19332424]

Item 103 Answer: B Item 104 Answer: B


Educational Objective: Manage mild cognitive Educational Objective: Evaluate a patient with
impairment. pleuritic chest pain.
This patient has mild cognitive impairment (MCI) as evi This patient should undergo radiography of the chest.
denced by reported memory loss, some impaired func He has severe COPD and findings consistent with spon
tioning, and no involvement of other cognitive domains. taneous secondary pneumothorax. These findings
Her Mini-Mental State Examination (MMSE) score of include sudden, sharp, nonradiating pleuritic chest pain
26 is typical for MCI. She does not demonstrate involve and shortness of breath with hyperresonance, decreased
ment of other domains of mental impairment that might breath sounds, and decreased chest wall expansion on the
suggest dementia, including problems with executive side of the pneumothorax in a patient with underlying
functioning, language difficulties, or activities of daily liv lung disease. Pneumothorax should be considered in any
ing. Patient concern about memory loss is more likely patient widi sudden onset of pleuritic chest pain and dysp
with MCI than it is with dementia, in which concerns are nea. The diagnostic test of choice if pneumothorax is sus
usually raised by family members. Although there is no pected is an upright chest radiograph. Findings on chest
widely accepted treatment for MCI, cognitive rehabilita radiograph include separation of the parietal and visceral
tion has been shown to have some effectiveness in
pleura by a collection of gas and the absence of vessels in
improving functioning in some patients. Cognitive reha this space. The diagnosis of pneumothorax may be diffi
bilitation is performed by neuropsychologists and occu cult in patients with COPD because the pleural line may
pational therapists and involves using external memorv be difficult to visualize in hyperlucent lung tissue, and a
aids as well as teaching patients organizational and atten
pneumothorax may be difficult to distinguish from a
tion skills.
large bulla.
In patients with Alzheimer disease, anticholinesterase Chest CT also can be used to diagnose a pneumoth
inhibitors, such as donepezil, galantamine, and rivastig orax. Chest CTs may be more sensitive in delineating
mine, may be tried because their use may result in modest smaller collections of gas in the pleural space and in pro
improvement of cognition, performance of activities of viding more information about the pulmonary par
daily living, and functioning, as determined by global enchyma and pleura. Plain film radiograph)' remains the
assessment. However, the use of anticholinesterase initial test of choice for most patients, however, and CT of
inhibitors has not been shown to delay die progression the chest should be reserved for patients in whom the
from MCI to dementia. chest radiograph does not provide information to guide
Although PET scanning can detect pathologic levels of further treatment or evaluation.
amyloid in patients with MCI and dementia, and possibly The patient's history and physical examination are
differentiate between the two, its use at this time is still classic for pneumothorax and his pain descriptors do
investigational. not strongly suggest ischemia or other primary cardio
Reassurance that the patient will not progress to vascular disease. An electrocardiogram or echocardio
dementia is not appropriate because persons with MCI gram, tests of choice to evaluate ischemic heart disease,
appear to be at increased risk for further declines in valvular heart disease or cardiomyopathy, would not be

228
Answers and Critiques

the first diagnostic test of choice for suspected KEY POINT


pneumothorax. Criteria for somatization disorder include a
KEY PO NT constellation of medically unexplained, persis
Pneumothorax should be considered in any tent symptoms, including gastrointestinal, pain,
pseudoneurologic, and sexual, that begin before
patient with sudden onset of pleuritic chest
age 30 years and cause significant impairment
pain and dyspnea, and the diagnostic test of for die patient.
choice if pneumothorax is suspected is an
upright chest radiograph. Bibliography
Oyama O, Paltoo C, Greengold I. Somatoform disorders. Am Fam
Bibliography Physician. 2007;76(9):1333-1338. [PMID: 18019877]
Noppen M. Spontaneous pneumothorax: epidemiology, pathophysi
ology and cause. Eur Respir Rev. 2010;19(117):217-219. [PMID:
20956196] Item 106 Answer: B
Educational Objective: Treat chronic bacterial
Item 105 Answer: D prostatitis.
Educational Objective: Diagnose somatization The most appropriate treatment for this patient is a 1-
disorder. month course of a fluoroquinolone antibiotic. This patient
has chronic bacterial prostatitis (National Institutes of
This patient most likely has somatization disorder.
Health category II), which presents with pain and urinary
Although medically unexplained symptoms are com
mon, as are patient concerns and anxiety regarding the symptoms with recurrent bacterial infection. The prostate
in patients with chronic bacterial prostatitis may be less
possible presence of disease for which they seek evalua inflamed than with acute prostatitis. The recommendation
tion, somatization disorder is a relatively rare psychiatric
for treatment of category II prostatitis is a prolonged (1
disease related to an extreme focus on medical symp
toms and their evaluation. DSM-IV criteria for somati month) course of a fluoroquinolone antibiotic such as
zation disorder include a constellation of symptoms, ciprofloxacin, which covers common bacterial infections of
the prostate with good penetration of the prostate.
including two gastrointestinal symptoms, four pain
A 1-week course of trimethoprim-sulfamethoxazole
symptoms, one pseudoneurologic symptom, and one would be appropriate for acute bacterial prostatitis (cate
sexual symptom. The symptoms must start before age
30 years, persist (although they often wax and wane), gory I prostatitis) or urinary tract infection; however, this
and cause significant impairment for the patient. In patient has had short-course antibiotics for three prior
infections, placing him in category II and warranting a
addition, each symptom must be medically unexplained
after evaluation. Somatization disorder should always longer course of antibiotics.
Category III prostatitis (chronic abacterial prostatitis/
be distinguished from depression with somatic features;
chronic pelvic pain syndrome) is noninfectious and dierefore
the latter will meet diagnostic criteria for depression.
does not respond to antibiotics. This patient's urinary findings
This patient meets DSM-IV criteria for somatization
of bacteria and leukocytes support an infectious cause of his
disorder.
Celiac disease may be difficult to diagnose and may svTnptoms, and not this form of prostatitis. There is some evi
dence diat cognitive-behavioral therapy may provide some ben
cause diffuse symptoms, including nausea, but would most
efit to patients with chronic pelvic pain syndrome, aldiough there
likely cause weight loss and odier gastrointestinal symp is not a role for this intervention in bacterial prostatitis. S)Tnp-
toms. In addition, celiac disease would not fully explain the
toms of category III chronic pelvic pain sviidrome are often
focal neurologic and other symptoms experienced by the
refractory, and enipathetic supportive care is often required.
patient. Finasteride is a 5-a-reductasc inhibitor that decreases
Patients who are malingering consciously fabricate
prostate volume and is used primarily in the treatment of
symptoms for some secondary gain, whereas patients with
benign prostatic hyperplasia. It does not have an established
somatoform disorders are unaware that dieir symptoms are
use in either acute or chronic bacterial prostatitis.
a manifestation of psychiatric disease. This patient's history
gives no suggestion of secondary gain. KEY POINT
Patients widi multiple sclerosis can have nausea. How The recommended treatment for chronic bacte
ever, this patient's neurologic symptoms affect the right rial prostatitis is a prolonged course of a fluo
face and body and thus do not follow neuroanatomy. Fur roquinolone antibiotic.
thermore, multiple sclerosis is an unlikely cause of the
patient's multiple somatic symptoms, and the patient's Bibliography
recent neuroimaging study did not show evidence of Touma NJ, Nickel JC. Prostatitis and chronic pelvic pain syndrome in
demvelination. men. Med Clin North Am. 2011;95:75-86. [PMID: 21095412]

229
Answers and Critiques

Item 107 Answer: B sensorineural hearing loss (SSNHL) is an alarming problem


Educational Objective: Manage acute sinusitis. that is defined as sensorineural hearing loss occurring in 3
days or less. Patients often report immediate or rapid hear
This patient with clinical findings typical of acute sinusitis
ing loss or loss of hearing upon awakening. Ninety percent
should be observed and given symptomatic treatment, such have unilateral hearing loss, and some have tinnitus, ear full
as chlorpheniramine. Most cases of acute sinusitis are ness, and vertigo. SSNHL constitutes a considerable diag
caused by viruses and typically resolve in 7 to 10 days widi nostic challenge because it may be caused by many condi
out directed therapy. The clinical presentation is not help
tions, including infection, neoplasm, trauma, autoimmune-
ful in determining whether the cause of symptoms is viral
disease, vascular events, and ototoxic drugs. Immediate
or bacterial. However, because most cases of "viral or bacte
otorhinolaryngologic referral is required. Improvement
rial sinusitis resolve spontaneously within 10 days, observa occurs in about two thirds of patients. Oral or intratym-
tion and treatment of the associated symptoms with anal
panic corticosteroids are usually given, although random
gesics and decongestants is appropriate. ized trials differ in their conclusions regarding efficacy.
Antibiotics are generally reserved for sinusitis accom Otic herpes zoster (Ramsay Hunt syndrome) is char
panied by high or continued fever or worsening symptoms, acterized by herpetic lesions in the external canal and ipsi
and even in this setting, their efficacy is not well docu lateral facial palsy neither of which is seen in diis patient.
mented. When used, an antibiotic focused on common res
Acyclovir may be considered in a clear case of Ramsay Hunt
piratory organisms is reasonable. syndrome but has been shown to be unhelpful in idiopathic
Nasal cultures have not been shown to bc helpful in SSNHL.
diagnosing a bacterial etiology for sinusitis or in guiding Neomycin, polymyxin B, and hydrocortisone car drops
antibiotic therapy. are a possible treatment for acute otitis externa. This patient
Sinus imaging is not part of the initial management of is unlikely to have otitis externa because she does not have
acute sinusitis because imaging results are frequendy abnor
otalgia, otorrhea, itching, or pain intensified by jaw motion.
mal in symptomatic patients with either a viral or bacterial She does not have internal tenderness when the tragus or
sinusitis, and also in a high percentage of asymptomatic
pinna is pushed or pulled. Her ear canal erythema is most
patients. Imaging is generally indicated in patients with a likely secondary to the trauma of recent cerumen removal
complicated presentation, such as those with visual changes than otitis externa.
or severe headache.
Triethanolamine ear drops may help to treat or prevent
Inhaled nasal corticosteroids are frequently pre cerumen impaction, but cerumen impaction causes con
scribed for acute symptom relief for sinusitis and have ductive hearing loss, not sudden sensorineural hearing loss.
some efficacy in this setting; however, the role of systemic
After her cerumen was successfully removed, the patient's
corticosteroids in acute sinusitis is not clear, and they are conductive hearing was intact, making diis an unlikely cause
not recommended.
of her sudden hearing loss in her left ear. Cerumen
KEY POINT impaction is also unlikely to cause tinnitus.
Most cases of viral or bacterial sinusitis resolve KEY POINT
spontaneously within 10 days, and observation Patients with sudden sensorineural hearing
and treatment of the associated symptoms with
loss should be urgently evaluated by
analgesics and decongestants is appropriate.
audiometry and considered for oral or
Bibliography intratympanic corticosteroid treatment by
an otorhinolaryngologist.
Chow AW, Benninger MS, Brook I, et al. IDSA clinical practice guide
line for acute bacterial rhinosinusitis in children and adults". Clin
Infect Dis. 2012;54(8):e72-cll2. [PMID: 22438350] Bibliography
Stachler RJ, Chandrasekhar SS, Archer SM, et al; American Academy
of Otolaryngology-Head and Neck Surgery. Clinical practice guide
line: sudden hearing loss. Otolaryngol Head Neck Surg
Item 108 Answer: D
2012;146(3 suppl):Sl-S35. [PMID: 22383545]
Educational Objective: Manage sudden sen
sorineural hearing loss.
Item 109 Answer: D
This woman widi sudden-onset unilateral sensorineural
hearing loss requires urgent audiometry and otorhino-
Educational Objective: Employ the principle of E
I
substituted judgment in managing care of a patient
laryngology referral because early diagnosis and treatment
without decisional capacity.
may be associated with improved outcomes. Based on die
initial examination, this patient does not have conductive
Although the patient is unconscious and unable to make his
hearing loss because she hears better when sound is trans own decisions, die ethical obligation of bodi die practitioner
mitted via air (through the external ear canal and middle- and the surrogate decision maker (his wife) is to continue to
ear) than when it is transmitted via bone vibration. Sudden make decisions that are consistent widi his previously

230
Answers and Critiques

expressed wishes and values. The principle, called "substi does not result in malpositioning of the testicle or an absent
tuted judgment," essentially asks, "What would the patient cremasteric reflex.
want if he or she could decide?" Clinical presentations of inguinal hernias can vary
In patients who present to the emergency department from an asymptomatic bulge to a feeling of groin or
unable to make decisions, lifesaving therapy is both ethical abdominal pressure to severe pain when incarceration or
and necessary under the principle of implied consent. strangulation occurs. A strangulated hernia may present as
However, once a patient's wishes are known, it is unethical a painful mass in the scrotum or as a tender bulge in the
to specifically defy those wishes simply because the patient inguinal area; signs of bowel obstruction may also be pres
has lost decisional capacity; therefore, it would bc unethi ent. This patient docs not have findings consistent with a
cal to transfuse the patient knowing that he specifically did strangulated inguinal hernia.
not want transfusion. Orchitis, an inflammation of the testicle, is usually
Obtaining a court-appointed guardian is not indicated caused by viral infection (mumps) or extension of a bacte
in this case because the patient clearly stated his views, and rial infection from epididymitis or urinary tract infection; in
subsequent care decisions will be made by his duly mumps, parotiditis begins about 5 days prior to orchitis.
appointed surrogate based on his wishes. The testicle is diffusely tender and may be swollen; the posi
Seeking the permission of the patient's wife to allow tion of die testicle in the scrotum is normal and the cre
transtiision or attempting to convince her that transfusion masteric reflex is present.
would be in her husband's best interest is ethically unac
KEY POINT
ceptable; all available evidence suggests diat he was consis
Testicular torsion is characterized by severe
tent in his wishes to avoid transfusion, and it would be inap
propriate to place her in a difficult ethical position, pain and an elevated high-riding testicle with
especially in this stressful situation. the longitudinal axis abnormally oriented trans
versely and an absent cremasteric reflex.
KEY POINT
When a patient is unable to make his or her Bibliography
own decisions, the ethical principle of substi Wampler SM, Llanes M. Common scrotal and testicular problems.
tuted judgment obliges surrogate decision mak Prim Care. 2010;37(3):613-626, x. [PMID: 20705202]
ers to make decisions that are consistent with
the patient's previously expressed wishes and
I t e m 111 A n s w e r : C
values.
Educational Objective: Treat bacterial vaginosis.
Bibliography The most appropriate treatment for this patient is a 7-day
Snyder L; American College of Physicians Ethics, Professionalism, and
Human Rights Committee. American College of Physicians Ethics oral regimen of metronidazole (500 mg twice daily). She
Manual: sixth edition. Ann Intern Med. 2012;156(1 Pt 2):73-104. has bacterial vaginosis (BV), a polymicrobial infection
[PMID: 22213573] characterized by an overgrowth of multiple anaerobic bac
teria. Although BV is not a sexually transmitted infection,
risk factors include lack of condom use and multiple or
I t e m 11 0 A n s w e r : D new sexual partners. BV can be diagnosed clinically using
Educational Objective: Diagnose testicular Amsel criteria, which include the following symptoms or
torsion. signs: (1) homogeneous tiiin discharge that coats the vagi
nal walls; (2) clue cells (epithelial cells with borders
This patient has testicular torsion, which occurs when the
obscured by small bacteria) on saline microscopy; (3) pH
testicle twists on the spermatic cord, leading to decreased
blood flow and ischemia. It is more common in children of vaginal fluid >4.5; and (4) fishy odor of vaginal dis
and in men younger than 30 years. Pain is usually sudden charge before or after the addition of 10% potassium
in onset and examination often reveals a high-riding testi hydroxide to the secretions (the "whiff test). The pres
cle with the longitudinal axis abnormally oriented trans ence of at least three of diese clinical findings has a high

versely. Absence of the cremasteric reflex on die affected sensitivity and specificity for diagnosing BV when com
side is nearly 99% sensitive for torsion. Treatment of torsion pared with Gram stain of collected secretions, which is the
includes rapid surgical decompression to resume blood gold standard. Because this woman is symptomatic, treat
flow. In the absence of rapid access to surgery, manual ment should be offered with either oral metronidazole,
decompression may be attempted. vaginal metronidazole gel, or vaginal clindamycin cream;
Men widi epididymitis typically present widi subacute patient preference should dictate treatment choice. Topi
onset of scrotal pain, dysuria, urinary frequency, and fever. cal clindamycin should be avoided during pregnancy as it
Inflammation and infection of the epididymis cause pain may increase the risk of adverse outcomes. Women treated
with oral metronidazole should be cautioned to avoid
localizing to the posterior and superior aspect of the testi
cle. The scrotum may be edematous and erythematous. It alcohol, which can cause a disulfiram-like reaction.

231
Answers and Critiques

This patient's abnormal vaginal pH and lack of yeast (premature ejaculation or hypoactive sexual desire disor
and hyphae on potassium hydroxide preparation make vul der). An adequate trial is generally deemed to constitute
vovaginal candidiasis an unlikely explanation for her symp patient reeducation and gradual escalation of the dose to
toms. Treatment with oral fluconazole or topical clotrima maximum dose and at least six attempts on maximum-
zole cream, effective dierapies for vulvovaginal candidiasis, dose therapy. PDE-5 inhibitors should be avoided in
is not warranted. patients receiving any form of nitrate dierapy.
This patient has multiple sexual partners, increasing Second-line dierapies for ED should be reserved for
her risk for trichomoniasis. Characteristic symptoms and men who fail to improve with lifestyle modifications or with
signs include a malodorous discharge with vulvar itching, use of PDE-5 inhibitors. These include alprostadil (either
burning, and postcoital bleeding. Although the vaginal pH intraurethral or intracavernosal), penile pumps, and penile
will be elevated and the whiff test may be positive (as noted prostheses. Intraurethral alprostadil is more effective than
with this patient), clue cells are not a characteristic finding that administered intracavernosally and is associated with
on saline microscopy, making trichomoniasis an unlikely fewer side effects.
diagnosis in this patient. Testosterone replacement therapy should be limited to
Oral metronidazole is also used for the treatment of patients with clinical symptoms and signs consistent with
trichomoniasis, but it is typically given as a single 2-g dose, androgen deficiency and a subnormal serum testosterone
which would not be appropriate for treatment of BV. level (generally an 8 am total testosterone level <200

KEY POINT
ng/dL [7 nmol/L]).
Bacterial vaginosis is the likely diagnosis in KEY POINT

women with at least three of the following fea First-line pharmacologic therapy for erectile
tures: (1) homogeneous thin discharge that dysfunction consists of phosphodiesterase type
coats the vaginal walls; (2) clue cells on saline 5 inhibitors.
microscopy; (3) pH of vaginal fluid >4.5; and
(4) fishy odor of vaginal discharge (positive Bibliography
"whiff' test). Tsertsvadzc A, Fink HA, Yazdi F, et al. Oral phosphodiesterase-5
inhibitors and hormonal treatments for erectile dysftinction: a sys
tematic review and meta-analysis. Ann Intern Med.
Bibliography 2009;151(9):650-661. [PMID: 19884626]
Wilson, JF. In the clinic. Vaginitis and cervicitis. Ann Intern Med.
2009;151(5):ITC3-1-ITC3-15. [PMID: 19721016]
I t e m 11 3 A n s w e r : C
I t e m 11 2 A n s w e r : C Educational Objective: Prevent medication errors
from occurring during a transition in care.
Educational Objective: Treat erectile
dysfunction. The most likely cause for this patient's readmission is a
medication error stemming from her medication changes at
Since this patient has not yet taken anything for his erec
tile dysfunction (ED), it would be most appropriate to discharge. It is likely that she cither did not receive or did
not take the medications at the increased dosages. One in
treat him with sildenafil, a phosphodiesterase type 5 (PDE-
five patients discharged from the hospital will suffer an
5) inhibitor. PDE-5 inhibitors are generally considered adverse event related to medical management within 3
first-line pharmacologic dierapy for ED and include silden
weeks of hospital discharge, with 66% of these being
afil, vardenafil, and tadalafil. These agents increase penile adverse events related to medications. Most medication
cyclic guanosine monophosphate (cGMP), facilitating errors result from inadequate communication by hospital
smooth muscle relaxation and allowing inflow of blood.
caregivers with patients and their primary care clinicians.
All of these drugs have been shown to improve erectile
Medication reconciliation is the process by which medica
function, as measured by successful sexual intercourse tions are reviewed at every step of die care process, widi a
attempts and improved scores on various survey instru focus on ensuring that the patient is taking only diose med
ments. There are no direct comparisons to support supe ications intended, and that this is clear to the patient and all
riority of any one agent, as the populations studied differ others involved in diat patient's care. Patients should
in various characteristics. receive a list of medications at the time of discharge, be
The PDE-5 inhibitors vary in their duration of informed of previous medications that have been discon
action, interaction with food and other medications, and tinued or changed, any new medications that have been
adverse effects. Treatment failure may result from lack of
added, and the reasons for these changes.
patient education or improper use (timing, taking with True diuretic resistance is uncommon, aldiough the
food, inadequate sexual stimulation, inadequate dose,
bioavailability of oral diuretics may be highly variable, par
inadequate trial), performance anxiety or unrealistic ticularly in die edematous state. She responded to intra
expectations, hypogonadism, or an incorrect diagnosis venous diuretics as an inpatient with a return of her weight

232
Answers and Critiques

to a nonedematous level, and her oral diuretic dose was diagnosis of otitis media and mild symptoms, it would be
appropriately increased at the time of discharge. Her rapid reasonable to withhold antibiotic dierapy.
decompensation from her normal baseline weight on an If an antibiotic was prescribed, amoxicillin is recom
increased dose of diuretic with the addition of a second mended as first-line therapy in adults. Erythromycin could
agent at the time of discharge makes clinically significant be used in a penicillin-allergic patient, but there is no evi
resistance to diuretics unlikely. dence that it is more efficacious.
Inadequate post-hospital follow-up is a potential An otorhinolaryngology consultation is not indicated
cause for readmission, particularly with complex patients at this time because the patient only has a URI.
who have had extended hospitalizations and multiple
KEY POINT
changes to their treatment regimen. In general, for most Do not routinely prescribe antibiotic therapy
patients admitted for heart failure exacerbation, a follow-
for adults with otitis media.
up appointment in 1 week should be scheduled at the
time of discharge, preferably with direct contact with the
Bibliography
primary care physician. This patient was scheduled for a 1 - Coco A, Vernacchio L, Horst M, Anderson A. Management of acute
week follow-up, but worsening of her symptoms shortly otitis media after publication of the 2004 AAP and AAFP clinical
after discharge suggests an issue with treatment of her ini practice guideline. Pediatrics. 2010;125(2):214-220. [PMID:
tial problem, or development of an additional medical 20100746]
complication.
Spironolactone has been shown to decrease mortality
Item 11 5 Answer: D
in selected patients with systolic heart failure. Its primary
Educational Objective: Manage secondary
complications are hyperkalemia and odier effects of aldos
terone blockade. However, it is unlikely to be an indepen insomnia.
dent cause of her acute heart failure decompensation. This patient's history of daytime fatigue and obesity' and
KEY POINT large neck size put him at risk for obstructive sleep apnea,
Hospitalized patients should receive a list of which can be diagnosed with an overnight polysomnog
medications at the time of discharge and be raphy study. It would be helpful to obtain corroborating
information from his wife regarding symptoms of snoring,
informed of previous medications that have
been discontinued or changed. gasping, other breathing problems, or abnormal leg
movements. Referral for polysomnography is indicated
when a primary sleep disorder is suspected (obstructive
Bibliography
Kripalani S, Jackson AT, Schnippcr JL, Coleman EA. Promoting effec sleep apnea, restless legs syndrome, periodic limb move
tive transitions of care at hospital discharge: a review of key issues ment disorder). A sleep study may also include multiple
for hospitalists. J Hosp Med. 2007;2(5):314-323. [PMID: sleep latency testing, in which a patient takes four or five
17935242] 20-minute naps, and sleep latency (the time from decid
ing to sleep to actually falling asleep) is measured. Sleep
I t e m 11 4 A n s w e r : D latency of less than 8 minutes is associated with hyper
somnia, which occurs with sleep disorders such as nar
Educational Objective: Manage upper respiratory
colepsy, insufficient sleep syndrome, medication adverse
tract infection with ear pain. effects, sleep apnea syndromes, and periodic limb move
This patient presents with signs and symptoms of a viral ments of sleep.

upper respiratory tract infection (URI). The recent devel Although alcohol can contribute to insomnia, this
opment of ear pain and the findings of a dull tympanic patient's pattern of alcohol use does not support its having
membrane with a small middle ear effusion are compatible a role in his symptoms, since he is having daily fatigue
with either otitis media or a viral URI without otitis media. symptoms despite only using alcohol on weekends.
Treatment of otitis media in adults has not been well stud Pharmacotherapy would not be appropriate in this
ied. There are no guidelines for antibiotic use in adults sep patient until a secondary cause of insomnia is ruled out.
arate from those for children. In children older than 2 years Prescription drug therapy for insomnia is reserved for
without severe illness, outcomes appear to be similar for patients with primary insomnia who have not benefited
observation without antibiotics compared with antibiotic from nonpharmacologic and behavioral therapies.
treatment. This strategy to reduce use of antimicrobials has Restless legs syndrome is a clinical diagnosis. Besides
not been evaluated in adults, and it is not known if antibi an urge to move the legs, other symptoms that patients
otics are associated with improved short- or long-term out may exhibit in support of this diagnosis include an
comes. However, antibiotic use is associated with adverse uncomfortable or unpleasant sensation in the legs that
effects and higher levels of antibiotic resistance that should may begin or worsen during periods of rest or inactivity,
be considered in conjunction with the lack of evidence is partially or totally relieved by movement as long as
regarding benefit. Considering the patient's equivocal activity continues, and is worse in the evening or night or

233
Answers and Critiques

is present only at night. Resdess legs syndrome can be onset of symptoms and prolonged course are not consistent
divided into primary and secondary forms. The primary widi septic arthritis.
form refers to patients without another condition known
KEY POINT
to be associated with restless legs syndrome. Conditions
associated with secondary restless legs syndrome include Osteonecrosis of the hip commonly presents
with dull, aching groin pain that is indolent in
pregnancy, end-stage kidney disease, and iron deficiency.
Since this patient does not have restless legs syndrome, onset; risk factors include corticosteroid use
iron studies are not indicated. and excessive alcohol use.

KEY POINT
Bibliography
Referral for polysomnography is indicated Amanatullah DF, Strauss EJ, Di Cesare PE. Current management
when a primary sleep disorder is suspected options for osteonecrosis of the femoral head: part 1, diagnosis and
nonoperative management. Am J Orthop (Belle Mead NJ).
(obstructive sleep apnea, restless legs syndrome, 201 l;40(9):E186-92. [PMID: 22022684]
periodic limb movement disorder).

Bibliography I t e m 11 7 A n s w e r : B
Wilson JF: In the clinic: Insomnia. Ann Intern Med. 2008;148(1)-
ITC13-1-ITC13-16. [PMID: 18166757] Educational Objective: Manage a patient with a
chronic pain syndrome.
This patient should be evaluated for possible intimate part
I t e m 11 6 A n s w e r : C ner violence. It is essential to perform a comprehensive
Educational Objective: Diagnose osteonecrosis of assessment in the evaluation of patients with chronic pain
the hip. syndromes. This patient has a chronic pain syndrome,
This patient's presentation is most consistent with fibromyalgia that had previously been well controlled but
has worsened recendy, significandy affecting her functional
osteonecrosis of the hip. Osteonecrosis of die hip com
status. In patients with chronic pain conditions, overlying
monly presents with dull, aching groin pain (most com
psychosocial stressors, including domestic violence, may
monly) or thigh or buttock pain diat is indolent in onset.
exacerbate or destabilize symptoms. Therefore, in addition
Occasionally, as is die case in this patient, severe pain may to questioning the patient more thoroughly about pain
be reported in die early stages as bone death is occurring.
onset and relieving and exacerbating factors, the provider
Corticosteroid use and excessive use of alcohol account for
should also inquire about threats to the patient's safety as
more than 90% of hip osteonecrosis cases. On examination,
well as other potential psychosocial events or situations that
patients have limited range of motion of the hip. During
the early stages of this disease, radiographic imaging may be may be contributing to her worsened clinical status.
There is no specific role for diagnostic testing or imag
normal. Eventually, patchy areas of sclerosis and lucency
ing in the evaluation of patients with most chronic pain syn
may be seen. Hip MRI is the most sensitive imaging test for dromes in the absence of objective physical examination
osteonecrosis and is typically positive early in the course of
the disease. findings or laboratory abnormalities suggesting a specific
Osteoarthritis of die hip can be established in patients underlying disorder. Although various studies are fre
with a history of chronic pain in the groin and medial thigh quendy obtained in patients widi chronic pain, they are typ
that worsens with activity and is relieved by rest. Aldiough ically not revealing, and abnormalities diat are identified on
osteoarthritis of the hip can present with slowly progressive diagnostic testing may not be the source of the patient's
pain. This patient is experiencing worsening of her typical
hip pain, severe pain at the onset of symptoms does not
fibromyalgia pain, and obtaining muscle and nerve con
usually occur. Radiographs in patients with osteoarthritis duction studies vvidiout focal symptoms or clinical findings
may show joint-space narrowing, subchondral sclerosis, and is unlikely to change management.
osteophyte formation. Although there is a poor correlation NSAIDs are most effective for treating the pain asso
between radiographic evidence of osteoarthritis and symp
ciated with rheumatoid arthritis, inflammatory
toms, it would be unusual for a patient with osteoardiritis
of die hip to have a normal radiographic series. arthropathies, and musculoskeletal pain. They are generally
ineffective in the management of neuropathic and muscu
Ll radiculopathy is rare. Symptoms include pain, lar pain syndromes, such as fibromyalgia, and can be asso
paresdiesia, and sensory loss in the groin. On examina ciated widi significant gastrointestinal and cardiovascular
tion, this patient has limited internal and external range of toxicities. Adding high-dose ibuprofen to this patient's pain
motion and pain with log-rolling of the hip. Each of these
regimen is unlikely to significantly improve her symptoms.
findings supports a joint etiology of this patient's hip pain Opioid therapy should generally be avoided in patients
and argues against lumbar disk disease at the Ll level. with chronic pain syndromes given the expected chronicity
Septic arthritis would be expected to present acutely of use, lack of demonstrated efficacy, and die potential for
widi fever and limited range of motion. This patient's slow
significant side effects and dependency.

234
Answers and Critiques

KEY POINT Bibliography


Patients with chronic pain syndromes should Carr DB, Schvvartzberg JG, Manning L, Scnipck J. Physician's
Guide to Assessing and Counseling Older Drivers. 2nd edition.
be evaluated for concurrent psychosocial stres
Washington, DC. NHTSA. 2010. Available at vvvvvv.ama-assn
sors, particularly in those in whom symptoms .org/ama/puli/physician-resourccs/public-health/promoting
have worsened without explanation. -healthy-lifestyles/geriatric-health/older-driver-safety/assessing
-counscling-oldcr-drivcrs.page.
Bibliography
Bradley LA. Pathophysiology of fibromyalgia. Am J Med.
2009;122(12 suppl):S22-30.'[PMID: 19962493] I t e m 11 9 A n s w e r : D
Educational Objective: Manage panic disorder.
Item 118 Answer: C The most appropriate management of this patient is to
Educational Objective: Manage risk for motor prescribe a selective serotonin reuptake inhibitor. Panic
vehicle accidents in an older adult driver. disorder is a syndrome characterized by sudden panic
attacks with the acute onset of somatic symptoms, which
This patient has a number of factors that increase his risk of may include chest pain, palpitations, sweating, nausea,
being involved in a motor vehicle accident and his physician dizziness, dyspnea, and numbness. These symptoms usu
has a responsibility to reduce this risk. This patient's risk ally last from 5 to 60 minutes. About 50% of patients
factors for a motor vehicle accident include his age, likely with panic disorder also have associated agoraphobia,
visual deficits, decreased motor function (including a his with fears of being in crowds or in places from which
tory of falling), and decreased cognitive function. The first escape would be difficult. Diagnosis is based on clinical
step in assessing driving ability in older adults is to ask the descriptors and setting, but care should be made to con
patient and family members about driving difficulties. This sider underlying medical disorders, such as cardiac dis
assessment should include questions about whether friends
ease, thyroid disease, or pheochromocytoma, particularly
and family members are worried about dieir driving, get in patients at increased risk for one of these disorders.
ting lost while driving, near misses, and recent accidents. A However, extensive testing is not necessary in most
more complete set of questions to assess driving risk can bc
patients with a characteristic presentation and normal
found in die "Am I a Safe Driver" self-assessment tool
physical examination and basic laboratory studies. Treat
(www.ama-assn.org/amal/pub/upload/mm/433/am_i ment options for panic disorder include medication and
_a_safe_drivcr.pdf). A positive response to any of the ques
tions suggests unsafe driving. psychotherapy. Cognitive-behavioral therapy (CBT) has
been shown to be the most effective psychotherapeutic
It would be premature to advise diis patient to stop
intervention in controlled trials. Selective serotonin reup
driving before assessing driving-related skills, providing the take inhibitors and serotonin-norepincphrine reuptake
patient widi information on safe driving, and suggesting inhibitors have been shown to be effective. Panic disor
that the patient enroll in a driving course designed to
der that is severe or refractory appears to be most
improve skills. Referral to a driver rehabilitation specialist amenable to the combination of CBT and pharma
can also assist in assessment and skill improvement.
Advising the patient to drive only locally is not advised cotherapy compared with either treatment alone.
because so-called "low-mileage" drivers may be at the This patient has classic symptoms of panic disorder and
no cardiac risk factors. It would be inappropriate to order
greatest risk. Older drivers who are having driving difficul
ties often self-restrict dieir driving, but local roads often further cardiac testing in the setting of a normal electro
have more hazards, including more signs and signals and cardiogram and classic symptoms. This patient's symptoms
are also atypical for gastroesophageal reflux disease, ren
confusing and congested intersections.
Guidelines for reporting patients to the department of dering empiric proton pump inhibitor therapy an incorrect
choice.
transportation vary by state and include immediate threats
to driving safety such as new seizures. Even in states that KEY POINT
require reporting of immediate threats to driving safety, Panic disorder is characterized by sudden panic
there is no indication to report this patient before a more-
attacks with the acute onset of somatic symp
complete evaluation is performed.
toms, which may include chest pain, palpita
tions, sweating, nausea, dizziness, dyspnea, and
The first step in assessing driving ability in numbness.
older adults is to ask the patient and family
members about driving difficulties, including Bibliography
whether friends and family members are wor Work Group on Panic Disorder; American Psychiatric Association;
Practice guideline for the treatment of patients with panic dis
ried about their driving, getting lost while dri order. Am J Psychiatry. 1998;155(5 suppl):l-34. [PMID:
ving, near misses, and recent accidents. 9585731]

235
Answers and Critiques

Item 120 Answer: A patients in the community have unrecognized OSA.


Educational Objective: Manage a patient with Patients with more than a negligible risk for OSA should be
harmful alcohol use. screened preoperatively. One instrument that may be used
is the STOP-BANG questionnaire, a screen based on eight
This patient is exhibiting harmful use of alcohol and should
parameters: Snoring, Tired, Observed stopping breathing
be counseled appropriately, including connecting her drink during sleep, high blood Pressure, BMI (>35), Age (>50
ing habits with the negative consequences that she has years), Neck circumference (>40 cm), and Gender (male).
recently experienced. Harmful drinking is drinking diat Patients with three or more positive responses have a high
causes physical or psychological harm. This patient's drink risk of OSA. Had he been screened, this patient would have
ing has resulted in serious illness as well as an arrest for dri scored at least 5 points (tiredness, hypertension, high BMI,
ving while intoxicated. Optimal management would age >50 years, male), and would have qualified for evalua
include a discussion of appropriate amounts of alcohol, tion for possible OSA.
negative consequences, and agreement of goals for reduc Although myocardial ischemia may lead to transient
ing alcohol intake. This should be performed in the setting respirator)' failure, diis patient has a negative cardiac his
of frequent follow-up and reassessment and should incor tory, no chest pain, and a normal electrocardiogram, mak
porate the patient's ideas about her drinking behaviors and ing this less likely dian OSA.
ways to change them, barriers she may face in reducing her Extubation following surgery is typically performed
alcohol consumption, and previous experiences with after careful assessment of stability following removal of
attempting to stop or reduce her drinking. sedating and paralyzing medications, with subsequent
Labeling a patient an alcoholic is neither productive monitoring in a postanesdiesia care unit. It is likely that he-
nor a medically useful term. Goals of managing this patient met criteria for extubation following his procedure, but
may not require complete abstinence, and abstinence may patients with unrecognized OSA can develop acute respi
be difficult for die patient to accomplish immediately.
ratory failure induced by the sleep apnea combined with die
Adjunct management strategies may include medica respiratory depression caused by narcotics for postoperative
tions or referral to Alcoholics Anonymous or psychiatry,
analgesia and the lingering effects of anesthesia.
but these measures are more effective when done in com
Sepsis can cause perioperative respiratory failure; how
bination with primary counseling. ever, patients with sepsis are typically febrile, hypotensive,
The National Institute on Alcohol Abuse and Alco and show evidence of a metabolic acidosis. The absence of
holism defines at-risk drinking as more than 14 drinks per
any of these findings makes sepsis a less likely cause of his
week or 4 drinks per occasion in men and more than 7
respiratory failure.
drinks per week or 3 drinks per occasion in women. How
KEY POINT
ever, harmful drinking is defined by consequences and not
by the quantity consumed. Patients with more than a negligible risk for
KEY POINT obstructive sleep apnea (OSA) should be
screened preoperatively; those with three or
Management of harmful drinking patterns more of the following parameters have a high
includes counseling to help patients connect the risk of OSA: snoring, tired, observed stopping
negative consequences to their drinking, discus breathing during sleep, hypertension, BMI
sion of appropriate amounts of alcohol, and
>35, age >50 years, neck circumference >40 cm,
agreement of goals for reducing alcohol intake, and male sex.
performed in a patient-centered manner in a
setting of frequent follow-up and reassessment. Bibliography
Vasu TS, Doghramji K, Cavallazzi R, et al. Obstructive sleep apnea
Bibliography syndrome and postoperative complications: clinical use of the
U.S. Preventive Services Task Force. Screening and behavioral coun STOP-RANG questionnaire. Arch Otolaryngol Head Neck Surg
seling interventions in primary care to reduce alcohol misuse: rec 20l0;l36(l()):1020-1024. [PMID: 20956751]
ommendation statement. Ann Intern Med. 2004;140(7)-554-556
[PMID: 15068984]
Item 122 Answer: C
Item 121 Answer: B Educational Objective: Diagnose rotator cuff
tendinitis.
Educational Objective: Manage a patient with
This patient most likely has rotator cuff impingement syn
suspected obstructive sleep apnea in the postopera
tive period. drome from underlying tendinitis. She presented with pain
in her shoulder that began after performing the repetitive
This patient most likely has undiagnosed obstructive sleep overhead motion of painting, and her pain is most pro
apnea (OSA). OSA is a major risk factor for perioperative nounced with abduction of her arm. On examination, her
pulmonary complications, although a large number of pain occurs between 60 and 120 degrees of abduction,

236
Answers and Critiques

which supports the diagnosis of rotator cuff tendinitis. She a C grade to HIV screening, making no recommendation
also has a positive Hawkins test, which has a high sensitiv for or against routine HIV screening.
ity (92%) but poor specificity (25%) for rotator cuff The National Cholesterol Education Program
impingement. (NCEP) recommends that screening be initiated at die age
Acromioclavicular joint degeneration is typically asso of 20 years and then continued at least every 5 years there
ciated with trauma (in younger patients) or osteoarthritis after if normal. This patient's lipid levels were normal 4
(in older patients). Palpable osteophytes may be present, years ago; therefore, it would not be appropriate to screen
and radiographs, if obtained, may demonstrate degenera him according to the NCEP guideline. The USPSTF rec
tive changes. It characteristically presents with pain that ommends lipid screening for all men 35 years or older and
occurs with shoulder adduction and abduction above 120 for men 20 to 35 years of age with increased cardiovascu
degrees. This diagnosis is unlikely in this patient given that lar risk. Because this patient is not at increased risk for atii-
she has no history of trauma and that there is no acromio erosclerotic heart disease, according to the USPSTF guide
clavicular joint tenderness on examination. lines, screening for hyperlipidemia should begin at the age
Adhesive capsulitis is caused by thickening of the cap of 35 years.
sule surrounding the glenohumeral joint. Adhesive capsuli The USPSTF recommends diabetes screening for all
tis presents with loss of both passive and active range of adults with a sustained blood pressure of 135/80 mm Hg
motion in multiple planes and patient reports of stiffness, or greater. In contrast, the American Diabetes Association
which arc not present in this patient. Also, pain is typically recommends screening all adults who are 45 years and
slow in onset and is located near the insertion of the deltoid older and all adults who have a BMI of 25 or greater who
muscle. have one or more additional risk factors (gestational dia
Rotator cuff tears are usually accompanied by weakness betes, hypertension, hyperlipidemia, family history of type
and loss of function. Examination findings include 2 diabetes mellitus in a first-degree relative). Screening for
supraspinatus weakness, weakness with external rotation, diabetes would be inappropriate in this patient owing to his
and a positive drop-arm test. The absence of weakness age, absence of hypertension or obesity, and lack of other
along widi die negative drop-arm test argues against the risk factors.
presence of a rotator cuff tear in this patient. There is no agreement among major groups related to
KEY PO NT screening for hypothyroidism. The American Academy of
Rotator cuff impingement syndrome due to Family Physicians and the American Association of Clinical
Endocrinologists recommend screening for hypothy
underlying tendinitis is a common cause of roidism in older women. The American Thyroid Associa
nontraumatic shoulder pain; characteristic find
tion recommends screening adults by measuring diyroid-
ings are pain with arm abduction and a positive
Hawkins test. stimulating hormone (TSH) beginning at age 35 years, but
the USPSTF does not recommend routine screening. This
Bibliography patient is not in a high-risk group defined by eidier age or
House J, Mooradian A. Evaluation and management of shoulder pain sex, and screening for diyroid disease widi a TSH level is
in primary care clinics. Soudi Med J. 2010; 103(11): 1129-1135. not appropriate.
[PMID: 20890250]
KEY POINT
The Centers for Disease Control and Preven
Item 123 Answer: C tion recommend that all persons between the
Educational Objective: Screen for HIV infection. ages of 13 and 64 years be screened for HTV
infection.
According to guidelines published by the Centers for Dis
ease Control and Prevention (CDC), this man should be Bibliography
screened for HrV infection using enzyme immunoassay Qaseem A, Snow V, Shekelle P, Hopkins R Jr, Owens DK; Clinical
antibody (EIA) testing. The guidelines recommend that all Efficacy Assessment Subcommittee, American College of Physi
cians. Screening for HIV in health care settings: a guidance state
persons between the ages of 13 and 64 years bc screened ment from the American College of Physicians and HIV Medicine
for HIV infection. This recommendation is based on evi Association. Ann Intern Med. 2009;150(2):125-131. [PMID:
dence from several studies that have demonstrated that 190470221
screening for HIV is effective even in low-prevalence set
tings. This is particularly true* when screening is coupled to
the availability of antiretroviral therapy. All positive results Item 124 Answer: A
using EIA testing should be confirmed by Western blot
Educational Objective: Manage an encounter
testing. Western blot testing should not be used as the ini with an impaired colleague.
tial screening test owing to its high rate of false-positive and
false-negative results. In contrast to the CDC guidelines, In this situation, there is considerable evidence diat die col
the U.S. Preventive Services Task Force (USPSTF) assigns league's judgment is impaired by both the amount and rate

237
Answers and Critiques

of alcohol consumption and her response to an expression as predicted by their Revised Cardiac Risk Index (clinical
of concern. The ethical obligation is to prevent her from risk factors), exercise tolerance, and the nature of the pro
CONT.
potentially harming a patient, which means contacting the posed surgery. Patients with a low or unknown exercise tol
hospital tonight. erance who have three or more clinical risk factors under
Reporting to state authorities on Monday may help going intermediate- or higher-risk surgery should be
this physician and her future patients but would not protect considered for preoperative cardiac testing if it will change
the patient she is admitting tonight. management; however, in this patient who is undergoing
Taking her aside later is unlikely to be helpful, and puts low-risk surgery, no testing is needed.
the unimpaired physician in die position of monitoring Percutaneous cardiac interventions have no demon
whatever action she may take to address the issue. strated value in die perioperative setting. Thus, periopera
Members of die medical profession have an obligation to tive cardiac risk assessment, if needed, should be done
protect die welfare of patients, which includes taking action through noninvasive stress testing. In general, die overrid
when a colleague puts patients at risk. Many states have ing theme of perioperative cardiac risk assessment is that
mandatory reporting statutes. Nevertheless, almost 75% of testing should only be done if the results will affect man
respondents in a 2009 survey did not believe diey had a duty agement, and prophylactic revascularization is rarely neces
to report a known impaired colleague to the relevant author
sary just to get a patient through surgery. This patient has
ity, raising significant concerns about physicians' understand no indications for cardiac catheterization.
ing of dieir personal and professional ethical obligations.
KEY POINT
KEY PO NT
In patients without active cardiac conditions
Members of the medical profession have an
undergoing low-risk surgery, preoperative car
obligation to protect the welfare of patients, diovascular testing is not routinely needed.
which includes taking action when a colleague
puts patients at risk. Bibliography
Fieisher LA, Beckman JA, Brown KA, et al. 2009 ACCF/AHA
Bibliography focused update on perioperative beta blockade incorporated into
Deslloches CM, Rao SR, Fromson J A, et al. Physicians' perceptions, the ACC/AHA 2007 guidelines on perioperative cardiovascular
preparedness for reporting, and experiences related to impaired and evaluation and care for noncardiac surgery: a report of the Ameri
incompetent colleagues. JAMA. 2010;304(2):187-193. [PMID: can College of Cardiology Foundation/American Fleart Associa
20628132] tion Task Force on Practice Guidelines. Circulation.
2009;120(21):el69-276. [PMID: 19884473]

Item 125 Answer: D


Educational Objective: Manage a patient under Item 126 Answer: C
going low-risk surgery. Educational Objective: Manage a patient who is
a victim of intimate partner violence.
This patient undergoing endoscopic sinus surgery should
proceed to surgery without further testing. Patients under The primary responsibility' of the provider for a patient who
going low-risk surgery, which includes endoscopic surgery, is a victim of intimate partner violence is to assist with
cataract surgery, superficial surgery, breast surgery, and health; assess for safety; and provide validation, support,
ambulatory surgery, do not need perioperative cardiac test and empathy.
ing unless they have high-risk ("active") conditions such as Leaving die abuser is neither necessary nor recom
an unstable coronary syndrome (myocardial infarction <30 mended without a well diought-out plan unless die patient
days ago, unstable or severe angina), decompensated heart is in imminent danger, in which case immediate intervention
failure, significant arrhythmia, or severe valvular disease. is indicated. Advising victims of intimate partner violence to
Although diis patient has three clinical risk factors (coro simply leave the situation, to utilize a shelter, contact an inti
nary artery disease, chronic kidney disease, diabetes melli mate partner violence counseling service, or press criminal
tus requiring insulin) for a major perioperative cardiac com charges is generally not helpful as die circumstances sur
plication, he does not require further cardiac testing rounding intimate partner abuse relationships are complex,
because the anticipated surgery is low risk. and die abused individual may have significant reasons for
The use of [.-blockers in the perioperative period is not pursuing these actions that need to be understood. An
controversial, with well-designed randomized controlled appreciation of the individual circumstances in the context of
trials showing both benefit and harm. Current recom a supportive relationship will help in developing a plan that
mendations are that patients currently on a (3-blocker may ultimately involve die use of these valuable resources.
should continue taking it. Thus, metoprolol should not It is not recommended that the potential abuser be
be discontinued. confronted directly or legal action be undertaken as an ini
Cardiac testing is reserved for patients undergoing tial step in most cases as this may potentially put the victim
surgery at the highest risk of a perioperative cardiac event in greater danger.

238
Answers and Critiques

Psychiatry intervention may bc necessary for refractory study whether this treatment is required to see the benefit
depression, or when the patient is deemed a risk to harm of drug H.
herself or others. However, this would not be an appropri There is no arbitrary level of risk of harm diat would
ate next step in management of this patient. impact a decision to use a medication; rather, each medica
A substantial number of patients remain in adverse tion should be evaluated according to its risk and benefit
relationships yet demonstrate improved health and health profile. Furthermore, in this trial, drug H had net benefit
outcomes after disclosure of their situation with appropri despite its rate of serious adverse events.
ate support and management. KEY POINT

KEY POINT Caution should be used when generalizing the


The primary responsibility of the provider for a results of randomized controlled trials to popu
lations other than those who would meet the
patient who is a victim of intimate partner vio
lence is to assist with health; assess for safety; inclusion and exclusion criteria of the study.
and provide validation, support, and empathy.
Bibliography
Ho PM, Peterson PN, Masoudi FA. Evaluating the evidence: is there
Bibliography a rigid hierarchy? Circulation. 2008;118(16):1675-1684. [PMID:
Zolotor AJ, Denham AC, Weil A. Intimate partner violence. Prim
18852378]
Care. 2009;36(1):167-179. [PMID: 19231608]

Item 128 Answer: A


Item 127 Answer: A
Educational Objective: Diagnose an autism spec
Educational Objective: Evaluate a randomized
trum disorder.
controlled trial for generalizability.
The diagnosis is autism spectrum disorder. Autism is char
This patient widi cardiomyopatiiy is asymptomatic, placing
acterized by a triad of impaired communication; impaired
her in the category of New York Heart Association social interactions; and restrictive, repetitive, and stereo
(NYHA) functional class I heart failure. Therefore, her typed behaviors and interests. In addition, classic autism is
heart failure is too mild for her to take drug H, which was
often associated with some degree of learning disability or
tested on patients with NYHA class III and IV heart failure. mental retardation and is typically diagnosed in early child
Randomized controlled trials (RCTs) are often con hood. Less severe variants include high-functioning autism
sidered the "gold standard" for evaluating new therapies
(HFA) and Asperger syndrome, which may not be diag
because their experimental design allows confounding vari nosed until adolescence or even aduldiood. HFA is a cate
ables diat might obscure the benefit of a therapy to be bal
gory of autistic disorder with less severe clinical features and
anced between groups. Thus, any finding in a well- without cognitive impairment. Asperger syndrome differs
designed RCT is typically considered valid. In order to from HFA in that early language development is not
maximize die ability of a given study to find a meaningful
delayed. Many experts believe that autism, HFA, and
result, RCTs are typically restricted to relatively homoge Asperger syndrome are variants along a single spectrum.
neous individuals who meet rigidly defined inclusion and The hallmark of obsessive-compulsive disorder is die
exclusion criteria. The proscribed nature of patient selection
presence of recurrent obsessions or compulsions diat are of
and intervention in RCTs dierefore make their conclusions sufficient severity to occupy at least 1 hour per day or to result
narrow. Clinicians must use caution when generalizing in marked distress or functional impairment. The person
these results to odier populations. Drug H was shown to shotdd recognize that the obsessions or compulsions are
be effective for patients with NYHA class III or IV heart excessive or unreasonable. Obsessions arc defined as persistent
failure, and it may not be effective for a patient widi more ideas, thoughts, impulses, or images diat are experienced as
mild heart failure, such as this patient. intrusive, inappropriate, and associated widi significant anxi
Although die mean age of participants in the trial was ety or distress. This patient has compulsions but has additional
63 years, die drug was tested on patients between the ages symptoms not characteristic of obsessive-compulsive disorder.
of 40 and 80 years. Thus, based on her age, the patient Schizophrenia is defined by the presence of psychosis.
would have been eligible for die trial, and age alone is not Psvchosis encompasses delusions, hallucinations, disorga
a reason to widihold the drug from her. nized speech, and disorganized or catatonic behavior. The
Despite rigid criteria for inclusion and exclusion in a diagnosis requires at least 6 mondis of symptoms, including
RCT, diere will still likely be some variability between indi 1 mondi or more of at least two active-phase symptoms, such
vidual patients included in a particular study, such as con as hallucinations, delusions, disorganized speech, grossly dis
current medications being used. Even with these differ organized or catatonic behavior; and negative symptoms,
ences, however, a net benefit of treatment was found in the such as flattened affect. There must also be significant impair
study population. Although most patients in the study ment in social or occupational function. This patient's symp
were already on a P-blocker, it cannot be inferred from the toms are not consistent widi schizophrenia.

239
Answers and Critiques

The primary feature of social anxiety disorder is a Persons born after 1980, health care workers, and
severe and persistent fear of social or performance situa diose born before 1980 who have a high risk for dissemi
tions, such as public speaking or taking an examination. nated varicella should receive a two-dose varicella vaccina
Persons with more generalized social anxiety disorder avoid tion series unless diey have serologic evidence of varicella
many occupational and social situations because of fears of immunity or physician-documented evidence of either vari
interacting widi other people. This patient's symptoms of cella or varicella vaccination. Patient or parent self-report is
emotional outbursts, ccholalia, and stereotyped interest in not considered reliable.
trains do not suggest social anxiety disorder.
KEY POINT
KEY POINT In patients who are HIV-positive, CD4 cell
Autism is characterized by a triad of impaired counts should be obtained before administering
communication; impaired social interactions; live attenuated vaccines.
and restrictive, repetitive, and stereotyped
behaviors and interests. Bibliography
Advisory Committee on Immunization Practices; Centers for Disease
Control and Prevention (CDC). Immunization of health-care per
Bibliography sonnel: recommendations of the Advisory Committee on Immu
Rao S, Salmon G. Autism spectrum disorders. Br J Hosp Med (Lond). nization Practices (ACIP). MMWR Recomm Rep. 2011;60(RR-
2010;71(12):699-703. [PMID: 21135768] 7):l-45. [PMID: 22108587]

Item 129 Answer: E Item 130 Answer: C


Educational Objective: Select an appropriate vac Educational Objective: Diagnose costochondritis.
cination strategy for an HIV-positive patient.
This patient most likely has costochondritis. The etiology of
A CD4 cell count should be obtained in this patient before chest pain can be determined in most cases after a carefiil his
any vaccines are administered. This woman is entering school tory and physical examination. Musculoskeletal chest pain
to become a certified nursing assistant; all healdi care work has an insidious onset and may last for hours to weeks. It is
ers should be vaccinated against or have serologic evidence of most recognizable when sharp and localized to a specific area
immunity to hepatitis B, varicella, measles, mumps, and of die chest; however, it can also be poorly localized. The
rubella. In addition, health care workers should receive a
pain may be worsened by turning, deep breathing, or arm
one-time tetanus, diphdieria, and acellular pertussis (Tdap) movement. Chest pain ma)' or may not be reproducible by
vaccine as well as annual influenza vaccination. In general, chest palpation (pain reproduced by palpation does not
vaccines can eidier be inactivated viral proteins or live atten exclude ischemic heart disease), and die cardiovascular exam
uated viruses. Inactivated viral proteins are safe to administer ination is often normal. Importantly, his findings are not con
in all patients, including those with HIV infection, except sistent widi an alternative cause of chest pain.
diose with a documented allergy to die vaccine or its growth The chest pain associated with acute pericarditis is typ
media (such as eggs). Live attenuated vaccines, on die odier
ically pleuritic in nature and is worsened when the patient
hand, should be withheld in patients with immune deficiency, lies down. A two- or diree-component friction rub is often
including HIV-positive patients with a CD4 cell count below present. This patient does not have any risk factors for peri
200/microliter. Therefore, before administering any vac carditis; specifically, diere is no history of recent viral infec
cines, however, it is important to determine whether or not tion, myocardial infarction, trauma, malignancy, medica
she is functionally immunodeficient. tions, connective tissue disease, or uremia. Pericarditis,
The measles, mumps, and rubella vaccine contains live, dierefore, is highly unlikely.
attenuated virus, and for this patient, it would be inappro Aortic dissection is generally described as a tearing or
priate to administer it before verifying that she is immuno ripping pain widi radiation to the back. It is more com
competent and also diat she needs revaccination (based on monly seen in patients widi a history of hypertension.
negative serologic studies). Although physical examination findings may be missed,
Live, attenuated influenza vaccine should be avoided asymmetric intensity of peripheral pulses has a positive like
in patients with immunodeficiency. Regardless of her CD4 lihood ratio of 5.7. This patient's chest pain description,
cell count, die inactivated influenza vaccine would likely be physical examination, and absence of risk factors are incon
preferred for this patient. sistent with aortic dissection.
Hepatitis B vaccination is indicated in health care This patient has no risk factors for cardiac disease. His
workers and in HIV-positive patients, as well as in patients history is inconsistent with descriptors diat increase the
with unknown hepatitis B status for whom hepatitis B vac probability of ischemic chest pain, including unstable
cination is needed. As for odier vaccines, it would be best angina. Specifically, there is no radiation to the arms, exer
to determine this patient's immune status before providing tional component, relief widi rest, diaphoresis, nausea,
die vaccine. vomiting, or pressure description. Considering the patient's

240
Answers and Critiques

age and description of his chest pain, the probability of Item 132 Answer: D
unstable angina or an acute coronary syndrome is low. Educational Objective: Diagnose ulnar nerve
KEY POINT entrapment at the level of the elbow.
Musculoskeletal chest pain has an insidious onset This patient's presentation is consistent widi ulnar nerve-
and may last for hours to weeks; it is most recog entrapment at the elbow (cubital tunnel syndrome). The
nizable when sharp and localized to a specific cubital tunnel is the path followed by the ulnar nerve as it
area of the chest; and the pain may be worsened passes around the elbow toward the hand. In this region,
by turning, deep breathing, or arm movement. the nerve is near the surface of the skin and dierefore sus
ceptible to injury. Injury may occur from mild and often
Bibliography unrecognized trauma, sustained pressure on the nerve as
Stochkendahl MJ, Christcnsen HW. Chest pain in focal muscu
may occur during sleep, or activities that involve sustained
loskeletal disorders. Med Clin North Am. 2010;94(2):259-273.
I PMID: 203809551 flexion of the elbow and stretching of the nerve. Diagnos-
tically, maneuvers that compress or stretch the injured
nerve, such as flexing the arm (as seen in this patient), result
Item 131 Answer: C in elbow pain. Paresthesias are also commonly present in
Educational Objective: Manage primary the ulnar nerve distribution in the hand. These paresthesias
are characteristically located on both the palmar and dorsal
dysmenorrhea.
surfaces of the hand, which contrasts with entrapment of
The most appropriate management option for this patient is the ulnar nerve at the wrist, which only involves the palmar
a trial of NSAID therapy, such as ibuprofen. This patient has surface.
dysmenorrhea associated widi normal menstrual cycles and Patients with lateral epicondylitis (tennis elbow) typi
no pelvic padiology. Initial treatment options for primary cally present with pain in the lateral elbow diat radiates
dysmenorrhea include NSAIDs and cyclooxygenase-2 down the forearm to the dorsal hand. On examination,
inhibitors, which inhibit die inflammation, vasoconstriction, there is tenderness to palpation at the location of the inser
and uterine ischemia that are diought to cause the symptoms. tion of extensor muscles on the lateral epicondyle. Pain is
If symptoms are not relieved with NSAID therapy or reproduced by forced extension of the wrist.
the patient requests contraception or is sexually active, a Patients with medial epicondylitis (golfer's elbow) typ
combination estrogen-progesterone contraceptive would ically present with pain in the medial elbow and proximal
be appropriate. Extended-cycle formulations are particu forearm. On examination, there is tenderness to palpation
larly useful for this indication. from the medial epicondyle to the pronator teres and flexor
Depot medroxyprogesterone acetate (DMPA) is a carpi radialis muscles. Pain can be reproduced with wrist
long-acting progesterone compound, administered intra flexion and resisted forearm supination.
muscularly or subcutaneously every 12 to 14 weeks. Long- Olecranon bursitis is characterized by pain in the pos
acting progesterone dierapy is a treatment option for dys terior elbow and swelling of the bursal sac that overlies the
menorrhea, and also provides contraception. For olecranon process. Range of motion of the elbow is not
adolescents and young adults, however, long-term use of limited.
progesterone therapy decreases bone mineral density owing KEY POINT
to prolonged estrogen deficiency. Therefore, such treat
Ulnar nerve entrapment at the elbow is charac
ment should be used widi caution for dysmenorrhea or for
terized by pain diat occurs with flexion of the
contraception in younger women based on the risks and
benefits of treatment in a given patient. In general, NSAIDs arm and paresthesias on both the palmar and
should be tried before hormonal therapy. dorsal surfaces of the hand, in die distribution
In the absence of worrisome symptoms such as severe- of die ulnar nerve.
pelvic pain or significant bleeding abnormalities, treatment
for dysmenorrhea may be initiated without further evalua Bibliography
Caliendro P, La Torre G, Padua R, Giannini F, Padua L. Treatment for
tion, such as pelvic imaging, hormonal testing, or gyneco ulnar neuropathy at the elbow. Cochrane Database Syst Rev.
logic referral. 2011;(2):CD006839. [PMID: 21328287]
KEY POINT
The first-line treatment for primary dysmenor
Item 133 Answer: D
rhea is NSAID therapy.
Educational Objective: Understand the princi
Bibliography ples of informed consent.
Harel Z. Dysmenorrhea in adolescents and young adults: from patho The principles of informed consent were not followed in
physiology to pharmacological treatment and management strate this case. The three key elements of informed consent are:
gies. Expert Opin Pharmacodier. 2008;9(15):2661-2672. | PMID:
18803452] understanding of the proposed treatment, understanding

241
Answers and Critiques

of alternatives to the proposed treatment, and understand Bibliography


ing die risks and benefits of both the treatment and die Nikolaidis I, Fouyas IP, Sandercock PA, Stadiam PF. Surgery for cer
alternatives. In diis case, the patient says he was not given vical radiculopathy or myelopathy. Cochrane Database Syst Rev.
either information about risks of antibiotics or alternative 20l0;(l):CD00l466. [PMID: 20091520]

options. This calls into question whether he received ade


quate information to give informed consent. Item 135 Answer: B
Informed consent applies to all health care decisions,
not only to invasive procedures. It requires an active dia Educational Objective: Diagnose interstitial cys
titis as a cause of chronic pelvic pain.
logue around the three key elements.
Many practitioners obtain written informed consent This most likely diagnosis in this patient with chronic
for invasive procedures because this entails documentation
pelvic pain is interstitial cystitis. Chronic pelvic pain is
of consent, but it does not exempt physicians from having defined as noncyclic pain of at least 6 months' duration
a discussion of risks, benefits, and alternatives for all tteat- that localizes to the anatomic pelvis, the anterior abdom
nients that have die potential to cause harm. inal wall at or below the umbilicus, the lumbosacral back,
KEY POINT or the buttocks, and is of sufficient severity to impair
The three key elements of informed consent quality of life. Potential causes of chronic pelvic pain
are: understanding of die proposed treatment, include interstitial cystitis, endometriosis, pelvic adhe

understanding of alternatives to the proposed sions, and irritable bowel syndrome. In this patient, the
combination of chronic pelvic pain in association with
treatment, and understanding the risks and
benefits of both the treatment and the unexplained urinary symptoms is most consistent with a
alternatives. diagnosis of interstitial cystitis. Interstitial cystitis is a
chronic inflammatory condition of the bladder that
Bibliography causes symptoms of urinary urgency, frequency, and
Snyder L; American College of Physicians Ft hies. Professionalism, and pelvic discomfort. The pelvic discomfort may be wors
Human Rights Committee. American College of Physicians Ethics ened by sexual intercourse, and patients may urinate
Manual: sixth edition. Ann Intern Med. 2012;156(lPt 2):73-104.
numerous times per day. Although urinalysis and urine
[PMID: 22213573]
cultures are almost always negative, most women with
interstitial cystitis have been treated empirically several
Item 134 Answer: D times for urinary tract infections.
Educational Objective: Treat chronic cervical Endometriosis is a common cause of chronic pelvic

radiculopathy with neurologic deficits. pain, and patients typically report severe dysmenorrhea,
cyclic pain, and dyspareunia. The absence of severe dys
This patient should be referred for surgical evaluation. He menorrhea and the noncyclic nature of this patient's
has clinical and imaging evidence of cervical radiculopathy, pelvic pain make endometriosis a less likely diagnosis.
and progressive weakness in the affected arm that correlates
Similarly, irritable bowel syndrome is unlikely to explain
with his disk herniation. Surgical intervention is generally her symptoms in the absence of any associated gastro
indicated in patients with progressive neurologic symptoms intestinal symptoms.
resulting from a defined anatomic abnormality to preserve Adhesions are diagnosed in 25% to 50% of women
and avoid permanent loss of function. with chronic pelvic pain. Pelvic adhesions typically form in
In the absence of progressive motor deficits, a conser the setting of acute or chronic inflammatory processes, such
vative approach is indicated as most patients experience as infection, or surgery. This patient has no history of pelvic
improvement in symptoms without more aggressive imag infection and has never had pelvic surgery, making this an
ing or intervention. Local corticosteroid injections may unlikely cause of her symptoms.
provide faster pain relief, although long-term outcomes are
similar to conservative therapy. Physical therapy is also a KEY POINT
useful intervention in patients with uncomplicated cervical Interstitial cystitis is a likely diagnosis in
radiculopathy to decrease discomfort and possibly women with chronic pelvic pain associated
strengthen the neck muscles to prevent recurrent episodes. with unexplained urinary symptoms; most
However, these interventions, including continued anal women with interstitial cystitis have been
gesics and rest, would not be appropriate in this patient treated empirically several times for urinary
with worsening neurologic function. tract infections.
KEY POINT
Bibliography
Surgical referral is indicated for patients with
Vercellini P, Somigliana F, Vigano P, Abbiati A, Barbara G, Fedcle L.
cervical radiculopathy widi progressive motor Chronic pelvic pain in women: etiology, pathogenesis, and diag
deficits. nostic approach. Gynecol Endocrinol. 2009;25(3):149-158.
| PMID: 19347704]

242
Answers and Critiques

Item 136 Answer: D Item 137 Answer: B


Educational Objective: Treat chronic neuro Educational Objective: Diagnose complicated
pathic pain with opioid therapy. grief.

Long-acting opioid therapy, such as sustained-release This patient is likely suffering from complicated grief stem
morphine, would be the best treatment option in this ming from his wife's death 9 months ago. Complicated
patient. Although opioid medications arc generally not grief, also referred to as complicated bereavement, is an
recommended for use in patients with chronic noncancer abnormal response to bereavement persisting more than 6
pain, they are appropriately considered in patients with months, at least 6 months after a death. Its chief symptom
moderate to severe neuropathic pain that has not is yearning for a loved one so intensely that all other desires
responded to adequate trials of multiple nonopioid ther are incapacitated. Although interventions have been
apies. As with all patients being considered for long-term attempted to prevent this severe grief reaction, none have-
opioid treatment, he should have a thorough under proved beneficial. It is important for health care providers
standing of its risks and benefits and should work together to be watchful for this disorder in bereaved survivors.
widi the physician to develop an opioid management Drugs with the potential for anticholinergic adverse
plan, or pain contract, that outlines agreed-upon goals effects, such as diphenhydramine, are generally not a good
and rules of treatment. choice in the elderly. Although the anticholinergic proper
Long-acting opioids provide more consistent serum ties of diphenhydramine could be responsible for acute con
drug levels and are generally preferred to intermittent dos fusion and hallucinations, diis patient's persistent symp
ing of shorter-acting agents that may lead to significant toms, such as anhedonia and early morning wakening,
fluctuations in pain level. Although this patient's pain is rea would not be explained by such a short-acting agent.
sonably well controlled on his current regimen of oxy- Generalized anxiety disorder is characterized by excessive
codone-acetaminophen, it is also a short-acting preparation anxiety and worry about a variety of events or activities over
and is less desirable for long-term use. When used chroni at least a 6-month period; difficult)' exercising control over
cally, the overall daily dose of acetaminophen needs to be worrying; several symptoms associated with the anxiety, such
taken into account; daily doses should be below 4 g and less as fatigue, irritability, restlessness, sleep disturbance, and diffi
in patients with liver disease, significant alcohol use, or culty concentrating; and functional impairment. Although an
those being treated with other potentially hepatotoxic anxiety disorder should also be considered in the differential
medications. diagnosis of this patient given his sleep disturbance, his lack of
Tramadol is a weak opioid agonist and may be effec excessive anxiety and worry make it less likely.
tive for mild, episodic neuropathic pain, although it may Although this patient could also have major depres
require a relatively high daily dose widi an increased risk of sion, his intense feelings that his wife is beside him are more-
associated side effects, such as gastrointestinal symptoms. It indicative of complicated grief marked by an unwillingness
also has a short period of action and requires multiple daily to accept her death. His sense that his wife is beside him
dosing, which would not be preferable in this patient. when he wakes up in the middle of the night and his visions
Methadone is a long-acting opioid that can be very of his wife's face when he closes his eyes are not consistent
effective for pain control but can be associated with sig with hallucinations because the patient is aware that they
nificant adverse cardiovascular outcomes, such as QT- are not real. Patients with true psychosis arc convinced their
interval prolongation, hypotension, and cardiac arrhyth hallucinations and delusions are real.
mias. It should be avoided in this patient with a history of KEY POINT
ischemic cardiomyopathy and conduction defects noted
Complicated grief is grief persisting more than
on the electrocardiogram.
6 months, at least 6 months after a death; its
KEY POINT chief symptom is yearning for a loved one so
Long-acting opioid therapy should be consid intensely that all other desires are incapacitated.
ered in a patient with moderate to severe neu
Bibliography
ropathic pain that has not responded to non
Wittouck C, Van Autreve S, De Jacgcrc E, Portzky G, van Heeringen
opioid dierapies. K. The prevention and treatment of complicated grief: a meta
analysis. Clin Psychol Rev. 201 l;31(l):69-78. [PMID: 21130937]
Bibliography
Bril V, England J, Franklin GM, et al; American Academy of Neurol
ogy; American Association of Neuromuscular and Flectrodiagnos- Item 138 Answer: D
tic Medicine; American Academy of Physical Medicine and Reha
bilitation. Evidence-based guideline: Treatment of painful diabetic Educational Objective: Diagnose rhinitis
neuropathy: report of the American Academy of Neurology, the medicamentosa.
American Association of Neuromuscular and Electrodiagnostic
Medicine, and the American Academy of Physical Medicine and This patient most likely has rhinitis medicamentosa. The
Rehabilitation. Neurology. 2011;76:1758-1765. [PMID:
21482920] key clue in this patient's presentation is ongoing use of a

243
Answers and Critiques

topical nasal vasoconstrictor (oxymetazoline). In some all women with cyclical mastalgia, and 20% of patients will
patients, this produces a syndrome of tolerance (rhinitis experience resolution of their pain without any interven
medicamentosa) in which continued use of die vasocon tion. This patient has recently increased her level of physi
strictor produces diminishing returns, but withdrawal of it cal activity, and lack of a supportive sports bra may have-
causes severe nasal congestion. Withdrawal of the offend exacerbated her discomfort.
ing agent is the only reliable way to treat this condition. Medical treatment is typically reserved for women who
Sometimes, use of nasal corticosteroids or nasal saline have severe and persistent pain that interferes with their
rinses can mitigate symptoms of decongestant withdrawal. quality of life. Danazol is the only treatment that has been
A 2011 trial randomized 60 patients with perennial approved by the FDA for cyclical mastalgia, although it
allergic rhinitis to fluticasone, oxymetazoline, a combina would not be appropriate in this patient without a trial of
tion of both agents, and placebo. After 4 weeks of treat nonmedical therapv-.
ment, no evidence of rhinitis medicamentosa was seen in A thorough history and physical examination are typ
either of the oxymetazoline-treated groups, raising die pos ically sufficient to rule out more serious causes of breast dis
sibility that rhinitis medicamentosa is much less common comfort. A patient can be diagnosed with benign mastalgia
than previously thought. if there is no evidence of extramammary causes of breast
Chronic rhinosinusitis is characterized by mucopuru discomfort, such as pneumonia, pleuritis, myocardial
lent drainage or facial pain or pressure typical of sinusitis. In ischemia, infection, or costochondritis. In the absence of a
the absence of these findings and the presence of ongoing palpable breast mass or skin changes suggestive of malig
oxymetazoline use, rhinitis medicamentosa is the more nancy, there is no role for diagnostic mammography in the
likely diagnosis. management of cyclical mastalgia.
Chronic nonallergic (vasomotor) rhinitis is a syndrome Tamoxifen has been used off-label for treatment of
characterized by the presence of at least one typical symp benign mastalgia, and rarely can be associated with hot
tom of rhinitis (sneezing, rhinorrhea, nasal congestion, flushes and menstrual irregularities. It could be considered
postnasal drainage) in the absence of a specific etiology. for this patient, but only if she does not benefit from con
Chronic nonallergic rhinitis is a possibility in this patient, servative measures, including use of a well-fitting bra.
but the diagnosis cannot be made until the oxymetazoline
KEY POINT
is stopped.
Granulomatosis with polyangiitis (Wegener granulo Conservative measures, including education,

matosis) is a necrotizing vasculitis that typically affects the reassurance, and the use of a well-fitting sup
respiratory tract and the kidneys. More than 70% of patients port bra, are recommended for all women widi
present with upper airway symptoms, particularly sinusitis. cyclical mastalgia and should bc tried before
Up to 90% of patients have pulmonary manifestations that initiating medical treatment.
can include cough, hemoptysis, or pleurisy. The patient's
findings are not consistent with granulomatosis with Bibliography
polyangiitis. Miltenburg DM, Speights VO Jr. Benign breast disease. Obstet
Gvnecol Clin North Am. 2008;35(2):285-300, i.\. [PMID:
KEY POINT 18486842 I
Ongoing use of a topical nasal vasoconstrictor
may cause rhinitis medicamentosa, in which Item 140 Answer: B
continued use of the vasoconstrictor produces
Educational Objective: Treat a depressive episode
diminishing returns, but withdrawal of it
causes severe nasal congestion. of bipolar disorder.

The most appropriate treatment for this patient is lamot


Bibliography
rigine. He most likely is having a depressive episode of
Doshi I. Rhinitis medicamentosa: what an otolaryngologist needs to
know. Eur Arch Otorhinolaryngol. 2009;266( 51:623-625. [PMID: bipolar disorder. Careful questioning of patients who pres
190968621 ent with depressive symptoms to elucidate a history of
hypomanic or manic episodes is important in order to iden
tify bipolar disorder. The Mood Disorders Questionnaire
Item 139 Answer: C (MDQ) is a relatively brief, validated questionnaire to
Educational Objective: Manage cyclical screen for bipolar disorder. A cut-off of seven or more pos
itive responses out of the thirteen items on question 1 yields
mastalgia.
a sensitivity of 73% and specificity of 90%. Pharmacother
This patient has cyclical mastalgia, which affects up to 40% apy for bipolar disorder is more complicated than for
of premenopausal women. It is most prominent during the
unipolar depression, and a psychiatrist should be involved
luteal phase of the menstrual cycle and is typically described in the care of most patients with the disorder. Lithium has
as a bilateral, throbbing discomfort. Education, reassur long been a mainstay of bipolar disorder therapy; however,
ance, and the use of a well-fitting bra are recommended for it has a narrow therapeutic window and is teratogenic,

244
Answers and Critiques

nephrotoxic, and can cause hypothyroidism. Alternative gemfibrozil to this patient's regimen will likely reduce her
first-line therapies include mood stabilizing agents, such as triglyceride levels by up to 50%, but will not substantially
lamotrigine, valproic acid, or carbamazepine. In addition, decrease her LDL cholesterol (typical reductions of 5%-
atypical antipsychotic agents, such as aripiprazole or queti 20%). Moreover, gemfibrozil raises the serum concentra
apine, can be used for frank mania. Simply prescribing anti tion of statins by two-fold, thereby increasing the risk for
depressants alone places the patient at risk for experiencing statin-induced myopathy. The risks adding gemfibrozil to
a frank manic episode. her statin therapv', therefore, outweigh the benefits for this
Duloxetine, a serotonin-norepinephrine reuptake patient.
inhibitor, is an appropriate therapy for die treatment of A review of data from several large clinical trials found
depressive disorders. Most classes of antidepressants have- that the risk of myopathy with the 80-mg dose of simvas
tatin was significantly higher than the risk observed with
equal efficacy (about 70% in most studies) and should be
chosen on the basis of previous patient response, side-effect other statin therapies. In patients who have not achieved
profile, and cost. This patient has bipolar disorder, and an goal LDL cholesterol levels with a 40-mg dose of simvas
antidepressant alone would not be die appropriate treatment. tatin, the FDA has now recommended that therapy be
switched to atorvastatin or rosuvastatin. These statins are
Benzodiazepines, such as lorazepam, are ineffective
more potent and can achieve LDL cholesterol goals at
single agents for depression or bipolar disorder, but may be
used as an adjunct therapy in patients with mania or hypo- lower doses, thereby decreasing the risk for statin-induced
mania. This patient is having a depressive episode, and there- myopathy. Increasing the simvastatin dose to 80 mg/d,
is no role for lorazepam in treating his condition. therefore, is inappropriate for this patient's management.
Selective serotonin reuptake inhibitors (SSRIs), such as Undiagnosed diabetes niellitus should be considered
in patients with unresponsive hyperlipidemia and may be
sertraline, may trigger a manic episode in patients with
bipolar disease and should not be used in this population. diagnosed by a hemoglobin Ak level of 6.5% or greater. In
this patient, her fasting glucose and triglyceride levels are
For this reason, in any patient being considered for initia
tion of SSRIs for depression, it is important that the history normal, making a diagnosis of diabetes less likely; therefore,
be assessed for manic symptoms. obtaining a hemoglobin A, level would not be an appro
priate next step in management.
KEY POINT
KEY POINT
In patients with depressive symptoms, it is
In patients with hyperlipidemia that is refrac
important to elucidate a history of hypomanic
or manic episodes in order to identify bipolar tory to medical dierapy, secondary causes,
including hypothyroidism, diabetes mellitus,
disorder, which is treated with mood stabilizers
rather than antidepressants. nephrotic syndrome, and obstructive liver dis
ease, should be considered.
Bibliography
Bibliography
Belmaker RH. Treatment of bipolar depression. N Engl ) Med
Ahvaili K, Alrasadi K, Awan Z, Genest J. Approach to the diagnosis
2007;356( 17): 1771 -1773. [PMID: 17392296] and management of lipoprotein disorders. Curr Opin Endocrinol
Diabetes Obes. 2009;l6(2):l32-140. [PMID: 19306526]

Item 141 Answer: D


Educational Objective: Diagnose secondary Item 142 Answer: D
causes of dyslipidemia. Educational Objective: Manage warfarin periop
This patient's serum thyroid-stimulating hormone level eratively in a high-risk patient.
should be measured. Her total cholesterol and LDL cho For this patient with a high risk for a perioperative throm
lesterol levels are markedly elevated despite adherence to boembolic event, the most appropriate treatment is to dis
treatment with simvastatin. Secondary causes of dyslipi continue warfarin 5 days before surgery and provide bridg
demia should be considered, as statin dierapy may be inef ing anticoagulation with a low-molecular-weight heparin
fective in the setting of untreated hypothyroidism, diabetes (LMWH), such as enoxaparin, until the morning of
mellitus, obstructive liver disease, or nephrotic syndrome. surgery. In general, patients using warfarin have three pos
There are several clinical clues that suggest the diagnosis of sible preoperative treatment options: stop warfarin, receive
hypothyroidism in this patient, including symptoms of bridging therapy with a parenteral anticoagulant, or con
fatigue and constipation and dry skin noted on physical tinue the warfarin. This patient has atrial fibrillation with a
examination. Undiagnosed thyroid disease is likely con high CHADS, score (l point each for diabetes mellitus,
tributing to this patient's apparent treatment-refractory heart failure, and hypertension, and 2 points for previous
dyslipidemia. stroke or transient ischemic attack [TIA] = 5) and her TIA
Gemfibrozil is a fibric acid derivative that is typically is recent, placing her at a high risk for thrombosis; such
used for the treatment of hypertriglyceridemia. Adding patients should not have anticoagulation withheld for a

245
Answers and Critiques

prolonged period of time. Thus, warfarin should be Antidepressants, when used as monotherapy for
changed to an agent with a shorter and more predictable anorexia nervosa, have not been shown to increase weight
half-life, usually LMWH. This agent is then withheld just gain or improve the underlying attitudes or behaviors asso
before surgery and restarted after surgery, thus minimizing ciated with anorexia nervosa. In particular, amitriptyline
the amount of time the patient is not therapeutically anti- and bupropion should be avoided in patients with anorexia
coagulated. nervosa because of the high risk of adverse effects of these
The effect of dose adjustment of warfarin on INR is agents when given in the setting of metabolic derange
hard to predict. Thus, it would be inappropriate to recom ments, which are common in this patient population.
mend a fixed half dose of warfarin. This may result in an Megestrol acetate is an appetite stimulant that is
inappropriately high INR level for surgery (as would con approved for palliative treatment of patients with cancer
tinuing the current dose of warfarin up to surgery) or a pro and AIDS complicated by anorexia and significant weight
longed period of time with an inadequate INR, putting the loss. It is not approved for patients with anorexia nervosa
patient at risk for thromboembolism. and has not been studied in this population.
In patients taking warfarin who have a low risk of Amenorrhea is a diagnostic criterion for anorexia ner
thromboembolism, including those with a history of vosa, and oral contraceptive pills will restore menses. How
venous thromboembolism more than 12 months ago and ever, diere are no convincing data to support this practice,
those with atrial fibrillation with a CHADS, score of 2 or and restoration of menses through hormonal manipulation
less, stopping warfarin without providing bridging antico may decrease a patient's motivation for weight gain.
agulation is acceptable. However, this patient's CHADS, KEY POINT
score is 5, and withholding anticoagulation for 5 davs pre
In patients with anorexia nervosa, cognitive-
operatively is not recommended.
behavioral therapy can be effective for reducing
KEY POINT
relapse after weight restoration has been
Patients taking warfarin who are at high risk of initiated.
postoperative venous thromboembolism and
are undergoing intermediate- or high-risk Bibliography
Bulik CM, lkrkman ND, Brownley KA, Sedway JA, Lohr KN.
surgery should have warfarin discontinued 5 Anorexia nervosa treatment: a systematic review of randomized
days before surgery and receive bridging anti controlled trials. Int J Eat Disord. 2007;40(4):310-320. [PMID:
coagulation, usually with low-molecular-weight 17370290]
heparin.

Bibliography Item 144 Answer: A


Douketis JD, Spyropoulos AC, Spencer FA, et al; American College Educational Objective: Treat chronic pain in an
ol Chest Physicians. Perioperative management of antithrombotic
therapy: Antithrombotic Therapy and Prevention of Thrombosis, elderly patient.
9th ed: American College of Chest Physicians Evidence-Based Clin
ical Practice Guidelines. Chest. 2012;141(2 suppl):e326S-e50S. Acetaminophen is the best medication for the initial treat
I PMID: 223152661 ment of this patient's chronic pain secondary to spinal
stenosis, and is recommended as first-line therapy for
chronic noncancer pain by the American Geriatrics Society.
Item 143 Answer: C It is effective for the relief of mechanical/compressive pain
Educational Objective: Treat anorexia nervosa. and has an excellent safety profile. This patient has no
underlying liver disease and does not drink alcohol, so he-
The most appropriate treatment for this patient with has no contraindications to this medication. He should not
anorexia nervosa is cognitive-behavioral therapy (CBT). exceed a dose of 4 g in 24 hours; the FDA has recently lim
Anorexia nervosa is characterized by an abnormally low ited die dose of prescription acetaminophen to 325 mg to
body weight (<85% of expected) in association with a fear decrease the risk of toxicity.
of gaining weight, an excessive emphasis of body weight Several medications are effective for treating mechani
on self-perception, and amenorrhea for at least three men
cal/compressive pain, including tricyclic antidepressants,
strual cycles. Overall recovery rates range from 35% to NSAIDs, acetaminophen, and duloxetine. However, older
85%, and many patients relapse. The goals of treatment patients are more likely to experience medication-related
are to restore healthy weight, treat physical and metabolic side effects, and comorbid conditions may predispose to
complications, address abnormal attitudes and feelings impaired drug metabolism. Careful selection of pain med
related to eating, and treat associated psychiatric condi ications is therefore essential. Tricyclic antidepressants, such
tions. Behavioral interventions include CBT, cognitive as amitriptyline, should be avoided in the elderly owing to
analytic therapy, and family therapy; among these, CBT their potential for adverse effects, including cardiac con
appears to be the most effective for reducing relapse after duction abnormalities, orthostasis, and anticholinergic
weight restoration has been initiated. effects.

246
Answers and Critiques

Muscle relaxants, such as cyclobenzaprine, have KEY POINT


demonstrated limited effectiveness in the treatment of Screening for depression is recommended for
mechanical/compressive pain. all adults as long as appropriate supports are
Although NSAIDs can be very effective for the relief in place to ensure adequate treatment and fol
of pain, their associated risks for gastrointestinal, cardio low up.
vascular, and renal toxicities limit their use in older
patients with comorbidities. This patient has a history of Bibliography
peptic ulcer disease, and the addition of ibuprofen to his O'Connor EA, Whitlock EP, Beil TL, Gaynes BN. Screening for
medication regimen may increase his risk for gastrointes depression in adult patients in primary care settings: a systematic evi
tinal bleeding. dence review. Ann Intern Med. 2009;151(ll):793-803. [PMID:
199491451
KEY POINT
First-line therapy for chronic pain in the elderly
is acetaminophen. Item 146 Answer: C
Educational Objective: Diagnose generalized
Bibliography anxiety disorder.
American Geriatrics Society Panel on the Pharmacologic Management
of Persistent Pain in Older Persons. Pharmacologic management of This patient most likely has generalized anxiety disorder
persistent pain in older persons. J Am Geriatr Soc. (GAD). Anxiety disorders are among the most common
2009;57(8):1331-1346. [PMID: 19573219] psychiatric disorders in the general population; of these,
GAD is the most common, occurring in approximately 4%
of die population. GAD is characterized by excessive anxi
Item 145 Answer: B
ety and worry about a variety of events or activities on most
Educational Objective: Screen for depression. days for at least 6 months, with difficulty controlling wor
This woman should be screened for depression. The rying. Associated symptoms include fatigue, irritability,
U.S. Preventive Services Task Force (USPSTF) recom resdessness, insomnia, and difficulty' concentrating. Patients
mends screening all adults for depression provided that with GAD often have comorbid anxiety disorders, depres

adequate resources are available to ensure adequate sion, or substance abuse. Patients with GAD often have
treatment and follow up. Little evidence supports using somatoform symptoms, which can make them high utiliz
one screening method over another. However, asking ers of health care resources.
two questions, "During the past 2 weeks, have you felt Attention-dcficit/hyperactivity disorder first manifests
in childhood and is characterized by inattention, hyperac
down, depressed, or hopeless?" and "During the past 2
weeks, have you felt little interest or pleasure in doing tivity, and impulsivity accompanied by functional impair
ment in at least two settings (home, work, school). This
things?" appears to have similar effectiveness compared
with longer instruments. patient's presentation is not consistent with attention-
One-time abdominal ultrasonography to screen for deficit/hyperactivity disorder.
abdominal aortic aneurysm (AAA) is recommended only Bipolar disorder is characterized by manic or hypo-
for men between the ages of 65 to 75 years who have pre manic mood episodes and depressive episodes. A manic
viously smoked. The USPSTF recommends against screen episode is marked by a persistent period of elevated mood,
ing women for AAA regardless of age or whether or not irritability, lack of need for sleep, racing thoughts, high
they have ever smoked. energy levels, increased talkativeness, spending sprees,
The USPSTF strongly recommends screening for hypersexuality, and increased self confidence, with possible-
delusions of grandeur or psychosis.
hepatitis B virus infection in all pregnant women at their
first prenatal visit. Owing to the low prevalence of A major depressive episode is diagnosed by the pres
ence of five or more of the following symptoms occurring
hepatitis B in the general population in the United
States, however, routine screening is not cost effective nearly every day during the same 2-week period, at least
and is not recommended. one of which is either depressed mood or loss of interest
The USPSTF recommends screening for osteoporosis or pleasure: depressed mood most of the day, loss of
in all women age 65 years or older and also in younger interest or pleasure in most activities, significant unin
women widi an elevated fracture risk. Screening this patient tentional weight or appetite gain or loss, insomnia or
for osteoporosis would not be appropriate as she is younger hypersomnia, psychomotor agitation or retardation,
than 65 years and does not have a fracture risk that is equal fatigue or loss of energy, feelings of worthlessncss or
to or greater than that of a 65-year-old woman without guilt, diminished concentration, or recurrent thoughts of
additional risk factors, such as alcoholism, corticosteroid death or suicide without a specific plan or prior attempt.
use for more than 3 months, low body mass, current This patient's symptom complex does not fit with a diag
tobacco use, dementia, or use of anticonvulsants. nosis of depression.

247
Answers and Critiques

KEY POINT KEY POINT


Generalized anxiety disorder is characterized For women with hypoactive sexual desire disor
by excessive anxiety and worry about a variety der, individual or couples sex dierapy may be
of events or activities on most days for at beneficial.
least 6 months, with difficulty controlling
worrying. Bibliography
Nappi RE, Martini E, Terreno E, et al. Management of hypoactive sex
ual desire disorder in women: current and emerging therapies. Int
Bibliography
J Wbmcns Health. 2010;2:167-175. [PMID: 21072309]
Kroenke K, Spitzer RL, Williams JP>, Monahan PO, Lowe B. Anxiety
disorders in primary care: prevalence, impairment, comorbidity, and
detection. Ann Intern Med. 2007;146(5):317-325. [PMID:
17339617] Item 148 Answer: A
Educational Objective: Diagnose macular

Item 147 degeneration.


Answer: A
Educational Objective: Manage hypoactive sexual This patient has age-related macular degeneration (AMD).
desire disorder. In the early stages, AMD is often asymptomatic. Common
symptoms, when present, include distortion of vision or a
This woman has hypoactive sexual desire disorder notable loss of central vision. Those with advanced AMD
(HSDD), which is diagnosed if a patient reports personal and profound visual loss may experience visual hallucina
distress associated with a persistent lack of sexual thoughts, tions (Charles Bonnet syndrome). In both dry and wet
desire for, or receptiveness to sexual activity. HSDD is the AMD, drusen are common findings. Drusen are amor
most common female sexual disorder, with prevalence phous deposits behind the retina that lead to visual loss
ranging from 12% to 19%. In studies of menopausal through direct (space occupying) and indirect (inflamma
women, predictors of sexual function included feelings for tory response) means. A few small, hard drusen are com
partner, change in partner status, and previous level of sex mon as people age, but numerous large, soft drusen are a
ual function. As this patient's current feelings about die
harbinger of severe AMD. Wet AMD, which is less com
relationship widi her partner may be causing distress, indi mon than dry AMD and typically more aggressive, is char
vidual or couples sex therapy, which provides information acterized by neovascularization with subsequent vessel leak
about the normal female sexual response and facilitates age and hemorrhage. Smoking is a risk factor for AMD.
communication about sexual issues, may be beneficial. Cataract, or any opacification of the otherwise optically
There is no FDA-approved medication for the treatment of clear lens behind the pupil and iris, is the most common
female HSDD. cause of blindness and low vision worldwide. Vision loss is
Sildenafil is an effective treatment for male erectile dys slowly progressive and is usually worse in bright light or at
function but generally is not indicated for the treatment of night, with glaring headlights while driving. This patient's
sexual disorders in women. It was found to be no better clear lens is not consistent with cataract.
than placebo for increasing the frequency of enjoyable sex Primary open angle glaucoma (POAG) is a progressive-
ual encounters or improving any aspect of sexual function optic neuropathy associated with increased intraocular pres
in women. It may be used to improve sexual arousal and sure. POAG is characterized by painless, gradual loss of
orgasm in women experiencing sexual dysfunction related peripheral vision in both eyes that may go unnoticed by the
to selective serotonin reuptake inhibitor therapy but does patient. In later stages, die central vision may also be
not impact sexual desire. affected. This patient's vision distortion and central vision
Systemic estrogen and progesterone therapy improves loss and the funduscopic findings are not consistent widi
many symptoms associated with menopause, including hot POAG.
flushes and vaginal atrophy (unresponsive to topical ther Retinal detachment is a separation of the neurosensory
apy). This patient has no signs or symptoms of vaginal atro layer of the retina from the choroid beneath. It may result
phy and does not report vasomotor symptoms, so this treat from trauma or occur spontaneously, particularly in persons
ment is not indicated. with myopia. Symptoms include diminished vision, pho-
Several studies have shown that systemic testosterone topsia (flashes of light), abrupt onset of multiple floaters in
therapy, either as monotherapy or in combination with the vision, or metamorphopsia (wavy vision). Funduscopic
estrogen therapy, increases sexual function scores and num examination reveals the folds of the retinal tear and detach
ber of satisfying sexual episodes in menopausal women but ment, which are not present in this patient.
has significant adverse effects. Treated women experienced
KEY PO NT
unwanted hair growth, vaginal bleeding, and had a trend
toward an increased incidence of breast cancer. The FDA Age-related macular degeneration causes pain
has not approved any testosterone therapv' for the treat less progressive vision loss, characterized by dis
ment of sexual dysftinction. tortion of vision and loss of central vision.

248
Answers and Critiques

Bibliography Item 150 Answer: A


lager KD, Mieler WF, Miller JW. Age-related macular degeneration. Educational Objective: Manage obesity with
N Engl J Med. 2008;358(24):2606-2617. [PMID: 18550876]
bariatric surgery.
This patient should be referred for bariatric surgery. For
Item 149 Answer: D patients with class III obesity (BMI >40) or class II obesity
Educational Objective: Assess cardiac risk in a (BMI 35.0-39.9) with obesity-related complications, the
patient scheduled for intermediate-risk surgery. National Institutes of Healdi Consensus Development Con
ference recommends consideration of bariatric surgery if diet,
This patient needs no further testing before surgery. Current exercise, and/or medication are ineffective. Patients should be
recommendations are to reserve preoperative cardiac evalua motivated and well informed about this option and undergo
tion for patients at highest risk for perioperative cardiac
multidisciplinary evaluation by a medical, surgical, psychiatric,
events undergoing intermediate- or high-risk surgeries when and nutritionist team. The most common procedure is gastric
testing would influence patient management. These are typ bypass surgery, but laparoscopic banding is becoming com
ically patients with both a low exercise tolerance and multi mon, as well. Bariatric surgery results in more dramatic and
ple risk factors. (Adequate exercise tolerance is defined as the sustained weight loss than nonsurgical interventions and leads
ability to perform physical exertion of >4 metabolic equiva to improvement in obesity-related complications (diabetes
lents [METs] without symptoms.) The Revised Cardiac Risk mellitus, obstructive sleep apnea, hypertension, and hyperlipi
Index (RCRI) is a validated risk assessment tool that assigns
demia). This patient has not attained his goal weight loss after
one point each for a history of ischemic heart disease, com a 6-month trial of diet and medication and has obesity-related
pensated or prior chronic heart failure, diabetes mellitus complications that likely will improve widi weight loss.
requiring insulin, chronic kidney disease, or history of cere Phentermine is a sympathomimetic drug that is FDA-
brovascular disease. Although this patient has poor exercise
approved for short-term use (up to 12 weeks) as an adjunc
tolerance and is undergoing an intermediate-risk procedure tive treatment of obesity. This patient's weight loss goal is
(ordiopedic surgery), he has no RCRI risk factors. The 45.4 kg (100 lb), which will take much longer than 12
American Heart Association/American College of Cardiol weeks. In addition, most persons regain any weight that is
ogy guidelines on perioperative cardiac evaluation recom lost with this medication upon its discontinuation.
mend diat patients widi no RCRI risk factors undergoing an
Restricting caloric intake to below 800 kcal/d (a very-
intermediate-risk procedure proceed to surgery without fur low-calorie diet) is no more effective for long-term weight
ther cardiac evaluation. Cardiac complications occur in less loss dian a moderate strategy of restricting intake to 500-
dian 1% of patients with zero or one RCRI risk factors. Thus, 1000 kcal/d below what is estimated to maintain current
despite diis patient's age and comorbidities, the current rec body weight. In addition, long-term compliance with a
ommendations are for no preoperative cardiac testing. very-low-calorie diet is nearly impossible.
Cardiac catheterization is not recommended as an ini Exercise is an important part of a comprehensive
tial preoperative cardiac evaluation, because of the good weight loss program that focuses on lifestyle modification.
test performance characteristics of stress tests and the inva However, the patient has already not benefited from an
sive nature a cardiac catheterization. exercise program. It is unlikely that exercise alone will meet
CT coronary angiography is a noninvasive means of his weight loss goals.
defining cardiac anatomy. It has not been studied for peri
KEY POINT
operative cardiac risk assessment.
Noninvasive pharmacologic sttcss testing, either dobuta Bariatric surgery should be considered for
mine stress echocardiography or nuclear perfusion stress tests, patients with BMI of 40 or greater or BMI of
are recommended for patients in whom preoperative cardiac 35.0 to 39.9 widi obesity-related complications
testing is indicated. However, this patient's cardiovascular lisk in whom diet, exercise, and/or medication are
is estimated to be low enough to not warrant stress testing. ineffective.
KEY POINT
Bibliography
Low-risk patients who are not undergoing Colquitt JL, Picot I, Lovcman E, Clegg AJ. Surgery for obesity.
higher-risk surgery can proceed to surgery Cochrane Database Syst Rev. 2009;(2):CD003641. |PMID:
without preoperative cardiac testing. 19370590|

Bibliography !
Fleisher LA, Beckman JA, Brown KA, et al. 2009 ACCF/AHA Item 151 Answer: A
focused update on perioperative beta blockade incorporated into Educational Objective: Screen for abdominal
the ACC/AHA 2007 guidelines on perioperative cardiovascular
evaluation and care for noncardiac surgery: a report of the Ameri aortic aneurysm.
can College of Cardiology Foundation/American Heart Associa This patient should undergo one-time abdominal ultra-
tion Task Force on Practice Guidelines. Circulation.
2009;120(21):el69-276. |PMID: 19884473] sonography to screen for an abdominal aortic aneurysm

249
Answers and Critiques

(AAA) because he is a man between the ages of 65 to 75 back pain secondary to spinal stenosis, and has continued
years who has ever smoked (defined as 100 lifetime ciga to experience pain out of proportion to usual postoperative
rettes). In the Multicentre Aneurysm Screening Study, a pop pain without evidence of ongoing spinal or nerve compres
ulation-based sample of 67,770 men between die ages of 65 sion. Additionally, his not participating in furdier treat
and 74 years were offered either screening for AAAs with an ment, including physical therapy and home exercise, is con
abdominal ultrasound or no screening. After 10 years of fol cerning. He is currendy on opioid therapy, which he has
low up, it was determined diat screening offered a 14% not tapered downward, and has risk factors that have been
absolute risk reduction in mortality from AAAs and a reduc associated with aberrant drug-taking behaviors, including
tion in all-cause mortality that was of borderline significance. age younger than 45 years, cigarette smoking, a history of
Coronary artery calcium (CAC) determination by elec depression, and a family history of alcohol abuse. The pos
tron beam CT has high sensitivity for detecting stenoses of sibility of opioid dependency should be discussed directly
greater than 50% but low specificity. Because of diis low with the patient and a plan for managing his opioid use
specificity, a 2007 American College of Cardiology Founda developed and mutually agreed upon, possibly including
tion/American Heart Association consensus document development of a pain management contract.
states diat determination of CAC by electron beam CT is not Amitriptyline can be a useful adjunctive therapy in die
recommended in asymptomatic persons. Therefore, it would management of chronic pain, and might be a useful
not be appropriate to order this screening test in this patient. adjunctive treatment in this patient, although it would be
Although this patient is a former smoker, the U.S. Pre inappropriate to start this therapy before addressing his
ventive Services Task Force (USPSTF) concludes that there- opioid use.
is insufficient evidence to recommend for or against screen It is generally preferable to avoid long-term opioid use
ing for lung cancer widi eidier chest radiograph)', low-dose for chronic pain. In situations in which they must be used,
CT, sputum cytology, or a combination of these tests. long-acting opioids, such as sustained-release morphine sul
The USPSTF recommends screening for lipid disor fate, are preferable to short-acting agents as diey avoid the
ders in men aged 35 years and older and in men or women serum peaks and troughs that occur with short-acting opi
aged 20 years and older who are at increased risk for car oids and provide more stable pain control.
diovascular disease. The Adult Treatment Panel of the Although this patient is post-spinal surgery, his surgi
National Cholesterol Education Program (ATP III) guide cal course has been uneventful and he has no focal findings
lines recommend that patients with 0 to 1 risk factor and a suggestive of an operative complication or intervening
normal fasting lipid profile or normal nonfasting total cho issue, such as infection. Therefore, further imaging is not
lesterol and HDL cholesterol levels do not have to be indicated.
screened again for 5 years; the USPSTF acknowledges diat
KEY POINT
the optimal screening interval is uncertain but that every 5
Opioid dependency should be suspected in
years is reasonable for low-risk persons. The patient's only
risk factor is hypertension and his lipid levels were normal patients with risk factors for aberrant drug use
1 year ago; therefore, there is no reason to repeat the test and an inability to stop opioid medications
after being treated with these agents for legiti
annually:
mate indications.
KEY POINT

One-time abdominal ultrasonography to screen Bibliography


for an abdominal aortic aneurysm is recom Dunn KM, Saunders KVV, Rutter CM, et al. Opioid prescriptions for
chronic pain and overdose, a cohort study. Ann Intern Med.
mended in men between die ages of 65 to 75
20l();l52(2):85-92. [PMID: 20083827]
years who have ever smoked (defined as 100
lifetime cigarettes).
Item 153 Answer: C
Bibliography
Educational Objective: Manage chronic primary
Thompson SG, Ashthon FIA, Gao L, Scott RA; Multicentre Aneurysm
insomnia.
Screening Study Group. Screening men for abdominal aortic
aneurysm: 10 year mortality and cost effectiveness results from the This patient should be counseled regarding sleep hygiene,
randomised Multicentre Aneurysm Screening Study. BMJ
2009;338:b2307. (PMID: 195532691 which refers to behavioral and environmental factors that
affect sleep. This patient is experiencing a primary sleep dis
turbance likely related to environmental factors in his new
Item 152 Answer: C apartment. Although sleep hygiene alone is often ineffec
Educational Objective: Avoid adverse outcomes tive in completely relieving insomnia, it provides a founda
associated with opioid treatment for chronic pain. tion of good sleep habits to which other therapies may be
added. In the case described, resolution may be as simple
This patient should be evaluated for opioid dependency. He as adjusting the temperature in the bedroom; other envi
has received definitive surgical treatment for chronic lower ronmental factors that may be contributing include noise

250
Answers and Critiques

level, bed comfort, and the patient's psychosocial adjust binge-eating, which may or may not be associated with self-
ment in the new apartment. induced vomiting or the misuse of medications. This
Although alcohol can contribute to insomnia or be patient's normal body weight and regular menses are more
used inappropriately to treat insomnia, this patient's pattern suggestive of bulimia nervosa rather than anorexia nervosa.
of alcohol use does not suggest either of these. He is not Notably, 30% of patients with the restricting subtype of
using alcohol to help him fall asleep, and he is having anorexia nervosa go on to develop bulimia nervosa, as seen
with this patient.
nightly insomnia despite only using alcohol on weekends.
When nonpharmacologic approaches such as improv Binge-eating disorder is more common than either
anorexia nervosa or bulimia nervosa, affecting 2% to 3% of
ing sleep hygiene are unsuccessful in treating insomnia,
the general population. It is differentiated from bulimia
pharmacotherapy may be indicated. Although many
nervosa in that there is no associated compensatory behav
patients take over-the-counter antihistamines as sleep aids,
ior after the binge.
they are not recommended owing to their potential anti
cholinergic side effects and next-day drowsiness. If this Night-eating syndrome is characterized by excessive
patient requires medication, the nonbenzodiazepine hyp eating at night, difficulty sleeping, and morning anorexia.
notics Zolpidem or zaleplon are considered first line It is a prevalent disorder in obese patients and those seek
because they do not alter sleep architecture and have a ing bariatric surgery.
favorable side-effect profile. Benzodiazepines, such as KEY POINT
diazepam, may be used as second-line agents, preferably Bulimia nervosa is an eating disorder character
for short-term use onlv. ized by recurrent episodes of binge eating with
KEY POINT subsequent compensatory behavior aimed at
The first-line treatment of insomnia is counsel preventing weight gain.
ing regarding sleep hygiene.
Bibliography
Sim LA, xMcAlpine DE, Grothe KB, Himes SM, Cockcrill RG, Clark
Bibliography MM. Identification and treatment of eating disorders in the primary
Wilson JF. In the clinic: Insomnia. Ann Intern Med. care setting. Mavo Clin Proc. 2010;85(8):746-751. [PMID:
2008;148(1):ITC13-1-ITC13-16. [PMID: L8166757] 20605951f

Item 154 Answer: C Item 155 Answer: D


Educational Objective: Diagnose bulimia Educational Objective: Diagnose meralgia
nervosa. paresthetica.
This patient most likely has bulimia nervosa, which is an This patient most likely has meralgia paresthetica, or
eating disorder characterized by recurrent episodes of binge entrapment of the lateral femoral cutaneous nerve beneath
eating with subsequent compensator)' behavior aimed at the inguinal ligament. Meralgia paresthetica is characterized
preventing weight gain. The compensator)' behaviors may by paresthesia (burning/numbness) located over the
include self-induced vomiting and misuse of medications, anterolateral thigh. There are no motor symptoms because
such as laxatives (purging subtype), or tasting and excessive the lateral femoral cutaneous nerve is a purely sensory
exercise. The purging subtype of bulimia nervosa may be nerve. Risk factors for developing meralgia paresthetica
suspected by metabolic abnormalities, including include diabetes mellitus, obesity, and the wearing of tight-
hypokalemia, hypomagnesemia, and metabolic alkalosis, as fitting pants or belts. On examination, dysesthesia or
seen in this patient. Diagnostic behavior includes engaging hypoesthesia is present in the distribution of the lateral
in bingeing and compensatory behaviors at least twice a femoral cutaneous nerve. The remainder of die examina
week for 3 months, and having one's self-perception be tion is typically normal.
excessively influenced by body weight and shape. Most Patients with greater trochanteric bursitis report pain
patients with bulimia nervosa have a normal weight; the in the region of the greater trochanter that is made worse
presence of dental caries, enlarged salivary glands, and scar widi lying on the affected side. On examination, diere is
ring on the dorsum of die hand are highly suggestive of tenderness to palpation approximately 2.5 cm (1 in) poster
purging behaviors. ior and superior to the greater trochanter.
Anorexia nervosa is characterized by an abnormally Iliotibial band syndrome typically presents with pain in
low bodv weight (<85% of expected) in association with an the anterolateral knee that is worse with running downhill
intense fear of gaining weight, an overemphasis of body or cycling. The pain is typically absent during rest. On
weight on self-evaluation, and amenorrhea for three con examination, patients frequendy have pain to palpation of
secutive menstrual cycles. The restricting subtype of the lateral femoral epicondyle.
anorexia nervosa is associated with regular caloric restric Lumbar disk herniation at L5 typically presents with
tion; the binge-eating/purging subtype is characterized by back pain that radiates down the lateral thigh to the

251
Answers and Critiques

foot. There is weakness in foot dorsiflexion, toe extension, Item 157 Answer: B
foot inversion, and foot eversion. The straight leg raising
Educational Objective: Diagnose Lemierre
test is more than 90% sensitive for lumbar radiculopathy
syndrome.
and its absence is strong evidence against lumbar disk dis
ease. Deep tendon reflexes are typically normal in L5 This patient should undergo CT of the neck with contrast.
radiculopathy. This patient's findings are not compatible She has fever, leukocytosis, sore throat, unilateral neck ten
with L5 radiculopathy. derness, and multiple densities on her chest radiograph,
suggestive of septic emboli. The combination of these fac
KEY POINT
tors points strongly toward Lemierre syndrome, which is
Meralgia paresthetica is a nerve entrapment
septic thrombosis of die internal jugular vein. The diagno
syndrome of die lateral femoral cutaneous sis should be suspected in anyone with pharyngitis, persis
nerve of die anterior thigh typified by pain and tent fever, neck pain, and septic pulmonary emboli. CT of
burning. the affected vessel with contrast would confirm the diag
nosis. Treatment should include intravenous antibiotics
Bibliography that cover streptococci, anaerobes, and (5-lactamase-pro-
Plante M, Wallace R, Busconi BD. Clinical diagnosis of hip pain. Clin
ducing organisms. Penicillins with (^-lactamase inhibitors
Sports Med. 2011;30(2):225-238. [PMIDr21419954] and carbapenems are both reasonable choices (such as
ampicillin-sulbactam, piperacillin-tazobactam, and ticar-
Item 156 Answer: C cillin-clavulanate).
Chest CT would better characterize the pulmonary
Educational Objective: Manage syncope in an
infiltrates, but such information would not provide specific
elderly patient. diagnostic information diat would guide therapy.
Cardiac arrhythmia is the most likely cause of syncope in Soft tissue radiography of the neck is incapable of
this patient, given his prodrome, time course, and complete detecting jugular vein filling defects or thromboses, which
are diagnostic of septic thrombophlebitis.
recovery immediately following the event. Cardiac causes of
syncope carry a high mortality, and patients with suspected Echocardiography would be helpful to rule out right-
cardiac causes warrant further evaluation in the hospital sided endocarditis as a cause of septic emboli; however,
there is nothing in the history or on the cardiac examina
regardless of age. High-risk patients requiring immediate
tion to suggest a cardiac source of septic emboli.
in-hospital telemetry are diose with exertional or supine
syncope, palpitations prior to the event, a family history of KEY POINT
sudden death, nonsustained ventricular tachycardia, and The diagnosis of septic thrombosis of the jugu
abnormal electrocardiographic findings. Thus, monitoring lar vein (Lemierre syndrome) should be sus
by telemetry in die hospital is appropriate for this high-risk pected in anyone with pharyngitis, persistent
patient. fever, neck pain and septic pulmonary emboli.
Neurologic abnormalities are uncommon causes of
syncope. Therefore, studies to assess for an intracranial or Bibliography
carotid process are very low yield in patients without new Centor RM, Samlowski R. Avoiding sore throat morbidity and mor
neurologic findings; thus, neither carotid Doppler ultra tality: when is it not "just a sore throat?". Am Fam Physician.
2011;83(1):26, 28. [PMID: 21888123]
sonography nor head CT scan is indicated. Brain imaging
may be appropriate to assess head trauma associated widi
his syncopal event, but not as a routine component of syn
Item 158 Answer: D
cope evaluation.
Educational Objective: Manage elevated liver
Echocardiography for evaluation of syncope is also low
yield except in patients suspected of having structural heart chemistry test results in a patient on statin therapy.
disease or with significant findings on cardiovascular exam No change in management of this patient's lipid levels is
ination, neither of which is the case in this patient. indicated, including repeat liver chemistry testing or change-
KEY POINT
in statin medication. Statins work by inhibiting the hepatic
Cardiac causes of syncope carry a high mortal HMG-CoA reductase enzyme, and can be associated with
an elevation of aminotransferase levels, and rarely hepato
ity, and patients with suspected cardiac causes
warrant furdier evaluation in the hospital. toxicity and acute liver failure. Aminotransferase elevations
less than three times the upper limit of normal may occur
in up to 3% of statin-treated patients. Conversely, statin-
Bibliography related hepatotoxicity (defined as alanine aminotransferase
Mendu ML, McAvay G, Lampert R, Stoehr J, Tinetti ME. Yield of
level more than three times the upper limit of normal and
diagnostic tests in evaluating syncopal episodes in older patients.
Arch Intern Med. 2009;169( 14).T299-1305. [PMID: total bilirubin level more than twice the upper limit of nor
196360311 mal) and acute liver failure are very rare. Acknowledging

252
Answers and Critiques

this, the FDA has recently recommended that baseline liver administered to inpatients and is part of the Joint Com
chemistry tests be measured prior to initiating statin ther mission's Core Measures for patients with pneumonia.
apy and then only as clinically indicated thereafter. Hepatitis B vaccination is reserved for persons at high
This patient has minor elevations of aminotransferase est risk for cither hepatitis B acquisition or its sequelae, as
levels diat were discovered incidentally. Statin-related minor well as persons who request die vaccine. This includes trav
elevation of aminotransferase levels is usually asymptomatic, elers to endemic regions as well as persons widi an increased
occurs within the first 12 weeks of therapy, and resolves risk of sexual, percutaneous, and mucosal transmission,
such as those with multiple sex partners or men who have
spontaneously without discontinuation of therapy. It is
sex widi men, injection drug users, and health care work
thought to represent a "leak" of liver enzymes related to
increased hepatocyte permeability; there are no associated ers. This patient has none of these risk factors and should
not be routinely vaccinated.
histopathologic changes. This phenomenon has been
observed with all of the statins but is more common with Human papillomavirus (HPV) vaccination is recom
mended for males between the ages of 11 and 21 years, and
higher doses.
Simvastatin should only be discontinued if there is is permitted in men ages 22 to 26 years. The rationale for
clinical evidence of drug-related hepatotoxicity. This vaccinating men is the prevention of genital w arts, anal car
cinoma (in men who have sex with men), and transmission
occurs most commonly in the setting of underlying liver
of HPV to women. This patient is 30 years old and the vac
conditions or as a result of drug interactions (such as
cination is not indicated.
acetaminophen).
In the setting of possible hepatotoxicity on treatment, Meningococcal vaccination is reserved for adolescents
and for adults living in college dormitories or military bar
persistent elevations of liver chemistry test results after dis racks or who are asplenic. It is not indicated in this patient.
continuation of the statin warrant further evaluation. Com
mon causes of liver disease should be sought, including KEY POINT

hepatitis C virus infection, nonalcoholic fatty liver disease, The pneumococcal polysaccharide vaccine is
and autoimmune hepatitis. Serum antibody studies and liver indicated for adults with asthma; it is consid
uldasonography may be helpful in this situation, and statin ered safe in patients widi mild to moderate ill
dierapy should be withheld until investigations are complete. ness and should not be routinely withheld in
KEY PO NT
hospitalized patients.
Baseline liver chemistry tests should be obtained
Bibliography
in patients prior to starting statin therapy; how Advisory Committee on Immunization Practices. Recommended
ever, routine follow-up of liver chemistry testing adult immunization schedule: United States, 2012. Ann Intern
is not needed and is indicated only if there is Med. 2012;156(3):211-217. [PMID: 22298576)
clinical evidence of liver dysfunction.
Item 160 Answer: B
Bibliography
FDA Drug Safety Communication: Important safety label changes Educational Objective: Manage a patient with
to cholesterol-lowering statin drugs. Additional Information type 2 diabetes mellitus postoperatively.
for Healthcare Professionals. Available at: WWW.fda.gov/
Drugs/DrugSafcry/ucm293101.htm#hcp. Accessed June 7,2012. This patient with type 2 diabetes mellitus should be given
long- and short-acting insulin postoperatively to control
her glucose level. Both the stress of surgery and anesthesia
Item 159 Answer: D independently contribute to intra- and postoperative hyper
Educational Objective: Appropriately administer glycemia. Therefore, many patients, even those who have
the pneumococcal vaccine in a young adult. previously been well controlled with diet or oral medica
tions, may need insulin during the perioperative period.
This young man widi asthma should receive the pneumo Oral agents are challenging to use perioperatively owing
coccal polysaccharide vaccine. Pneumococcal vaccination is to variability in die patient's ability to eat as well as die long-
currendy indicated for adults aged 65 years and older and acting nature of diese agents; in general, these medications
for adults younger than 65 years who have risk factors for should be avoided during the perioperative period.
acquisition of pneumococcal disease or morbidity from it. Sliding scale insulin, the administration of short-act
This includes persons with chronic cardiovascular disease ing insulin in a dose based on periodic blood glucose
(including hypertension), chronic pulmonary disease measurements (usually every 4-6 hours), has been a tra
(including asthma), chronic liver disease, diabetes mellitus, ditional method of treating patients with diabetes in
alcoholism, and persons who smoke. Vaccination is gener acute care and perioperative settings. However, because
ally considered safe in patients with mild to moderate illness basal hypoglycemic treatment is not provided and the
and should not be routinely withheld in hospitalized dosing of insulin is "retrospective" based on current glu
patients. In fact, pneumococcal vaccination is routinely cose levels, control with this approach is typically poor,

253
Answers and Critiques

with the potential for significant fluctuations in glucose pharyngeal wall. In high-risk patients, a negative antigen
levels. Providing a basal level of long-acting insulin with test should be confirmed by throat culture.
CONT.
as-needed short-acting insulin is the preferred method of No guidelines recommend antibiotic treatment with
glucose control in acute care settings. out further testing. Some recommend treating patients
The optimal plasma glucose level postoperatively is with three or four Centor criteria while test results are
controversial. Overt hyperglycemia likely contributes to pending, although guidelines differ on this point.
postoperative dehydration as well as poor wound healing,
KEY POINT
although there is no reduction in mortality when intensive
insulin therapies are used to lower plasma glucose level to Use of the four-point Centor criteria is a rea
a target of 80 to 180 mg/dL (4.4 to 10.0 mmol/L) among sonable way to triage patients with pharyngitis
patients in die postoperative period. Therefore, most to empiric treatment with antibiotics, sympto
experts advocate using insulin to keep random glucose level matic treatment only, or testing with treatment
below 180 mg/dL. As this patient's plasma glucose level is if the test is positive.
250 mg/dL (13.9 mmol/L), she should be treated to
reduce her glucose level. Bibliography
Wessels MR. Clinical practice. Streptococcal pharyngitis. N Engl I
KEY POINT Med. 201l;364(7):64S-655. [ PMID: 21323542]
Owing to the stresses of surgery and die poten
tial adverse effects of oral diabetic agents, many
patients with type 2 diabetes niellitus require Item 162 Answer: A
insulin during die perioperative period even if Educational Objective: Manage vaccination in a
their diabetes was well controlled with diet or patient in whom a multidose vaccination series has
oral medications previously. been interrupted.

This patient who received only the first of a three-dose series


Bibliography
Lipshutz AK, Cropper MA. Perioperative glycemic control: an evi of hepatitis B vaccine 3 years ago should finish the series.
dence-based review. Anesthesiology. 2009;110(2):408-421. Many vaccines require multiple doses to achieve an optimal
| PMID: 191941671 immune response. However, the interval between doses is a
minimum interval, not a maximum, and a longer dian desired
interval is not thought to reduce die overall antibody con
Item 161 Answer: B centration following completion of die series. Thus, in
Educational Objective: Manage acute
patients with a prolonged interval since die previous dose of
pharyngitis. vaccine, the series does not need to bc restarted but should be
This patient should be given a rapid streptococcal antigen resumed with the next injection as soon as possible after die
test before beginning antibiotic therapy. The patient's pri missed prescribed interval and completed as recommended.
An adequate response to immunization against hepati
mary symptoms (fever, cough, and sore throat) are com
patible with either a viral upper respiratory tract infection tis B virus is suggested by the presence of greater than 10 mil-
or streptococcal pharyngitis. The Centor criteria (presence liunits/mL of anti-HBs antibody in the blood. However,
of fever >38.1 C [100.5 F], tonsillar exudates, tender because the seroconversion rate in a general patient popula
cervical lymphadenopathy; absence of cough) predict the tion is approximately 95%, antibody titer testing to confirm
likelihood of streptococcal pharyngitis and is a reasonable an adequate response following completion of a vaccination
way to triage patients with pharyngitis to empiric treat series is not routinely indicated except in certain high-risk
ment with antibiotics, symptomatic treatment only, or test patient populations (such as health care workers at high risk
ing with treatment if the test is positive. Patients with all for exposure to bodily fluids, hemodialysis patients, and those
four criteria have a 40% or greater chance of having group who may be repeatedly exposed to hepatitis B virus). As this
A ^-hemolytic streptococcal (GABHS) pharyngitis; patient has not yet completed the vaccination series, antibody
patients with zero or one criterion have a low (<3%) prob titer testing is not currently indicated and would not affect
ability of GABHS pharyngitis. Patients with two criteria, die recommendation to complete his remaining injections.
such as this patient, or three criteria have an intermediate He would require post-vaccination antibody testing only if
probability of GABHS pharyngitis; for these patients, he is in a known high-risk group or if there are concerns
some guidelines recommend throat culture and others rec regarding his ability to generate an immune response owing
ommend the rapid antigen detection test (RADT) widi to his existing liver disease.
confirmation of negative results. The advantage of RADT Hepatitis B vaccine is currently indicated for adults at
is the immediate availability of the results. RADT has com increased risk of seroconversion or increased risk of com
parable sensitivity and specificity to throat culture. The plications for hepatitis, such as patients with chronic liver
throat swab for either culture or RADT should be obtained disease. The patient has chronic liver disease and thus
from both tonsils or tonsillar fossae and the posterior should complete a vaccination series against hepatitis B.

254
Answers and Critiques

KEY POINT media highly unlikely as a diagnostic possibility'. In addition,


In patients with a prolonged interval since die acute otitis media is associated with signs of middle ear effu
sion and middle car inflammation (erythema of die tym
previous dose of a multiple-dose vaccine, die
series should be resumed rather than restarted. panic membrane), which are not present in this patient.
KEY POINT
Bibliography Symptoms of otitis externa include otalgia, itching
Poland GA, Jacobson RM. Clinical practice: prevention of hepatitis 15
with the hepatitis B vaccine. N Engl J Med. 2004;351(27):2832- or fullness, and pain intensified by jaw motion;
2838. [Erratum in: N Engl I Med. 2005;352(22):2362 and N Engl signs include internal tenderness when die tragus
J Med. 2005;352(7):740]. [PMID: 15625334] or pinna is pushed or pulled and diffuse ear canal
edema, purulent debris, and erydicma.
Item 163 Answer: A Bibliography
Educational Objective: Diagnose acute otitis Osguthorpe JD, Nielsen DR. Otitis externa: Review and clinical
externa. update. Am Fam Physician. 2006;74(9):1510-1516. [PMID:
171118891
This patient most likely has uncomplicated acute otitis
externa. Her swimming puts her at risk for otitis externa
owing to moist conditions created by daily water immersion. Item 164 Answer: C
Symptoms include otalgia, itching or fullness widi or without Educational Objective: Recommend contracep
hearing loss, and pain intensified by jaw motion. Signs include tion options for a woman who smokes.
internal tenderness when the tragus or pinna is pushed or
The best hormonal contraception option for this 38-year-
pulled and diffuse ear canal edema, purulent debris, and ery
old woman who smokes is a progesterone-only prepara
thema, widi or vvidiout otorrhea. Otitis externa can cause ery
thema of the tympanic membrane and mimic otitis media. In tion. Women older than 35 years who smoke more dian 15
otitis externa, however, pneumatic otoscopy shows good tym cigarettes daily should not be prescribed estrogen-con
panic membrane mobility. Management consists of clearing taining preparations because of the increased risk of
die canal of as much debris as possible to optimize penetra thromboembolic disease. A family history of stroke itself is
tion of ototopical agents as well as to visualize the tympanic not a contraindication to the use of estrogen-containing
membrane to ensure it is intact before initiating treatment. preparations, although a personal history of stroke or
thromboembolic disease is; progesterone-only contracep
Topical agents have been the mainstay of dierapy for uncom
tives are considered safe in these women. Progesterone-
plicated otitis, aldiough there is a paucity of data regarding the
effectiveness of one topical treatment compared with another. only options for women with contraindications to estrogen
An ototopical agent containing neomycin, polymyxin B, and include the "mini-pill," long-acting progesterone com
hydrocortisone is frequendy used and is effective when given pounds (such as depot medroxyprogesterone acetate),
for 7 to 10 days. Mild otitis externa can bc treated with a subcutaneous progesterone implants, and progesterone-
dilute acetic acid solution. containing intrauterine devices.
Whereas an allergic reaction to plastic ear plugs should Estrogen-only patches are never appropriate for con
be considered, the purulent discharge and the much higher traception; they may be used as hormone replacement dier
likelihood of diis being bacterial acute otitis externa make apy in postmenopausal women without an intact uterus.
a delayed type (type IV) hypersensitivity reaction unlikely. Combined estrogen-progesterone preparations are
Delayed hypersensitivity reactions (contact dermatitis) arc- available in the form of patches and vaginal rings, which
typically characterized by erythema and edema with vesicles avoid first-pass hepatic metabolism and may limit estrogen's
or bullae that often rupture, leaving a crust. Allergic reac effects on die liver and on lipids. These products do not
tions to the plastic in hearing aids, metal in earrings, or even negate the thrombogenic effects of estrogen, however, and
to otic suspension drops used to treat otitis externa should so diey are still contraindicated in women who smoke.
always be considered in the differential diagnosis of an KEY POINT
inflamed external auditory canal.
Estrogen-containing contraceptives are con
Malignant otitis externa is a much more serious entity in
which the infection in the ear canal spreads to the cartilage traindicated in women older than 35 years who
and bones nearby. It is frequendy accompanied by fever, sig smoke more than 15 cigarettes daily because of
nificant pain, and otorrhea, and patients usually appear much die increased risk of thromboembolic disease.
more ill dian diis healthy-appearing woman with localized
ear discomfort. On physical examination, granulation tissue- Bibliography
ACOG Committee on Practice Bullctins-Gynecology. ACOG practice
is often visible along the inferior margin of die external canal. bulletin. No. 73: Use of hormonal contraception in women with
Pain with tugging on the pinna and movement of the coexisting medical conditions. Obstet Gynecol. 2()06;107(6):1453-
tragus and an inflamed external auditory canal make otitis 1472. [PMID: 16738183]

255
Answers and Critiques

Item 165 Answer: E (ED), and ED is a warning sign of future cardiovascular


Educational Objective: Recognize the adverse events similar in magnitude to smoking or a family his
effects of glaucoma treatment options. tory of myocardial infarction. It is essential to accurately
assess cardiovascular risk prior to treating ED. According
This patient's timolol drops should be discontinued. Glau to the Second Princeton Consensus Conference risk clas
coma is a frequent cause of blindness in the elderly and is sification for sexual activity, this patient would be classi
characterized by increased intraocular pressure causing fied as having low cardiovascular risk as he is asympto
damage to the optic nerve. Many classes of drugs, local and matic and has fewer than three of the following major
systemic, have been used to reduce intraocular pressure, by cardiovascular risk factors: age, hypertension, diabetes
either decreasing inflow or increasing outflow of the aque mellitus, smoking, dyslipidemia, sedentary lifestyle, and
ous humor. Timolol decreases the inflow of aqueous humor family history of premature coronarv' artery disease.
and is generally well tolerated, but even locally applied Although he underwent prior coronary revascularization,
drugs can have systemic side effects. Most adverse reactions this intervention was successful, it was performed more
of timolol are manifestations of its therapeutic effect and than 8 weeks ago, and he is currently asymptomatic. As
may include heart block, sinus bradycardia, and hypoten a result of his low cardiovascular risk classification, it is
sion; most reactions are not serious and can be alleviated by appropriate to initiate therapy for his ED without per
eliminating die drug or decreasing the dosage. Other asso forming further cardiac evaluation. Because he is not on
ciated adverse effects of topical p-blocker therapy include a nitrate drug, first-line therapy with a PDE-5 inhibitor
bronchospasm, decreased libido, central nervous system would be most appropriate.
depression, and mood swings. Testosterone replacement therapy should only bc ini
Amlodipine, a systemic antihypertensive agent, has the tiated in patients with ED who have symptoms and signs of
main side effects of hypotension, peripheral edema, dizzi hypogonadism and whose testosterone level is measured
ness, and headache. Fewer than 1% of patients taking and found to be low.
amlodipine experience bradycardia, but a cause and effect Although stopping his metoprolol may improve his
relationship has not been established. ED, the cardiovascular mortality benefit of this medication
Carbonic anhydrase inhibitors, which may be adminis makes it unwise to stop.
tered orally or topically, reduce intraocular pressure by
KEY PO NT
decreasing aqueous humor inflow. Dorzolamide, a topical
carbonic anhydrase inhibitor, has fewer side effects than sys Patients with coronary artery disease who have
temic carbonic anhydrase inhibitors, such as acetazolamide, successfully undergone previous coronary revas
which can cause acidosis, malaise, hirsutism, diarrhea, and cularization, are without cardiovascular symp
blood dyscrasias. toms, and have fewer than three major cardio
Latanoprost, a topical prostaglandin analogue, vascular risk factors are considered to be at low
increases outflow of the aqueous humor. It can cause flu- risk and can safely engage in sexual activity
like symptoms and muscle aches. without cardiac evaluation.
ACE inhibitors, such as lisinopril, have adverse
effects of cough, hyperkalemia, and kidney failure but Bibliography
not bradycardia. Schwartz BG, Kloner RA. Clinical cardiology: physician update: erec
tile dysfunction and cardiovascular disease. Circulation.
KEY POINT 20ll;123(l):98-101.[PMID:212000l6|
Timolol, a topically applied (3- blocker for treat
ment of glaucoma, may have systemic adverse
Item 167 Answer: A
effects, including bradycardia and heart block.
Educational Objective: Treat plantar fasciitis.
Bibliography The most appropriate next step in the management of this
I-'ogagnolo P, Rossetti L. Medical treatment of glaucoma: present and
future. Expert Opin Invesrig Drugs. 201l;2()(7):947-959. Epub patient is arch supports. Plantar fasciitis is the most com
2011 May 3. [PMID: 21534887] mon cause of heel pain in adults. Initial therapy for plantar
fasciitis should focus on nonpharmacologic measures with
acetaminophen or NSAIDs for pain control. Although this
Item 166 Answer: A patient has been doing heel stretches and has been using
Educational Objective: Manage erectile dysfunc acetaminophen, her pes planus has not yet been addressed.
Pes planus, the loss of the arch of the foot, leads to
tion in a patient with coronary artery disease.
decreased cushioning with standing and walking and allows
The most appropriate treatment for this man with erec the redistribution of forces across the foot and ankle, com
tile dysfunction is initiation of a phosphodiesterase type monly resulting in pain and exacerbation of other foot
5 (PDE-5) inhibitor, such as sildenafil. Cardiovascular issues. Arch supports to correct her pes planus should
disease is common in men with erectile dysfunction therefore be tried. The patient should also be educated that

256
Answers and Critiques

the expected period of time until recovery is long and is surgery as well as the patient's clinical risk for a periopera
often measured in months. tive cardiac event, usually derived from the number of
Although corticosteroid injections (15 to 30 mg of Revised Cardiac Risk Index (RCRI) risk factors present and
methylprednisolone injected into the origin of the plantar exercise tolerance. Stress testing is reserved for patients
fascia) appear to provide short-term improvement in symp undergoing non-low risk surgeries who have three or more
toms, one meta-analysis found that there was no improve RCRI risk factors and are unable to achieve four metabolic
ment in long-term outcomes. Given the lack of proven ben equivalents (METs) of exercise without symptoms sugges
efit in long-term outcomes, corticosteroid injection is best tive of angina, and in whom testing will result in changes in
reserved for patients who do not respond to a conservative- management. Aldiough this patient will be undergoing an
management plan that has included addressing pes planus, intermediate-risk procedure, she has two RCRI risk factors
if present. (diabetes niellitus, chronic kidney disease with serum crea
Multiple studies have investigated the role of extracor tinine level >2.0 mg/dL [177 pmol/L]) and a good self-
poreal shock wave therapy in the treatment of plantar fasci reported level of exercise. Therefore, no further testing is
itis with conflicting results. In a meta-analysis that only required.
included studies considered to be of high quality, there was Electrocardiographic stress testing, and pharmacologic
no statistically significant benefit observed. Consequently, stress testing for those unable to physically exercise, can be
although extracorporeal shock wave therapv' is well toler used for perioperative risk stratification. However, based on
ated (the only significant side effect is a temporary increase this patient's good functional capacity, as seen by her abil
in pain), it should not be routinely recommended owing to ity to exercise to a reasonable rate without significant diffi
a lack of evidence supporting its use. culty, no further cardiovascular testing is needed.
Plantar fascia release surgery should be reserved for Preoperative percutaneous coronary interventions
have not been shown to improve postoperative outcomes
patients with refractory plantar fasciitis.
and are not routinely indicated preoperatively even in high-
KEY POINT risk patients. In patients for whom testing is required, stress
Initial therapy for plantar fasciitis should focus testing is preferred. In addition, coronarv' angiography
on nonpharmacologic measures with aceta would put this patient at risk for worsening kidney injury
minophen or NSAIDs for pain control. and should be avoided unless absolutely necessary.

KEY POINT
Bibliography
In patients undergoing non-low risk surgeries,
.bung C. In the clinic. Plantar fasciitis. Ann Intern Med. 2012;156( I
pt 1):ITC1-1-16. [PMID: 22213510] preoperative stress testing is reserved for those
who have three or more clinical cardiac risk fac
tors and are unable to achieve four metabolic
Item 168 Answer: D
equivalents (METs) of exercise without symp
Educational Objective: Manage perioperative risk toms suggestive of angina.
in a patient undergoing intermediate-risk surgery.
Bibliography
This patient can proceed to surgery without further testing. Fleishcr LA, Beckman IA, Brown KA, et al. 2009 ACCF/AHA
The American College of Cardiology/American Heart focused update on perioperative beta blockade incorporated into
Association recommendations for perioperative cardiac risk die ACC/AHA 2007 guidelines on perioperative cardiovascular
evaluation and care for noncardiac surgery: a report of rhe Ameri
assessment suggest that preoperative stress testing be can College of Cardiology Foundation/American Heart Associa
reserved for patients at greatest likelihood of a perioperative tion Task Force on Practice Guidelines. Circulation.
cardiac event. Risk is based on the nature of the planned 2009;120(21):el69-276. [PMID: 19884473]

257
Index

Note: Page numbers followed by f anil I denote figures and tables, respectively. American Society of Addictive Medicine, alcohol abuse screening recommenda
Test questions arc indicated by Q. tion from, 8
American Urological Society, 78
Abdominal aortic aneurysms Aminotransferases, statin-related increase in, 65, 66t, 67, Ql 58
screening for, 7t, 8, Q151 Anal cancer, screening for, 10
surgical repair of, 128 Anal fissures, 108
Abortitacicnts, 86 Analgesic ladder, 29-30, 29f, 35, Q65
Abscesses, anorectal, 108 Analgesics. See also Opioid analgesics
Absolute risk (AR), 3, 3t, Q58 perioperative use of, 126t
Absolute risk reduction (ARR), 3, 3t, 4 Anchoring heuristic, 2 It, Q12
Accountable care organizations (ACOs), 6t Androgen deficiency, 77, 77\, Q40
ACE inhibitors, cough and, 37, Q81 Anemia, perioperative management of, 131
Acetaminophen Angiotensin-converting enzyme inhibitors. See ACE inhibitors
for plantar fasciitis, Q167 Ankle, sprains of, 61, Q59
use in liver disease, 35 Anorectal abscesses, 108
use in older patients, 35, Q144 Anorectal disorders, 107-108
Achilles tendon Anorectal pain, chronic, 108
rupture of, 62 Anorexia, 32
tendinopathy of, 61-62 Anorexia nervosa, 113-114, Q143
Acromioclavicular joint, degeneration of, 56, Q26 Anterior cruciate ligament
Activities of daily living (ADLs), 116-117, 117t examination of, 59, 59f
Acupuncture, 35 injuries to, 59
Acute coronary syndrome (ACS) Anterior drawer test, 59, 591"
chest pain'in, 48, 49, Q101
Anticoagulant therapv, perioperative management of, 126t, 129-130, 1311,
hospitalization for, 49 132,Q142
Adhesive capsulitis, 55-56
Antidepressants, 109, HOt, Q56
Adolescents for insomnia, 44
medical confidentiality for, 26 Antiemetics, 32
sexually active, 18 Antihistamines, for insomnia, 44, 45
Adrenal insufficiency, perioperative management of, 132-133
Anti-La/SSB antibodies, Q44
Advance directives, 24
Antiplatelet medications, perioperative management of, 126t, 130-131, 132
Alcohol dependence, 17
Anti-Ro/SSA antibodies, Q44
Alcohol use/abuse Antiseizure medications, perioperative management of, 126t
insomnia in, 44, Q153
Anxiety, in anorexia nervosa, 114
interventions for, 18, Q120
in physicians, 27, 27t Anxiety disorders, 110-112, Q146
Aortic dissection, chest pain in, 49, 50
screening for, 7t, 8, 9, 17-18
Alcohol Use Disorders Identification Test (AUDIT), 17 Apley scratch test, 54, 54t
Apolipoprotein A-l, 64
a-blockers, for benign prostatic hyperplasia, 78, 78t, Q70
Alzheimer's disease, 118 Apolipoprotein li, 64
Apprehension test, 54, 54t, 5Sf
Amenorrhea, anorexia nervosa and, 113
American Academy of Family Physicians, cancer screening guidelines from, 10 Arrhythmias, as syncope cause, 46, Q156
Arteritis, temporal, 99, Q95
American Academy of Ophthalmology, 119
Arthritis, septic, of the knee, 58
American Cancer Society
cancer screening guidelines from, 10 Aspergillosis, in immunocompromised patients, 39
cervical cancer screening guidelines from, lOt Aspirin
as adjunct to dyslipidemia management, 69, 691
mammography recommendations from, 9-10
gastrointestinal bleeding and, 69, 691
osteoporosis screening guidelines from, 9 for metabolic syndrome, 68, 68t
American College of Cardiology/American Heart Association, 125,127, Q151, Q168
American College of Chest Physicians, 36 perioperative management of, 130, 132
American College of Obstetricians and Gynecologists Asthma
mammography recommendation from, 9 cough-variant, 37, 38f, 39
as indication for pneumococcal vaccination, 14, 16, Q159
pelvic pain treatment recommendation from, 92
American College of Physicians Atherosclerosis, 68
cancer screening guidelines from, 10 Atorvastatin, 64-65
clinical practice guidelines from, 4, 7 Attention-deticit/hyperactivity disorder, 115, Q93
evidence-based medicine resources from, 4 Autism spectrum disorders, 115-116, Q128
High-Value Cost-Conscious Clare initiative of, 23
low back pain diagnostic testing recommendation from, 51 Bacteriuria, screening for, 7t, 9
Raker (popliteal) cyst, 61
position on physician-assisted suicide, 25, Q83
American Diabetes Association, diabetes screening recommendation from, 70 Bariatric surgery, 73-74, 731", 74t, Q150
American Geriatrics Society, 35, 1 17 Bayes theorem, 4
American Heart Association P-blockers, perioperative use of, 126t, 127, 128, Q125
Behavioral counseling, 16
optimal triglyceride levels guidelines from, 63
stroke prevention guidelines from, 69 Behavioral health. .SVr Mental and behavioral health
American Medical Association Behavioral therapy. See also Cognitive-behavioral therapy
alcohol abuse screening recommendation from, 8 for obesity, 72
mammography recommendation from, 9 for urinary incontinence, 121-122
older driver evaluation recommendation from, 120 Benign paroxysmal positional vertigo, 41, 42f, 43, Q14, Q20
position on physician-assisted suicide, 25, Q83 Benign prostatic hyperplasia, 78, 78t, QI8, Q70

259
Index

Benzodiazepines gastrointestinal causes of, 49


contraindication for cancer pain, 30 hospitalization for, 49-50
for insomnia, 44, 45, 45t, Q153 misdiagnosis of, 47
Bereavement, 32, 109 musculoskeletal causes of, 49, Ql 30
Bias, in study design, 1 pleuritic, 49, 50, Q104
lead-time, 7 pulmonary causes of, 49
length, 7 Child abuse, IS
Bile acid scquestrants, 65, 65t, Q75 mandated reporting of, 26
Binge-eating disorder, 114, Q154 Children, medical confidentiality for, 26
Bipolar disorder, 109, 112, QUO Chlamydial infection, 18
Bladder, overactive, in benign prostatic hypertrophy, 78 bacterial vaginosis as risk (actor for, 92
Bladder cancer, 10 screening for, 7t, 9, 11
Bladder outlet obstruction (BOO), 78 Chronic diseases, screening for, 7t, 8-9
Blepharitis, 95 Chronic fatigue, 39-40
Body dysmorphic disorder, 113, Q85 Chronic fatigue syndrome, 39-40, Q7, Q72
Body mass index (BMI), 69, 70, 70t, Q150 Chronic obstructive pulmonary disease (COPD)
/MC/l///{/{G42 gene mutations, 9 as perioperative pulmonary complications cause, 128, I28t
Breast, cystic, 83 preoperative care in, 128, Q47
Breast cancer, 84 Chronic pain
hormone therapy and, 88-89 in elderly patients, Q144
screening for, 7t, 9-10, 11 noncancer, 33-36, 34t, Ql 17
Breast masses, palpable, evaluation of, S2-84, 83t, Q45, Q90 pelvic
Breast pain, 84 in men, 80-81
Brief advice, 16 in women, 91-92,91t, Q135
Brief interventions, 16 Chronic Prostatitis Symptom Index, 80
Brief Pain Inventory (BPI), 33 Cirrhosis, perioperative management of", 133-134, Q62
Bronchitis, nonasthmatic eosinophilic, 37, 38f, 39, Q77 Clinical decision support, 22
Bulimia nervosa, 114, Q154 Clinical practice guidelines, 4, 7
Bupropion, 17 Clopidogrel, perioperative management of, 130, 132
Burning mouth syndrome, 106 Clue cells, 92,92f
Bursitis "Clunk" sign, 54t
olecranon, 56, QI32 Cluster-randomized trials, 1, 2t
pes anserine, 60, Q79 Coagulopathies, preoperative evaluation of, 131
prepatellar, 60 Cochrane collection, 4
trochanteric, 58 Codeine, 31t
Cognitive-behavioral therapy
CAGK questionnaire, for alcohol abuse diagnosis, 17 for anxiety disorders, 111
Caloric restriction, for weight loss, 72, Q51 for chronic fatigue, 40
Canadian Task Force on Preventive Health Care, 8 for chronic pain, 34-35
Canadian Task Force on the Periodic Health Examination, 119 for depression, 109
Cancer. Sec also specific types of cancer for eating disorders, 114
clinical course of, 28, 28f for insomnia, 44
constipation in, 30 for obsessive-compulsive disorder, 112, Q3
dyspnea in, 32 for social anxiety disorder, 111
fatigue in, 30 Cohort studies, 2, 2t
low back pain in, 511 Colon cancer, 107
screening for, 7t, 9-10 screening for, 7t, 11, Q48
Cancer pain, 29-30, 3 It, Q19, Q65 Colonoscopy, 107, Q48
Candidiasis, vulvovaginal, 92, 93, Q80 Common symptoms, 32-50
Capsulitis, adhesive, 55-56 chest pain, 47-50
Cardiac risk assessment, perioperative, 125, 128t, Q149, Q168 chronic fatigue, 39-40
Cardiac Risk Index (RCRI), Revised, 125, 128t, Q168 chronic noncancer pain, 33-36
Cardiovascular disease cough, 36-39
chest pain in, 48-50, 48t, Q101 dizziness, 4013
clinical course of, 28f edema, 50
dyslipidemia as risk factor for, 62-64, 63t insomnia, 4315
risk stratification for, Q66 syncope, 45-47
syncope in, 46, 47 Comparative effectiveness research (CFR), 3
Cardiovascular medications, perioperative management of, 126t Compression stockings, 50, Q22
Cardiovascular perioperative medicine, 125, 127-128, 1271", 128t,Q125
Computerized physician order entry (CPOE), 22
Cardiovascular testing, preoperative, 125, Q125 Concierge medicine, 6t
Carpal tunnel syndrome, 57, Q2 Condoms, 85t, 86
Case-control studies, 2,2t Confidence interval (CI), 1
Case series, 2
Confidentiality, 25-26
Cataracts, 97 Conflicts of interest, 26, 27t
Cauda equina syndrome, 50, 51t Confounding, I
Celecoxib, 35 Conjunctivitis, 93-94, 95
Cellulitis, orbital, 99 bacterial, 94, 94f
Centers for Disease Control and Prevention (CDC), 9 viral, 93, 94f,Qll
Advisory Committee on Immunization Practices, 12 Constipation, in cancer patients, 30
Centor criteria, for pharyngitis triage, 104, Q161 Contraception
Central adiposity, 70
emergency, 85t, 86
Central retinal artery occlusion, 99, 99f, Q95 for men, 85t, 86
Central retinal vein occlusion, 99, Q6I for women, 84-86, Q69, Ql 64
Cerumen, impaction of, 103 for women smokers, 84, 86, Q164
Cervical cancer, screening for, 7t, 10, Kit, 11, Q34 Control groups, Q92
Cervical neurogenic pain, 53 Conversion disorder, 1 12-1 13
Charter on Medical Professionalism, 23, 23t Core measure sets, 20
Chemical injuries, ocular, 99 Cornea
Chest pain, 47-50 abrasions to, 94-95
cardiovascular causes of", 48-50, 48t, Q101 edema of, 97
differential diagnosis of, 47-49 ulcers of, 95

2 6 0
Index

Coronary artery bypass grafting, 125, 130 Drivers, older, 120, Ql 18


Coronary artery disease (CAD) Drop-arm test, 54t
chest pain in, 48, 48t Drug abuse, 18
erectile dysfunction management in, Q166 of opioids, 36
screening for, 8-9 in physicians, 27, 27t
Coronary heart disease (CHD) Drug Abuse Screening Test (DAST-10), 18
dyslipidemia management in, 63, 63t Dry eye, 97-98, Q44^
hormone therapy and, 88 Durable power of attorney for healthcare, 24
Coronary revascularization, 125 Dysequilibrium, 43
Corticosteroids, 30, 32 Dyslipidemia
perioperative management of, 126t, 132-133 in elderly patients, 68-69
Costochondritis, Q130 evaluation of, 62-64, 63t, Q141
Cough, 36-39 management of, 64-67, 65t, 66t, 68t, Q66
ACE inhibitors and, Q81 in pregnancy, Q75
acute, 2>7 screening for, 62
chronic, 37-39, 38f, Q38 Dysmenorrhea, 90-91, Ql 31
in gastroesophageal reflux disease, 37, 38f, 39, Q38 Dyspareunia, 82, 88, 89, Q5
in immunocompromised patients, 39 Dyspnea, at end of lite, 32, Q6
in nonasthmatic eosinophilic bronchitis, 37, Q77
subacute, 37-39 Far, nose, mouth and throat disorders, 100-107
C-reactive protein, 64, Q66 cerumen impaction, 103
Creatine kinase, 65, 67 epistaxis, 105
Cross-arm test, 54t hearing loss, 100-101, 1011
Crossed straight-leg raise test, 51 oral health, 105-106
Cross-sectional studies, 2 otitis externa, 102-103
Cubital tunnel syndrome, 56, Q132 otitis media, 102
Cyclooxygenase-2 (COX-2) inhibitors, 35 temporomandibular disorders, 106-107
Cystitis, interstitial, 91, 911, Q135 tinnitus, 101-102
Cysts upper respiratory tract infections, 103-105
Baker (popliteal), 61 Eating disorder not otherwise specified (EDNOS), 114
breast, 83 Eating disorders, 113-114, Q143, Q154
epididymal, 79 domestic violence and, 19
ganglion, 57 in men, 77
Cytochrome P-450 3A4 isoenzyme inhibitors, 64-65 Ecologic fallacy, 2-3
Edema, 50
Decisional capacity. 24-25, 26, Q109 corneal, 97
Delirium, in terminally ill patients, 32 of the knee, 58-59
Dementia, 118 venous stasis-related, 50, Q22
clinical course of, 28f Edmonton Symptom Assessment Scale, 29
Dental disease/infection, 105, 106 Effect size, 4
Depression, 108-110 Eicosapentaenoic acid, 66-67
in anorexia nervosa, 114 Elbow, golfer's, 56, Q132
in bipolar disorder, 109, 112, Q140 Elbow pain, 56
in chronic fatigue, 40 Elder abuse, 18
in elderly patients, 119, Q99 Elderly patients. See Geriatric medicine
in low back pain, 50 Electrocardiography (ECG), for syncope evaluation, 46, 47
management of, 109, 1 Kit, Q56 Electronic Health Record (EHR) Incentive program, 6t
in pain, 35 Electronic health records (EIlRs), 22
Electronic prescribing, 6t
postpartum, 109
screening for, 7t, 8, Q145 Embolism, pulmonary, chest pain in, 49
in terminally ill patients, 32 Endocrine diseases, perioperative management of, 132-133
Depressive disorders, diagnosis of, 108-109 End-of-life care, 25, 28-29, 281". See aba Palliative care
lie Quervain tenosynovitis, 57, Q96 Endolymphatic hydrops, 41
Dexamethasone, 32 Endometriosis, 91, 911, 92
Diabetes mellitus Endophthalmitis, 97,99
dyslipidemia management in, 63 Epicondylitis, 56, Q132
metabolic syndrome and, 67 Epidemiologic studies, 2-3
obesity and, 70, Q51, Q57 Epididymitis, 78, 79, Q42
perioperative management of, 132, Q160 Epiphora, 98
prevention of, 72 Episcleritis, 95, 95f
screening for, 7t, 8, 70 Epistaxis, 105, Q4
Diabetes Recognition Program (DRP), 6t Epley maneuver, 41,42f
Erectile dysfunction, 74-76, 75t, 76t, Ql 12
Diagnostic errors, 20, 21t, Q12
in cardiovascular disease patients, 75, 76t, Q166
Diagnostic tests
interpretation of, 3-4, 5t management of, 75, 76t, Q112, Q166
preoperative, 124, Q8 Errors, medical. See Medical errors
properties of, 4, 5t Estrogen therapy
Diet for hypoactive sexual desire disorder, 82, Q147
for dyslipidemia management, 64 for menopause symptoms, 88-89, 89t, Q25
for weight loss, 72, Q51 perioperative, 126t
Dietary behavioral counseling, 16 Ethics, 22-27, 23t
Difficult patient encounters, 116 Euthanasia, 25
Diphenhydramine, for insomnia, 44, 45 Evidence, sources of, 4
Diphtheria vaccine, 12t, 13 Evidence-based medicine, 4
DIRF score, 35-36 Exercise, for weight control, 72, Q51
Discharge, from hospital, 21, Q94 Experimental studies, l,2t
Discharge summaries, 21, 22t External validity, 1
Diuretics, for edema, 50, Q22 Eye disorders, 93-100
Dix-Hallpike maneuver, 41, 41t, Q14, Q50 blepharitis, 95
Dizziness, 40-43, Q55 cataracts, 97
nonspecific, 43 conjunctivitis, 93-94, 94f, 95, Ql 1
Domestic violence, 18-19, Ql 17, Q126 corneal conditions, 94-95, 97

261
Index

Eve disorders (continued) primary open angle, 96-97, 97t


dry eye, 97-98, Q44 Globe, traumatic injury to, 99
episcleritis, 95, 951" Glossitis, atrophic, 106
excessive tearing, 98 Golfer's elbow, 56, Q132
eye emergencies, 99-100, 99t Gonorrhea
glaucoma, 96-97, 97t, 119, Q95, Q165 bacterial vaginosis and, 92
macular degeneration, 96, 96f, 119, Q148 screening for, 7t, 9, 11
retinal detachment, 98, 98f, Q29 Granulomatosis
retinal vascular occlusion, 99, 991" with polyangiitis, 104, Q138
scleritis, 95 Wegener, 104, Q138
subconjunctival hematoma, 94, 941" Grief, 32, 109, Q137
uveitis, 95 Groin pain, 58
Eye examinations, in elderly patients, 119 Group A streptococcal pharyngitis, 104
Ezetimibe, 65, 65t, 66
Halitosis, 106
FABER test, 58, 581" Hallux valgus, 62
Falls, in elderly patients Hamate bone, fractures of, 57
management of, Q87 Hand, innervation of, 56f
patient assessment after, 117-118, Q78 Hand pain, 57
prevention of, 117-118, 119, Q67 Hawkins test, 54, 54i, 55f
Family history, 11 Hazard ratio, 3
Fatigue Healthcare Effectiveness Data and Information Set (HEDIS), 6, 6t
chronic, 39-10, Q7, Q72 Healthcare industry, conflicts of"interest with, 26, 27t
end-stage illness-related, 30 Healthcare services, unsafe and low-value, 23, 24
Feeding tubes, 32 Health care workers, vaccinations for, 15, 16
Fenofibrate, 66, Q41 Health examinations, periodic, 8
Fentanyl transdermal patches, 30, 3It, Q19 Health information technology (HIT), 22
Fibrates, 64-65, 65t, 66, 67, 68, Q41, Q66 Hearing loss
Fibroadenomas, 83 cerumen impaction and, 103
Fish oil supplementation, cardioprotective effects of, 66-67 common causes of, 100, lOIt
5a-reductase inhibitors, for benign prostatic hyperplasia, 78, 78t, Q70 evaluation of, 100
Fluvastatin, 65 in older adults, 119
Folic acid supplementation, 87 otitis media and, 102
Follicle-stimulating hormone (FSH), 87-88 screening for, 119
Foot pain sudden sensorineural, 100, Q20, QI08
forefoot, 62 unilateral sensorineural, 100, Q20
hindfoot, 61-62 Fleart failure, preoperative, 127
midfoot, 62 Hematochezia, 107, Q48
Forefoot pain, 62 Hematologic perioperative medicine, 129-132
Fractures Hematoma, subconjunctival, 94, 94f
compression, as low back pain cause, 511 Hemoptysis, 39
of wrist and hand, 57
Hemorrhage
Frail elderly. See also Geriatric medicine abnormal uterine, 90, Q53
clinical course in, 281" rectal, 107, Q48
outcome measures for, 5 Hemorrhoids, 107, Q48
urinary incontinence screening in, 121 Heparin therapy, perioperative, 129-130
FRAMES model, of brief interventions, 16 Hepatitis, perioperative management of, 133
Functional assessment, of elderly patients, 116-120 Hepatitis A vaccine, I2t, 15
Hepatitis B, 18
Gabapentin, 35, Q65 screening for, 7t, 9, 11
Gait analysis, in elderly patients, 117
Hepatitis B vaccine, 12t, 15, 15t, 16, Q162
Gastric bypass surgery, 73, 7M, 74t, Q15
Hepatitis C, 18
Gastroesophageal reflux disease (GERD) screening for, 7t, 9, 11
chest pain in, 49
Hepatotoxicity, statin-related, 65, Q158
cough in, 37, 381", 39, Q38 Herbal supplements
Gastrointestinal medications, perioperative management of, 126t for menopausal symptoms, 89
Generalizability, of study results, Q127 for pain, 35
Generalized anxiety disorder (GAD), 110-111, Q146
preoperative discontinuation of, 126t
Genetic counseling, referral for, 11-12, Ql, Q16
Hernia, inguinal, 81
Genetic testing, 11, Ql, Q16 Herpes simplex virus, 18
Genital herpes, bacterial vaginosis as risk factor for, 92
Herpes zoster ophthalmicus, 99-100
Geographic tongue, 106 Herpes zoster virus, chest pain and, 47
Geriatric medicine, 116-124 Herpes zoster virus vaccine, 12t, 14
acetaminophen use, 35, Q144 Heuristics, 211, Ql2
chronic pain management, Q144
High-density lipoprotein (HDL) cholesterol, 62, 63i, 64, 65t, 66
depression, 119, Q99 low-carbohydrate diet and, 72
dizziness, 40, Q55 smoking cessation and, 67
driving, 120, Ql 18 Hindfoot pain, 61-62
dyslipidemia management, 68-69 I lip, osteonecrosis of, 58, Ql 16
falls, 117-118, 119, Q67, Q78, Q87 Hip pain, 58, 58f
frail elderly, 5, 28f, 121 Home safety, for elderly patients, 118
functional assessment, 116-120 Hormone therapy. Sec also Estrogen therapy; Testosterone therapv
levels of care in, 120 for menopause symptoms, 88-S9, 89t, Q25, Q60
polypharmacy, 120-121, Q46 Hospice care, 28-29. Sir also Palliative care
pressure ulcers, 123-124, 124t, Q17, Q24 Hot flushes, 88
syncope management, Q156 "Housemaid's knee," 60
urinary incontinence, 121-123, 122f Human immunodeficiency virus (HIV) infection
Glaucoma, 119 bacterial vaginosis as risk factor for, 92
acute angle closure, 97, Q95
screening for, 7t, 9, 11, Q123
adverse effects of treatment for, 96, 97t, Q165 sexual transmission of, 18

262
Index

Human immunodeficiency virus-positive patients, vaccination of, 13, Q129 Lifestyle risk factors, interventions for, 16-19
Human papillomavirus, 10, 18 Life-sustaining treatment, withholding or withdrawing of, 25, Q43
Human papillomavirus vaccine, 12t, 14-15 Likelihood ratio (LR), 4, 5t
Hutchinson sign, 99-100 Liver chemistry tests, in statin use, 65, Ql 58
Hydrocele, 79 Liver disease, perioperative management ot, 133-134
Hydrocodone, 311 Living wills, 24
Hydromorphone, 30, 3It, Ql9 Long-term care, 120
Hyperlipidemia. See Dyslipidemia Lorcaserin, 73
Hyperplasia Lovastatin, 64-65
benign prostatic, 78, 78t, Q18, Q70 Low back pain, acute, 50-52, 5It, Q91
gingival, 105 Low-density lipoprotein (LDL) cholesterol
Hypertension evaluation of, 62-63
pulmonary, as chest pain cause, 49 goal level of, 62, 63, 63t
screening for, 7t, 8, Q102 management of, 63t, 64-66, 65t, 67, 68t, 69, Q52, Q75
Hypertriglyceridemia, 63-64 metabolic syndrome and, 68, 68t
management of, 65t, 66-67, Q4I pregnancy and, 65t, Q75
Hypoactive sexual desire disorder, 81-82, Q5, Q147 stroke and, 69
Hypochondriasis, 113 Lower respiratory tract infections, as cough cause, 37
Hypoglycemic agents, perioperative management of, 126t Lower urinary tract symptoms (LUTS), in men, 78
Hypotension, orthostatic, 46, 47, Q86 Lp(a) lipoprotein, 64
Lumbar disk disease, 58
Iliotibial band syndrome, 60 Lung volume expansion, perioperative, 128, 129, Q47
Immunization. See also specific vaccines
for adults, 12-16, 12t Macular degeneration, 96, 96f, 119, Q148
multiple-dose, 15, Q162 Malcolm Baldrige Model for Performance Excellence, 19
preconception administration of, 87 Mammography
Immunocompromised patients. Sec also Human immunodeficiency virus (HIV) for palpable breast mass evaluation, 83
infection screening, 7t, 9-10, 11
cough in, 39 Mastalgia, 84, Q139
pneumococcal immunization in, 14, 14t McMurray test, 59, 591"
Immunomodulators, perioperative management of, 126t Measles, mumps, rubella vaccine, 12t, 15, 16
Infection, low back pain and. Sir Medial collateral ligament injuries, 59
Infectious diseases Medial-lateral grind test, 59, 61
reporting of, 26 Median nerve, 56f
screening for, 7t, 9 Medical errors, 19-22, 21t, Q12
Inferior vena cava filters, 129 disclosure of, 26
Influenza vaccine, 12t, 13, 15, 87, Q71 medication-related, 20, Ql 13
Informed consent, 24, Q133 sources of, 20
Insomnia, 43-45 "Swiss cheese" model of, 19, 191"
anxiety disorders and, 110 Medical literature, interpretation of, 1-4
evaluation of, 44, Ql 15 statistical analysis, 3-4
management of, 4445, 45t, Ql 15, Ql 53 study design, 1-3, 2t
Institute for Clinical Systems Improvement, 33
Institute of Medicine (IOM) terminology, 3t
To Err is Human: Bttildbitj a Safer Health System, 19 Medically unexplained symptoms (MUSs), 112
Medicare, 28
fall prevention recommendations of, 117 Medication errors, 20, Ql 13
obesity definition of, 69 Medications. See also specific medications
vitamin D supplementation recommendation of, 118
erectile dysfunction and, 74-75, 75t
Instructive directives, 24
Instrumental activities of daily living (IADLs), 116-117, 117t perioperative management of, 125, 126t, Q82
Insulin resistance, 68 teratogenic, 86, 87t, Q32
Insulin therapy, perioperative, 132, Q160 weight gain and, 70t
Intermittent explosive disorder, 112 Melatonin, for insomnia, 4415
Internal validity, in study design, 1 Melatonin-receptor agonists, for insomnia, 41 15, 45t
Meniere disease, 41, Q14, Q20
International Index of Erectile Dysfiinction-5 (IIEF-5), 75
Intimate partner violence, 18-19, Ql 17, Q126 Meningococcal vaccine, 12t, 15
Menisci
Intraocular pressure (IOP), 96-97
examination of, 59, 591", 61
Irritable bowel syndrome, 91, 92
injuries to, 59
Joint Commission (formerly JCAHO), 20, 22 Menometrorrhagia, 90
Joint National Committee on Prevention, Detection, Evaluation, and Menopause
Treatment of High Blood Pressure, 8, QI02 urogenital symptom management in, 89, Q25
vasomotor symptom management in, 88-89, 89t, Q60
Kegel exercises, 121, Q33, Q88 Menorrhagia, 90
Men's health, 74-81
Keratoconjunctivitis sicca, 97-98, Q44 acute testicular and scrotal pain, 78-79
Kidney disease, perioperative management of, 133
Knee androgen deficiency, 77, 77t, Q40
ligamentous injuries to, 59 benign prostatic hyperplasia, 78, 78t, Q18, Q70
osteoarthritis of, 58, 59 epididymal cysts, 79-80
Knee pain, 58-61 hernia, 81
diagnosis and evaluation of, 58-59, 591", 601" hydrocele, 79
pelvic pain, 80-81
Lachman test, 59, 59f prostatitis, 80-81, 8()t
Lacrimation, 98 sexual dysfunction, 74-77, 75t, 76t
Lateral femoral cutaneous nerve, entrapment syndrome ot, 58, Q155 varicocele, 79-80
Laxatives, 30 Mental and behavioral health, 108-1 16
Lemierre syndrome, 104, Q157 anxiety disorders, 110-112, Q146
Libido, decreased attention-deficit/hyperactivity disorder, 115, Q93
in men, 77 autism spectrum disorders, 115-116
in women, 81-82, Q5, Q147 bipolar disorder, 109, 1 12, Q140

263
Index

Mental and behavioral health (continued) for elbow pain, 56


depression, 108-110 for foot pain, 62
anorexia nervosa and, 114 for knee pain, 60
bipolar disorder and, 109, 112,Q140 for low back pain, 52
chronic fatigue and, 40 for neck pain, 53
in elderly patients, 119, Q99
nephrotoxicity of, 133
low back pain and, 50 for pelvic pain, 80,81,91
management of, 109, 1 lOt, Q56 for rotator cuff disorders, 55
pain and, 35 use in elderly patients, 35
postpartum, 109 for wrist .md hand pain, 57
screening for, 7t, 8, Q145 Nosebleed, 105
in terminally ill patients, 32 Number needed to harm (NNFI; 3 t , 4
difficult patient encounters, 116, Q73 Number needed to treat (NNT), 31, 3t, Q58
eating disorders, 19, 77, 113-114, Q143, Q154
intermittent explosive disorder, 112 Obesity, 69-74
schizophrenia, 77, 114-115, Q68 adverse health effects of, 70, 74, Q57
somatoform disorders, 112-113, Q105 definition of, 69-70, 70t
Meralgia paresthetica, 58, Q155 epidemiology of, 69
Meta-analyses, 3 erectile dysfunction in, 74
Metabolic equivalents (METs), 125, I27f, QI68 evaluation of, 70
Metabolic syndrome, 67-68, 67t, Q64 screening for, 7t, 8, 70
"ABCDE" approach to management of, 67-68, 68t treatment of, 71-74, 71f, 73f, 74t, Q51, Q150
Methadone, 30, 36, Ql 36 Observational studies, 1-3, 2t
Metrorrhagia, 90 Obsessive-compulsive disorder, 111-112, 113, Q3
Midfoot pain, 62 Obstructive sleep apnea (OSA), perioperative management of, 128-129, I29t,
Migraine, ocular/retinal, Q95 Q121
Mild cognitive impairment, 118-119, 120, Q35, QI03 Odds ratio, 3
Mini-Mental State Examination (MMSE), 118 Older patients. See Geriatric medicine
Morphine, 30,3It, Q19 Olecranon bursitis, 56, Q132
Motivational interviewing, 16, 72, Q63 Omega-3 fatty acids, 65t
Motor vehicle accidents, in older drivers, 120, Ql 18 Opioid analgesics, 29-30, 29f, 3It, 34t, 35-36
Mucosal lesions, oral, 105, 106 abuse of, 30, 35, 36, Q152
Multiple sclerosis, perioperative management of, 134 for cancer pain, 29-30, 3It, Q19, Q65
Mumps vaccine, 12t, 15, 16 for chronic neuropathic pain, 34t, 35, QI36
Muscle relaxants for chronic noncancer pain, 34t, 35-36
for low back pain, 52
constipation and, 30
for neck pain, 53 for low back pain, 52
Musculoskeletal pain, 50-62 for neck pain, 53
acute low back, 50-52, 5It, Q9I Oral cancer, 10, 105
ankle and foot, 61-62, 611", Q59 screening for, 106
elbow, 56, 56f Oral contraceptives, 84, 85t
hip, 58, 58f for abnormal uterine bleeding, 90
knee, 58-61, 59f, 60f for dysmenorrhea, 90-91
neck, 52-53, Q37, Ql34 Oral health, 105-106
shoulder, 53-56, 54f, 54t, 551" Oral infections, 106
wrist and hand, 57 Oral ulcers, 106
Myocardial infarction Orchitis, 78-79
chest pain in, 50
Orgasmic disorder, 82
perioperative, 125 Orlistat, 72-73
Myopathy, statins and, 64, 66t ORYX initiative, 20
Osteoarthritis
National Cholesterol Education Program Adult Treatment Panel III (ATP III), of the hand, 57
8, 62, 63, 63t, 64, 65, 69, Ql51 of the hip, 58
National Committee for Quality Assurance (NCQA), 6 of the knee, 58, 59
National Comprehensive Cancer Network, 9 Osteonecrosis, of the hip, 58, Ql 16
National Institutes of Health (NIH) Osteoporosis, screening for, 7t, 8, 9
Chronic Prostatitis Symptom Index, 80 Otitis externa, 102-103, Q163
obesity screening guidelines from, 70 Otitis media, 102
National Quality Forum, 26 Ottawa ankle .\m\ foot rules, 61, 61 f
Nausea, chronic, 32 Outcome measures, of healthcare, 5-6
Neck pain, 52-53, Q37, Q134 Ovarian cancer, 10
Neer test, 54f, 54, 54t Overdiagnosis, 7
Negative likelihood ratio (LR-), 5t Overuse syndrome, of the knee, 59-60
Negative predictive value (NPV), 4, 5t Overweight, 70t
Neural tube defects, 87 Oxycodone, 30, 311
Neuritis, optic, 99, 100
Neurologic disease, perioperative management of, 134 Pain
Neuroma, Morton, 62, Q84 cancer, 29-30, 3It, Q19, Q65
Neuronitis, vestibular, 41, 43, Q14 chest, 47-50
Neuropathic pain, 34t, 35, Q136 cardiovascular causes of, 48-50, 48t, Q101
Niacin, 66 differential diagnosis of, 47-49
Nicotinic acid, 65t, 66, 67 gastrointestinal causes of, 49
Noble test, 60 hospitalization for, 49-50
Nonsteroidal anti-inflammatory drugs (NSAIDs), 29, 29f, 35 misdiagnosis of, 47
for abnormal uterine bleeding, 90 musculoskeletal causes of, 49, Q130
for acromioclavicular joint degeneration, 56 pleuritic, 49, 50, Q104
for adhesive capsulitis, 56 pulmonary causes of, 49
for ankle pain, 61 chronic
for cancer pain, 29, Q65 in elderly patients, Q144
for chronic pain, Ql 17 noncancer, 33-36, 34t, Ql 17
for dysmenorrhea, 91, Q131 groin, 58

264
Index

musculoskeletal, 50-62 Posterior vitreous detachment, 98


acute low back, 50-52, 5It, Q91 Posttest odds, 5l
ankle and foot, 61-62, 6If, Q59 Posttest probability, 5l
elbow, 56, 56f Posttraumatic stress disorder (PTSD), 111, Q100
hip, 58, 58f Pravastatin, 65, Q52
knee, 58-61, 59f,60f Preconception counseling, 86-87, 87t
neck, 52-53, Q37, Ql 34 Predictive value, of diagnostic tests, 4
shoulder, 53-56, 54f, 54t, 55f Pregabalin, 35
wrist and hand, 57 Pregnancy
alcohol abuse screening during, 17
neuropathic, 34t, 35, Q136
palliative care management of, 29-30, 291, 311 hyperlipidemia management and, Q75
pelvic pain, chronic medication management during, 86, 87t, Q32
in men, 80-81 preconception counseling for, 86-87, 87t
in women, 91-92, 91t,Q135 vaccination during, 13
scrotal, 78-79 vulvovaginal candidiasis during, 93, Q80
testicular, 78-79 Premature ejaculation, 76
Pain contracts, 36 Premenstrual dysphoric disorder, 109
Palliative care, 27-32 Preoperative risk assessment, 124
assessment and communication in, 29, Q36, Q89 Preoperative testing, 124, Q8
hospice, 28-29 Prepatellar bursitis, 60
nonhospice, 28, 29, Q76 Pressure ulcers, 123-124, 124t, Q17, Q24
symptom management in, 29-32 Presyncope, 43
Pancreatic cancer, 10 Pretest odds, 5t
Panic disorder, 110, 111, Ql 19 Pretest probability, 5t
Parkinson disease, perioperative management of", 134 Prevalence, 5t
Patellofemoral pain syndrome, 59-60, Q54 Professionalism, 22-27, 23t
Patient-centered medical home (PCMH), 6t Prostate cancer, screening for, 10, Q49
Patient Health Questionnaire (PHQ-9), 109, 119 Prostate-specific antigen (PSA), 10, 77
Patient identification, 22 Prostatitis, 80-81, 80i, Q106
Patient refusal, of treatment, 25 Proxy directives, 24
Patients, difficult encounters with, 116, Q73 Pruritus ani, 108
Patient safety, 19-22 Psychiatric medications, perioperative management of, 126t
health information technology and, 22 PubMed, 4
national goals for, 22 Pulmonary medications, perioperative management of, 126t
Pay for performance (P4P), 6t Pulmonary perioperative medicine, 128-129, 128t, 129t
Pelvic adhesions, 91, 911 P value, 4
Pelvic floor muscle training, 121, Q33, Q88
Pelvic pain, chronic Quality improvement, 6
in men, 80-81 measurement of, 20
in women, 91 -92, 911, Q135 Quality improvement models, 19-20,201"
Pelvic varices, 91,911 Quasi-experimental study design, 1, 2t
Percutaneous coronary intervention (PCI), 125, 127
Performance measures, 5-6, 6t Radial nerve, 561'
Pericarditis, acute, chest pain in, 49, Q39 Radiculopathy
Periodontal disease, 105, 106 cervical, 134, Q37
Perioperative medicine, 124-134 low back pain in, 51, 5It, 52
cardiovascular, 125, 127-128, 1271", 128t Raloxifene, 10
endocrine, 132-133 Randomized controlled trials (RCTs), 1, 2t, Q127
general recommendations for, 124-125 Rectum, bleeding from, 107, Q48
hematologic, 129-132 Relative risk, 3
kidney disease, 133 Relative risk reduction (RRR), 3, 3t
liver disease, 133-134 Relative risk (RR), 3, 3t, Q58
neurologic disease, 134 Relocation test, 54t
pulmonary, 128-129, 128t Respiratory depression, morphine and, 30
Pertussis, 37 Retinal detachment, 98, 981", Q29
Pertussis vaccine, 12t, 13,87 Retinal vascular occlusion, 99, 991"
Pes anserine bursitis, 60, Q79 Revised Cardiac Risk Index (RCRI), 125, 128t, Q168
Pharyngitis, 104, Q161 Rhabdomyolysis, statins and, 66t
Phosphodiesterase inhibitors, 75-76, 78, Q166 Rheumatoid arthritis, of the knee, 58
Physical activity. .Sir also Exercise Rhinitis
promotion of, 16 allergic, 103-104
Physician-assisted suicide, 25, Q83 nonallergic, 104, Q138
Physician health programs, 27 Rhinitis medicamentosa, 104, Q138
Physician Quality Reporting System (PQRS), 6t Rhinorrhea, 103
Physicians, impaired, 27, 27t, Q124 Rhinosinusitis, 37, Q13
Piriformis syndrome, 58 Rinne test, 100, Q20
Pivot test, 591" Romberg test, 51
Plan-Do-Study-Act (PDSA) model, for quality improvement, 19, 20f, Q30 Rosuvastatin, 65
Plantar fasciitis, 61, Q167 Rotator cuff disorders, 54-55, 56, Q23, Q122
Platelet deficiencies, perioperative management of, 131 Routine care, of healthy patients, 419
Pleuritis, as chest pain cause, 49, 50 family history, 1 1
Pneumococcal polysaccharide vaccine, 12t, 14, 14t, 16, Q74.Q159 genetic testing and counseling, 11-12
Pneumonia immunization, 12-16, 12t
Pneumocystis jirovccii, 39 lifestyle risk factors management, 16-19
as shoulder pain cause, 53 screening tests, 6-11
Pneumothorax, as chest pain cause, 49, Q104 Roux-en-Y gastric bypass surgery, 73, 731", 74t, Q15
Polvmenorrhagia, 90 Rubella vaccine, 12t, 15, 16,87
Polypharmacy, 120-121, Q46
Positive Qkelihood ratio (LR+), 5t Sacroiliitis, 58
Positive predictive value ( PPV). -1. 5t Scaphoid bone, fractures of, 57
Postconcussion syndrome, 111 Schirmer test, Q44
Posterior drawer test, 59 Schizophrenia, 114-115, Q68

265
Index

Sciatica, 51-52, 5It Statistical analysis, 31


Scleritis, 95 Sterilization
SCOFF questionnaire, for eating disorders screening, 113, 113t, 114 female, 85t, 86
Screening, 6-11 male, 85t
for abdominal aortic aneurysms, 7t, 8, Q151 STOP-BANG questionnaire, for sleep apnea, 128-129, 129t
for alcohol abuse, 7t, 8,9, 17-18 Straight-leg raise test, 51
for breast cancer, 7t, 9-10, 11 Stress testing, preoperative, 125, Q168
for cancer, 7t, 9-10 Stroke
for cervical cancer, 7t, 10, lOt, 11, Q34 perioperative, 134
for colon cancer, 7t, 11, Q48 prevention of, 69
for coronary artery disease, 8-9 Study design, 1-3, 2t
for depression, 7t, 8, Q145 experimental studies, 1, 2t
for diabetes mellitus, 7t, 8, 70 observational studies, 1-3, 2t
for drug abuse, 18 validity, 1
for dyslipidemia, 62 Substance use disorders, 17-18. See also Alcohol use/abuse; Drug abuse
for eating disorders, 113, 113t, 114 domestic violence and, 19
for hearing loss, 119 in physicians, 27, 27t
for hepatitis B, 7t, 9, 11 Substituted judgment, 24-25, Q109
for hepatitis C, 7t, 9, 11 Suicide
during history and physical examination, 8 bipolar disorder and, 112
for human immunodeficiency virus (HIV) infection, 7t, 9,11, Q123 physician-assisted, 25, Q83
for infectious diseases, 7t, 9 tramadol and, 35
for obesity, 7t, 8, 70 Sulcus sign, 54t
for oral cancer, 106 Surrogate decision-making, 24-25
for osteoporosis, 7t, 8, 9 Sympathomimetic drugs, for weight loss, 72
for prostate cancer, 10, Q49 Syncope, 45-47
for sexually transmitted diseases, 7t, 9, 11, 18 cardiac causes of, 46, 47, Q21, Q156
for urinary incontinence, 121, 1221" diagnostic evaluation of, 4617, Q31
Seasonal affective disorder, 109 in elderly patients, 46, 47, Q21, Q156
Sedative-hypnotics, for insomnia, 44, 45t management of, 47
Seizure disorders, perioperative management of, 134 neurocardiogenic, 45-46, 47
Selective serotonin reuptake inhibitors (SSRIs), 109, HOt, 111, 114,119 orthostatic, 46, 47, Q86
Sensitivity, of diagnostic tests, 4, 5t risk stratification of, 47
Sexual arousal disorder, 82 vasovagal, Q86
Sexual aversion disorder, 81-82, Q5 Syphilis, screening for, 7t, 11
Sexual behavior Systematic reviews, 3
risky, 18, 19
sexual contact between physician and patient, 26 Tamoxifen, 10
Sexual behavior counseling, 18 Tarsal tunnel syndrome, 62
Sexual desire disorders, 81-82, Q5, Q147 Tearing, excessive, 98
Sexual dysfunction Temporomandibular disorders, 106-107, Q97
in men, 74-77, 75t, 76t, Ql 12, Q166 Tendinitis, rotator cuff, 54, 55, Q122
in women, 81-82, 81t, 88, 89, Q5, Q147 Tendinopathy, Achilles, 61-62
Sexually transmitted diseases Tennis elbow, 56, Q132
in adolescents, 18 Tenosynovitis, de Quervain, 57, Q96
screening for, 7t, 9, 11, 18 Teratogens, 86, 87t, Q32
in victims of intimate partner violence, 18-19 Terminally ill patients
Sexual pain disorders, 82 end-of-life care for, 25, 28-29, 28f
Shoulder, "frozen," 55-56 palliative care for, 27-32
Shoulder pain, 53-56, 54f, 54t, 55f Testicular torsion, 78, 79, Ql 10
Sigmoidoscopy, 107, Q48 Testosterone deficiency, 77, 77k, Q40
Simvastatin, 64-65, 66, Q158 'Testosterone therapy, 77, Q166
Sinusitis, 103, Q107 Tests. Sec Diagnostic tests
Six Sigma model, for quality improvement, 19-20 Tetanus, diphtheria, pertussis vaccine, 12t, 13, 15, 16, 87
Sjogren syndrome, 98, Q44 3 Incontinence Questions (3IQ), 121, 122f
Skin cancer, 10 Thromboembolic disease, estrogen-containing contraceptives and, 84, 86,
Sleep hygiene, 44, Q153 Q164
Smoking, 17 'Thrombosis, perioperative deep venous, 129-130, 13()t, 131t, Q28
abdominal aortic aneurysm and, 7t, Q151 Thrombotic thrombocytopenic purpura, 131
cough and, 37 'Thyroid diseases, perioperative management of, 132
erectile dysfunction and, 74 Thyroid medications, perioperative management of, 126t
implication for female contraception, 84, 86, Q164 Tilt-table testing, for syncope, 47, Q31
sinusitis and, 103 "Timed "Up and Go" (TUG) test, 117, Q78
Smoking cessation Tincl sign, 62
chronic cough and, 37 Tinnitus, 101-102
counseling in, 17, 18, Q10, Q27 pulsatile, 101, Q9
high-density lipoprotein cholesterol levels and, 67 To Err is Hitman: Building a Safer Health System (Institute of Medicine), 19
pharmacotherapy in, 17, 18 'Tongue syndromes, 106
Social anxiety disorder, 111 Toxic shock syndrome, 105
Somatization, 50, 112, Q105 Tramadol
Somatoform disorders, 112-113, Q105 for low back pain, 52
Specificity, ofdiagnostic tests, 4, 5t for noncancer chronic pain, 35, Q136
Speed maneuver, 54t Transitions in care, 20-21, 22t, Q94, Q113
Spinal cord compression, 53, Q37 Transjugular intrahepatic portosystemic shunt (TIPS), 134
Spinal stenosis, low back pain and, 51, 5It, 52 Transurethral resection of the prostate (TURP), 78
Spirometry, preoperative, 128, 129 Traumatic brain injury, 111
Spondyloarthropathies, 50 Trichomoniasis, 92-93
Spurling test, 54t Tricyclic antidepressants
"Squeeze" test, 61 for pain, 35
Statins, 64-65, 65t, 68, Q52 perioperative management of, 126t
adverse effects of, 64-65, 66t, Q158 "Triglycerides, 63-64

2 6 6
Index

Trochanteric bursitis, 58 Very-low-density lipoprotein (VLDL) cholesterol, 63


'Tubal ligation, 85t, 86 Visual acuity lest, 119
'Tuberculosis, in immunocompromised patients, 39 Visual impairment, in older adults, 119
TWEAK test, 17 Vitamin B,2 deficiency
atrophic glossitis and, 106
Ulcers in bariatric surgery patients, 74, 74t, Q15
corneal, 95 Vitamin D supplementation, for older adults, 118, Q87
oral, 106
pressure, 123-124, 124t, Q17, Q24 Warfarin, perioperative management of, 126t, 129-130, 131t, 132,
Ulnar nerve, 561*
Q142
entrapment of, 56, Q132 Weber test, 100
Ultrasonography, for abdominal aortic aneurysm screening, 8, Q151 Wegener granulomatosis, 104, Q138
Underweight, 70t Weight gain, medication-related, 70t
United States Preventive Services'Task Force (USPSTF)
obesity management recommendations from, 72 Weight loss, for obesity control, 71, Q51
Whooping cough. See Pertussis
screening recommendations from, 7, 7t, 8, 9, Kit, 11, 62, 64, 70, 106,
119, Q151 Withholding/withdrawing, of treatment, 25, Q43
Women's health, 81-93
Upper airway cough syndrome (UACS), 37, 381" abnormal uterine bleeding, 90, Q53
Upper respiratory tract infections, 103-105
cough in, 37 breast cancer, 84
ear pain in, Ql 14 breast masses, 82-84, 83t
Urinary incontinence, 121-123, 1221", Q33, Q88 breast pain, 84
Uterine bleeding, abnormal, 90, Q53 chronic pelvic pain, 91-92,91t
Uveitis, 95 contraception, 84-86
dysmenorrhea, 90-91
Vaginal estrogen therapy, 89, Q25 menopause, 87-89, 89t
Vaginal symptoms, postmenopausal, 89, Q25 postpartum depression, 109
Vaginismus, 82, Q5 preconception counseling, 86-87, 87t
Vaginosis, bacterial, 92,921", QUI premenstrual dysphoric disorder, 109
Valgus stress test, 591", 601" sexual dysfunction, 81-82, 81t, 88, 89, Q5, QI47
Validity, threats to, 1,Q92 vaginitis, 92-93
Value, in healthcare, 5 Women's Health Initiative, 88
Varenicline, 17 World Health Organization, analgesic ladder of, 29-30, 291", 35
Varicella vaccine, 12t, 13-14, 15, 16,87
Wrist pain, 57
Varicocele, 79-80
Varus stress test, 59, 59f, 601"
Xerophthalmia, 97-98, Q44
Vasectomy, 85t
Vasomotor symptoms, of menopause, 88-89, 89t, Q60 Xerostomia, 105, 106
Venous stasis, as edema cause, 50, Q22
Vertigo, 40-43 Yergason test, 54, 54t, 55f
benign paroxysmal positional, 41, 421", 43, Q14, Q20, Q98
central, 4It, 43, Q14, Q50 Zaleplon, for insomnia, 44, 45t, Q153
peripheral, 41, 4It, 421", 43, QI4, Q98 Zolpidem, for insomnia, 44, 45t, Q153

267
El
M NAME AND ADDRESS (Please complete.)

Last Name First Name Middle Initial


ACP
American College of Physicians
INTERNA! MEPICINI I IJ.KI.it. )ur AJuIk
Medical Knowledge
Self-Assessment
Program 16
Address TO EARN AMA PRA CATEGORY 1 CREDITS
YOU MUST:
Address cont.
1. Answer all questions.
2. Score a minimum of 50% correct.
City State ZIP Code

TO EARN FREE SAME-DAY


Country AMA PRA CATEGORY 1 CREDITS ONLINE:

1. Answer all of your questions.


Email address
2. Go to mksap.acponline.org and access the
appropriate answer sheet.
3. Transcribe your answers and submit for CME
D
I Order Number credits.
4. You can also enter your answers directly at
(Use the Order Number on your MKSAP materials packing slip.)
mksap.acponline.org without first using this
answer sheet.

To Submit Your Answer Sheet by Mail or FAX for


a $10 Administrative Fee per Answer Sheet:
1. Answer all of your questions and calculate
a ACP ID Number your score.
2. Complete boxes A-F.
(Refer to packing slip in your MKSAP materials
for your ACP ID Number.) 3. Complete payment information.
4. Send the answer sheet and payment information to
ACP, using the FAX number/address listed below.

COMPLETE FORM BELOW ONLY IF YOU SUBMIT BY MAIL OR FAX


Last Name First Name Ml

Payment Information. Must remit in US funds, drawn on a US bank.

The processing fee for each paper answer sheet is $10.

Check, made payable to ACP, enclosed

C h a r g e t o ( ] h / / S A | ^ ) , - ; - !

Card Number

Expiration Date Security code (3 or 4 digit #s)


MM YY

Signature

Fax to: 215-351-2799 Mail to:


Member and Customer Service
American College of Physicians
Questions? 190 N. Independence Mall West
Go to mskap.acponline.org or email custserv@acponline.org Philadelphia, PA 19106-1572
TESTTYPE
Maximum
Number of B Enter your score here.
CME CREDITS CLAIMED
Credits ON SECTION Instructions for calculating
O Cardiovascular Medicine 18 (1 hour = 1 credit) your own score
CD Dermatology 10 are found in front of the
O Gastroenterology and self-assessment test
Hepatology 14 in each book.
O Hematology and Oncology 20 Enter the number of
14 credits earned on You must receive
O Neurology
the test to the nearest a minimum score of
O Rheumatology 14
quarter hour. Physicians 50% correct.
O Endocrinology and Metabolism 12
should claim only the
O General Internal Medicine 24
credit commensurate
CD Infectious Disease 16 with the extent of
O Nephrology 16 their participation in
O Pulmonary and Critical the activity. Credit Submission Date:
Care Medicine 16

1 CS>CDCDCDCD 46 CS) CD CD CD CD 91 CS) CD CD CD CD 36 CS) CD CD CD CD


2 CD (DCS) CD 47 CS) CD CD CD CD 92 CS)CDCDCDCD 37 CS) CD CD CD CD
3 C (D C GD CD 48 CS) CD CD CD CD 93 CS) CD CD CD CD 38 CS) CD CD CD CD
4(S)CDCDCDCD 49 CS) CD CD CD CD 94 CS) CD CD CD CD 39 CS) CD CD CD CD
5 CS) CD CO GD CD 50 CS) CD CD CD CD 95 CS) CD CD CD CD 40 CS) CD CD CD CD

6 CDC CD CD 51 CS) CD CD CD CD 96 CS) CD CD CD CD 41 CS) CD CD CD CD


7 CD CD CD 52 CS) CD CD CD CD 97 CS)CDCDCDCD 42 CS) CD CD CD CD
8 CD CD CD CD CD 53 CS) CD CD CD CD 98 CS) CD CD CD CD 43 CS) CD CD CD CD
9CDCDCDCDCD 54 CS) CD CD CD CD 99 CS) CD CD CD CD 44 CS) CD CD CD CD
I oC5) CD CD CD CD 55 CS) CD CD CD CD 100 CS) CD CD CD CD 45 CS) CD CD CD CD
II CD CD CD CD CD 56 CS) CD CD CD CD 01 CS) CD CD CD CD 46 CS) CD CD CD CD
12 CS) CD CD CD CD 57 CS) CD CD CD CD 02 CS) CD CD CD CD 47 CS) CD CD CD CD
13 CS) CD CD CD CD 58 CS) CD CD CD CD 03 CS) CD CD CD CD 48 CS) CD CD CD CD
14 CS) CD CD CD CD 59 CS) CD CD CD CD 04 CS) CD CD CD CD 49 CS) CD CD CD CD
15 CS) CD CD CD CD 60 CS) CD CD CD CD 05 CS) CD CD CD CD so CS) CD CD CD CD
16 CS) CD CD CD CD 61 CS) CD CD CD CD 06 CS) CD CD CD CD 51 CS) CD CD CD CD
17 CS) CD CD CD CD 62 CS) CD CD CD CD 07 CS) CD CD CD CD 52 CS) CD CD CD CD
18 CS) CD CD CD CD 63 CS) CD CD CD CD 08 CS) CD CD CD CD 53 CS) CD CD CD CD
19 CS) CD CD CD CD 64 CS) CD CD CD CD 09 CS) CD CD CD CD 54 CS) CD CD CD CD
20 CS) CD CD CD CD 65 CS) CD CD CD CD 10CS) CD CD CD CD 55 CS) CD CD CD CD
21 CS) CD CD CD CD 66 CS) CD CD CD CD 11 CS)CDCDCDCD 56 CS) CD CD CD CD
22 CS) CD CD CD CD 67 CS) CD CD CD CD 12 CS) CD CD CD CD 57 CS) CD CD CD CD
23 CS) CD CD CD CD 68 CS) CD CD CD CD 13 CS) CD CD CD CD 58 CS) CD CD CD CD
24 CS) CD CD CD CD 69 CS) CD CD CD CD 14 CS) CD CD CD CD 59 CS) CD CD CD CD
25 CS) CD CD CD CD 70 CS) CD CD CD CD 15 CS) CD CD CD CD 60 CS) CD CD CD CD
26 CS) CD CD CD CD 71 CS) CD CD CD CD 16 CS) CD CD CD CD 61 CS) CD CD CD CD
27 CS) CD CD CD CD 72 CS) CD CD CD CD 17 CS) CD CD CD CD 62 CS) CD CD CD CD
28 CS) CD CD CD CD 73 CS) CD CD CD CD 18 CS) CD CD CD CD 63 CS) CD CD CD CD
29 CS) CD CD CD CD 74 CS) CD CD CD CD 19 CS) CD CD CD CD 64 CS) CD CD CD CD
30 CS) CD CD CD CD 75 CS) CD CD CD CD 20 CS) CD CD CD CD 65 CS) CD CD CD CD
31 CS) CD CD CD CD 76 CS) CD CD CD CD 21 CS) CD CD CD CD 66 CS) CD CD CD CD
32 CS) CD CD CD CD 77 CS) CD CD CD CD 22 CS) CD CD CD CD 67 CS) CD CD CD CD
33 CS) CD CD CD CD 78 CS) CD CD CD CD 23 CS) CD CD CD CD 68 CS) CD CD CD CD
34 CS) CD CD CD CD 79 CS) CD CD CD CD 24 CS) CD CD CD CD 69 CS) CD CD CD CD
35 CS) CD CD CD CD so CS) CD CD CD CD 25 CS) CD CD CD CD 70 CS) CD CD CD CD
36 CS) CD CD CD CD 81 CS) CD CD CD CD 26 CS) CD CD CD CD 71 CS) CD CD CD CD
37 CS) CD CD CD CD 82 CS) CD CD CD CD 27 CS) CD CD CD CD 72 CS) CD CD CD CD
38 CS) CD CD CD CD 83 CS) CD CD CD CD 28 CS) CD CD CD CD 73 CS) CD CD CD CD
39 CS) CD CD CD CD 84 CS) CD CD CD CD 29 CS) CD CD CD CD 74 CS) CD CD CD CD
40 CS) CD CD GD CD 85 CS) CD CD CD CD 30 CS) CD CD CD CD 75 CS) CD CD CD CD
41 CS) CD CD CD CD 86 CS) CD CD CD CD 31 CS) CD CD CD CD 76 CS) CD CD CD CD
42 CS) CD CD CD CD 87 CS) CD CD CD CD 32 CS) CD CD CD CD 77 CS) CD CD CD CD
43 CS) CD CD CD CD 88 CS) CD CD CD CD 33 CS) CD CD CD CD 78 CS) CD CD CD CD
44 CS) CD CD CD CD 89 CS) CD CD CD CD 34 CS) CD CD CD CD 79 CS) CD CD CD CD
45 CS) CD CD CD CD 90 CS) CD CD CD CD 35 CS) CD CD CD CD 80 CS) CD CD CD CD

MK
MKSAP16
General Internal Medicine

The Best Review and Test Questions in Medicine:

Routine Care of the Healthy Patient

Patient Safety

' Professionalism and Ethics

Palliative Care

Common Symptoms

' Musculoskeletal Pain

Obesity

Men's and Women's Health

Eye, Ear, Nose, Mouth, and Throat Disorders

Mental and Behavioral Health

Geriatric Medicine

Perioperative Medicine

Plus much more . . .

ACP
ISBN 978-1-938245-08-4
90000

American College of Physicians 9 781938"245084


internal meoicine I Doctors for Adults

You might also like